date explanation q number question answer 20120101 It's our annual year-end news quiz, compiled with the help of Kathie Baker and Tim Goodman. You are given new names in the news — people you probably never heard of before 2011, but who became famous during the past 12 months. Explain why they're famous. 1 Kim Jong Un He's the new North Korean leader 20120101 It's our annual year-end news quiz, compiled with the help of Kathie Baker and Tim Goodman. You are given new names in the news — people you probably never heard of before 2011, but who became famous during the past 12 months. Explain why they're famous. 2 Gilad Shalit Gilad Shalit is the Israeli soldier who was released from captivity in exchange for many Palestinians 20120101 It's our annual year-end news quiz, compiled with the help of Kathie Baker and Tim Goodman. You are given new names in the news — people you probably never heard of before 2011, but who became famous during the past 12 months. Explain why they're famous. 3 Mohammed Bouazizi He was a Tunisian street vendor whose self-immolation martyrdom triggered the Arab Spring and led to the overthrow of the Tunisian government 20120101 It's our annual year-end news quiz, compiled with the help of Kathie Baker and Tim Goodman. You are given new names in the news — people you probably never heard of before 2011, but who became famous during the past 12 months. Explain why they're famous. 5 Kris Humphries He married Kim Kardashian for 72 days 20120101 It's our annual year-end news quiz, compiled with the help of Kathie Baker and Tim Goodman. You are given new names in the news — people you probably never heard of before 2011, but who became famous during the past 12 months. Explain why they're famous. 6 Watson "the computer that played ""Jeopardy!""" 20120101 It's our annual year-end news quiz, compiled with the help of Kathie Baker and Tim Goodman. You are given new names in the news — people you probably never heard of before 2011, but who became famous during the past 12 months. Explain why they're famous. 7 Siri the new intelligence software system for the iPhone that answers questions for you. 20130414 Every answer is a two-word phrase in which the first word starts with O. Drop the O, and you'll get a new word that ends the phrase. 1 male resident of Muscat Oman man 20130414 Every answer is a two-word phrase in which the first word starts with O. Drop the O, and you'll get a new word that ends the phrase. 2 statue of a lion made out of a butter substitute oleo leo 20130414 Every answer is a two-word phrase in which the first word starts with O. Drop the O, and you'll get a new word that ends the phrase. 3 very wise Arkansas Indian Osage sage 20130414 Every answer is a two-word phrase in which the first word starts with O. Drop the O, and you'll get a new word that ends the phrase. 4 area high in the atmosphere where the air is thin ozone zone 20130414 Every answer is a two-word phrase in which the first word starts with O. Drop the O, and you'll get a new word that ends the phrase. 5 pumpkin colored stove orange range 20130414 Every answer is a two-word phrase in which the first word starts with O. Drop the O, and you'll get a new word that ends the phrase. 6 accommodate R&B singer Mary J oblige Blige 20130414 Every answer is a two-word phrase in which the first word starts with O. Drop the O, and you'll get a new word that ends the phrase. 7 grouchy muppet’s marks left by wounds Oscar’s scars 20130414 Every answer is a two-word phrase in which the first word starts with O. Drop the O, and you'll get a new word that ends the phrase. 8 job to find what is left out omission mission 20200809 Every answer today is the name of a famous person whose first initial and last name, in order, spell a word. For example, take Benjamin Rush, signer of the Declaration of Independence. The B of Benjamin + his last name spells BRUSH. I'll give you clues to the parts. You give me the names. 1 Oscar-winning actor (3,5) — expression of appreciation Tom Hanks, thanks 20200809 Every answer today is the name of a famous person whose first initial and last name, in order, spell a word. For example, take Benjamin Rush, signer of the Declaration of Independence. The B of Benjamin + his last name spells BRUSH. I'll give you clues to the parts. You give me the names. 2 Singer with the Supremes (5,4) — worthless stuff Diana Ross, dross 20200809 Every answer today is the name of a famous person whose first initial and last name, in order, spell a word. For example, take Benjamin Rush, signer of the Declaration of Independence. The B of Benjamin + his last name spells BRUSH. I'll give you clues to the parts. You give me the names. 3 Former baseball star (4,4) — writing that's not poetry Pete Rose, prose 20200809 Every answer today is the name of a famous person whose first initial and last name, in order, spell a word. For example, take Benjamin Rush, signer of the Declaration of Independence. The B of Benjamin + his last name spells BRUSH. I'll give you clues to the parts. You give me the names. 4 Comedian and former host of the Oscars (5,4) — earthenware pot Chris Rock, crock 20200809 Every answer today is the name of a famous person whose first initial and last name, in order, spell a word. For example, take Benjamin Rush, signer of the Declaration of Independence. The B of Benjamin + his last name spells BRUSH. I'll give you clues to the parts. You give me the names. 5 Singer with the group Hole (8,4) — garlic bulb Courtney Love, clove 20200809 Every answer today is the name of a famous person whose first initial and last name, in order, spell a word. For example, take Benjamin Rush, signer of the Declaration of Independence. The B of Benjamin + his last name spells BRUSH. I'll give you clues to the parts. You give me the names. 6 Oldtime comedian with a radio show (4,5) — opposite of risen Fred Allen, fallen 20200809 Every answer today is the name of a famous person whose first initial and last name, in order, spell a word. For example, take Benjamin Rush, signer of the Declaration of Independence. The B of Benjamin + his last name spells BRUSH. I'll give you clues to the parts. You give me the names. 7 "Co-star of """"Desperate Housewives"""" (4,7) — former British prime minister" Teri Hatcher 20190623 Every answer today is the name of a fabric or clothing material. You name it from its anagram. 1 LIKES - E silk 20190623 Every answer today is the name of a fabric or clothing material. You name it from its anagram. 2 ON LOW - N wool 20190623 Every answer today is the name of a fabric or clothing material. You name it from its anagram. 3 NOTARY - T rayon 20190623 Every answer today is the name of a fabric or clothing material. You name it from its anagram. 4 MAIDEN - A denim 20190623 Every answer today is the name of a fabric or clothing material. You name it from its anagram. 5 ANTICS - C satin 20190623 Every answer today is the name of a fabric or clothing material. You name it from its anagram. 6 ARCHLY - H lycra 20190623 Every answer today is the name of a fabric or clothing material. You name it from its anagram. 7 RECIPE - I crepe 20190623 Every answer today is the name of a fabric or clothing material. You name it from its anagram. 8 CONTORT - R cotton 20190623 Every answer today is the name of a fabric or clothing material. You name it from its anagram. 9 CALDRON - L dacron 20190623 Every answer today is the name of a fabric or clothing material. You name it from its anagram. 10 REMATCHES - T cashmere 20190623 Every answer today is the name of a fabric or clothing material. You name it from its anagram. 11 YOUR CROWD - W corduroy 20170709 I'm going to give you clues for some 5-letter words. Switch the 2nd and 4th letters to make a new word that answers the second clue. 1 Christmas song / Reef material carol, coral 20170709 I'm going to give you clues for some 5-letter words. Switch the 2nd and 4th letters to make a new word that answers the second clue. 2 No longer fresh / Roof material stale, slate 20170709 I'm going to give you clues for some 5-letter words. Switch the 2nd and 4th letters to make a new word that answers the second clue. 3 "Versions in the testing stage / Motel in """"Psycho""""""" betas, Bates 20170709 I'm going to give you clues for some 5-letter words. Switch the 2nd and 4th letters to make a new word that answers the second clue. 4 Sheets, pillowcases, etc. / First leader of the Soviet Union linen, Lenin 20170709 I'm going to give you clues for some 5-letter words. Switch the 2nd and 4th letters to make a new word that answers the second clue. 5 Song in the Bible / Blood component psalm, plasm 20170709 I'm going to give you clues for some 5-letter words. Switch the 2nd and 4th letters to make a new word that answers the second clue. 6 Tree secretion / Up resin, risen 20170709 I'm going to give you clues for some 5-letter words. Switch the 2nd and 4th letters to make a new word that answers the second clue. 7 Trail left by a snail / Grin slime, smile 20170709 I'm going to give you clues for some 5-letter words. Switch the 2nd and 4th letters to make a new word that answers the second clue. 8 Male singing voice / Cartridge contents tenor, toner 20170709 I'm going to give you clues for some 5-letter words. Switch the 2nd and 4th letters to make a new word that answers the second clue. 9 Marsh bird / Backbone snipe, spine 20170709 I'm going to give you clues for some 5-letter words. Switch the 2nd and 4th letters to make a new word that answers the second clue. 10 Group of experts / Word before code or colony panel, penal 20170709 I'm going to give you clues for some 5-letter words. Switch the 2nd and 4th letters to make a new word that answers the second clue. 11 Temperamental sorts / ___ Cup divas, Davis 20170709 I'm going to give you clues for some 5-letter words. Switch the 2nd and 4th letters to make a new word that answers the second clue. 12 Home / Material for a Southwest home abode, adobe 20181209 Every answer today is a familiar three-word phrase or title in which the first and last words are the same. I'll give you the middle word (and a clue, if necessary). You tell me the phrase. 1 THINGS — Phrase suggesting priority first things first 20181209 Every answer today is a familiar three-word phrase or title in which the first and last words are the same. I'll give you the middle word (and a clue, if necessary). You tell me the phrase. 2 EAT — cutthroat dog eat dog 20181209 Every answer today is a familiar three-word phrase or title in which the first and last words are the same. I'll give you the middle word (and a clue, if necessary). You tell me the phrase. 3 SWEET — What you say after returning from a long trip home sweet home 20181209 Every answer today is a familiar three-word phrase or title in which the first and last words are the same. I'll give you the middle word (and a clue, if necessary). You tell me the phrase. 4 AGAINST — Not give up despite impossible odds hope against hope 20181209 Every answer today is a familiar three-word phrase or title in which the first and last words are the same. I'll give you the middle word (and a clue, if necessary). You tell me the phrase. 5 MEANS — an emphatic refusal no means no 20181209 Every answer today is a familiar three-word phrase or title in which the first and last words are the same. I'll give you the middle word (and a clue, if necessary). You tell me the phrase. 6 VERSUS — a classic feature of Mad magazine Spy vs. Spy, or Kramer vs. Kramer 20181209 Every answer today is a familiar three-word phrase or title in which the first and last words are the same. I'll give you the middle word (and a clue, if necessary). You tell me the phrase. 7 SAY — Don't give up, there's always a possibility never say never 20181209 Every answer today is a familiar three-word phrase or title in which the first and last words are the same. I'll give you the middle word (and a clue, if necessary). You tell me the phrase. 8 BABY — Catchphrase said by Sarah Palin at the 2008 Republican National Convention drill baby drill 20181209 Every answer today is a familiar three-word phrase or title in which the first and last words are the same. I'll give you the middle word (and a clue, if necessary). You tell me the phrase. 9 "THOUSAND — The number of """"Arabian Nights""""" one thousand one 20181209 Every answer today is a familiar three-word phrase or title in which the first and last words are the same. I'll give you the middle word (and a clue, if necessary). You tell me the phrase. 10 CITY — State capital north of Austin, Texas Oklahoma City, Oklahoma 20181209 Every answer today is a familiar three-word phrase or title in which the first and last words are the same. I'll give you the middle word (and a clue, if necessary). You tell me the phrase. 11 "GLORIOUS — Song from """"Oliver!"""" with the lyric """"Just picture a great big steak / Fried, roasted, or stewed""""" Food Glorious Food 20181209 Every answer today is a familiar three-word phrase or title in which the first and last words are the same. I'll give you the middle word (and a clue, if necessary). You tell me the phrase. 12 BLOODY — 1971 film with Glenda Jackson and Peter Finch Sunday Bloody Sunday 20181209 Every answer today is a familiar three-word phrase or title in which the first and last words are the same. I'll give you the middle word (and a clue, if necessary). You tell me the phrase. 13 JAMES — 007's introduction Bond, James Bond 20181209 Every answer today is a familiar three-word phrase or title in which the first and last words are the same. I'll give you the middle word (and a clue, if necessary). You tell me the phrase. 14 NOT — Faux apology sorry, not sorry 20171231 "This is my annual """"New Names in the News Quiz."""" I'll name some people and things you had probably never heard of until 2017 but that sprang to prominence during the past 12 months. You tell me who or what they are.1. Doug Jones 2. Gal Gadot3. Anthony Scaramucci4. Jack Phillips5. Emmerson Mnangagwa6. Tom Steyer7. Salvator Mundi" 1 Doug Jones Democratic upset winner in Alabama who ran against Roy Moore in a special election for Jeff Sessions' seat in the Senate 20171231 "This is my annual """"New Names in the News Quiz."""" I'll name some people and things you had probably never heard of until 2017 but that sprang to prominence during the past 12 months. You tell me who or what they are.1. Doug Jones 2. Gal Gadot3. Anthony Scaramucci4. Jack Phillips5. Emmerson Mnangagwa6. Tom Steyer7. Salvator Mundi" 2 Gal Gadot Israeli actress who starred as Wonder Woman 20171231 "This is my annual """"New Names in the News Quiz."""" I'll name some people and things you had probably never heard of until 2017 but that sprang to prominence during the past 12 months. You tell me who or what they are.1. Doug Jones 2. Gal Gadot3. Anthony Scaramucci4. Jack Phillips5. Emmerson Mnangagwa6. Tom Steyer7. Salvator Mundi" 3 Anthony Scaramucci President Trump's communications director for just 10 days 20171231 "This is my annual """"New Names in the News Quiz."""" I'll name some people and things you had probably never heard of until 2017 but that sprang to prominence during the past 12 months. You tell me who or what they are.1. Doug Jones 2. Gal Gadot3. Anthony Scaramucci4. Jack Phillips5. Emmerson Mnangagwa6. Tom Steyer7. Salvator Mundi" 4 Jack Phillips Colorado baker who refused to bake a wedding cake for a gay couple; his case is now before the Supreme Court 20171231 "This is my annual """"New Names in the News Quiz."""" I'll name some people and things you had probably never heard of until 2017 but that sprang to prominence during the past 12 months. You tell me who or what they are.1. Doug Jones 2. Gal Gadot3. Anthony Scaramucci4. Jack Phillips5. Emmerson Mnangagwa6. Tom Steyer7. Salvator Mundi" 5 Emmerson Mnangagwa New president of Zimbabwe, successor to Robert Mugabe 20171231 "This is my annual """"New Names in the News Quiz."""" I'll name some people and things you had probably never heard of until 2017 but that sprang to prominence during the past 12 months. You tell me who or what they are.1. Doug Jones 2. Gal Gadot3. Anthony Scaramucci4. Jack Phillips5. Emmerson Mnangagwa6. Tom Steyer7. Salvator Mundi" 6 Tom Steyer Billionaire activist who has spent millions of dollars in a campaign to impeach President Trump 20171231 "This is my annual """"New Names in the News Quiz."""" I'll name some people and things you had probably never heard of until 2017 but that sprang to prominence during the past 12 months. You tell me who or what they are.1. Doug Jones 2. Gal Gadot3. Anthony Scaramucci4. Jack Phillips5. Emmerson Mnangagwa6. Tom Steyer7. Salvator Mundi" 7 Salvator Mundi Painting by Leonardo da Vinci that sold at Christie's for a record $450.3 million 20150517 "This week's on-air puzzle is similar to last week's, only a little harder. Every answer is the name of a country. For each word given, ignore the vowels. The consonants, in order from left to right, are the same consonants in the same order as in the country. For example, given the word """"omelet,"""" the answer is """"Malta.""""" 1 supine Spain 20150517 "This week's on-air puzzle is similar to last week's, only a little harder. Every answer is the name of a country. For each word given, ignore the vowels. The consonants, in order from left to right, are the same consonants in the same order as in the country. For example, given the word """"omelet,"""" the answer is """"Malta.""""" 2 climb Colombia 20150517 "This week's on-air puzzle is similar to last week's, only a little harder. Every answer is the name of a country. For each word given, ignore the vowels. The consonants, in order from left to right, are the same consonants in the same order as in the country. For example, given the word """"omelet,"""" the answer is """"Malta.""""" 3 hinders Honduras 20150517 "This week's on-air puzzle is similar to last week's, only a little harder. Every answer is the name of a country. For each word given, ignore the vowels. The consonants, in order from left to right, are the same consonants in the same order as in the country. For example, given the word """"omelet,"""" the answer is """"Malta.""""" 4 rouses Russia 20150517 "This week's on-air puzzle is similar to last week's, only a little harder. Every answer is the name of a country. For each word given, ignore the vowels. The consonants, in order from left to right, are the same consonants in the same order as in the country. For example, given the word """"omelet,"""" the answer is """"Malta.""""" 5 encourage Nicaragua 20150517 "This week's on-air puzzle is similar to last week's, only a little harder. Every answer is the name of a country. For each word given, ignore the vowels. The consonants, in order from left to right, are the same consonants in the same order as in the country. For example, given the word """"omelet,"""" the answer is """"Malta.""""" 6 cloned Iceland 20150517 "This week's on-air puzzle is similar to last week's, only a little harder. Every answer is the name of a country. For each word given, ignore the vowels. The consonants, in order from left to right, are the same consonants in the same order as in the country. For example, given the word """"omelet,"""" the answer is """"Malta.""""" 7 smile Somolia 20150517 "This week's on-air puzzle is similar to last week's, only a little harder. Every answer is the name of a country. For each word given, ignore the vowels. The consonants, in order from left to right, are the same consonants in the same order as in the country. For example, given the word """"omelet,"""" the answer is """"Malta.""""" 8 userlink Sri Lanka 20150517 "This week's on-air puzzle is similar to last week's, only a little harder. Every answer is the name of a country. For each word given, ignore the vowels. The consonants, in order from left to right, are the same consonants in the same order as in the country. For example, given the word """"omelet,"""" the answer is """"Malta.""""" 9 origination Argentina 20150517 "This week's on-air puzzle is similar to last week's, only a little harder. Every answer is the name of a country. For each word given, ignore the vowels. The consonants, in order from left to right, are the same consonants in the same order as in the country. For example, given the word """"omelet,"""" the answer is """"Malta.""""" 10 sterile Australia 20150405 "Every answer is a made-up, two-word phrase in which the first word has seven letters. Drop its first and last letters to get a five-letter word that is the second part of the phrase. For example: Bottled water from France that is not normal is """"deviant Evian.""""" 1 opposed to profits against gains 20150405 "Every answer is a made-up, two-word phrase in which the first word has seven letters. Drop its first and last letters to get a five-letter word that is the second part of the phrase. For example: Bottled water from France that is not normal is """"deviant Evian.""""" 2 catch sight of slight hobbles glimpse limps 20150405 "Every answer is a made-up, two-word phrase in which the first word has seven letters. Drop its first and last letters to get a five-letter word that is the second part of the phrase. For example: Bottled water from France that is not normal is """"deviant Evian.""""" 3 former Alaska governor with a milky white hue opaline Palin 20150405 "Every answer is a made-up, two-word phrase in which the first word has seven letters. Drop its first and last letters to get a five-letter word that is the second part of the phrase. For example: Bottled water from France that is not normal is """"deviant Evian.""""" 4 narrow walkway covered with a colorful curvy shirt pattern Paisley aisle 20150405 "Every answer is a made-up, two-word phrase in which the first word has seven letters. Drop its first and last letters to get a five-letter word that is the second part of the phrase. For example: Bottled water from France that is not normal is """"deviant Evian.""""" 5 great deal of wreckage from a ship (5-letter word comes first this time) lotsa flotsam 20141214 These are some business-related puzzles made for the New York Times' DealBook conference in New York last Thursday. Every answer is the name of a Fortune 200 company — that is, one of the top 200 corporations according to the 2014 list in Fortune magazine. 1 What company’s name is appropriately inside the phrase METAL COATING? Alcoa 20141214 These are some business-related puzzles made for the New York Times' DealBook conference in New York last Thursday. Every answer is the name of a Fortune 200 company — that is, one of the top 200 corporations according to the 2014 list in Fortune magazine. 2 What company’s name is an anagram of ICE POPS? PepsiCo 20141214 These are some business-related puzzles made for the New York Times' DealBook conference in New York last Thursday. Every answer is the name of a Fortune 200 company — that is, one of the top 200 corporations according to the 2014 list in Fortune magazine. 3 What company’s name consists of 8 letters, 4 of which are L’s? Eli Lilly 20141214 These are some business-related puzzles made for the New York Times' DealBook conference in New York last Thursday. Every answer is the name of a Fortune 200 company — that is, one of the top 200 corporations according to the 2014 list in Fortune magazine. 4 What company’s name becomes the name of an Olympic sport if you change the third letter to an X? Boeing (making boxing) 20141214 These are some business-related puzzles made for the New York Times' DealBook conference in New York last Thursday. Every answer is the name of a Fortune 200 company — that is, one of the top 200 corporations according to the 2014 list in Fortune magazine. 5 What company’s name is pig Latin for an insect? eBay 20141214 These are some business-related puzzles made for the New York Times' DealBook conference in New York last Thursday. Every answer is the name of a Fortune 200 company — that is, one of the top 200 corporations according to the 2014 list in Fortune magazine. 6 What company’s name consists of the names of two presidents’ names? (a tricky clue) Johnson & Johnson 20131229 Today’s on-air puzzle is called, Now You Know Them. 1 Rob Ford mayor of Toronto who admitted using crack cocaine 20131229 Today’s on-air puzzle is called, Now You Know Them. 2 Edward Snowden NSA whistleblower 20131229 Today’s on-air puzzle is called, Now You Know Them. 3 George Alexander Louis Windsor Royal baby born to Prince William and Duchess Catherine 20131229 Today’s on-air puzzle is called, Now You Know Them. 4 North West Baby of Kim Kardashian and Kanye West 20131229 Today’s on-air puzzle is called, Now You Know Them. 5 Hassan Rouhani New president of Iran, regarded as a moderate 20131229 Today’s on-air puzzle is called, Now You Know Them. 6 Janet Yellen President Obama’s nominee to be the new Chairman of the Federal Reserve Board 20131229 Today’s on-air puzzle is called, Now You Know Them. 7 Magnus Carlsen new world chess champion 20131229 Today’s on-air puzzle is called, Now You Know Them. 8 Sunny the Obama’s new dog 20140413 "Three words that start with the same letter will be presented in a group. Find a word that shares the same first letter as the three, and that can follow each word within the group to complete a compound word or a familiar two-word phrase. Example: """"big,"""" """"broad,"""" """"boy""""; the answer would be """"band"""" to get """"big band,"""" """"broadband"""" and """"boy band.""""" 1 calling, credit and Christmas card 20140413 "Three words that start with the same letter will be presented in a group. Find a word that shares the same first letter as the three, and that can follow each word within the group to complete a compound word or a familiar two-word phrase. Example: """"big,"""" """"broad,"""" """"boy""""; the answer would be """"band"""" to get """"big band,"""" """"broadband"""" and """"boy band.""""" 2 cable, cattle and company car 20140413 "Three words that start with the same letter will be presented in a group. Find a word that shares the same first letter as the three, and that can follow each word within the group to complete a compound word or a familiar two-word phrase. Example: """"big,"""" """"broad,"""" """"boy""""; the answer would be """"band"""" to get """"big band,"""" """"broadband"""" and """"boy band.""""" 3 electric, eagle and evil eye 20140413 "Three words that start with the same letter will be presented in a group. Find a word that shares the same first letter as the three, and that can follow each word within the group to complete a compound word or a familiar two-word phrase. Example: """"big,"""" """"broad,"""" """"boy""""; the answer would be """"band"""" to get """"big band,"""" """"broadband"""" and """"boy band.""""" 4 fist, fire and food fight 20140413 "Three words that start with the same letter will be presented in a group. Find a word that shares the same first letter as the three, and that can follow each word within the group to complete a compound word or a familiar two-word phrase. Example: """"big,"""" """"broad,"""" """"boy""""; the answer would be """"band"""" to get """"big band,"""" """"broadband"""" and """"boy band.""""" 5 guessing, golf and ground game 20140413 "Three words that start with the same letter will be presented in a group. Find a word that shares the same first letter as the three, and that can follow each word within the group to complete a compound word or a familiar two-word phrase. Example: """"big,"""" """"broad,"""" """"boy""""; the answer would be """"band"""" to get """"big band,"""" """"broadband"""" and """"boy band.""""" 6 pie, pizza and Peter pan 20140413 "Three words that start with the same letter will be presented in a group. Find a word that shares the same first letter as the three, and that can follow each word within the group to complete a compound word or a familiar two-word phrase. Example: """"big,"""" """"broad,"""" """"boy""""; the answer would be """"band"""" to get """"big band,"""" """"broadband"""" and """"boy band.""""" 7 table, tank and tree top 20200126 Every answer is a word, name or phrase in which the only consonants are T and P, repeated as often as necessary. 1 Something Idaho is famous for producing potato 20200126 Every answer is a word, name or phrase in which the only consonants are T and P, repeated as often as necessary. 2 Small, as in women’s dress sizes petite 20200126 Every answer is a word, name or phrase in which the only consonants are T and P, repeated as often as necessary. 3 to proceed very quitely tiptoe 20200126 Every answer is a word, name or phrase in which the only consonants are T and P, repeated as often as necessary. 4 a baked entree usually with meat and vegetables potpie 20200126 Every answer is a word, name or phrase in which the only consonants are T and P, repeated as often as necessary. 5 artificial hairpiece to cover a bald spot toupee 20200126 Every answer is a word, name or phrase in which the only consonants are T and P, repeated as often as necessary. 6 a marionette for example puppet 20200126 Every answer is a word, name or phrase in which the only consonants are T and P, repeated as often as necessary. 7 a calming drug like heroin or morphine opiate 20200126 Every answer is a word, name or phrase in which the only consonants are T and P, repeated as often as necessary. 8 an amount that is produced as in a factory output 20200126 Every answer is a word, name or phrase in which the only consonants are T and P, repeated as often as necessary. 9 paradise a perfect place utopia 20200126 Every answer is a word, name or phrase in which the only consonants are T and P, repeated as often as necessary. 10 tempest locale in a saying teapot 20200126 Every answer is a word, name or phrase in which the only consonants are T and P, repeated as often as necessary. 11 excellent or very best tiptop 20200126 Every answer is a word, name or phrase in which the only consonants are T and P, repeated as often as necessary. 12 capital of Taiwan Taipei 20200126 Every answer is a word, name or phrase in which the only consonants are T and P, repeated as often as necessary. 13 capital of Tahiti Pape'ete 20200126 Every answer is a word, name or phrase in which the only consonants are T and P, repeated as often as necessary. 14 sound of a beating heart hyphenated pit-a-pat 20200126 Every answer is a word, name or phrase in which the only consonants are T and P, repeated as often as necessary. 15 what hunger increases appetite 20200126 Every answer is a word, name or phrase in which the only consonants are T and P, repeated as often as necessary. 16 another name for miniature golf putt-putt 20161030 "This week's puzzle is called """"SuperPACs."""" Every answer is a familiar two-word phrase or name in which the first word starts with PA- and the second word starts with C." 1 waterway connecting the Atlantic and the Pacific Panama Canal 20161030 "This week's puzzle is called """"SuperPACs."""" Every answer is a familiar two-word phrase or name in which the first word starts with PA- and the second word starts with C." 2 a bent wire for holding office sheets together paper clip 20161030 "This week's puzzle is called """"SuperPACs."""" Every answer is a familiar two-word phrase or name in which the first word starts with PA- and the second word starts with C." 3 what Comcast is to NBC Universal parent company 20161030 "This week's puzzle is called """"SuperPACs."""" Every answer is a familiar two-word phrase or name in which the first word starts with PA- and the second word starts with C." 4 police officers’ vehicle patrol car 20161030 "This week's puzzle is called """"SuperPACs."""" Every answer is a familiar two-word phrase or name in which the first word starts with PA- and the second word starts with C." 5 renowned 20th Century cellist Pablo Casals 20161030 "This week's puzzle is called """"SuperPACs."""" Every answer is a familiar two-word phrase or name in which the first word starts with PA- and the second word starts with C." 6 country singer whose hits include Crazy and I Fall to Pieces Patsy Cline 20161030 "This week's puzzle is called """"SuperPACs."""" Every answer is a familiar two-word phrase or name in which the first word starts with PA- and the second word starts with C." 7 container for a Benjamin Moore or Sherwin Williams product paint can 20161030 "This week's puzzle is called """"SuperPACs."""" Every answer is a familiar two-word phrase or name in which the first word starts with PA- and the second word starts with C." 8 device for remotely watching Chinese animals at a zoo panda cam 20161030 "This week's puzzle is called """"SuperPACs."""" Every answer is a familiar two-word phrase or name in which the first word starts with PA- and the second word starts with C." 9 person who shows up at a social event without an invitation party crasher 20161030 "This week's puzzle is called """"SuperPACs."""" Every answer is a familiar two-word phrase or name in which the first word starts with PA- and the second word starts with C." 10 words repeated before baker’s man in a children’s rhyme patty cake 20120603 "Every answer is the name of a world capital. You'll be given clues to its phonetic parts, and you name the capital. For example, given the clues """"person from Bangkok"""" and """"salary,"""" the answer would be Taipei (""""Thai"""" plus """"pay"""")." 1 Actress Taylor, informally; covering for a hot dog Lisbon 20120603 "Every answer is the name of a world capital. You'll be given clues to its phonetic parts, and you name the capital. For example, given the clues """"person from Bangkok"""" and """"salary,"""" the answer would be Taipei (""""Thai"""" plus """"pay"""")." 2 A Muslim's place of worship; a cry of surprise Moscow 20120603 "Every answer is the name of a world capital. You'll be given clues to its phonetic parts, and you name the capital. For example, given the clues """"person from Bangkok"""" and """"salary,"""" the answer would be Taipei (""""Thai"""" plus """"pay"""")." 3 Wildebeest; sandwich shop New Delhi 20120603 "Every answer is the name of a world capital. You'll be given clues to its phonetic parts, and you name the capital. For example, given the clues """"person from Bangkok"""" and """"salary,"""" the answer would be Taipei (""""Thai"""" plus """"pay"""")." 4 Chesapeake, for example; road or highway Beiruit 20120603 "Every answer is the name of a world capital. You'll be given clues to its phonetic parts, and you name the capital. For example, given the clues """"person from Bangkok"""" and """"salary,"""" the answer would be Taipei (""""Thai"""" plus """"pay"""")." 5 expected, as payment; a tournament position in which one advances without playing Dubai 20120603 "Every answer is the name of a world capital. You'll be given clues to its phonetic parts, and you name the capital. For example, given the clues """"person from Bangkok"""" and """"salary,"""" the answer would be Taipei (""""Thai"""" plus """"pay"""")." 6 A sound of disapproval, as in a stadium; a snarling dog; to seize, as control Bucharest 20120603 "Every answer is the name of a world capital. You'll be given clues to its phonetic parts, and you name the capital. For example, given the clues """"person from Bangkok"""" and """"salary,"""" the answer would be Taipei (""""Thai"""" plus """"pay"""")." 7 Opposite of heaven, to croon; a lock opener Helsinki 20160508 Given a four-letter word and a six-letter word, rearrange the letters of one of them to get a synonym of the other. 1 dare, peruse read 20160508 Given a four-letter word and a six-letter word, rearrange the letters of one of them to get a synonym of the other. 2 tame, coiled docile 20160508 Given a four-letter word and a six-letter word, rearrange the letters of one of them to get a synonym of the other. 3 peek, retain keep 20160508 Given a four-letter word and a six-letter word, rearrange the letters of one of them to get a synonym of the other. 4 clam, serene calm 20160508 Given a four-letter word and a six-letter word, rearrange the letters of one of them to get a synonym of the other. 5 taco, jacket coat 20160508 Given a four-letter word and a six-letter word, rearrange the letters of one of them to get a synonym of the other. 6 suds, halter lather 20160508 Given a four-letter word and a six-letter word, rearrange the letters of one of them to get a synonym of the other. 7 grin, circle ring 20160508 Given a four-letter word and a six-letter word, rearrange the letters of one of them to get a synonym of the other. 8 rote, ripped tore 20160508 Given a four-letter word and a six-letter word, rearrange the letters of one of them to get a synonym of the other. 9 wolf, stream flow 20160508 Given a four-letter word and a six-letter word, rearrange the letters of one of them to get a synonym of the other. 10 dial, placed laid 20160508 Given a four-letter word and a six-letter word, rearrange the letters of one of them to get a synonym of the other. 11 hear, silent listen 20121209 "Every answer is a six-letter word containing """"QU"""" somewhere inside it. You'll be given anagrams of the remaining four letters. You name the words (No answer is a plural or a word formed by adding """"s."""")." 1 rave quaver 20121209 "Every answer is a six-letter word containing """"QU"""" somewhere inside it. You'll be given anagrams of the remaining four letters. You name the words (No answer is a plural or a word formed by adding """"s."""")." 2 leap plaque 20121209 "Every answer is a six-letter word containing """"QU"""" somewhere inside it. You'll be given anagrams of the remaining four letters. You name the words (No answer is a plural or a word formed by adding """"s."""")." 3 Nita quaint 20121209 "Every answer is a six-letter word containing """"QU"""" somewhere inside it. You'll be given anagrams of the remaining four letters. You name the words (No answer is a plural or a word formed by adding """"s."""")." 4 rims squirm 20121209 "Every answer is a six-letter word containing """"QU"""" somewhere inside it. You'll be given anagrams of the remaining four letters. You name the words (No answer is a plural or a word formed by adding """"s."""")." 5 some mosque 20121209 "Every answer is a six-letter word containing """"QU"""" somewhere inside it. You'll be given anagrams of the remaining four letters. You name the words (No answer is a plural or a word formed by adding """"s."""")." 6 sear square 20121209 "Every answer is a six-letter word containing """"QU"""" somewhere inside it. You'll be given anagrams of the remaining four letters. You name the words (No answer is a plural or a word formed by adding """"s."""")." 7 lice clique 20121209 "Every answer is a six-letter word containing """"QU"""" somewhere inside it. You'll be given anagrams of the remaining four letters. You name the words (No answer is a plural or a word formed by adding """"s."""")." 8 lees sequel 20140608 "Every answer is a familiar two-word phrase or name in which the first word ends in E and the second word starts with Z. For example, given, """"popular blush wine,"""" the answer would be: White zinfandel." 1 the area in front of a batter where a pitcher aims strike zone 20140608 "Every answer is a familiar two-word phrase or name in which the first word ends in E and the second word starts with Z. For example, given, """"popular blush wine,"""" the answer would be: White zinfandel." 2 the lowest possible temperature approximately -273 degrees Celsius absolute zero 20140608 "Every answer is a familiar two-word phrase or name in which the first word ends in E and the second word starts with Z. For example, given, """"popular blush wine,"""" the answer would be: White zinfandel." 3 a twelve year cycle that includes the Year of the Rat Year of the Ox and Year of the Tiger Chinese Zodiac 20140608 "Every answer is a familiar two-word phrase or name in which the first word ends in E and the second word starts with Z. For example, given, """"popular blush wine,"""" the answer would be: White zinfandel." 4 body of water in north central Switzerland Lake Zurich 20140608 "Every answer is a familiar two-word phrase or name in which the first word ends in E and the second word starts with Z. For example, given, """"popular blush wine,"""" the answer would be: White zinfandel." 5 19th Century French writer involved in the Dreyfus Affair Emile Zola 20140608 "Every answer is a familiar two-word phrase or name in which the first word ends in E and the second word starts with Z. For example, given, """"popular blush wine,"""" the answer would be: White zinfandel." 6 Capital city and its country where Robert Mugabe is president Harare, Zimbabwe 20140608 "Every answer is a familiar two-word phrase or name in which the first word ends in E and the second word starts with Z. For example, given, """"popular blush wine,"""" the answer would be: White zinfandel." 7 scientific study of organisms in the ocean marine zoology 20140608 "Every answer is a familiar two-word phrase or name in which the first word ends in E and the second word starts with Z. For example, given, """"popular blush wine,"""" the answer would be: White zinfandel." 8 food decoration made by scraping the skin of a citrus fruit orange zest 20140608 "Every answer is a familiar two-word phrase or name in which the first word ends in E and the second word starts with Z. For example, given, """"popular blush wine,"""" the answer would be: White zinfandel." 9 slangy rhyming phrase shouted when you are excited wowie zowie 20210110 Today, I've brought a game of Categories based on the word FIRST. For each category I give, name something in it starting with each of the letters F-I-R-S-T. Any answer that works is OK, and you can give the answers in any order. 1 State capitals, F Frankfurt (KY), not Fargo (ND) 20210110 Today, I've brought a game of Categories based on the word FIRST. For each category I give, name something in it starting with each of the letters F-I-R-S-T. Any answer that works is OK, and you can give the answers in any order. 2 State capitals, I Indianapolis (IN) 20210110 Today, I've brought a game of Categories based on the word FIRST. For each category I give, name something in it starting with each of the letters F-I-R-S-T. Any answer that works is OK, and you can give the answers in any order. 3 State capitals, R Raleigh (NC), Richmond (VA) 20210110 Today, I've brought a game of Categories based on the word FIRST. For each category I give, name something in it starting with each of the letters F-I-R-S-T. Any answer that works is OK, and you can give the answers in any order. 4 State capitals, S Salem (OR), Sacramento (CA), Salt Lake City (UT), Santa Fe (NM), Springfield (IL), St. Paul (MN) 20210110 Today, I've brought a game of Categories based on the word FIRST. For each category I give, name something in it starting with each of the letters F-I-R-S-T. Any answer that works is OK, and you can give the answers in any order. 5 State capitals, T Tallahassee (FL), Topeka (KS), Trenton (NJ) 20210110 Today, I've brought a game of Categories based on the word FIRST. For each category I give, name something in it starting with each of the letters F-I-R-S-T. Any answer that works is OK, and you can give the answers in any order. 6 Parts of the human body, F face, feet, forearm 20210110 Today, I've brought a game of Categories based on the word FIRST. For each category I give, name something in it starting with each of the letters F-I-R-S-T. Any answer that works is OK, and you can give the answers in any order. 7 Parts of the human body, I index finger, incisor, iris, intestines 20210110 Today, I've brought a game of Categories based on the word FIRST. For each category I give, name something in it starting with each of the letters F-I-R-S-T. Any answer that works is OK, and you can give the answers in any order. 8 Parts of the human body, R ribs, retina 20210110 Today, I've brought a game of Categories based on the word FIRST. For each category I give, name something in it starting with each of the letters F-I-R-S-T. Any answer that works is OK, and you can give the answers in any order. 9 Parts of the human body, S stomach 20210110 Today, I've brought a game of Categories based on the word FIRST. For each category I give, name something in it starting with each of the letters F-I-R-S-T. Any answer that works is OK, and you can give the answers in any order. 10 Parts of the human body, T toe, teeth 20210110 Today, I've brought a game of Categories based on the word FIRST. For each category I give, name something in it starting with each of the letters F-I-R-S-T. Any answer that works is OK, and you can give the answers in any order. 11 Terms in mathematics, F factor 20210110 Today, I've brought a game of Categories based on the word FIRST. For each category I give, name something in it starting with each of the letters F-I-R-S-T. Any answer that works is OK, and you can give the answers in any order. 12 Terms in mathematics, I integer, integral 20210110 Today, I've brought a game of Categories based on the word FIRST. For each category I give, name something in it starting with each of the letters F-I-R-S-T. Any answer that works is OK, and you can give the answers in any order. 13 Terms in mathematics, R rhombus, ratio 20210110 Today, I've brought a game of Categories based on the word FIRST. For each category I give, name something in it starting with each of the letters F-I-R-S-T. Any answer that works is OK, and you can give the answers in any order. 14 Terms in mathematics, S set, square, sum, subtract, sine 20210110 Today, I've brought a game of Categories based on the word FIRST. For each category I give, name something in it starting with each of the letters F-I-R-S-T. Any answer that works is OK, and you can give the answers in any order. 15 Terms in mathematics, T theorem, triangle, tetrahedron, trapezoid 20210110 Today, I've brought a game of Categories based on the word FIRST. For each category I give, name something in it starting with each of the letters F-I-R-S-T. Any answer that works is OK, and you can give the answers in any order. 16 Appliances you plug in, F fan, not frying pan 20210110 Today, I've brought a game of Categories based on the word FIRST. For each category I give, name something in it starting with each of the letters F-I-R-S-T. Any answer that works is OK, and you can give the answers in any order. 17 Appliances you plug in, I iron, intercom, immersion blender 20210110 Today, I've brought a game of Categories based on the word FIRST. For each category I give, name something in it starting with each of the letters F-I-R-S-T. Any answer that works is OK, and you can give the answers in any order. 18 Appliances you plug in, R refrigerator, range 20210110 Today, I've brought a game of Categories based on the word FIRST. For each category I give, name something in it starting with each of the letters F-I-R-S-T. Any answer that works is OK, and you can give the answers in any order. 19 Appliances you plug in, S stove 20210110 Today, I've brought a game of Categories based on the word FIRST. For each category I give, name something in it starting with each of the letters F-I-R-S-T. Any answer that works is OK, and you can give the answers in any order. 20 Appliances you plug in, T television, toaster 20130818 You're given two words starting with the letter S. For each pair, give a third word — also starting with S — that can follow the first one and precede the second one, in each case to complete a compound word or a familiar two-word phrase. 1 shooting, studded star 20130818 You're given two words starting with the letter S. For each pair, give a third word — also starting with S — that can follow the first one and precede the second one, in each case to complete a compound word or a familiar two-word phrase. 2 silk, stuffer stocking 20130818 You're given two words starting with the letter S. For each pair, give a third word — also starting with S — that can follow the first one and precede the second one, in each case to complete a compound word or a familiar two-word phrase. 3 swizzle, shift stick 20130818 You're given two words starting with the letter S. For each pair, give a third word — also starting with S — that can follow the first one and precede the second one, in each case to complete a compound word or a familiar two-word phrase. 4 side, stool step 20130818 You're given two words starting with the letter S. For each pair, give a third word — also starting with S — that can follow the first one and precede the second one, in each case to complete a compound word or a familiar two-word phrase. 5 sterling, screen silver 20130818 You're given two words starting with the letter S. For each pair, give a third word — also starting with S — that can follow the first one and precede the second one, in each case to complete a compound word or a familiar two-word phrase. 6 Sunday, spirit school 20130818 You're given two words starting with the letter S. For each pair, give a third word — also starting with S — that can follow the first one and precede the second one, in each case to complete a compound word or a familiar two-word phrase. 7 sea, shaker salt 20130818 You're given two words starting with the letter S. For each pair, give a third word — also starting with S — that can follow the first one and precede the second one, in each case to complete a compound word or a familiar two-word phrase. 8 safe, symbol sex 20130818 You're given two words starting with the letter S. For each pair, give a third word — also starting with S — that can follow the first one and precede the second one, in each case to complete a compound word or a familiar two-word phrase. 9 shutter, skate speed 20130818 You're given two words starting with the letter S. For each pair, give a third word — also starting with S — that can follow the first one and precede the second one, in each case to complete a compound word or a familiar two-word phrase. 10 slide, stopper show 20190414 "Every answer is a word or name in which the second syllable sounds like the letter """"K."""" The syllable is always accented, and there's at least one syllable after it." 1 Time off from work when you travel somewhere vacation 20190414 "Every answer is a word or name in which the second syllable sounds like the letter """"K."""" The syllable is always accented, and there's at least one syllable after it." 2 Mount Etna or Mount St. Helens volcano 20190414 "Every answer is a word or name in which the second syllable sounds like the letter """"K."""" The syllable is always accented, and there's at least one syllable after it." 3 17-year locust cicada 20190414 "Every answer is a word or name in which the second syllable sounds like the letter """"K."""" The syllable is always accented, and there's at least one syllable after it." 4 Distracting from real life, as fantasy novels escapist 20190414 "Every answer is a word or name in which the second syllable sounds like the letter """"K."""" The syllable is always accented, and there's at least one syllable after it." 5 A place location 20190414 "Every answer is a word or name in which the second syllable sounds like the letter """"K."""" The syllable is always accented, and there's at least one syllable after it." 6 One's profession vocation 20190414 "Every answer is a word or name in which the second syllable sounds like the letter """"K."""" The syllable is always accented, and there's at least one syllable after it." 7 Irish dramatist Seán O'Casey 20190414 "Every answer is a word or name in which the second syllable sounds like the letter """"K."""" The syllable is always accented, and there's at least one syllable after it." 8 National park along the coast of Maine Acadia 20190414 "Every answer is a word or name in which the second syllable sounds like the letter """"K."""" The syllable is always accented, and there's at least one syllable after it." 10 Dirty tricks, sleight of hand chicanery 20190414 "Every answer is a word or name in which the second syllable sounds like the letter """"K."""" The syllable is always accented, and there's at least one syllable after it." 11 Lacking the skill to do something incapable 20190414 "Every answer is a word or name in which the second syllable sounds like the letter """"K."""" The syllable is always accented, and there's at least one syllable after it." 12 Occurring naturally on a 24-hour cycle circadian 20111113 "Every answer is the name of a world capital. You'll be given a four-letter word. The first two letters are the first two letters of the city's name, and the last two are the last two letters of the country's name. For example, if you were given """"loin,"""" the answer would be London, Great Britain." 1 main (European countries here) Madrid, Spain 20111113 "Every answer is the name of a world capital. You'll be given a four-letter word. The first two letters are the first two letters of the city's name, and the last two are the last two letters of the country's name. For example, if you were given """"loin,"""" the answer would be London, Great Britain." 2 pace Paris, France 20111113 "Every answer is the name of a world capital. You'll be given a four-letter word. The first two letters are the first two letters of the city's name, and the last two are the last two letters of the country's name. For example, if you were given """"loin,"""" the answer would be London, Great Britain." 3 sten Stockholm, Sweden 20111113 "Every answer is the name of a world capital. You'll be given a four-letter word. The first two letters are the first two letters of the city's name, and the last two are the last two letters of the country's name. For example, if you were given """"loin,"""" the answer would be London, Great Britain." 4 rend Reykjavik, Iceland 20111113 "Every answer is the name of a world capital. You'll be given a four-letter word. The first two letters are the first two letters of the city's name, and the last two are the last two letters of the country's name. For example, if you were given """"loin,"""" the answer would be London, Great Britain." 5 sale (not in Europe any more) Santiago, Chile 20111113 "Every answer is the name of a world capital. You'll be given a four-letter word. The first two letters are the first two letters of the city's name, and the last two are the last two letters of the country's name. For example, if you were given """"loin,"""" the answer would be London, Great Britain." 6 khan Khartoum, Sudan 20161211 We're in the merry month of December. Every answer this week is a two-word phrase or name in which the first word starts DE- and the second word starts C. 1 a place to sit on a ship overlooking the ocean deck chair 20161211 We're in the merry month of December. Every answer this week is a two-word phrase or name in which the first word starts DE- and the second word starts C. 2 a warning sign along a road in a wooded area deer crossing 20161211 We're in the merry month of December. Every answer this week is a two-word phrase or name in which the first word starts DE- and the second word starts C. 3 a peice of plastic you use to take money out of a bank debit card 20161211 We're in the merry month of December. Every answer this week is a two-word phrase or name in which the first word starts DE- and the second word starts C. 4 a small vehicle wheeled around a restaurant with cakes, pies, etc. dessert cart 20161211 We're in the merry month of December. Every answer this week is a two-word phrase or name in which the first word starts DE- and the second word starts C. 5 What, “when I was a boy,” is, grammatically speaking dependent clause 20161211 We're in the merry month of December. Every answer this week is a two-word phrase or name in which the first word starts DE- and the second word starts C. 6 political event held in 2016 in Philadelphia and in 2012 in Charlotte, North Carolina Democratic Convention 20150809 For each word given, think of another word starting with the same two letters that can follow it to complete a familiar compound word. 1 cat call 20150809 For each word given, think of another word starting with the same two letters that can follow it to complete a familiar compound word. 2 corn cob 20150809 For each word given, think of another word starting with the same two letters that can follow it to complete a familiar compound word. 3 fan fare 20150809 For each word given, think of another word starting with the same two letters that can follow it to complete a familiar compound word. 4 fist fight 20150809 For each word given, think of another word starting with the same two letters that can follow it to complete a familiar compound word. 5 hitch hike 20150809 For each word given, think of another word starting with the same two letters that can follow it to complete a familiar compound word. 6 hot house 20150809 For each word given, think of another word starting with the same two letters that can follow it to complete a familiar compound word. 7 land lady 20150809 For each word given, think of another word starting with the same two letters that can follow it to complete a familiar compound word. 8 lime light 20150809 For each word given, think of another word starting with the same two letters that can follow it to complete a familiar compound word. 9 marsh mallow 20150809 For each word given, think of another word starting with the same two letters that can follow it to complete a familiar compound word. 10 rum runner 20110313 "You are given a sentence that contains two blanks. The word in the first blank contains a J sound. Change that to a CH sound to get the second word and complete the sentence. For example, given: """"___ ___ in a person's popularity leads to many a Google ___""""; the answer would be: """"A SURGE in a person's popularity leads to many a Google SEARCH.""""" 1 After his recent Oscar nomination for Best Actor, Jeff _____ may be getting too big for his _____. Bridges, britches 20110313 "You are given a sentence that contains two blanks. The word in the first blank contains a J sound. Change that to a CH sound to get the second word and complete the sentence. For example, given: """"___ ___ in a person's popularity leads to many a Google ___""""; the answer would be: """"A SURGE in a person's popularity leads to many a Google SEARCH.""""" 2 For the baker, it was a _____ of honor to make a _____ of cookies for the visiting dignitary. badge, batch 20110313 "You are given a sentence that contains two blanks. The word in the first blank contains a J sound. Change that to a CH sound to get the second word and complete the sentence. For example, given: """"___ ___ in a person's popularity leads to many a Google ___""""; the answer would be: """"A SURGE in a person's popularity leads to many a Google SEARCH.""""" 3 Two kids in the school cafeteria made a _____ for the last peanut butter jelly sandwich on the _____ tray. lunge, lunch 20110313 "You are given a sentence that contains two blanks. The word in the first blank contains a J sound. Change that to a CH sound to get the second word and complete the sentence. For example, given: """"___ ___ in a person's popularity leads to many a Google ___""""; the answer would be: """"A SURGE in a person's popularity leads to many a Google SEARCH.""""" 4 On the Simpsons, a new law against having blue hair in public incited _____ to _____ in protest. Marge, march 20110313 "You are given a sentence that contains two blanks. The word in the first blank contains a J sound. Change that to a CH sound to get the second word and complete the sentence. For example, given: """"___ ___ in a person's popularity leads to many a Google ___""""; the answer would be: """"A SURGE in a person's popularity leads to many a Google SEARCH.""""" 5 A gold stripe just over the mountain _____ made Jebediah a _____ man. ridge, rich 20110313 "You are given a sentence that contains two blanks. The word in the first blank contains a J sound. Change that to a CH sound to get the second word and complete the sentence. For example, given: """"___ ___ in a person's popularity leads to many a Google ___""""; the answer would be: """"A SURGE in a person's popularity leads to many a Google SEARCH.""""" 6 We have an extensive amount of shade in our yard provided by a _____ _____ tree. large, larch 20110313 "You are given a sentence that contains two blanks. The word in the first blank contains a J sound. Change that to a CH sound to get the second word and complete the sentence. For example, given: """"___ ___ in a person's popularity leads to many a Google ___""""; the answer would be: """"A SURGE in a person's popularity leads to many a Google SEARCH.""""" 7 If you get distracted while trying to _____ the top of your crème brûlée, it’s a _____ you’ll burn it. singe, cinch 20110313 "You are given a sentence that contains two blanks. The word in the first blank contains a J sound. Change that to a CH sound to get the second word and complete the sentence. For example, given: """"___ ___ in a person's popularity leads to many a Google ___""""; the answer would be: """"A SURGE in a person's popularity leads to many a Google SEARCH.""""" 8 On the baseball diamond, a _____ flyin’ in front of the mound disrupted Randy Johnson’s _____. pigeon, pitchin’ 20120930 "Every answer today is a six-letter word or name that has a repeated two-letter pair, like """"eraser,"""" which has E-R twice, or """"regret,"""" which has R-E twice. The repeated pair of letters can appear anywhere in the word. You'll be given the pair of letters and a clue, and you provide the words." 1 VE, type of fabric velvet 20120930 "Every answer today is a six-letter word or name that has a repeated two-letter pair, like """"eraser,"""" which has E-R twice, or """"regret,"""" which has R-E twice. The repeated pair of letters can appear anywhere in the word. You'll be given the pair of letters and a clue, and you provide the words." 2 CO, architectural style Rococo 20120930 "Every answer today is a six-letter word or name that has a repeated two-letter pair, like """"eraser,"""" which has E-R twice, or """"regret,"""" which has R-E twice. The repeated pair of letters can appear anywhere in the word. You'll be given the pair of letters and a clue, and you provide the words." 3 UK, footwear mukluk 20120930 "Every answer today is a six-letter word or name that has a repeated two-letter pair, like """"eraser,"""" which has E-R twice, or """"regret,"""" which has R-E twice. The repeated pair of letters can appear anywhere in the word. You'll be given the pair of letters and a clue, and you provide the words." 4 TI, artist Titian 20120930 "Every answer today is a six-letter word or name that has a repeated two-letter pair, like """"eraser,"""" which has E-R twice, or """"regret,"""" which has R-E twice. The repeated pair of letters can appear anywhere in the word. You'll be given the pair of letters and a clue, and you provide the words." 5 GE, English King George 20120930 "Every answer today is a six-letter word or name that has a repeated two-letter pair, like """"eraser,"""" which has E-R twice, or """"regret,"""" which has R-E twice. The repeated pair of letters can appear anywhere in the word. You'll be given the pair of letters and a clue, and you provide the words." 6 ON, world capital London 20120930 "Every answer today is a six-letter word or name that has a repeated two-letter pair, like """"eraser,"""" which has E-R twice, or """"regret,"""" which has R-E twice. The repeated pair of letters can appear anywhere in the word. You'll be given the pair of letters and a clue, and you provide the words." 7 AR, mountain Ararat 20120930 "Every answer today is a six-letter word or name that has a repeated two-letter pair, like """"eraser,"""" which has E-R twice, or """"regret,"""" which has R-E twice. The repeated pair of letters can appear anywhere in the word. You'll be given the pair of letters and a clue, and you provide the words." 8 TO, product of a vegetable garden tomato 20140330 This week's challenge comes from a book called Puzzles & Pleasantries, published by the Old Farmer's Almanac. Since April Fool's Day is coming up on Tuesday, the challenge questions are all April Fool-ish. Each one has appeared in a past year of the Old Farmer's Almanac, and each has a legitimate answer. 1 What goes up the hill and down the hill and yet stands still? a road 20140330 This week's challenge comes from a book called Puzzles & Pleasantries, published by the Old Farmer's Almanac. Since April Fool's Day is coming up on Tuesday, the challenge questions are all April Fool-ish. Each one has appeared in a past year of the Old Farmer's Almanac, and each has a legitimate answer. 2 What flowers can be found between the nose and the chin? tulips 20140330 This week's challenge comes from a book called Puzzles & Pleasantries, published by the Old Farmer's Almanac. Since April Fool's Day is coming up on Tuesday, the challenge questions are all April Fool-ish. Each one has appeared in a past year of the Old Farmer's Almanac, and each has a legitimate answer. 3 What is lengthened by being cut at both ends? a ditch (not smiles) 20140330 This week's challenge comes from a book called Puzzles & Pleasantries, published by the Old Farmer's Almanac. Since April Fool's Day is coming up on Tuesday, the challenge questions are all April Fool-ish. Each one has appeared in a past year of the Old Farmer's Almanac, and each has a legitimate answer. 4 What never asks questions but requires frequent answers? telephone (Malcolm's answer) or doorbell (original answer) 20140330 This week's challenge comes from a book called Puzzles & Pleasantries, published by the Old Farmer's Almanac. Since April Fool's Day is coming up on Tuesday, the challenge questions are all April Fool-ish. Each one has appeared in a past year of the Old Farmer's Almanac, and each has a legitimate answer. 5 What is it that is full of holes, yet holds water? a sponge 20191013 "Every answer today is a familiar phrase in the form """"___ to the ___."""" I'll give you the first word of the phrase. You fill in the end." 1 (starting with four-letter answers) word wise 20191013 "Every answer today is a familiar phrase in the form """"___ to the ___."""" I'll give you the first word of the phrase. You fill in the end." 2 key city 20191013 "Every answer today is a familiar phrase in the form """"___ to the ___."""" I'll give you the first word of the phrase. You fill in the end." 3 exception rule 20191013 "Every answer today is a familiar phrase in the form """"___ to the ___."""" I'll give you the first word of the phrase. You fill in the end." 4 talk hand 20191013 "Every answer today is a familiar phrase in the form """"___ to the ___."""" I'll give you the first word of the phrase. You fill in the end." 5 cut (5-letter answers now) chase, quick 20191013 "Every answer today is a familiar phrase in the form """"___ to the ___."""" I'll give you the first word of the phrase. You fill in the end." 6 off races 20191013 "Every answer today is a familiar phrase in the form """"___ to the ___."""" I'll give you the first word of the phrase. You fill in the end." 7 hail chief 20191013 "Every answer today is a familiar phrase in the form """"___ to the ___."""" I'll give you the first word of the phrase. You fill in the end." 8 pedal metal 20191013 "Every answer today is a familiar phrase in the form """"___ to the ___."""" I'll give you the first word of the phrase. You fill in the end." 9 dressed nines 20191013 "Every answer today is a familiar phrase in the form """"___ to the ___."""" I'll give you the first word of the phrase. You fill in the end." 10 preach choir 20191013 "Every answer today is a familiar phrase in the form """"___ to the ___."""" I'll give you the first word of the phrase. You fill in the end." 11 back (6-letter answers now) future 20191013 "Every answer today is a familiar phrase in the form """"___ to the ___."""" I'll give you the first word of the phrase. You fill in the end." 12 heir throne 20191013 "Every answer today is a familiar phrase in the form """"___ to the ___."""" I'll give you the first word of the phrase. You fill in the end." 13 rise (answers are 8 or more letters now) occasion 20191013 "Every answer today is a familiar phrase in the form """"___ to the ___."""" I'll give you the first word of the phrase. You fill in the end." 14 nose grindstone 20191013 "Every answer today is a familiar phrase in the form """"___ to the ___."""" I'll give you the first word of the phrase. You fill in the end." 15 good (answers have more than one word now) last drop 20191013 "Every answer today is a familiar phrase in the form """"___ to the ___."""" I'll give you the first word of the phrase. You fill in the end." 16 journey Center of the Earth 20210214 "Every answer today is a familiar phrase in the form """"___ of the ___."""" I'll give you anagrams of the first and last words. You give me the phrases. Ex. SING of the ITEMS --> SIGN OF THE TIMES1. TASTE of the RAT2. LAST of the HEART3. LILTS of the THING4. AMEN of the MAGE5. STOOL of the TREAD6. SHAPES of the MONO7. HIPS of the RESTED8. TABLET of the BUGLE9. LICENSE of the BALMS10. DEN of the NILE" 1 TASTE of the RAT state of the art 20210214 "Every answer today is a familiar phrase in the form """"___ of the ___."""" I'll give you anagrams of the first and last words. You give me the phrases. Ex. SING of the ITEMS --> SIGN OF THE TIMES1. TASTE of the RAT2. LAST of the HEART3. LILTS of the THING4. AMEN of the MAGE5. STOOL of the TREAD6. SHAPES of the MONO7. HIPS of the RESTED8. TABLET of the BUGLE9. LICENSE of the BALMS10. DEN of the NILE" 2 LAST of the HEART salt of the Earth 20210214 "Every answer today is a familiar phrase in the form """"___ of the ___."""" I'll give you anagrams of the first and last words. You give me the phrases. Ex. SING of the ITEMS --> SIGN OF THE TIMES1. TASTE of the RAT2. LAST of the HEART3. LILTS of the THING4. AMEN of the MAGE5. STOOL of the TREAD6. SHAPES of the MONO7. HIPS of the RESTED8. TABLET of the BUGLE9. LICENSE of the BALMS10. DEN of the NILE" 4 AMEN of the MAGE name of the game 20210214 "Every answer today is a familiar phrase in the form """"___ of the ___."""" I'll give you anagrams of the first and last words. You give me the phrases. Ex. SING of the ITEMS --> SIGN OF THE TIMES1. TASTE of the RAT2. LAST of the HEART3. LILTS of the THING4. AMEN of the MAGE5. STOOL of the TREAD6. SHAPES of the MONO7. HIPS of the RESTED8. TABLET of the BUGLE9. LICENSE of the BALMS10. DEN of the NILE" 5 STOOL of the TREAD tools of the trade 20210214 "Every answer today is a familiar phrase in the form """"___ of the ___."""" I'll give you anagrams of the first and last words. You give me the phrases. Ex. SING of the ITEMS --> SIGN OF THE TIMES1. TASTE of the RAT2. LAST of the HEART3. LILTS of the THING4. AMEN of the MAGE5. STOOL of the TREAD6. SHAPES of the MONO7. HIPS of the RESTED8. TABLET of the BUGLE9. LICENSE of the BALMS10. DEN of the NILE" 6 SHAPES of the MONO phases of the Moon 20210214 "Every answer today is a familiar phrase in the form """"___ of the ___."""" I'll give you anagrams of the first and last words. You give me the phrases. Ex. SING of the ITEMS --> SIGN OF THE TIMES1. TASTE of the RAT2. LAST of the HEART3. LILTS of the THING4. AMEN of the MAGE5. STOOL of the TREAD6. SHAPES of the MONO7. HIPS of the RESTED8. TABLET of the BUGLE9. LICENSE of the BALMS10. DEN of the NILE" 7 HIPS of the RESTED ship of the desert 20210214 "Every answer today is a familiar phrase in the form """"___ of the ___."""" I'll give you anagrams of the first and last words. You give me the phrases. Ex. SING of the ITEMS --> SIGN OF THE TIMES1. TASTE of the RAT2. LAST of the HEART3. LILTS of the THING4. AMEN of the MAGE5. STOOL of the TREAD6. SHAPES of the MONO7. HIPS of the RESTED8. TABLET of the BUGLE9. LICENSE of the BALMS10. DEN of the NILE" 8 TABLET of the BUGLE Battle of the Bulge 20210214 "Every answer today is a familiar phrase in the form """"___ of the ___."""" I'll give you anagrams of the first and last words. You give me the phrases. Ex. SING of the ITEMS --> SIGN OF THE TIMES1. TASTE of the RAT2. LAST of the HEART3. LILTS of the THING4. AMEN of the MAGE5. STOOL of the TREAD6. SHAPES of the MONO7. HIPS of the RESTED8. TABLET of the BUGLE9. LICENSE of the BALMS10. DEN of the NILE" 9 LICENSE of the BALMS silence of the lambs 20210214 "Every answer today is a familiar phrase in the form """"___ of the ___."""" I'll give you anagrams of the first and last words. You give me the phrases. Ex. SING of the ITEMS --> SIGN OF THE TIMES1. TASTE of the RAT2. LAST of the HEART3. LILTS of the THING4. AMEN of the MAGE5. STOOL of the TREAD6. SHAPES of the MONO7. HIPS of the RESTED8. TABLET of the BUGLE9. LICENSE of the BALMS10. DEN of the NILE" 10 DEN of the NILE end of the line 20200412 Today I've brought a word ladder with sort of an Easter egg theme. The object is to convert CHICK to HATCH, changing one letter at a time, making a new word each step of the way. I'll give you clues for the steps. You tell me the words. 1 The sound of a camera. Then change one letter in that to make ... click 20200412 Today I've brought a word ladder with sort of an Easter egg theme. The object is to convert CHICK to HATCH, changing one letter at a time, making a new word each step of the way. I'll give you clues for the steps. You tell me the words. 2 A timepiece [etc.] clock 20200412 Today I've brought a word ladder with sort of an Easter egg theme. The object is to convert CHICK to HATCH, changing one letter at a time, making a new word each step of the way. I'll give you clues for the steps. You tell me the words. 3 Unit of a city street block 20200412 Today I've brought a word ladder with sort of an Easter egg theme. The object is to convert CHICK to HATCH, changing one letter at a time, making a new word each step of the way. I'll give you clues for the steps. You tell me the words. 4 Opposite of white black 20200412 Today I've brought a word ladder with sort of an Easter egg theme. The object is to convert CHICK to HATCH, changing one letter at a time, making a new word each step of the way. I'll give you clues for the steps. You tell me the words. 5 Devoid of writing or marks blank 20200412 Today I've brought a word ladder with sort of an Easter egg theme. The object is to convert CHICK to HATCH, changing one letter at a time, making a new word each step of the way. I'll give you clues for the steps. You tell me the words. 6 Long, flat piece of lumber plank 20200412 Today I've brought a word ladder with sort of an Easter egg theme. The object is to convert CHICK to HATCH, changing one letter at a time, making a new word each step of the way. I'll give you clues for the steps. You tell me the words. 7 Carpenter's tool plane 20200412 Today I've brought a word ladder with sort of an Easter egg theme. The object is to convert CHICK to HATCH, changing one letter at a time, making a new word each step of the way. I'll give you clues for the steps. You tell me the words. 8 Locale place 20200412 Today I've brought a word ladder with sort of an Easter egg theme. The object is to convert CHICK to HATCH, changing one letter at a time, making a new word each step of the way. I'll give you clues for the steps. You tell me the words. 9 Informal word of parting peace 20200412 Today I've brought a word ladder with sort of an Easter egg theme. The object is to convert CHICK to HATCH, changing one letter at a time, making a new word each step of the way. I'll give you clues for the steps. You tell me the words. 10 A juicy fruit peach 20200412 Today I've brought a word ladder with sort of an Easter egg theme. The object is to convert CHICK to HATCH, changing one letter at a time, making a new word each step of the way. I'll give you clues for the steps. You tell me the words. 11 Locale for a summer getaway beach 20200412 Today I've brought a word ladder with sort of an Easter egg theme. The object is to convert CHICK to HATCH, changing one letter at a time, making a new word each step of the way. I'll give you clues for the steps. You tell me the words. 12 Seat for a pianist bench 20200412 Today I've brought a word ladder with sort of an Easter egg theme. The object is to convert CHICK to HATCH, changing one letter at a time, making a new word each step of the way. I'll give you clues for the steps. You tell me the words. 13 Group, as of flowers bunch 20200412 Today I've brought a word ladder with sort of an Easter egg theme. The object is to convert CHICK to HATCH, changing one letter at a time, making a new word each step of the way. I'll give you clues for the steps. You tell me the words. 14 Drink served in a big bowl punch 20200412 Today I've brought a word ladder with sort of an Easter egg theme. The object is to convert CHICK to HATCH, changing one letter at a time, making a new word each step of the way. I'll give you clues for the steps. You tell me the words. 15 Tiny amount, as of salt pinch 20200412 Today I've brought a word ladder with sort of an Easter egg theme. The object is to convert CHICK to HATCH, changing one letter at a time, making a new word each step of the way. I'll give you clues for the steps. You tell me the words. 16 Throw pitch 20200412 Today I've brought a word ladder with sort of an Easter egg theme. The object is to convert CHICK to HATCH, changing one letter at a time, making a new word each step of the way. I'll give you clues for the steps. You tell me the words. 17 What a pirate might have over one eye patch 20200412 Today I've brought a word ladder with sort of an Easter egg theme. The object is to convert CHICK to HATCH, changing one letter at a time, making a new word each step of the way. I'll give you clues for the steps. You tell me the words. 18 What chicks do hatch 20110410 "Every answer is a pair of homophones, like """"wait"""" and """"weight."""" You are given a word that can precede one homophone and follow the other, in each case, to complete a familiar two-word phrase. For example, given """"chess"""" and """"pipe,"""" the answer would be """"piece"""" and """"peace.""""" 1 anthracite _____ slaw coal, cole 20110410 "Every answer is a pair of homophones, like """"wait"""" and """"weight."""" You are given a word that can precede one homophone and follow the other, in each case, to complete a familiar two-word phrase. For example, given """"chess"""" and """"pipe,"""" the answer would be """"piece"""" and """"peace.""""" 2 bus _____ shake fare, fair 20110410 "Every answer is a pair of homophones, like """"wait"""" and """"weight."""" You are given a word that can precede one homophone and follow the other, in each case, to complete a familiar two-word phrase. For example, given """"chess"""" and """"pipe,"""" the answer would be """"piece"""" and """"peace.""""" 3 honeymoon _____ sixteen suite, sweet 20110410 "Every answer is a pair of homophones, like """"wait"""" and """"weight."""" You are given a word that can precede one homophone and follow the other, in each case, to complete a familiar two-word phrase. For example, given """"chess"""" and """"pipe,"""" the answer would be """"piece"""" and """"peace.""""" 4 false _____ sharing prophet, profit 20110410 "Every answer is a pair of homophones, like """"wait"""" and """"weight."""" You are given a word that can precede one homophone and follow the other, in each case, to complete a familiar two-word phrase. For example, given """"chess"""" and """"pipe,"""" the answer would be """"piece"""" and """"peace.""""" 5 anchors _____ game aweigh, away 20131013 "Today's puzzle is an insider's test. Every answer is a familiar two-word phrase or name with the consecutive letters T-E-S-T. Specifically, the first word will end with -TE and the second word will start ST-. For example, given """"sheer force,"""" you would say """"brute strength.""""" 1 The CS of CSA Confederate States 20131013 "Today's puzzle is an insider's test. Every answer is a familiar two-word phrase or name with the consecutive letters T-E-S-T. Specifically, the first word will end with -TE and the second word will start ST-. For example, given """"sheer force,"""" you would say """"brute strength.""""" 2 someone working for a masters degree, for example graduate student 20131013 "Today's puzzle is an insider's test. Every answer is a familiar two-word phrase or name with the consecutive letters T-E-S-T. Specifically, the first word will end with -TE and the second word will start ST-. For example, given """"sheer force,"""" you would say """"brute strength.""""" 3 cut of meat that supposedly only takes 60 seconds to cook minute steak 20131013 "Today's puzzle is an insider's test. Every answer is a familiar two-word phrase or name with the consecutive letters T-E-S-T. Specifically, the first word will end with -TE and the second word will start ST-. For example, given """"sheer force,"""" you would say """"brute strength.""""" 4 a dark brown mark that is hard to wash out of clothes chocolate stain 20131013 "Today's puzzle is an insider's test. Every answer is a familiar two-word phrase or name with the consecutive letters T-E-S-T. Specifically, the first word will end with -TE and the second word will start ST-. For example, given """"sheer force,"""" you would say """"brute strength.""""" 5 someone you have never seen before in your life complete stranger 20131013 "Today's puzzle is an insider's test. Every answer is a familiar two-word phrase or name with the consecutive letters T-E-S-T. Specifically, the first word will end with -TE and the second word will start ST-. For example, given """"sheer force,"""" you would say """"brute strength.""""" 6 bottles of wine that a vintner keeps for his own use private stock 20131013 "Today's puzzle is an insider's test. Every answer is a familiar two-word phrase or name with the consecutive letters T-E-S-T. Specifically, the first word will end with -TE and the second word will start ST-. For example, given """"sheer force,"""" you would say """"brute strength.""""" 7 mark on an envelope or package that says what day it is date stamp 20131013 "Today's puzzle is an insider's test. Every answer is a familiar two-word phrase or name with the consecutive letters T-E-S-T. Specifically, the first word will end with -TE and the second word will start ST-. For example, given """"sheer force,"""" you would say """"brute strength.""""" 8 If you meant to leave at 6:00 and you didn’t actually leave until 6:30 or 7:00, you got a late start 20131013 "Today's puzzle is an insider's test. Every answer is a familiar two-word phrase or name with the consecutive letters T-E-S-T. Specifically, the first word will end with -TE and the second word will start ST-. For example, given """"sheer force,"""" you would say """"brute strength.""""" 9 they’re plucked on classical musical instruments lute strings 20131013 "Today's puzzle is an insider's test. Every answer is a familiar two-word phrase or name with the consecutive letters T-E-S-T. Specifically, the first word will end with -TE and the second word will start ST-. For example, given """"sheer force,"""" you would say """"brute strength.""""" 10 what you hold in your hand for a toy that flies in the wind kite string 20131013 "Today's puzzle is an insider's test. Every answer is a familiar two-word phrase or name with the consecutive letters T-E-S-T. Specifically, the first word will end with -TE and the second word will start ST-. For example, given """"sheer force,"""" you would say """"brute strength.""""" 11 nickname for New Hampshire Granite State 20150111 It's another geographical puzzle this week. For each familiar two-word phrase and name, take one or more letters from the start of the first word plus one or more letters from the start of the second word. Read them in order from left to right to name a country. 1 space invaders Spain 20150111 It's another geographical puzzle this week. For each familiar two-word phrase and name, take one or more letters from the start of the first word plus one or more letters from the start of the second word. Read them in order from left to right to name a country. 2 marriage license Mali 20150111 It's another geographical puzzle this week. For each familiar two-word phrase and name, take one or more letters from the start of the first word plus one or more letters from the start of the second word. Read them in order from left to right to name a country. 3 congressional oversight Congo 20150111 It's another geographical puzzle this week. For each familiar two-word phrase and name, take one or more letters from the start of the first word plus one or more letters from the start of the second word. Read them in order from left to right to name a country. 4 international dialing India 20150111 It's another geographical puzzle this week. For each familiar two-word phrase and name, take one or more letters from the start of the first word plus one or more letters from the start of the second word. Read them in order from left to right to name a country. 5 Charles Adams Chad 20150111 It's another geographical puzzle this week. For each familiar two-word phrase and name, take one or more letters from the start of the first word plus one or more letters from the start of the second word. Read them in order from left to right to name a country. 6 Malcolm Taylor Malta 20150111 It's another geographical puzzle this week. For each familiar two-word phrase and name, take one or more letters from the start of the first word plus one or more letters from the start of the second word. Read them in order from left to right to name a country. 7 owners manual Oman 20150111 It's another geographical puzzle this week. For each familiar two-word phrase and name, take one or more letters from the start of the first word plus one or more letters from the start of the second word. Read them in order from left to right to name a country. 8 Beethoven's Ninth Benin 20150111 It's another geographical puzzle this week. For each familiar two-word phrase and name, take one or more letters from the start of the first word plus one or more letters from the start of the second word. Read them in order from left to right to name a country. 9 hair tie Haiti 20150111 It's another geographical puzzle this week. For each familiar two-word phrase and name, take one or more letters from the start of the first word plus one or more letters from the start of the second word. Read them in order from left to right to name a country. 10 personal use Peru 20150111 It's another geographical puzzle this week. For each familiar two-word phrase and name, take one or more letters from the start of the first word plus one or more letters from the start of the second word. Read them in order from left to right to name a country. 11 intermediate range Iran 20150111 It's another geographical puzzle this week. For each familiar two-word phrase and name, take one or more letters from the start of the first word plus one or more letters from the start of the second word. Read them in order from left to right to name a country. 12 Malaga wine Malawi 20191208 Every answer is a word, name or phrase in which the only consonants are B and L, repeated as often as necessary. All the other letters are vowels. 1 1. Holy book bible 20191208 Every answer is a word, name or phrase in which the only consonants are B and L, repeated as often as necessary. All the other letters are vowels. 2 2. Reason why you couldn't have committed the crime alibi 20191208 Every answer is a word, name or phrase in which the only consonants are B and L, repeated as often as necessary. All the other letters are vowels. 3 3. Record company label 20191208 Every answer is a word, name or phrase in which the only consonants are B and L, repeated as often as necessary. All the other letters are vowels. 4 "4. """"The Hobbit"""" hero ___ Baggins" Bilbo 20191208 Every answer is a word, name or phrase in which the only consonants are B and L, repeated as often as necessary. All the other letters are vowels. 5 5. Tower of ___ Babel 20191208 Every answer is a word, name or phrase in which the only consonants are B and L, repeated as often as necessary. All the other letters are vowels. 6 6. Talk rapidly and foolishly babble 20191208 Every answer is a word, name or phrase in which the only consonants are B and L, repeated as often as necessary. All the other letters are vowels. 7 7. Move up and down, as a doll's head bobble 20191208 Every answer is a word, name or phrase in which the only consonants are B and L, repeated as often as necessary. All the other letters are vowels. 8 8. Air-filled sphere sometimes made by soap bubble 20191208 Every answer is a word, name or phrase in which the only consonants are B and L, repeated as often as necessary. All the other letters are vowels. 9 9. Legally obligated liable 20191208 Every answer is a word, name or phrase in which the only consonants are B and L, repeated as often as necessary. All the other letters are vowels. 10 10. Spanish explorer who discovered the Pacific in 1513 Balboa 20191208 Every answer is a word, name or phrase in which the only consonants are B and L, repeated as often as necessary. All the other letters are vowels. 11 11. Woodcutter who foiled 40 thieves (two words) Ali Baba 20191208 Every answer is a word, name or phrase in which the only consonants are B and L, repeated as often as necessary. All the other letters are vowels. 12 12. Victim of written defamation libelee 20191208 Every answer is a word, name or phrase in which the only consonants are B and L, repeated as often as necessary. All the other letters are vowels. 13 "13. Singer Patti with the 1975 #1 hit """"Lady Marmalade""""" LaBelle 20191208 Every answer is a word, name or phrase in which the only consonants are B and L, repeated as often as necessary. All the other letters are vowels. 14 14. What you might get in the mail once a month for heating your home (two words) oil bill 20191208 Every answer is a word, name or phrase in which the only consonants are B and L, repeated as often as necessary. All the other letters are vowels. 15 15. Like the hours that a lawyer charges for billable 20110109 "Every answer is a made up two-word phrase in which the two words look like they should rhyme, but they don't. For example, given the clue """"desires trousers,"""" the answer would be """"wants pants.""""" 1 listens to grizzlies hears bears 20110109 "Every answer is a made up two-word phrase in which the two words look like they should rhyme, but they don't. For example, given the clue """"desires trousers,"""" the answer would be """"wants pants.""""" 2 exhibits cattle shows cows 20110109 "Every answer is a made up two-word phrase in which the two words look like they should rhyme, but they don't. For example, given the clue """"desires trousers,"""" the answer would be """"wants pants.""""" 3 holds stinging insects clasps wasps 20110109 "Every answer is a made up two-word phrase in which the two words look like they should rhyme, but they don't. For example, given the clue """"desires trousers,"""" the answer would be """"wants pants.""""" 4 provides mates for husbands gives wives 20110109 "Every answer is a made up two-word phrase in which the two words look like they should rhyme, but they don't. For example, given the clue """"desires trousers,"""" the answer would be """"wants pants.""""" 5 outlaws graceful birds bans swans 20110109 "Every answer is a made up two-word phrase in which the two words look like they should rhyme, but they don't. For example, given the clue """"desires trousers,"""" the answer would be """"wants pants.""""" 6 wheels out Barbies rolls dolls 20110109 "Every answer is a made up two-word phrase in which the two words look like they should rhyme, but they don't. For example, given the clue """"desires trousers,"""" the answer would be """"wants pants.""""" 7 injures hunting dogs wounds hounds 20110109 "Every answer is a made up two-word phrase in which the two words look like they should rhyme, but they don't. For example, given the clue """"desires trousers,"""" the answer would be """"wants pants.""""" 8 cuts plumbing items in two halves valves 20110109 "Every answer is a made up two-word phrase in which the two words look like they should rhyme, but they don't. For example, given the clue """"desires trousers,"""" the answer would be """"wants pants.""""" 9 takes the rough edge off magicians’ sticks sands wands 20110109 "Every answer is a made up two-word phrase in which the two words look like they should rhyme, but they don't. For example, given the clue """"desires trousers,"""" the answer would be """"wants pants.""""" 10 interrupts a series of wins breaks streaks 20110109 "Every answer is a made up two-word phrase in which the two words look like they should rhyme, but they don't. For example, given the clue """"desires trousers,"""" the answer would be """"wants pants.""""" 11 is a pedicurist does toes 20170129 Every answer today is a made up two-word phrase, in which the first word has six letters. Its last three letters spell the second word that will complete the phrase. 1 Heavy weight in Massachusetts' capital Boston ton 20170129 Every answer today is a made up two-word phrase, in which the first word has six letters. Its last three letters spell the second word that will complete the phrase. 2 Hero pilot who lives in a royal home palace ace 20170129 Every answer today is a made up two-word phrase, in which the first word has six letters. Its last three letters spell the second word that will complete the phrase. 3 Young lady who is very careful about spending money frugal gal 20170129 Every answer today is a made up two-word phrase, in which the first word has six letters. Its last three letters spell the second word that will complete the phrase. 4 Container from Mr. Spock's home planet Vulcan can 20170129 Every answer today is a made up two-word phrase, in which the first word has six letters. Its last three letters spell the second word that will complete the phrase. 5 Bear's home that is concealed hidden den 20170129 Every answer today is a made up two-word phrase, in which the first word has six letters. Its last three letters spell the second word that will complete the phrase. 6 Prohibition on headwear for Sikhs turban ban 20170129 Every answer today is a made up two-word phrase, in which the first word has six letters. Its last three letters spell the second word that will complete the phrase. 7 Collection of things in the room where you store clothes closet set 20170129 Every answer today is a made up two-word phrase, in which the first word has six letters. Its last three letters spell the second word that will complete the phrase. 8 Thin line of seats in a theater narrow row 20170129 Every answer today is a made up two-word phrase, in which the first word has six letters. Its last three letters spell the second word that will complete the phrase. 9 Boston basketball player's peculiar mannerism Celtic tic 20170129 Every answer today is a made up two-word phrase, in which the first word has six letters. Its last three letters spell the second word that will complete the phrase. 10 What an Italian tourist city has when its canals are frozen Venice ice 20170129 Every answer today is a made up two-word phrase, in which the first word has six letters. Its last three letters spell the second word that will complete the phrase. 11 """""My name is Lassie and I'm a German Shepherd,"""" for example" collie lie 20170129 Every answer today is a made up two-word phrase, in which the first word has six letters. Its last three letters spell the second word that will complete the phrase. 12 Meat sold in Batman's hometown Gotham ham 20170129 Every answer today is a made up two-word phrase, in which the first word has six letters. Its last three letters spell the second word that will complete the phrase. 13 Mafia chief in England's capitol London don 20170129 Every answer today is a made up two-word phrase, in which the first word has six letters. Its last three letters spell the second word that will complete the phrase. 14 How things are done in Oslo Norway way 20140209 "Today's puzzle is called ""Break Loose."" Every answer is a familiar two-word phrase in which the first word has a long-A vowel sound (as in ""break""), and the second word has a long-U vowel sound (as in ""loose"")." 1 beverage from Welch’s grape juice 20140209 "Today's puzzle is called ""Break Loose."" Every answer is a familiar two-word phrase in which the first word has a long-A vowel sound (as in ""break""), and the second word has a long-U vowel sound (as in ""loose"")." 2 education after kindergarten grade school 20140209 "Today's puzzle is called ""Break Loose."" Every answer is a familiar two-word phrase in which the first word has a long-A vowel sound (as in ""break""), and the second word has a long-U vowel sound (as in ""loose"")." 3 part of a building in which you might play ping pong or billiards game room 20140209 "Today's puzzle is called ""Break Loose."" Every answer is a familiar two-word phrase in which the first word has a long-A vowel sound (as in ""break""), and the second word has a long-U vowel sound (as in ""loose"")." 4 what an astronaut wears space suit 20140209 "Today's puzzle is called ""Break Loose."" Every answer is a familiar two-word phrase in which the first word has a long-A vowel sound (as in ""break""), and the second word has a long-U vowel sound (as in ""loose"")." 5 durable top for a house slate roof 20140209 "Today's puzzle is called ""Break Loose."" Every answer is a familiar two-word phrase in which the first word has a long-A vowel sound (as in ""break""), and the second word has a long-U vowel sound (as in ""loose"")." 6 vodka brand that shares its name with a dull colored fowl Grey Goose 20140209 "Today's puzzle is called ""Break Loose."" Every answer is a familiar two-word phrase in which the first word has a long-A vowel sound (as in ""break""), and the second word has a long-U vowel sound (as in ""loose"")." 7 third bimonthly issue in a calendar year May June 20140209 "Today's puzzle is called ""Break Loose."" Every answer is a familiar two-word phrase in which the first word has a long-A vowel sound (as in ""break""), and the second word has a long-U vowel sound (as in ""loose"")." 8 narrow slide often next to an elevator, in which postal letters are dropped mail shoot 20140209 "Today's puzzle is called ""Break Loose."" Every answer is a familiar two-word phrase in which the first word has a long-A vowel sound (as in ""break""), and the second word has a long-U vowel sound (as in ""loose"")." 9 what people who agree are said to sing same tune 20140209 "Today's puzzle is called ""Break Loose."" Every answer is a familiar two-word phrase in which the first word has a long-A vowel sound (as in ""break""), and the second word has a long-U vowel sound (as in ""loose"")." 10 footwear for wet weather rain boots 20140209 "Today's puzzle is called ""Break Loose."" Every answer is a familiar two-word phrase in which the first word has a long-A vowel sound (as in ""break""), and the second word has a long-U vowel sound (as in ""loose"")." 11 unvarnished facts plain truths 20140209 I'm going to give you two words. Move one letter from one of them and insert it in the other — without changing the order of any of the letters — to get two synonyms. 12 number one Beatles song with the lyric “Don’t make it bad / Take a sad song and make it better” Hey Jude 20161016 I'm going to give you two words. Move one letter from one of them and insert it in the other — without changing the order of any of the letters — to get two synonyms. 1 fiend, rally friend, ally 20161016 I'm going to give you two words. Move one letter from one of them and insert it in the other — without changing the order of any of the letters — to get two synonyms. 2 annual, repel annul, repeal 20161016 I'm going to give you two words. Move one letter from one of them and insert it in the other — without changing the order of any of the letters — to get two synonyms. 3 strip, tumble trip, stumble 20161016 I'm going to give you two words. Move one letter from one of them and insert it in the other — without changing the order of any of the letters — to get two synonyms. 4 dray, rid dry, arid 20161016 I'm going to give you two words. Move one letter from one of them and insert it in the other — without changing the order of any of the letters — to get two synonyms. 5 carful, weary careful, wary 20161016 I'm going to give you two words. Move one letter from one of them and insert it in the other — without changing the order of any of the letters — to get two synonyms. 6 though, drone through, done 20150927 Every answer this week is a familiar two-word phrase or name, in which the first word starts with the letters C and A in that order, and the second word starts with P. 1 a group of commuters in a single auto car poolers 20150927 Every answer this week is a familiar two-word phrase or name, in which the first word starts with the letters C and A in that order, and the second word starts with P. 2 a bird used to transmit messenges carrier pigeon 20150927 Every answer this week is a familiar two-word phrase or name, in which the first word starts with the letters C and A in that order, and the second word starts with P. 3 product of Del Monte or Green Giant canned peas or canned peaches 20150927 Every answer this week is a familiar two-word phrase or name, in which the first word starts with the letters C and A in that order, and the second word starts with P. 4 1920’s slang for something really cool cat’s pajamas 20150927 Every answer this week is a familiar two-word phrase or name, in which the first word starts with the letters C and A in that order, and the second word starts with P. 5 university in San Luis Obispo Cal Poly 20150927 Every answer this week is a familiar two-word phrase or name, in which the first word starts with the letters C and A in that order, and the second word starts with P. 6 baking utensil cake pan 20150927 Every answer this week is a familiar two-word phrase or name, in which the first word starts with the letters C and A in that order, and the second word starts with P. 7 something sent overseas to help those in need care package 20150927 Every answer this week is a familiar two-word phrase or name, in which the first word starts with the letters C and A in that order, and the second word starts with P. 8 a social occassion to play bridge or canasta card party 20150927 Every answer this week is a familiar two-word phrase or name, in which the first word starts with the letters C and A in that order, and the second word starts with P. 9 group of travelers written about by Chaucer Canterbury Pilgrims 20150927 Every answer this week is a familiar two-word phrase or name, in which the first word starts with the letters C and A in that order, and the second word starts with P. 10 security for a university campus police 20150927 Every answer this week is a familiar two-word phrase or name, in which the first word starts with the letters C and A in that order, and the second word starts with P. 11 Secretary of Education, or Attorney General cabinet post or cabinet position 20150927 Every answer this week is a familiar two-word phrase or name, in which the first word starts with the letters C and A in that order, and the second word starts with P. 12 most straightforward and desireable contest awards cash prize 20150531 "Because tomorrow is June 1st, today's game is one of categories, based on the word """"first."""" For each category, name something in it starting with each of the letters F-I-R-S-T. For example, if the category were """"Two-Syllable Boys' Names,"""" you might say Francis, Isaac, Richard, Simon and Tony." 1 state capitals, F Frankfort (KY); not Fargo 20150531 "Because tomorrow is June 1st, today's game is one of categories, based on the word """"first."""" For each category, name something in it starting with each of the letters F-I-R-S-T. For example, if the category were """"Two-Syllable Boys' Names,"""" you might say Francis, Isaac, Richard, Simon and Tony." 2 state capitals, I Indianapolis 20150531 "Because tomorrow is June 1st, today's game is one of categories, based on the word """"first."""" For each category, name something in it starting with each of the letters F-I-R-S-T. For example, if the category were """"Two-Syllable Boys' Names,"""" you might say Francis, Isaac, Richard, Simon and Tony." 3 state capitals, R Raleigh, Richmond 20150531 "Because tomorrow is June 1st, today's game is one of categories, based on the word """"first."""" For each category, name something in it starting with each of the letters F-I-R-S-T. For example, if the category were """"Two-Syllable Boys' Names,"""" you might say Francis, Isaac, Richard, Simon and Tony." 4 state capitals, S Salem 20150531 "Because tomorrow is June 1st, today's game is one of categories, based on the word """"first."""" For each category, name something in it starting with each of the letters F-I-R-S-T. For example, if the category were """"Two-Syllable Boys' Names,"""" you might say Francis, Isaac, Richard, Simon and Tony." 5 state capitals, T Topeka 20150531 "Because tomorrow is June 1st, today's game is one of categories, based on the word """"first."""" For each category, name something in it starting with each of the letters F-I-R-S-T. For example, if the category were """"Two-Syllable Boys' Names,"""" you might say Francis, Isaac, Richard, Simon and Tony." 6 foreign makes of car, F Fiat 20150531 "Because tomorrow is June 1st, today's game is one of categories, based on the word """"first."""" For each category, name something in it starting with each of the letters F-I-R-S-T. For example, if the category were """"Two-Syllable Boys' Names,"""" you might say Francis, Isaac, Richard, Simon and Tony." 7 foreign makes of car, I Isuzu 20150531 "Because tomorrow is June 1st, today's game is one of categories, based on the word """"first."""" For each category, name something in it starting with each of the letters F-I-R-S-T. For example, if the category were """"Two-Syllable Boys' Names,"""" you might say Francis, Isaac, Richard, Simon and Tony." 8 foreign makes of car, R Rolls Royce, Range Rover, Renault 20150531 "Because tomorrow is June 1st, today's game is one of categories, based on the word """"first."""" For each category, name something in it starting with each of the letters F-I-R-S-T. For example, if the category were """"Two-Syllable Boys' Names,"""" you might say Francis, Isaac, Richard, Simon and Tony." 9 foreign makes of car, S Saab 20150531 "Because tomorrow is June 1st, today's game is one of categories, based on the word """"first."""" For each category, name something in it starting with each of the letters F-I-R-S-T. For example, if the category were """"Two-Syllable Boys' Names,"""" you might say Francis, Isaac, Richard, Simon and Tony." 10 foreign makes of car, T Toyota 20150531 "Because tomorrow is June 1st, today's game is one of categories, based on the word """"first."""" For each category, name something in it starting with each of the letters F-I-R-S-T. For example, if the category were """"Two-Syllable Boys' Names,"""" you might say Francis, Isaac, Richard, Simon and Tony." 11 parts of a book, F frontispiece 20150531 "Because tomorrow is June 1st, today's game is one of categories, based on the word """"first."""" For each category, name something in it starting with each of the letters F-I-R-S-T. For example, if the category were """"Two-Syllable Boys' Names,"""" you might say Francis, Isaac, Richard, Simon and Tony." 12 parts of a book, I index 20150531 "Because tomorrow is June 1st, today's game is one of categories, based on the word """"first."""" For each category, name something in it starting with each of the letters F-I-R-S-T. For example, if the category were """"Two-Syllable Boys' Names,"""" you might say Francis, Isaac, Richard, Simon and Tony." 13 parts of a book, R recto, right-hand page 20150531 "Because tomorrow is June 1st, today's game is one of categories, based on the word """"first."""" For each category, name something in it starting with each of the letters F-I-R-S-T. For example, if the category were """"Two-Syllable Boys' Names,"""" you might say Francis, Isaac, Richard, Simon and Tony." 14 parts of a book, S slip cover, spine 20150531 "Because tomorrow is June 1st, today's game is one of categories, based on the word """"first."""" For each category, name something in it starting with each of the letters F-I-R-S-T. For example, if the category were """"Two-Syllable Boys' Names,"""" you might say Francis, Isaac, Richard, Simon and Tony." 15 parts of a book, T table of contents 20150531 "Because tomorrow is June 1st, today's game is one of categories, based on the word """"first."""" For each category, name something in it starting with each of the letters F-I-R-S-T. For example, if the category were """"Two-Syllable Boys' Names,"""" you might say Francis, Isaac, Richard, Simon and Tony." 16 things to take to the beach, F flip-flops 20150531 "Because tomorrow is June 1st, today's game is one of categories, based on the word """"first."""" For each category, name something in it starting with each of the letters F-I-R-S-T. For example, if the category were """"Two-Syllable Boys' Names,"""" you might say Francis, Isaac, Richard, Simon and Tony." 17 things to take to the beach, I inner tube, icebox 20150531 "Because tomorrow is June 1st, today's game is one of categories, based on the word """"first."""" For each category, name something in it starting with each of the letters F-I-R-S-T. For example, if the category were """"Two-Syllable Boys' Names,"""" you might say Francis, Isaac, Richard, Simon and Tony." 18 things to take to the beach, R radio 20150531 "Because tomorrow is June 1st, today's game is one of categories, based on the word """"first."""" For each category, name something in it starting with each of the letters F-I-R-S-T. For example, if the category were """"Two-Syllable Boys' Names,"""" you might say Francis, Isaac, Richard, Simon and Tony." 19 things to take to the beach, S sun screen 20150531 "Because tomorrow is June 1st, today's game is one of categories, based on the word """"first."""" For each category, name something in it starting with each of the letters F-I-R-S-T. For example, if the category were """"Two-Syllable Boys' Names,"""" you might say Francis, Isaac, Richard, Simon and Tony." 20 things to take to the beach, T towel 20170611 "This puzzle is called """"BBC,"""" as in the British media giant. You get two words starting with B. Come up with a word starting with C that can complete each of the first two words to complete a compound word or a familiar two-word phrase. Ex. Bob Burmese ---> CAT [bobcat, Burmese cat]" 1 baseball, bottle (3-letter answers) cap 20170611 "This puzzle is called """"BBC,"""" as in the British media giant. You get two words starting with B. Come up with a word starting with C that can complete each of the first two words to complete a compound word or a familiar two-word phrase. Ex. Bob Burmese ---> CAT [bobcat, Burmese cat]" 2 buzz, budget cut 20170611 "This puzzle is called """"BBC,"""" as in the British media giant. You get two words starting with B. Come up with a word starting with C that can complete each of the first two words to complete a compound word or a familiar two-word phrase. Ex. Bob Burmese ---> CAT [bobcat, Burmese cat]" 3 bar, building (4-letter answer) code 20170611 "This puzzle is called """"BBC,"""" as in the British media giant. You get two words starting with B. Come up with a word starting with C that can complete each of the first two words to complete a compound word or a familiar two-word phrase. Ex. Bob Burmese ---> CAT [bobcat, Burmese cat]" 4 booster, basic (5-letter answers now) cable 20170611 "This puzzle is called """"BBC,"""" as in the British media giant. You get two words starting with B. Come up with a word starting with C that can complete each of the first two words to complete a compound word or a familiar two-word phrase. Ex. Bob Burmese ---> CAT [bobcat, Burmese cat]" 5 basketball, bankruptcy court 20170611 "This puzzle is called """"BBC,"""" as in the British media giant. You get two words starting with B. Come up with a word starting with C that can complete each of the first two words to complete a compound word or a familiar two-word phrase. Ex. Bob Burmese ---> CAT [bobcat, Burmese cat]" 6 body, brownie (6-letter answer) camera 20170611 "This puzzle is called """"BBC,"""" as in the British media giant. You get two words starting with B. Come up with a word starting with C that can complete each of the first two words to complete a compound word or a familiar two-word phrase. Ex. Bob Burmese ---> CAT [bobcat, Burmese cat]" 7 bean, bargain (7-letter answer) counter 20171112 I'm going to give you some words. For each word, move one letter to a different position to spell another word. 1 HURLED hurdle 20171112 I'm going to give you some words. For each word, move one letter to a different position to spell another word. 2 GALLERY allergy 20171112 I'm going to give you some words. For each word, move one letter to a different position to spell another word. 3 INFIELD infidel 20171112 I'm going to give you some words. For each word, move one letter to a different position to spell another word. 4 SOOTHER shooter 20171112 I'm going to give you some words. For each word, move one letter to a different position to spell another word. 5 STARTLE starlet 20171112 I'm going to give you some words. For each word, move one letter to a different position to spell another word. 6 AMENDER meander 20171112 I'm going to give you some words. For each word, move one letter to a different position to spell another word. 7 WITHER writhe (Danny’s answer), whiter (Will’s intended answer) 20171112 I'm going to give you some words. For each word, move one letter to a different position to spell another word. 8 SPARED parsed 20171112 I'm going to give you some words. For each word, move one letter to a different position to spell another word. 9 TOPICAL optical 20171112 I'm going to give you some words. For each word, move one letter to a different position to spell another word. 10 PARLEYS parsley 20171112 I'm going to give you some words. For each word, move one letter to a different position to spell another word. 11 RETAIN retina 20171112 I'm going to give you some words. For each word, move one letter to a different position to spell another word. 12 NEAREST earnest 20171112 I'm going to give you some words. For each word, move one letter to a different position to spell another word. 13 SIMMERED immersed 20171112 I'm going to give you some words. For each word, move one letter to a different position to spell another word. 14 SHORE horse shoer 20190310 Today's puzzle is based on the names of famous writers of mysteries, crime fiction and spy novels. Their last names have been anagrammed. You name the writers. 1 ITCHIER + S (Agatha) Christie 20190310 Today's puzzle is based on the names of famous writers of mysteries, crime fiction and spy novels. Their last names have been anagrammed. You name the writers. 2 RANCHED + L (Raymond) Chandler 20190310 Today's puzzle is based on the names of famous writers of mysteries, crime fiction and spy novels. Their last names have been anagrammed. You name the writers. 3 ESSAY + R (Dorothy) Sayers 20190310 Today's puzzle is based on the names of famous writers of mysteries, crime fiction and spy novels. Their last names have been anagrammed. You name the writers. 4 WEAKEST + L (Donald) Westlake 20190310 Today's puzzle is based on the names of famous writers of mysteries, crime fiction and spy novels. Their last names have been anagrammed. You name the writers. 5 LENDER + L (Ruth) Rendell 20190310 Today's puzzle is based on the names of famous writers of mysteries, crime fiction and spy novels. Their last names have been anagrammed. You name the writers. 6 ORDEAL + N (Elmore) Leonard 20190310 Today's puzzle is based on the names of famous writers of mysteries, crime fiction and spy novels. Their last names have been anagrammed. You name the writers. 7 CAREER + L (John; 2 word last name) La Carre 20190310 Today's puzzle is based on the names of famous writers of mysteries, crime fiction and spy novels. Their last names have been anagrammed. You name the writers. 8 SALONS + R (Stieg) Larsson 20190310 Today's puzzle is based on the names of famous writers of mysteries, crime fiction and spy novels. Their last names have been anagrammed. You name the writers. 9 SPANIEL + L (Mickey) Spillane 20190310 Today's puzzle is based on the names of famous writers of mysteries, crime fiction and spy novels. Their last names have been anagrammed. You name the writers. 10 MINGLE + F (Ian) Fleming 20190310 Today's puzzle is based on the names of famous writers of mysteries, crime fiction and spy novels. Their last names have been anagrammed. You name the writers. 11 RANGED + R (Erle Stanley) Gardner 20190310 Today's puzzle is based on the names of famous writers of mysteries, crime fiction and spy novels. Their last names have been anagrammed. You name the writers. 12 BLAME + R (Eric) Ambler 20190310 Today's puzzle is based on the names of famous writers of mysteries, crime fiction and spy novels. Their last names have been anagrammed. You name the writers. 13 MESA + J (P.D.) James 20190310 Today's puzzle is based on the names of famous writers of mysteries, crime fiction and spy novels. Their last names have been anagrammed. You name the writers. 14 THE MAT + M (Dashiell) Hammett 20190310 Today's puzzle is based on the names of famous writers of mysteries, crime fiction and spy novels. Their last names have been anagrammed. You name the writers. 15 COLONNADE + Y (Arthur; 2 word last name) Conan Doyle 20200614 "This is a followup puzzle to last week's """"Lost ID's."""" It's called """"Replacement ID's."""" I'm going to give you some words. Each word contains the consecutive letters I-D somewhere in it. Change the I-D to two new letters to get a new familiar word.Example: Stride --> Stroke or Strafe1. Fidget2. Ideate3. Bridal4. Rancid5. Fiddle6. Afraid (hyph.)7. Collide8. Provide9. Humidity10. Consider11. Diffident12. President Last week's challenge: This challenge came from listener Chad Graham, of St. Louis. Name a well-known restaurant chain. Rearrange its letters to name a large area in the United States. This area has a two-word name. What is it?" 1 Fidget forget 20200614 "This is a followup puzzle to last week's """"Lost ID's."""" It's called """"Replacement ID's."""" I'm going to give you some words. Each word contains the consecutive letters I-D somewhere in it. Change the I-D to two new letters to get a new familiar word.Example: Stride --> Stroke or Strafe1. Fidget2. Ideate3. Bridal4. Rancid5. Fiddle6. Afraid (hyph.)7. Collide8. Provide9. Humidity10. Consider11. Diffident12. President Last week's challenge: This challenge came from listener Chad Graham, of St. Louis. Name a well-known restaurant chain. Rearrange its letters to name a large area in the United States. This area has a two-word name. What is it?" 2 Ideate create or oleate 20200614 "This is a followup puzzle to last week's """"Lost ID's."""" It's called """"Replacement ID's."""" I'm going to give you some words. Each word contains the consecutive letters I-D somewhere in it. Change the I-D to two new letters to get a new familiar word.Example: Stride --> Stroke or Strafe1. Fidget2. Ideate3. Bridal4. Rancid5. Fiddle6. Afraid (hyph.)7. Collide8. Provide9. Humidity10. Consider11. Diffident12. President Last week's challenge: This challenge came from listener Chad Graham, of St. Louis. Name a well-known restaurant chain. Rearrange its letters to name a large area in the United States. This area has a two-word name. What is it?" 3 Bridal bromal, brumal or brutal 20200614 "This is a followup puzzle to last week's """"Lost ID's."""" It's called """"Replacement ID's."""" I'm going to give you some words. Each word contains the consecutive letters I-D somewhere in it. Change the I-D to two new letters to get a new familiar word.Example: Stride --> Stroke or Strafe1. Fidget2. Ideate3. Bridal4. Rancid5. Fiddle6. Afraid (hyph.)7. Collide8. Provide9. Humidity10. Consider11. Diffident12. President Last week's challenge: This challenge came from listener Chad Graham, of St. Louis. Name a well-known restaurant chain. Rearrange its letters to name a large area in the United States. This area has a two-word name. What is it?" 4 Rancid rancho or rancor 20200614 "This is a followup puzzle to last week's """"Lost ID's."""" It's called """"Replacement ID's."""" I'm going to give you some words. Each word contains the consecutive letters I-D somewhere in it. Change the I-D to two new letters to get a new familiar word.Example: Stride --> Stroke or Strafe1. Fidget2. Ideate3. Bridal4. Rancid5. Fiddle6. Afraid (hyph.)7. Collide8. Provide9. Humidity10. Consider11. Diffident12. President Last week's challenge: This challenge came from listener Chad Graham, of St. Louis. Name a well-known restaurant chain. Rearrange its letters to name a large area in the United States. This area has a two-word name. What is it?" 5 Fiddle fondle or fuddle 20200614 "This is a followup puzzle to last week's """"Lost ID's."""" It's called """"Replacement ID's."""" I'm going to give you some words. Each word contains the consecutive letters I-D somewhere in it. Change the I-D to two new letters to get a new familiar word.Example: Stride --> Stroke or Strafe1. Fidget2. Ideate3. Bridal4. Rancid5. Fiddle6. Afraid (hyph.)7. Collide8. Provide9. Humidity10. Consider11. Diffident12. President Last week's challenge: This challenge came from listener Chad Graham, of St. Louis. Name a well-known restaurant chain. Rearrange its letters to name a large area in the United States. This area has a two-word name. What is it?" 6 Afraid (hyph.) A-frame 20200614 "This is a followup puzzle to last week's """"Lost ID's."""" It's called """"Replacement ID's."""" I'm going to give you some words. Each word contains the consecutive letters I-D somewhere in it. Change the I-D to two new letters to get a new familiar word.Example: Stride --> Stroke or Strafe1. Fidget2. Ideate3. Bridal4. Rancid5. Fiddle6. Afraid (hyph.)7. Collide8. Provide9. Humidity10. Consider11. Diffident12. President Last week's challenge: This challenge came from listener Chad Graham, of St. Louis. Name a well-known restaurant chain. Rearrange its letters to name a large area in the United States. This area has a two-word name. What is it?" 8 Provide provoke 20200614 "This is a followup puzzle to last week's """"Lost ID's."""" It's called """"Replacement ID's."""" I'm going to give you some words. Each word contains the consecutive letters I-D somewhere in it. Change the I-D to two new letters to get a new familiar word.Example: Stride --> Stroke or Strafe1. Fidget2. Ideate3. Bridal4. Rancid5. Fiddle6. Afraid (hyph.)7. Collide8. Provide9. Humidity10. Consider11. Diffident12. President Last week's challenge: This challenge came from listener Chad Graham, of St. Louis. Name a well-known restaurant chain. Rearrange its letters to name a large area in the United States. This area has a two-word name. What is it?" 9 Humidity humanity 20200614 "This is a followup puzzle to last week's """"Lost ID's."""" It's called """"Replacement ID's."""" I'm going to give you some words. Each word contains the consecutive letters I-D somewhere in it. Change the I-D to two new letters to get a new familiar word.Example: Stride --> Stroke or Strafe1. Fidget2. Ideate3. Bridal4. Rancid5. Fiddle6. Afraid (hyph.)7. Collide8. Provide9. Humidity10. Consider11. Diffident12. President Last week's challenge: This challenge came from listener Chad Graham, of St. Louis. Name a well-known restaurant chain. Rearrange its letters to name a large area in the United States. This area has a two-word name. What is it?" 10 Consider consoler or consumer 20200614 "This is a followup puzzle to last week's """"Lost ID's."""" It's called """"Replacement ID's."""" I'm going to give you some words. Each word contains the consecutive letters I-D somewhere in it. Change the I-D to two new letters to get a new familiar word.Example: Stride --> Stroke or Strafe1. Fidget2. Ideate3. Bridal4. Rancid5. Fiddle6. Afraid (hyph.)7. Collide8. Provide9. Humidity10. Consider11. Diffident12. President Last week's challenge: This challenge came from listener Chad Graham, of St. Louis. Name a well-known restaurant chain. Rearrange its letters to name a large area in the United States. This area has a two-word name. What is it?" 11 Diffident different or diffluent 20200614 "This is a followup puzzle to last week's """"Lost ID's."""" It's called """"Replacement ID's."""" I'm going to give you some words. Each word contains the consecutive letters I-D somewhere in it. Change the I-D to two new letters to get a new familiar word.Example: Stride --> Stroke or Strafe1. Fidget2. Ideate3. Bridal4. Rancid5. Fiddle6. Afraid (hyph.)7. Collide8. Provide9. Humidity10. Consider11. Diffident12. President Last week's challenge: This challenge came from listener Chad Graham, of St. Louis. Name a well-known restaurant chain. Rearrange its letters to name a large area in the United States. This area has a two-word name. What is it?" 12 President prescient 20160410 Every answer this week is an anagram of a six-letter girl's name. I'll give you the name and a synonym of its anagram. You tell me the anagram. 1 Teresa, a spring holiday Easter 20160410 Every answer this week is an anagram of a six-letter girl's name. I'll give you the name and a synonym of its anagram. You tell me the anagram. 2 Thelma, a Shakespeare play Hamlet 20160410 Every answer this week is an anagram of a six-letter girl's name. I'll give you the name and a synonym of its anagram. You tell me the anagram. 3 Lauren, imaginary unreal 20160410 Every answer this week is an anagram of a six-letter girl's name. I'll give you the name and a synonym of its anagram. You tell me the anagram. 4 Marsha, religious retreat for Hindus ashram 20160410 Every answer this week is an anagram of a six-letter girl's name. I'll give you the name and a synonym of its anagram. You tell me the anagram. 5 Marion, key of Beethoven's Symphony No. 7 (2 word answer) A minor 20191110 Here's a list of seven-letter words. For each one, change the first and last letters — but only the first and last letters — to make a new, uncapitalized seven-letter word. Both the first and last letters have to change. 1 PARTOOK cartoon 20191110 Here's a list of seven-letter words. For each one, change the first and last letters — but only the first and last letters — to make a new, uncapitalized seven-letter word. Both the first and last letters have to change. 2 TERSELY herself 20191110 Here's a list of seven-letter words. For each one, change the first and last letters — but only the first and last letters — to make a new, uncapitalized seven-letter word. Both the first and last letters have to change. 3 GUNROOM sunroof 20191110 Here's a list of seven-letter words. For each one, change the first and last letters — but only the first and last letters — to make a new, uncapitalized seven-letter word. Both the first and last letters have to change. 4 HELLISH cellist 20191110 Here's a list of seven-letter words. For each one, change the first and last letters — but only the first and last letters — to make a new, uncapitalized seven-letter word. Both the first and last letters have to change. 5 LORELEI foreleg 20191110 Here's a list of seven-letter words. For each one, change the first and last letters — but only the first and last letters — to make a new, uncapitalized seven-letter word. Both the first and last letters have to change. 6 CARCASS sarcasm 20191110 Here's a list of seven-letter words. For each one, change the first and last letters — but only the first and last letters — to make a new, uncapitalized seven-letter word. Both the first and last letters have to change. 7 GORDIAN cordial 20191110 Here's a list of seven-letter words. For each one, change the first and last letters — but only the first and last letters — to make a new, uncapitalized seven-letter word. Both the first and last letters have to change. 8 LIGNITE dignity 20191110 Here's a list of seven-letter words. For each one, change the first and last letters — but only the first and last letters — to make a new, uncapitalized seven-letter word. Both the first and last letters have to change. 9 PARTING martini 20190428 Every answer today is the name of a famous person with only one name — either because they literally had only one name or because they chose to go by one name in show business. I'll give you anagrams. You name the people. 1 perch - p Cher 20190428 Every answer today is the name of a famous person with only one name — either because they literally had only one name or because they chose to go by one name in show business. I'll give you anagrams. You name the people. 2 yearn - r Enya 20190428 Every answer today is the name of a famous person with only one name — either because they literally had only one name or because they chose to go by one name in show business. I'll give you anagrams. You name the people. 3 sleep - s Pele 20190428 Every answer today is the name of a famous person with only one name — either because they literally had only one name or because they chose to go by one name in show business. I'll give you anagrams. You name the people. 4 operas - r Aesop 20190428 Every answer today is the name of a famous person with only one name — either because they literally had only one name or because they chose to go by one name in show business. I'll give you anagrams. You name the people. 5 cupular - c RuPaul 20190428 Every answer today is the name of a famous person with only one name — either because they literally had only one name or because they chose to go by one name in show business. I'll give you anagrams. You name the people. 6 porcine - o Prince 20190428 Every answer today is the name of a famous person with only one name — either because they literally had only one name or because they chose to go by one name in show business. I'll give you anagrams. You name the people. 7 immense - s Eminem 20190428 Every answer today is the name of a famous person with only one name — either because they literally had only one name or because they chose to go by one name in show business. I'll give you anagrams. You name the people. 8 monologs - g Solomon 20190428 Every answer today is the name of a famous person with only one name — either because they literally had only one name or because they chose to go by one name in show business. I'll give you anagrams. You name the people. 9 ergonomic - c Geronimo 20190428 Every answer today is the name of a famous person with only one name — either because they literally had only one name or because they chose to go by one name in show business. I'll give you anagrams. You name the people. 10 bracelike - k Liberace 20130609 "Categories are given based on the name """"Homer,"""" the name of a town in Alaska. Name something in the category beginning with each of the letters H-O-M-E-R. For example, if the category were """"Chemical Elements,"""" you might say Helium, Oxygen, Magnesium, Einsteinium, and Radon. You can give the answers in any order." 1 O M 20130609 "Categories are given based on the name """"Homer,"""" the name of a town in Alaska. Name something in the category beginning with each of the letters H-O-M-E-R. For example, if the category were """"Chemical Elements,"""" you might say Helium, Oxygen, Magnesium, Einsteinium, and Radon. You can give the answers in any order." 2 E R 20130609 "Categories are given based on the name """"Homer,"""" the name of a town in Alaska. Name something in the category beginning with each of the letters H-O-M-E-R. For example, if the category were """"Chemical Elements,"""" you might say Helium, Oxygen, Magnesium, Einsteinium, and Radon. You can give the answers in any order." 3 trees hazelnut, hemlock, hickory 20130609 "Categories are given based on the name """"Homer,"""" the name of a town in Alaska. Name something in the category beginning with each of the letters H-O-M-E-R. For example, if the category were """"Chemical Elements,"""" you might say Helium, Oxygen, Magnesium, Einsteinium, and Radon. You can give the answers in any order." 4 oak maple 20130609 "Categories are given based on the name """"Homer,"""" the name of a town in Alaska. Name something in the category beginning with each of the letters H-O-M-E-R. For example, if the category were """"Chemical Elements,"""" you might say Helium, Oxygen, Magnesium, Einsteinium, and Radon. You can give the answers in any order." 5 elm redwood rubber 20130609 "Categories are given based on the name """"Homer,"""" the name of a town in Alaska. Name something in the category beginning with each of the letters H-O-M-E-R. For example, if the category were """"Chemical Elements,"""" you might say Helium, Oxygen, Magnesium, Einsteinium, and Radon. You can give the answers in any order." 6 makes of automobile that are no longer made Hummer, Hudson, Hupmobile 20130609 "Categories are given based on the name """"Homer,"""" the name of a town in Alaska. Name something in the category beginning with each of the letters H-O-M-E-R. For example, if the category were """"Chemical Elements,"""" you might say Helium, Oxygen, Magnesium, Einsteinium, and Radon. You can give the answers in any order." 7 Oldsmobile Mercury 20130609 "Categories are given based on the name """"Homer,"""" the name of a town in Alaska. Name something in the category beginning with each of the letters H-O-M-E-R. For example, if the category were """"Chemical Elements,"""" you might say Helium, Oxygen, Magnesium, Einsteinium, and Radon. You can give the answers in any order." 8 Edsel REO 20130609 "Categories are given based on the name """"Homer,"""" the name of a town in Alaska. Name something in the category beginning with each of the letters H-O-M-E-R. For example, if the category were """"Chemical Elements,"""" you might say Helium, Oxygen, Magnesium, Einsteinium, and Radon. You can give the answers in any order." 9 things commonly found in a refrigerator hamburgers 20130609 "Categories are given based on the name """"Homer,"""" the name of a town in Alaska. Name something in the category beginning with each of the letters H-O-M-E-R. For example, if the category were """"Chemical Elements,"""" you might say Helium, Oxygen, Magnesium, Einsteinium, and Radon. You can give the answers in any order." 10 olives, orange juice, oleo mayonaise milk, mustard 20130609 "Categories are given based on the name """"Homer,"""" the name of a town in Alaska. Name something in the category beginning with each of the letters H-O-M-E-R. For example, if the category were """"Chemical Elements,"""" you might say Helium, Oxygen, Magnesium, Einsteinium, and Radon. You can give the answers in any order." 11 eggs relish 20180408 "Tonight I'm going to be on the Fox sitcom """"Brooklyn Nine-Nine."""" In today's puzzle I'm going to tell you my experience in filming the show. Anytime you hear an error of fact, logic, or word usage, say """"Buzz!"""" and explain why. And just so you know, no specialized knowledge is needed." 1 ABC FOX 20180408 "Tonight I'm going to be on the Fox sitcom """"Brooklyn Nine-Nine."""" In today's puzzle I'm going to tell you my experience in filming the show. Anytime you hear an error of fact, logic, or word usage, say """"Buzz!"""" and explain why. And just so you know, no specialized knowledge is needed." 2 Brookline, Massachusetts Brooklyn, New York 20180408 "Tonight I'm going to be on the Fox sitcom """"Brooklyn Nine-Nine."""" In today's puzzle I'm going to tell you my experience in filming the show. Anytime you hear an error of fact, logic, or word usage, say """"Buzz!"""" and explain why. And just so you know, no specialized knowledge is needed." 3 New York Post New York Times 20180408 "Tonight I'm going to be on the Fox sitcom """"Brooklyn Nine-Nine."""" In today's puzzle I'm going to tell you my experience in filming the show. Anytime you hear an error of fact, logic, or word usage, say """"Buzz!"""" and explain why. And just so you know, no specialized knowledge is needed." 4 San Francisco Los Angeles 20180408 "Tonight I'm going to be on the Fox sitcom """"Brooklyn Nine-Nine."""" In today's puzzle I'm going to tell you my experience in filming the show. Anytime you hear an error of fact, logic, or word usage, say """"Buzz!"""" and explain why. And just so you know, no specialized knowledge is needed." 5 Brooklyn 9-1-1 Brooklyn Nine-Nine 20180408 "Tonight I'm going to be on the Fox sitcom """"Brooklyn Nine-Nine."""" In today's puzzle I'm going to tell you my experience in filming the show. Anytime you hear an error of fact, logic, or word usage, say """"Buzz!"""" and explain why. And just so you know, no specialized knowledge is needed." 6 everyone on the set were super nice everyone ... was ... 20180408 "Tonight I'm going to be on the Fox sitcom """"Brooklyn Nine-Nine."""" In today's puzzle I'm going to tell you my experience in filming the show. Anytime you hear an error of fact, logic, or word usage, say """"Buzz!"""" and explain why. And just so you know, no specialized knowledge is needed." 7 recreates several square blocks of make-believe Brooklyn Either Brooklyn is not make-believe or the set does not “recreate” them. 20180408 "Tonight I'm going to be on the Fox sitcom """"Brooklyn Nine-Nine."""" In today's puzzle I'm going to tell you my experience in filming the show. Anytime you hear an error of fact, logic, or word usage, say """"Buzz!"""" and explain why. And just so you know, no specialized knowledge is needed." 8 ASCAP SAG-AFTRA 20180408 "Tonight I'm going to be on the Fox sitcom """"Brooklyn Nine-Nine."""" In today's puzzle I'm going to tell you my experience in filming the show. Anytime you hear an error of fact, logic, or word usage, say """"Buzz!"""" and explain why. And just so you know, no specialized knowledge is needed." 10 Susan Samberg Susan Stamberg 20180408 "Tonight I'm going to be on the Fox sitcom """"Brooklyn Nine-Nine."""" In today's puzzle I'm going to tell you my experience in filming the show. Anytime you hear an error of fact, logic, or word usage, say """"Buzz!"""" and explain why. And just so you know, no specialized knowledge is needed." 11 8:30 am 8:30 pm 20170430 "I'm going to read some sentences. Each sentence has two blanks. Put two five-letter homophones in the blanks to complete the sentence. For example: If I said, """"Criminals went to a Pittsburgh factory late at night to ___ ___,"""" you'd say they went their to """"steal steel.""""" 1 Because I never took a class in penmanship, I never learned to ___ ___. write right 20170430 "I'm going to read some sentences. Each sentence has two blanks. Put two five-letter homophones in the blanks to complete the sentence. For example: If I said, """"Criminals went to a Pittsburgh factory late at night to ___ ___,"""" you'd say they went their to """"steal steel.""""" 2 The driver jammed his foot on the pedal so hard, he made the ___ ___. brake break 20170430 "I'm going to read some sentences. Each sentence has two blanks. Put two five-letter homophones in the blanks to complete the sentence. For example: If I said, """"Criminals went to a Pittsburgh factory late at night to ___ ___,"""" you'd say they went their to """"steal steel.""""" 3 One of the Shakespeare character messed up the line 'Double, double toil in trouble,' but I couldn't identify ___ ___. which witch 20170430 "I'm going to read some sentences. Each sentence has two blanks. Put two five-letter homophones in the blanks to complete the sentence. For example: If I said, """"Criminals went to a Pittsburgh factory late at night to ___ ___,"""" you'd say they went their to """"steal steel.""""" 4 The increase in car rental rates really hits the pocket book hard. So that latest charge from ___ ___. Hertz hurts 20170430 "I'm going to read some sentences. Each sentence has two blanks. Put two five-letter homophones in the blanks to complete the sentence. For example: If I said, """"Criminals went to a Pittsburgh factory late at night to ___ ___,"""" you'd say they went their to """"steal steel.""""" 5 The host of The Price Is Right is bringing in boxes from his car, but how many can ___ ___? Carey carry 20101031 "You are given categories. For each of the categories, name an item starting with each of the letters in """"ghost."""" For example, given the category """"girls' names,"""" answers could be Greta, Hilda, Olive, Sue and Trish." 1 musical instruments, G guitar 20101031 "You are given categories. For each of the categories, name an item starting with each of the letters in """"ghost."""" For example, given the category """"girls' names,"""" answers could be Greta, Hilda, Olive, Sue and Trish." 2 musical instruments, H harp 20101031 "You are given categories. For each of the categories, name an item starting with each of the letters in """"ghost."""" For example, given the category """"girls' names,"""" answers could be Greta, Hilda, Olive, Sue and Trish." 3 musical instruments, O oboe 20101031 "You are given categories. For each of the categories, name an item starting with each of the letters in """"ghost."""" For example, given the category """"girls' names,"""" answers could be Greta, Hilda, Olive, Sue and Trish." 4 musical instruments, S sousaphone 20101031 "You are given categories. For each of the categories, name an item starting with each of the letters in """"ghost."""" For example, given the category """"girls' names,"""" answers could be Greta, Hilda, Olive, Sue and Trish." 5 musical instruments, T trumpet 20101031 "You are given categories. For each of the categories, name an item starting with each of the letters in """"ghost."""" For example, given the category """"girls' names,"""" answers could be Greta, Hilda, Olive, Sue and Trish." 6 fruits, G guava 20101031 "You are given categories. For each of the categories, name an item starting with each of the letters in """"ghost."""" For example, given the category """"girls' names,"""" answers could be Greta, Hilda, Olive, Sue and Trish." 7 fruits, H huckleberry 20101031 "You are given categories. For each of the categories, name an item starting with each of the letters in """"ghost."""" For example, given the category """"girls' names,"""" answers could be Greta, Hilda, Olive, Sue and Trish." 8 fruits, O orange 20101031 "You are given categories. For each of the categories, name an item starting with each of the letters in """"ghost."""" For example, given the category """"girls' names,"""" answers could be Greta, Hilda, Olive, Sue and Trish." 9 fruits, S strawberry 20101031 "You are given categories. For each of the categories, name an item starting with each of the letters in """"ghost."""" For example, given the category """"girls' names,"""" answers could be Greta, Hilda, Olive, Sue and Trish." 10 fruits, T tomato 20101031 "You are given categories. For each of the categories, name an item starting with each of the letters in """"ghost."""" For example, given the category """"girls' names,"""" answers could be Greta, Hilda, Olive, Sue and Trish." 11 common names for newspapers, G Globe 20101031 "You are given categories. For each of the categories, name an item starting with each of the letters in """"ghost."""" For example, given the category """"girls' names,"""" answers could be Greta, Hilda, Olive, Sue and Trish." 12 common names for newspapers, H Herald 20101031 "You are given categories. For each of the categories, name an item starting with each of the letters in """"ghost."""" For example, given the category """"girls' names,"""" answers could be Greta, Hilda, Olive, Sue and Trish." 13 common names for newspapers, O Observer 20101031 "You are given categories. For each of the categories, name an item starting with each of the letters in """"ghost."""" For example, given the category """"girls' names,"""" answers could be Greta, Hilda, Olive, Sue and Trish." 14 common names for newspapers, S Sentinel 20101031 "You are given categories. For each of the categories, name an item starting with each of the letters in """"ghost."""" For example, given the category """"girls' names,"""" answers could be Greta, Hilda, Olive, Sue and Trish." 15 common names for newspapers, T Times 20101031 "You are given categories. For each of the categories, name an item starting with each of the letters in """"ghost."""" For example, given the category """"girls' names,"""" answers could be Greta, Hilda, Olive, Sue and Trish." 16 Olympic host cities, G Grenoble 20101031 "You are given categories. For each of the categories, name an item starting with each of the letters in """"ghost."""" For example, given the category """"girls' names,"""" answers could be Greta, Hilda, Olive, Sue and Trish." 17 Olympic host cities, H Helsinki 20101031 "You are given categories. For each of the categories, name an item starting with each of the letters in """"ghost."""" For example, given the category """"girls' names,"""" answers could be Greta, Hilda, Olive, Sue and Trish." 18 Olympic host cities, O Oslo 20101031 "You are given categories. For each of the categories, name an item starting with each of the letters in """"ghost."""" For example, given the category """"girls' names,"""" answers could be Greta, Hilda, Olive, Sue and Trish." 19 Olympic host cities, S Salt Lake City 20101031 "You are given categories. For each of the categories, name an item starting with each of the letters in """"ghost."""" For example, given the category """"girls' names,"""" answers could be Greta, Hilda, Olive, Sue and Trish." 20 Olympic host cities, T Torino 20101031 "You are given categories. For each of the categories, name an item starting with each of the letters in """"ghost."""" For example, given the category """"girls' names,"""" answers could be Greta, Hilda, Olive, Sue and Trish." 21 things seen in a car interior, G gear shift, glove compartment, gas pedal, gas gage 20101031 "You are given categories. For each of the categories, name an item starting with each of the letters in """"ghost."""" For example, given the category """"girls' names,"""" answers could be Greta, Hilda, Olive, Sue and Trish." 22 things seen in a car interior, H headlight, horn, headrest 20101031 "You are given categories. For each of the categories, name an item starting with each of the letters in """"ghost."""" For example, given the category """"girls' names,"""" answers could be Greta, Hilda, Olive, Sue and Trish." 23 things seen in a car interior, O odometer 20101031 "You are given categories. For each of the categories, name an item starting with each of the letters in """"ghost."""" For example, given the category """"girls' names,"""" answers could be Greta, Hilda, Olive, Sue and Trish." 24 things seen in a car interior, S seat belt 20101031 "You are given categories. For each of the categories, name an item starting with each of the letters in """"ghost."""" For example, given the category """"girls' names,"""" answers could be Greta, Hilda, Olive, Sue and Trish." 25 things seen in a car interior, T turn signal 20150614 For the blank in each provided sentence, put in the name of a color to complete the sentence in a punny way. 1 To avoid a ____ the wound heal naturally, S scarlet 20150614 For the blank in each provided sentence, put in the name of a color to complete the sentence in a punny way. 2 For many older movie-goers, action films these days are too ___, V violet 20150614 For the blank in each provided sentence, put in the name of a color to complete the sentence in a punny way. 3 We stayed up all night watching for shooting stars, but we didn’t _____, S sienna 20150614 For the blank in each provided sentence, put in the name of a color to complete the sentence in a punny way. 4 The attorney said to her client, _____ lawyer, I recommend you keep quiet, A azure 20150614 For the blank in each provided sentence, put in the name of a color to complete the sentence in a punny way. 5 Louis Armstrong was a professional ___, J jasmine 20150614 For the blank in each provided sentence, put in the name of a color to complete the sentence in a punny way. 6 In packing my suitcase, I made room for some more items, and now _____, I indigo 20200726 I'm going to give you some words. For each one, change one letter to two new letters to name a country. 1 1. Fence France 20200726 I'm going to give you some words. For each one, change one letter to two new letters to name a country. 2 2. Grace Greece 20200726 I'm going to give you some words. For each one, change one letter to two new letters to name a country. 3 3. Brawl Brazil 20200726 I'm going to give you some words. For each one, change one letter to two new letters to name a country. 4 4. No Say Norway 20200726 I'm going to give you some words. For each one, change one letter to two new letters to name a country. 5 5. Polar Poland 20200726 I'm going to give you some words. For each one, change one letter to two new letters to name a country. 6 6. Debark Denmark 20200726 I'm going to give you some words. For each one, change one letter to two new letters to name a country. 7 7. Brunt Brunei 20200726 I'm going to give you some words. For each one, change one letter to two new letters to name a country. 8 8. Mondo Monaco 20200726 I'm going to give you some words. For each one, change one letter to two new letters to name a country. 9 9. Malaria Malasia 20200726 I'm going to give you some words. For each one, change one letter to two new letters to name a country. 10 10. Panda (three different answers) Panama, Rwanda and Uganda 20120812 "You are given the ends of the names of three things that are all in the same category. You name the category. For example, """"fur,"""" """"dine"""" and """"sten"""" are all ends of chemical elements (sulfur, iodine, tungsten)." 1 arch, gust, ember months (March, August, September) 20120812 "You are given the ends of the names of three things that are all in the same category. You name the category. For example, """"fur,"""" """"dine"""" and """"sten"""" are all ends of chemical elements (sulfur, iodine, tungsten)." 2 bra, corn, mini signs of the zodiac (Libra, Capricorn, Gemini) 20120812 "You are given the ends of the names of three things that are all in the same category. You name the category. For example, """"fur,"""" """"dine"""" and """"sten"""" are all ends of chemical elements (sulfur, iodine, tungsten)." 3 sea, sis, verbs books of the Bible (Hosea, Genesis, Proverbs) 20120812 "You are given the ends of the names of three things that are all in the same category. You name the category. For example, """"fur,"""" """"dine"""" and """"sten"""" are all ends of chemical elements (sulfur, iodine, tungsten)." 4 pet, bone, soon musical instruments (trumpet, trombone, bassoon) 20120812 "You are given the ends of the names of three things that are all in the same category. You name the category. For example, """"fur,"""" """"dine"""" and """"sten"""" are all ends of chemical elements (sulfur, iodine, tungsten)." 5 den, many, pain European countries (Sweden, Germany, Spain) 20120812 "You are given the ends of the names of three things that are all in the same category. You name the category. For example, """"fur,"""" """"dine"""" and """"sten"""" are all ends of chemical elements (sulfur, iodine, tungsten)." 6 mute, stiff, eagle breeds of dog (malamute, mastiff, beagle) 20120812 "You are given the ends of the names of three things that are all in the same category. You name the category. For example, """"fur,"""" """"dine"""" and """"sten"""" are all ends of chemical elements (sulfur, iodine, tungsten)." 7 love, lox, gold flowers (foxglove, phlox marigold) 20120812 "You are given the ends of the names of three things that are all in the same category. You name the category. For example, """"fur,"""" """"dine"""" and """"sten"""" are all ends of chemical elements (sulfur, iodine, tungsten)." 8 see, cut, sin states (Tennessee, Connecticut, Wisconsin) 20120812 "You are given the ends of the names of three things that are all in the same category. You name the category. For example, """"fur,"""" """"dine"""" and """"sten"""" are all ends of chemical elements (sulfur, iodine, tungsten)." 9 din, ella, Asia Disney movies (Aladdin, Cinderella, Fantasia) 20201025 I'm going to give you some common five-letter words in Spanish. For each one, rearrange the letters to spell a common, uncapitalized word in English.Example: CESTA (basket) --> CASTE1. TODOS (all or every)2. TRUCO (trick)3. BANCO (bank)4. ARROZ (rice)5. CINCO (five)6. JABON (soap)7. TORRE (tower)8. PECHO (chest)9. HUESO (bone)10. ODIAR (to hate) 2 TRUCO (trick) court 20201025 I'm going to give you some common five-letter words in Spanish. For each one, rearrange the letters to spell a common, uncapitalized word in English.Example: CESTA (basket) --> CASTE1. TODOS (all or every)2. TRUCO (trick)3. BANCO (bank)4. ARROZ (rice)5. CINCO (five)6. JABON (soap)7. TORRE (tower)8. PECHO (chest)9. HUESO (bone)10. ODIAR (to hate) 3 BANCO (bank) bacon 20201025 I'm going to give you some common five-letter words in Spanish. For each one, rearrange the letters to spell a common, uncapitalized word in English.Example: CESTA (basket) --> CASTE1. TODOS (all or every)2. TRUCO (trick)3. BANCO (bank)4. ARROZ (rice)5. CINCO (five)6. JABON (soap)7. TORRE (tower)8. PECHO (chest)9. HUESO (bone)10. ODIAR (to hate) 4 ARROZ (rice) razor 20201025 I'm going to give you some common five-letter words in Spanish. For each one, rearrange the letters to spell a common, uncapitalized word in English.Example: CESTA (basket) --> CASTE1. TODOS (all or every)2. TRUCO (trick)3. BANCO (bank)4. ARROZ (rice)5. CINCO (five)6. JABON (soap)7. TORRE (tower)8. PECHO (chest)9. HUESO (bone)10. ODIAR (to hate) 5 CINCO (five) conic 20201025 I'm going to give you some common five-letter words in Spanish. For each one, rearrange the letters to spell a common, uncapitalized word in English.Example: CESTA (basket) --> CASTE1. TODOS (all or every)2. TRUCO (trick)3. BANCO (bank)4. ARROZ (rice)5. CINCO (five)6. JABON (soap)7. TORRE (tower)8. PECHO (chest)9. HUESO (bone)10. ODIAR (to hate) 6 JABON (soap) banjo 20201025 I'm going to give you some common five-letter words in Spanish. For each one, rearrange the letters to spell a common, uncapitalized word in English.Example: CESTA (basket) --> CASTE1. TODOS (all or every)2. TRUCO (trick)3. BANCO (bank)4. ARROZ (rice)5. CINCO (five)6. JABON (soap)7. TORRE (tower)8. PECHO (chest)9. HUESO (bone)10. ODIAR (to hate) 7 TORRE (tower) retro 20201025 I'm going to give you some common five-letter words in Spanish. For each one, rearrange the letters to spell a common, uncapitalized word in English.Example: CESTA (basket) --> CASTE1. TODOS (all or every)2. TRUCO (trick)3. BANCO (bank)4. ARROZ (rice)5. CINCO (five)6. JABON (soap)7. TORRE (tower)8. PECHO (chest)9. HUESO (bone)10. ODIAR (to hate) 8 PECHO (chest) epoch 20201025 I'm going to give you some common five-letter words in Spanish. For each one, rearrange the letters to spell a common, uncapitalized word in English.Example: CESTA (basket) --> CASTE1. TODOS (all or every)2. TRUCO (trick)3. BANCO (bank)4. ARROZ (rice)5. CINCO (five)6. JABON (soap)7. TORRE (tower)8. PECHO (chest)9. HUESO (bone)10. ODIAR (to hate) 9 HUESO (bone) house 20201025 I'm going to give you some common five-letter words in Spanish. For each one, rearrange the letters to spell a common, uncapitalized word in English.Example: CESTA (basket) --> CASTE1. TODOS (all or every)2. TRUCO (trick)3. BANCO (bank)4. ARROZ (rice)5. CINCO (five)6. JABON (soap)7. TORRE (tower)8. PECHO (chest)9. HUESO (bone)10. ODIAR (to hate) 10 ODIAR (to hate) radio 20140511 "Today's puzzle is a game of categories based on the word """"phlox."""" You'll be given some categories. For each one, name something in the category beginning with each of the letters P, H, L, O and X. For example, if the category were """"comic books,"""" you might say """"Phantom, Hopalong Cassidy, Lone Ranger, Our Gang Comics and X-men."""" Any answer that works is okay, and you can give the answers in any order." 1 H L 20140511 "Today's puzzle is a game of categories based on the word """"phlox."""" You'll be given some categories. For each one, name something in the category beginning with each of the letters P, H, L, O and X. For example, if the category were """"comic books,"""" you might say """"Phantom, Hopalong Cassidy, Lone Ranger, Our Gang Comics and X-men."""" Any answer that works is okay, and you can give the answers in any order." 2 O X 20140511 "Today's puzzle is a game of categories based on the word """"phlox."""" You'll be given some categories. For each one, name something in the category beginning with each of the letters P, H, L, O and X. For example, if the category were """"comic books,"""" you might say """"Phantom, Hopalong Cassidy, Lone Ranger, Our Gang Comics and X-men."""" Any answer that works is okay, and you can give the answers in any order." 3 chemical elements potassium 20140511 "Today's puzzle is a game of categories based on the word """"phlox."""" You'll be given some categories. For each one, name something in the category beginning with each of the letters P, H, L, O and X. For example, if the category were """"comic books,"""" you might say """"Phantom, Hopalong Cassidy, Lone Ranger, Our Gang Comics and X-men."""" Any answer that works is okay, and you can give the answers in any order." 4 helium lithium 20140511 "Today's puzzle is a game of categories based on the word """"phlox."""" You'll be given some categories. For each one, name something in the category beginning with each of the letters P, H, L, O and X. For example, if the category were """"comic books,"""" you might say """"Phantom, Hopalong Cassidy, Lone Ranger, Our Gang Comics and X-men."""" Any answer that works is okay, and you can give the answers in any order." 5 oxygen, osmium, not Oregon xenon 20140511 "Today's puzzle is a game of categories based on the word """"phlox."""" You'll be given some categories. For each one, name something in the category beginning with each of the letters P, H, L, O and X. For example, if the category were """"comic books,"""" you might say """"Phantom, Hopalong Cassidy, Lone Ranger, Our Gang Comics and X-men."""" Any answer that works is okay, and you can give the answers in any order." 6 musical instruments piano 20140511 "Today's puzzle is a game of categories based on the word """"phlox."""" You'll be given some categories. For each one, name something in the category beginning with each of the letters P, H, L, O and X. For example, if the category were """"comic books,"""" you might say """"Phantom, Hopalong Cassidy, Lone Ranger, Our Gang Comics and X-men."""" Any answer that works is okay, and you can give the answers in any order." 7 harpsi- chord lyre 20140511 "Today's puzzle is a game of categories based on the word """"phlox."""" You'll be given some categories. For each one, name something in the category beginning with each of the letters P, H, L, O and X. For example, if the category were """"comic books,"""" you might say """"Phantom, Hopalong Cassidy, Lone Ranger, Our Gang Comics and X-men."""" Any answer that works is okay, and you can give the answers in any order." 8 organ xylophone 20140511 "Today's puzzle is a game of categories based on the word """"phlox."""" You'll be given some categories. For each one, name something in the category beginning with each of the letters P, H, L, O and X. For example, if the category were """"comic books,"""" you might say """"Phantom, Hopalong Cassidy, Lone Ranger, Our Gang Comics and X-men."""" Any answer that works is okay, and you can give the answers in any order." 9 Fortune 500 companies Phillips 66 20140511 "Today's puzzle is a game of categories based on the word """"phlox."""" You'll be given some categories. For each one, name something in the category beginning with each of the letters P, H, L, O and X. For example, if the category were """"comic books,"""" you might say """"Phantom, Hopalong Cassidy, Lone Ranger, Our Gang Comics and X-men."""" Any answer that works is okay, and you can give the answers in any order." 10 Hess, HP, Home Depot Lockheed Martin 20140511 "Today's puzzle is a game of categories based on the word """"phlox."""" You'll be given some categories. For each one, name something in the category beginning with each of the letters P, H, L, O and X. For example, if the category were """"comic books,"""" you might say """"Phantom, Hopalong Cassidy, Lone Ranger, Our Gang Comics and X-men."""" Any answer that works is okay, and you can give the answers in any order." 11 Oracle, Occidental PetroleumOffice Depot Xerox 20190609 "This puzzle is called """"Lost Arts."""" I'm going to give you clues for two words. The first word has the consecutive letters A-R-T somewhere inside it. Lose the """"art,"""" close up the remaining letters, and you'll get a new word that answers the second clue." 1 Beginning / What bees do starting, sting 20190609 "This puzzle is called """"Lost Arts."""" I'm going to give you clues for two words. The first word has the consecutive letters A-R-T somewhere inside it. Lose the """"art,"""" close up the remaining letters, and you'll get a new word that answers the second clue." 2 Ancient Greek warrior / A bridge Spartan, span 20190609 "This puzzle is called """"Lost Arts."""" I'm going to give you clues for two words. The first word has the consecutive letters A-R-T somewhere inside it. Lose the """"art,"""" close up the remaining letters, and you'll get a new word that answers the second clue." 3 Cocktail often served with an olive / Short short skirt, martini, mini 20190609 "This puzzle is called """"Lost Arts."""" I'm going to give you clues for two words. The first word has the consecutive letters A-R-T somewhere inside it. Lose the """"art,"""" close up the remaining letters, and you'll get a new word that answers the second clue." 4 Person who serves drinks / Drinking spree bartender, bender 20190609 "This puzzle is called """"Lost Arts."""" I'm going to give you clues for two words. The first word has the consecutive letters A-R-T somewhere inside it. Lose the """"art,"""" close up the remaining letters, and you'll get a new word that answers the second clue." 5 "Written grant of rights as for a college / Best Actress Oscar winner for ”Moonstruck""" charter 20190609 "This puzzle is called """"Lost Arts."""" I'm going to give you clues for two words. The first word has the consecutive letters A-R-T somewhere inside it. Lose the """"art,"""" close up the remaining letters, and you'll get a new word that answers the second clue." 6 Zealous supporter of one side in politics / Resident of an Italian city with a leaning tower partisan, Pisan 20190609 "This puzzle is called """"Lost Arts."""" I'm going to give you clues for two words. The first word has the consecutive letters A-R-T somewhere inside it. Lose the """"art,"""" close up the remaining letters, and you'll get a new word that answers the second clue." 7 Exchanging goods without using money / Alaska's ___ Strait bartering, Bering 20190609 "This puzzle is called """"Lost Arts."""" I'm going to give you clues for two words. The first word has the consecutive letters A-R-T somewhere inside it. Lose the """"art,"""" close up the remaining letters, and you'll get a new word that answers the second clue." 8 Ravaged by military conflict (hyphenated) / Threadbare war-torn, worn 20190609 "This puzzle is called """"Lost Arts."""" I'm going to give you clues for two words. The first word has the consecutive letters A-R-T somewhere inside it. Lose the """"art,"""" close up the remaining letters, and you'll get a new word that answers the second clue." 9 Person who steals autos (2 words) / The 'C' of C.E.O. car thief, chief 20190609 "This puzzle is called """"Lost Arts."""" I'm going to give you clues for two words. The first word has the consecutive letters A-R-T somewhere inside it. Lose the """"art,"""" close up the remaining letters, and you'll get a new word that answers the second clue." 10 Oblongs enclosing Egyptian hieroglyphs / Sofas cartouches, couches 20140907 Every answer is a familiar two-word phrase in which the two words start with the same consonant or pair of consonants. Given rhymes for the words, you name the words. 2 snobby course hobby horse 20140907 Every answer is a familiar two-word phrase in which the two words start with the same consonant or pair of consonants. Given rhymes for the words, you name the words. 3 minting stress printing press 20140907 Every answer is a familiar two-word phrase in which the two words start with the same consonant or pair of consonants. Given rhymes for the words, you name the words. 4 scrabble browser rabble rouser 20140907 Every answer is a familiar two-word phrase in which the two words start with the same consonant or pair of consonants. Given rhymes for the words, you name the words. 5 rolling call bowling ball 20140907 Every answer is a familiar two-word phrase in which the two words start with the same consonant or pair of consonants. Given rhymes for the words, you name the words. 6 louder stuff powder puff 20140907 Every answer is a familiar two-word phrase in which the two words start with the same consonant or pair of consonants. Given rhymes for the words, you name the words. 7 tricky house Mickey Mouse 20140907 Every answer is a familiar two-word phrase in which the two words start with the same consonant or pair of consonants. Given rhymes for the words, you name the words. 8 splash mud flash flood 20140907 Every answer is a familiar two-word phrase in which the two words start with the same consonant or pair of consonants. Given rhymes for the words, you name the words. 9 class puzzler gas guzzler 20140907 Every answer is a familiar two-word phrase in which the two words start with the same consonant or pair of consonants. Given rhymes for the words, you name the words. 10 docile mule fossil fuel 20140907 Every answer is a familiar two-word phrase in which the two words start with the same consonant or pair of consonants. Given rhymes for the words, you name the words. 11 painless deal stainless steel 20140907 Every answer is a familiar two-word phrase in which the two words start with the same consonant or pair of consonants. Given rhymes for the words, you name the words. 13 wrench size french fries 20140907 Every answer is a familiar two-word phrase in which the two words start with the same consonant or pair of consonants. Given rhymes for the words, you name the words. 14 diesel bird weasel word 20140907 Every answer is a familiar two-word phrase in which the two words start with the same consonant or pair of consonants. Given rhymes for the words, you name the words. 15 dressing frame guessing game 20130428 "For each given category, name something in the category where the first letter is also the first letter of the category. For example, given """"Military Ranks,"""" you would say """"Major.""""" 1 Weekdays Wednesday 20130428 "For each given category, name something in the category where the first letter is also the first letter of the category. For example, given """"Military Ranks,"""" you would say """"Major.""""" 2 card suits clubs 20130428 "For each given category, name something in the category where the first letter is also the first letter of the category. For example, given """"Military Ranks,"""" you would say """"Major.""""" 3 positions in baseball pitcher 20130428 "For each given category, name something in the category where the first letter is also the first letter of the category. For example, given """"Military Ranks,"""" you would say """"Major.""""" 4 U.S. states Utah 20130428 "For each given category, name something in the category where the first letter is also the first letter of the category. For example, given """"Military Ranks,"""" you would say """"Major.""""" 5 provinces of Canada Prince Edward Island 20130428 "For each given category, name something in the category where the first letter is also the first letter of the category. For example, given """"Military Ranks,"""" you would say """"Major.""""" 6 Clue weapons candlestick 20130428 "For each given category, name something in the category where the first letter is also the first letter of the category. For example, given """"Military Ranks,"""" you would say """"Major.""""" 7 Old Testament books Obadiah 20130428 "For each given category, name something in the category where the first letter is also the first letter of the category. For example, given """"Military Ranks,"""" you would say """"Major.""""" 8 Central American countries Costa Rica 20130428 "For each given category, name something in the category where the first letter is also the first letter of the category. For example, given """"Military Ranks,"""" you would say """"Major.""""" 9 Host cities of the Summer Olympics Helsinki 20130428 "For each given category, name something in the category where the first letter is also the first letter of the category. For example, given """"Military Ranks,"""" you would say """"Major.""""" 10 Albums by the Beatles Abby Road 20130428 "For each given category, name something in the category where the first letter is also the first letter of the category. For example, given """"Military Ranks,"""" you would say """"Major.""""" 11 Reindeer for Santa Claus Rudolph 20130428 "For each given category, name something in the category where the first letter is also the first letter of the category. For example, given """"Military Ranks,"""" you would say """"Major.""""" 12 English queens’ names Elizabeth 20130428 "For each given category, name something in the category where the first letter is also the first letter of the category. For example, given """"Military Ranks,"""" you would say """"Major.""""" 13 Old MacDonald’s farm animal sounds oink oink 20200927 I'm going to read you some sentences. Each sentence conceals the name of a state capital in consecutive letters. You name the capitals. 1 1. You can't rent only one trailer. Trenton (NJ) 20200927 I'm going to read you some sentences. Each sentence conceals the name of a state capital in consecutive letters. You name the capitals. 2 2. How does golf rank for television? Frankfort (KY) 20200927 I'm going to read you some sentences. Each sentence conceals the name of a state capital in consecutive letters. You name the capitals. 3 3. That's not too pleasant a feeling. Santa Fe (NM) 20200927 I'm going to read you some sentences. Each sentence conceals the name of a state capital in consecutive letters. You name the capitals. 4 4. Dad prepared flapjacks once. Jackson (MS) 20200927 I'm going to read you some sentences. Each sentence conceals the name of a state capital in consecutive letters. You name the capitals. 5 5. The numeral eight comes before nine. Raliegh (NC) 20200927 I'm going to read you some sentences. Each sentence conceals the name of a state capital in consecutive letters. You name the capitals. 6 6. We fly to Zurich Monday. Richmond (VA) 20200927 I'm going to read you some sentences. Each sentence conceals the name of a state capital in consecutive letters. You name the capitals. 7 7. Are you and Eric on cordial terms? Concord (NH) 20200927 I'm going to read you some sentences. Each sentence conceals the name of a state capital in consecutive letters. You name the capitals. 8 8. I wanna polish off dinner. Annapolis (MD) 20131110 "Every answer is the name of a state capital, to be identified from its anagram. For example, given """"banally"""" minus the letter L, the answer would be """"Albany.""""" 1 camels -C Salem (Oregon) 20131110 "Every answer is the name of a state capital, to be identified from its anagram. For example, given """"banally"""" minus the letter L, the answer would be """"Albany.""""" 2 so be it -T Boise (Idaho) 20131110 "Every answer is the name of a state capital, to be identified from its anagram. For example, given """"banally"""" minus the letter L, the answer would be """"Albany.""""" 3 vendor -N Dover (Delaware) 20131110 "Every answer is the name of a state capital, to be identified from its anagram. For example, given """"banally"""" minus the letter L, the answer would be """"Albany.""""" 4 tsunami - M Austin (Texas) 20131110 "Every answer is the name of a state capital, to be identified from its anagram. For example, given """"banally"""" minus the letter L, the answer would be """"Albany.""""" 5 palimony -N Olympia (Washington) 20131110 "Every answer is the name of a state capital, to be identified from its anagram. For example, given """"banally"""" minus the letter L, the answer would be """"Albany.""""" 6 snarling -R Lansing (Michigan) 20131110 "Every answer is the name of a state capital, to be identified from its anagram. For example, given """"banally"""" minus the letter L, the answer would be """"Albany.""""" 7 admonish -H Madison (Wisconsin) 20131110 "Every answer is the name of a state capital, to be identified from its anagram. For example, given """"banally"""" minus the letter L, the answer would be """"Albany.""""" 8 nonspatial -T Annapolis (Maryland) 20131110 "Every answer is the name of a state capital, to be identified from its anagram. For example, given """"banally"""" minus the letter L, the answer would be """"Albany.""""" 9 chlorinates -I Charleston (West Virginia) 20151212 Three words will be given, starting with the letters F, B, and I respectively. Find a word that can follow each one to complete a compound word or a familiar two-word phrase. 1 floor, bulletin, ironing board 20151212 Three words will be given, starting with the letters F, B, and I respectively. Find a word that can follow each one to complete a compound word or a familiar two-word phrase. 3 fun, brokerage, ice house 20151212 Three words will be given, starting with the letters F, B, and I respectively. Find a word that can follow each one to complete a compound word or a familiar two-word phrase. 4 family, best, invisible man 20151212 Three words will be given, starting with the letters F, B, and I respectively. Find a word that can follow each one to complete a compound word or a familiar two-word phrase. 5 Florida, backspace, ignition key 20151212 Three words will be given, starting with the letters F, B, and I respectively. Find a word that can follow each one to complete a compound word or a familiar two-word phrase. 6 first, birth, interest rate 20200823 "Today's puzzle is titled """"Low and Inside."""" Every answer is a word or name that has the syllable """"low"""" somewhere inside it (not at the start or the end). The """"low"""" syllable is always accented." 1 Greeting in Hawaii aloha 20200823 "Today's puzzle is titled """"Low and Inside."""" Every answer is a word or name that has the syllable """"low"""" somewhere inside it (not at the start or the end). The """"low"""" syllable is always accented." 2 House speaker Nancy Pelosi 20200823 "Today's puzzle is titled """"Low and Inside."""" Every answer is a word or name that has the syllable """"low"""" somewhere inside it (not at the start or the end). The """"low"""" syllable is always accented." 3 Tennis star Martina Navratilova 20200823 "Today's puzzle is titled """"Low and Inside."""" Every answer is a word or name that has the syllable """"low"""" somewhere inside it (not at the start or the end). The """"low"""" syllable is always accented." 4 Spanish city that hosted the 1992 Olympics Barcelona 20200823 "Today's puzzle is titled """"Low and Inside."""" Every answer is a word or name that has the syllable """"low"""" somewhere inside it (not at the start or the end). The """"low"""" syllable is always accented." 5 Powerful land in ancient Mesopotamia Babylonia 20200823 "Today's puzzle is titled """"Low and Inside."""" Every answer is a word or name that has the syllable """"low"""" somewhere inside it (not at the start or the end). The """"low"""" syllable is always accented." 6 Everyday sandwich meat bologna 20200823 "Today's puzzle is titled """"Low and Inside."""" Every answer is a word or name that has the syllable """"low"""" somewhere inside it (not at the start or the end). The """"low"""" syllable is always accented." 7 Sounding pleasant to the ear, as music melodious 20200823 "Today's puzzle is titled """"Low and Inside."""" Every answer is a word or name that has the syllable """"low"""" somewhere inside it (not at the start or the end). The """"low"""" syllable is always accented." 8 Informal, as speech colloquial 20200823 "Today's puzzle is titled """"Low and Inside."""" Every answer is a word or name that has the syllable """"low"""" somewhere inside it (not at the start or the end). The """"low"""" syllable is always accented." 9 Religious scholar theologian 20200823 "Today's puzzle is titled """"Low and Inside."""" Every answer is a word or name that has the syllable """"low"""" somewhere inside it (not at the start or the end). The """"low"""" syllable is always accented." 10 Kind of tube that a fertilized egg passes through fallopian 20200823 "Today's puzzle is titled """"Low and Inside."""" Every answer is a word or name that has the syllable """"low"""" somewhere inside it (not at the start or the end). The """"low"""" syllable is always accented." 11 Relating to or involved in crime felonious 20200823 "Today's puzzle is titled """"Low and Inside."""" Every answer is a word or name that has the syllable """"low"""" somewhere inside it (not at the start or the end). The """"low"""" syllable is always accented." 12 Running away to get married eloping 20200209 Every answer to this puzzle is the name of an Academy Award-winning Best Actor or Best Actress. Change one letter in each word from a two-word phrase to name the person. 1 Top Ranks Tom Hanks 20200209 Every answer to this puzzle is the name of an Academy Award-winning Best Actor or Best Actress. Change one letter in each word from a two-word phrase to name the person. 2 Jolly Hinter Holly Hunter 20200209 Every answer to this puzzle is the name of an Academy Award-winning Best Actor or Best Actress. Change one letter in each word from a two-word phrase to name the person. 3 Am Pacing Al Pacino 20200209 Every answer to this puzzle is the name of an Academy Award-winning Best Actor or Best Actress. Change one letter in each word from a two-word phrase to name the person. 4 Mean Peon Sean Penn 20200209 Every answer to this puzzle is the name of an Academy Award-winning Best Actor or Best Actress. Change one letter in each word from a two-word phrase to name the person. 5 Wary Copper Gary Cooper 20200209 Every answer to this puzzle is the name of an Academy Award-winning Best Actor or Best Actress. Change one letter in each word from a two-word phrase to name the person. 6 Clank Cable Clark Gable 20200209 Every answer to this puzzle is the name of an Academy Award-winning Best Actor or Best Actress. Change one letter in each word from a two-word phrase to name the person. 7 Salty Yield Sally Field 20200209 Every answer to this puzzle is the name of an Academy Award-winning Best Actor or Best Actress. Change one letter in each word from a two-word phrase to name the person. 8 Kissy Spaces Sissy Spacek 20200209 Every answer to this puzzle is the name of an Academy Award-winning Best Actor or Best Actress. Change one letter in each word from a two-word phrase to name the person. 9 Singer Rowers Ginger Rogers 20200209 Every answer to this puzzle is the name of an Academy Award-winning Best Actor or Best Actress. Change one letter in each word from a two-word phrase to name the person. 10 Sane Woman Jane Wyman 20200209 Every answer to this puzzle is the name of an Academy Award-winning Best Actor or Best Actress. Change one letter in each word from a two-word phrase to name the person. 11 Grape Jelly Grace Kelly 20200209 Every answer to this puzzle is the name of an Academy Award-winning Best Actor or Best Actress. Change one letter in each word from a two-word phrase to name the person. 12 Maroon Brands Marlon Brando 20120311 Every answer consists of two words that are opposites. You are given rhymes for the words, and you give the opposites. 1 coy and curl boy and girl 20120311 Every answer consists of two words that are opposites. You are given rhymes for the words, and you give the opposites. 3 dove and date love and hate 20120311 Every answer consists of two words that are opposites. You are given rhymes for the words, and you give the opposites. 4 pappy and pad happy and sad 20120311 Every answer consists of two words that are opposites. You are given rhymes for the words, and you give the opposites. 5 callow and keep shallow and deep 20120311 Every answer consists of two words that are opposites. You are given rhymes for the words, and you give the opposites. 6 barge and ball large and small 20120311 Every answer consists of two words that are opposites. You are given rhymes for the words, and you give the opposites. 7 smack and smite black and white 20120311 Every answer consists of two words that are opposites. You are given rhymes for the words, and you give the opposites. 8 whiskey and waif risky and safe 20120311 Every answer consists of two words that are opposites. You are given rhymes for the words, and you give the opposites. 9 pong and peak strong and weak 20120311 Every answer consists of two words that are opposites. You are given rhymes for the words, and you give the opposites. 10 gore and geese war and peace 20170108 This puzzle is a welcome for Lulu to the program. We're going to take it from the top. Every answer is a compound word or familiar two-word phrase in which the first part starts TO- and the second part starts P-. 1 Product from Crest or Colgate toothpaste 20170108 This puzzle is a welcome for Lulu to the program. We're going to take it from the top. Every answer is a compound word or familiar two-word phrase in which the first part starts TO- and the second part starts P-. 2 Product from Charmin or Cottonelle toilet paper 20170108 This puzzle is a welcome for Lulu to the program. We're going to take it from the top. Every answer is a compound word or familiar two-word phrase in which the first part starts TO- and the second part starts P-. 3 Place on a highway where you have to stop and pay money toll plaza 20170108 This puzzle is a welcome for Lulu to the program. We're going to take it from the top. Every answer is a compound word or familiar two-word phrase in which the first part starts TO- and the second part starts P-. 4 Native American sculpture in the Northwest in which giant heads are carved into a tree totem pole 20170108 This puzzle is a welcome for Lulu to the program. We're going to take it from the top. Every answer is a compound word or familiar two-word phrase in which the first part starts TO- and the second part starts P-. 5 Rock singer and musician with the Heartbreakers Tom Petty 20170108 This puzzle is a welcome for Lulu to the program. We're going to take it from the top. Every answer is a compound word or familiar two-word phrase in which the first part starts TO- and the second part starts P-. 6 Little dog with a fuzzy coat and a high-pitched bark toy poodle 20170108 This puzzle is a welcome for Lulu to the program. We're going to take it from the top. Every answer is a compound word or familiar two-word phrase in which the first part starts TO- and the second part starts P-. 7 "Fraternity event seen in the film """"Animal House""""" toga party 20170108 This puzzle is a welcome for Lulu to the program. We're going to take it from the top. Every answer is a compound word or familiar two-word phrase in which the first part starts TO- and the second part starts P-. 8 Drunkard tosspot 20170108 This puzzle is a welcome for Lulu to the program. We're going to take it from the top. Every answer is a compound word or familiar two-word phrase in which the first part starts TO- and the second part starts P-. 9 Hybrid car from Japan Toyota Prius 20170108 This puzzle is a welcome for Lulu to the program. We're going to take it from the top. Every answer is a compound word or familiar two-word phrase in which the first part starts TO- and the second part starts P-. 10 Pointing device on a laptop computer used as a substitute for a mouse touchpad 20170108 This puzzle is a welcome for Lulu to the program. We're going to take it from the top. Every answer is a compound word or familiar two-word phrase in which the first part starts TO- and the second part starts P-. 11 Procedure for inserting a titanium bar in part of the mouth; or as Will stated it on air, “a small ornamental stud inserted into part of the mouth.” tunge piercing 20170108 This puzzle is a welcome for Lulu to the program. We're going to take it from the top. Every answer is a compound word or familiar two-word phrase in which the first part starts TO- and the second part starts P-. 12 Something smoked by Sherlock Holmes and Santa Claus tobacco pipe 20170108 This puzzle is a welcome for Lulu to the program. We're going to take it from the top. Every answer is a compound word or familiar two-word phrase in which the first part starts TO- and the second part starts P-. 13 Set of things for sightseers to see at an all-inclusive price tour package 20170108 This puzzle is a welcome for Lulu to the program. We're going to take it from the top. Every answer is a compound word or familiar two-word phrase in which the first part starts TO- and the second part starts P-. 14 Paint that's applied by a pedicurist toenail polish 20121111 Sunday is Veterans Day, so we have a game of categories based on flags. Given some categories, for each one name something in the category beginning with each of the letters F, L, A, G and S. 1 L A 20121111 Sunday is Veterans Day, so we have a game of categories based on flags. Given some categories, for each one name something in the category beginning with each of the letters F, L, A, G and S. 2 G S 20121111 Sunday is Veterans Day, so we have a game of categories based on flags. Given some categories, for each one name something in the category beginning with each of the letters F, L, A, G and S. 3 Things in a fireplace fire, flame, flue 20121111 Sunday is Veterans Day, so we have a game of categories based on flags. Given some categories, for each one name something in the category beginning with each of the letters F, L, A, G and S. 4 liner, log andiron 20121111 Sunday is Veterans Day, so we have a game of categories based on flags. Given some categories, for each one name something in the category beginning with each of the letters F, L, A, G and S. 5 gas, grate spark, spoke, smoke, soot 20121111 Sunday is Veterans Day, so we have a game of categories based on flags. Given some categories, for each one name something in the category beginning with each of the letters F, L, A, G and S. 6 sports football 20121111 Sunday is Veterans Day, so we have a game of categories based on flags. Given some categories, for each one name something in the category beginning with each of the letters F, L, A, G and S. 7 Lacrosse, not line dancing archery, athletics 20121111 Sunday is Veterans Day, so we have a game of categories based on flags. Given some categories, for each one name something in the category beginning with each of the letters F, L, A, G and S. 8 gymnastics, golf soccer, skiing, swimming 20121111 Sunday is Veterans Day, so we have a game of categories based on flags. Given some categories, for each one name something in the category beginning with each of the letters F, L, A, G and S. 9 college majors folklore, French, finance 20121111 Sunday is Veterans Day, so we have a game of categories based on flags. Given some categories, for each one name something in the category beginning with each of the letters F, L, A, G and S. 10 Latin, linguistics art, astronomy 20121111 Sunday is Veterans Day, so we have a game of categories based on flags. Given some categories, for each one name something in the category beginning with each of the letters F, L, A, G and S. 11 German, geology sociology 20121111 Sunday is Veterans Day, so we have a game of categories based on flags. Given some categories, for each one name something in the category beginning with each of the letters F, L, A, G and S. 12 newspaper comics Flinstones, Far Side, Family Circle 20121111 Sunday is Veterans Day, so we have a game of categories based on flags. Given some categories, for each one name something in the category beginning with each of the letters F, L, A, G and S. 13 L’il Abner Apt. 3G, Archie, Ally Oop, Andy Capp 20121111 Sunday is Veterans Day, so we have a game of categories based on flags. Given some categories, for each one name something in the category beginning with each of the letters F, L, A, G and S. 14 Garfield Superman, Spiderman, Shoe, Steve Canyon 20120429 "Every answer today is a familiar three-word phrase in which the second word is """"and"""" and the first word starts with the letter L. You'll be given the last word of the phrase, and you must identify the first word, starting with """"L."""" For example, given """"master,"""" the answer would be """"lord,"""" as in """"lord and master.""""" 1 order law 20120429 "Every answer today is a familiar three-word phrase in which the second word is """"and"""" and the first word starts with the letter L. You'll be given the last word of the phrase, and you must identify the first word, starting with """"L."""" For example, given """"master,"""" the answer would be """"lord,"""" as in """"lord and master.""""" 2 learn live 20120429 "Every answer today is a familiar three-word phrase in which the second word is """"and"""" and the first word starts with the letter L. You'll be given the last word of the phrase, and you must identify the first word, starting with """"L."""" For example, given """"master,"""" the answer would be """"lord,"""" as in """"lord and master.""""" 3 right left 20120429 "Every answer today is a familiar three-word phrase in which the second word is """"and"""" and the first word starts with the letter L. You'll be given the last word of the phrase, and you must identify the first word, starting with """"L."""" For example, given """"master,"""" the answer would be """"lord,"""" as in """"lord and master.""""" 4 found lost 20120429 "Every answer today is a familiar three-word phrase in which the second word is """"and"""" and the first word starts with the letter L. You'll be given the last word of the phrase, and you must identify the first word, starting with """"L."""" For example, given """"master,"""" the answer would be """"lord,"""" as in """"lord and master.""""" 5 clear loud 20120429 "Every answer today is a familiar three-word phrase in which the second word is """"and"""" and the first word starts with the letter L. You'll be given the last word of the phrase, and you must identify the first word, starting with """"L."""" For example, given """"master,"""" the answer would be """"lord,"""" as in """"lord and master.""""" 6 small large 20120429 "Every answer today is a familiar three-word phrase in which the second word is """"and"""" and the first word starts with the letter L. You'll be given the last word of the phrase, and you must identify the first word, starting with """"L."""" For example, given """"master,"""" the answer would be """"lord,"""" as in """"lord and master.""""" 7 Clark Lewis or Lois 20120429 "Every answer today is a familiar three-word phrase in which the second word is """"and"""" and the first word starts with the letter L. You'll be given the last word of the phrase, and you must identify the first word, starting with """"L."""" For example, given """"master,"""" the answer would be """"lord,"""" as in """"lord and master.""""" 8 width length 20120429 "Every answer today is a familiar three-word phrase in which the second word is """"and"""" and the first word starts with the letter L. You'll be given the last word of the phrase, and you must identify the first word, starting with """"L."""" For example, given """"master,"""" the answer would be """"lord,"""" as in """"lord and master.""""" 9 inside low 20120429 "Every answer today is a familiar three-word phrase in which the second word is """"and"""" and the first word starts with the letter L. You'll be given the last word of the phrase, and you must identify the first word, starting with """"L."""" For example, given """"master,"""" the answer would be """"lord,"""" as in """"lord and master.""""" 10 gentlemen ladies 20120429 "Every answer today is a familiar three-word phrase in which the second word is """"and"""" and the first word starts with the letter L. You'll be given the last word of the phrase, and you must identify the first word, starting with """"L."""" For example, given """"master,"""" the answer would be """"lord,"""" as in """"lord and master.""""" 11 Hardy Laurel 20120429 "Every answer today is a familiar three-word phrase in which the second word is """"and"""" and the first word starts with the letter L. You'll be given the last word of the phrase, and you must identify the first word, starting with """"L."""" For example, given """"master,"""" the answer would be """"lord,"""" as in """"lord and master.""""" 12 McCartney Lennon 20120429 "Every answer today is a familiar three-word phrase in which the second word is """"and"""" and the first word starts with the letter L. You'll be given the last word of the phrase, and you must identify the first word, starting with """"L."""" For example, given """"master,"""" the answer would be """"lord,"""" as in """"lord and master.""""" 13 times life 20120429 "Every answer today is a familiar three-word phrase in which the second word is """"and"""" and the first word starts with the letter L. You'll be given the last word of the phrase, and you must identify the first word, starting with """"L."""" For example, given """"master,"""" the answer would be """"lord,"""" as in """"lord and master.""""" 14 key lock 20120429 "Every answer today is a familiar three-word phrase in which the second word is """"and"""" and the first word starts with the letter L. You'll be given the last word of the phrase, and you must identify the first word, starting with """"L."""" For example, given """"master,"""" the answer would be """"lord,"""" as in """"lord and master.""""" 15 kisses love 20120429 "Every answer today is a familiar three-word phrase in which the second word is """"and"""" and the first word starts with the letter L. You'll be given the last word of the phrase, and you must identify the first word, starting with """"L."""" For example, given """"master,"""" the answer would be """"lord,"""" as in """"lord and master.""""" 16 behold lo 20120429 "Every answer today is a familiar three-word phrase in which the second word is """"and"""" and the first word starts with the letter L. You'll be given the last word of the phrase, and you must identify the first word, starting with """"L."""" For example, given """"master,"""" the answer would be """"lord,"""" as in """"lord and master.""""" 17 The Tramp Lady 20120429 "Every answer today is a familiar three-word phrase in which the second word is """"and"""" and the first word starts with the letter L. You'll be given the last word of the phrase, and you must identify the first word, starting with """"L."""" For example, given """"master,"""" the answer would be """"lord,"""" as in """"lord and master.""""" 18 justice for all liberty 20191222 This week's puzzle is a variation on last week's. It's more like a quiz. I'm going to give four words. Three of them have something in common. I'll tell you what that something in common is. You tell me which word is the odd one out. 1 Words that are both flowers and girls' names: Violet, Lily, Iris, Cowslip cowslip 20191222 This week's puzzle is a variation on last week's. It's more like a quiz. I'm going to give four words. Three of them have something in common. I'll tell you what that something in common is. You tell me which word is the odd one out. 2 Words that start the names of state capitals: Big, Rich, Mad, Tall big 20191222 This week's puzzle is a variation on last week's. It's more like a quiz. I'm going to give four words. Three of them have something in common. I'll tell you what that something in common is. You tell me which word is the odd one out. 3 Adjectives that are the titles of well-known movies: Frozen, Notorious, Sweet, Unforgiven sweet 20191222 This week's puzzle is a variation on last week's. It's more like a quiz. I'm going to give four words. Three of them have something in common. I'll tell you what that something in common is. You tell me which word is the odd one out. 4 "Words in the first verse of """"The Star-Spangled Banner"""": Bright, Light, Night, Right" right 20191222 This week's puzzle is a variation on last week's. It's more like a quiz. I'm going to give four words. Three of them have something in common. I'll tell you what that something in common is. You tell me which word is the odd one out. 5 English words that have completely different meanings in French: Main, Pain, Rue, Wet wet 20191222 This week's puzzle is a variation on last week's. It's more like a quiz. I'm going to give four words. Three of them have something in common. I'll tell you what that something in common is. You tell me which word is the odd one out. 6 Three most commonly misspelled words in internet newsgroups (according to a survey by Cornell Kimball): Embarrassment, Millennium, Minuscule, Separate seperate 20190825 For each 5-letter word, add two letters at the end to complete a common, uncapitalized 7-letter word that is unrelated in meaning to the first one. 1 EXTRA extract 20190825 For each 5-letter word, add two letters at the end to complete a common, uncapitalized 7-letter word that is unrelated in meaning to the first one. 2 SURGE surgery or surgeon 20190825 For each 5-letter word, add two letters at the end to complete a common, uncapitalized 7-letter word that is unrelated in meaning to the first one. 3 MARSH marshal 20190825 For each 5-letter word, add two letters at the end to complete a common, uncapitalized 7-letter word that is unrelated in meaning to the first one. 4 PLACE placebo 20190825 For each 5-letter word, add two letters at the end to complete a common, uncapitalized 7-letter word that is unrelated in meaning to the first one. 5 SHELL shellac 20190825 For each 5-letter word, add two letters at the end to complete a common, uncapitalized 7-letter word that is unrelated in meaning to the first one. 6 BLANK blanket 20190825 For each 5-letter word, add two letters at the end to complete a common, uncapitalized 7-letter word that is unrelated in meaning to the first one. 7 INFER interno 20190825 For each 5-letter word, add two letters at the end to complete a common, uncapitalized 7-letter word that is unrelated in meaning to the first one. 8 INTER interim 20190825 For each 5-letter word, add two letters at the end to complete a common, uncapitalized 7-letter word that is unrelated in meaning to the first one. 9 COMMA command 20190825 For each 5-letter word, add two letters at the end to complete a common, uncapitalized 7-letter word that is unrelated in meaning to the first one. 10 COLON colonel 20190825 For each 5-letter word, add two letters at the end to complete a common, uncapitalized 7-letter word that is unrelated in meaning to the first one. 11 WHIST whistle 20190825 For each 5-letter word, add two letters at the end to complete a common, uncapitalized 7-letter word that is unrelated in meaning to the first one. 12 FINES finesse 20190825 For each 5-letter word, add two letters at the end to complete a common, uncapitalized 7-letter word that is unrelated in meaning to the first one. 13 MAXIM maximum 20190825 For each 5-letter word, add two letters at the end to complete a common, uncapitalized 7-letter word that is unrelated in meaning to the first one. 14 AGAIN against 20190825 For each 5-letter word, add two letters at the end to complete a common, uncapitalized 7-letter word that is unrelated in meaning to the first one. 15 VIOLA violate 20190825 For each 5-letter word, add two letters at the end to complete a common, uncapitalized 7-letter word that is unrelated in meaning to the first one. 16 DISCO discord 20190825 For each 5-letter word, add two letters at the end to complete a common, uncapitalized 7-letter word that is unrelated in meaning to the first one. 17 PLATE plateau 20190825 For each 5-letter word, add two letters at the end to complete a common, uncapitalized 7-letter word that is unrelated in meaning to the first one. 18 BREAD breadth 20150510 "Every answer is the name of a well-known U.S. city. For every word given, ignore the vowels. The word's consonants are the same consonants appearing in the same order as those in the city's name. For example, given the word """"amiable,"""" the answer is """"Mobile"""" (Alabama)." 1 baffle, New York Buffalo 20150510 "Every answer is the name of a well-known U.S. city. For every word given, ignore the vowels. The word's consonants are the same consonants appearing in the same order as those in the city's name. For example, given the word """"amiable,"""" the answer is """"Mobile"""" (Alabama)." 2 climbs, Ohio Columbus 20150510 "Every answer is the name of a well-known U.S. city. For every word given, ignore the vowels. The word's consonants are the same consonants appearing in the same order as those in the city's name. For example, given the word """"amiable,"""" the answer is """"Mobile"""" (Alabama)." 3 atlas, Oklahoma Tulsa 20150510 "Every answer is the name of a well-known U.S. city. For every word given, ignore the vowels. The word's consonants are the same consonants appearing in the same order as those in the city's name. For example, given the word """"amiable,"""" the answer is """"Mobile"""" (Alabama)." 4 stately, Washington Seattle 20150510 "Every answer is the name of a well-known U.S. city. For every word given, ignore the vowels. The word's consonants are the same consonants appearing in the same order as those in the city's name. For example, given the word """"amiable,"""" the answer is """"Mobile"""" (Alabama)." 5 dolls, Texas Dallas 20150510 "Every answer is the name of a well-known U.S. city. For every word given, ignore the vowels. The word's consonants are the same consonants appearing in the same order as those in the city's name. For example, given the word """"amiable,"""" the answer is """"Mobile"""" (Alabama)." 6 guest, Maine Augusta 20150510 "Every answer is the name of a well-known U.S. city. For every word given, ignore the vowels. The word's consonants are the same consonants appearing in the same order as those in the city's name. For example, given the word """"amiable,"""" the answer is """"Mobile"""" (Alabama)." 7 foreseen, California Fresno 20150510 "Every answer is the name of a well-known U.S. city. For every word given, ignore the vowels. The word's consonants are the same consonants appearing in the same order as those in the city's name. For example, given the word """"amiable,"""" the answer is """"Mobile"""" (Alabama)." 8 talent, Georgia Atlanta 20150510 "Every answer is the name of a well-known U.S. city. For every word given, ignore the vowels. The word's consonants are the same consonants appearing in the same order as those in the city's name. For example, given the word """"amiable,"""" the answer is """"Mobile"""" (Alabama)." 9 forage, North Dakota Fargo 20150510 "Every answer is the name of a well-known U.S. city. For every word given, ignore the vowels. The word's consonants are the same consonants appearing in the same order as those in the city's name. For example, given the word """"amiable,"""" the answer is """"Mobile"""" (Alabama)." 10 abstain, Massachusetts Boston 20150510 "Every answer is the name of a well-known U.S. city. For every word given, ignore the vowels. The word's consonants are the same consonants appearing in the same order as those in the city's name. For example, given the word """"amiable,"""" the answer is """"Mobile"""" (Alabama)." 11 styles, Missouri St. Louis 20150510 "Every answer is the name of a well-known U.S. city. For every word given, ignore the vowels. The word's consonants are the same consonants appearing in the same order as those in the city's name. For example, given the word """"amiable,"""" the answer is """"Mobile"""" (Alabama)." 12 au pair, Illinois Peoria 20150510 "Every answer is the name of a well-known U.S. city. For every word given, ignore the vowels. The word's consonants are the same consonants appearing in the same order as those in the city's name. For example, given the word """"amiable,"""" the answer is """"Mobile"""" (Alabama)." 13 no charge, Alaska Anchorage 20140615 Today's puzzle is geographical. Every answer is the name of a river — identify it using its anagram minus a letter. Example: Top minus T = Po (River). 1 ELFIN - F NILE 20140615 Today's puzzle is geographical. Every answer is the name of a river — identify it using its anagram minus a letter. Example: Top minus T = Po (River). 2 COOING - I CONGO 20140615 Today's puzzle is geographical. Every answer is the name of a river — identify it using its anagram minus a letter. Example: Top minus T = Po (River). 3 ENRICH - C RHINE or RHEIN (German spelling) 20140615 Today's puzzle is geographical. Every answer is the name of a river — identify it using its anagram minus a letter. Example: Top minus T = Po (River). 4 BOILER - B LOIRE 20140615 Today's puzzle is geographical. Every answer is the name of a river — identify it using its anagram minus a letter. Example: Top minus T = Po (River). 5 NIECES - C SEINE 20140615 Today's puzzle is geographical. Every answer is the name of a river — identify it using its anagram minus a letter. Example: Top minus T = Po (River). 6 HAMSTER - R THAMES 20140615 Today's puzzle is geographical. Every answer is the name of a river — identify it using its anagram minus a letter. Example: Top minus T = Po (River). 7 SUBDEAN - S DANUBE 20140615 Today's puzzle is geographical. Every answer is the name of a river — identify it using its anagram minus a letter. Example: Top minus T = Po (River). 8 SNAGGED - D GANGES 20140615 Today's puzzle is geographical. Every answer is the name of a river — identify it using its anagram minus a letter. Example: Top minus T = Po (River). 9 CORONOID - D ORINOCO 20140615 Today's puzzle is geographical. Every answer is the name of a river — identify it using its anagram minus a letter. Example: Top minus T = Po (River). 10 CORRIGENDA - C RIO GRANDE 20111030 "You will be given the name of a famous person without the first and last letters of their first and last names. Determine the missing letters to add onto the name. For example, if you are given """"err row,"""" the answer would be """"Jerry Brown.""""" 1 lady nigh Gladys Knight 20111030 "You will be given the name of a famous person without the first and last letters of their first and last names. Determine the missing letters to add onto the name. For example, if you are given """"err row,"""" the answer would be """"Jerry Brown.""""" 2 all truther Sally Struthers 20111030 "You will be given the name of a famous person without the first and last letters of their first and last names. Determine the missing letters to add onto the name. For example, if you are given """"err row,"""" the answer would be """"Jerry Brown.""""" 3 live ton Oliver Stone 20111030 "You will be given the name of a famous person without the first and last letters of their first and last names. Determine the missing letters to add onto the name. For example, if you are given """"err row,"""" the answer would be """"Jerry Brown.""""" 4 arlo rand Marlo Brando 20111030 "You will be given the name of a famous person without the first and last letters of their first and last names. Determine the missing letters to add onto the name. For example, if you are given """"err row,"""" the answer would be """"Jerry Brown.""""" 5 oh dam John Adams 20111030 "You will be given the name of a famous person without the first and last letters of their first and last names. Determine the missing letters to add onto the name. For example, if you are given """"err row,"""" the answer would be """"Jerry Brown.""""" 6 mine (one-named persona) Eminem 20141012 Every answer today is the name of a popular prime-time TV series from this century, on either broadcast or cable. Identify the shows from their anagrams. For example, OBLIGE + V = BIG LOVE. 1 leg + E Glee, on FOX 20141012 Every answer today is the name of a popular prime-time TV series from this century, on either broadcast or cable. Identify the shows from their anagrams. For example, OBLIGE + V = BIG LOVE. 2 shoe + U House, on FOX 20141012 Every answer today is the name of a popular prime-time TV series from this century, on either broadcast or cable. Identify the shows from their anagrams. For example, OBLIGE + V = BIG LOVE. 3 rigs + L Girls, on HBO with Lena Dunham 20141012 Every answer today is the name of a popular prime-time TV series from this century, on either broadcast or cable. Identify the shows from their anagrams. For example, OBLIGE + V = BIG LOVE. 4 curbs + S Scrubs 20141012 Every answer today is the name of a popular prime-time TV series from this century, on either broadcast or cable. Identify the shows from their anagrams. For example, OBLIGE + V = BIG LOVE. 5 treed + X Dexter 20141012 Every answer today is the name of a popular prime-time TV series from this century, on either broadcast or cable. Identify the shows from their anagrams. For example, OBLIGE + V = BIG LOVE. 6 maned + M, on AMC two words from now on Mad Men 20141012 Every answer today is the name of a popular prime-time TV series from this century, on either broadcast or cable. Identify the shows from their anagrams. For example, OBLIGE + V = BIG LOVE. 7 writhe + E, HBO, set in Baltimore The Wire 20141012 Every answer today is the name of a popular prime-time TV series from this century, on either broadcast or cable. Identify the shows from their anagrams. For example, OBLIGE + V = BIG LOVE. 8 cheviot + E (cheviot is a breed of sheep) The Voice 20141012 Every answer today is the name of a popular prime-time TV series from this century, on either broadcast or cable. Identify the shows from their anagrams. For example, OBLIGE + V = BIG LOVE. 9 solaced + C, CBS crime drama Cold Case 20190203 "Every answer is a word or name that has the accented syllable """"no"""" somewhere inside it." 1 Children's character whose nose grows when he lies Pinocchio 20190203 "Every answer is a word or name that has the accented syllable """"no"""" somewhere inside it." 2 Wine-making region of California Sonoma 20190203 "Every answer is a word or name that has the accented syllable """"no"""" somewhere inside it." 3 Trance-like state in which a person may be easily manipulated hypnosis 20190203 "Every answer is a word or name that has the accented syllable """"no"""" somewhere inside it." 4 Site of a 1986 nuclear disaster in Ukraine Chernobyl 20190203 "Every answer is a word or name that has the accented syllable """"no"""" somewhere inside it." 5 Brilliantly exploding star supernova 20190203 "Every answer is a word or name that has the accented syllable """"no"""" somewhere inside it." 6 One of the longest rivers in South America Orinoco 20190203 "Every answer is a word or name that has the accented syllable """"no"""" somewhere inside it." 7 French city that hosted the 1968 Winter Olympics Grenoble 20190203 "Every answer is a word or name that has the accented syllable """"no"""" somewhere inside it." 8 Wisconsin city on Lake Michigan Kenosha 20190203 "Every answer is a word or name that has the accented syllable """"no"""" somewhere inside it." 9 Ladies' man Casanova 20190203 "Every answer is a word or name that has the accented syllable """"no"""" somewhere inside it." 10 Old name for Istanbul Constantinople 20111211 Add a letter to create new words in a series of word teasers. 1 An animal starting with O; add an F to name another animal ox, fox 20111211 Add a letter to create new words in a series of word teasers. 2 A musical instrument starting with L; add an F to name another musical instrument lute, flute 20111211 Add a letter to create new words in a series of word teasers. 3 A part of a hat starting with R; add a B to name another part of the hat rim, brim 20111211 Add a letter to create new words in a series of word teasers. 4 A bird starting with E; add a T to name another bird ern, tern 20111211 Add a letter to create new words in a series of word teasers. 5 part of a bar-b-que starting with P; add an S to name another part of a bar-b-que pit, spit 20111211 Add a letter to create new words in a series of word teasers. 6 A boy’s name starting with A; add an R to make another boy’s name; add a B to that to make a girl’s name Andy, Randy, Brandy 20161113 "Insert the letters A and R into the middle of the first clue to get the answer to the second clue. For example, when given the clues """"small argument"""" and """"a tax on imports,"""" the answer would be """"tiff"""" and """"tariff.""""" 1 a ballad, for example; a Polynesian wrap song, sarong 20161113 "Insert the letters A and R into the middle of the first clue to get the answer to the second clue. For example, when given the clues """"small argument"""" and """"a tax on imports,"""" the answer would be """"tiff"""" and """"tariff.""""" 2 having uneven terrain; 2016 election name hilly, Hillary 20161113 "Insert the letters A and R into the middle of the first clue to get the answer to the second clue. For example, when given the clues """"small argument"""" and """"a tax on imports,"""" the answer would be """"tiff"""" and """"tariff.""""" 3 a prize; toward the back reward, rearward 20161113 "Insert the letters A and R into the middle of the first clue to get the answer to the second clue. For example, when given the clues """"small argument"""" and """"a tax on imports,"""" the answer would be """"tiff"""" and """"tariff.""""" 4 shiny, as a photo; a brief dictionary glossy, glossary 20161113 "Insert the letters A and R into the middle of the first clue to get the answer to the second clue. For example, when given the clues """"small argument"""" and """"a tax on imports,"""" the answer would be """"tiff"""" and """"tariff.""""" 5 sound mental health; hygienic and clean sanity, sanitary 20200510 "In honor of Mother's Day, every answer today is a word that ends in the letters """"MA."""" I'll give you an anagram of the letters that precede MA. You tell me the words." 1 ARK karma 20200510 "In honor of Mother's Day, every answer today is a word that ends in the letters """"MA."""" I'll give you an anagram of the letters that precede MA. You tell me the words." 2 OAR aroma 20200510 "In honor of Mother's Day, every answer today is a word that ends in the letters """"MA."""" I'll give you an anagram of the letters that precede MA. You tell me the words." 3 LAPS plasma 20200510 "In honor of Mother's Day, every answer today is a word that ends in the letters """"MA."""" I'll give you an anagram of the letters that precede MA. You tell me the words." 4 GIST stigma 20200510 "In honor of Mother's Day, every answer today is a word that ends in the letters """"MA."""" I'll give you an anagram of the letters that precede MA. You tell me the words." 5 NICE cinema 20200510 "In honor of Mother's Day, every answer today is a word that ends in the letters """"MA."""" I'll give you an anagram of the letters that precede MA. You tell me the words." 6 HATS asthma 20200510 "In honor of Mother's Day, every answer today is a word that ends in the letters """"MA."""" I'll give you an anagram of the letters that precede MA. You tell me the words." 7 LIMED dilemma 20200510 "In honor of Mother's Day, every answer today is a word that ends in the letters """"MA."""" I'll give you an anagram of the letters that precede MA. You tell me the words." 8 RADIO diorama 20200510 "In honor of Mother's Day, every answer today is a word that ends in the letters """"MA."""" I'll give you an anagram of the letters that precede MA. You tell me the words." 9 CHAIRS charisma 20200510 "In honor of Mother's Day, every answer today is a word that ends in the letters """"MA."""" I'll give you an anagram of the letters that precede MA. You tell me the words." 10 EARLDOM melodrama 20201213 "Every answer today is a familiar phrase in the form """"___ & ___,"""" in which the first two letters of the first word plus the first two letters of the last word themselves spell a word. I'll give you that word as well as a definition of the phrase. You tell me the phrase." 1 BOAR — Native American weapon bow and arrow 20201213 "Every answer today is a familiar phrase in the form """"___ & ___,"""" in which the first two letters of the first word plus the first two letters of the last word themselves spell a word. I'll give you that word as well as a definition of the phrase. You tell me the phrase." 2 CAMO — cunning game that a feline and rodent might play cat and mouse 20201213 "Every answer today is a familiar phrase in the form """"___ & ___,"""" in which the first two letters of the first word plus the first two letters of the last word themselves spell a word. I'll give you that word as well as a definition of the phrase. You tell me the phrase." 3 "EVAN — old-fashioned phrase meaning """"occasionally""""" ever and anon 20201213 "Every answer today is a familiar phrase in the form """"___ & ___,"""" in which the first two letters of the first word plus the first two letters of the last word themselves spell a word. I'll give you that word as well as a definition of the phrase. You tell me the phrase." 4 ABBE — more than could possibly be expected above and beyond 20201213 "Every answer today is a familiar phrase in the form """"___ & ___,"""" in which the first two letters of the first word plus the first two letters of the last word themselves spell a word. I'll give you that word as well as a definition of the phrase. You tell me the phrase." 5 LASE — amphibious land and sea 20201213 "Every answer today is a familiar phrase in the form """"___ & ___,"""" in which the first two letters of the first word plus the first two letters of the last word themselves spell a word. I'll give you that word as well as a definition of the phrase. You tell me the phrase." 6 SHAW — overwhelm through a show of force shock and awe 20201213 "Every answer today is a familiar phrase in the form """"___ & ___,"""" in which the first two letters of the first word plus the first two letters of the last word themselves spell a word. I'll give you that word as well as a definition of the phrase. You tell me the phrase." 7 BACH — heavy restraint to keep a prisoner from escaping ball and chain 20201213 "Every answer today is a familiar phrase in the form """"___ & ___,"""" in which the first two letters of the first word plus the first two letters of the last word themselves spell a word. I'll give you that word as well as a definition of the phrase. You tell me the phrase." 8 HOVE — like the positions of the x and y axes on a graph horizontal and vertical 20110904 In a chosen category, name something else in the same category in which the last two letters of Will's word are the first two letters of the answer. Example: for TUBA, the answer might be BASSOON, because TUBA is a musical instrument ending in BA, and BA is the start of BASSOON. 1 CONNECTICUT Utah 20110904 In a chosen category, name something else in the same category in which the last two letters of Will's word are the first two letters of the answer. Example: for TUBA, the answer might be BASSOON, because TUBA is a musical instrument ending in BA, and BA is the start of BASSOON. 2 OBAMA Madison 20110904 In a chosen category, name something else in the same category in which the last two letters of Will's word are the first two letters of the answer. Example: for TUBA, the answer might be BASSOON, because TUBA is a musical instrument ending in BA, and BA is the start of BASSOON. 3 OMEGA Gamma 20110904 In a chosen category, name something else in the same category in which the last two letters of Will's word are the first two letters of the answer. Example: for TUBA, the answer might be BASSOON, because TUBA is a musical instrument ending in BA, and BA is the start of BASSOON. 4 ATLANTA Tallahassee 20110904 In a chosen category, name something else in the same category in which the last two letters of Will's word are the first two letters of the answer. Example: for TUBA, the answer might be BASSOON, because TUBA is a musical instrument ending in BA, and BA is the start of BASSOON. 5 GENESIS Isaiah 20110904 In a chosen category, name something else in the same category in which the last two letters of Will's word are the first two letters of the answer. Example: for TUBA, the answer might be BASSOON, because TUBA is a musical instrument ending in BA, and BA is the start of BASSOON. 6 GATOR orange 20110904 In a chosen category, name something else in the same category in which the last two letters of Will's word are the first two letters of the answer. Example: for TUBA, the answer might be BASSOON, because TUBA is a musical instrument ending in BA, and BA is the start of BASSOON. 7 OTHELLO Lover’s Labours Lost 20110904 In a chosen category, name something else in the same category in which the last two letters of Will's word are the first two letters of the answer. Example: for TUBA, the answer might be BASSOON, because TUBA is a musical instrument ending in BA, and BA is the start of BASSOON. 8 CROW Owl 20110904 In a chosen category, name something else in the same category in which the last two letters of Will's word are the first two letters of the answer. Example: for TUBA, the answer might be BASSOON, because TUBA is a musical instrument ending in BA, and BA is the start of BASSOON. 9 MATISSE seurat 20110904 In a chosen category, name something else in the same category in which the last two letters of Will's word are the first two letters of the answer. Example: for TUBA, the answer might be BASSOON, because TUBA is a musical instrument ending in BA, and BA is the start of BASSOON. 10 NATIONAL alamo 20110904 In a chosen category, name something else in the same category in which the last two letters of Will's word are the first two letters of the answer. Example: for TUBA, the answer might be BASSOON, because TUBA is a musical instrument ending in BA, and BA is the start of BASSOON. 11 PEACH cherry 20110904 In a chosen category, name something else in the same category in which the last two letters of Will's word are the first two letters of the answer. Example: for TUBA, the answer might be BASSOON, because TUBA is a musical instrument ending in BA, and BA is the start of BASSOON. 12 VOLVO Volkswagen 20210228 "Today's puzzle is called A++. I'm going to give you clues for two things. Say what they are. Then put the letter """"A"""" at the start to make a word." 1 Hydroelectric facility / Insect that scurries adamant 20210228 "Today's puzzle is called A++. I'm going to give you clues for two things. Say what they are. Then put the letter """"A"""" at the start to make a word." 2 Old horse / Male sheep anagram 20210228 "Today's puzzle is called A++. I'm going to give you clues for two things. Say what they are. Then put the letter """"A"""" at the start to make a word." 3 Hot dog holder / Waltz or minuet abundance 20210228 "Today's puzzle is called A++. I'm going to give you clues for two things. Say what they are. Then put the letter """"A"""" at the start to make a word." 4 Where a scientist works / Fall flower alabaster 20210228 "Today's puzzle is called A++. I'm going to give you clues for two things. Say what they are. Then put the letter """"A"""" at the start to make a word." 5 Untruth / Country or land alienation 20210228 "Today's puzzle is called A++. I'm going to give you clues for two things. Say what they are. Then put the letter """"A"""" at the start to make a word." 6 Colorado ski resort / Skill availability 20130331 "You will be given some words. For each one, you provide a four-letter word that can follow the first to complete a familiar two-word phrase. The four letters of the second word can always be found inside the first word. For example, given """"personal,"""" the answer would be """"loan.""""" 1 taste test 20130331 "You will be given some words. For each one, you provide a four-letter word that can follow the first to complete a familiar two-word phrase. The four letters of the second word can always be found inside the first word. For example, given """"personal,"""" the answer would be """"loan.""""" 2 maiden name 20130331 "You will be given some words. For each one, you provide a four-letter word that can follow the first to complete a familiar two-word phrase. The four letters of the second word can always be found inside the first word. For example, given """"personal,"""" the answer would be """"loan.""""" 3 local call 20130331 "You will be given some words. For each one, you provide a four-letter word that can follow the first to complete a familiar two-word phrase. The four letters of the second word can always be found inside the first word. For example, given """"personal,"""" the answer would be """"loan.""""" 4 billiard ball 20130331 "You will be given some words. For each one, you provide a four-letter word that can follow the first to complete a familiar two-word phrase. The four letters of the second word can always be found inside the first word. For example, given """"personal,"""" the answer would be """"loan.""""" 5 hornet’s nest 20130331 "You will be given some words. For each one, you provide a four-letter word that can follow the first to complete a familiar two-word phrase. The four letters of the second word can always be found inside the first word. For example, given """"personal,"""" the answer would be """"loan.""""" 6 telephone pole 20130331 "You will be given some words. For each one, you provide a four-letter word that can follow the first to complete a familiar two-word phrase. The four letters of the second word can always be found inside the first word. For example, given """"personal,"""" the answer would be """"loan.""""" 7 convection oven 20130331 "You will be given some words. For each one, you provide a four-letter word that can follow the first to complete a familiar two-word phrase. The four letters of the second word can always be found inside the first word. For example, given """"personal,"""" the answer would be """"loan.""""" 8 Hippocratic Oath 20130331 "You will be given some words. For each one, you provide a four-letter word that can follow the first to complete a familiar two-word phrase. The four letters of the second word can always be found inside the first word. For example, given """"personal,"""" the answer would be """"loan.""""" 9 friendly fire 20130331 "You will be given some words. For each one, you provide a four-letter word that can follow the first to complete a familiar two-word phrase. The four letters of the second word can always be found inside the first word. For example, given """"personal,"""" the answer would be """"loan.""""" 10 industrial arts 20130331 "You will be given some words. For each one, you provide a four-letter word that can follow the first to complete a familiar two-word phrase. The four letters of the second word can always be found inside the first word. For example, given """"personal,"""" the answer would be """"loan.""""" 11 broken bone 20130331 "You will be given some words. For each one, you provide a four-letter word that can follow the first to complete a familiar two-word phrase. The four letters of the second word can always be found inside the first word. For example, given """"personal,"""" the answer would be """"loan.""""" 12 dormitory room 20120624 "You are given three words starting with the letter """"F."""" The answer is a word that can follow each of those three words to complete a familiar two-word phrase. For example, if given """"flag, father's and field,"""" the answer would be """"day.""""" 1 flower, feather, four-poster (3-letter answers) bed 20120624 "You are given three words starting with the letter """"F."""" The answer is a word that can follow each of those three words to complete a familiar two-word phrase. For example, if given """"flag, father's and field,"""" the answer would be """"day.""""" 2 fishing, fuel, fly rod 20120624 "You are given three words starting with the letter """"F."""" The answer is a word that can follow each of those three words to complete a familiar two-word phrase. For example, if given """"flag, father's and field,"""" the answer would be """"day.""""" 3 foul, felt, filter tip 20120624 "You are given three words starting with the letter """"F."""" The answer is a word that can follow each of those three words to complete a familiar two-word phrase. For example, if given """"flag, father's and field,"""" the answer would be """"day.""""" 4 fig, fir, family (4-letter answers) tree 20120624 "You are given three words starting with the letter """"F."""" The answer is a word that can follow each of those three words to complete a familiar two-word phrase. For example, if given """"flag, father's and field,"""" the answer would be """"day.""""" 5 fault, firing, finish line 20120624 "You are given three words starting with the letter """"F."""" The answer is a word that can follow each of those three words to complete a familiar two-word phrase. For example, if given """"flag, father's and field,"""" the answer would be """"day.""""" 6 flight, floor, five-year plan 20120624 "You are given three words starting with the letter """"F."""" The answer is a word that can follow each of those three words to complete a familiar two-word phrase. For example, if given """"flag, father's and field,"""" the answer would be """"day.""""" 7 farmer’s free, flea (6-letter answers) market 20120624 "You are given three words starting with the letter """"F."""" The answer is a word that can follow each of those three words to complete a familiar two-word phrase. For example, if given """"flag, father's and field,"""" the answer would be """"day.""""" 8 filling, fire, flagship (7-letter answers) station 20150208 For each familiar two-word phrase, use the first three letters of the first word and the first three letters of the second word to start two other words that have opposite meanings of each other. Example: Health food = HEAD, FOOT 1 talent show tall, short 20150208 For each familiar two-word phrase, use the first three letters of the first word and the first three letters of the second word to start two other words that have opposite meanings of each other. Example: Health food = HEAD, FOOT 2 small stuff smart, stupid 20150208 For each familiar two-word phrase, use the first three letters of the first word and the first three letters of the second word to start two other words that have opposite meanings of each other. Example: Health food = HEAD, FOOT 3 financial statement finish, start 20150208 For each familiar two-word phrase, use the first three letters of the first word and the first three letters of the second word to start two other words that have opposite meanings of each other. Example: Health food = HEAD, FOOT 4 plain words play, work 20150208 For each familiar two-word phrase, use the first three letters of the first word and the first three letters of the second word to start two other words that have opposite meanings of each other. Example: Health food = HEAD, FOOT 5 strategic weapon strong, weak 20150208 For each familiar two-word phrase, use the first three letters of the first word and the first three letters of the second word to start two other words that have opposite meanings of each other. Example: Health food = HEAD, FOOT 6 chemical exposure cheap, expensive 20150208 For each familiar two-word phrase, use the first three letters of the first word and the first three letters of the second word to start two other words that have opposite meanings of each other. Example: Health food = HEAD, FOOT 7 tennis tournament tender, tough 20150208 For each familiar two-word phrase, use the first three letters of the first word and the first three letters of the second word to start two other words that have opposite meanings of each other. Example: Health food = HEAD, FOOT 8 Louis Quinze loud, quiet 20150208 For each familiar two-word phrase, use the first three letters of the first word and the first three letters of the second word to start two other words that have opposite meanings of each other. Example: Health food = HEAD, FOOT 9 impovised explosive import, export 20200712 I'm going to give you some words that contain the consecutive letters I-T. For each word, change the I-T to two new letters to make a new word. Example: TITULAR --> TUBULAR1. IGNITE2. COMMIT3. CITRUS4. EXCITE5. CITRIC6. INVITE7. FINITE8. MARITAL9. EXPLOIT10. SMITTEN11. BACKBITE 1 IGNITE ignore 20200712 I'm going to give you some words that contain the consecutive letters I-T. For each word, change the I-T to two new letters to make a new word. Example: TITULAR --> TUBULAR1. IGNITE2. COMMIT3. CITRUS4. EXCITE5. CITRIC6. INVITE7. FINITE8. MARITAL9. EXPLOIT10. SMITTEN11. BACKBITE 2 COMMIT common 20200712 I'm going to give you some words that contain the consecutive letters I-T. For each word, change the I-T to two new letters to make a new word. Example: TITULAR --> TUBULAR1. IGNITE2. COMMIT3. CITRUS4. EXCITE5. CITRIC6. INVITE7. FINITE8. MARITAL9. EXPLOIT10. SMITTEN11. BACKBITE 3 CITRUS chorus 20200712 I'm going to give you some words that contain the consecutive letters I-T. For each word, change the I-T to two new letters to make a new word. Example: TITULAR --> TUBULAR1. IGNITE2. COMMIT3. CITRUS4. EXCITE5. CITRIC6. INVITE7. FINITE8. MARITAL9. EXPLOIT10. SMITTEN11. BACKBITE 4 EXCITE excuse 20200712 I'm going to give you some words that contain the consecutive letters I-T. For each word, change the I-T to two new letters to make a new word. Example: TITULAR --> TUBULAR1. IGNITE2. COMMIT3. CITRUS4. EXCITE5. CITRIC6. INVITE7. FINITE8. MARITAL9. EXPLOIT10. SMITTEN11. BACKBITE 5 CITRIC cleric 20200712 I'm going to give you some words that contain the consecutive letters I-T. For each word, change the I-T to two new letters to make a new word. Example: TITULAR --> TUBULAR1. IGNITE2. COMMIT3. CITRUS4. EXCITE5. CITRIC6. INVITE7. FINITE8. MARITAL9. EXPLOIT10. SMITTEN11. BACKBITE 6 INVITE invoke 20200712 I'm going to give you some words that contain the consecutive letters I-T. For each word, change the I-T to two new letters to make a new word. Example: TITULAR --> TUBULAR1. IGNITE2. COMMIT3. CITRUS4. EXCITE5. CITRIC6. INVITE7. FINITE8. MARITAL9. EXPLOIT10. SMITTEN11. BACKBITE 7 FINITE finale 20200712 I'm going to give you some words that contain the consecutive letters I-T. For each word, change the I-T to two new letters to make a new word. Example: TITULAR --> TUBULAR1. IGNITE2. COMMIT3. CITRUS4. EXCITE5. CITRIC6. INVITE7. FINITE8. MARITAL9. EXPLOIT10. SMITTEN11. BACKBITE 8 MARITAL marshal, martial 20200712 I'm going to give you some words that contain the consecutive letters I-T. For each word, change the I-T to two new letters to make a new word. Example: TITULAR --> TUBULAR1. IGNITE2. COMMIT3. CITRUS4. EXCITE5. CITRIC6. INVITE7. FINITE8. MARITAL9. EXPLOIT10. SMITTEN11. BACKBITE 9 EXPLOIT explore, explode 20200712 I'm going to give you some words that contain the consecutive letters I-T. For each word, change the I-T to two new letters to make a new word. Example: TITULAR --> TUBULAR1. IGNITE2. COMMIT3. CITRUS4. EXCITE5. CITRIC6. INVITE7. FINITE8. MARITAL9. EXPLOIT10. SMITTEN11. BACKBITE 10 SMITTEN smarten 20200712 I'm going to give you some words that contain the consecutive letters I-T. For each word, change the I-T to two new letters to make a new word. Example: TITULAR --> TUBULAR1. IGNITE2. COMMIT3. CITRUS4. EXCITE5. CITRIC6. INVITE7. FINITE8. MARITAL9. EXPLOIT10. SMITTEN11. BACKBITE 11 BACKBITE backbone 20141123 NOTE: Due to the Thanksgiving holiday, the deadline for this week's puzzle will be on Wednesday at 3 p.m. Eastern. 1 parrot, horn carrot, corn (veggies) 20141123 NOTE: Due to the Thanksgiving holiday, the deadline for this week's puzzle will be on Wednesday at 3 p.m. Eastern. 2 Leon, fickle neon, nickel (elements) 20141123 NOTE: Due to the Thanksgiving holiday, the deadline for this week's puzzle will be on Wednesday at 3 p.m. Eastern. 3 mover, carding Hoover, Harding (presidents) 20141123 NOTE: Due to the Thanksgiving holiday, the deadline for this week's puzzle will be on Wednesday at 3 p.m. Eastern. 4 canyon, teach banyan, beech (trees) 20141123 NOTE: Due to the Thanksgiving holiday, the deadline for this week's puzzle will be on Wednesday at 3 p.m. Eastern. 5 tango, felon mango, melon (fruits) 20141123 NOTE: Due to the Thanksgiving holiday, the deadline for this week's puzzle will be on Wednesday at 3 p.m. Eastern. 6 linger, put finger, foot (body parts) 20141123 NOTE: Due to the Thanksgiving holiday, the deadline for this week's puzzle will be on Wednesday at 3 p.m. Eastern. 7 Fonda, Monday Honda, Hyundai (car makers) 20201011 Every answer today is a word or phrase in which the only consonants are P and R — repeated as often as necessary. The other letters are all vowels. 1 What beats rock but loses to scissors in a children's game paper 20201011 Every answer today is a word or phrase in which the only consonants are P and R — repeated as often as necessary. The other letters are all vowels. 2 Musician with a flute piper 20201011 Every answer today is a word or phrase in which the only consonants are P and R — repeated as often as necessary. The other letters are all vowels. 3 Partner of salt pepper 20201011 Every answer today is a word or phrase in which the only consonants are P and R — repeated as often as necessary. The other letters are all vowels. 4 Fix as an appliance repair 20201011 Every answer today is a word or phrase in which the only consonants are P and R — repeated as often as necessary. The other letters are all vowels. 5 Brand of bottled water Perrier 20201011 Every answer today is a word or phrase in which the only consonants are P and R — repeated as often as necessary. The other letters are all vowels. 6 Open grassland as in Kansas and Nebraska prairie 20201011 Every answer today is a word or phrase in which the only consonants are P and R — repeated as often as necessary. The other letters are all vowels. 7 Like reasoning by theoretical deduction (2 words) a priori 20201011 Every answer today is a word or phrase in which the only consonants are P and R — repeated as often as necessary. The other letters are all vowels. 8 Come into view again reappear 20110710 "Every answer ends with the letter F, and you are given an anagram of the letters before the F. For example, given the word """"flub,"""" the answer would be """"bluff.""""" 1 ward dwarf 20110710 "Every answer ends with the letter F, and you are given an anagram of the letters before the F. For example, given the word """"flub,"""" the answer would be """"bluff.""""" 2 cars scarf 20110710 "Every answer ends with the letter F, and you are given an anagram of the letters before the F. For example, given the word """"flub,"""" the answer would be """"bluff.""""" 3 fast staff 20110710 "Every answer ends with the letter F, and you are given an anagram of the letters before the F. For example, given the word """"flub,"""" the answer would be """"bluff.""""" 4 Ashe sheaf 20110710 "Every answer ends with the letter F, and you are given an anagram of the letters before the F. For example, given the word """"flub,"""" the answer would be """"bluff.""""" 5 fins sniff 20110710 "Every answer ends with the letter F, and you are given an anagram of the letters before the F. For example, given the word """"flub,"""" the answer would be """"bluff.""""" 6 aced decaf 20110710 "Every answer ends with the letter F, and you are given an anagram of the letters before the F. For example, given the word """"flub,"""" the answer would be """"bluff.""""" 7 frug gruff 20110710 "Every answer ends with the letter F, and you are given an anagram of the letters before the F. For example, given the word """"flub,"""" the answer would be """"bluff.""""" 8 tiles itself 20110710 "Every answer ends with the letter F, and you are given an anagram of the letters before the F. For example, given the word """"flub,"""" the answer would be """"bluff.""""" 9 fisher sheriff 20110710 "Every answer ends with the letter F, and you are given an anagram of the letters before the F. For example, given the word """"flub,"""" the answer would be """"bluff.""""" 10 fat slob blastoff 20110710 "Every answer ends with the letter F, and you are given an anagram of the letters before the F. For example, given the word """"flub,"""" the answer would be """"bluff.""""" 11 bare toes (2 word answer) roast beef 20110213 "Every answer is a word or phrase in which the first syllable is E-U. For example, give the clue """"Holy Communion,"""" the answer would be """"Eucharist.""""" 1 river through Iraq Euphrates 20110213 "Every answer is a word or phrase in which the first syllable is E-U. For example, give the clue """"Holy Communion,"""" the answer would be """"Eucharist.""""" 2 writer Welty Eudora 20110213 "Every answer is a word or phrase in which the first syllable is E-U. For example, give the clue """"Holy Communion,"""" the answer would be """"Eucharist.""""" 3 game with 32 cards euchre 20110213 "Every answer is a word or phrase in which the first syllable is E-U. For example, give the clue """"Holy Communion,"""" the answer would be """"Eucharist.""""" 4 Oregon city that is also a boy’s name Eugene 20110213 "Every answer is a word or phrase in which the first syllable is E-U. For example, give the clue """"Holy Communion,"""" the answer would be """"Eucharist.""""" 5 Kennedy who married Sargent Shriver Eunice 20110213 "Every answer is a word or phrase in which the first syllable is E-U. For example, give the clue """"Holy Communion,"""" the answer would be """"Eucharist.""""" 6 tree whose leaves are eaten by koalas eucalyptus 20110213 "Every answer is a word or phrase in which the first syllable is E-U. For example, give the clue """"Holy Communion,"""" the answer would be """"Eucharist.""""" 7 so-called Father of Geometry; also a city in Ohio Euclid 20110213 "Every answer is a word or phrase in which the first syllable is E-U. For example, give the clue """"Holy Communion,"""" the answer would be """"Eucharist.""""" 8 speech for a funeral eulogy 20110213 "Every answer is a word or phrase in which the first syllable is E-U. For example, give the clue """"Holy Communion,"""" the answer would be """"Eucharist.""""" 9 man in charge of a harem eunich; not Brett Farve 20110213 "Every answer is a word or phrase in which the first syllable is E-U. For example, give the clue """"Holy Communion,"""" the answer would be """"Eucharist.""""" 10 extreme happiness euphoria 20110213 "Every answer is a word or phrase in which the first syllable is E-U. For example, give the clue """"Holy Communion,"""" the answer would be """"Eucharist.""""" 11 mercy killing euthanasia 20110213 "Every answer is a word or phrase in which the first syllable is E-U. For example, give the clue """"Holy Communion,"""" the answer would be """"Eucharist.""""" 12 pleasing to the ear euphonic or euphonious 20110213 "Every answer is a word or phrase in which the first syllable is E-U. For example, give the clue """"Holy Communion,"""" the answer would be """"Eucharist.""""" 13 number 1 or number 2, for a bathroom activity euphemism 20110213 "Every answer is a word or phrase in which the first syllable is E-U. For example, give the clue """"Holy Communion,"""" the answer would be """"Eucharist.""""" 14 the muse of music Euterpe 20110213 "Every answer is a word or phrase in which the first syllable is E-U. For example, give the clue """"Holy Communion,"""" the answer would be """"Eucharist.""""" 15 the furies in Greek myth Eumenidies 20110213 "Every answer is a word or phrase in which the first syllable is E-U. For example, give the clue """"Holy Communion,"""" the answer would be """"Eucharist.""""" 16 a cry after a discovery eureka 20160214 Change one letter of each word and rearrange the result to get a new word that can follow it, to complete a common two-word phrase. 1 RUSH, changing the S to an O HOUR 20160214 Change one letter of each word and rearrange the result to get a new word that can follow it, to complete a common two-word phrase. 3 LONG, changing the L GONE 20160214 Change one letter of each word and rearrange the result to get a new word that can follow it, to complete a common two-word phrase. 4 WORN, changing the R DOWN 20160214 Change one letter of each word and rearrange the result to get a new word that can follow it, to complete a common two-word phrase. 5 MIXED, changing the X MEDIA 20160214 Change one letter of each word and rearrange the result to get a new word that can follow it, to complete a common two-word phrase. 6 EDGAR, changing the R DEGAS 20160214 Change one letter of each word and rearrange the result to get a new word that can follow it, to complete a common two-word phrase. 7 METAL, changing the M to a P PLATE 20160214 Change one letter of each word and rearrange the result to get a new word that can follow it, to complete a common two-word phrase. 8 EQUAL, changing the Q to a V VALUE 20160306 In a series of categories, name something in the category starting with each of the letters W-I-N-D-S. Any answer that works is fine, and you can give the answers in any order. 1 US States, W Wynoming 20160306 In a series of categories, name something in the category starting with each of the letters W-I-N-D-S. Any answer that works is fine, and you can give the answers in any order. 2 US States, I Indiana 20160306 In a series of categories, name something in the category starting with each of the letters W-I-N-D-S. Any answer that works is fine, and you can give the answers in any order. 3 US States, N Nebraska 20160306 In a series of categories, name something in the category starting with each of the letters W-I-N-D-S. Any answer that works is fine, and you can give the answers in any order. 4 US States, D Delaware 20160306 In a series of categories, name something in the category starting with each of the letters W-I-N-D-S. Any answer that works is fine, and you can give the answers in any order. 5 US States, S South Dakota 20160306 In a series of categories, name something in the category starting with each of the letters W-I-N-D-S. Any answer that works is fine, and you can give the answers in any order. 6 Things set on a table at a restaurant, W water 20160306 In a series of categories, name something in the category starting with each of the letters W-I-N-D-S. Any answer that works is fine, and you can give the answers in any order. 7 Things set on a table at a restaurant, I ice, ice bucket 20160306 In a series of categories, name something in the category starting with each of the letters W-I-N-D-S. Any answer that works is fine, and you can give the answers in any order. 8 Things set on a table at a restaurant, N napkin 20160306 In a series of categories, name something in the category starting with each of the letters W-I-N-D-S. Any answer that works is fine, and you can give the answers in any order. 9 Things set on a table at a restaurant, D dish 20160306 In a series of categories, name something in the category starting with each of the letters W-I-N-D-S. Any answer that works is fine, and you can give the answers in any order. 10 Things set on a table at a restaurant, S spoon 20160306 In a series of categories, name something in the category starting with each of the letters W-I-N-D-S. Any answer that works is fine, and you can give the answers in any order. 11 Birds, W wren 20160306 In a series of categories, name something in the category starting with each of the letters W-I-N-D-S. Any answer that works is fine, and you can give the answers in any order. 12 Birds, I ibis 20160306 In a series of categories, name something in the category starting with each of the letters W-I-N-D-S. Any answer that works is fine, and you can give the answers in any order. 13 Birds, N nene goose 20160306 In a series of categories, name something in the category starting with each of the letters W-I-N-D-S. Any answer that works is fine, and you can give the answers in any order. 14 Birds, D duck, dove, dodo 20160306 In a series of categories, name something in the category starting with each of the letters W-I-N-D-S. Any answer that works is fine, and you can give the answers in any order. 15 Birds, S swallow 20160306 In a series of categories, name something in the category starting with each of the letters W-I-N-D-S. Any answer that works is fine, and you can give the answers in any order. 16 One-word movie titles, W Wings 20160306 In a series of categories, name something in the category starting with each of the letters W-I-N-D-S. Any answer that works is fine, and you can give the answers in any order. 17 One-word movie titles, I Ishtar 20160306 In a series of categories, name something in the category starting with each of the letters W-I-N-D-S. Any answer that works is fine, and you can give the answers in any order. 18 One-word movie titles, N Notorious 20160306 In a series of categories, name something in the category starting with each of the letters W-I-N-D-S. Any answer that works is fine, and you can give the answers in any order. 19 One-word movie titles, D Drive 20160306 In a series of categories, name something in the category starting with each of the letters W-I-N-D-S. Any answer that works is fine, and you can give the answers in any order. 20 One-word movie titles, S Spotlight 20160306 In a series of categories, name something in the category starting with each of the letters W-I-N-D-S. Any answer that works is fine, and you can give the answers in any order. 21 Words with a Z, W wizard 20160306 In a series of categories, name something in the category starting with each of the letters W-I-N-D-S. Any answer that works is fine, and you can give the answers in any order. 22 Words with a Z, I influenza 20160306 In a series of categories, name something in the category starting with each of the letters W-I-N-D-S. Any answer that works is fine, and you can give the answers in any order. 23 Words with a Z, N nozzle 20160306 In a series of categories, name something in the category starting with each of the letters W-I-N-D-S. Any answer that works is fine, and you can give the answers in any order. 24 Words with a Z, D doze 20160306 In a series of categories, name something in the category starting with each of the letters W-I-N-D-S. Any answer that works is fine, and you can give the answers in any order. 25 Words with a Z, S size 20190113 Every answer today is the name of a U.S. state capital. Which state capital: 1 Which state capital rhymes with noisy? Boise (Idaho) 20190113 Every answer today is the name of a U.S. state capital. Which state capital: 2 Which state capital contains the letters of COMBS in left-to-right order? Columbus (Ohio) 20190113 Every answer today is the name of a U.S. state capital. Which state capital: 3 Which state capital consists of the chemical symbols of gold and sulfur, followed by the full name of another chemical element? Austin (Texas) 20190113 Every answer today is the name of a U.S. state capital. Which state capital: 4 Which state capital contains a silent O as its third letter, in the southwest? Phoenix (Arizona) 20190113 Every answer today is the name of a U.S. state capital. Which state capital: 5 Which state capital ends in an O sound, but does not contain the letter O? Juneau (Alaska) 20190113 Every answer today is the name of a U.S. state capital. Which state capital: 6 Which state capital consists of a candidate for president in 1988 plus a president in the 1970’s? Hartford (Connecticut) 20160717 Here are two clues for two words. Insert the letters V-E somewhere inside the first word to get the second one. 1 genuine; unmask real, reveal 20160717 Here are two clues for two words. Insert the letters V-E somewhere inside the first word to get the second one. 2 gossip; cause to change course dirt, divert 20160717 Here are two clues for two words. Insert the letters V-E somewhere inside the first word to get the second one. 3 large furry animal; smallish furry animal bear, beaver 20160717 Here are two clues for two words. Insert the letters V-E somewhere inside the first word to get the second one. 4 low in fat; at yeast to lean, leaven 20160717 Here are two clues for two words. Insert the letters V-E somewhere inside the first word to get the second one. 5 transparent; a heavy knife clear, cleaver 20160717 Here are two clues for two words. Insert the letters V-E somewhere inside the first word to get the second one. 6 wrath; one who retaliates anger, avenger 20160717 Here are two clues for two words. Insert the letters V-E somewhere inside the first word to get the second one. 7 a person who is connected by blood; the last book of the New Testament relation, Revelation 20200913 "Every answer to this puzzle is a word or name that has the accented syllable """"lee"""" (in any spelling) somewhere inside it." 1 "City in Ohio sometimes called """"holy""""" Toledo 20200913 "Every answer to this puzzle is a word or name that has the accented syllable """"lee"""" (in any spelling) somewhere inside it." 2 Supreme Court justice Samuel Alito 20200913 "Every answer to this puzzle is a word or name that has the accented syllable """"lee"""" (in any spelling) somewhere inside it." 3 Pledge of ___ Allegience 20200913 "Every answer to this puzzle is a word or name that has the accented syllable """"lee"""" (in any spelling) somewhere inside it." 4 Opposite of an atheist believer 20200913 "Every answer to this puzzle is a word or name that has the accented syllable """"lee"""" (in any spelling) somewhere inside it." 5 Island off the coast of California Catalina 20200913 "Every answer to this puzzle is a word or name that has the accented syllable """"lee"""" (in any spelling) somewhere inside it." 6 Resident of California's largest city Angelino 20200913 "Every answer to this puzzle is a word or name that has the accented syllable """"lee"""" (in any spelling) somewhere inside it." 7 "One-named """"Queen of Tejano Music""""" Selena 20200913 "Every answer to this puzzle is a word or name that has the accented syllable """"lee"""" (in any spelling) somewhere inside it." 8 Italian film director Federico Fellini 20200913 "Every answer to this puzzle is a word or name that has the accented syllable """"lee"""" (in any spelling) somewhere inside it." 9 Leonardo da Vinci painting (two words) Mona Lisa 20150913 Every answer this week is the first and last name of one of the major Republican candidates for president. Identify the candidates from the anagrams given. 1 lunar pad Rand Paul 20150913 Every answer this week is the first and last name of one of the major Republican candidates for president. Identify the candidates from the anagrams given. 2 truced + z Ted Cruz 20150913 Every answer this week is the first and last name of one of the major Republican candidates for president. Identify the candidates from the anagrams given. 3 comb our hair - h Marco Rubio 20150913 Every answer this week is the first and last name of one of the major Republican candidates for president. Identify the candidates from the anagrams given. 4 California + RY Carly Fiorina 20150913 Every answer this week is the first and last name of one of the major Republican candidates for president. Identify the candidates from the anagrams given. 5 corn + beans Ben Carson 20120129 "Today's puzzle is the pits. Every answer is a familiar two-word phrase or name in which the first word starts with """"PI"""" and the second word starts with """"T.""""" 1 an eating surface at a park picnic table 20120129 "Today's puzzle is the pits. Every answer is a familiar two-word phrase or name in which the first word starts with """"PI"""" and the second word starts with """"T.""""" 2 cones grow it pine tree 20120129 "Today's puzzle is the pits. Every answer is a familiar two-word phrase or name in which the first word starts with """"PI"""" and the second word starts with """"T.""""" 3 pepperoni, sausage or mushrooms pizza topping 20120129 "Today's puzzle is the pits. Every answer is a familiar two-word phrase or name in which the first word starts with """"PI"""" and the second word starts with """"T.""""" 4 a classic English poem, of circa 1536, written in the style of Chaucer, with “The” Pilgrim’s Tale 20120129 "Today's puzzle is the pits. Every answer is a familiar two-word phrase or name in which the first word starts with """"PI"""" and the second word starts with """"T.""""" 5 mouth feature of many a punk rocker pierced tongue 20120129 "Today's puzzle is the pits. Every answer is a familiar two-word phrase or name in which the first word starts with """"PI"""" and the second word starts with """"T.""""" 6 former Canadian Prime Minister Pierre Trudeau 20120129 "Today's puzzle is the pits. Every answer is a familiar two-word phrase or name in which the first word starts with """"PI"""" and the second word starts with """"T.""""" 7 chest of stolen gold, silver, precious gems, etc. pirate’s treasure 20111009 Identify errors of fact, logic and grammar in a short essay on Christopher Columbus. 1 Christopher Columbus was born in Genoa in 1451, almost exactly in the middle of the Fourteenth Century. 1451 is in the middle of the Fifthteenth Century. 20111009 Identify errors of fact, logic and grammar in a short essay on Christopher Columbus. 2 His father was a gondolier in Genoa’s famous canals. The famous gondolas and canals are in Venice. 20111009 Identify errors of fact, logic and grammar in a short essay on Christopher Columbus. 3 As a young man, Columbus began seafaring career, sailing throughout the Mediterranean, along the coast of Africa, and as far north as England, and possibly Iceland. No errors according to Will. 20111009 Identify errors of fact, logic and grammar in a short essay on Christopher Columbus. 4 Being ambitious, geography, astronomy and history were subjects of special interest to him. Misplaced subject according to Tony and Will. “Ambitious” does not modify “geography, astronomy and history.” I would call this a misplaced modifier. However, the sentence would be stronger if written in the active voice with Columbus as the subject (“Being ambitious, Columbus took special interest in geography, astronomy and history.”). 20111009 Identify errors of fact, logic and grammar in a short essay on Christopher Columbus. 5 For many years, the Portuguese had been eager to find a sea route to China and the Far East, in order to trade in spices and other riches there. No errors according to Will. 20111009 Identify errors of fact, logic and grammar in a short essay on Christopher Columbus. 6 Portuguese explorers had sailed around Tierra del Fuego at the southern tip of Africa, and then they went east to India and China. Tierra del Fuego is the southern tip of South America. The Cape of Good Hope is the southern tip of Africa. 20111009 Identify errors of fact, logic and grammar in a short essay on Christopher Columbus. 7 Columbus decided he would like to find a shortcut to the east by traveling west. No errors according to Will. 20111009 Identify errors of fact, logic and grammar in a short essay on Christopher Columbus. 8 After several rebuffs, Columbus received financial and other support from Portugal’s King Ferdinand and Queen Isabela. Ferdinand and Isabela were sovereigns of Spain, not Portugal. 20111009 Identify errors of fact, logic and grammar in a short essay on Christopher Columbus. 9 In August, 1492 and a group of fellow explorers departed in three ships, the Niña, the Pinta and the Santa Catalina. It is the Santa Maria, not the Santa Catalina. 20111009 Identify errors of fact, logic and grammar in a short essay on Christopher Columbus. 10 First, they sailed to the Canary Islands which are named after the famously colorful birds there. The Canary Islands are named after the Latin word for dogs, canares. 20111009 Identify errors of fact, logic and grammar in a short essay on Christopher Columbus. 11 Then they used the favorable currents to sail west, citing land in Florida on October 12th. Columbus never saw any part of the mainland United States. The first spotting of land on October 12th was in the Bahamas. 20111009 Identify errors of fact, logic and grammar in a short essay on Christopher Columbus. 12 Columbus thought he’d reached the Far East, or the Indies as the area was then known. No errors according to Will. 20111009 Identify errors of fact, logic and grammar in a short essay on Christopher Columbus. 13 This is why today we call the indigenous peoples of the Americas Indians. No errors according to Will. 20111009 Identify errors of fact, logic and grammar in a short essay on Christopher Columbus. 14 Altogether, Columbus made four voyages to the New World and never knew that he hadn’t reached the Far East. No errors according to Will. 20160131 I'm going to give you some six-letter words. For each one, insert two letters in the exact center to complete a familiar eight-letter word. 1 evince evidence 20160131 I'm going to give you some six-letter words. For each one, insert two letters in the exact center to complete a familiar eight-letter word. 2 callus calculus 20160131 I'm going to give you some six-letter words. For each one, insert two letters in the exact center to complete a familiar eight-letter word. 3 candle canoodle 20160131 I'm going to give you some six-letter words. For each one, insert two letters in the exact center to complete a familiar eight-letter word. 4 Harare hardware 20160131 I'm going to give you some six-letter words. For each one, insert two letters in the exact center to complete a familiar eight-letter word. 5 cellar cellular 20160131 I'm going to give you some six-letter words. For each one, insert two letters in the exact center to complete a familiar eight-letter word. 6 sanity sanctity 20160131 I'm going to give you some six-letter words. For each one, insert two letters in the exact center to complete a familiar eight-letter word. 7 docent document 20160131 I'm going to give you some six-letter words. For each one, insert two letters in the exact center to complete a familiar eight-letter word. 8 banter banister 20160131 I'm going to give you some six-letter words. For each one, insert two letters in the exact center to complete a familiar eight-letter word. 9 metric meteoric 20160131 I'm going to give you some six-letter words. For each one, insert two letters in the exact center to complete a familiar eight-letter word. 10 cookin coonskin 20191124 Every answer is a made-up two-word phrase in which the first word starts with the letter C, and the second word is the same as the first but without the C. 1 1. A Scottish boy wearing clothing clad lad 20191124 Every answer is a made-up two-word phrase in which the first word starts with the letter C, and the second word is the same as the first but without the C. 2 2. Pile that doesn't cost much cheap heap 20191124 Every answer is a made-up two-word phrase in which the first word starts with the letter C, and the second word is the same as the first but without the C. 3 3. Security device for a timepiece clock lock 20191124 Every answer is a made-up two-word phrase in which the first word starts with the letter C, and the second word is the same as the first but without the C. 4 4. Where a group of witches bakes a cake coven oven 20191124 Every answer is a made-up two-word phrase in which the first word starts with the letter C, and the second word is the same as the first but without the C. 5 5. One who assesses the quality of a feature of the lunar surface crater rater 20191124 Every answer is a made-up two-word phrase in which the first word starts with the letter C, and the second word is the same as the first but without the C. 6 6. Black bird that is cowardly craven raven 20191124 Every answer is a made-up two-word phrase in which the first word starts with the letter C, and the second word is the same as the first but without the C. 7 7. Wifey's mate who's a little overweight chubby hubby 20191124 Every answer is a made-up two-word phrase in which the first word starts with the letter C, and the second word is the same as the first but without the C. 8 8. A two-point score in horseshoes that is more obvious cleaner leaner 20191124 Every answer is a made-up two-word phrase in which the first word starts with the letter C, and the second word is the same as the first but without the C. 9 9. One jumping on a helicopter, informally chopper hopper 20191124 Every answer is a made-up two-word phrase in which the first word starts with the letter C, and the second word is the same as the first but without the C. 10 10. One abandoning a butcher's knife cleaver leaver 20191124 Every answer is a made-up two-word phrase in which the first word starts with the letter C, and the second word is the same as the first but without the C. 11 "11. One running his tongue over an answering device on """"Jeopardy!""""" clicker licker 20191124 Every answer is a made-up two-word phrase in which the first word starts with the letter C, and the second word is the same as the first but without the C. 12 12. One who wakes up a reveler carouser arouser 20130113 "Given three three-letter words, give a three-letter word that can follow each to complete a familiar six-letter word. None of the words in a set will be related in meaning. For example, given """"dam,"""" """"man"""" and """"sew,"""" the answer would be """"age,"""" which results in """"damage,"""" """"manage"""" and """"sewage.""""" 1 par, poi, sea son 20130113 "Given three three-letter words, give a three-letter word that can follow each to complete a familiar six-letter word. None of the words in a set will be related in meaning. For example, given """"dam,"""" """"man"""" and """"sew,"""" the answer would be """"age,"""" which results in """"damage,"""" """"manage"""" and """"sewage.""""" 3 arc, fat, was her 20130113 "Given three three-letter words, give a three-letter word that can follow each to complete a familiar six-letter word. None of the words in a set will be related in meaning. For example, given """"dam,"""" """"man"""" and """"sew,"""" the answer would be """"age,"""" which results in """"damage,"""" """"manage"""" and """"sewage.""""" 4 deb, don, rot ate 20130113 "Given three three-letter words, give a three-letter word that can follow each to complete a familiar six-letter word. None of the words in a set will be related in meaning. For example, given """"dam,"""" """"man"""" and """"sew,"""" the answer would be """"age,"""" which results in """"damage,"""" """"manage"""" and """"sewage.""""" 5 pal, pan, win try 20130113 "Given three three-letter words, give a three-letter word that can follow each to complete a familiar six-letter word. None of the words in a set will be related in meaning. For example, given """"dam,"""" """"man"""" and """"sew,"""" the answer would be """"age,"""" which results in """"damage,"""" """"manage"""" and """"sewage.""""" 6 fin, imp, tam ale 20130113 "Given three three-letter words, give a three-letter word that can follow each to complete a familiar six-letter word. None of the words in a set will be related in meaning. For example, given """"dam,"""" """"man"""" and """"sew,"""" the answer would be """"age,"""" which results in """"damage,"""" """"manage"""" and """"sewage.""""" 7 ear, fro, too thy 20151011 I'm going to tell a little story. Call me out on every error of fact, logic or word usage you hear. No specialized knowledge is needed. Ready? Here goes: 1 Sunday, Oct. 10 Today is Sunday, October 11 20151011 I'm going to tell a little story. Call me out on every error of fact, logic or word usage you hear. No specialized knowledge is needed. Ready? Here goes: 2 Greece, Macedonia and Bulgaria, in southwest Europe Greece, Macedonia and Bulgaria, are in southeast Europe 20151011 I'm going to tell a little story. Call me out on every error of fact, logic or word usage you hear. No specialized knowledge is needed. Ready? Here goes: 3 uno momento, as they say in Spain un momento, is the correct Spanish 20151011 I'm going to tell a little story. Call me out on every error of fact, logic or word usage you hear. No specialized knowledge is needed. Ready? Here goes: 4 Thessaloniki, Greece, on the Adriatic Sea Thessaloniki, Greece, is on the Aegean Sea 20151011 I'm going to tell a little story. Call me out on every error of fact, logic or word usage you hear. No specialized knowledge is needed. Ready? Here goes: 5 Thessaloniki, Greece, is famous for its neo-Roman architecture Thessaloniki, Greece, would be famous for its neoclassical Greek architecture 20151011 I'm going to tell a little story. Call me out on every error of fact, logic or word usage you hear. No specialized knowledge is needed. Ready? Here goes: 6 I'm looking forward to abjuring abjuring means avoiding 20151011 I'm going to tell a little story. Call me out on every error of fact, logic or word usage you hear. No specialized knowledge is needed. Ready? Here goes: 7 Skopje, Macedonia, which hosted the recent Winter Olympics the recent Winter Olympics were in Sochi, Russia 20151011 I'm going to tell a little story. Call me out on every error of fact, logic or word usage you hear. No specialized knowledge is needed. Ready? Here goes: 8 Like Russia, Macedonia uses the Latin alphabet Will said that Macedonia uses the Cyrillic alphabet which is also used in Russia 20151011 I'm going to tell a little story. Call me out on every error of fact, logic or word usage you hear. No specialized knowledge is needed. Ready? Here goes: 9 For five days I'll be in Bulgaria for the reason I mentioned — the World Crossword Championship. Will had said that “The main purpose of my trip is to attend the World Puzzle Championship in Bulgaria[.]” Not table tennis as Lourdes answered. 20151011 I'm going to tell a little story. Call me out on every error of fact, logic or word usage you hear. No specialized knowledge is needed. Ready? Here goes: 10 The event will be held in the capital, Bucharest. Sofia is the capital of Bulgaria. Bucharest is the capital of Romania. 20151011 I'm going to tell a little story. Call me out on every error of fact, logic or word usage you hear. No specialized knowledge is needed. Ready? Here goes: 11 Competitors are coming from countries on all seven continents. Antartica has no countries. 20151011 I'm going to tell a little story. Call me out on every error of fact, logic or word usage you hear. No specialized knowledge is needed. Ready? Here goes: 12 All the puzzles in the championship are language- and culture-neutral, like sudoku and crosswords. crosswords are language and culture dependent, not language and culture neutral 20151011 I'm going to tell a little story. Call me out on every error of fact, logic or word usage you hear. No specialized knowledge is needed. Ready? Here goes: 13 After an exhausting 11-day trip Will had said it was a 10-day trip 20151011 I'm going to tell a little story. Call me out on every error of fact, logic or word usage you hear. No specialized knowledge is needed. Ready? Here goes: 14 I will be happy to fly home to New York, setting my watch ahead seven hours on the way. New York is currently seven hours BEHIND Bulgaria 20150712 Today's puzzle involves wordplay on some well-known Canadian place names. Example: 1 Which Canadian province is an anagram for ORATION? Ontario 20150712 Today's puzzle involves wordplay on some well-known Canadian place names. Example: 2 Which Canadian province is an anagram for AVOCATIONS? Nova Scotia 20150712 Today's puzzle involves wordplay on some well-known Canadian place names. Example: 3 Which Canadian province starts and ends with initials of two college degrees? Manitoba 20150712 Today's puzzle involves wordplay on some well-known Canadian place names. Example: 4 What Canadian city consists of two boy’s names, one after the other? Calgary 20150712 Today's puzzle involves wordplay on some well-known Canadian place names. Example: 5 What Canadian city, in ten letters, would name a major landmark in Washington, DC, if you changed and R to a U? Whitehorse 20150712 Today's puzzle involves wordplay on some well-known Canadian place names. Example: 6 What province is hidden in consecutive letters in this question, “Is the boutique becoming popular?” Quebec 20140713 "Today's puzzle is called """"Ch-Ch-Ch-Changes."""" Every answer is a word starting with the letters """"ch,"""" and your clue will be an anagram of the word." 1 CH + cane chance 20140713 "Today's puzzle is called """"Ch-Ch-Ch-Changes."""" Every answer is a word starting with the letters """"ch,"""" and your clue will be an anagram of the word." 2 CH + lies chisel (not chalice) 20140713 "Today's puzzle is called """"Ch-Ch-Ch-Changes."""" Every answer is a word starting with the letters """"ch,"""" and your clue will be an anagram of the word." 3 CH + sour chorus 20140713 "Today's puzzle is called """"Ch-Ch-Ch-Changes."""" Every answer is a word starting with the letters """"ch,"""" and your clue will be an anagram of the word." 4 CH + nose chosen 20140713 "Today's puzzle is called """"Ch-Ch-Ch-Changes."""" Every answer is a word starting with the letters """"ch,"""" and your clue will be an anagram of the word." 5 CH + more chrome 20140713 "Today's puzzle is called """"Ch-Ch-Ch-Changes."""" Every answer is a word starting with the letters """"ch,"""" and your clue will be an anagram of the word." 6 CH + plea chapel 20140713 "Today's puzzle is called """"Ch-Ch-Ch-Changes."""" Every answer is a word starting with the letters """"ch,"""" and your clue will be an anagram of the word." 7 CH + rube cherub 20140713 "Today's puzzle is called """"Ch-Ch-Ch-Changes."""" Every answer is a word starting with the letters """"ch,"""" and your clue will be an anagram of the word." 8 CH + eyes cheesy 20140713 "Today's puzzle is called """"Ch-Ch-Ch-Changes."""" Every answer is a word starting with the letters """"ch,"""" and your clue will be an anagram of the word." 9 CH + grain chagrin 20140713 "Today's puzzle is called """"Ch-Ch-Ch-Changes."""" Every answer is a word starting with the letters """"ch,"""" and your clue will be an anagram of the word." 10 CH + times chemist 20140713 "Today's puzzle is called """"Ch-Ch-Ch-Changes."""" Every answer is a word starting with the letters """"ch,"""" and your clue will be an anagram of the word." 11 CH + dread cheddar 20140713 "Today's puzzle is called """"Ch-Ch-Ch-Changes."""" Every answer is a word starting with the letters """"ch,"""" and your clue will be an anagram of the word." 12 CH + rowed chowder 20140713 "Today's puzzle is called """"Ch-Ch-Ch-Changes."""" Every answer is a word starting with the letters """"ch,"""" and your clue will be an anagram of the word." 13 CH + ratio chariot 20140713 "Today's puzzle is called """"Ch-Ch-Ch-Changes."""" Every answer is a word starting with the letters """"ch,"""" and your clue will be an anagram of the word." 14 CH + prate chapter 20140713 "Today's puzzle is called """"Ch-Ch-Ch-Changes."""" Every answer is a word starting with the letters """"ch,"""" and your clue will be an anagram of the word." 15 CH + marina chairman 20140713 "Today's puzzle is called """"Ch-Ch-Ch-Changes."""" Every answer is a word starting with the letters """"ch,"""" and your clue will be an anagram of the word." 16 CH + refuel cheerful 20110731 "Each answer is a pair of homophones, which are words that sound alike but are spelled differently. You are given two words: One would precede one of the homophones, the other would follow the other, to complete a familiar two-word phrase. For example, given the words """"hay"""" and """"jumper,"""" the answer would be """"bale"""" as in """"hay bale"""" and """"bail"""" as in """"bail jumper.""""" 1 chow ____ and ____ attraction mein, main 20110731 "Each answer is a pair of homophones, which are words that sound alike but are spelled differently. You are given two words: One would precede one of the homophones, the other would follow the other, to complete a familiar two-word phrase. For example, given the words """"hay"""" and """"jumper,"""" the answer would be """"bale"""" as in """"hay bale"""" and """"bail"""" as in """"bail jumper.""""" 2 24 ____ and ____ stick karat, carrot 20110731 "Each answer is a pair of homophones, which are words that sound alike but are spelled differently. You are given two words: One would precede one of the homophones, the other would follow the other, to complete a familiar two-word phrase. For example, given the words """"hay"""" and """"jumper,"""" the answer would be """"bale"""" as in """"hay bale"""" and """"bail"""" as in """"bail jumper.""""" 3 debt ____ and ____ wax ceiling, sealing 20110731 "Each answer is a pair of homophones, which are words that sound alike but are spelled differently. You are given two words: One would precede one of the homophones, the other would follow the other, to complete a familiar two-word phrase. For example, given the words """"hay"""" and """"jumper,"""" the answer would be """"bale"""" as in """"hay bale"""" and """"bail"""" as in """"bail jumper.""""" 4 honeymoon ____ and ____ tooth suite, sweet 20110731 "Each answer is a pair of homophones, which are words that sound alike but are spelled differently. You are given two words: One would precede one of the homophones, the other would follow the other, to complete a familiar two-word phrase. For example, given the words """"hay"""" and """"jumper,"""" the answer would be """"bale"""" as in """"hay bale"""" and """"bail"""" as in """"bail jumper.""""" 5 anchors ____ and ____ game aweigh, away 20110731 "Each answer is a pair of homophones, which are words that sound alike but are spelled differently. You are given two words: One would precede one of the homophones, the other would follow the other, to complete a familiar two-word phrase. For example, given the words """"hay"""" and """"jumper,"""" the answer would be """"bale"""" as in """"hay bale"""" and """"bail"""" as in """"bail jumper.""""" 6 paper ____ and ____ beer route, root 20110731 "Each answer is a pair of homophones, which are words that sound alike but are spelled differently. You are given two words: One would precede one of the homophones, the other would follow the other, to complete a familiar two-word phrase. For example, given the words """"hay"""" and """"jumper,"""" the answer would be """"bale"""" as in """"hay bale"""" and """"bail"""" as in """"bail jumper.""""" 7 white ____ and ____ watchman knight, night 20110731 "Each answer is a pair of homophones, which are words that sound alike but are spelled differently. You are given two words: One would precede one of the homophones, the other would follow the other, to complete a familiar two-word phrase. For example, given the words """"hay"""" and """"jumper,"""" the answer would be """"bale"""" as in """"hay bale"""" and """"bail"""" as in """"bail jumper.""""" 8 track ____ and ____ thermometer meet, meat 20110731 "Each answer is a pair of homophones, which are words that sound alike but are spelled differently. You are given two words: One would precede one of the homophones, the other would follow the other, to complete a familiar two-word phrase. For example, given the words """"hay"""" and """"jumper,"""" the answer would be """"bale"""" as in """"hay bale"""" and """"bail"""" as in """"bail jumper.""""" 9 marine ____ and ____ sample corps, core 20110731 "Each answer is a pair of homophones, which are words that sound alike but are spelled differently. You are given two words: One would precede one of the homophones, the other would follow the other, to complete a familiar two-word phrase. For example, given the words """"hay"""" and """"jumper,"""" the answer would be """"bale"""" as in """"hay bale"""" and """"bail"""" as in """"bail jumper.""""" 10 800 pound ____ and ____ warfare gorilla, guerrilla 20201122 Every answer today is a word or phrase in which the only consonants are F and R — repeated as often as necessary. All the other letters are vowels. 1 Monk FRIAR 20201122 Every answer today is a word or phrase in which the only consonants are F and R — repeated as often as necessary. All the other letters are vowels. 2 Worker who puts shingles on a house ROOFER 20201122 Every answer today is a word or phrase in which the only consonants are F and R — repeated as often as necessary. All the other letters are vowels. 3 More just FAIRER 20201122 Every answer today is a word or phrase in which the only consonants are F and R — repeated as often as necessary. All the other letters are vowels. 4 Less likely to happen IFFIER 20201122 Every answer today is a word or phrase in which the only consonants are F and R — repeated as often as necessary. All the other letters are vowels. 5 Brother, in France FRèRE (FRERE) 20201122 Every answer today is a word or phrase in which the only consonants are F and R — repeated as often as necessary. All the other letters are vowels. 6 Cost of flying on a plane AIRFARE 20201122 Every answer today is a word or phrase in which the only consonants are F and R — repeated as often as necessary. All the other letters are vowels. 7 Basketball official REFEREE 20201122 Every answer today is a word or phrase in which the only consonants are F and R — repeated as often as necessary. All the other letters are vowels. 8 A long way away (two words) FAR OFF 20201122 Every answer today is a word or phrase in which the only consonants are F and R — repeated as often as necessary. All the other letters are vowels. 9 One who sells mink coats FURRIER 20201122 Every answer today is a word or phrase in which the only consonants are F and R — repeated as often as necessary. All the other letters are vowels. 10 One who shoes horses FERRIER 20201122 Every answer today is a word or phrase in which the only consonants are F and R — repeated as often as necessary. All the other letters are vowels. 11 Illicit relationship with a married person AFFAIR 20201122 Every answer today is a word or phrase in which the only consonants are F and R — repeated as often as necessary. All the other letters are vowels. 12 Without cost (two words) FOR FREE 20201122 Every answer today is a word or phrase in which the only consonants are F and R — repeated as often as necessary. All the other letters are vowels. 13 Frills or ornamentation on clothes FROU-FROU 20201122 Every answer today is a word or phrase in which the only consonants are F and R — repeated as often as necessary. All the other letters are vowels. 14 Disreputable or undesirable people, informally RIFFRAFF 20201122 Every answer today is a word or phrase in which the only consonants are F and R — repeated as often as necessary. All the other letters are vowels. 15 Send quickly and angrily, as a letter (two words) FIRE OFF 20201122 Every answer today is a word or phrase in which the only consonants are F and R — repeated as often as necessary. All the other letters are vowels. 16 Repeated sound of a small dog (two words) ROOF ROOF or ARF ARF 20170521 Every answer today is the name of a well-known U.S. city in 7 letters. I'm going to give you a word or phrase that contains those letters in left-to-right order, but not consecutively. You name the city. Ex. INTERVENTION (New Jersey) --> Trenton 1. LANDSCAPING (Michigan)2. METAMORPHOSIS (Tennessee)3. CHEMICAL GROUP (Illinois)4. GROUNDSWELL (New Mexico)5. AMBIVALENCE (Texas)6. LIEUTENANT COLONEL (Nebraska)7. MALDISTRIBUTION (Wisconsin)8. ULTRAVIOLET LIGHT (North Carolina)9. WHITE CHRISTMAS (Kansas)10. RHETORICAL QUESTION (Texas)11. BODYBUILDER (Colorado) 1 landscaping, Michigan Lansing 20170521 Every answer today is the name of a well-known U.S. city in 7 letters. I'm going to give you a word or phrase that contains those letters in left-to-right order, but not consecutively. You name the city. Ex. INTERVENTION (New Jersey) --> Trenton 1. LANDSCAPING (Michigan)2. METAMORPHOSIS (Tennessee)3. CHEMICAL GROUP (Illinois)4. GROUNDSWELL (New Mexico)5. AMBIVALENCE (Texas)6. LIEUTENANT COLONEL (Nebraska)7. MALDISTRIBUTION (Wisconsin)8. ULTRAVIOLET LIGHT (North Carolina)9. WHITE CHRISTMAS (Kansas)10. RHETORICAL QUESTION (Texas)11. BODYBUILDER (Colorado) 2 metamorphosis, Tennessee Memphis 20170521 Every answer today is the name of a well-known U.S. city in 7 letters. I'm going to give you a word or phrase that contains those letters in left-to-right order, but not consecutively. You name the city. Ex. INTERVENTION (New Jersey) --> Trenton 1. LANDSCAPING (Michigan)2. METAMORPHOSIS (Tennessee)3. CHEMICAL GROUP (Illinois)4. GROUNDSWELL (New Mexico)5. AMBIVALENCE (Texas)6. LIEUTENANT COLONEL (Nebraska)7. MALDISTRIBUTION (Wisconsin)8. ULTRAVIOLET LIGHT (North Carolina)9. WHITE CHRISTMAS (Kansas)10. RHETORICAL QUESTION (Texas)11. BODYBUILDER (Colorado) 3 chemical group, Illinois Chicago 20170521 Every answer today is the name of a well-known U.S. city in 7 letters. I'm going to give you a word or phrase that contains those letters in left-to-right order, but not consecutively. You name the city. Ex. INTERVENTION (New Jersey) --> Trenton 1. LANDSCAPING (Michigan)2. METAMORPHOSIS (Tennessee)3. CHEMICAL GROUP (Illinois)4. GROUNDSWELL (New Mexico)5. AMBIVALENCE (Texas)6. LIEUTENANT COLONEL (Nebraska)7. MALDISTRIBUTION (Wisconsin)8. ULTRAVIOLET LIGHT (North Carolina)9. WHITE CHRISTMAS (Kansas)10. RHETORICAL QUESTION (Texas)11. BODYBUILDER (Colorado) 4 ground swell, New Mexico Roswell 20170521 Every answer today is the name of a well-known U.S. city in 7 letters. I'm going to give you a word or phrase that contains those letters in left-to-right order, but not consecutively. You name the city. Ex. INTERVENTION (New Jersey) --> Trenton 1. LANDSCAPING (Michigan)2. METAMORPHOSIS (Tennessee)3. CHEMICAL GROUP (Illinois)4. GROUNDSWELL (New Mexico)5. AMBIVALENCE (Texas)6. LIEUTENANT COLONEL (Nebraska)7. MALDISTRIBUTION (Wisconsin)8. ULTRAVIOLET LIGHT (North Carolina)9. WHITE CHRISTMAS (Kansas)10. RHETORICAL QUESTION (Texas)11. BODYBUILDER (Colorado) 6 lieutenant colonel, Nebraska Lincoln 20170521 Every answer today is the name of a well-known U.S. city in 7 letters. I'm going to give you a word or phrase that contains those letters in left-to-right order, but not consecutively. You name the city. Ex. INTERVENTION (New Jersey) --> Trenton 1. LANDSCAPING (Michigan)2. METAMORPHOSIS (Tennessee)3. CHEMICAL GROUP (Illinois)4. GROUNDSWELL (New Mexico)5. AMBIVALENCE (Texas)6. LIEUTENANT COLONEL (Nebraska)7. MALDISTRIBUTION (Wisconsin)8. ULTRAVIOLET LIGHT (North Carolina)9. WHITE CHRISTMAS (Kansas)10. RHETORICAL QUESTION (Texas)11. BODYBUILDER (Colorado) 7 maldistributed, Wisconsin Madison 20170521 Every answer today is the name of a well-known U.S. city in 7 letters. I'm going to give you a word or phrase that contains those letters in left-to-right order, but not consecutively. You name the city. Ex. INTERVENTION (New Jersey) --> Trenton 1. LANDSCAPING (Michigan)2. METAMORPHOSIS (Tennessee)3. CHEMICAL GROUP (Illinois)4. GROUNDSWELL (New Mexico)5. AMBIVALENCE (Texas)6. LIEUTENANT COLONEL (Nebraska)7. MALDISTRIBUTION (Wisconsin)8. ULTRAVIOLET LIGHT (North Carolina)9. WHITE CHRISTMAS (Kansas)10. RHETORICAL QUESTION (Texas)11. BODYBUILDER (Colorado) 8 ultraviolet light, North Carolina Raleigh 20170521 Every answer today is the name of a well-known U.S. city in 7 letters. I'm going to give you a word or phrase that contains those letters in left-to-right order, but not consecutively. You name the city. Ex. INTERVENTION (New Jersey) --> Trenton 1. LANDSCAPING (Michigan)2. METAMORPHOSIS (Tennessee)3. CHEMICAL GROUP (Illinois)4. GROUNDSWELL (New Mexico)5. AMBIVALENCE (Texas)6. LIEUTENANT COLONEL (Nebraska)7. MALDISTRIBUTION (Wisconsin)8. ULTRAVIOLET LIGHT (North Carolina)9. WHITE CHRISTMAS (Kansas)10. RHETORICAL QUESTION (Texas)11. BODYBUILDER (Colorado) 9 white Christmas, Kansas Wichita 20170521 Every answer today is the name of a well-known U.S. city in 7 letters. I'm going to give you a word or phrase that contains those letters in left-to-right order, but not consecutively. You name the city. Ex. INTERVENTION (New Jersey) --> Trenton 1. LANDSCAPING (Michigan)2. METAMORPHOSIS (Tennessee)3. CHEMICAL GROUP (Illinois)4. GROUNDSWELL (New Mexico)5. AMBIVALENCE (Texas)6. LIEUTENANT COLONEL (Nebraska)7. MALDISTRIBUTION (Wisconsin)8. ULTRAVIOLET LIGHT (North Carolina)9. WHITE CHRISTMAS (Kansas)10. RHETORICAL QUESTION (Texas)11. BODYBUILDER (Colorado) 10 rhetorical question, Texas Houston 20170521 Every answer today is the name of a well-known U.S. city in 7 letters. I'm going to give you a word or phrase that contains those letters in left-to-right order, but not consecutively. You name the city. Ex. INTERVENTION (New Jersey) --> Trenton 1. LANDSCAPING (Michigan)2. METAMORPHOSIS (Tennessee)3. CHEMICAL GROUP (Illinois)4. GROUNDSWELL (New Mexico)5. AMBIVALENCE (Texas)6. LIEUTENANT COLONEL (Nebraska)7. MALDISTRIBUTION (Wisconsin)8. ULTRAVIOLET LIGHT (North Carolina)9. WHITE CHRISTMAS (Kansas)10. RHETORICAL QUESTION (Texas)11. BODYBUILDER (Colorado) 11 body builder, Colorado Boulder 20170219 I'm going to give you a word plus a category. Change one letter in the word to name something in the category. Then change a different letter in the first word to name something else in the category. 1 HOUSE, Animal MOUSE 20170219 I'm going to give you a word plus a category. Change one letter in the word to name something in the category. Then change a different letter in the first word to name something else in the category. 2 BULK, Male animal BUCK, HORSE (not LOUSE?) 20170219 I'm going to give you a word plus a category. Change one letter in the word to name something in the category. Then change a different letter in the first word to name something else in the category. 3 IMAN, Country IRAN, BULL 20170219 I'm going to give you a word plus a category. Change one letter in the word to name something in the category. Then change a different letter in the first word to name something else in the category. 4 CRAM, Seafood CLAM, OMAN 20170219 I'm going to give you a word plus a category. Change one letter in the word to name something in the category. Then change a different letter in the first word to name something else in the category. 5 LURE, Musical instrument LYRE, CRAB 20170219 I'm going to give you a word plus a category. Change one letter in the word to name something in the category. Then change a different letter in the first word to name something else in the category. 6 HEARD, Part of a human body BEARD, HEART 20170219 I'm going to give you a word plus a category. Change one letter in the word to name something in the category. Then change a different letter in the first word to name something else in the category. 7 GRAIN, Another part of the body BRAIN, GROIN 20170219 I'm going to give you a word plus a category. Change one letter in the word to name something in the category. Then change a different letter in the first word to name something else in the category. 8 RAIL, Precipitation HAIL, RAIN 20170219 I'm going to give you a word plus a category. Change one letter in the word to name something in the category. Then change a different letter in the first word to name something else in the category. 9 "GRANT, Word meaning ""big""" GIANT, GRAND 20170219 I'm going to give you a word plus a category. Change one letter in the word to name something in the category. Then change a different letter in the first word to name something else in the category. 10 SPOOL, Something seen in a kitchen SPOON, STOOL 20170219 I'm going to give you a word plus a category. Change one letter in the word to name something in the category. Then change a different letter in the first word to name something else in the category. 11 LACERS, NBA team LAKERS, PACERS 20130811 Every answer is an anagram of a word that has the letters A-B-C in it. 1 ABC + is basic 20130811 Every answer is an anagram of a word that has the letters A-B-C in it. 2 ABC + us scuba 20130811 Every answer is an anagram of a word that has the letters A-B-C in it. 3 ABC + one beacon 20130811 Every answer is an anagram of a word that has the letters A-B-C in it. 4 ABC + nor carbon 20130811 Every answer is an anagram of a word that has the letters A-B-C in it. 5 ABC + fir fabric 20130811 Every answer is an anagram of a word that has the letters A-B-C in it. 6 ABC + jet abject 20130811 Every answer is an anagram of a word that has the letters A-B-C in it. 7 ABC + mere embrace 20130811 Every answer is an anagram of a word that has the letters A-B-C in it. 8 ABC + pole placebo 20130811 Every answer is an anagram of a word that has the letters A-B-C in it. 9 ABC + only balcony 20200322 Every answer is a rhyming two-word phrase, in which each word has three syllables. 1 How-to book that comes out once a year annual manual 20200322 Every answer is a rhyming two-word phrase, in which each word has three syllables. 2 Recollection of a university in Atlanta Emory memory 20200322 Every answer is a rhyming two-word phrase, in which each word has three syllables. 3 Careful examination of a rebellion on a ship mutiny scrutiny 20200322 Every answer is a rhyming two-word phrase, in which each word has three syllables. 4 Personal journal written with hot language fiery diary 20200322 Every answer is a rhyming two-word phrase, in which each word has three syllables. 5 Yorkshire dog that's more jolly merrier terrier 20200322 Every answer is a rhyming two-word phrase, in which each word has three syllables. 6 Sweepstakes involving clay ware pottery lottery 20200322 Every answer is a rhyming two-word phrase, in which each word has three syllables. 7 Devotion to kings and queens royalty loyalty 20200322 Every answer is a rhyming two-word phrase, in which each word has three syllables. 8 Event sent by God involving a ball spherical miracle 20200322 Every answer is a rhyming two-word phrase, in which each word has three syllables. 9 Curtains that have the texture of writing material papery drapery 20200322 Every answer is a rhyming two-word phrase, in which each word has three syllables. 10 Dressing table for a sea cow manatee vanity 20200322 Every answer is a rhyming two-word phrase, in which each word has three syllables. 11 Small group of people who certify documents notary coterie 20200322 Every answer is a rhyming two-word phrase, in which each word has three syllables. 12 Frozen stick in the shape of a three-wheeled children's conveyance tricycle icicle 20170409 Previous audio and Web versions of this segment were incorrect.The singer referred to in the on-air challenge starred in the movie Moonstruck, not Moonshine. 1 Singer who starred in Silkwood, Mask, and Moonshine / Reserve, as a plane or bus, for private use Cher, charter 20170409 Previous audio and Web versions of this segment were incorrect.The singer referred to in the on-air challenge starred in the movie Moonstruck, not Moonshine. 2 Strait between Alaska and Russia / Trading Bering, bartering 20170409 Previous audio and Web versions of this segment were incorrect.The singer referred to in the on-air challenge starred in the movie Moonstruck, not Moonshine. 3 Light brown / Scottish cloth pattern tan, tartan 20170409 Previous audio and Web versions of this segment were incorrect.The singer referred to in the on-air challenge starred in the movie Moonstruck, not Moonshine. 4 Drunken spree / Someone who might help with that drunken spree bender, bartender 20170409 Previous audio and Web versions of this segment were incorrect.The singer referred to in the on-air challenge starred in the movie Moonstruck, not Moonshine. 5 Sofas / Graphic elements in ancient hieroglyphics couches, cartouches 20130217 "The name given to this week's puzzle, """"Dear Mr. President, What's Your Name,"""" as usual appeared only at the NPR site. In honor of Presidents Day, every answer is the last name of a U.S. president. Each clue is a word or phrase that is a president's last name in which two letters have been changed [without rearrangement]. For example, given the clue """"CARpEt,"""" the answer would be """"CARTER.""""" 1 Klingon Clinton 20130217 "The name given to this week's puzzle, """"Dear Mr. President, What's Your Name,"""" as usual appeared only at the NPR site. In honor of Presidents Day, every answer is the last name of a U.S. president. Each clue is a word or phrase that is a president's last name in which two letters have been changed [without rearrangement]. For example, given the clue """"CARpEt,"""" the answer would be """"CARTER.""""" 2 hurling Harding 20130217 "The name given to this week's puzzle, """"Dear Mr. President, What's Your Name,"""" as usual appeared only at the NPR site. In honor of Presidents Day, every answer is the last name of a U.S. president. Each clue is a word or phrase that is a president's last name in which two letters have been changed [without rearrangement]. For example, given the clue """"CARpEt,"""" the answer would be """"CARTER.""""" 3 Joins in Johnson 20130217 "The name given to this week's puzzle, """"Dear Mr. President, What's Your Name,"""" as usual appeared only at the NPR site. In honor of Presidents Day, every answer is the last name of a U.S. president. Each clue is a word or phrase that is a president's last name in which two letters have been changed [without rearrangement]. For example, given the clue """"CARpEt,"""" the answer would be """"CARTER.""""" 4 sad iron Madison 20130217 "The name given to this week's puzzle, """"Dear Mr. President, What's Your Name,"""" as usual appeared only at the NPR site. In honor of Presidents Day, every answer is the last name of a U.S. president. Each clue is a word or phrase that is a president's last name in which two letters have been changed [without rearrangement]. For example, given the clue """"CARpEt,"""" the answer would be """"CARTER.""""" 5 hennery Kennedy 20130217 "The name given to this week's puzzle, """"Dear Mr. President, What's Your Name,"""" as usual appeared only at the NPR site. In honor of Presidents Day, every answer is the last name of a U.S. president. Each clue is a word or phrase that is a president's last name in which two letters have been changed [without rearrangement]. For example, given the clue """"CARpEt,"""" the answer would be """"CARTER.""""" 6 toucan Truman 20130217 "The name given to this week's puzzle, """"Dear Mr. President, What's Your Name,"""" as usual appeared only at the NPR site. In honor of Presidents Day, every answer is the last name of a U.S. president. Each clue is a word or phrase that is a president's last name in which two letters have been changed [without rearrangement]. For example, given the clue """"CARpEt,"""" the answer would be """"CARTER.""""" 7 reason Reagan 20130217 "The name given to this week's puzzle, """"Dear Mr. President, What's Your Name,"""" as usual appeared only at the NPR site. In honor of Presidents Day, every answer is the last name of a U.S. president. Each clue is a word or phrase that is a president's last name in which two letters have been changed [without rearrangement]. For example, given the clue """"CARpEt,"""" the answer would be """"CARTER.""""" 8 parlor Taylor 20130217 "The name given to this week's puzzle, """"Dear Mr. President, What's Your Name,"""" as usual appeared only at the NPR site. In honor of Presidents Day, every answer is the last name of a U.S. president. Each clue is a word or phrase that is a president's last name in which two letters have been changed [without rearrangement]. For example, given the clue """"CARpEt,"""" the answer would be """"CARTER.""""" 9 pile on Wilson 20130217 "The name given to this week's puzzle, """"Dear Mr. President, What's Your Name,"""" as usual appeared only at the NPR site. In honor of Presidents Day, every answer is the last name of a U.S. president. Each clue is a word or phrase that is a president's last name in which two letters have been changed [without rearrangement]. For example, given the clue """"CARpEt,"""" the answer would be """"CARTER.""""" 10 adios Adams 20200524 I'm going to name two things that are in the same category. And that category is itself part of another category, which I'll also name. You tell me the category in the middle. Example: Volkswagen, Chrysler/Rock groups -->(The) Cars1. Robin, Finch/Hitchcock movies2. Seven, 100/Books of the Bible3. Hip hip hooray!, Go team!/Sitcoms4. Merlin, Harry Potter/N.B.A. teams5. Mary, Elizabeth/Boroughs of New York City6. Amazon, Nile/Female comedians7. Roasted, Salted/Comic strips8. Potatoes, Rice/Chain stores9. Proofreader, Auditor/Board games 1 Robin, Finch/Hitchcock movies (The) Birds 20200524 I'm going to name two things that are in the same category. And that category is itself part of another category, which I'll also name. You tell me the category in the middle. Example: Volkswagen, Chrysler/Rock groups -->(The) Cars1. Robin, Finch/Hitchcock movies2. Seven, 100/Books of the Bible3. Hip hip hooray!, Go team!/Sitcoms4. Merlin, Harry Potter/N.B.A. teams5. Mary, Elizabeth/Boroughs of New York City6. Amazon, Nile/Female comedians7. Roasted, Salted/Comic strips8. Potatoes, Rice/Chain stores9. Proofreader, Auditor/Board games 2 Seven, 100/Books of the Bible Numbers 20200524 I'm going to name two things that are in the same category. And that category is itself part of another category, which I'll also name. You tell me the category in the middle. Example: Volkswagen, Chrysler/Rock groups -->(The) Cars1. Robin, Finch/Hitchcock movies2. Seven, 100/Books of the Bible3. Hip hip hooray!, Go team!/Sitcoms4. Merlin, Harry Potter/N.B.A. teams5. Mary, Elizabeth/Boroughs of New York City6. Amazon, Nile/Female comedians7. Roasted, Salted/Comic strips8. Potatoes, Rice/Chain stores9. Proofreader, Auditor/Board games 3 Hip hip hooray!, Go team!/Sitcoms Cheers 20200524 I'm going to name two things that are in the same category. And that category is itself part of another category, which I'll also name. You tell me the category in the middle. Example: Volkswagen, Chrysler/Rock groups -->(The) Cars1. Robin, Finch/Hitchcock movies2. Seven, 100/Books of the Bible3. Hip hip hooray!, Go team!/Sitcoms4. Merlin, Harry Potter/N.B.A. teams5. Mary, Elizabeth/Boroughs of New York City6. Amazon, Nile/Female comedians7. Roasted, Salted/Comic strips8. Potatoes, Rice/Chain stores9. Proofreader, Auditor/Board games 4 Merlin, Harry Potter/N.B.A. teams Wizards 20200524 I'm going to name two things that are in the same category. And that category is itself part of another category, which I'll also name. You tell me the category in the middle. Example: Volkswagen, Chrysler/Rock groups -->(The) Cars1. Robin, Finch/Hitchcock movies2. Seven, 100/Books of the Bible3. Hip hip hooray!, Go team!/Sitcoms4. Merlin, Harry Potter/N.B.A. teams5. Mary, Elizabeth/Boroughs of New York City6. Amazon, Nile/Female comedians7. Roasted, Salted/Comic strips8. Potatoes, Rice/Chain stores9. Proofreader, Auditor/Board games 5 Mary, Elizabeth/Boroughs of New York City Queens 20200524 I'm going to name two things that are in the same category. And that category is itself part of another category, which I'll also name. You tell me the category in the middle. Example: Volkswagen, Chrysler/Rock groups -->(The) Cars1. Robin, Finch/Hitchcock movies2. Seven, 100/Books of the Bible3. Hip hip hooray!, Go team!/Sitcoms4. Merlin, Harry Potter/N.B.A. teams5. Mary, Elizabeth/Boroughs of New York City6. Amazon, Nile/Female comedians7. Roasted, Salted/Comic strips8. Potatoes, Rice/Chain stores9. Proofreader, Auditor/Board games 6 Amazon, Nile/Female comedians (Joan) Rivers 20200524 I'm going to name two things that are in the same category. And that category is itself part of another category, which I'll also name. You tell me the category in the middle. Example: Volkswagen, Chrysler/Rock groups -->(The) Cars1. Robin, Finch/Hitchcock movies2. Seven, 100/Books of the Bible3. Hip hip hooray!, Go team!/Sitcoms4. Merlin, Harry Potter/N.B.A. teams5. Mary, Elizabeth/Boroughs of New York City6. Amazon, Nile/Female comedians7. Roasted, Salted/Comic strips8. Potatoes, Rice/Chain stores9. Proofreader, Auditor/Board games 8 Potatoes, Rice/Chain stores Staples 20200524 I'm going to name two things that are in the same category. And that category is itself part of another category, which I'll also name. You tell me the category in the middle. Example: Volkswagen, Chrysler/Rock groups -->(The) Cars1. Robin, Finch/Hitchcock movies2. Seven, 100/Books of the Bible3. Hip hip hooray!, Go team!/Sitcoms4. Merlin, Harry Potter/N.B.A. teams5. Mary, Elizabeth/Boroughs of New York City6. Amazon, Nile/Female comedians7. Roasted, Salted/Comic strips8. Potatoes, Rice/Chain stores9. Proofreader, Auditor/Board games 9 Proofreader, Auditor/Board games Checkers 20130714 You're given a series of clues, and every answer is the name of a U.S. state capital. 1 What state capital is contained in the word EXHAUSTING? Austin (Texas) 20130714 You're given a series of clues, and every answer is the name of a U.S. state capital. 2 What state capital starts with the name of a month? There are two answers. Juneau (Alaska) and Augusta (Maine) 20130714 You're given a series of clues, and every answer is the name of a U.S. state capital. 3 What state capital has three pairs of double letters? Tallahassee (Florida) 20130714 You're given a series of clues, and every answer is the name of a U.S. state capital. 4 What state capital has a three word name in which all three words have the same number of letters? Its state also has this many letter letters in its name. Salt Lake City, Utah 20130714 You're given a series of clues, and every answer is the name of a U.S. state capital. 5 What state capital is an anagram of MOST ARCANE? Sacramento (California) 20130714 You're given a series of clues, and every answer is the name of a U.S. state capital. 6 What state capital changes from two syllables to one when its first letter is changed from a B to a P? Boise (Idaho) 20130714 You're given a series of clues, and every answer is the name of a U.S. state capital. 7 What state capital is the name of a dance in 10 letters? Charleston (West Virginia) 20201227 Every answer today is an eight-letter word that consists of a three-letter word inserted inside a five-letter word. I'll give you a clue to the three-letter word as well as the five-letter word itself. You tell me the eight-letter word. 1 Tennis court divider + MAGIC magnetic 20201227 Every answer today is an eight-letter word that consists of a three-letter word inserted inside a five-letter word. I'll give you a clue to the three-letter word as well as the five-letter word itself. You tell me the eight-letter word. 2 Oil installation + IRATE irrigate 20201227 Every answer today is an eight-letter word that consists of a three-letter word inserted inside a five-letter word. I'll give you a clue to the three-letter word as well as the five-letter word itself. You tell me the eight-letter word. 3 What a hot dog goes in + TRIAL tribunal 20201227 Every answer today is an eight-letter word that consists of a three-letter word inserted inside a five-letter word. I'll give you a clue to the three-letter word as well as the five-letter word itself. You tell me the eight-letter word. 4 Everything + BROOM ballroom 20201227 Every answer today is an eight-letter word that consists of a three-letter word inserted inside a five-letter word. I'll give you a clue to the three-letter word as well as the five-letter word itself. You tell me the eight-letter word. 5 Have dinner + ENTRY entreaty 20201227 Every answer today is an eight-letter word that consists of a three-letter word inserted inside a five-letter word. I'll give you a clue to the three-letter word as well as the five-letter word itself. You tell me the eight-letter word. 6 Opposite of young + DRUMS doldrums 20201227 Every answer today is an eight-letter word that consists of a three-letter word inserted inside a five-letter word. I'll give you a clue to the three-letter word as well as the five-letter word itself. You tell me the eight-letter word. 7 To employ + CAROL carousel 20201227 Every answer today is an eight-letter word that consists of a three-letter word inserted inside a five-letter word. I'll give you a clue to the three-letter word as well as the five-letter word itself. You tell me the eight-letter word. 8 Health resort + DITCH dispatch 20201227 Every answer today is an eight-letter word that consists of a three-letter word inserted inside a five-letter word. I'll give you a clue to the three-letter word as well as the five-letter word itself. You tell me the eight-letter word. 9 Fruit preserves + BENIN Benjamin 20201227 Every answer today is an eight-letter word that consists of a three-letter word inserted inside a five-letter word. I'll give you a clue to the three-letter word as well as the five-letter word itself. You tell me the eight-letter word. 10 Outdoor children's game + SHORE shortage 20201227 Every answer today is an eight-letter word that consists of a three-letter word inserted inside a five-letter word. I'll give you a clue to the three-letter word as well as the five-letter word itself. You tell me the eight-letter word. 11 Cry of disgust + LATER laughter 20201227 Every answer today is an eight-letter word that consists of a three-letter word inserted inside a five-letter word. I'll give you a clue to the three-letter word as well as the five-letter word itself. You tell me the eight-letter word. 12 Be successful at bat + GRAPE graphite 20201227 Every answer today is an eight-letter word that consists of a three-letter word inserted inside a five-letter word. I'll give you a clue to the three-letter word as well as the five-letter word itself. You tell me the eight-letter word. 13 Affirmative response + EIGHT eyesight 20120108 Each answer is a five-letter word or phrase containing the letters N, O, T, E plus one other letter. Answer the clues to get the words. 1 NOTE + B, a type of steak T-bone 20120108 Each answer is a five-letter word or phrase containing the letters N, O, T, E plus one other letter. Answer the clues to get the words. 2 NOTE + C, French word for story conte 20120108 Each answer is a five-letter word or phrase containing the letters N, O, T, E plus one other letter. Answer the clues to get the words. 3 NOTE + D, in good shape toned 20120108 Each answer is a five-letter word or phrase containing the letters N, O, T, E plus one other letter. Answer the clues to get the words. 4 NOTE + G, mount, two word phrase get on 20120108 Each answer is a five-letter word or phrase containing the letters N, O, T, E plus one other letter. Answer the clues to get the words. 5 NOTE + I, attach, two words tie on 20120108 Each answer is a five-letter word or phrase containing the letters N, O, T, E plus one other letter. Answer the clues to get the words. 6 NOTE + L, at a slow tempo, in music lento 20120108 Each answer is a five-letter word or phrase containing the letters N, O, T, E plus one other letter. Answer the clues to get the words. 7 NOTE + M, a French impressionist Monet 20120108 Each answer is a five-letter word or phrase containing the letters N, O, T, E plus one other letter. Answer the clues to get the words. 8 NOTE + P, bridge, in Italian ponte 20120108 Each answer is a five-letter word or phrase containing the letters N, O, T, E plus one other letter. Answer the clues to get the words. 9 NOTE + R, contents of a certain cartridge toner 20120108 Each answer is a five-letter word or phrase containing the letters N, O, T, E plus one other letter. Answer the clues to get the words. 10 NOTE + S, beginning onset 20120108 Each answer is a five-letter word or phrase containing the letters N, O, T, E plus one other letter. Answer the clues to get the words. 11 NOTE + T, Grand _____ National Park Teton 20120325 Every answer is a word or phrase containing the consecutive letters A-X. You'll be given clues and anagrams to the answers. 1 moi + ax, self-evident truth axiom 20120325 Every answer is a word or phrase containing the consecutive letters A-X. You'll be given clues and anagrams to the answers. 2 roth + ax, part of the body between the neck and the abdomen thorax 20120325 Every answer is a word or phrase containing the consecutive letters A-X. You'll be given clues and anagrams to the answers. 3 mince + ax, premium cable channel Cinemax 20120325 Every answer is a word or phrase containing the consecutive letters A-X. You'll be given clues and anagrams to the answers. 4 roles + ax, lead singer for Guns and Roses (2 word answer) Axl Rose 20120325 Every answer is a word or phrase containing the consecutive letters A-X. You'll be given clues and anagrams to the answers. 5 amber + ax, 1930’s heavyweight boxing champ (2 word name) Max Baer 20120325 Every answer is a word or phrase containing the consecutive letters A-X. You'll be given clues and anagrams to the answers. 6 main chef + ax, office item for sending documents long distance fax machine 20120325 Every answer is a word or phrase containing the consecutive letters A-X. You'll be given clues and anagrams to the answers. 7 silage + ax, the Milky Way and others galaxies 20120325 Every answer is a word or phrase containing the consecutive letters A-X. You'll be given clues and anagrams to the answers. 8 others + ax, creators of big practical jokes or frauds hoaxters 20160612 I've brought a puzzle based on the name Oahu. Oddly, it has three syllables in only four letters. Answer my clues for other words and names that have three syllables in four letters. 1 neighborhood, vicinity area 20160612 I've brought a puzzle based on the name Oahu. Oddly, it has three syllables in only four letters. Answer my clues for other words and names that have three syllables in four letters. 2 Verde opera Aida 20160612 I've brought a puzzle based on the name Oahu. Oddly, it has three syllables in only four letters. Answer my clues for other words and names that have three syllables in four letters. 3 a song in an opera aria 20160612 I've brought a puzzle based on the name Oahu. Oddly, it has three syllables in only four letters. Answer my clues for other words and names that have three syllables in four letters. 4 Capital of Samoa Apia 20160612 I've brought a puzzle based on the name Oahu. Oddly, it has three syllables in only four letters. Answer my clues for other words and names that have three syllables in four letters. 5 early state in presidential campaigns Iowa 20160612 I've brought a puzzle based on the name Oahu. Oddly, it has three syllables in only four letters. Answer my clues for other words and names that have three syllables in four letters. 6 brainstorm idea 20160612 I've brought a puzzle based on the name Oahu. Oddly, it has three syllables in only four letters. Answer my clues for other words and names that have three syllables in four letters. 7 brand billed as milk’s favorite cookie Oreo 20160612 I've brought a puzzle based on the name Oahu. Oddly, it has three syllables in only four letters. Answer my clues for other words and names that have three syllables in four letters. 8 Greek letter after theta iota 20160612 I've brought a puzzle based on the name Oahu. Oddly, it has three syllables in only four letters. Answer my clues for other words and names that have three syllables in four letters. 9 villain in Othello Iago 20170813 "Every answer today is a familiar three-word phrase in the form """"___ of ___."""" I'll give you anagrams of the words in the blanks You identify the phrases." 1 cat of dog act of God 20170813 "Every answer today is a familiar three-word phrase in the form """"___ of ___."""" I'll give you anagrams of the words in the blanks You identify the phrases." 2 taco of Mars coat of arms 20170813 "Every answer today is a familiar three-word phrase in the form """"___ of ___."""" I'll give you anagrams of the words in the blanks You identify the phrases." 3 stream of rats master of arts 20170813 "Every answer today is a familiar three-word phrase in the form """"___ of ___."""" I'll give you anagrams of the words in the blanks You identify the phrases." 4 stapler of pairs plaster of Paris 20170813 "Every answer today is a familiar three-word phrase in the form """"___ of ___."""" I'll give you anagrams of the words in the blanks You identify the phrases." 5 side of charm ides of March 20170813 "Every answer today is a familiar three-word phrase in the form """"___ of ___."""" I'll give you anagrams of the words in the blanks You identify the phrases." 6 mage of hornets Game of Thrones 20160110 Every answer this week is the name of an article of apparel — something to wear. Name the items from the anagram given. 1 pace cape 20160110 Every answer this week is the name of an article of apparel — something to wear. Name the items from the anagram given. 2 wong gown 20160110 Every answer this week is the name of an article of apparel — something to wear. Name the items from the anagram given. 3 goat toga 20160110 Every answer this week is the name of an article of apparel — something to wear. Name the items from the anagram given. 4 hose shoe 20160110 Every answer this week is the name of an article of apparel — something to wear. Name the items from the anagram given. 5 fibers briefs 20160110 Every answer this week is the name of an article of apparel — something to wear. Name the items from the anagram given. 6 haunts sun hat 20160110 Every answer this week is the name of an article of apparel — something to wear. Name the items from the anagram given. 7 repaid diapers 20160110 Every answer this week is the name of an article of apparel — something to wear. Name the items from the anagram given. 8 ungreased dungarees 20190714 I'm going to give you some words and phrases. Each conceals the name of something in left-to-right order, although not consecutively. Hint: Every answer has exactly 7 letters. 1 REVERSE SPLIT — Mountain Everest 20190714 I'm going to give you some words and phrases. Each conceals the name of something in left-to-right order, although not consecutively. Hint: Every answer has exactly 7 letters. 2 AFFRANCHISE — Pope Francis 20190714 I'm going to give you some words and phrases. Each conceals the name of something in left-to-right order, although not consecutively. Hint: Every answer has exactly 7 letters. 4 SERVICE STATION — Norse explorer Ericson 20190714 I'm going to give you some words and phrases. Each conceals the name of something in left-to-right order, although not consecutively. Hint: Every answer has exactly 7 letters. 5 CARDIAC ARREST — South American capital Caracas 20190714 I'm going to give you some words and phrases. Each conceals the name of something in left-to-right order, although not consecutively. Hint: Every answer has exactly 7 letters. 6 MOTOR CONTROL — Canadian city Toronto 20190714 I'm going to give you some words and phrases. Each conceals the name of something in left-to-right order, although not consecutively. Hint: Every answer has exactly 7 letters. 7 TIME INTERVAL — Roman goddess Minerva 20190714 I'm going to give you some words and phrases. Each conceals the name of something in left-to-right order, although not consecutively. Hint: Every answer has exactly 7 letters. 8 PETROLEUM JELLY — Ancient astronomer Ptolemy 20190714 I'm going to give you some words and phrases. Each conceals the name of something in left-to-right order, although not consecutively. Hint: Every answer has exactly 7 letters. 9 THEORETICALLY — Conservative commentator O’Reilly 20190714 I'm going to give you some words and phrases. Each conceals the name of something in left-to-right order, although not consecutively. Hint: Every answer has exactly 7 letters. 10 NO-WIN SITUATION — Brand of cigarettes Winston 20190714 I'm going to give you some words and phrases. Each conceals the name of something in left-to-right order, although not consecutively. Hint: Every answer has exactly 7 letters. 11 SEARCH PARTY — Aviator Earhart 20210124 Every answer today is a word or name that sounds like it starts with two spoken letters of the alphabet. 1 Place for camels to drink oasis (OA) 20210124 Every answer today is a word or name that sounds like it starts with two spoken letters of the alphabet. 2 Capital of Austria Vienna (VN) 20210124 Every answer today is a word or name that sounds like it starts with two spoken letters of the alphabet. 3 Try to equal or excel emulate (MU) 20210124 Every answer today is a word or name that sounds like it starts with two spoken letters of the alphabet. 4 Boredom tedium (TD) 20210124 Every answer today is a word or name that sounds like it starts with two spoken letters of the alphabet. 5 9 x 9 eighty-one (AT) 20210124 Every answer today is a word or name that sounds like it starts with two spoken letters of the alphabet. 6 Activity of secret agents espionage (SP) 20210124 Every answer today is a word or name that sounds like it starts with two spoken letters of the alphabet. 7 One involved in trickery deceiver (DC) 20210124 Every answer today is a word or name that sounds like it starts with two spoken letters of the alphabet. 8 Following orders (3 phonetic letters from now on) obedient (OBD) 20210124 Every answer today is a word or name that sounds like it starts with two spoken letters of the alphabet. 9 Online travel agency that competes with Travelocity Expedia (XPD) 20210124 Every answer today is a word or name that sounds like it starts with two spoken letters of the alphabet. 10 "[Fill in the blank:] """"___, my dear Watson""""" elementary (LMN) 20171209 Change one letter in each word to name something to eat. 1 BREAK bread 20171209 Change one letter in each word to name something to eat. 2 BUDGE fudge 20171209 Change one letter in each word to name something to eat. 4 BARON bacon 20171209 Change one letter in each word to name something to eat. 5 FILED filet 20171209 Change one letter in each word to name something to eat. 6 DUMBO gumbo 20171209 Change one letter in each word to name something to eat. 7 LIVEN liver 20171209 Change one letter in each word to name something to eat. 8 PRUDE prune 20171209 Change one letter in each word to name something to eat. 9 PASTY pasta (not Cornish Pasty) 20171209 Change one letter in each word to name something to eat. 10 CHILD chili 20171209 Change one letter in each word to name something to eat. 11 BUTLER butter 20171209 Change one letter in each word to name something to eat. 12 TICKLE pickle 20171209 Change one letter in each word to name something to eat. 13 BOOKIE cookie 20171209 Change one letter in each word to name something to eat. 14 POODLE noodle 20171209 Change one letter in each word to name something to eat. 15 CLEESE cheese 20171209 Change one letter in each word to name something to eat. 17 RAN OUT ragout 20171209 Change one letter in each word to name something to eat. 18 DELISH relish 20151129 For each category given, I'll name something in the category that closely follows the name of the category alphabetically.You tell me the only other thing in the category that fits between these two things alphabetically. 1 card suits; diamonds clubs 20151129 For each category given, I'll name something in the category that closely follows the name of the category alphabetically.You tell me the only other thing in the category that fits between these two things alphabetically. 2 presidents; Roosevelt Reagan 20151129 For each category given, I'll name something in the category that closely follows the name of the category alphabetically.You tell me the only other thing in the category that fits between these two things alphabetically. 3 Old Testament books; Psalms Proverbs 20151129 For each category given, I'll name something in the category that closely follows the name of the category alphabetically.You tell me the only other thing in the category that fits between these two things alphabetically. 4 Disney dwarfs; Dopey Doc 20151129 For each category given, I'll name something in the category that closely follows the name of the category alphabetically.You tell me the only other thing in the category that fits between these two things alphabetically. 5 Kardashians; Kim Khloé 20140112 "Every answer is a word that begins and ends with the letter A. You'll be given an anagram of the letters between the A's. For example, given ""ern,"" you would say, ""arena.""" 1 rot aorta 20140112 "Every answer is a word that begins and ends with the letter A. You'll be given an anagram of the letters between the A's. For example, given ""ern,"" you would say, ""arena.""" 2 pen apnea 20140112 "Every answer is a word that begins and ends with the letter A. You'll be given an anagram of the letters between the A's. For example, given ""ern,"" you would say, ""arena.""" 3 clap alpaca 20140112 "Every answer is a word that begins and ends with the letter A. You'll be given an anagram of the letters between the A's. For example, given ""ern,"" you would say, ""arena.""" 4 dram armada 20140112 "Every answer is a word that begins and ends with the letter A. You'll be given an anagram of the letters between the A's. For example, given ""ern,"" you would say, ""arena.""" 5 laze azalea 20140112 "Every answer is a word that begins and ends with the letter A. You'll be given an anagram of the letters between the A's. For example, given ""ern,"" you would say, ""arena.""" 6 mine anemia 20140112 "Every answer is a word that begins and ends with the letter A. You'll be given an anagram of the letters between the A's. For example, given ""ern,"" you would say, ""arena.""" 7 Deng agenda 20140112 "Every answer is a word that begins and ends with the letter A. You'll be given an anagram of the letters between the A's. For example, given ""ern,"" you would say, ""arena.""" 8 ice dam academia 20140112 "Every answer is a word that begins and ends with the letter A. You'll be given an anagram of the letters between the A's. For example, given ""ern,"" you would say, ""arena.""" 9 in sheets anesthesia 20140112 "Every answer is a word that begins and ends with the letter A. You'll be given an anagram of the letters between the A's. For example, given ""ern,"" you would say, ""arena.""" 10 crime (answers are now geographical places) America 20140112 "Every answer is a word that begins and ends with the letter A. You'll be given an anagram of the letters between the A's. For example, given ""ern,"" you would say, ""arena.""" 11 renting Argentina 20140112 "Every answer is a word that begins and ends with the letter A. You'll be given an anagram of the letters between the A's. For example, given ""ern,"" you would say, ""arena.""" 12 rituals Australia 20130210 "Every answer is a three-letter word that ends a familiar two-word phrase. The clue will be the first word of the phrase. The three letters in your answer will always be found, in some order, inside the first word. For example, if the clue is ""arctic,"" the answer could be ""air."" " 1 carpenter ant 20130210 "Every answer is a three-letter word that ends a familiar two-word phrase. The clue will be the first word of the phrase. The three letters in your answer will always be found, in some order, inside the first word. For example, if the clue is ""arctic,"" the answer could be ""air.""" 2 abstract art 20130210 "Every answer is a three-letter word that ends a familiar two-word phrase. The clue will be the first word of the phrase. The three letters in your answer will always be found, in some order, inside the first word. For example, if the clue is ""arctic,"" the answer could be ""air.""" 3 electric eel 20130210 "Every answer is a three-letter word that ends a familiar two-word phrase. The clue will be the first word of the phrase. The three letters in your answer will always be found, in some order, inside the first word. For example, if the clue is ""arctic,"" the answer could be ""air.""" 4 tennis net 20130210 "Every answer is a three-letter word that ends a familiar two-word phrase. The clue will be the first word of the phrase. The three letters in your answer will always be found, in some order, inside the first word. For example, if the clue is ""arctic,"" the answer could be ""air.""" 5 double bed 20130210 "Every answer is a three-letter word that ends a familiar two-word phrase. The clue will be the first word of the phrase. The three letters in your answer will always be found, in some order, inside the first word. For example, if the clue is ""arctic,"" the answer could be ""air.""" 6 drilling rig 20130210 "Every answer is a three-letter word that ends a familiar two-word phrase. The clue will be the first word of the phrase. The three letters in your answer will always be found, in some order, inside the first word. For example, if the clue is ""arctic,"" the answer could be ""air.""" 7 young gun 20130210 "Every answer is a three-letter word that ends a familiar two-word phrase. The clue will be the first word of the phrase. The three letters in your answer will always be found, in some order, inside the first word. For example, if the clue is ""arctic,"" the answer could be ""air.""" 8 switch hit 20130210 "Every answer is a three-letter word that ends a familiar two-word phrase. The clue will be the first word of the phrase. The three letters in your answer will always be found, in some order, inside the first word. For example, if the clue is ""arctic,"" the answer could be ""air.""" 9 race car 20130210 "Every answer is a three-letter word that ends a familiar two-word phrase. The clue will be the first word of the phrase. The three letters in your answer will always be found, in some order, inside the first word. For example, if the clue is ""arctic,"" the answer could be ""air.""" 10 grab bag 20130210 "Every answer is a three-letter word that ends a familiar two-word phrase. The clue will be the first word of the phrase. The three letters in your answer will always be found, in some order, inside the first word. For example, if the clue is ""arctic,"" the answer could be ""air.""" 11 stage set 20130210 "Every answer is a three-letter word that ends a familiar two-word phrase. The clue will be the first word of the phrase. The three letters in your answer will always be found, in some order, inside the first word. For example, if the clue is ""arctic,"" the answer could be ""air.""" 12 legal age 20130210 "Every answer is a three-letter word that ends a familiar two-word phrase. The clue will be the first word of the phrase. The three letters in your answer will always be found, in some order, inside the first word. For example, if the clue is ""arctic,"" the answer could be ""air.""" 13 copper ore 20130210 "Every answer is a three-letter word that ends a familiar two-word phrase. The clue will be the first word of the phrase. The three letters in your answer will always be found, in some order, inside the first word. For example, if the clue is ""arctic,"" the answer could be ""air.""" 14 butcher’s cut 20110130 You are given clues, and each answer has the letters S, N, O and W in it. 1 Anagram the letters of SNOW to make two common words owns, sown, not wons 20110130 You are given clues, and each answer has the letters S, N, O and W in it. 2 The last name of what U.S. president contains the letters S, N, O and W in left to right order (although not consecutively) [20th Century president after 1950] Eisenhower 20110130 You are given clues, and each answer has the letters S, N, O and W in it. 3 A 7-letter word for part of the day contains the letter, S, N, O and W in left-to-right order, but not consecutively. What is it? sundown 20110130 You are given clues, and each answer has the letters S, N, O and W in it. 4 Imagine the phrase NO SNOW written in capital letters. Double one of the letters and turn the result upside- down and you will make what familiar 7-letter word? monsoon 20110130 You are given clues, and each answer has the letters S, N, O and W in it. 5 There is a fictional character from story and film whose name contains two W’s in a row. The first half of the name is SNOW, what is the second half? White 20110130 You are given clues, and each answer has the letters S, N, O and W in it. 6 Two compound words starting with SNOW end in the letters ALL. What are they? snowball, snowfall (not snow squall because it is a 2- word phrase 20110130 You are given clues, and each answer has the letters S, N, O and W in it. 7 Riddle: What does a snowman use to keep his pants up? snow belt 20110130 You are given clues, and each answer has the letters S, N, O and W in it. 8 Riddle: Where can you save snow so it will collect interest? snow bank 20110130 You are given clues, and each answer has the letters S, N, O and W in it. 9 Change one letter in SNOW to a new letter to make a common 4-letter word. The new letter is silent. What is the word? know 20180318 Every answer today is a made up two-word phrase in which the first word starts DE- and the second word sounds like the first one without the DE-. 1 Hates exams detests tests 20180318 Every answer today is a made up two-word phrase in which the first word starts DE- and the second word sounds like the first one without the DE-. 2 Lowers trenches filled with water as means of defending castles demotes moats 20180318 Every answer today is a made up two-word phrase in which the first word starts DE- and the second word sounds like the first one without the DE-. 3 Says exactly what monetary penalties will be defines fines 20180318 Every answer today is a made up two-word phrase in which the first word starts DE- and the second word sounds like the first one without the DE-. 4 Figures out which cards are the twos deduces deuces 20180318 Every answer today is a made up two-word phrase in which the first word starts DE- and the second word sounds like the first one without the DE-. 5 Puts off the arrival of Hawaiian garlands delays leis 20180318 Every answer today is a made up two-word phrase in which the first word starts DE- and the second word sounds like the first one without the DE-. 6 Puts off the arrival of sables and minks defers furs 20180318 Every answer today is a made up two-word phrase in which the first word starts DE- and the second word sounds like the first one without the DE-. 7 Argues about whether worms or flies are better for catching fish debates baits 20180318 Every answer today is a made up two-word phrase in which the first word starts DE- and the second word sounds like the first one without the DE-. 8 Shows that two beds stacked one on the other are worthless debunks bunks 20180318 Every answer today is a made up two-word phrase in which the first word starts DE- and the second word sounds like the first one without the DE-. 9 Reduces the number of folds in paper decreases creases, or depletes pleats 20180318 Every answer today is a made up two-word phrase in which the first word starts DE- and the second word sounds like the first one without the DE-. 10 Brings supplies for pate de foie gras delivers livers 20180318 Every answer today is a made up two-word phrase in which the first word starts DE- and the second word sounds like the first one without the DE-. 11 Makes fun of the roller coaster and merry-go-round derides rides 20180318 Every answer today is a made up two-word phrase in which the first word starts DE- and the second word sounds like the first one without the DE-. 12 Represents old-fashioned writers through words denotes notes 20180318 Every answer today is a made up two-word phrase in which the first word starts DE- and the second word sounds like the first one without the DE-. 13 Says no to designers Calvin and Anne declines Kleins 20120729 "Every answer is the name of a former Olympic sport. Given an anagram, you name the sport. For example, """"flog"""" becomes """"golf.""""" 1 sent in tennis 20120729 "Every answer is the name of a former Olympic sport. Given an anagram, you name the sport. For example, """"flog"""" becomes """"golf.""""" 2 a cherry archery 20120729 "Every answer is the name of a former Olympic sport. Given an anagram, you name the sport. For example, """"flog"""" becomes """"golf.""""" 3 I signal sailing 20120729 "Every answer is the name of a former Olympic sport. Given an anagram, you name the sport. For example, """"flog"""" becomes """"golf.""""" 4 lob aloft football 20120729 "Every answer is the name of a former Olympic sport. Given an anagram, you name the sport. For example, """"flog"""" becomes """"golf.""""" 5 red gases dressage 20120729 "Every answer is the name of a former Olympic sport. Given an anagram, you name the sport. For example, """"flog"""" becomes """"golf.""""" 6 its no hog shooting 20120729 "Every answer is the name of a former Olympic sport. Given an anagram, you name the sport. For example, """"flog"""" becomes """"golf.""""" 7 halt intro triathlon 20120729 "Every answer is the name of a former Olympic sport. Given an anagram, you name the sport. For example, """"flog"""" becomes """"golf.""""" 8 primal tone trampoline 20120729 "Every answer is the name of a former Olympic sport. Given an anagram, you name the sport. For example, """"flog"""" becomes """"golf.""""" 9 nine battles table tennis 20171008 I'm going to read you some sentences. Each sentence contains two words that phonetically sound like a world capital. The words will be in left-to-right order, but not necessarily consecutive. 1 The opening bell rang for the grade school. Belgrade (Serbia) 20171008 I'm going to read you some sentences. Each sentence contains two words that phonetically sound like a world capital. The words will be in left-to-right order, but not necessarily consecutive. 2 The Chinese kid trying to sell me a Buddha figure was such a pest. Budapest (Hungary) 20171008 I'm going to read you some sentences. Each sentence contains two words that phonetically sound like a world capital. The words will be in left-to-right order, but not necessarily consecutive. 3 Mom got a bag of golf balls for dad on his birthday. Baghdad (Iraq) 20171008 I'm going to read you some sentences. Each sentence contains two words that phonetically sound like a world capital. The words will be in left-to-right order, but not necessarily consecutive. 4 The Russians watched a black car whisk past Lenin's Tomb. Khartoum (Sudan) 20171008 I'm going to read you some sentences. Each sentence contains two words that phonetically sound like a world capital. The words will be in left-to-right order, but not necessarily consecutive. 5 The dress that sis wore is one I never saw before. Warsaw (Poland) 20171008 I'm going to read you some sentences. Each sentence contains two words that phonetically sound like a world capital. The words will be in left-to-right order, but not necessarily consecutive. 6 We got a good view of the triple play from our seats in Row E. Tripoli (Libya) 20171008 I'm going to read you some sentences. Each sentence contains two words that phonetically sound like a world capital. The words will be in left-to-right order, but not necessarily consecutive. 7 Baseball fans in the Bay area root for the Giants. Beirut (Lebanon) 20171008 I'm going to read you some sentences. Each sentence contains two words that phonetically sound like a world capital. The words will be in left-to-right order, but not necessarily consecutive. 8 After a meal at a Thai restaurant, I rose to pay the bill. Taipei (Taiwan) 20171008 I'm going to read you some sentences. Each sentence contains two words that phonetically sound like a world capital. The words will be in left-to-right order, but not necessarily consecutive. 9 If the kids bang your car, don't let them give you some cock and bull story. Bangkok (Thailand) 20171008 I'm going to read you some sentences. Each sentence contains two words that phonetically sound like a world capital. The words will be in left-to-right order, but not necessarily consecutive. 10 It sounds like hell when you sing off-key. Helsinki (Finland) 20171008 I'm going to read you some sentences. Each sentence contains two words that phonetically sound like a world capital. The words will be in left-to-right order, but not necessarily consecutive. 11 If you see Bill Nye in the front row, be quiet. Nairobi (Kenya) 20171008 I'm going to read you some sentences. Each sentence contains two words that phonetically sound like a world capital. The words will be in left-to-right order, but not necessarily consecutive. 12 "After taking his cat to the vet, the man wondered, """"What should I do now?""""" Kathmandu (Nepal) 20171008 I'm going to read you some sentences. Each sentence contains two words that phonetically sound like a world capital. The words will be in left-to-right order, but not necessarily consecutive. 13 Even in the poor light, I could see the dinosaur's toe prints. Port-au-Prince (Haiti) 20171008 I'm going to read you some sentences. Each sentence contains two words that phonetically sound like a world capital. The words will be in left-to-right order, but not necessarily consecutive. 14 """Am I the one who stirred the pot? I am!""" Amsterdam (Netherlands) 20130512 You are given two words starting with M-A. The answer is a third word that can follow the first one and precede the second one, in each case to complete a compound word or a familiar two-word phrase. 1 Mandarin, marmalade orange 20130512 You are given two words starting with M-A. The answer is a third word that can follow the first one and precede the second one, in each case to complete a compound word or a familiar two-word phrase. 2 malted, maid milk 20130512 You are given two words starting with M-A. The answer is a third word that can follow the first one and precede the second one, in each case to complete a compound word or a familiar two-word phrase. 3 Marco, mallet Polo 20130512 You are given two words starting with M-A. The answer is a third word that can follow the first one and precede the second one, in each case to complete a compound word or a familiar two-word phrase. 4 main, map street 20130512 You are given two words starting with M-A. The answer is a third word that can follow the first one and precede the second one, in each case to complete a compound word or a familiar two-word phrase. 5 magnetic, mall strip 20130512 You are given two words starting with M-A. The answer is a third word that can follow the first one and precede the second one, in each case to complete a compound word or a familiar two-word phrase. 6 mail, machine slot 20130512 You are given two words starting with M-A. The answer is a third word that can follow the first one and precede the second one, in each case to complete a compound word or a familiar two-word phrase. 7 Matthew, Mason Perry 20130512 You are given two words starting with M-A. The answer is a third word that can follow the first one and precede the second one, in each case to complete a compound word or a familiar two-word phrase. 8 mad, market money 20130512 You are given two words starting with M-A. The answer is a third word that can follow the first one and precede the second one, in each case to complete a compound word or a familiar two-word phrase. 9 mathematical, manners table 20170910 For each word, change one letter in it to name a drink. 1 COFFER coffee 20170910 For each word, change one letter in it to name a drink. 2 TOPIC tonic 20170910 For each word, change one letter in it to name a drink. 3 WIDER cider 20170910 For each word, change one letter in it to name a drink. 4 WHISKER whiskey 20170910 For each word, change one letter in it to name a drink. 5 PINCH punch 20170910 For each word, change one letter in it to name a drink. 6 SCORCH scotch 20170910 For each word, change one letter in it to name a drink. 7 MARTINS martini 20170910 For each word, change one letter in it to name a drink. 8 POSTER porter 20170910 For each word, change one letter in it to name a drink. 9 MILD milk 20170910 For each word, change one letter in it to name a drink. 10 BEAR beer 20170910 For each word, change one letter in it to name a drink. 11 LEAD mead 20170910 For each word, change one letter in it to name a drink. 12 WINK wine 20170910 For each word, change one letter in it to name a drink. 13 COLD cola, not Colt 45 20170910 For each word, change one letter in it to name a drink. 14 LAKE sake 20170910 For each word, change one letter in it to name a drink. 15 BRANDS brandy 20170910 For each word, change one letter in it to name a drink. 16 SO LONG oolong 20191006 I'm going to give you some five-letter words. For each one, add two letters at the end to create a common, uncapitalized seven-letter word that is unrelated in meaning to the first one. 1 belie believe 20191006 I'm going to give you some five-letter words. For each one, add two letters at the end to create a common, uncapitalized seven-letter word that is unrelated in meaning to the first one. 2 sever several 20191006 I'm going to give you some five-letter words. For each one, add two letters at the end to create a common, uncapitalized seven-letter word that is unrelated in meaning to the first one. 3 trick trickle 20191006 I'm going to give you some five-letter words. For each one, add two letters at the end to create a common, uncapitalized seven-letter word that is unrelated in meaning to the first one. 4 hatch hatchet 20191006 I'm going to give you some five-letter words. For each one, add two letters at the end to create a common, uncapitalized seven-letter word that is unrelated in meaning to the first one. 5 prude prudent 20191006 I'm going to give you some five-letter words. For each one, add two letters at the end to create a common, uncapitalized seven-letter word that is unrelated in meaning to the first one. 6 scour scourge 20191006 I'm going to give you some five-letter words. For each one, add two letters at the end to create a common, uncapitalized seven-letter word that is unrelated in meaning to the first one. 7 opera operate 20191006 I'm going to give you some five-letter words. For each one, add two letters at the end to create a common, uncapitalized seven-letter word that is unrelated in meaning to the first one. 8 scalp scalpel 20191006 I'm going to give you some five-letter words. For each one, add two letters at the end to create a common, uncapitalized seven-letter word that is unrelated in meaning to the first one. 9 masse masseur 20191006 I'm going to give you some five-letter words. For each one, add two letters at the end to create a common, uncapitalized seven-letter word that is unrelated in meaning to the first one. 10 shmoo shmooze 20191006 I'm going to give you some five-letter words. For each one, add two letters at the end to create a common, uncapitalized seven-letter word that is unrelated in meaning to the first one. 11 again   against 20191006 I'm going to give you some five-letter words. For each one, add two letters at the end to create a common, uncapitalized seven-letter word that is unrelated in meaning to the first one. 12 colon   colonel 20190210 I'm going to read you two words. Think of a third word that can follow my first one and precede my second one, in each case to complete a common two-word phrase. As a help, each answer starts with the letter O. 1 Pecking Form order 20190210 I'm going to read you two words. Think of a third word that can follow my first one and precede my second one, in each case to complete a common two-word phrase. As a help, each answer starts with the letter O. 2 Snake Well oil 20190210 I'm going to read you two words. Think of a third word that can follow my first one and precede my second one, in each case to complete a common two-word phrase. As a help, each answer starts with the letter O. 3 Oval Hours office 20190210 I'm going to read you two words. Think of a third word that can follow my first one and precede my second one, in each case to complete a common two-word phrase. As a help, each answer starts with the letter O. 4 Agent Juice orange 20190210 I'm going to read you two words. Think of a third word that can follow my first one and precede my second one, in each case to complete a common two-word phrase. As a help, each answer starts with the letter O. 5 Second Poll opinion 20190210 I'm going to read you two words. Think of a third word that can follow my first one and precede my second one, in each case to complete a common two-word phrase. As a help, each answer starts with the letter O. 6 Soap House opera 20190210 I'm going to read you two words. Think of a third word that can follow my first one and precede my second one, in each case to complete a common two-word phrase. As a help, each answer starts with the letter O. 7 House Grinder organ 20190210 I'm going to read you two words. Think of a third word that can follow my first one and precede my second one, in each case to complete a common two-word phrase. As a help, each answer starts with the letter O. 8 U.S. Sesame open 20190210 I'm going to read you two words. Think of a third word that can follow my first one and precede my second one, in each case to complete a common two-word phrase. As a help, each answer starts with the letter O. 9 Green Rings onion 20190210 I'm going to read you two words. Think of a third word that can follow my first one and precede my second one, in each case to complete a common two-word phrase. As a help, each answer starts with the letter O. 10 Polar Sex opposite 20190210 I'm going to read you two words. Think of a third word that can follow my first one and precede my second one, in each case to complete a common two-word phrase. As a help, each answer starts with the letter O. 11 Golden Knocks opportunity 20101107 "You are given a word and must provide a second word to complete a familiar two-word phrase. The first letter of the word must be the last letter of the word given, and the last letter of the word must be the first letter of the word given. For example, given the clue ""photo,"" the answer would be ""op.""" 1 goose (3 letter answers) egg 20101107 "You are given a word and must provide a second word to complete a familiar two-word phrase. The first letter of the word must be the last letter of the word given, and the last letter of the word must be the first letter of the word given. For example, given the clue ""photo,"" the answer would be ""op.""" 2 plastic cup, cap 20101107 "You are given a word and must provide a second word to complete a familiar two-word phrase. The first letter of the word must be the last letter of the word given, and the last letter of the word must be the first letter of the word given. For example, given the clue ""photo,"" the answer would be ""op.""" 3 Tea Act 20101107 "You are given a word and must provide a second word to complete a familiar two-word phrase. The first letter of the word must be the last letter of the word given, and the last letter of the word must be the first letter of the word given. For example, given the clue ""photo,"" the answer would be ""op.""" 4 salad (4 letter answers now) days 20101107 "You are given a word and must provide a second word to complete a familiar two-word phrase. The first letter of the word must be the last letter of the word given, and the last letter of the word must be the first letter of the word given. For example, given the clue ""photo,"" the answer would be ""op.""" 5 lap pool 20101107 "You are given a word and must provide a second word to complete a familiar two-word phrase. The first letter of the word must be the last letter of the word given, and the last letter of the word must be the first letter of the word given. For example, given the clue ""photo,"" the answer would be ""op.""" 6 traffic (5 letter answers now) court 20101107 "You are given a word and must provide a second word to complete a familiar two-word phrase. The first letter of the word must be the last letter of the word given, and the last letter of the word must be the first letter of the word given. For example, given the clue ""photo,"" the answer would be ""op.""" 7 socioeconomic class 20101107 "You are given a word and must provide a second word to complete a familiar two-word phrase. The first letter of the word must be the last letter of the word given, and the last letter of the word must be the first letter of the word given. For example, given the clue ""photo,"" the answer would be ""op.""" 8 electric (6 letter answers) charge, candle 20101107 "You are given a word and must provide a second word to complete a familiar two-word phrase. The first letter of the word must be the last letter of the word given, and the last letter of the word must be the first letter of the word given. For example, given the clue ""photo,"" the answer would be ""op.""" 9 nether region 20101107 "You are given a word and must provide a second word to complete a familiar two-word phrase. The first letter of the word must be the last letter of the word given, and the last letter of the word must be the first letter of the word given. For example, given the clue ""photo,"" the answer would be ""op.""" 10 Emeril (7 letters) Lagasse 20101107 "You are given a word and must provide a second word to complete a familiar two-word phrase. The first letter of the word must be the last letter of the word given, and the last letter of the word must be the first letter of the word given. For example, given the clue ""photo,"" the answer would be ""op.""" 11 National Lampoon 20101107 "You are given a word and must provide a second word to complete a familiar two-word phrase. The first letter of the word must be the last letter of the word given, and the last letter of the word must be the first letter of the word given. For example, given the clue ""photo,"" the answer would be ""op.""" 12 radio (8 letters) operator 20101107 "You are given a word and must provide a second word to complete a familiar two-word phrase. The first letter of the word must be the last letter of the word given, and the last letter of the word must be the first letter of the word given. For example, given the clue ""photo,"" the answer would be ""op.""" 13 enfant (French) terrible 20160814 "The phrases """"peace of mind"""" and """"state of mind"""" differ only in their first words. Their ends, """"of mind,"""" are the same. Here are the first words of other familiar pairs of three-word phrases that have """"of"""" in the middle and the same words at the end. Find the final word in the phrase." 1 facts, quality of life 20160814 "The phrases """"peace of mind"""" and """"state of mind"""" differ only in their first words. Their ends, """"of mind,"""" are the same. Here are the first words of other familiar pairs of three-word phrases that have """"of"""" in the middle and the same words at the end. Find the final word in the phrase." 2 can, diet of worms 20160814 "The phrases """"peace of mind"""" and """"state of mind"""" differ only in their first words. Their ends, """"of mind,"""" are the same. Here are the first words of other familiar pairs of three-word phrases that have """"of"""" in the middle and the same words at the end. Find the final word in the phrase." 3 maid, medal of honor 20160814 "The phrases """"peace of mind"""" and """"state of mind"""" differ only in their first words. Their ends, """"of mind,"""" are the same. Here are the first words of other familiar pairs of three-word phrases that have """"of"""" in the middle and the same words at the end. Find the final word in the phrase." 4 nick, waste of time 20160814 "The phrases """"peace of mind"""" and """"state of mind"""" differ only in their first words. Their ends, """"of mind,"""" are the same. Here are the first words of other familiar pairs of three-word phrases that have """"of"""" in the middle and the same words at the end. Find the final word in the phrase." 5 break, time of day 20160814 "The phrases """"peace of mind"""" and """"state of mind"""" differ only in their first words. Their ends, """"of mind,"""" are the same. Here are the first words of other familiar pairs of three-word phrases that have """"of"""" in the middle and the same words at the end. Find the final word in the phrase." 6 wheel, soldier of fortune 20160814 "The phrases """"peace of mind"""" and """"state of mind"""" differ only in their first words. Their ends, """"of mind,"""" are the same. Here are the first words of other familiar pairs of three-word phrases that have """"of"""" in the middle and the same words at the end. Find the final word in the phrase." 7 line, ball of fire 20160814 "The phrases """"peace of mind"""" and """"state of mind"""" differ only in their first words. Their ends, """"of mind,"""" are the same. Here are the first words of other familiar pairs of three-word phrases that have """"of"""" in the middle and the same words at the end. Find the final word in the phrase." 8 school, train of thought 20160814 "The phrases """"peace of mind"""" and """"state of mind"""" differ only in their first words. Their ends, """"of mind,"""" are the same. Here are the first words of other familiar pairs of three-word phrases that have """"of"""" in the middle and the same words at the end. Find the final word in the phrase." 9 park, figure, freedom of speech 20110612 "You are given three words, starting with the letters T, O and P, and must come up with a fourth word that can follow the words and complete a familiar two-word phrase. For example, given the words """"taste,"""" """"oral"""" and """"paternity,"""" the answer would be """"test,"""" as in """"taste test,"""" """"oral test"""" and """"paternity test.""""" 1 ticket, oval, patent office 20110612 "You are given three words, starting with the letters T, O and P, and must come up with a fourth word that can follow the words and complete a familiar two-word phrase. For example, given the words """"taste,"""" """"oral"""" and """"paternity,"""" the answer would be """"test,"""" as in """"taste test,"""" """"oral test"""" and """"paternity test.""""" 2 ten-gallon, old, Panama hat 20110612 "You are given three words, starting with the letters T, O and P, and must come up with a fourth word that can follow the words and complete a familiar two-word phrase. For example, given the words """"taste,"""" """"oral"""" and """"paternity,"""" the answer would be """"test,"""" as in """"taste test,"""" """"oral test"""" and """"paternity test.""""" 3 tree, open, publishing house 20110612 "You are given three words, starting with the letters T, O and P, and must come up with a fourth word that can follow the words and complete a familiar two-word phrase. For example, given the words """"taste,"""" """"oral"""" and """"paternity,"""" the answer would be """"test,"""" as in """"taste test,"""" """"oral test"""" and """"paternity test.""""" 4 tree, onion, piston ring 20110612 "You are given three words, starting with the letters T, O and P, and must come up with a fourth word that can follow the words and complete a familiar two-word phrase. For example, given the words """"taste,"""" """"oral"""" and """"paternity,"""" the answer would be """"test,"""" as in """"taste test,"""" """"oral test"""" and """"paternity test.""""" 5 triple, outer, parking space 20110612 "You are given three words, starting with the letters T, O and P, and must come up with a fourth word that can follow the words and complete a familiar two-word phrase. For example, given the words """"taste,"""" """"oral"""" and """"paternity,"""" the answer would be """"test,"""" as in """"taste test,"""" """"oral test"""" and """"paternity test.""""" 6 telephone, ordinal, prime number 20110612 "You are given three words, starting with the letters T, O and P, and must come up with a fourth word that can follow the words and complete a familiar two-word phrase. For example, given the words """"taste,"""" """"oral"""" and """"paternity,"""" the answer would be """"test,"""" as in """"taste test,"""" """"oral test"""" and """"paternity test.""""" 7 tapas, oyster, parallel bar 20110612 "You are given three words, starting with the letters T, O and P, and must come up with a fourth word that can follow the words and complete a familiar two-word phrase. For example, given the words """"taste,"""" """"oral"""" and """"paternity,"""" the answer would be """"test,"""" as in """"taste test,"""" """"oral test"""" and """"paternity test.""""" 8 throne, operating, pool room 20110612 "You are given three words, starting with the letters T, O and P, and must come up with a fourth word that can follow the words and complete a familiar two-word phrase. For example, given the words """"taste,"""" """"oral"""" and """"paternity,"""" the answer would be """"test,"""" as in """"taste test,"""" """"oral test"""" and """"paternity test.""""" 9 tar, orchestra, peach pit 20110612 "You are given three words, starting with the letters T, O and P, and must come up with a fourth word that can follow the words and complete a familiar two-word phrase. For example, given the words """"taste,"""" """"oral"""" and """"paternity,"""" the answer would be """"test,"""" as in """"taste test,"""" """"oral test"""" and """"paternity test.""""" 10 tootsie, onion, piano roll 20110612 "You are given three words, starting with the letters T, O and P, and must come up with a fourth word that can follow the words and complete a familiar two-word phrase. For example, given the words """"taste,"""" """"oral"""" and """"paternity,"""" the answer would be """"test,"""" as in """"taste test,"""" """"oral test"""" and """"paternity test.""""" 11 tea, old, punching bag 20110612 "You are given three words, starting with the letters T, O and P, and must come up with a fourth word that can follow the words and complete a familiar two-word phrase. For example, given the words """"taste,"""" """"oral"""" and """"paternity,"""" the answer would be """"test,"""" as in """"taste test,"""" """"oral test"""" and """"paternity test.""""" 12 tray, operating, periodic table 20140831 Every answer this week is a made-up two-word phrase, in which both words start with 'S' and they're anagrams of each other. 1 A wise man playing video games from a Nintendo rival. Sega sage 20140831 Every answer this week is a made-up two-word phrase, in which both words start with 'S' and they're anagrams of each other. 2 An event at which a performing circus animal is marked down in price. seal sale 20140831 Every answer this week is a made-up two-word phrase, in which both words start with 'S' and they're anagrams of each other. 3 A healing ointment for a person in bondage. slave salve 20140831 Every answer this week is a made-up two-word phrase, in which both words start with 'S' and they're anagrams of each other. 4 A dirty mark on a shiny piece of fabric. satin stain 20140831 Every answer this week is a made-up two-word phrase, in which both words start with 'S' and they're anagrams of each other. 5 A Spanish gentleman’s sleep sounds. senor’s snores 20140831 Every answer this week is a made-up two-word phrase, in which both words start with 'S' and they're anagrams of each other. 6 An angel that could accompany you up Mt. Everest Sherpa seraph 20140831 Every answer this week is a made-up two-word phrase, in which both words start with 'S' and they're anagrams of each other. 7 Golfer Sam’s hard-topped cars Snead’s sedans 20140831 Every answer this week is a made-up two-word phrase, in which both words start with 'S' and they're anagrams of each other. 8 An attempt by an Iroquois tribe to communicate with the dead. Seneca seance 20140831 Every answer this week is a made-up two-word phrase, in which both words start with 'S' and they're anagrams of each other. 9 A conspicuous soda cracker. salient saltine 20140831 Every answer this week is a made-up two-word phrase, in which both words start with 'S' and they're anagrams of each other. 10 A fish packed tightly in a can that has more beach grit in it. sandier sardine 20140309 For each five-letter word provided, insert two letters after the first letter to complete a familiar seven-letter word. 1 march on, monarch 20140309 For each five-letter word provided, insert two letters after the first letter to complete a familiar seven-letter word. 2 bathe re, breathe 20140309 For each five-letter word provided, insert two letters after the first letter to complete a familiar seven-letter word. 3 feign or, foreign 20140309 For each five-letter word provided, insert two letters after the first letter to complete a familiar seven-letter word. 4 onion pi, opinion 20140309 For each five-letter word provided, insert two letters after the first letter to complete a familiar seven-letter word. 6 clone yc, cyclone 20140309 For each five-letter word provided, insert two letters after the first letter to complete a familiar seven-letter word. 7 villa an, vanilla 20140309 For each five-letter word provided, insert two letters after the first letter to complete a familiar seven-letter word. 8 rival ev, revival 20140309 For each five-letter word provided, insert two letters after the first letter to complete a familiar seven-letter word. 9 elude xc, exclude 20140309 For each five-letter word provided, insert two letters after the first letter to complete a familiar seven-letter word. 10 gaffe ir, giraffe 20200112 "Every answer is a compound word in which the vowel sound in each half is a short """"a"""" — as in """"grassland"""" or """"madcap.""""" 1 Father's father granddad 20200112 "Every answer is a compound word in which the vowel sound in each half is a short """"a"""" — as in """"grassland"""" or """"madcap.""""" 2 Item in a levee sandbag 20200112 "Every answer is a compound word in which the vowel sound in each half is a short """"a"""" — as in """"grassland"""" or """"madcap.""""" 3 Slang term for someone who talks gasbab 20200112 "Every answer is a compound word in which the vowel sound in each half is a short """"a"""" — as in """"grassland"""" or """"madcap.""""" 4 Symbol in a tweet hashtag 20200112 "Every answer is a compound word in which the vowel sound in each half is a short """"a"""" — as in """"grassland"""" or """"madcap.""""" 5 Sudden remembrance of something past flashback, not aha 20200112 "Every answer is a compound word in which the vowel sound in each half is a short """"a"""" — as in """"grassland"""" or """"madcap.""""" 6 Carrying case with straps that's worn over the shoulders backpack, or napsack 20200112 "Every answer is a compound word in which the vowel sound in each half is a short """"a"""" — as in """"grassland"""" or """"madcap.""""" 7 Pancake flapjack 20200112 "Every answer is a compound word in which the vowel sound in each half is a short """"a"""" — as in """"grassland"""" or """"madcap.""""" 8 Game of 21 blackjack 20200112 "Every answer is a compound word in which the vowel sound in each half is a short """"a"""" — as in """"grassland"""" or """"madcap.""""" 9 Low-growing weed that can quickly take over a lawn crabgrass 20200112 "Every answer is a compound word in which the vowel sound in each half is a short """"a"""" — as in """"grassland"""" or """"madcap.""""" 10 Old-fashioned timer that you periodically turn over sandglass 20200112 "Every answer is a compound word in which the vowel sound in each half is a short """"a"""" — as in """"grassland"""" or """"madcap.""""" 11 Place where taxis line up cabstand 20200112 "Every answer is a compound word in which the vowel sound in each half is a short """"a"""" — as in """"grassland"""" or """"madcap.""""" 12 What a pirate might be made to walk gangplank 20200112 "Every answer is a compound word in which the vowel sound in each half is a short """"a"""" — as in """"grassland"""" or """"madcap.""""" 13 Done hurriedly and carelessly slapjack 20200112 "Every answer is a compound word in which the vowel sound in each half is a short """"a"""" — as in """"grassland"""" or """"madcap.""""" 14 Make individually without machine help handcraft 20200112 "Every answer is a compound word in which the vowel sound in each half is a short """"a"""" — as in """"grassland"""" or """"madcap.""""" 15 Item you step onto when you step out of the shower bathmat 20130908 Each of the following answers is a made-up, two-word phrase in which the two words are homophones, and both words start with the letter C. 1 bank draft from Prague Czech check 20130908 Each of the following answers is a made-up, two-word phrase in which the two words are homophones, and both words start with the letter C. 3 bean dish that has been refrigerated chilly chili 20130908 Each of the following answers is a made-up, two-word phrase in which the two words are homophones, and both words start with the letter C. 4 place to play golf that has a rough surface coarse course 20130908 Each of the following answers is a made-up, two-word phrase in which the two words are homophones, and both words start with the letter C. 5 small seedless raisin that is now in use current currant 20130908 Each of the following answers is a made-up, two-word phrase in which the two words are homophones, and both words start with the letter C. 6 reason for a crow’s noise caw’s cause 20130908 Each of the following answers is a made-up, two-word phrase in which the two words are homophones, and both words start with the letter C. 7 drink to celebrate with at the University of Illinois Champaign champagne 20130908 Each of the following answers is a made-up, two-word phrase in which the two words are homophones, and both words start with the letter C. 8 ability to handle pennies intelligently [second words start with letters other than C from now on] cents sense 20130908 Each of the following answers is a made-up, two-word phrase in which the two words are homophones, and both words start with the letter C. 9 line up of billiard sticks cue queue 20130908 Each of the following answers is a made-up, two-word phrase in which the two words are homophones, and both words start with the letter C. 10 bits of corn from certain army officers colonels’ kernels 20130908 Each of the following answers is a made-up, two-word phrase in which the two words are homophones, and both words start with the letter C. 11 one with a business in the basement cellar seller 20141109 "Given a category, name something whose first two letters are the first and last letters of the category. For example, given """"Animal,"""" you would say """"Alligator"""" or """"Alpaca.""""" 1 country Cyprus 20141109 "Given a category, name something whose first two letters are the first and last letters of the category. For example, given """"Animal,"""" you would say """"Alligator"""" or """"Alpaca.""""" 2 color crimson 20141109 "Given a category, name something whose first two letters are the first and last letters of the category. For example, given """"Animal,"""" you would say """"Alligator"""" or """"Alpaca.""""" 3 tree teak (or maybe tea) 20141109 "Given a category, name something whose first two letters are the first and last letters of the category. For example, given """"Animal,"""" you would say """"Alligator"""" or """"Alpaca.""""" 4 measure megabyte, meter 20141109 "Given a category, name something whose first two letters are the first and last letters of the category. For example, given """"Animal,"""" you would say """"Alligator"""" or """"Alpaca.""""" 5 occupation oncologist 20141109 "Given a category, name something whose first two letters are the first and last letters of the category. For example, given """"Animal,"""" you would say """"Alligator"""" or """"Alpaca.""""" 6 TV show Twin Peaks, Two and a Half Men 20141109 "Given a category, name something whose first two letters are the first and last letters of the category. For example, given """"Animal,"""" you would say """"Alligator"""" or """"Alpaca.""""" 7 best picture Ben Hur, A Beautiful Mind, The Best Years of Our Lives 20141109 "Given a category, name something whose first two letters are the first and last letters of the category. For example, given """"Animal,"""" you would say """"Alligator"""" or """"Alpaca.""""" 8 leaf vegetable lettuce 20190728 "This week's puzzle is called """"High C's."""" Every answer is a word or name with the accented syllable """"see"""" (in any spelling) somewhere inside it." 1 Place in ancient Rome where gladiators fought Colosseum 20190728 "This week's puzzle is called """"High C's."""" Every answer is a word or name with the accented syllable """"see"""" (in any spelling) somewhere inside it." 2 Pill used in medical tests that has no medical effect placebo 20190728 "This week's puzzle is called """"High C's."""" Every answer is a word or name with the accented syllable """"see"""" (in any spelling) somewhere inside it." 3 Resident of Nashville or Memphis Tennesseean 20190728 "This week's puzzle is called """"High C's."""" Every answer is a word or name with the accented syllable """"see"""" (in any spelling) somewhere inside it." 4 The noun that a pronoun refers back to antecedent 20190728 "This week's puzzle is called """"High C's."""" Every answer is a word or name with the accented syllable """"see"""" (in any spelling) somewhere inside it." 5 Like some hairlines and tides receding 20190728 "This week's puzzle is called """"High C's."""" Every answer is a word or name with the accented syllable """"see"""" (in any spelling) somewhere inside it." 6 Classic Notre Dame football coach Ara Parseghian 20190728 "This week's puzzle is called """"High C's."""" Every answer is a word or name with the accented syllable """"see"""" (in any spelling) somewhere inside it." 7 Establishment in Las Vegas or Monte Carlo casino 20190728 "This week's puzzle is called """"High C's."""" Every answer is a word or name with the accented syllable """"see"""" (in any spelling) somewhere inside it." 8 Fashion designer Oleg Cassini 20190728 "This week's puzzle is called """"High C's."""" Every answer is a word or name with the accented syllable """"see"""" (in any spelling) somewhere inside it." 9 Baseball Hall-of-Fame pitcher Mike Mussina 20190728 "This week's puzzle is called """"High C's."""" Every answer is a word or name with the accented syllable """"see"""" (in any spelling) somewhere inside it." 10 Public building for concerts and lectures lyceum 20190728 "This week's puzzle is called """"High C's."""" Every answer is a word or name with the accented syllable """"see"""" (in any spelling) somewhere inside it." 11 Not meant to be taken seriously, as a comment facetious 20190728 "This week's puzzle is called """"High C's."""" Every answer is a word or name with the accented syllable """"see"""" (in any spelling) somewhere inside it." 12 Part of a theater stage in front of the curtain proscenium 20190728 "This week's puzzle is called """"High C's."""" Every answer is a word or name with the accented syllable """"see"""" (in any spelling) somewhere inside it." 13 Player who's the target of a football pass receiver 20151108 "This week's puzzle is called """"Bus Fare."""" Every answer is a familiar two-word phrase, in which the first word starts with BU- and the second word starts with S." 1 nickname for Ohio Buckeye State 20151108 "This week's puzzle is called """"Bus Fare."""" Every answer is a familiar two-word phrase, in which the first word starts with BU- and the second word starts with S." 2 an insect repellent, comes in a can bug spray 20151108 "This week's puzzle is called """"Bus Fare."""" Every answer is a familiar two-word phrase, in which the first word starts with BU- and the second word starts with S." 3 Crayola color that is reddish brown burnt sienna 20151108 "This week's puzzle is called """"Bus Fare."""" Every answer is a familiar two-word phrase, in which the first word starts with BU- and the second word starts with S." 4 place to buy fresh meat butcher shop 20151108 "This week's puzzle is called """"Bus Fare."""" Every answer is a familiar two-word phrase, in which the first word starts with BU- and the second word starts with S." 5 one of two passenger positions in the front of a sports car bucket seat 20151108 "This week's puzzle is called """"Bus Fare."""" Every answer is a familiar two-word phrase, in which the first word starts with BU- and the second word starts with S." 6 informal spontaneous discussion bull session 20151108 "This week's puzzle is called """"Bus Fare."""" Every answer is a familiar two-word phrase, in which the first word starts with BU- and the second word starts with S." 7 what George W. Bush graduated from at Harvard business school 20151108 "This week's puzzle is called """"Bus Fare."""" Every answer is a familiar two-word phrase, in which the first word starts with BU- and the second word starts with S." 8 pulpy orange vegetable often pureed for soups butternut squash 20151108 "This week's puzzle is called """"Bus Fare."""" Every answer is a familiar two-word phrase, in which the first word starts with BU- and the second word starts with S." 9 extra amount when a government’s income exceeds its expenditures budget surplus 20151108 "This week's puzzle is called """"Bus Fare."""" Every answer is a familiar two-word phrase, in which the first word starts with BU- and the second word starts with S." 10 1960’s folk rock group with the hit, For What It’s Worth Buffalo Springfield 20151108 "This week's puzzle is called """"Bus Fare."""" Every answer is a familiar two-word phrase, in which the first word starts with BU- and the second word starts with S." 11 company that used to advertise with a series of humorous signs along highways Burma Shave 20151108 "This week's puzzle is called """"Bus Fare."""" Every answer is a familiar two-word phrase, in which the first word starts with BU- and the second word starts with S." 12 you might have one of these for a political candidate on the back of your car bumper sticker 20160710 Every answer this week is a word with a double-B, like RABBIT. Rearrange the letters in the word provided to get the answer. 1 iron + BB ribbon 20160710 Every answer this week is a word with a double-B, like RABBIT. Rearrange the letters in the word provided to get the answer. 2 hays + BB shabby 20160710 Every answer this week is a word with a double-B, like RABBIT. Rearrange the letters in the word provided to get the answer. 3 lure + BB rubble (not burble) 20160710 Every answer this week is a word with a double-B, like RABBIT. Rearrange the letters in the word provided to get the answer. 4 seas + BB abbess 20160710 Every answer this week is a word with a double-B, like RABBIT. Rearrange the letters in the word provided to get the answer. 5 ogle + BB gobble 20160710 Every answer this week is a word with a double-B, like RABBIT. Rearrange the letters in the word provided to get the answer. 6 yowl + BB wobbly 20160710 Every answer this week is a word with a double-B, like RABBIT. Rearrange the letters in the word provided to get the answer. 7 shine + BB nebbish 20160710 Every answer this week is a word with a double-B, like RABBIT. Rearrange the letters in the word provided to get the answer. 8 mulled + BB dumbbell 20180114 I'm going to give you clues for two words. Reverse the last two letters of the first word to get the second word. 1 "Squiggle over a Spanish """"n"""" / Like a bathroom floor" tilde, tiled 20180114 I'm going to give you clues for two words. Reverse the last two letters of the first word to get the second word. 2 Move along slowly / Thin pancake creep, crepe 20180114 I'm going to give you clues for two words. Reverse the last two letters of the first word to get the second word. 3 Variety of grape for wine-making / Spotted horse pinot, pinto 20180114 I'm going to give you clues for two words. Reverse the last two letters of the first word to get the second word. 4 Something that's 90 degrees on a square / Heavenly figure angle, angel 20180114 I'm going to give you clues for two words. Reverse the last two letters of the first word to get the second word. 5 "Bread-making need / Irish poet who wrote """"The Lake Isle of Innisfree""""" yeast, Yeats 20180114 I'm going to give you clues for two words. Reverse the last two letters of the first word to get the second word. 6 English county / More timid shire, shier 20180114 I'm going to give you clues for two words. Reverse the last two letters of the first word to get the second word. 7 Feature on a necklace / Applauds clasp, claps 20180114 I'm going to give you clues for two words. Reverse the last two letters of the first word to get the second word. 8 First sign of the zodiac / Get out of bed Aries, arise 20120909 "You are given sentences with two blanks. Put a word starting with R in the first blank. Then move that R to the end to make a new word that goes in the second blank to complete the sentence. For example, given the sentence, """"The door of the Indian ___ was left slightly ___,"""" the answers would be """"raja"""" and """"ajar.""""" 1 When the chef burned himself on the _____, he exploded in _____. range, anger 20120909 "You are given sentences with two blanks. Put a word starting with R in the first blank. Then move that R to the end to make a new word that goes in the second blank to complete the sentence. For example, given the sentence, """"The door of the Indian ___ was left slightly ___,"""" the answers would be """"raja"""" and """"ajar.""""" 2 The traveler’s _____ across Asia passed through _____ Mongolia. route, Outer 20120909 "You are given sentences with two blanks. Put a word starting with R in the first blank. Then move that R to the end to make a new word that goes in the second blank to complete the sentence. For example, given the sentence, """"The door of the Indian ___ was left slightly ___,"""" the answers would be """"raja"""" and """"ajar.""""" 3 Years ago, members of the Pawnee and Kiowa tribes would _____ all _____ the plains. rove, over 20120909 "You are given sentences with two blanks. Put a word starting with R in the first blank. Then move that R to the end to make a new word that goes in the second blank to complete the sentence. For example, given the sentence, """"The door of the Indian ___ was left slightly ___,"""" the answers would be """"raja"""" and """"ajar.""""" 4 A _____ in Southwest Arizona is just the sort of property the local news _____ wants to buy. ranco, anchor 20120909 "You are given sentences with two blanks. Put a word starting with R in the first blank. Then move that R to the end to make a new word that goes in the second blank to complete the sentence. For example, given the sentence, """"The door of the Indian ___ was left slightly ___,"""" the answers would be """"raja"""" and """"ajar.""""" 5 The actor who seemed distant and _____ during the audition turned out to be quite the _____ when he got on the stage. remote, emoter 20120909 "You are given sentences with two blanks. Put a word starting with R in the first blank. Then move that R to the end to make a new word that goes in the second blank to complete the sentence. For example, given the sentence, """"The door of the Indian ___ was left slightly ___,"""" the answers would be """"raja"""" and """"ajar.""""" 6 Between Madonna and her ex-husband, Guy _____, it was always the latter who was _____ to get going. Ritchie, itchier 20150308 "Every answer is the name of a well-known U.S. city and its state. One or more letters from the start of the city's name plus one or more letters from the start of the state's name are run together to spell a word. I'll give you the word. You tell me the city and state. For example, given """"latex,"""" the answer would be """"Laredo, Texas.""""" 1 provider, 10 Providence, Rhode Island 20150308 "Every answer is the name of a well-known U.S. city and its state. One or more letters from the start of the city's name plus one or more letters from the start of the state's name are run together to spell a word. I'll give you the word. You tell me the city and state. For example, given """"latex,"""" the answer would be """"Laredo, Texas.""""" 3 chillin, 7 Chicago, Illinois 20150308 "Every answer is the name of a well-known U.S. city and its state. One or more letters from the start of the city's name plus one or more letters from the start of the state's name are run together to spell a word. I'll give you the word. You tell me the city and state. For example, given """"latex,"""" the answer would be """"Laredo, Texas.""""" 4 savage, 8 Savannah, Georgia 20150308 "Every answer is the name of a well-known U.S. city and its state. One or more letters from the start of the city's name plus one or more letters from the start of the state's name are run together to spell a word. I'll give you the word. You tell me the city and state. For example, given """"latex,"""" the answer would be """"Laredo, Texas.""""" 5 mover, 10 Montpelier, Vermont 20150308 "Every answer is the name of a well-known U.S. city and its state. One or more letters from the start of the city's name plus one or more letters from the start of the state's name are run together to spell a word. I'll give you the word. You tell me the city and state. For example, given """"latex,"""" the answer would be """"Laredo, Texas.""""" 6 deco, 6 Denver, Colorado 20150308 "Every answer is the name of a well-known U.S. city and its state. One or more letters from the start of the city's name plus one or more letters from the start of the state's name are run together to spell a word. I'll give you the word. You tell me the city and state. For example, given """"latex,"""" the answer would be """"Laredo, Texas.""""" 7 poor, 8 Portland, Oregon 20150308 "Every answer is the name of a well-known U.S. city and its state. One or more letters from the start of the city's name plus one or more letters from the start of the state's name are run together to spell a word. I'll give you the word. You tell me the city and state. For example, given """"latex,"""" the answer would be """"Laredo, Texas.""""" 8 liar, 10 (2 words) Little Rock, Arkansas 20171029 Every answer is a compound word with two hyphens in which the part in the middle has two letters. 1 Carousel merry-go-round 20171029 Every answer is a compound word with two hyphens in which the part in the middle has two letters. 2 Officer during a meeting who stands near the door sargent-at-arms 20171029 Every answer is a compound word with two hyphens in which the part in the middle has two letters. 3 Basic piece of lumber two-by-four 20171029 Every answer is a compound word with two hyphens in which the part in the middle has two letters. 4 Street that's a dead end cul-de-sac 20171029 Every answer is a compound word with two hyphens in which the part in the middle has two letters. 5 Relative of one's spouse who is stereotypically hard to get along with mother-in-law (answer by Mike); father-in-law (answer by Lulu) 20171029 Every answer is a compound word with two hyphens in which the part in the middle has two letters. 6 Smarty-pants; a person who pretends to have all the answers know-it-all 20171029 Every answer is a compound word with two hyphens in which the part in the middle has two letters. 7 State flower of Alaska, whose name suggests you'll always remember it forget-me-not 20171029 Every answer is a compound word with two hyphens in which the part in the middle has two letters. 8 Capital of Haiti Port-au-Prince 20171029 Every answer is a compound word with two hyphens in which the part in the middle has two letters. 9 Person who can represent you in a legal case attorney-in-fact 20171029 Every answer is a compound word with two hyphens in which the part in the middle has two letters. 10 "French name for blackjack, which translates as """"twenty-one""""" vingt-et-un 20171029 Every answer is a compound word with two hyphens in which the part in the middle has two letters. 11 Decorative symbol with a French name Flor-de-Lis 20171029 Every answer is a compound word with two hyphens in which the part in the middle has two letters. 12 Basic kind of knot used to tie a necktie four-in-hand 20171029 Every answer is a compound word with two hyphens in which the part in the middle has two letters. 13 Person who is lazy and irresponsible ne’er-do-well 20171029 Every answer is a compound word with two hyphens in which the part in the middle has two letters. 14 Relatively affluent well-to-do 20120708 "For each six-letter word given, rearrange the letters to make two three-letter words that rhyme. Example: For the word """"tweets,"""" the pair of rhyming three-letter words would be """"wet"""" and """"set.""""" 1 attach cat, hat 20120708 "For each six-letter word given, rearrange the letters to make two three-letter words that rhyme. Example: For the word """"tweets,"""" the pair of rhyming three-letter words would be """"wet"""" and """"set.""""" 2 papacy cap, yap 20120708 "For each six-letter word given, rearrange the letters to make two three-letter words that rhyme. Example: For the word """"tweets,"""" the pair of rhyming three-letter words would be """"wet"""" and """"set.""""" 3 inning gin, inn 20120708 "For each six-letter word given, rearrange the letters to make two three-letter words that rhyme. Example: For the word """"tweets,"""" the pair of rhyming three-letter words would be """"wet"""" and """"set.""""" 4 poorer pro, roe 20120708 "For each six-letter word given, rearrange the letters to make two three-letter words that rhyme. Example: For the word """"tweets,"""" the pair of rhyming three-letter words would be """"wet"""" and """"set.""""" 5 sheiks she, ski 20120708 "For each six-letter word given, rearrange the letters to make two three-letter words that rhyme. Example: For the word """"tweets,"""" the pair of rhyming three-letter words would be """"wet"""" and """"set.""""" 6 neuron one, run 20120708 "For each six-letter word given, rearrange the letters to make two three-letter words that rhyme. Example: For the word """"tweets,"""" the pair of rhyming three-letter words would be """"wet"""" and """"set.""""" 7 yields die, sly 20120708 "For each six-letter word given, rearrange the letters to make two three-letter words that rhyme. Example: For the word """"tweets,"""" the pair of rhyming three-letter words would be """"wet"""" and """"set.""""" 8 errata are, tar 20120708 "For each six-letter word given, rearrange the letters to make two three-letter words that rhyme. Example: For the word """"tweets,"""" the pair of rhyming three-letter words would be """"wet"""" and """"set.""""" 9 A-sharp rah, spa 20121230 "This week is the annual """"new names in the news"""" quiz. You're given some names that you probably never heard of before 2012, but who made news during the past 12 months. You say who they are. These names were compiled with the help of Kathie Baker and Tim Goodman, who were players on previous year-end quizzes." 1 Mohamed Morsi New president of Egypt 20121230 "This week is the annual """"new names in the news"""" quiz. You're given some names that you probably never heard of before 2012, but who made news during the past 12 months. You say who they are. These names were compiled with the help of Kathie Baker and Tim Goodman, who were players on previous year-end quizzes." 2 Jeremy Lin Basketball player who inspired “Linsanity” with the Knicks and then moved to the Houston Rockets 20121230 "This week is the annual """"new names in the news"""" quiz. You're given some names that you probably never heard of before 2012, but who made news during the past 12 months. You say who they are. These names were compiled with the help of Kathie Baker and Tim Goodman, who were players on previous year-end quizzes." 3 Francesco Schettino Captain of the Costa Concordia when it ran aground 20121230 "This week is the annual """"new names in the news"""" quiz. You're given some names that you probably never heard of before 2012, but who made news during the past 12 months. You say who they are. These names were compiled with the help of Kathie Baker and Tim Goodman, who were players on previous year-end quizzes." 4 Gabby Douglas Olympic gymnast 20121230 "This week is the annual """"new names in the news"""" quiz. You're given some names that you probably never heard of before 2012, but who made news during the past 12 months. You say who they are. These names were compiled with the help of Kathie Baker and Tim Goodman, who were players on previous year-end quizzes." 5 Paula Broadwell Gen. Petraeus’ biographer with whom he had an affair 20121230 "This week is the annual """"new names in the news"""" quiz. You're given some names that you probably never heard of before 2012, but who made news during the past 12 months. You say who they are. These names were compiled with the help of Kathie Baker and Tim Goodman, who were players on previous year-end quizzes." 6 Oscar Pistorius South African racer, first double leg amputee to participate in the Olympics 20121230 "This week is the annual """"new names in the news"""" quiz. You're given some names that you probably never heard of before 2012, but who made news during the past 12 months. You say who they are. These names were compiled with the help of Kathie Baker and Tim Goodman, who were players on previous year-end quizzes." 7 Felix Baumgartner Jumped from a balloon 39 km high and became the first person to break the sound barrier without vehicular power 20121230 "This week is the annual """"new names in the news"""" quiz. You're given some names that you probably never heard of before 2012, but who made news during the past 12 months. You say who they are. These names were compiled with the help of Kathie Baker and Tim Goodman, who were players on previous year-end quizzes." 8 Malala Yousafzai Pakistani teenager and blogger for educating girls shot in the head and neck in an assassination attempt by Taliban gunmen 20121230 "This week is the annual """"new names in the news"""" quiz. You're given some names that you probably never heard of before 2012, but who made news during the past 12 months. You say who they are. These names were compiled with the help of Kathie Baker and Tim Goodman, who were players on previous year-end quizzes." 9 Maria Alyokhina, Nadezhda Tolokonnikova, Yekaterina Samutsevich, Russian punk rockers and Pussy Riot members jailed for mocking Putin 20121230 "This week is the annual """"new names in the news"""" quiz. You're given some names that you probably never heard of before 2012, but who made news during the past 12 months. You say who they are. These names were compiled with the help of Kathie Baker and Tim Goodman, who were players on previous year-end quizzes." 10 Psy South Korean rapper of Gangnam Style -- the first video to get over one billion hits on YouTube 20180211 For each word, drop the first letter and rearrange those that remain to name part of the human body. 1 OGLE LEG 20180211 For each word, drop the first letter and rearrange those that remain to name part of the human body. 2 PINCH CHIN 20180211 For each word, drop the first letter and rearrange those that remain to name part of the human body. 3 BONES NOSE 20180211 For each word, drop the first letter and rearrange those that remain to name part of the human body. 4 DECAF FACE 20180211 For each word, drop the first letter and rearrange those that remain to name part of the human body. 5 KNISH SHIN 20180211 For each word, drop the first letter and rearrange those that remain to name part of the human body. 6 ITCHES CHEST 20180211 For each word, drop the first letter and rearrange those that remain to name part of the human body. 7 ROTORS TORSO 20180211 For each word, drop the first letter and rearrange those that remain to name part of the human body. 8 GATHER HEART 20180211 For each word, drop the first letter and rearrange those that remain to name part of the human body. 9 PITSAW WAIST 20180211 For each word, drop the first letter and rearrange those that remain to name part of the human body. 10 EIGHTH THIGH 20180211 For each word, drop the first letter and rearrange those that remain to name part of the human body. 11 SLIPUP PUPIL 20180211 For each word, drop the first letter and rearrange those that remain to name part of the human body. 13 OUTGONE TONGUE 20180211 For each word, drop the first letter and rearrange those that remain to name part of the human body. 14 EARWORM MARROW 20180211 For each word, drop the first letter and rearrange those that remain to name part of the human body. 15 RELATION TOENAIL 20180211 For each word, drop the first letter and rearrange those that remain to name part of the human body. 16 HAMBONED ABDOMEN 20160911 "If you alphabetized the eight planets in reverse — that is, by their ending letters — the next-to-last name on the list would be Venus, which ends in """"s."""" The last planet on the list would be Mercury, which ends in """"y."""" I'm going to give you some categories. For each one, I'll give you the next-to-last member of the category, if all the names in it were alphabetized backward. You tell me the last name." 1 numbers from one to ten; eight six 20160911 "If you alphabetized the eight planets in reverse — that is, by their ending letters — the next-to-last name on the list would be Venus, which ends in """"s."""" The last planet on the list would be Mercury, which ends in """"y."""" I'm going to give you some categories. For each one, I'll give you the next-to-last member of the category, if all the names in it were alphabetized backward. You tell me the last name." 2 Beatles (by first name); John Ringo 20160911 "If you alphabetized the eight planets in reverse — that is, by their ending letters — the next-to-last name on the list would be Venus, which ends in """"s."""" The last planet on the list would be Mercury, which ends in """"y."""" I'm going to give you some categories. For each one, I'll give you the next-to-last member of the category, if all the names in it were alphabetized backward. You tell me the last name." 3 chess pieces; bishop knight 20160911 "If you alphabetized the eight planets in reverse — that is, by their ending letters — the next-to-last name on the list would be Venus, which ends in """"s."""" The last planet on the list would be Mercury, which ends in """"y."""" I'm going to give you some categories. For each one, I'll give you the next-to-last member of the category, if all the names in it were alphabetized backward. You tell me the last name." 4 Disney dwarfs; Happy Sneezy 20160911 "If you alphabetized the eight planets in reverse — that is, by their ending letters — the next-to-last name on the list would be Venus, which ends in """"s."""" The last planet on the list would be Mercury, which ends in """"y."""" I'm going to give you some categories. For each one, I'll give you the next-to-last member of the category, if all the names in it were alphabetized backward. You tell me the last name." 5 top ten car rental companies; Thrifty Hertz 20160911 "If you alphabetized the eight planets in reverse — that is, by their ending letters — the next-to-last name on the list would be Venus, which ends in """"s."""" The last planet on the list would be Mercury, which ends in """"y."""" I'm going to give you some categories. For each one, I'll give you the next-to-last member of the category, if all the names in it were alphabetized backward. You tell me the last name." 6 forms of music recognized by the Gammy’s; country jazz 20160911 "If you alphabetized the eight planets in reverse — that is, by their ending letters — the next-to-last name on the list would be Venus, which ends in """"s."""" The last planet on the list would be Mercury, which ends in """"y."""" I'm going to give you some categories. For each one, I'll give you the next-to-last member of the category, if all the names in it were alphabetized backward. You tell me the last name." 7 Ivy League schools; Princeton Brown 20160911 "If you alphabetized the eight planets in reverse — that is, by their ending letters — the next-to-last name on the list would be Venus, which ends in """"s."""" The last planet on the list would be Mercury, which ends in """"y."""" I'm going to give you some categories. For each one, I'll give you the next-to-last member of the category, if all the names in it were alphabetized backward. You tell me the last name." 8 modern birthstones; ruby topaz 20160911 "If you alphabetized the eight planets in reverse — that is, by their ending letters — the next-to-last name on the list would be Venus, which ends in """"s."""" The last planet on the list would be Mercury, which ends in """"y."""" I'm going to give you some categories. For each one, I'll give you the next-to-last member of the category, if all the names in it were alphabetized backward. You tell me the last name." 9 Major League Baseball teams; Red Sox White Sox 20191027 The three B's in classical music are Bach, Beethoven and Brahms. But in this puzzle they're three words starting with B. Each set can be followed by a fourth word to complete a compound word or a familiar two-word phrase. 1 1. (starting with three-letter answers) Bow Black Bolo tie 20191027 The three B's in classical music are Bach, Beethoven and Brahms. But in this puzzle they're three words starting with B. Each set can be followed by a fourth word to complete a compound word or a familiar two-word phrase. 2 2. Bar Bell Bunny hop 20191027 The three B's in classical music are Bach, Beethoven and Brahms. But in this puzzle they're three words starting with B. Each set can be followed by a fourth word to complete a compound word or a familiar two-word phrase. 3 3. Boom Ballot Bread box 20191027 The three B's in classical music are Bach, Beethoven and Brahms. But in this puzzle they're three words starting with B. Each set can be followed by a fourth word to complete a compound word or a familiar two-word phrase. 4 4. Brick Berlin Border (4-letter answers now) wall 20191027 The three B's in classical music are Bach, Beethoven and Brahms. But in this puzzle they're three words starting with B. Each set can be followed by a fourth word to complete a compound word or a familiar two-word phrase. 5 5. Building Bar Binary code 20191027 The three B's in classical music are Bach, Beethoven and Brahms. But in this puzzle they're three words starting with B. Each set can be followed by a fourth word to complete a compound word or a familiar two-word phrase. 6 6. Big Bank Booster shot 20191027 The three B's in classical music are Bach, Beethoven and Brahms. But in this puzzle they're three words starting with B. Each set can be followed by a fourth word to complete a compound word or a familiar two-word phrase. 7 7. Blood Bee Bottom line 20191027 The three B's in classical music are Bach, Beethoven and Brahms. But in this puzzle they're three words starting with B. Each set can be followed by a fourth word to complete a compound word or a familiar two-word phrase. 8 8. Ball Board Blame game 20191027 The three B's in classical music are Bach, Beethoven and Brahms. But in this puzzle they're three words starting with B. Each set can be followed by a fourth word to complete a compound word or a familiar two-word phrase. 9 9. Boat Bird Boarding (5 letter answers now) house 20191027 The three B's in classical music are Bach, Beethoven and Brahms. But in this puzzle they're three words starting with B. Each set can be followed by a fourth word to complete a compound word or a familiar two-word phrase. 10 10. Butter Bowie Butcher knife 20191027 The three B's in classical music are Bach, Beethoven and Brahms. But in this puzzle they're three words starting with B. Each set can be followed by a fourth word to complete a compound word or a familiar two-word phrase. 11 11. Bulletin Bill Backgammon board 20120513 "The word """"mother"""" has a surprising property. If you move the first two letters to the end, you get """"thermo,"""" the prefix for """"heat."""" Every answer today is another six-letter word that, when you move the first two letters to the end, you get another word or phrase." 1 join the armed forces; pay attention enlist, listen 20120513 "The word """"mother"""" has a surprising property. If you move the first two letters to the end, you get """"thermo,"""" the prefix for """"heat."""" Every answer today is another six-letter word that, when you move the first two letters to the end, you get another word or phrase." 2 feature on a zebra; most ready to pick, as fruit stripe, ripest 20120513 "The word """"mother"""" has a surprising property. If you move the first two letters to the end, you get """"thermo,"""" the prefix for """"heat."""" Every answer today is another six-letter word that, when you move the first two letters to the end, you get another word or phrase." 3 to maintain, as principles; armed robbery uphold, hold up 20120513 "The word """"mother"""" has a surprising property. If you move the first two letters to the end, you get """"thermo,"""" the prefix for """"heat."""" Every answer today is another six-letter word that, when you move the first two letters to the end, you get another word or phrase." 4 accompany to a party; Spanish conquistador whose expedition caused the downfall of the Aztecs escort, Cortes 20120513 "The word """"mother"""" has a surprising property. If you move the first two letters to the end, you get """"thermo,"""" the prefix for """"heat."""" Every answer today is another six-letter word that, when you move the first two letters to the end, you get another word or phrase." 5 Novelist Laurence; novelist Hemingway; a bird, for example (“double” answer) Sterne, Ernest, nester 20200426 "I'm going to read you some sentences. Each sentence ends in """"_____ to _____."""" Put two homophones in the blanks to complete it." 1 The thieves prowling around the Pittsburgh mill were looking for some _____ to _____. steel to steal 20200426 "I'm going to read you some sentences. Each sentence ends in """"_____ to _____."""" Put two homophones in the blanks to complete it." 2 To prepare the dough for the oven, the baker will first _____ to _____. need to knead 20200426 "I'm going to read you some sentences. Each sentence ends in """"_____ to _____."""" Put two homophones in the blanks to complete it." 3 Since my foot surgery, my toes have gotten better, but I still have to wait for my _____ to _____. heel to heal 20200426 "I'm going to read you some sentences. Each sentence ends in """"_____ to _____."""" Put two homophones in the blanks to complete it." 4 To make some money, the illustrator for the old Disney film has a sample _____ to _____. cel to sell; not frame 20200426 "I'm going to read you some sentences. Each sentence ends in """"_____ to _____."""" Put two homophones in the blanks to complete it." 5 Seeing the physician's boat moving erratically, the Coast Guard ordered the _____ to _____. doc to dock 20200426 "I'm going to read you some sentences. Each sentence ends in """"_____ to _____."""" Put two homophones in the blanks to complete it." 6 Regulations may not allow you to speak, but you always have the _____ to _____. right to write 20130929 "Every answer is a made-up, two-word phrase in which each word has two or more syllables. The first vowel sound in the first word is a short """"e."""" Change that short """"e"""" to a short """"a"""" sound, and phonetically you'll get the second word of the phrase. For example, given """"energetic backwoods father,"""" you would say """"peppy pappy.""""" 1 superior cake mix better batter 20130929 "Every answer is a made-up, two-word phrase in which each word has two or more syllables. The first vowel sound in the first word is a short """"e."""" Change that short """"e"""" to a short """"a"""" sound, and phonetically you'll get the second word of the phrase. For example, given """"energetic backwoods father,"""" you would say """"peppy pappy.""""" 2 money paid to the government in the Lone Star State Texas taxes 20130929 "Every answer is a made-up, two-word phrase in which each word has two or more syllables. The first vowel sound in the first word is a short """"e."""" Change that short """"e"""" to a short """"a"""" sound, and phonetically you'll get the second word of the phrase. For example, given """"energetic backwoods father,"""" you would say """"peppy pappy.""""" 3 Late Night host Jimmy, after committing a serious crime felon Fallon 20130929 "Every answer is a made-up, two-word phrase in which each word has two or more syllables. The first vowel sound in the first word is a short """"e."""" Change that short """"e"""" to a short """"a"""" sound, and phonetically you'll get the second word of the phrase. For example, given """"energetic backwoods father,"""" you would say """"peppy pappy.""""" 4 hot tasting salad ingredient, with a slightly crimson hue reddish radish 20130929 "Every answer is a made-up, two-word phrase in which each word has two or more syllables. The first vowel sound in the first word is a short """"e."""" Change that short """"e"""" to a short """"a"""" sound, and phonetically you'll get the second word of the phrase. For example, given """"energetic backwoods father,"""" you would say """"peppy pappy.""""" 5 framework composed of leafy heads from the garden lettuce lattice 20130929 "Every answer is a made-up, two-word phrase in which each word has two or more syllables. The first vowel sound in the first word is a short """"e."""" Change that short """"e"""" to a short """"a"""" sound, and phonetically you'll get the second word of the phrase. For example, given """"energetic backwoods father,"""" you would say """"peppy pappy.""""" 6 disorderly mob made of revolutionaries rebel rabble 20130929 "Every answer is a made-up, two-word phrase in which each word has two or more syllables. The first vowel sound in the first word is a short """"e."""" Change that short """"e"""" to a short """"a"""" sound, and phonetically you'll get the second word of the phrase. For example, given """"energetic backwoods father,"""" you would say """"peppy pappy.""""" 7 affliction involving the musical part of a song melody malady 20130303 "You will be given two words starting with the letter P. Name a third word starting with P that can follow the first one and precede the second one, in each case to complete a familiar two-word phrase. For example, given ""peer"" and ""point,"" you would say ""pressure,"" as in ""peer pressure"" and ""pressure point.""" 1 poison, pal pen 20130303 "You will be given two words starting with the letter P. Name a third word starting with P that can follow the first one and precede the second one, in each case to complete a familiar two-word phrase. For example, given ""peer"" and ""point,"" you would say ""pressure,"" as in ""peer pressure"" and ""pressure point.""" 2 pepperoni, parlor pizza 20130303 "You will be given two words starting with the letter P. Name a third word starting with P that can follow the first one and precede the second one, in each case to complete a familiar two-word phrase. For example, given ""peer"" and ""point,"" you would say ""pressure,"" as in ""peer pressure"" and ""pressure point.""" 3 political, pooper party 20130303 "You will be given two words starting with the letter P. Name a third word starting with P that can follow the first one and precede the second one, in each case to complete a familiar two-word phrase. For example, given ""peer"" and ""point,"" you would say ""pressure,"" as in ""peer pressure"" and ""pressure point.""" 4 parallel, place park 20130303 "You will be given two words starting with the letter P. Name a third word starting with P that can follow the first one and precede the second one, in each case to complete a familiar two-word phrase. For example, given ""peer"" and ""point,"" you would say ""pressure,"" as in ""peer pressure"" and ""pressure point.""" 5 pretty, pincher penny 20130303 "You will be given two words starting with the letter P. Name a third word starting with P that can follow the first one and precede the second one, in each case to complete a familiar two-word phrase. For example, given ""peer"" and ""point,"" you would say ""pressure,"" as in ""peer pressure"" and ""pressure point.""" 6 penthouse, peeve pet 20130303 "You will be given two words starting with the letter P. Name a third word starting with P that can follow the first one and precede the second one, in each case to complete a familiar two-word phrase. For example, given ""peer"" and ""point,"" you would say ""pressure,"" as in ""peer pressure"" and ""pressure point.""" 7 pike, performance peak 20200308 I'm going to give you two sets of three letters. Put the same two letters in front of each trio to complete a compound word or familiar two-word phrase. Ex. Ite Ale --> White Whale 1. Und Bin2. Ass Own 3. Bby Rse 4. Ste Ter 5. Dar Nge6. Ash OOD 7. Wer Int 8. Ell Ock 9. Tor Uth 10. Own Ead 11. Ain Ack 12. Ole Eat 13. And Ill 14. Tch Ker (two answers) 1 UND BIN round robin 20200308 I'm going to give you two sets of three letters. Put the same two letters in front of each trio to complete a compound word or familiar two-word phrase. Ex. Ite Ale --> White Whale 1. Und Bin2. Ass Own 3. Bby Rse 4. Ste Ter 5. Dar Nge6. Ash OOD 7. Wer Int 8. Ell Ock 9. Tor Uth 10. Own Ead 11. Ain Ack 12. Ole Eat 13. And Ill 14. Tch Ker (two answers) 2 ASS OWN class clown 20200308 I'm going to give you two sets of three letters. Put the same two letters in front of each trio to complete a compound word or familiar two-word phrase. Ex. Ite Ale --> White Whale 1. Und Bin2. Ass Own 3. Bby Rse 4. Ste Ter 5. Dar Nge6. Ash OOD 7. Wer Int 8. Ell Ock 9. Tor Uth 10. Own Ead 11. Ain Ack 12. Ole Eat 13. And Ill 14. Tch Ker (two answers) 3 BBY RSE hobby horse 20200308 I'm going to give you two sets of three letters. Put the same two letters in front of each trio to complete a compound word or familiar two-word phrase. Ex. Ite Ale --> White Whale 1. Und Bin2. Ass Own 3. Bby Rse 4. Ste Ter 5. Dar Nge6. Ash OOD 7. Wer Int 8. Ell Ock 9. Tor Uth 10. Own Ead 11. Ain Ack 12. Ole Eat 13. And Ill 14. Tch Ker (two answers) 4 STE TER waste water 20200308 I'm going to give you two sets of three letters. Put the same two letters in front of each trio to complete a compound word or familiar two-word phrase. Ex. Ite Ale --> White Whale 1. Und Bin2. Ass Own 3. Bby Rse 4. Ste Ter 5. Dar Nge6. Ash OOD 7. Wer Int 8. Ell Ock 9. Tor Uth 10. Own Ead 11. Ain Ack 12. Ole Eat 13. And Ill 14. Tch Ker (two answers) 5 DAR NGE radar range 20200308 I'm going to give you two sets of three letters. Put the same two letters in front of each trio to complete a compound word or familiar two-word phrase. Ex. Ite Ale --> White Whale 1. Und Bin2. Ass Own 3. Bby Rse 4. Ste Ter 5. Dar Nge6. Ash OOD 7. Wer Int 8. Ell Ock 9. Tor Uth 10. Own Ead 11. Ain Ack 12. Ole Eat 13. And Ill 14. Tch Ker (two answers) 6 ASH OOD flash flood 20200308 I'm going to give you two sets of three letters. Put the same two letters in front of each trio to complete a compound word or familiar two-word phrase. Ex. Ite Ale --> White Whale 1. Und Bin2. Ass Own 3. Bby Rse 4. Ste Ter 5. Dar Nge6. Ash OOD 7. Wer Int 8. Ell Ock 9. Tor Uth 10. Own Ead 11. Ain Ack 12. Ole Eat 13. And Ill 14. Tch Ker (two answers) 7 WER INT power point 20200308 I'm going to give you two sets of three letters. Put the same two letters in front of each trio to complete a compound word or familiar two-word phrase. Ex. Ite Ale --> White Whale 1. Und Bin2. Ass Own 3. Bby Rse 4. Ste Ter 5. Dar Nge6. Ash OOD 7. Wer Int 8. Ell Ock 9. Tor Uth 10. Own Ead 11. Ain Ack 12. Ole Eat 13. And Ill 14. Tch Ker (two answers) 8 TOR UTH motor mouth 20200308 I'm going to give you two sets of three letters. Put the same two letters in front of each trio to complete a compound word or familiar two-word phrase. Ex. Ite Ale --> White Whale 1. Und Bin2. Ass Own 3. Bby Rse 4. Ste Ter 5. Dar Nge6. Ash OOD 7. Wer Int 8. Ell Ock 9. Tor Uth 10. Own Ead 11. Ain Ack 12. Ole Eat 13. And Ill 14. Tch Ker (two answers) 9 OWN EAD brown bread 20200308 I'm going to give you two sets of three letters. Put the same two letters in front of each trio to complete a compound word or familiar two-word phrase. Ex. Ite Ale --> White Whale 1. Und Bin2. Ass Own 3. Bby Rse 4. Ste Ter 5. Dar Nge6. Ash OOD 7. Wer Int 8. Ell Ock 9. Tor Uth 10. Own Ead 11. Ain Ack 12. Ole Eat 13. And Ill 14. Tch Ker (two answers) 10 AIN ACK train track 20200308 I'm going to give you two sets of three letters. Put the same two letters in front of each trio to complete a compound word or familiar two-word phrase. Ex. Ite Ale --> White Whale 1. Und Bin2. Ass Own 3. Bby Rse 4. Ste Ter 5. Dar Nge6. Ash OOD 7. Wer Int 8. Ell Ock 9. Tor Uth 10. Own Ead 11. Ain Ack 12. Ole Eat 13. And Ill 14. Tch Ker (two answers) 11 OLE EAT whole wheat 20200308 I'm going to give you two sets of three letters. Put the same two letters in front of each trio to complete a compound word or familiar two-word phrase. Ex. Ite Ale --> White Whale 1. Und Bin2. Ass Own 3. Bby Rse 4. Ste Ter 5. Dar Nge6. Ash OOD 7. Wer Int 8. Ell Ock 9. Tor Uth 10. Own Ead 11. Ain Ack 12. Ole Eat 13. And Ill 14. Tch Ker (two answers) 12 TCH KER (2 answers) match maker, hitch hiker 20170730 Every answer today is a familiar two-word phrase or name with the stress pattern da-DAH-da da-DAH-da. 1 State capital east of Indianapolis, Indiana Columbus, Ohio 20170730 Every answer today is a familiar two-word phrase or name with the stress pattern da-DAH-da da-DAH-da. 2 Mountain range in the Far West Sierra Nevada 20170730 Every answer today is a familiar two-word phrase or name with the stress pattern da-DAH-da da-DAH-da. 3 Song in The Lion King with a Swahili title Hakuna Matata 20170730 Every answer today is a familiar two-word phrase or name with the stress pattern da-DAH-da da-DAH-da. 4 Treaty respecting the humane treatment of P.O.W.'s Geneva Convention 20170730 Every answer today is a familiar two-word phrase or name with the stress pattern da-DAH-da da-DAH-da. 5 Material for a pharaoh's scroll Egyptian papyrus 20170730 Every answer today is a familiar two-word phrase or name with the stress pattern da-DAH-da da-DAH-da. 6 Longtime popular car from Japan Toyota Carrolla 20170730 Every answer today is a familiar two-word phrase or name with the stress pattern da-DAH-da da-DAH-da. 7 Person with a government post gotten by ballot, not appointment elected official 20170730 Every answer today is a familiar two-word phrase or name with the stress pattern da-DAH-da da-DAH-da. 8 Medical term for chest pain angina pectoris 20170730 Every answer today is a familiar two-word phrase or name with the stress pattern da-DAH-da da-DAH-da. 9 White House daughter from 2009 to 2017 Malia Obama 20190421 I'm going to read you some sentences. Each one conceals the name of a U.S. city both phonetically and by spelling. Name the cities. 1 The governor did a handspring fielding questions in Illinois. Springfield 20190421 I'm going to read you some sentences. Each one conceals the name of a U.S. city both phonetically and by spelling. Name the cities. 2 My grandmother would belittle rocking chairs from Arkansas. Little Rock 20190421 I'm going to read you some sentences. Each one conceals the name of a U.S. city both phonetically and by spelling. Name the cities. 3 I'm looking for semipro vocational training in Utah. Provo 20190421 I'm going to read you some sentences. Each one conceals the name of a U.S. city both phonetically and by spelling. Name the cities. 4 Everyone hated to see Wilbur bankrupted in California. Burbank 20190421 I'm going to read you some sentences. Each one conceals the name of a U.S. city both phonetically and by spelling. Name the cities. 5 Let's plan singalongs all around Michigan. Lansing 20190421 I'm going to read you some sentences. Each one conceals the name of a U.S. city both phonetically and by spelling. Name the cities. 6 That's some weirdo vernacular in Delaware. Dover 20190421 I'm going to read you some sentences. Each one conceals the name of a U.S. city both phonetically and by spelling. Name the cities. 7 The diners ordered some jumbo calamari in Florida. Ocala 20190421 I'm going to read you some sentences. Each one conceals the name of a U.S. city both phonetically and by spelling. Name the cities. 8 You mistakenly called Cornwall a wall about a year ago in Washington. Walla Walla 20140914 "Every answer today is a word or name with an accented syllable """"pee"""" — spelled in any way — but always occurring inside the word, never at the start or end. For example, one saying the same thing again and again would be """"repeater.""""" 1 the capital of Kansas Topeka 20140914 "Every answer today is a word or name with an accented syllable """"pee"""" — spelled in any way — but always occurring inside the word, never at the start or end. For example, one saying the same thing again and again would be """"repeater.""""" 2 underwater missile torpedo 20140914 "Every answer today is a word or name with an accented syllable """"pee"""" — spelled in any way — but always occurring inside the word, never at the start or end. For example, one saying the same thing again and again would be """"repeater.""""" 3 Britannica or World Book encyclopedia 20140914 "Every answer today is a word or name with an accented syllable """"pee"""" — spelled in any way — but always occurring inside the word, never at the start or end. For example, one saying the same thing again and again would be """"repeater.""""" 4 chirp or moo onomatopoeia 20140914 "Every answer today is a word or name with an accented syllable """"pee"""" — spelled in any way — but always occurring inside the word, never at the start or end. For example, one saying the same thing again and again would be """"repeater.""""" 5 a person from Manila Filipino 20140914 "Every answer today is a word or name with an accented syllable """"pee"""" — spelled in any way — but always occurring inside the word, never at the start or end. For example, one saying the same thing again and again would be """"repeater.""""" 6 from France or Germany, for example European 20140914 "Every answer today is a word or name with an accented syllable """"pee"""" — spelled in any way — but always occurring inside the word, never at the start or end. For example, one saying the same thing again and again would be """"repeater.""""" 7 Italian dish made of thinly sliced veal scaloppini 20140914 "Every answer today is a word or name with an accented syllable """"pee"""" — spelled in any way — but always occurring inside the word, never at the start or end. For example, one saying the same thing again and again would be """"repeater.""""" 8 Calgary’s professional football team Stampeders 20140914 "Every answer today is a word or name with an accented syllable """"pee"""" — spelled in any way — but always occurring inside the word, never at the start or end. For example, one saying the same thing again and again would be """"repeater.""""" 9 fancy term for baldness alopecia 20140914 "Every answer today is a word or name with an accented syllable """"pee"""" — spelled in any way — but always occurring inside the word, never at the start or end. For example, one saying the same thing again and again would be """"repeater.""""" 10 popular travel web site Expedia 20140914 "Every answer today is a word or name with an accented syllable """"pee"""" — spelled in any way — but always occurring inside the word, never at the start or end. For example, one saying the same thing again and again would be """"repeater.""""" 11 a demand to appear in court subpoena 20201108 I'm going to give you some words. Add the letters Q-U and rearrange the result to get a new word. The Q-U can appear anywhere in the answer. 1 CRONE + QU CONQUER 20201108 I'm going to give you some words. Add the letters Q-U and rearrange the result to get a new word. The Q-U can appear anywhere in the answer. 2 TREAT + QU QUARTET 20201108 I'm going to give you some words. Add the letters Q-U and rearrange the result to get a new word. The Q-U can appear anywhere in the answer. 4 A ROBE + QU BAROQUE 20201108 I'm going to give you some words. Add the letters Q-U and rearrange the result to get a new word. The Q-U can appear anywhere in the answer. 5 BIBLE + QU QUIBBLE 20201108 I'm going to give you some words. Add the letters Q-U and rearrange the result to get a new word. The Q-U can appear anywhere in the answer. 6 REBUS + QU BRUSQUE 20201108 I'm going to give you some words. Add the letters Q-U and rearrange the result to get a new word. The Q-U can appear anywhere in the answer. 7 RECTO + QU CROQUET 20201108 I'm going to give you some words. Add the letters Q-U and rearrange the result to get a new word. The Q-U can appear anywhere in the answer. 8 NNIE + QU QUININE 20201108 I'm going to give you some words. Add the letters Q-U and rearrange the result to get a new word. The Q-U can appear anywhere in the answer. 9 DIALS + QU SQUALID 20201108 I'm going to give you some words. Add the letters Q-U and rearrange the result to get a new word. The Q-U can appear anywhere in the answer. 10 CLEAR + QU LACQUER 20201108 I'm going to give you some words. Add the letters Q-U and rearrange the result to get a new word. The Q-U can appear anywhere in the answer. 11 I RULE + QU LIQUEUR 20080629 Nowadays, the letters IT stand for information technology. But they're also the initials of some other familiar two-word phrases and names. You name them from the clues. 1 two children developed from the same egg identical twins 20080629 Nowadays, the letters IT stand for information technology. But they're also the initials of some other familiar two-word phrases and names. You name them from the clues. 2 the IRS collects income tax 20080629 Nowadays, the letters IT stand for information technology. But they're also the initials of some other familiar two-word phrases and names. You name them from the clues. 3 summer drink, often served with a mint leaf ice tea 20080629 Nowadays, the letters IT stand for information technology. But they're also the initials of some other familiar two-word phrases and names. You name them from the clues. 4 cause of pain at the end of the foot ingrown toenail 20080629 Nowadays, the letters IT stand for information technology. But they're also the initials of some other familiar two-word phrases and names. You name them from the clues. 5 Stanford-Binet is the most famous oneintelligence test IQ test, or 20080629 Nowadays, the letters IT stand for information technology. But they're also the initials of some other familiar two-word phrases and names. You name them from the clues. 6 modern crime that may involve your SocialSecurity number identity theft 20080629 Nowadays, the letters IT stand for information technology. But they're also the initials of some other familiar two-word phrases and names. You name them from the clues. 7 twelve time NBA all star who went on tocoach the Pacers and Knicks Isiah Thomas 20080629 Nowadays, the letters IT stand for information technology. But they're also the initials of some other familiar two-word phrases and names. You name them from the clues. 8 it slants to the rightisosceles triangle italic type, not 20080629 Nowadays, the letters IT stand for information technology. But they're also the initials of some other familiar two-word phrases and names. You name them from the clues. 9 court game often played in winter underartificial light indoor tennis 20080629 Nowadays, the letters IT stand for information technology. But they're also the initials of some other familiar two-word phrases and names. You name them from the clues. 10 "it might end, ""or else""" idle threat 20070304 I'm going to give you two three-letter words. The first word can be found reading in consecutive letters in the name of a U.S. state capital, and the second word can be found reading in consecutive letters of that capital's state. 0 ton, set (New England) Boston, Massachusetts 20070304 I'm going to give you two three-letter words. The first word can be found reading in consecutive letters in the name of a U.S. state capital, and the second word can be found reading in consecutive letters of that capital's state. 1 has, rid (south) Tallahassee, Florida 20070304 I'm going to give you two three-letter words. The first word can be found reading in consecutive letters in the name of a U.S. state capital, and the second word can be found reading in consecutive letters of that capital's state. 2 ram, for (far west) Sacramento, California 20070304 I'm going to give you two three-letter words. The first word can be found reading in consecutive letters in the name of a U.S. state capital, and the second word can be found reading in consecutive letters of that capital's state. 3 sin, chi Lansing, Michigan 20070304 I'm going to give you two three-letter words. The first word can be found reading in consecutive letters in the name of a U.S. state capital, and the second word can be found reading in consecutive letters of that capital's state. 4 inc, bra Lincoln, Nebraska 20070304 I'm going to give you two three-letter words. The first word can be found reading in consecutive letters in the name of a U.S. state capital, and the second word can be found reading in consecutive letters of that capital's state. 5 ale, ego (far west) Salem, Oregon 20070304 I'm going to give you two three-letter words. The first word can be found reading in consecutive letters in the name of a U.S. state capital, and the second word can be found reading in consecutive letters of that capital's state. 6 err, out (upper plains)Dakota Pierre (pronounced PEER), South 20070304 I'm going to give you two three-letter words. The first word can be found reading in consecutive letters in the name of a U.S. state capital, and the second word can be found reading in consecutive letters of that capital's state. 7 nap, ian Indianapolis, Indiana 20070304 I'm going to give you two three-letter words. The first word can be found reading in consecutive letters in the name of a U.S. state capital, and the second word can be found reading in consecutive letters of that capital's state. 8 len, tan (mountain states) Helena, Montana 20070304 I'm going to give you two three-letter words. The first word can be found reading in consecutive letters in the name of a U.S. state capital, and the second word can be found reading in consecutive letters of that capital's state. 9 bur, van (easternmid-Atlantic state) Harrisburg, Pennsylvania 20071104 "In the on-air puzzle, every answer is a familiar two-word phrase, or name, with the initials ""DH.""" 0 unlikely candidate to win a race dark horse 20071104 "In the on-air puzzle, every answer is a familiar two-word phrase, or name, with the initials ""DH.""" 1 absolute tie dead heat 20071104 "In the on-air puzzle, every answer is a familiar two-word phrase, or name, with the initials ""DH.""" 2 noted beach on Oahu Diamond Head 20071104 "In the on-air puzzle, every answer is a familiar two-word phrase, or name, with the initials ""DH.""" 3 Bruce Willis film with three sequels Die Hard 20071104 "In the on-air puzzle, every answer is a familiar two-word phrase, or name, with the initials ""DH.""" 4 "Henrik Ibsen play preceded by ""A""" Doll's House 20071104 "In the on-air puzzle, every answer is a familiar two-word phrase, or name, with the initials ""DH.""" 5 a person who has guests over for a meal dinner host 20071104 "In the on-air puzzle, every answer is a familiar two-word phrase, or name, with the initials ""DH.""" 6 something you turn to enter a room door handle 20071104 "In the on-air puzzle, every answer is a familiar two-word phrase, or name, with the initials ""DH.""" 7 DNA's shape double helix 20071104 "In the on-air puzzle, every answer is a familiar two-word phrase, or name, with the initials ""DH.""" 8 it breathes fire in a Chinese New Year parade dragon's head 20071104 "In the on-air puzzle, every answer is a familiar two-word phrase, or name, with the initials ""DH.""" 9 "film character who said, ""Go ahead, make myday.""" Dirty Harry 20071104 "In the on-air puzzle, every answer is a familiar two-word phrase, or name, with the initials ""DH.""" 10 a place to do a waltz or a cha-cha dance hall 20071104 "In the on-air puzzle, every answer is a familiar two-word phrase, or name, with the initials ""DH.""" 11 person working on a ship deck hand 20071104 "In the on-air puzzle, every answer is a familiar two-word phrase, or name, with the initials ""DH.""" 12 it's important for maintaining healthy teeth dental hygiene 20071104 "In the on-air puzzle, every answer is a familiar two-word phrase, or name, with the initials ""DH.""" 13 an interstate, for example, with a median divided highway 20071104 "In the on-air puzzle, every answer is a familiar two-word phrase, or name, with the initials ""DH.""" 14 "Robert De Niro film set in Vietnam, with thearticle ""The.""" Deer Hunter 20071104 "In the on-air puzzle, every answer is a familiar two-word phrase, or name, with the initials ""DH.""" 15 French for 18 dix-huit 20071104 "In the on-air puzzle, every answer is a familiar two-word phrase, or name, with the initials ""DH.""" 17 wit that's unlikely to make you laugh dry humor 20071104 "In the on-air puzzle, every answer is a familiar two-word phrase, or name, with the initials ""DH.""" 18 opening at a bakery? (a little tricky) donut hole 20070715 I'm going to give you clues for two words. The first word starts with the letters T, H. Change the T-H to S-H and phonetically, you'll get a new word that answers the second clue. 0 robber; a group of papers thief, sheaf 20070715 I'm going to give you clues for two words. The first word starts with the letters T, H. Change the T-H to S-H and phonetically, you'll get a new word that answers the second clue. 1 to express appreciation to; partof the leg thank,shank 20070715 I'm going to give you clues for two words. The first word starts with the letters T, H. Change the T-H to S-H and phonetically, you'll get a new word that answers the second clue. 2 part of the leg; retiring thigh, shy 20070715 I'm going to give you clues for two words. The first word starts with the letters T, H. Change the T-H to S-H and phonetically, you'll get a new word that answers the second clue. 3 a Quaker possessive; to twinkle thine, shine 20070715 I'm going to give you clues for two words. The first word starts with the letters T, H. Change the T-H to S-H and phonetically, you'll get a new word that answers the second clue. 4 McBeth, for example; classic1950's western Thane, Shane 20070715 I'm going to give you clues for two words. The first word starts with the letters T, H. Change the T-H to S-H and phonetically, you'll get a new word that answers the second clue. 5 part of a rose; like a sheepwithout its wool thorn, shorn 20070715 I'm going to give you clues for two words. The first word starts with the letters T, H. Change the T-H to S-H and phonetically, you'll get a new word that answers the second clue. 6 kind of street; a nag through, shrew 20070715 I'm going to give you clues for two words. The first word starts with the letters T, H. Change the T-H to S-H and phonetically, you'll get a new word that answers the second clue. 7 a baseball position, informally;prepared eggs, in a way third,shirred 20070715 I'm going to give you clues for two words. The first word starts with the letters T, H. Change the T-H to S-H and phonetically, you'll get a new word that answers the second clue. 8 like screws; kind of wheatshredded threaded, 20070715 I'm going to give you clues for two words. The first word starts with the letters T, H. Change the T-H to S-H and phonetically, you'll get a new word that answers the second clue. 9 to send; loud and high pitched thrill, shrill 20070715 I'm going to give you clues for two words. The first word starts with the letters T, H. Change the T-H to S-H and phonetically, you'll get a new word that answers the second clue. 10 actress in Pulp Fiction; a Uniongeneral in the Civil War Thurman, Sherman 20070715 I'm going to give you clues for two words. The first word starts with the letters T, H. Change the T-H to S-H and phonetically, you'll get a new word that answers the second clue. 11 person or thing doing very wellin its environment; McGovern'srunning mate in 1972 thriver, Shriver 20070715 I'm going to give you clues for two words. The first word starts with the letters T, H. Change the T-H to S-H and phonetically, you'll get a new word that answers the second clue. 12 economy; it may be short thrift, shrift 20070715 I'm going to give you clues for two words. The first word starts with the letters T, H. Change the T-H to S-H and phonetically, you'll get a new word that answers the second clue. 13 a noted choreographer (Twila,Liane added); quick witted Tharp, sharp 20090111 "Every answer consists of two letters of the alphabet that sound like a word or name. Fill in the blanks to get them. For example, if the clue is ___ watermelon, then the answer would be, """"C-D"""" as in seedy watermelon." 0 poison _____ IV, ivy 20090111 "Every answer consists of two letters of the alphabet that sound like a word or name. Fill in the blanks to get them. For example, if the clue is ___ watermelon, then the answer would be, """"C-D"""" as in seedy watermelon." 2 Indian _____ TP, tipi 20090111 "Every answer consists of two letters of the alphabet that sound like a word or name. Fill in the blanks to get them. For example, if the clue is ___ watermelon, then the answer would be, """"C-D"""" as in seedy watermelon." 3 _____ listening EZ, easy 20090111 "Every answer consists of two letters of the alphabet that sound like a word or name. Fill in the blanks to get them. For example, if the clue is ___ watermelon, then the answer would be, """"C-D"""" as in seedy watermelon." 4 _____ doll QP, Kewpie, not Barbie 20090111 "Every answer consists of two letters of the alphabet that sound like a word or name. Fill in the blanks to get them. For example, if the clue is ___ watermelon, then the answer would be, """"C-D"""" as in seedy watermelon." 5 _____ calories (junk food) MT, empty 20090111 "Every answer consists of two letters of the alphabet that sound like a word or name. Fill in the blanks to get them. For example, if the clue is ___ watermelon, then the answer would be, """"C-D"""" as in seedy watermelon." 6 _____ Degeneres LN, Ellen 20090111 "Every answer consists of two letters of the alphabet that sound like a word or name. Fill in the blanks to get them. For example, if the clue is ___ watermelon, then the answer would be, """"C-D"""" as in seedy watermelon." 7 baked _____ (Italian food) ZT, ziti 20090111 "Every answer consists of two letters of the alphabet that sound like a word or name. Fill in the blanks to get them. For example, if the clue is ___ watermelon, then the answer would be, """"C-D"""" as in seedy watermelon." 8 _____ Blake UB, Eubie 20090111 "Every answer consists of two letters of the alphabet that sound like a word or name. Fill in the blanks to get them. For example, if the clue is ___ watermelon, then the answer would be, """"C-D"""" as in seedy watermelon." 9 19__ (a year) AT, eighty (80) 20090111 "Every answer consists of two letters of the alphabet that sound like a word or name. Fill in the blanks to get them. For example, if the clue is ___ watermelon, then the answer would be, """"C-D"""" as in seedy watermelon." 10 daytime _____ (an award) ME, Emmy 20090111 "Every answer consists of two letters of the alphabet that sound like a word or name. Fill in the blanks to get them. For example, if the clue is ___ watermelon, then the answer would be, """"C-D"""" as in seedy watermelon." 11 _____ pie (term of endearment) QT, cutie 20070902 "In this week's on-air puzzle, every answer is the first and last name of a famous person. Will gives a two-word phrase; change one letter in each word to name the person. Here's a hint: The letter you change to is the same in the first and last names. For example, for Seal Pens the answer would be Sean Penn; each letter is changed to an """"N.""""" 0 Salty Fiend Sally Field 20070902 "In this week's on-air puzzle, every answer is the first and last name of a famous person. Will gives a two-word phrase; change one letter in each word to name the person. Here's a hint: The letter you change to is the same in the first and last names. For example, for Seal Pens the answer would be Sean Penn; each letter is changed to an """"N.""""" 1 Party Hearse Patty Hearst 20070902 "In this week's on-air puzzle, every answer is the first and last name of a famous person. Will gives a two-word phrase; change one letter in each word to name the person. Here's a hint: The letter you change to is the same in the first and last names. For example, for Seal Pens the answer would be Sean Penn; each letter is changed to an """"N.""""" 2 Mix Furrow Mia Farrow 20070902 "In this week's on-air puzzle, every answer is the first and last name of a famous person. Will gives a two-word phrase; change one letter in each word to name the person. Here's a hint: The letter you change to is the same in the first and last names. For example, for Seal Pens the answer would be Sean Penn; each letter is changed to an """"N.""""" 3 Any Ladders Ann Landers 20070902 "In this week's on-air puzzle, every answer is the first and last name of a famous person. Will gives a two-word phrase; change one letter in each word to name the person. Here's a hint: The letter you change to is the same in the first and last names. For example, for Seal Pens the answer would be Sean Penn; each letter is changed to an """"N.""""" 4 Best Packs Pert Parks 20070902 "In this week's on-air puzzle, every answer is the first and last name of a famous person. Will gives a two-word phrase; change one letter in each word to name the person. Here's a hint: The letter you change to is the same in the first and last names. For example, for Seal Pens the answer would be Sean Penn; each letter is changed to an """"N.""""" 5 Carry Ponds Barry Bonds 20070902 "In this week's on-air puzzle, every answer is the first and last name of a famous person. Will gives a two-word phrase; change one letter in each word to name the person. Here's a hint: The letter you change to is the same in the first and last names. For example, for Seal Pens the answer would be Sean Penn; each letter is changed to an """"N.""""" 6 Hunk Apron Hank Aaron 20070902 "In this week's on-air puzzle, every answer is the first and last name of a famous person. Will gives a two-word phrase; change one letter in each word to name the person. Here's a hint: The letter you change to is the same in the first and last names. For example, for Seal Pens the answer would be Sean Penn; each letter is changed to an """"N.""""" 7 Red Burner Ted Turner 20070902 "In this week's on-air puzzle, every answer is the first and last name of a famous person. Will gives a two-word phrase; change one letter in each word to name the person. Here's a hint: The letter you change to is the same in the first and last names. For example, for Seal Pens the answer would be Sean Penn; each letter is changed to an """"N.""""" 8 Pierce Cutie Pierre Curie 20070902 "In this week's on-air puzzle, every answer is the first and last name of a famous person. Will gives a two-word phrase; change one letter in each word to name the person. Here's a hint: The letter you change to is the same in the first and last names. For example, for Seal Pens the answer would be Sean Penn; each letter is changed to an """"N.""""" 9 March Poll Marco Polo 20070902 "In this week's on-air puzzle, every answer is the first and last name of a famous person. Will gives a two-word phrase; change one letter in each word to name the person. Here's a hint: The letter you change to is the same in the first and last names. For example, for Seal Pens the answer would be Sean Penn; each letter is changed to an """"N.""""" 10 Meter Wiper Peter Piper 20100411 Every answer is a six-letter word, name or phrase in which the first five letters read backward and forward the same. In other words, if you drop the last letter, what's left will be a palindrome. 0 place to park a car garage 20100411 Every answer is a six-letter word, name or phrase in which the first five letters read backward and forward the same. In other words, if you drop the last letter, what's left will be a palindrome. 1 title for a married French lady Madame 20100411 Every answer is a six-letter word, name or phrase in which the first five letters read backward and forward the same. In other words, if you drop the last letter, what's left will be a palindrome. 2 university name in Chicago and New Orleans Loyola 20100411 Every answer is a six-letter word, name or phrase in which the first five letters read backward and forward the same. In other words, if you drop the last letter, what's left will be a palindrome. 3 high school course in government civics 20100411 Every answer is a six-letter word, name or phrase in which the first five letters read backward and forward the same. In other words, if you drop the last letter, what's left will be a palindrome. 4 maker of the Corolla Toyota 20100411 Every answer is a six-letter word, name or phrase in which the first five letters read backward and forward the same. In other words, if you drop the last letter, what's left will be a palindrome. 5 brand of athletic shoe Adidas 20100411 Every answer is a six-letter word, name or phrase in which the first five letters read backward and forward the same. In other words, if you drop the last letter, what's left will be a palindrome. 6 mount where Noah landed Ararat 20100411 Every answer is a six-letter word, name or phrase in which the first five letters read backward and forward the same. In other words, if you drop the last letter, what's left will be a palindrome. 7 exhausts, or depletes (2 word answer) uses up 20080330 "In this week's puzzle, every answer is a familiar three-word phrase in the form of """"______ of ______."""" The first word starts with M, and you are given the third word. For example, given """"production"""" the answer would be """"means"""" for """"means of production."""" " 0 magnesia milk 20080330 "In this week's puzzle, every answer is a familiar three-word phrase in the form of """"______ of ______."""" The first word starts with M, and you are given the third word. For example, given """"production"""" the answer would be """"means"""" for """"means of production."""" " 1 dimes march 20080330 "In this week's puzzle, every answer is a familiar three-word phrase in the form of """"______ of ______."""" The first word starts with M, and you are given the third word. For example, given """"production"""" the answer would be """"means"""" for """"means of production."""" " 2 Zorro mark 20080330 "In this week's puzzle, every answer is a familiar three-word phrase in the form of """"______ of ______."""" The first word starts with M, and you are given the third word. For example, given """"production"""" the answer would be """"means"""" for """"means of production."""" " 3 steel made or man 20080330 "In this week's puzzle, every answer is a familiar three-word phrase in the form of """"______ of ______."""" The first word starts with M, and you are given the third word. For example, given """"production"""" the answer would be """"means"""" for """"means of production."""" " 4 truth moment 20080330 "In this week's puzzle, every answer is a familiar three-word phrase in the form of """"______ of ______."""" The first word starts with M, and you are given the third word. For example, given """"production"""" the answer would be """"means"""" for """"means of production."""" " 5 parliament member 20080330 "In this week's puzzle, every answer is a familiar three-word phrase in the form of """"______ of ______."""" The first word starts with M, and you are given the third word. For example, given """"production"""" the answer would be """"means"""" for """"means of production."""" " 6 pearl mother 20080330 "In this week's puzzle, every answer is a familiar three-word phrase in the form of """"______ of ______."""" The first word starts with M, and you are given the third word. For example, given """"production"""" the answer would be """"means"""" for """"means of production."""" " 7 ceremonies master 20080330 "In this week's puzzle, every answer is a familiar three-word phrase in the form of """"______ of ______."""" The first word starts with M, and you are given the third word. For example, given """"production"""" the answer would be """"means"""" for """"means of production."""" " 8 fact matter 20080330 "In this week's puzzle, every answer is a familiar three-word phrase in the form of """"______ of ______."""" The first word starts with M, and you are given the third word. For example, given """"production"""" the answer would be """"means"""" for """"means of production."""" " 9 life meaning 20090222 "You are given some quotes with the ending, """"Tom ________."""" The word that goes in the blank is a synonym for """"said"""" that completes the statement in a punny way. For example, given, """"'That was the best all-night dance party ever,' Tom ________,"""" the answer would be """"raved.""""" 1 That's too much whale fat, Tom _____. blubbered 20090222 "You are given some quotes with the ending, """"Tom ________."""" The word that goes in the blank is a synonym for """"said"""" that completes the statement in a punny way. For example, given, """"'That was the best all-night dance party ever,' Tom ________,"""" the answer would be """"raved.""""" 2 You wrecked my Swedish car, Tom _____.on SAAB) sobbed (pun 20090222 "You are given some quotes with the ending, """"Tom ________."""" The word that goes in the blank is a synonym for """"said"""" that completes the statement in a punny way. For example, given, """"'That was the best all-night dance party ever,' Tom ________,"""" the answer would be """"raved.""""" 3 Well I would say a breakfast dish made on an ironis my very favorite, Tom _____. waffled 20090222 "You are given some quotes with the ending, """"Tom ________."""" The word that goes in the blank is a synonym for """"said"""" that completes the statement in a punny way. For example, given, """"'That was the best all-night dance party ever,' Tom ________,"""" the answer would be """"raved.""""" 4 The best barrier between the yards wouldn't haveto be a fence, Tom _____. hedged 20090222 "You are given some quotes with the ending, """"Tom ________."""" The word that goes in the blank is a synonym for """"said"""" that completes the statement in a punny way. For example, given, """"'That was the best all-night dance party ever,' Tom ________,"""" the answer would be """"raved.""""" 5 For the thousandth time, quit betting on the race nagged, not 20090222 "You are given some quotes with the ending, """"Tom ________."""" The word that goes in the blank is a synonym for """"said"""" that completes the statement in a punny way. For example, given, """"'That was the best all-night dance party ever,' Tom ________,"""" the answer would be """"raved.""""" 6 horses, Tom _____. (starting with N) neighed 20090222 "You are given some quotes with the ending, """"Tom ________."""" The word that goes in the blank is a synonym for """"said"""" that completes the statement in a punny way. For example, given, """"'That was the best all-night dance party ever,' Tom ________,"""" the answer would be """"raved.""""" 7 Don't you hate plumbers who bend over with theirpants too low, Tom _____. (starting with C) cracked 20090222 "You are given some quotes with the ending, """"Tom ________."""" The word that goes in the blank is a synonym for """"said"""" that completes the statement in a punny way. For example, given, """"'That was the best all-night dance party ever,' Tom ________,"""" the answer would be """"raved.""""" 8 Somehow, you will have to get the water from the piped up, not 20090222 "You are given some quotes with the ending, """"Tom ________."""" The word that goes in the blank is a synonym for """"said"""" that completes the statement in a punny way. For example, given, """"'That was the best all-night dance party ever,' Tom ________,"""" the answer would be """"raved.""""" 9 basement to the top floors, Tom _____. (two wordanswers with initials P and U) pumped up 20090222 "You are given some quotes with the ending, """"Tom ________."""" The word that goes in the blank is a synonym for """"said"""" that completes the statement in a punny way. For example, given, """"'That was the best all-night dance party ever,' Tom ________,"""" the answer would be """"raved.""""" 10 This ellipsis has only two dots, Tom _____. (twowords, with initials P and O) pointed out 20070826 "This week's on-air puzzle goes to ""excess."" Every answer is a familiar two-word phrase, in which the first word ends in ""X"" and the second word starts with ""S.""" 1 Marilyn Monroe or Pamela Anderson sex symbol 20070826 "This week's on-air puzzle goes to ""excess."" Every answer is a familiar two-word phrase, in which the first word ends in ""X"" and the second word starts with ""S.""" 2 where rich people protect their money from theIRS tax shelter 20070826 "This week's on-air puzzle goes to ""excess."" Every answer is a familiar two-word phrase, in which the first word ends in ""X"" and the second word starts with ""S.""" 3 NBA team from Arizona Phoenix Suns 20070826 "This week's on-air puzzle goes to ""excess."" Every answer is a familiar two-word phrase, in which the first word ends in ""X"" and the second word starts with ""S.""" 4 revolver with half a dozen chambers six shooter 20070826 "This week's on-air puzzle goes to ""excess."" Every answer is a familiar two-word phrase, in which the first word ends in ""X"" and the second word starts with ""S.""" 5 items sent by mail in a 2001 post office scare anthrax spores 20070826 "This week's on-air puzzle goes to ""excess."" Every answer is a familiar two-word phrase, in which the first word ends in ""X"" and the second word starts with ""S.""" 6 French for 17 dix-sept 20070826 "This week's on-air puzzle goes to ""excess."" Every answer is a familiar two-word phrase, in which the first word ends in ""X"" and the second word starts with ""S.""" 7 one who transmits a document by telephone fax sender 20081130 "For each word given, add a letter at the start, the end or somewhere inside to name a well-known U.S. city. For example, given the word """"flit,"""" the answer would be """"Flint.""""" 0 caper Casper, Wyoming 20081130 "For each word given, add a letter at the start, the end or somewhere inside to name a well-known U.S. city. For example, given the word """"flit,"""" the answer would be """"Flint.""""" 1 dove Dover, Delaware (or Ohio) 20081130 "For each word given, add a letter at the start, the end or somewhere inside to name a well-known U.S. city. For example, given the word """"flit,"""" the answer would be """"Flint.""""" 2 Mimi Miami, Florida 20081130 "For each word given, add a letter at the start, the end or somewhere inside to name a well-known U.S. city. For example, given the word """"flit,"""" the answer would be """"Flint.""""" 3 august Augusta, Maine 20081130 "For each word given, add a letter at the start, the end or somewhere inside to name a well-known U.S. city. For example, given the word """"flit,"""" the answer would be """"Flint.""""" 4 sale Salem, Massachusetts (or Oregon) 20081130 "For each word given, add a letter at the start, the end or somewhere inside to name a well-known U.S. city. For example, given the word """"flit,"""" the answer would be """"Flint.""""" 5 butt Butte, Montana 20081130 "For each word given, add a letter at the start, the end or somewhere inside to name a well-known U.S. city. For example, given the word """"flit,"""" the answer would be """"Flint.""""" 6 Poland (add two letters intothe same place from now on) Portland, Maine (or Oregon) 20081130 "For each word given, add a letter at the start, the end or somewhere inside to name a well-known U.S. city. For example, given the word """"flit,"""" the answer would be """"Flint.""""" 7 June Juneau, Alaska 20081130 "For each word given, add a letter at the start, the end or somewhere inside to name a well-known U.S. city. For example, given the word """"flit,"""" the answer would be """"Flint.""""" 8 spoke Spokane, Washington 20081130 "For each word given, add a letter at the start, the end or somewhere inside to name a well-known U.S. city. For example, given the word """"flit,"""" the answer would be """"Flint.""""" 9 boon Boston, Massachusetts 20081130 "For each word given, add a letter at the start, the end or somewhere inside to name a well-known U.S. city. For example, given the word """"flit,"""" the answer would be """"Flint.""""" 10 Saul St. Paul, Minnesota 20081130 "For each word given, add a letter at the start, the end or somewhere inside to name a well-known U.S. city. For example, given the word """"flit,"""" the answer would be """"Flint.""""" 11 province Providence, Rhode Island 20080824 "Each word you're given has the letters C-H within it. Rearrange the letters to come up with an anagram that begins with C-H. For example, """"inch"""" becomes """"chin.""""" 0 much chum 20080824 "Each word you're given has the letters C-H within it. Rearrange the letters to come up with an anagram that begins with C-H. For example, """"inch"""" becomes """"chin.""""" 2 peach cheap 20080824 "Each word you're given has the letters C-H within it. Rearrange the letters to come up with an anagram that begins with C-H. For example, """"inch"""" becomes """"chin.""""" 3 aches chase 20080824 "Each word you're given has the letters C-H within it. Rearrange the letters to come up with an anagram that begins with C-H. For example, """"inch"""" becomes """"chin.""""" 4 march charm 20080824 "Each word you're given has the letters C-H within it. Rearrange the letters to come up with an anagram that begins with C-H. For example, """"inch"""" becomes """"chin.""""" 5 echoic choice 20080824 "Each word you're given has the letters C-H within it. Rearrange the letters to come up with an anagram that begins with C-H. For example, """"inch"""" becomes """"chin.""""" 6 ratchet chatter 20080824 "Each word you're given has the letters C-H within it. Rearrange the letters to come up with an anagram that begins with C-H. For example, """"inch"""" becomes """"chin.""""" 7 arching chagrin 20080824 "Each word you're given has the letters C-H within it. Rearrange the letters to come up with an anagram that begins with C-H. For example, """"inch"""" becomes """"chin.""""" 8 snitcher christen 20080824 "Each word you're given has the letters C-H within it. Rearrange the letters to come up with an anagram that begins with C-H. For example, """"inch"""" becomes """"chin.""""" 9 archaism charisma 20080608 In this week's on-air challenge, every answer is a word or familiar phrase of no more than 12 letters. The word contains at least four T's. 1 snitch tattle tale 20080608 In this week's on-air challenge, every answer is a word or familiar phrase of no more than 12 letters. The word contains at least four T's. 2 Coke vs. Pepsi competition, for example taste test 20080608 In this week's on-air challenge, every answer is a word or familiar phrase of no more than 12 letters. The word contains at least four T's. 3 Oscar award statuette 20080608 In this week's on-air challenge, every answer is a word or familiar phrase of no more than 12 letters. The word contains at least four T's. 4 stuff put in the bottom of a cat box toabsorb odors kitty litter 20080608 In this week's on-air challenge, every answer is a word or familiar phrase of no more than 12 letters. The word contains at least four T's. 5 ice cream containing many kinds of fruit tutti frutti 20080608 In this week's on-air challenge, every answer is a word or familiar phrase of no more than 12 letters. The word contains at least four T's. 6 one of the native people of Namibia notedfor their fierceness Hottentot 20080608 In this week's on-air challenge, every answer is a word or familiar phrase of no more than 12 letters. The word contains at least four T's. 7 ready for action, as on a date hot to trot 20080608 In this week's on-air challenge, every answer is a word or familiar phrase of no more than 12 letters. The word contains at least four T's. 8 one-named Venetian painter of the ItalianRenaissance Tintoretto 20080608 In this week's on-air challenge, every answer is a word or familiar phrase of no more than 12 letters. The word contains at least four T's. 9 image once seen on TV after a station wentoff the air test pattern 20080608 In this week's on-air challenge, every answer is a word or familiar phrase of no more than 12 letters. The word contains at least four T's. 10 perfect vision twenty twenty 20080608 In this week's on-air challenge, every answer is a word or familiar phrase of no more than 12 letters. The word contains at least four T's. 11 see-saw teeter totter 20081019 "You are given a word. Drop two letters so that the remaining letters, in order, spell the name of a world capital. For example, given """"backup,"""" the answer would be """"Baku,"""" the capital of Azerbaijan." 0 parties Paris, France 20081019 "You are given a word. Drop two letters so that the remaining letters, in order, spell the name of a world capital. For example, given """"backup,"""" the answer would be """"Baku,"""" the capital of Azerbaijan." 1 Clairol Cairo, Egypt 20081019 "You are given a word. Drop two letters so that the remaining letters, in order, spell the name of a world capital. For example, given """"backup,"""" the answer would be """"Baku,"""" the capital of Azerbaijan." 2 Delphic Delhi, India 20081019 "You are given a word. Drop two letters so that the remaining letters, in order, spell the name of a world capital. For example, given """"backup,"""" the answer would be """"Baku,"""" the capital of Azerbaijan." 3 heathens Athens, Greece 20081019 "You are given a word. Drop two letters so that the remaining letters, in order, spell the name of a world capital. For example, given """"backup,"""" the answer would be """"Baku,"""" the capital of Azerbaijan." 4 accrual Accra, Ghana 20081019 "You are given a word. Drop two letters so that the remaining letters, in order, spell the name of a world capital. For example, given """"backup,"""" the answer would be """"Baku,"""" the capital of Azerbaijan." 5 Tammany Amman, Jordan 20081019 "You are given a word. Drop two letters so that the remaining letters, in order, spell the name of a world capital. For example, given """"backup,"""" the answer would be """"Baku,"""" the capital of Azerbaijan." 6 mantilla Manila, Philippines 20070325 This week's on air puzzle involves two clues that will yield a solution using a seven-letter word and the middle five letters of that very same word. 0 an industrial plant; a person in a play factory, actor 20070325 This week's on air puzzle involves two clues that will yield a solution using a seven-letter word and the middle five letters of that very same word. 1 a saying; a vagabond proverb, rover 20070325 This week's on air puzzle involves two clues that will yield a solution using a seven-letter word and the middle five letters of that very same word. 2 to supervise; poem oversee, verse 20070325 This week's on air puzzle involves two clues that will yield a solution using a seven-letter word and the middle five letters of that very same word. 3 a grand speech; proportion oration, ratio 20070325 This week's on air puzzle involves two clues that will yield a solution using a seven-letter word and the middle five letters of that very same word. 4 gas that may build up in a mine; afamous Allen methane, Ethan 20070325 This week's on air puzzle involves two clues that will yield a solution using a seven-letter word and the middle five letters of that very same word. 5 neighbored; a city in Montana abutted, Butte 20070325 This week's on air puzzle involves two clues that will yield a solution using a seven-letter word and the middle five letters of that very same word. 6 a Greek letter (after xi, a Greek O);very very small omicron, micro 20070325 This week's on air puzzle involves two clues that will yield a solution using a seven-letter word and the middle five letters of that very same word. 7 exercised too much; composer of Latraviata overdid, Verdi 20070325 This week's on air puzzle involves two clues that will yield a solution using a seven-letter word and the middle five letters of that very same word. 8 like 1.3 billion of the world's people;dancer Gregory Chinese, Hines 20070325 This week's on air puzzle involves two clues that will yield a solution using a seven-letter word and the middle five letters of that very same word. 9 to overshadow; office fasteners eclipse, clips 20070325 This week's on air puzzle involves two clues that will yield a solution using a seven-letter word and the middle five letters of that very same word. 10 severe punishment; like some kitchens beating, eat in 20070325 This week's on air puzzle involves two clues that will yield a solution using a seven-letter word and the middle five letters of that very same word. 11 teach; former gold coin of Europe educate, ducat 20070325 This week's on air puzzle involves two clues that will yield a solution using a seven-letter word and the middle five letters of that very same word. 12 to desert; one of the railroads inMonopoly abandon, B and O 20071021 "In this week's on-air puzzle, every answer is a word starting with the letter V. Use an anagram of the clue to find the answer. For example, if the clue is """"serve,"""" the answer would be """"verse.""""" 0 rival viral 20071021 "In this week's on-air puzzle, every answer is a word starting with the letter V. Use an anagram of the clue to find the answer. For example, if the clue is """"serve,"""" the answer would be """"verse.""""" 1 lovely volley 20071021 "In this week's on-air puzzle, every answer is a word starting with the letter V. Use an anagram of the clue to find the answer. For example, if the clue is """"serve,"""" the answer would be """"verse.""""" 2 caveat vacate 20071021 "In this week's on-air puzzle, every answer is a word starting with the letter V. Use an anagram of the clue to find the answer. For example, if the clue is """"serve,"""" the answer would be """"verse.""""" 3 devote vetoed 20071021 "In this week's on-air puzzle, every answer is a word starting with the letter V. Use an anagram of the clue to find the answer. For example, if the clue is """"serve,"""" the answer would be """"verse.""""" 4 Irving (airline or islands) Virgin 20071021 "In this week's on-air puzzle, every answer is a word starting with the letter V. Use an anagram of the clue to find the answer. For example, if the clue is """"serve,"""" the answer would be """"verse.""""" 5 livery (biblical word) verily 20071021 "In this week's on-air puzzle, every answer is a word starting with the letter V. Use an anagram of the clue to find the answer. For example, if the clue is """"serve,"""" the answer would be """"verse.""""" 6 selves (general word for ship) vessel 20071021 "In this week's on-air puzzle, every answer is a word starting with the letter V. Use an anagram of the clue to find the answer. For example, if the clue is """"serve,"""" the answer would be """"verse.""""" 7 Latvian (adjective that describes a knight or HalFoster's Prince) valiant 20071021 "In this week's on-air puzzle, every answer is a word starting with the letter V. Use an anagram of the clue to find the answer. For example, if the clue is """"serve,"""" the answer would be """"verse.""""" 8 saviour various 20071021 "In this week's on-air puzzle, every answer is a word starting with the letter V. Use an anagram of the clue to find the answer. For example, if the clue is """"serve,"""" the answer would be """"verse.""""" 9 observe verbose 20071021 "In this week's on-air puzzle, every answer is a word starting with the letter V. Use an anagram of the clue to find the answer. For example, if the clue is """"serve,"""" the answer would be """"verse.""""" 10 Octavian (something you might go on in the Summer) vacation 20071021 "In this week's on-air puzzle, every answer is a word starting with the letter V. Use an anagram of the clue to find the answer. For example, if the clue is """"serve,"""" the answer would be """"verse.""""" 11 dovetail (past tense of a verb; broke, as a law) violated 20071021 "In this week's on-air puzzle, every answer is a word starting with the letter V. Use an anagram of the clue to find the answer. For example, if the clue is """"serve,"""" the answer would be """"verse.""""" 12 Levantine Valentine 20071125 "Every answer involves reduplicative words. An example is ping pong, whereby the first half is the same as the second half, only the vowel changes. So if the clue is ""table tennis,"" then the answer would be ""ping pong.”" 0 sound of a doorbell ding dong 20071125 "Every answer involves reduplicative words. An example is ping pong, whereby the first half is the same as the second half, only the vowel changes. So if the clue is ""table tennis,"" then the answer would be ""ping pong.”" 1 sound of a clock tick tock 20071125 "Every answer involves reduplicative words. An example is ping pong, whereby the first half is the same as the second half, only the vowel changes. So if the clue is ""table tennis,"" then the answer would be ""ping pong.”" 2 completely reverse one's politicalposition flip flop 20071125 "Every answer involves reduplicative words. An example is ping pong, whereby the first half is the same as the second half, only the vowel changes. So if the clue is ""table tennis,"" then the answer would be ""ping pong.”" 3 rap music hip hop 20071125 "Every answer involves reduplicative words. An example is ping pong, whereby the first half is the same as the second half, only the vowel changes. So if the clue is ""table tennis,"" then the answer would be ""ping pong.”" 4 a movie ape King Kong 20071125 "Every answer involves reduplicative words. An example is ping pong, whereby the first half is the same as the second half, only the vowel changes. So if the clue is ""table tennis,"" then the answer would be ""ping pong.”" 5 voice with repetitive rhythm sing song 20071125 "Every answer involves reduplicative words. An example is ping pong, whereby the first half is the same as the second half, only the vowel changes. So if the clue is ""table tennis,"" then the answer would be ""ping pong.”" 6 to traverse again and again criss-cross 20071125 "Every answer involves reduplicative words. An example is ping pong, whereby the first half is the same as the second half, only the vowel changes. So if the clue is ""table tennis,"" then the answer would be ""ping pong.”" 7 sound of horse hooves clip clop 20071125 "Every answer involves reduplicative words. An example is ping pong, whereby the first half is the same as the second half, only the vowel changes. So if the clue is ""table tennis,"" then the answer would be ""ping pong.”" 8 apex or peak tip top 20071125 "Every answer involves reduplicative words. An example is ping pong, whereby the first half is the same as the second half, only the vowel changes. So if the clue is ""table tennis,"" then the answer would be ""ping pong.”" 9 to make sharp turns, left and right(I and A vowel changes from now on) zig zag 20071125 "Every answer involves reduplicative words. An example is ping pong, whereby the first half is the same as the second half, only the vowel changes. So if the clue is ""table tennis,"" then the answer would be ""ping pong.”" 10 to talk, informally chit chat 20071125 "Every answer involves reduplicative words. An example is ping pong, whereby the first half is the same as the second half, only the vowel changes. So if the clue is ""table tennis,"" then the answer would be ""ping pong.”" 11 disreputable persons riff raff, not flim flams 20071125 "Every answer involves reduplicative words. An example is ping pong, whereby the first half is the same as the second half, only the vowel changes. So if the clue is ""table tennis,"" then the answer would be ""ping pong.”" 12 deception or fraud flim flam 20071125 "Every answer involves reduplicative words. An example is ping pong, whereby the first half is the same as the second half, only the vowel changes. So if the clue is ""table tennis,"" then the answer would be ""ping pong.”" 13 tchotchke knick knack 20071125 "Every answer involves reduplicative words. An example is ping pong, whereby the first half is the same as the second half, only the vowel changes. So if the clue is ""table tennis,"" then the answer would be ""ping pong.”" 14 Hershey's chocolate bar with a waferinside Kit Kat, not Tic Tac 20071125 "Every answer involves reduplicative words. An example is ping pong, whereby the first half is the same as the second half, only the vowel changes. So if the clue is ""table tennis,"" then the answer would be ""ping pong.”" 15 rap duo, _____ Twins Ying Yang 20071125 "Every answer involves reduplicative words. An example is ping pong, whereby the first half is the same as the second half, only the vowel changes. So if the clue is ""table tennis,"" then the answer would be ""ping pong.”" 16 to waste time dilly dally 20071125 "Every answer involves reduplicative words. An example is ping pong, whereby the first half is the same as the second half, only the vowel changes. So if the clue is ""table tennis,"" then the answer would be ""ping pong.”" 17 sound of rain, or little feet pitter patter 20071125 "Every answer involves reduplicative words. An example is ping pong, whereby the first half is the same as the second half, only the vowel changes. So if the clue is ""table tennis,"" then the answer would be ""ping pong.”" 18 unable to take a firm position wishy washy 20080720 "In this week's on-air puzzle, you are given two words. Move a letter from one word to the other to make two new words that complete the familiar phrase, """"___________ and ___________."""" For example, given """"liven"""" and """"Lear,"""" the answer would be """"live and learn."""" The letter that's moved can come from either word. " 0 wear, pace war and peace 20080720 "In this week's on-air puzzle, you are given two words. Move a letter from one word to the other to make two new words that complete the familiar phrase, """"___________ and ___________."""" For example, given """"liven"""" and """"Lear,"""" the answer would be """"live and learn."""" The letter that's moved can come from either word. " 1 bridge, room bride and groom 20080720 "In this week's on-air puzzle, you are given two words. Move a letter from one word to the other to make two new words that complete the familiar phrase, """"___________ and ___________."""" For example, given """"liven"""" and """"Lear,"""" the answer would be """"live and learn."""" The letter that's moved can come from either word. " 2 shard, fat hard and fast 20080720 "In this week's on-air puzzle, you are given two words. Move a letter from one word to the other to make two new words that complete the familiar phrase, """"___________ and ___________."""" For example, given """"liven"""" and """"Lear,"""" the answer would be """"live and learn."""" The letter that's moved can come from either word. " 3 stick, ired sick and tired 20080720 "In this week's on-air puzzle, you are given two words. Move a letter from one word to the other to make two new words that complete the familiar phrase, """"___________ and ___________."""" For example, given """"liven"""" and """"Lear,"""" the answer would be """"live and learn."""" The letter that's moved can come from either word. " 4 strop, stat stop and start 20080720 "In this week's on-air puzzle, you are given two words. Move a letter from one word to the other to make two new words that complete the familiar phrase, """"___________ and ___________."""" For example, given """"liven"""" and """"Lear,"""" the answer would be """"live and learn."""" The letter that's moved can come from either word. " 5 stares, strips stars and stripes 20070923 "In this week's on-air puzzle, every answer is a word or phrase used in football, delivered in the form of an anagram. For example, if the word given for the clue is """"saps"""" the answer would be """"pass.""""" 0 cask sack 20070923 "In this week's on-air puzzle, every answer is a word or phrase used in football, delivered in the form of an anagram. For example, if the word given for the clue is """"saps"""" the answer would be """"pass.""""" 1 recent center 20070923 "In this week's on-air puzzle, every answer is a word or phrase used in football, delivered in the form of an anagram. For example, if the word given for the clue is """"saps"""" the answer would be """"pass.""""" 2 melanin lineman 20070923 "In this week's on-air puzzle, every answer is a word or phrase used in football, delivered in the form of an anagram. For example, if the word given for the clue is """"saps"""" the answer would be """"pass.""""" 3 dies off offside 20070923 "In this week's on-air puzzle, every answer is a word or phrase used in football, delivered in the form of an anagram. For example, if the word given for the clue is """"saps"""" the answer would be """"pass.""""" 4 aplenty penalty 20070923 "In this week's on-air puzzle, every answer is a word or phrase used in football, delivered in the form of an anagram. For example, if the word given for the clue is """"saps"""" the answer would be """"pass.""""" 5 spiking pigskin 20070923 "In this week's on-air puzzle, every answer is a word or phrase used in football, delivered in the form of an anagram. For example, if the word given for the clue is """"saps"""" the answer would be """"pass.""""" 6 go postal goal post 20070923 "In this week's on-air puzzle, every answer is a word or phrase used in football, delivered in the form of an anagram. For example, if the word given for the clue is """"saps"""" the answer would be """"pass.""""" 7 remove it overtime 20070923 "In this week's on-air puzzle, every answer is a word or phrase used in football, delivered in the form of an anagram. For example, if the word given for the clue is """"saps"""" the answer would be """"pass.""""" 8 creep into reception 20070923 "In this week's on-air puzzle, every answer is a word or phrase used in football, delivered in the form of an anagram. For example, if the word given for the clue is """"saps"""" the answer would be """"pass.""""" 9 dunner (2 word phrase answersfrom now on) end run 20070923 "In this week's on-air puzzle, every answer is a word or phrase used in football, delivered in the form of an anagram. For example, if the word given for the clue is """"saps"""" the answer would be """"pass.""""" 11 snowdrift first down 20070923 "In this week's on-air puzzle, every answer is a word or phrase used in football, delivered in the form of an anagram. For example, if the word given for the clue is """"saps"""" the answer would be """"pass.""""" 12 dangerous nose guard 20080203 Every answer is a familiar two-word phrase in which the first word starts with the letter C and the second word starts with P-A. 0 famous Las Vegas hotel and casino Caesars Palace 20080203 Every answer is a familiar two-word phrase in which the first word starts with the letter C and the second word starts with P-A. 1 walk on role cameo part 20080203 Every answer is a familiar two-word phrase in which the first word starts with the letter C and the second word starts with P-A. 2 a sheet that allows you to make copies ona typewriter carbon paper 20080203 Every answer is a familiar two-word phrase in which the first word starts with the letter C and the second word starts with P-A. 3 a social gathering with drinks cocktail party 20080203 Every answer is a familiar two-word phrase in which the first word starts with the letter C and the second word starts with P-A. 4 where to read Blondie and Beetle Bailey comics page 20080203 Every answer is a familiar two-word phrase in which the first word starts with the letter C and the second word starts with P-A. 5 tree that might be on a desert island coconut palm 20080203 Every answer is a familiar two-word phrase in which the first word starts with the letter C and the second word starts with P-A. 6 something wonderful in 1920's slang cat's pajamas 20080203 Every answer is a familiar two-word phrase in which the first word starts with the letter C and the second word starts with P-A. 7 implement for a boat on a lake canoe paddle 20080203 Every answer is a familiar two-word phrase in which the first word starts with the letter C and the second word starts with P-A. 8 birthplace of a kid in a 1980's doll craze cabbage patch 20080203 Every answer is a familiar two-word phrase in which the first word starts with the letter C and the second word starts with P-A. 9 baker's container cake pan 20080203 Every answer is a familiar two-word phrase in which the first word starts with the letter C and the second word starts with P-A. 10 what a citizen of Quebec or Ontariocarries when traveling abroad Canadian passport 20080203 Every answer is a familiar two-word phrase in which the first word starts with the letter C and the second word starts with P-A. 11 long-time home for San Francisco Giants Candlestick Park 20080203 Every answer is a familiar two-word phrase in which the first word starts with the letter C and the second word starts with P-A. 12 humanitarian gift that is sent abroad CARE package 20080203 Every answer is a familiar two-word phrase in which the first word starts with the letter C and the second word starts with P-A. 13 extra money for a soldier during war combat pay 20080203 Every answer is a familiar two-word phrase in which the first word starts with the letter C and the second word starts with P-A. 14 in slang, a place to sleep, temporarily,for free crash pad 20080203 Every answer is a familiar two-word phrase in which the first word starts with the letter C and the second word starts with P-A. 15 NFL team in Charlotte Carolina Panthers 20080203 Every answer is a familiar two-word phrase in which the first word starts with the letter C and the second word starts with P-A. 16 by definition, it is not intercepted completed pass 20071216 In this week's on-air puzzle, take two four-letter words and put three letters in front of both of them to complete two common seven-letter words. For example: for the words, GAIN and RACK, the added three-letter word would be BAR to form BARGAIN AND BARRACK. 0 give, tune forgive, fortune 20071216 In this week's on-air puzzle, take two four-letter words and put three letters in front of both of them to complete two common seven-letter words. For example: for the words, GAIN and RACK, the added three-letter word would be BAR to form BARGAIN AND BARRACK. 1 arch, soon monarch, monsoon 20071216 In this week's on-air puzzle, take two four-letter words and put three letters in front of both of them to complete two common seven-letter words. For example: for the words, GAIN and RACK, the added three-letter word would be BAR to form BARGAIN AND BARRACK. 2 mach, rage stomach, storage 20071216 In this week's on-air puzzle, take two four-letter words and put three letters in front of both of them to complete two common seven-letter words. For example: for the words, GAIN and RACK, the added three-letter word would be BAR to form BARGAIN AND BARRACK. 3 grin, lice chagrin, chalice 20071216 In this week's on-air puzzle, take two four-letter words and put three letters in front of both of them to complete two common seven-letter words. For example: for the words, GAIN and RACK, the added three-letter word would be BAR to form BARGAIN AND BARRACK. 4 lain, lore explain, explore 20071216 In this week's on-air puzzle, take two four-letter words and put three letters in front of both of them to complete two common seven-letter words. For example: for the words, GAIN and RACK, the added three-letter word would be BAR to form BARGAIN AND BARRACK. 5 sage, tiff massage, mastiff 20071216 In this week's on-air puzzle, take two four-letter words and put three letters in front of both of them to complete two common seven-letter words. For example: for the words, GAIN and RACK, the added three-letter word would be BAR to form BARGAIN AND BARRACK. 6 each, rove impeach, improve 20071216 In this week's on-air puzzle, take two four-letter words and put three letters in front of both of them to complete two common seven-letter words. For example: for the words, GAIN and RACK, the added three-letter word would be BAR to form BARGAIN AND BARRACK. 7 ache, icky panache, panicky 20071216 In this week's on-air puzzle, take two four-letter words and put three letters in front of both of them to complete two common seven-letter words. For example: for the words, GAIN and RACK, the added three-letter word would be BAR to form BARGAIN AND BARRACK. 8 bang, riff shebang, sheriff 20071216 In this week's on-air puzzle, take two four-letter words and put three letters in front of both of them to complete two common seven-letter words. For example: for the words, GAIN and RACK, the added three-letter word would be BAR to form BARGAIN AND BARRACK. 9 lacy, lout fallacy, fallout 20071216 In this week's on-air puzzle, take two four-letter words and put three letters in front of both of them to complete two common seven-letter words. For example: for the words, GAIN and RACK, the added three-letter word would be BAR to form BARGAIN AND BARRACK. 10 ashy, urge splashy, splurge 20090215 "Every answer is the name of an animal you might see in the zoo. Name the animals from their anagrams. For example, given """"oil"""" plus """"N,"""" the answer would be """"lion.""""" 0 reb + a bear 20090215 "Every answer is the name of an animal you might see in the zoo. Name the animals from their anagrams. For example, given """"oil"""" plus """"N,"""" the answer would be """"lion.""""" 1 Ezra + b zebra 20090215 "Every answer is the name of an animal you might see in the zoo. Name the animals from their anagrams. For example, given """"oil"""" plus """"N,"""" the answer would be """"lion.""""" 2 NAPA + d panda 20090215 "Every answer is the name of an animal you might see in the zoo. Name the animals from their anagrams. For example, given """"oil"""" plus """"N,"""" the answer would be """"lion.""""" 3 rite + g tiger 20090215 "Every answer is the name of an animal you might see in the zoo. Name the animals from their anagrams. For example, given """"oil"""" plus """"N,"""" the answer would be """"lion.""""" 4 nabob + o baboon 20090215 "Every answer is the name of an animal you might see in the zoo. Name the animals from their anagrams. For example, given """"oil"""" plus """"N,"""" the answer would be """"lion.""""" 5 Rawls + u walrus 20090215 "Every answer is the name of an animal you might see in the zoo. Name the animals from their anagrams. For example, given """"oil"""" plus """"N,"""" the answer would be """"lion.""""" 6 allege + z gazelle 20090215 "Every answer is the name of an animal you might see in the zoo. Name the animals from their anagrams. For example, given """"oil"""" plus """"N,"""" the answer would be """"lion.""""" 7 gaffer + i giraffe 20090215 "Every answer is the name of an animal you might see in the zoo. Name the animals from their anagrams. For example, given """"oil"""" plus """"N,"""" the answer would be """"lion.""""" 8 enrapt + h panther 20090215 "Every answer is the name of an animal you might see in the zoo. Name the animals from their anagrams. For example, given """"oil"""" plus """"N,"""" the answer would be """"lion.""""" 9 Gloria + l gorilla 20090215 "Every answer is the name of an animal you might see in the zoo. Name the animals from their anagrams. For example, given """"oil"""" plus """"N,"""" the answer would be """"lion.""""" 10 reload + p leopard 20090215 "Every answer is the name of an animal you might see in the zoo. Name the animals from their anagrams. For example, given """"oil"""" plus """"N,"""" the answer would be """"lion.""""" 11 heeltap + n, a large animal you may see in a circus elephant 20090215 "Every answer is the name of an animal you might see in the zoo. Name the animals from their anagrams. For example, given """"oil"""" plus """"N,"""" the answer would be """"lion.""""" 12 needier + r reindeer 20090215 "Every answer is the name of an animal you might see in the zoo. Name the animals from their anagrams. For example, given """"oil"""" plus """"N,"""" the answer would be """"lion.""""" 13 mechanize + p chimpanzee 20081214 "You are given two seven-letter words. Rearrange the letters in one of the words to get a synonym of the other. For example, if the clue is """"gratify"""" and """"eluding,"""" the answer would be """"indulge,"""" which is an anagram of eluding and means gratify. The anagram can be the first or second word." 0 sincere, ingenue genuine 20081214 "You are given two seven-letter words. Rearrange the letters in one of the words to get a synonym of the other. For example, if the clue is """"gratify"""" and """"eluding,"""" the answer would be """"indulge,"""" which is an anagram of eluding and means gratify. The anagram can be the first or second word." 1 witness, verbose observe 20081214 "You are given two seven-letter words. Rearrange the letters in one of the words to get a synonym of the other. For example, if the clue is """"gratify"""" and """"eluding,"""" the answer would be """"indulge,"""" which is an anagram of eluding and means gratify. The anagram can be the first or second word." 2 relieve, sausage assuage 20081214 "You are given two seven-letter words. Rearrange the letters in one of the words to get a synonym of the other. For example, if the clue is """"gratify"""" and """"eluding,"""" the answer would be """"indulge,"""" which is an anagram of eluding and means gratify. The anagram can be the first or second word." 3 phantom, respect specter 20081214 "You are given two seven-letter words. Rearrange the letters in one of the words to get a synonym of the other. For example, if the clue is """"gratify"""" and """"eluding,"""" the answer would be """"indulge,"""" which is an anagram of eluding and means gratify. The anagram can be the first or second word." 4 ecstasy, toenail elation 20081214 "You are given two seven-letter words. Rearrange the letters in one of the words to get a synonym of the other. For example, if the clue is """"gratify"""" and """"eluding,"""" the answer would be """"indulge,"""" which is an anagram of eluding and means gratify. The anagram can be the first or second word." 5 donator, twister tornado 20081214 "You are given two seven-letter words. Rearrange the letters in one of the words to get a synonym of the other. For example, if the clue is """"gratify"""" and """"eluding,"""" the answer would be """"indulge,"""" which is an anagram of eluding and means gratify. The anagram can be the first or second word." 6 eternal, sealegs ageless 20090405 "Every answer is a familiar two-word phrase, in which each word has a short """"A"""" vowel sound. For example, given the clue """"A pest weed in lawns,"""" the answer would be """"crab grass.""""" 1 a product of Hefty trash bag 20090405 "Every answer is a familiar two-word phrase, in which each word has a short """"A"""" vowel sound. For example, given the clue """"A pest weed in lawns,"""" the answer would be """"crab grass.""""" 2 a group of sheets of paper on which you doodle orwrite notes scratch pad 20090405 "Every answer is a familiar two-word phrase, in which each word has a short """"A"""" vowel sound. For example, given the clue """"A pest weed in lawns,"""" the answer would be """"crab grass.""""" 3 an annoying thing that accompanies a sitcom laugh track 20090405 "Every answer is a familiar two-word phrase, in which each word has a short """"A"""" vowel sound. For example, given the clue """"A pest weed in lawns,"""" the answer would be """"crab grass.""""" 4 medical image of inside the body cat scan 20090405 "Every answer is a familiar two-word phrase, in which each word has a short """"A"""" vowel sound. For example, given the clue """"A pest weed in lawns,"""" the answer would be """"crab grass.""""" 5 protection from poison in the air gas mask 20090405 "Every answer is a familiar two-word phrase, in which each word has a short """"A"""" vowel sound. For example, given the clue """"A pest weed in lawns,"""" the answer would be """"crab grass.""""" 6 where you might hang your chapeau hat rack 20090405 "Every answer is a familiar two-word phrase, in which each word has a short """"A"""" vowel sound. For example, given the clue """"A pest weed in lawns,"""" the answer would be """"crab grass.""""" 7 group that included Frank Sinatra, Dean Martin andSammy Davis, Jr. Rat Pack 20090405 "Every answer is a familiar two-word phrase, in which each word has a short """"A"""" vowel sound. For example, given the clue """"A pest weed in lawns,"""" the answer would be """"crab grass.""""" 8 activity for Sammy Davis, Jr., and Gregory Hines tap dance 20090405 "Every answer is a familiar two-word phrase, in which each word has a short """"A"""" vowel sound. For example, given the clue """"A pest weed in lawns,"""" the answer would be """"crab grass.""""" 9 what you might get at a club to allow reentry hand stamp 20090405 "Every answer is a familiar two-word phrase, in which each word has a short """"A"""" vowel sound. For example, given the clue """"A pest weed in lawns,"""" the answer would be """"crab grass.""""" 10 something that prevents you from slipping in theshower bath mat 20090405 "Every answer is a familiar two-word phrase, in which each word has a short """"A"""" vowel sound. For example, given the clue """"A pest weed in lawns,"""" the answer would be """"crab grass.""""" 11 according to superstition, it brings bad luck if itruns in front of you black cat 20090405 "Every answer is a familiar two-word phrase, in which each word has a short """"A"""" vowel sound. For example, given the clue """"A pest weed in lawns,"""" the answer would be """"crab grass.""""" 12 winner of all the tricks in a bridge hand grand slam 20080106 "In the on-air puzzle, every answer is the name of a well-known company. You must name the company from its anagram. For example, given ""Coal plus A,"" the answer would be, ""Alcoa.""" 0 pale + P, computer maker Apple 20080106 "In the on-air puzzle, every answer is the name of a well-known company. You must name the company from its anagram. For example, given ""Coal plus A,"" the answer would be, ""Alcoa.""" 1 scam + Y, retail company Macy's 20080106 "In the on-air puzzle, every answer is the name of a well-known company. You must name the company from its anagram. For example, given ""Coal plus A,"" the answer would be, ""Alcoa.""" 2 ahoy + O, a way to get on the internet Yahoo 20080106 "In the on-air puzzle, every answer is the name of a well-known company. You must name the company from its anagram. For example, given ""Coal plus A,"" the answer would be, ""Alcoa.""" 3 coons + U, oil company Sunoco 20080106 "In the on-air puzzle, every answer is the name of a well-known company. You must name the company from its anagram. For example, given ""Coal plus A,"" the answer would be, ""Alcoa.""" 4 pound + T, chemical company Du Pont 20080106 "In the on-air puzzle, every answer is the name of a well-known company. You must name the company from its anagram. For example, given ""Coal plus A,"" the answer would be, ""Alcoa.""" 5 litho + N, hotel company Hilton 20080106 "In the on-air puzzle, every answer is the name of a well-known company. You must name the company from its anagram. For example, given ""Coal plus A,"" the answer would be, ""Alcoa.""" 6 metal + T, toy company Mattel 20080106 "In the on-air puzzle, every answer is the name of a well-known company. You must name the company from its anagram. For example, given ""Coal plus A,"" the answer would be, ""Alcoa.""" 7 prize + F, pharmaceutical company Pfizer 20080106 "In the on-air puzzle, every answer is the name of a well-known company. You must name the company from its anagram. For example, given ""Coal plus A,"" the answer would be, ""Alcoa.""" 8 copies + P, full name of beverage company PepsiCo 20080106 "In the on-air puzzle, every answer is the name of a well-known company. You must name the company from its anagram. For example, given ""Coal plus A,"" the answer would be, ""Alcoa.""" 9 accost + M, telecom company Comcast 20080106 "In the on-air puzzle, every answer is the name of a well-known company. You must name the company from its anagram. For example, given ""Coal plus A,"" the answer would be, ""Alcoa.""" 10 traitor + M, hotelier Marriott 20080106 "In the on-air puzzle, every answer is the name of a well-known company. You must name the company from its anagram. For example, given ""Coal plus A,"" the answer would be, ""Alcoa.""" 11 poor Will + H, appliance company Whirlpool 20071118 In the on-air puzzle, given a word, the response is a word that completes a common two-word phrase. The first two letters of a given word will be the last two letters of the player's word, reversed. 0 bar (3-letter answers for now) tab 20071118 In the on-air puzzle, given a word, the response is a word that completes a common two-word phrase. The first two letters of a given word will be the last two letters of the player's word, reversed. 1 pace lap 20071118 In the on-air puzzle, given a word, the response is a word that completes a common two-word phrase. The first two letters of a given word will be the last two letters of the player's word, reversed. 2 teacher's pet 20071118 In the on-air puzzle, given a word, the response is a word that completes a common two-word phrase. The first two letters of a given word will be the last two letters of the player's word, reversed. 3 tight fit 20071118 In the on-air puzzle, given a word, the response is a word that completes a common two-word phrase. The first two letters of a given word will be the last two letters of the player's word, reversed. 4 dowsing rod 20071118 In the on-air puzzle, given a word, the response is a word that completes a common two-word phrase. The first two letters of a given word will be the last two letters of the player's word, reversed. 5 garbage bag 20071118 In the on-air puzzle, given a word, the response is a word that completes a common two-word phrase. The first two letters of a given word will be the last two letters of the player's word, reversed. 6 Aegean Sea 20071118 In the on-air puzzle, given a word, the response is a word that completes a common two-word phrase. The first two letters of a given word will be the last two letters of the player's word, reversed. 7 lemon (4-letter answer now) peel 20071118 In the on-air puzzle, given a word, the response is a word that completes a common two-word phrase. The first two letters of a given word will be the last two letters of the player's word, reversed. 8 motel room 20071118 In the on-air puzzle, given a word, the response is a word that completes a common two-word phrase. The first two letters of a given word will be the last two letters of the player's word, reversed. 9 navy bean 20071118 In the on-air puzzle, given a word, the response is a word that completes a common two-word phrase. The first two letters of a given word will be the last two letters of the player's word, reversed. 10 pink slip 20071118 In the on-air puzzle, given a word, the response is a word that completes a common two-word phrase. The first two letters of a given word will be the last two letters of the player's word, reversed. 11 end zone 20071118 In the on-air puzzle, given a word, the response is a word that completes a common two-word phrase. The first two letters of a given word will be the last two letters of the player's word, reversed. 12 drug card or lord 20071118 In the on-air puzzle, given a word, the response is a word that completes a common two-word phrase. The first two letters of a given word will be the last two letters of the player's word, reversed. 13 pork chop 20071118 In the on-air puzzle, given a word, the response is a word that completes a common two-word phrase. The first two letters of a given word will be the last two letters of the player's word, reversed. 14 dog food 20080224 "In this week's on-air puzzle, clues are given for two words. Each word has two syllables. The first syllable of the first word has a short """"U"""" sound as in """"uh."""" Change this to a long """"O"""" sound and, phonetically, you'll get a new word that answers the second clue. Example: For the clue, """"absolutely beautiful"""" and """"an old style punishment,"""" the answer would be """"stunning"""" and """"stoning."""" " 1 General George at the Battle of LittleBig Horn; something to set a drink on Custer, coaster 20080224 "In this week's on-air puzzle, clues are given for two words. Each word has two syllables. The first syllable of the first word has a short """"U"""" sound as in """"uh."""" Change this to a long """"O"""" sound and, phonetically, you'll get a new word that answers the second clue. Example: For the clue, """"absolutely beautiful"""" and """"an old style punishment,"""" the answer would be """"stunning"""" and """"stoning."""" " 2 to say, under one's breath; an engine mutter, motor 20080224 "In this week's on-air puzzle, clues are given for two words. Each word has two syllables. The first syllable of the first word has a short """"U"""" sound as in """"uh."""" Change this to a long """"O"""" sound and, phonetically, you'll get a new word that answers the second clue. Example: For the clue, """"absolutely beautiful"""" and """"an old style punishment,"""" the answer would be """"stunning"""" and """"stoning."""" " 3 a light four-wheeled carriage; a scoreof one over par buggy, bogie 20080224 "In this week's on-air puzzle, clues are given for two words. Each word has two syllables. The first syllable of the first word has a short """"U"""" sound as in """"uh."""" Change this to a long """"O"""" sound and, phonetically, you'll get a new word that answers the second clue. Example: For the clue, """"absolutely beautiful"""" and """"an old style punishment,"""" the answer would be """"stunning"""" and """"stoning."""" " 4 a ravine; a hockey position gully, goalie 20080224 "In this week's on-air puzzle, clues are given for two words. Each word has two syllables. The first syllable of the first word has a short """"U"""" sound as in """"uh."""" Change this to a long """"O"""" sound and, phonetically, you'll get a new word that answers the second clue. Example: For the clue, """"absolutely beautiful"""" and """"an old style punishment,"""" the answer would be """"stunning"""" and """"stoning."""" " 5 a gas guzzling General Motors vehicle;a baseball hit over the wall Hummer, homer 20080224 "In this week's on-air puzzle, clues are given for two words. Each word has two syllables. The first syllable of the first word has a short """"U"""" sound as in """"uh."""" Change this to a long """"O"""" sound and, phonetically, you'll get a new word that answers the second clue. Example: For the clue, """"absolutely beautiful"""" and """"an old style punishment,"""" the answer would be """"stunning"""" and """"stoning."""" " 6 a passage through a mountain; havingharmony and melody tunnel, tonal 20080224 "In this week's on-air puzzle, clues are given for two words. Each word has two syllables. The first syllable of the first word has a short """"U"""" sound as in """"uh."""" Change this to a long """"O"""" sound and, phonetically, you'll get a new word that answers the second clue. Example: For the clue, """"absolutely beautiful"""" and """"an old style punishment,"""" the answer would be """"stunning"""" and """"stoning."""" " 7 having a healthy red complexion; anassistant to a rock band ruddy, roadie 20080224 "In this week's on-air puzzle, clues are given for two words. Each word has two syllables. The first syllable of the first word has a short """"U"""" sound as in """"uh."""" Change this to a long """"O"""" sound and, phonetically, you'll get a new word that answers the second clue. Example: For the clue, """"absolutely beautiful"""" and """"an old style punishment,"""" the answer would be """"stunning"""" and """"stoning."""" " 8 a little fat; actor Maguire tubby, Tobey 20080224 "In this week's on-air puzzle, clues are given for two words. Each word has two syllables. The first syllable of the first word has a short """"U"""" sound as in """"uh."""" Change this to a long """"O"""" sound and, phonetically, you'll get a new word that answers the second clue. Example: For the clue, """"absolutely beautiful"""" and """"an old style punishment,"""" the answer would be """"stunning"""" and """"stoning."""" " 9 period of play in polo; a snug necklace chukker, choker 20080224 "In this week's on-air puzzle, clues are given for two words. Each word has two syllables. The first syllable of the first word has a short """"U"""" sound as in """"uh."""" Change this to a long """"O"""" sound and, phonetically, you'll get a new word that answers the second clue. Example: For the clue, """"absolutely beautiful"""" and """"an old style punishment,"""" the answer would be """"stunning"""" and """"stoning."""" " 10 fortunate; trickster god in Norse myth lucky, Loki 20080224 "In this week's on-air puzzle, clues are given for two words. Each word has two syllables. The first syllable of the first word has a short """"U"""" sound as in """"uh."""" Change this to a long """"O"""" sound and, phonetically, you'll get a new word that answers the second clue. Example: For the clue, """"absolutely beautiful"""" and """"an old style punishment,"""" the answer would be """"stunning"""" and """"stoning."""" " 11 like some Groucho quips; a small horse puny, pony 20080224 "In this week's on-air puzzle, clues are given for two words. Each word has two syllables. The first syllable of the first word has a short """"U"""" sound as in """"uh."""" Change this to a long """"O"""" sound and, phonetically, you'll get a new word that answers the second clue. Example: For the clue, """"absolutely beautiful"""" and """"an old style punishment,"""" the answer would be """"stunning"""" and """"stoning."""" " 12 fair, as a day at the beach; big namein audio and video equipment sunny, Sony 20080224 "In this week's on-air puzzle, clues are given for two words. Each word has two syllables. The first syllable of the first word has a short """"U"""" sound as in """"uh."""" Change this to a long """"O"""" sound and, phonetically, you'll get a new word that answers the second clue. Example: For the clue, """"absolutely beautiful"""" and """"an old style punishment,"""" the answer would be """"stunning"""" and """"stoning."""" " 13 machines that fill envelopes; a brandof frozen dinners stuffers, Stouffer's 20080727 In the on-air puzzle, you must complete a two-word phrase with a word that starts with the third and fourth letters of the word given to you. 0 postage stamp 20080727 In the on-air puzzle, you must complete a two-word phrase with a word that starts with the third and fourth letters of the word given to you. 1 clerical error 20080727 In the on-air puzzle, you must complete a two-word phrase with a word that starts with the third and fourth letters of the word given to you. 2 liberty bell 20080727 In the on-air puzzle, you must complete a two-word phrase with a word that starts with the third and fourth letters of the word given to you. 3 straight razor 20080727 In the on-air puzzle, you must complete a two-word phrase with a word that starts with the third and fourth letters of the word given to you. 4 second course, coming, cousin 20080727 In the on-air puzzle, you must complete a two-word phrase with a word that starts with the third and fourth letters of the word given to you. 5 short order 20080727 In the on-air puzzle, you must complete a two-word phrase with a word that starts with the third and fourth letters of the word given to you. 6 silent letter 20080727 In the on-air puzzle, you must complete a two-word phrase with a word that starts with the third and fourth letters of the word given to you. 7 estate tax 20080727 In the on-air puzzle, you must complete a two-word phrase with a word that starts with the third and fourth letters of the word given to you. 8 intelligence test 20080727 In the on-air puzzle, you must complete a two-word phrase with a word that starts with the third and fourth letters of the word given to you. 9 Pythagorean theorem 20090208 "Given a four-letter word, name another four-letter word that starts with the last letter of the given word. When the two words are read successively, overlapping on the shared letter, they'll spell a familiar seven-letter word. For example, given """"prop,"""" the answer would be """"pose""""; put together, the two words form """"propose.""""" 0 term mite, termite 20090208 "Given a four-letter word, name another four-letter word that starts with the last letter of the given word. When the two words are read successively, overlapping on the shared letter, they'll spell a familiar seven-letter word. For example, given """"prop,"""" the answer would be """"pose""""; put together, the two words form """"propose.""""" 1 aver rage, average 20090208 "Given a four-letter word, name another four-letter word that starts with the last letter of the given word. When the two words are read successively, overlapping on the shared letter, they'll spell a familiar seven-letter word. For example, given """"prop,"""" the answer would be """"pose""""; put together, the two words form """"propose.""""" 2 port (answer is something found in a cafeteria) tray, portray 20090208 "Given a four-letter word, name another four-letter word that starts with the last letter of the given word. When the two words are read successively, overlapping on the shared letter, they'll spell a familiar seven-letter word. For example, given """"prop,"""" the answer would be """"pose""""; put together, the two words form """"propose.""""" 3 earn nest, earnest 20090208 "Given a four-letter word, name another four-letter word that starts with the last letter of the given word. When the two words are read successively, overlapping on the shared letter, they'll spell a familiar seven-letter word. For example, given """"prop,"""" the answer would be """"pose""""; put together, the two words form """"propose.""""" 4 fort tune, fortune 20090208 "Given a four-letter word, name another four-letter word that starts with the last letter of the given word. When the two words are read successively, overlapping on the shared letter, they'll spell a familiar seven-letter word. For example, given """"prop,"""" the answer would be """"pose""""; put together, the two words form """"propose.""""" 5 stew ward, steward 20090208 "Given a four-letter word, name another four-letter word that starts with the last letter of the given word. When the two words are read successively, overlapping on the shared letter, they'll spell a familiar seven-letter word. For example, given """"prop,"""" the answer would be """"pose""""; put together, the two words form """"propose.""""" 6 lamp prey, lamprey 20090208 "Given a four-letter word, name another four-letter word that starts with the last letter of the given word. When the two words are read successively, overlapping on the shared letter, they'll spell a familiar seven-letter word. For example, given """"prop,"""" the answer would be """"pose""""; put together, the two words form """"propose.""""" 7 pars snip, parsnip 20090208 "Given a four-letter word, name another four-letter word that starts with the last letter of the given word. When the two words are read successively, overlapping on the shared letter, they'll spell a familiar seven-letter word. For example, given """"prop,"""" the answer would be """"pose""""; put together, the two words form """"propose.""""" 8 mess sage, message 20090208 "Given a four-letter word, name another four-letter word that starts with the last letter of the given word. When the two words are read successively, overlapping on the shared letter, they'll spell a familiar seven-letter word. For example, given """"prop,"""" the answer would be """"pose""""; put together, the two words form """"propose.""""" 9 grim mace, grimace 20090208 "Given a four-letter word, name another four-letter word that starts with the last letter of the given word. When the two words are read successively, overlapping on the shared letter, they'll spell a familiar seven-letter word. For example, given """"prop,"""" the answer would be """"pose""""; put together, the two words form """"propose.""""" 10 cord dial, cordial 20090208 "Given a four-letter word, name another four-letter word that starts with the last letter of the given word. When the two words are read successively, overlapping on the shared letter, they'll spell a familiar seven-letter word. For example, given """"prop,"""" the answer would be """"pose""""; put together, the two words form """"propose.""""" 11 bill (answer is an animal) lion, billion 20090208 "Given a four-letter word, name another four-letter word that starts with the last letter of the given word. When the two words are read successively, overlapping on the shared letter, they'll spell a familiar seven-letter word. For example, given """"prop,"""" the answer would be """"pose""""; put together, the two words form """"propose.""""" 12 grad dual, gradual 20090802 "Every answer is a familiar proverb or saying. Given a two-word phrase, one of the words is in that proverb or saying, and the other is an anagram of a word in it. The anagram can be either word in the phrase. Example: MORE ROADS. The answer would be: """"All ROADS lead to ROME.""""" 1 rolling notes A rolling stone gathers no moss. 20090802 "Every answer is a familiar proverb or saying. Given a two-word phrase, one of the words is in that proverb or saying, and the other is an anagram of a word in it. The anagram can be either word in the phrase. Example: MORE ROADS. The answer would be: """"All ROADS lead to ROME.""""" 2 gold ton All that glimmers is not gold. 20090802 "Every answer is a familiar proverb or saying. Given a two-word phrase, one of the words is in that proverb or saying, and the other is an anagram of a word in it. The anagram can be either word in the phrase. Example: MORE ROADS. The answer would be: """"All ROADS lead to ROME.""""" 3 two hubs A bird in the tree is worth two in the bush. 20090802 "Every answer is a familiar proverb or saying. Given a two-word phrase, one of the words is in that proverb or saying, and the other is an anagram of a word in it. The anagram can be either word in the phrase. Example: MORE ROADS. The answer would be: """"All ROADS lead to ROME.""""" 4 raw loveMake love not war All is fair in love and war.. 20090802 "Every answer is a familiar proverb or saying. Given a two-word phrase, one of the words is in that proverb or saying, and the other is an anagram of a word in it. The anagram can be either word in the phrase. Example: MORE ROADS. The answer would be: """"All ROADS lead to ROME.""""" 5 bed layer Early to bed and early to rise makes a man 20090802 "Every answer is a familiar proverb or saying. Given a two-word phrase, one of the words is in that proverb or saying, and the other is an anagram of a word in it. The anagram can be either word in the phrase. Example: MORE ROADS. The answer would be: """"All ROADS lead to ROME.""""" 6 healthy, wealthy and wise. 20090802 "Every answer is a familiar proverb or saying. Given a two-word phrase, one of the words is in that proverb or saying, and the other is an anagram of a word in it. The anagram can be either word in the phrase. Example: MORE ROADS. The answer would be: """"All ROADS lead to ROME.""""" 7 diet time Time and tide wait for no man. 20090802 "Every answer is a familiar proverb or saying. Given a two-word phrase, one of the words is in that proverb or saying, and the other is an anagram of a word in it. The anagram can be either word in the phrase. Example: MORE ROADS. The answer would be: """"All ROADS lead to ROME.""""" 8 fiction granters Truth is stranger than fiction. 20090802 "Every answer is a familiar proverb or saying. Given a two-word phrase, one of the words is in that proverb or saying, and the other is an anagram of a word in it. The anagram can be either word in the phrase. Example: MORE ROADS. The answer would be: """"All ROADS lead to ROME.""""" 9 nights end All good things come to an end. 20091213 "This week's puzzle is called """"I Will Lead."""" Each clue is a word that contains the letter """"I"""" inside. Rearrange the letters to get a new word that starts with the letter """"I."""" For example, if the clue is """"Susie,"""" the answer would be """"issue.""""" 0 coin icon 20091213 "This week's puzzle is called """"I Will Lead."""" Each clue is a word that contains the letter """"I"""" inside. Rearrange the letters to get a new word that starts with the letter """"I."""" For example, if the clue is """"Susie,"""" the answer would be """"issue.""""" 1 aide idea 20091213 "This week's puzzle is called """"I Will Lead."""" Each clue is a word that contains the letter """"I"""" inside. Rearrange the letters to get a new word that starts with the letter """"I."""" For example, if the clue is """"Susie,"""" the answer would be """"issue.""""" 2 Annie inane 20091213 "This week's puzzle is called """"I Will Lead."""" Each clue is a word that contains the letter """"I"""" inside. Rearrange the letters to get a new word that starts with the letter """"I."""" For example, if the clue is """"Susie,"""" the answer would be """"issue.""""" 3 runic incur 20091213 "This week's puzzle is called """"I Will Lead."""" Each clue is a word that contains the letter """"I"""" inside. Rearrange the letters to get a new word that starts with the letter """"I."""" For example, if the clue is """"Susie,"""" the answer would be """"issue.""""" 4 bedim imbed 20091213 "This week's puzzle is called """"I Will Lead."""" Each clue is a word that contains the letter """"I"""" inside. Rearrange the letters to get a new word that starts with the letter """"I."""" For example, if the clue is """"Susie,"""" the answer would be """"issue.""""" 5 sunlit insult 20091213 "This week's puzzle is called """"I Will Lead."""" Each clue is a word that contains the letter """"I"""" inside. Rearrange the letters to get a new word that starts with the letter """"I."""" For example, if the clue is """"Susie,"""" the answer would be """"issue.""""" 6 Renoir ironer (OR REIN) 20091213 "This week's puzzle is called """"I Will Lead."""" Each clue is a word that contains the letter """"I"""" inside. Rearrange the letters to get a new word that starts with the letter """"I."""" For example, if the clue is """"Susie,"""" the answer would be """"issue.""""" 7 cinema iceman 20091213 "This week's puzzle is called """"I Will Lead."""" Each clue is a word that contains the letter """"I"""" inside. Rearrange the letters to get a new word that starts with the letter """"I."""" For example, if the clue is """"Susie,"""" the answer would be """"issue.""""" 8 stifle itself 20091213 "This week's puzzle is called """"I Will Lead."""" Each clue is a word that contains the letter """"I"""" inside. Rearrange the letters to get a new word that starts with the letter """"I."""" For example, if the clue is """"Susie,"""" the answer would be """"issue.""""" 9 region ignore 20091213 "This week's puzzle is called """"I Will Lead."""" Each clue is a word that contains the letter """"I"""" inside. Rearrange the letters to get a new word that starts with the letter """"I."""" For example, if the clue is """"Susie,"""" the answer would be """"issue.""""" 10 umpire impure 20091213 "This week's puzzle is called """"I Will Lead."""" Each clue is a word that contains the letter """"I"""" inside. Rearrange the letters to get a new word that starts with the letter """"I."""" For example, if the clue is """"Susie,"""" the answer would be """"issue.""""" 11 excitation intoxicate 20081116 The letters S-B stand for seat belt. They also stand for some other familiar two word phrases. See if you can get them from the clues. 1 a piece of furniture that opens up forsleeping sofa bed 20081116 The letters S-B stand for seat belt. They also stand for some other familiar two word phrases. See if you can get them from the clues. 2 an annual football showdown Super Bowl 20081116 The letters S-B stand for seat belt. They also stand for some other familiar two word phrases. See if you can get them from the clues. 3 it slows down cars in a parking lot speed bump 20081116 The letters S-B stand for seat belt. They also stand for some other familiar two word phrases. See if you can get them from the clues. 4 restaurant feature with lettuce, carrots,tomatoes, etc. salad bar 20081116 The letters S-B stand for seat belt. They also stand for some other familiar two word phrases. See if you can get them from the clues. 5 military plane that's hard to detect stealth bomber 20081116 The letters S-B stand for seat belt. They also stand for some other familiar two word phrases. See if you can get them from the clues. 6 what a very fast plane produces sonic boom 20081116 The letters S-B stand for seat belt. They also stand for some other familiar two word phrases. See if you can get them from the clues. 7 a bit of someone's quote on the TV news sound bite 20081116 The letters S-B stand for seat belt. They also stand for some other familiar two word phrases. See if you can get them from the clues. 8 a big yellow vehicle school bus 20081116 The letters S-B stand for seat belt. They also stand for some other familiar two word phrases. See if you can get them from the clues. 9 a standard feature of a fair and democraticelection secret ballot 20081116 The letters S-B stand for seat belt. They also stand for some other familiar two word phrases. See if you can get them from the clues. 10 equipment at the World Cup soccer ball 20081116 The letters S-B stand for seat belt. They also stand for some other familiar two word phrases. See if you can get them from the clues. 11 victor at the Battle of Little Bighorn Sitting Bull 20081116 The letters S-B stand for seat belt. They also stand for some other familiar two word phrases. See if you can get them from the clues. 12 Bill Clinton's religious affiliation Southern Baptist 20081116 The letters S-B stand for seat belt. They also stand for some other familiar two word phrases. See if you can get them from the clues. 13 item of camping equipment sleeping bag 20081116 The letters S-B stand for seat belt. They also stand for some other familiar two word phrases. See if you can get them from the clues. 14 portable advertising sign sandwich board 20081116 The letters S-B stand for seat belt. They also stand for some other familiar two word phrases. See if you can get them from the clues. 15 baseball hit designed to advance a base runner sacrifice bunt 20081116 The letters S-B stand for seat belt. They also stand for some other familiar two word phrases. See if you can get them from the clues. 16 shade of hair that is reddish yellow strawberry blonde 20081116 The letters S-B stand for seat belt. They also stand for some other familiar two word phrases. See if you can get them from the clues. 17 hospital on a ship; an on the Enterprise onStar Trek sickbay 20081116 The letters S-B stand for seat belt. They also stand for some other familiar two word phrases. See if you can get them from the clues. 18 God Supreme Being 20080316 "In this week's puzzle, you are given two things and must name what they have in common. Each answer has six letters. For example, given """"typewriter"""" and """"Christmas gift,"""" the answer is """"ribbon.""""" 0 nose, San Francisco Bay bridge 20080316 "In this week's puzzle, you are given two things and must name what they have in common. Each answer has six letters. For example, given """"typewriter"""" and """"Christmas gift,"""" the answer is """"ribbon.""""" 1 credit card, skunk stripe 20080316 "In this week's puzzle, you are given two things and must name what they have in common. Each answer has six letters. For example, given """"typewriter"""" and """"Christmas gift,"""" the answer is """"ribbon.""""" 2 jack-in-the-box, a year spring 20080316 "In this week's puzzle, you are given two things and must name what they have in common. Each answer has six letters. For example, given """"typewriter"""" and """"Christmas gift,"""" the answer is """"ribbon.""""" 3 compass, phonograph needle 20080316 "In this week's puzzle, you are given two things and must name what they have in common. Each answer has six letters. For example, given """"typewriter"""" and """"Christmas gift,"""" the answer is """"ribbon.""""" 4 kangaroo, pool table pocket 20080316 "In this week's puzzle, you are given two things and must name what they have in common. Each answer has six letters. For example, given """"typewriter"""" and """"Christmas gift,"""" the answer is """"ribbon.""""" 5 bathroom, soon-to-be-bride shower 20080316 "In this week's puzzle, you are given two things and must name what they have in common. Each answer has six letters. For example, given """"typewriter"""" and """"Christmas gift,"""" the answer is """"ribbon.""""" 6 laundromat, hardware storetemperature, college graduate washer degree 20080316 "In this week's puzzle, you are given two things and must name what they have in common. Each answer has six letters. For example, given """"typewriter"""" and """"Christmas gift,"""" the answer is """"ribbon.""""" 7 door, CB user handle 20080316 "In this week's puzzle, you are given two things and must name what they have in common. Each answer has six letters. For example, given """"typewriter"""" and """"Christmas gift,"""" the answer is """"ribbon.""""" 8 railway station, beer pub porter 20080316 "In this week's puzzle, you are given two things and must name what they have in common. Each answer has six letters. For example, given """"typewriter"""" and """"Christmas gift,"""" the answer is """"ribbon.""""" 9 baseball game, cake mix bowl batter 20080316 "In this week's puzzle, you are given two things and must name what they have in common. Each answer has six letters. For example, given """"typewriter"""" and """"Christmas gift,"""" the answer is """"ribbon.""""" 10 ship, TV news program anchor 20081012 "Every answer is a two-word, rhyming phrase in which both words have three syllables. For example, given the clue """"unexplained circumstance of the past,"""" the answer would be """"history mystery.""""" 1 monthly pay for working at an art store gallery salary 20081012 "Every answer is a two-word, rhyming phrase in which both words have three syllables. For example, given the clue """"unexplained circumstance of the past,"""" the answer would be """"history mystery.""""" 2 better looking guy paying for a kidnappee handsomer ransomer 20081012 "Every answer is a two-word, rhyming phrase in which both words have three syllables. For example, given the clue """"unexplained circumstance of the past,"""" the answer would be """"history mystery.""""" 3 an average amount of boringness medium tedium 20081012 "Every answer is a two-word, rhyming phrase in which both words have three syllables. For example, given the clue """"unexplained circumstance of the past,"""" the answer would be """"history mystery.""""" 4 chance drawing for some clay ware pottery lottery 20081012 "Every answer is a two-word, rhyming phrase in which both words have three syllables. For example, given the clue """"unexplained circumstance of the past,"""" the answer would be """"history mystery.""""" 5 not as neat a Xerox machine sloppier copier 20081012 "Every answer is a two-word, rhyming phrase in which both words have three syllables. For example, given the clue """"unexplained circumstance of the past,"""" the answer would be """"history mystery.""""" 6 a more sadistic worker with gems crueler jeweler 20081012 "Every answer is a two-word, rhyming phrase in which both words have three syllables. For example, given the clue """"unexplained circumstance of the past,"""" the answer would be """"history mystery.""""" 7 faithfulness to the crown royalty loyalty 20081012 "Every answer is a two-word, rhyming phrase in which both words have three syllables. For example, given the clue """"unexplained circumstance of the past,"""" the answer would be """"history mystery.""""" 8 whoever is the current leader living inthe White House resident president 20081012 "Every answer is a two-word, rhyming phrase in which both words have three syllables. For example, given the clue """"unexplained circumstance of the past,"""" the answer would be """"history mystery.""""" 9 merriment for average churchgoers laity gaiety 20081012 "Every answer is a two-word, rhyming phrase in which both words have three syllables. For example, given the clue """"unexplained circumstance of the past,"""" the answer would be """"history mystery.""""" 10 a group of friends at an internationalservice club rotary coterie 20070729 "In this week's on-air puzzle, crossword clues point the way to the answer, which will rhyme with the first word of the clue. For example, if the clue is """"Home of the Vatican"""" the answer would be """"Rome.""""" 0 cash is hidden in one stash 20070729 "In this week's on-air puzzle, crossword clues point the way to the answer, which will rhyme with the first word of the clue. For example, if the clue is """"Home of the Vatican"""" the answer would be """"Rome.""""" 2 sneak a look peak 20070729 "In this week's on-air puzzle, crossword clues point the way to the answer, which will rhyme with the first word of the clue. For example, if the clue is """"Home of the Vatican"""" the answer would be """"Rome.""""" 3 big drink from a bottle swig 20070729 "In this week's on-air puzzle, crossword clues point the way to the answer, which will rhyme with the first word of the clue. For example, if the clue is """"Home of the Vatican"""" the answer would be """"Rome.""""" 4 group of USO entertainers troupe 20070729 "In this week's on-air puzzle, crossword clues point the way to the answer, which will rhyme with the first word of the clue. For example, if the clue is """"Home of the Vatican"""" the answer would be """"Rome.""""" 5 need for an oboe player reed 20070729 "In this week's on-air puzzle, crossword clues point the way to the answer, which will rhyme with the first word of the clue. For example, if the clue is """"Home of the Vatican"""" the answer would be """"Rome.""""" 6 some alcohol rum 20070729 "In this week's on-air puzzle, crossword clues point the way to the answer, which will rhyme with the first word of the clue. For example, if the clue is """"Home of the Vatican"""" the answer would be """"Rome.""""" 7 soul singer Natalie Cole 20070729 "In this week's on-air puzzle, crossword clues point the way to the answer, which will rhyme with the first word of the clue. For example, if the clue is """"Home of the Vatican"""" the answer would be """"Rome.""""" 8 4 to 3, perhaps score 20070729 "In this week's on-air puzzle, crossword clues point the way to the answer, which will rhyme with the first word of the clue. For example, if the clue is """"Home of the Vatican"""" the answer would be """"Rome.""""" 9 sped away fled 20070729 "In this week's on-air puzzle, crossword clues point the way to the answer, which will rhyme with the first word of the clue. For example, if the clue is """"Home of the Vatican"""" the answer would be """"Rome.""""" 10 huff puff 20070729 "In this week's on-air puzzle, crossword clues point the way to the answer, which will rhyme with the first word of the clue. For example, if the clue is """"Home of the Vatican"""" the answer would be """"Rome.""""" 11 put this in a shoe foot 20070729 "In this week's on-air puzzle, crossword clues point the way to the answer, which will rhyme with the first word of the clue. For example, if the clue is """"Home of the Vatican"""" the answer would be """"Rome.""""" 12 form of flying bees swarm 20070729 "In this week's on-air puzzle, crossword clues point the way to the answer, which will rhyme with the first word of the clue. For example, if the clue is """"Home of the Vatican"""" the answer would be """"Rome.""""" 13 not in good working order any more shot, not rot or caput 20070729 "In this week's on-air puzzle, crossword clues point the way to the answer, which will rhyme with the first word of the clue. For example, if the clue is """"Home of the Vatican"""" the answer would be """"Rome.""""" 14 word before rail or dimension third 20070729 "In this week's on-air puzzle, crossword clues point the way to the answer, which will rhyme with the first word of the clue. For example, if the clue is """"Home of the Vatican"""" the answer would be """"Rome.""""" 15 quit a place, in slang split, not flip 20070729 "In this week's on-air puzzle, crossword clues point the way to the answer, which will rhyme with the first word of the clue. For example, if the clue is """"Home of the Vatican"""" the answer would be """"Rome.""""" 16 dish at a Jewish deli knish, not fish 20070729 "In this week's on-air puzzle, crossword clues point the way to the answer, which will rhyme with the first word of the clue. For example, if the clue is """"Home of the Vatican"""" the answer would be """"Rome.""""" 17 use a bed for a short while snooze 20081228 You are given the names of people whom you had probably never heard of before 2008, but who sprang to national or international prominence during the past 12 months. Who are they? 1 Samuel Joseph Wurzelbacherpresidential election Joe the Plumber during the 2008 20081228 You are given the names of people whom you had probably never heard of before 2008, but who sprang to national or international prominence during the past 12 months. Who are they? 2 Michael Phelps14th gold medal in 2008 Olympic swimmer who won his 20081228 You are given the names of people whom you had probably never heard of before 2008, but who sprang to national or international prominence during the past 12 months. Who are they? 3 Rod Blagojevichfor trying to sell Obama'ssenate seat governor of Illinois indicted 20081228 You are given the names of people whom you had probably never heard of before 2008, but who sprang to national or international prominence during the past 12 months. Who are they? 4 David Patersonfirst blind governor governor of New York, America's 20081228 You are given the names of people whom you had probably never heard of before 2008, but who sprang to national or international prominence during the past 12 months. Who are they? 5 Dmitry Medvedev president of Russia 20081228 You are given the names of people whom you had probably never heard of before 2008, but who sprang to national or international prominence during the past 12 months. Who are they? 6 Bernard Madoffchairman of NASDAQ, chargedthis month with running a $50billion Ponzi scheme investment businessman, former 20081228 You are given the names of people whom you had probably never heard of before 2008, but who sprang to national or international prominence during the past 12 months. Who are they? 7 Usain Boltrecords in the 100 and 200meter dashes in the Olympics Jamaican runner who set world 20081228 You are given the names of people whom you had probably never heard of before 2008, but who sprang to national or international prominence during the past 12 months. Who are they? 8 Rob Pattinsonvampire Edward Cullen in thecurrent movie Twilight actor who plays 108-year-old 20081228 You are given the names of people whom you had probably never heard of before 2008, but who sprang to national or international prominence during the past 12 months. Who are they? 9 Kazuo Kawasaki (hint: designer of Sarah Palin's 20081228 You are given the names of people whom you had probably never heard of before 2008, but who sprang to national or international prominence during the past 12 months. Who are they? 10 eyeglasses) glasses 20081228 You are given the names of people whom you had probably never heard of before 2008, but who sprang to national or international prominence during the past 12 months. Who are they? 11 Muntadhar al-Zaidi (hint: Iraqi journalist who threw his 20081228 You are given the names of people whom you had probably never heard of before 2008, but who sprang to national or international prominence during the past 12 months. Who are they? 12 shoes) shoes at George W. Bush 20070513 I'm going to give you some words. For each one, change one of the letters in the word to M-A to spell a new word. 0 table tamale 20070513 I'm going to give you some words. For each one, change one of the letters in the word to M-A to spell a new word. 1 dishy dismay 20070513 I'm going to give you some words. For each one, change one of the letters in the word to M-A to spell a new word. 2 optic optima 20070513 I'm going to give you some words. For each one, change one of the letters in the word to M-A to spell a new word. 3 slice malice 20070513 I'm going to give you some words. For each one, change one of the letters in the word to M-A to spell a new word. 4 sucker smacker 20070513 I'm going to give you some words. For each one, change one of the letters in the word to M-A to spell a new word. 5 plunge plumage 20070513 I'm going to give you some words. For each one, change one of the letters in the word to M-A to spell a new word. 6 flagon flagman 20070513 I'm going to give you some words. For each one, change one of the letters in the word to M-A to spell a new word. 7 unkind mankind 20070513 I'm going to give you some words. For each one, change one of the letters in the word to M-A to spell a new word. 8 Nordic nomadic 20070513 I'm going to give you some words. For each one, change one of the letters in the word to M-A to spell a new word. 9 priory primary 20070513 I'm going to give you some words. For each one, change one of the letters in the word to M-A to spell a new word. 10 auteur amateur 20070513 I'm going to give you some words. For each one, change one of the letters in the word to M-A to spell a new word. 11 Schultz schmaltz 20070513 I'm going to give you some words. For each one, change one of the letters in the word to M-A to spell a new word. 12 rebinder remainder 20070513 I'm going to give you some words. For each one, change one of the letters in the word to M-A to spell a new word. 13 shopworn shopwoman 20090301 "Every clue is an anagram for the name of a Shakespeare character. For example, given, """"real,"""" the answer would be """"Lear.""""" 0 Moore Romeo 20090301 "Every clue is an anagram for the name of a Shakespeare character. For example, given, """"real,"""" the answer would be """"Lear.""""" 1 pairs Paris 20090301 "Every clue is an anagram for the name of a Shakespeare character. For example, given, """"real,"""" the answer would be """"Lear.""""" 2 ailer Ariel 20090301 "Every clue is an anagram for the name of a Shakespeare character. For example, given, """"real,"""" the answer would be """"Lear.""""" 3 anger Regan (King Lear's daughter) 20090301 "Every clue is an anagram for the name of a Shakespeare character. For example, given, """"real,"""" the answer would be """"Lear.""""" 4 Thelma Hamlet 20090301 "Every clue is an anagram for the name of a Shakespeare character. For example, given, """"real,"""" the answer would be """"Lear.""""" 5 ascare Caeser 20090301 "Every clue is an anagram for the name of a Shakespeare character. For example, given, """"real,"""" the answer would be """"Lear.""""" 6 tropia Portia 20090301 "Every clue is an anagram for the name of a Shakespeare character. For example, given, """"real,"""" the answer would be """"Lear.""""" 7 stealer Laertes 20090301 "Every clue is an anagram for the name of a Shakespeare character. For example, given, """"real,"""" the answer would be """"Lear.""""" 8 proposer Prospero 20090301 "Every clue is an anagram for the name of a Shakespeare character. For example, given, """"real,"""" the answer would be """"Lear.""""" 9 ancestral Lancaster 20080706 You are given two things from the same category. Name a third thing that's in the same category and falls between the two things alphabetically. For example, given Psycho and Rebecca, the answer would be Rear Window, because all three are Hitchcock films. 0 Harrison, McCartney Lennon (John) members of The Beatles 20080706 You are given two things from the same category. Name a third thing that's in the same category and falls between the two things alphabetically. For example, given Psycho and Rebecca, the answer would be Rear Window, because all three are Hitchcock films. 1 orange, violet redcolors of the rainbow 20080706 You are given two things from the same category. Name a third thing that's in the same category and falls between the two things alphabetically. For example, given Psycho and Rebecca, the answer would be Rear Window, because all three are Hitchcock films. 2 Earth, Mars Jupiterplanets 20080706 You are given two things from the same category. Name a third thing that's in the same category and falls between the two things alphabetically. For example, given Psycho and Rebecca, the answer would be Rear Window, because all three are Hitchcock films. 3 pawn, rook queenchess pieces 20080706 You are given two things from the same category. Name a third thing that's in the same category and falls between the two things alphabetically. For example, given Psycho and Rebecca, the answer would be Rear Window, because all three are Hitchcock films. 4 Malachi, Matthew Markbooks of the Bible 20080706 You are given two things from the same category. Name a third thing that's in the same category and falls between the two things alphabetically. For example, given Psycho and Rebecca, the answer would be Rear Window, because all three are Hitchcock films. 5 Atlanta, Austin Auguststate capitals 20080706 You are given two things from the same category. Name a third thing that's in the same category and falls between the two things alphabetically. For example, given Psycho and Rebecca, the answer would be Rear Window, because all three are Hitchcock films. 6 Minnesota, Ohio State Northwestern Big Ten schools 20080706 You are given two things from the same category. Name a third thing that's in the same category and falls between the two things alphabetically. For example, given Psycho and Rebecca, the answer would be Rear Window, because all three are Hitchcock films. 7 pitcher, second base right fieldbaseball positions 20080907 "You are given clues that end in a six-letter word. Rearrange the letters in the last word to get the answer to the clue. For example, given """"Jewels a pirate buries,"""" the answer would be """"rubies.""""" 0 turkey part that may be moistened with a baster breast 20080907 "You are given clues that end in a six-letter word. Rearrange the letters in the last word to get the answer to the clue. For example, given """"Jewels a pirate buries,"""" the answer would be """"rubies.""""" 1 when a bridge is out, where traffic is routed detour 20080907 "You are given clues that end in a six-letter word. Rearrange the letters in the last word to get the answer to the clue. For example, given """"Jewels a pirate buries,"""" the answer would be """"rubies.""""" 2 important member of a religious circle cleric 20080907 "You are given clues that end in a six-letter word. Rearrange the letters in the last word to get the answer to the clue. For example, given """"Jewels a pirate buries,"""" the answer would be """"rubies.""""" 3 You might need one to get through the military sector escort 20080907 "You are given clues that end in a six-letter word. Rearrange the letters in the last word to get the answer to the clue. For example, given """"Jewels a pirate buries,"""" the answer would be """"rubies.""""" 4 game for bridge haters hearts 20080907 "You are given clues that end in a six-letter word. Rearrange the letters in the last word to get the answer to the clue. For example, given """"Jewels a pirate buries,"""" the answer would be """"rubies.""""" 5 big holiday for St. Teresa Easter 20080907 "You are given clues that end in a six-letter word. Rearrange the letters in the last word to get the answer to the clue. For example, given """"Jewels a pirate buries,"""" the answer would be """"rubies.""""" 6 what you cannot do to something that is silent listen 20080907 "You are given clues that end in a six-letter word. Rearrange the letters in the last word to get the answer to the clue. For example, given """"Jewels a pirate buries,"""" the answer would be """"rubies.""""" 7 rock whose chances of hitting the Earth are remote meteor 20080907 "You are given clues that end in a six-letter word. Rearrange the letters in the last word to get the answer to the clue. For example, given """"Jewels a pirate buries,"""" the answer would be """"rubies.""""" 8 Island a thousand miles east northeast of Bogota Tobago 20080907 "You are given clues that end in a six-letter word. Rearrange the letters in the last word to get the answer to the clue. For example, given """"Jewels a pirate buries,"""" the answer would be """"rubies.""""" 9 "what someone might be who posts a sign, ""quiet please""" asleep 20080803 In the on-air puzzle, you are given clues to two words, each with two syllables. The vowel sound in the first syllable of the first word is a long A. Change that vowel to a short I, and phonetically you will get a new word that answers the second clue. 1 William Penn, for example; more astute Quaker, quicker 20080803 In the on-air puzzle, you are given clues to two words, each with two syllables. The vowel sound in the first syllable of the first word is a long A. Change that vowel to a short I, and phonetically you will get a new word that answers the second clue. 2 occurring every 24 hours; lulu daily, dilly 20080803 In the on-air puzzle, you are given clues to two words, each with two syllables. The vowel sound in the first syllable of the first word is a long A. Change that vowel to a short I, and phonetically you will get a new word that answers the second clue. 3 a worker who makes bread; to argue baker, bicker 20080803 In the on-air puzzle, you are given clues to two words, each with two syllables. The vowel sound in the first syllable of the first word is a long A. Change that vowel to a short I, and phonetically you will get a new word that answers the second clue. 4 heavy; clever weighty, witty 20080803 In the on-air puzzle, you are given clues to two words, each with two syllables. The vowel sound in the first syllable of the first word is a long A. Change that vowel to a short I, and phonetically you will get a new word that answers the second clue. 5 taste; 1920's car, informally flavor, Flivver 20080803 In the on-air puzzle, you are given clues to two words, each with two syllables. The vowel sound in the first syllable of the first word is a long A. Change that vowel to a short I, and phonetically you will get a new word that answers the second clue. 6 procrastinator's word; trash later, litter 20080803 In the on-air puzzle, you are given clues to two words, each with two syllables. The vowel sound in the first syllable of the first word is a long A. Change that vowel to a short I, and phonetically you will get a new word that answers the second clue. 7 to rouse from sleep; a witch waken, Wiccan 20080803 In the on-air puzzle, you are given clues to two words, each with two syllables. The vowel sound in the first syllable of the first word is a long A. Change that vowel to a short I, and phonetically you will get a new word that answers the second clue. 8 watch brand; a person who is mentallyderanged and does bad things Seiko, sicko 20080803 In the on-air puzzle, you are given clues to two words, each with two syllables. The vowel sound in the first syllable of the first word is a long A. Change that vowel to a short I, and phonetically you will get a new word that answers the second clue. 9 banality; quiet staleness, stillness 20080803 In the on-air puzzle, you are given clues to two words, each with two syllables. The vowel sound in the first syllable of the first word is a long A. Change that vowel to a short I, and phonetically you will get a new word that answers the second clue. 10 Italian city; parts of baby bottles Naples, nipples 20090823 "From two given four-letter words, rearrange the letters of one of them to get a synonym of the other. For example, given """"each"""" and """"pain,"""" the answer is """"ache,"""" because """"ache"""" is an anagram of """"each,"""" and it means """"pain.""""" 0 peel, grin ring 20090823 "From two given four-letter words, rearrange the letters of one of them to get a synonym of the other. For example, given """"each"""" and """"pain,"""" the answer is """"ache,"""" because """"ache"""" is an anagram of """"each,"""" and it means """"pain.""""" 1 bare, dune nude 20090823 "From two given four-letter words, rearrange the letters of one of them to get a synonym of the other. For example, given """"each"""" and """"pain,"""" the answer is """"ache,"""" because """"ache"""" is an anagram of """"each,"""" and it means """"pain.""""" 2 gear, fury rage 20090823 "From two given four-letter words, rearrange the letters of one of them to get a synonym of the other. For example, given """"each"""" and """"pain,"""" the answer is """"ache,"""" because """"ache"""" is an anagram of """"each,"""" and it means """"pain.""""" 3 burn, arch char 20090823 "From two given four-letter words, rearrange the letters of one of them to get a synonym of the other. For example, given """"each"""" and """"pain,"""" the answer is """"ache,"""" because """"ache"""" is an anagram of """"each,"""" and it means """"pain.""""" 4 oven, link kiln 20090823 "From two given four-letter words, rearrange the letters of one of them to get a synonym of the other. For example, given """"each"""" and """"pain,"""" the answer is """"ache,"""" because """"ache"""" is an anagram of """"each,"""" and it means """"pain.""""" 5 felt, gone left 20090823 "From two given four-letter words, rearrange the letters of one of them to get a synonym of the other. For example, given """"each"""" and """"pain,"""" the answer is """"ache,"""" because """"ache"""" is an anagram of """"each,"""" and it means """"pain.""""" 6 info, sewn news 20090823 "From two given four-letter words, rearrange the letters of one of them to get a synonym of the other. For example, given """"each"""" and """"pain,"""" the answer is """"ache,"""" because """"ache"""" is an anagram of """"each,"""" and it means """"pain.""""" 7 rove, atop over 20090823 "From two given four-letter words, rearrange the letters of one of them to get a synonym of the other. For example, given """"each"""" and """"pain,"""" the answer is """"ache,"""" because """"ache"""" is an anagram of """"each,"""" and it means """"pain.""""" 8 rapt, role part 20090823 "From two given four-letter words, rearrange the letters of one of them to get a synonym of the other. For example, given """"each"""" and """"pain,"""" the answer is """"ache,"""" because """"ache"""" is an anagram of """"each,"""" and it means """"pain.""""" 9 whip, golf flog 20090823 "From two given four-letter words, rearrange the letters of one of them to get a synonym of the other. For example, given """"each"""" and """"pain,"""" the answer is """"ache,"""" because """"ache"""" is an anagram of """"each,"""" and it means """"pain.""""" 10 thin, clue hint 20080120 In the on-air puzzle, you are given a word and must drop two letters so that the remaining letters, in order, spell a color or shade. 0 train tan, not tin 20080120 In the on-air puzzle, you are given a word and must drop two letters so that the remaining letters, in order, spell a color or shade. 1 blouse blue 20080120 In the on-air puzzle, you are given a word and must drop two letters so that the remaining letters, in order, spell a color or shade. 2 grouse rose 20080120 In the on-air puzzle, you are given a word and must drop two letters so that the remaining letters, in order, spell a color or shade. 3 greasy grey or gray 20080120 In the on-air puzzle, you are given a word and must drop two letters so that the remaining letters, in order, spell a color or shade. 4 age-old gold 20080120 In the on-air puzzle, you are given a word and must drop two letters so that the remaining letters, in order, spell a color or shade. 5 pucker puce 20080120 In the on-air puzzle, you are given a word and must drop two letters so that the remaining letters, in order, spell a color or shade. 6 reborn ebon 20080120 In the on-air puzzle, you are given a word and must drop two letters so that the remaining letters, in order, spell a color or shade. 7 grubby ruby 20080120 In the on-air puzzle, you are given a word and must drop two letters so that the remaining letters, in order, spell a color or shade. 8 cordial coral 20080120 In the on-air puzzle, you are given a word and must drop two letters so that the remaining letters, in order, spell a color or shade. 9 Clemson lemon 20080120 In the on-air puzzle, you are given a word and must drop two letters so that the remaining letters, in order, spell a color or shade. 10 outlive olive 20080120 In the on-air puzzle, you are given a word and must drop two letters so that the remaining letters, in order, spell a color or shade. 11 whither white 20080120 In the on-air puzzle, you are given a word and must drop two letters so that the remaining letters, in order, spell a color or shade. 13 greyhen green 20080120 In the on-air puzzle, you are given a word and must drop two letters so that the remaining letters, in order, spell a color or shade. 14 poacher ocher 20080120 In the on-air puzzle, you are given a word and must drop two letters so that the remaining letters, in order, spell a color or shade. 15 macaroon maroon 20080120 In the on-air puzzle, you are given a word and must drop two letters so that the remaining letters, in order, spell a color or shade. 16 outrange orange 20100314 "This game involves anagrams, and every word contains the letters A, B and C. For example, use A-B-C along with the letters L, O and T to get a type of metal. The answer is """"cobalt.""""" 0 ABC + one lighthousebeacon 20100314 "This game involves anagrams, and every word contains the letters A, B and C. For example, use A-B-C along with the letters L, O and T to get a type of metal. The answer is """"cobalt.""""" 1 ABC + tom fightingcombat 20100314 "This game involves anagrams, and every word contains the letters A, B and C. For example, use A-B-C along with the letters L, O and T to get a type of metal. The answer is """"cobalt.""""" 2 ABC + fir material for clothingfabric 20100314 "This game involves anagrams, and every word contains the letters A, B and C. For example, use A-B-C along with the letters L, O and T to get a type of metal. The answer is """"cobalt.""""" 3 ABC + rode material for draperybrocade 20100314 "This game involves anagrams, and every word contains the letters A, B and C. For example, use A-B-C along with the letters L, O and T to get a type of metal. The answer is """"cobalt.""""" 4 ABC + Lyon setting in Romeo and Julietbalcony 20100314 "This game involves anagrams, and every word contains the letters A, B and C. For example, use A-B-C along with the letters L, O and T to get a type of metal. The answer is """"cobalt.""""" 5 ABC + tine place to store thingscabinet 20100314 "This game involves anagrams, and every word contains the letters A, B and C. For example, use A-B-C along with the letters L, O and T to get a type of metal. The answer is """"cobalt.""""" 6 ABC + proud place to store thingscupboard 20100314 "This game involves anagrams, and every word contains the letters A, B and C. For example, use A-B-C along with the letters L, O and T to get a type of metal. The answer is """"cobalt.""""" 7 ABC + holer unmarried manbachelor 20100314 "This game involves anagrams, and every word contains the letters A, B and C. For example, use A-B-C along with the letters L, O and T to get a type of metal. The answer is """"cobalt.""""" 8 ABC + foils some glassesbifocals 20100314 "This game involves anagrams, and every word contains the letters A, B and C. For example, use A-B-C along with the letters L, O and T to get a type of metal. The answer is """"cobalt.""""" 9 ABC + creak seafood appetizer orcrabcake 20100314 "This game involves anagrams, and every word contains the letters A, B and C. For example, use A-B-C along with the letters L, O and T to get a type of metal. The answer is """"cobalt.""""" 10 ABC + censure NFL football team Buccaneers 20070909 "In this week's on-air puzzle, every answer is a six-letter word that can be broken into two consecutive three-letter words to answer the clues. For example, if the question is """"What body part is a number plus a boy's name,"""" the answer would be """"tendon,"""" which includes """"ten"""" and """"don.""""" 1 What bird consists of a golf score plus nonsense?What kind of puzzle is a dance plus a carpenter's tool? parrot jigsaw 20070909 "In this week's on-air puzzle, every answer is a six-letter word that can be broken into two consecutive three-letter words to answer the clues. For example, if the question is """"What body part is a number plus a boy's name,"""" the answer would be """"tendon,"""" which includes """"ten"""" and """"don.""""" 2 What fabric is a place to sleep plus a heavy weight? cotton 20070909 "In this week's on-air puzzle, every answer is a six-letter word that can be broken into two consecutive three-letter words to answer the clues. For example, if the question is """"What body part is a number plus a boy's name,"""" the answer would be """"tendon,"""" which includes """"ten"""" and """"don.""""" 3 What planet consists of the abbreviation for a weekdayplus a container? Saturn 20070909 "In this week's on-air puzzle, every answer is a six-letter word that can be broken into two consecutive three-letter words to answer the clues. For example, if the question is """"What body part is a number plus a boy's name,"""" the answer would be """"tendon,"""" which includes """"ten"""" and """"don.""""" 4 What animal consists of the abbreviation for a weekdayplus an item on a chain? monkey 20070909 "In this week's on-air puzzle, every answer is a six-letter word that can be broken into two consecutive three-letter words to answer the clues. For example, if the question is """"What body part is a number plus a boy's name,"""" the answer would be """"tendon,"""" which includes """"ten"""" and """"don.""""" 5 What northeast river consists of a Paul Newman filmplus a member of the family? Hudson 20070909 "In this week's on-air puzzle, every answer is a six-letter word that can be broken into two consecutive three-letter words to answer the clues. For example, if the question is """"What body part is a number plus a boy's name,"""" the answer would be """"tendon,"""" which includes """"ten"""" and """"don.""""" 6 What item to take on vacation consists of a car enginepart plus a (long) period of time? camera 20070909 "In this week's on-air puzzle, every answer is a six-letter word that can be broken into two consecutive three-letter words to answer the clues. For example, if the question is """"What body part is a number plus a boy's name,"""" the answer would be """"tendon,"""" which includes """"ten"""" and """"don.""""" 7 What zodiac symbol (not a sign) consists of a curvedgeometric line plus a pronoun? archer 20070909 "In this week's on-air puzzle, every answer is a six-letter word that can be broken into two consecutive three-letter words to answer the clues. For example, if the question is """"What body part is a number plus a boy's name,"""" the answer would be """"tendon,"""" which includes """"ten"""" and """"don.""""" 8 What form of writing consists of a famous writer plusan attempt? poetry 20071230 Will Shortz names people who became famous during the past twelve months, whom you probably never heard of before 2007. The player tells why they're famous. 0 Lisa Nowakcross-country ria romantic rival NASAde to confrontAstronaut who took a 20071230 Will Shortz names people who became famous during the past twelve months, whom you probably never heard of before 2007. The player tells why they're famous. 1 Nicolas Sarkozy New President of France 20071230 Will Shortz names people who became famous during the past twelve months, whom you probably never heard of before 2007. The player tells why they're famous. 2 Sanjaya Malakarsixth season of notwithstanding from the judges seventh place finisher in theAmerican Idol,low ratings 20071230 Will Shortz names people who became famous during the past twelve months, whom you probably never heard of before 2007. The player tells why they're famous. 3 Joey Chestnut 66 in 12won the 92nd Annual Nathan's 20071230 Will Shortz names people who became famous during the past twelve months, whom you probably never heard of before 2007. The player tells why they're famous. 4 "minutessix-time defendiTakeru ""Tsunamiconsuming 66 HDBbuns) in 12 minu" "Hot Dog Eating Contest, beatingng champion"" Kobayashi bys (hot dogs andtes" 20071230 Will Shortz names people who became famous during the past twelve months, whom you probably never heard of before 2007. The player tells why they're famous. 5 Mark Ecko $750,000, heTo determine the fate of the 20071230 Will Shortz names people who became famous during the past twelve months, whom you probably never heard of before 2007. The player tells why they're famous. 6 paidwww.vote756.com visitors vote onthink should happage visitors voball to the Hallbranding the balasterisk. ball, he started a website,[1], which let what theypen to it. Webted to give the of Fame,l with an 20071230 Will Shortz names people who became famous during the past twelve months, whom you probably never heard of before 2007. The player tells why they're famous. 7 Michael Mukasey new U.S. Attorney General 20071230 Will Shortz names people who became famous during the past twelve months, whom you probably never heard of before 2007. The player tells why they're famous. 8 Gillian Gibbonsafter students vMuhammad school teacheroted to name itarrested in Sudan for blasphemy 20071230 Will Shortz names people who became famous during the past twelve months, whom you probably never heard of before 2007. The player tells why they're famous. 9 Andrew Meyer "student,Said ""Don't tase me, bro!"" when" 20071230 Will Shortz names people who became famous during the past twelve months, whom you probably never heard of before 2007. The player tells why they're famous. 10 University of arrested (and then tased) at 20071230 Will Shortz names people who became famous during the past twelve months, whom you probably never heard of before 2007. The player tells why they're famous. 11 Florida atGainesville the end of a John Kerry forum. 20071230 Will Shortz names people who became famous during the past twelve months, whom you probably never heard of before 2007. The player tells why they're famous. 12 Charles Simonyi paid over $20Paid for Soyuz ride to the 20071230 Will Shortz names people who became famous during the past twelve months, whom you probably never heard of before 2007. The player tells why they're famous. 13 million International Space Station. 20071230 Will Shortz names people who became famous during the past twelve months, whom you probably never heard of before 2007. The player tells why they're famous. 14 Troublefrom Leona Helmshuman grandchildout of the will Maltese dogley after tworen were leftinhereted $12 million trust 20090419 "Every answer is the name of a popular magazine. Name the title of the magazine from the anagram. For example, given """"weird,"""" the answer would be """"Wired.""""" 0 item Time 20090419 "Every answer is the name of a popular magazine. Name the title of the magazine from the anagram. For example, given """"weird,"""" the answer would be """"Wired.""""" 1 Laurel Allure 20090419 "Every answer is the name of a popular magazine. Name the title of the magazine from the anagram. For example, given """"weird,"""" the answer would be """"Wired.""""" 2 adrape Parade 20090419 "Every answer is the name of a popular magazine. Name the title of the magazine from the anagram. For example, given """"weird,"""" the answer would be """"Wired.""""" 3 queries Esquire 20090419 "Every answer is the name of a popular magazine. Name the title of the magazine from the anagram. For example, given """"weird,"""" the answer would be """"Wired.""""" 4 ten-four Fortune 20090419 "Every answer is the name of a popular magazine. Name the title of the magazine from the anagram. For example, given """"weird,"""" the answer would be """"Wired.""""" 5 sharper Harpers 20090419 "Every answer is the name of a popular magazine. Name the title of the magazine from the anagram. For example, given """"weird,"""" the answer would be """"Wired.""""" 6 brooked Redbook 20090419 "Every answer is the name of a popular magazine. Name the title of the magazine from the anagram. For example, given """"weird,"""" the answer would be """"Wired.""""" 7 entraps Parents 20090419 "Every answer is the name of a popular magazine. Name the title of the magazine from the anagram. For example, given """"weird,"""" the answer would be """"Wired.""""" 8 senesce Essence 20090419 "Every answer is the name of a popular magazine. Name the title of the magazine from the anagram. For example, given """"weird,"""" the answer would be """"Wired.""""" 9 modern tort Motor Trend 20070708 "In the on-air puzzle, clues are given to two words. If you add ""C-O"" in front of the first word, you'll get a new word that is completely unrelated to the first one." 1 animal doc; desire vet, covet 20070708 "In the on-air puzzle, clues are given to two words. If you add ""C-O"" in front of the first word, you'll get a new word that is completely unrelated to the first one." 2 voting district; one who turns tail ward, coward 20070708 "In the on-air puzzle, clues are given to two words. If you add ""C-O"" in front of the first word, you'll get a new word that is completely unrelated to the first one." 3 masked animal, for short; where a butterflycomes from coon, cocoon 20070708 "In the on-air puzzle, clues are given to two words. If you add ""C-O"" in front of the first word, you'll get a new word that is completely unrelated to the first one." 4 encountered; a sight in the night sky met, comet 20070708 "In the on-air puzzle, clues are given to two words. If you add ""C-O"" in front of the first word, you'll get a new word that is completely unrelated to the first one." 5 a person from Tulsa, informally; a treat froma bakery Okie, cookie 20070708 "In the on-air puzzle, clues are given to two words. If you add ""C-O"" in front of the first word, you'll get a new word that is completely unrelated to the first one." 6 resting atop; a means for getting money off agrocery item upon, coupon 20070708 "In the on-air puzzle, clues are given to two words. If you add ""C-O"" in front of the first word, you'll get a new word that is completely unrelated to the first one." 7 Michael who won two Oscars; a habit ofSherlock Holmes Caine, cocaine 20070708 "In the on-air puzzle, clues are given to two words. If you add ""C-O"" in front of the first word, you'll get a new word that is completely unrelated to the first one." 8 a cry of disgust; a flu symptom ugh, cough 20070708 "In the on-air puzzle, clues are given to two words. If you add ""C-O"" in front of the first word, you'll get a new word that is completely unrelated to the first one." 9 lady's partner; convincing gent, cogent 20070708 "In the on-air puzzle, clues are given to two words. If you add ""C-O"" in front of the first word, you'll get a new word that is completely unrelated to the first one." 10 a person from Latvia, Lithuania or Estonia; a Balt (or Baltic), 20070708 "In the on-air puzzle, clues are given to two words. If you add ""C-O"" in front of the first word, you'll get a new word that is completely unrelated to the first one." 11 metal that is number 27 on the periodic table cobalt 20070708 "In the on-air puzzle, clues are given to two words. If you add ""C-O"" in front of the first word, you'll get a new word that is completely unrelated to the first one." 12 like monks; plentiful pious, copious 20070708 "In the on-air puzzle, clues are given to two words. If you add ""C-O"" in front of the first word, you'll get a new word that is completely unrelated to the first one." 13 a strip down the middle of a highway; a funnyguy median, comedian 20080622 "You are given two words. Change one letter in each of them to make two new words that rhyme. For example, given """"pulse"""" and """"verge,"""" the answer would be """"purse"""" and """"verse.""""" 0 crown, prude croon, prune 20080622 "You are given two words. Change one letter in each of them to make two new words that rhyme. For example, given """"pulse"""" and """"verge,"""" the answer would be """"purse"""" and """"verse.""""" 1 chance, mango change, mange 20080622 "You are given two words. Change one letter in each of them to make two new words that rhyme. For example, given """"pulse"""" and """"verge,"""" the answer would be """"purse"""" and """"verse.""""" 2 suits, green suite, Greek 20080622 "You are given two words. Change one letter in each of them to make two new words that rhyme. For example, given """"pulse"""" and """"verge,"""" the answer would be """"purse"""" and """"verse.""""" 3 heart, worn heard, word 20080622 "You are given two words. Change one letter in each of them to make two new words that rhyme. For example, given """"pulse"""" and """"verge,"""" the answer would be """"purse"""" and """"verse.""""" 4 theme, flail there, flair 20080622 "You are given two words. Change one letter in each of them to make two new words that rhyme. For example, given """"pulse"""" and """"verge,"""" the answer would be """"purse"""" and """"verse.""""" 5 boughs, tact bought, taut 20080622 "You are given two words. Change one letter in each of them to make two new words that rhyme. For example, given """"pulse"""" and """"verge,"""" the answer would be """"purse"""" and """"verse.""""" 6 tartan, chapter tartar, charter 20071209 In this week's on-air puzzle, every answer is a familiar two-word phrase or name, in which every word contains the consecutive letters R-I. For example: What a person who might fail an FBI background check might pose. The answer would be: secuRIty RIsk. 0 a black person who lives in the United States African American 20071209 In this week's on-air puzzle, every answer is a familiar two-word phrase or name, in which every word contains the consecutive letters R-I. For example: What a person who might fail an FBI background check might pose. The answer would be: secuRIty RIsk. 1 co-discoverer of radium Marie Curie 20071209 In this week's on-air puzzle, every answer is a familiar two-word phrase or name, in which every word contains the consecutive letters R-I. For example: What a person who might fail an FBI background check might pose. The answer would be: secuRIty RIsk. 2 a power tool with a bit electric drill 20071209 In this week's on-air puzzle, every answer is a familiar two-word phrase or name, in which every word contains the consecutive letters R-I. For example: What a person who might fail an FBI background check might pose. The answer would be: secuRIty RIsk. 3 tributary of the Mississippi explored byLewis and Clark Missouri River 20071209 In this week's on-air puzzle, every answer is a familiar two-word phrase or name, in which every word contains the consecutive letters R-I. For example: What a person who might fail an FBI background check might pose. The answer would be: secuRIty RIsk. 4 lead dancer in a tutu prima ballerina 20071209 In this week's on-air puzzle, every answer is a familiar two-word phrase or name, in which every word contains the consecutive letters R-I. For example: What a person who might fail an FBI background check might pose. The answer would be: secuRIty RIsk. 5 geometrical figure that may have sides oflength 3, 4 and 5 right triangle 20071209 In this week's on-air puzzle, every answer is a familiar two-word phrase or name, in which every word contains the consecutive letters R-I. For example: What a person who might fail an FBI background check might pose. The answer would be: secuRIty RIsk. 6 Russian who was the first man in space Yuri Gagarin 20071209 In this week's on-air puzzle, every answer is a familiar two-word phrase or name, in which every word contains the consecutive letters R-I. For example: What a person who might fail an FBI background check might pose. The answer would be: secuRIty RIsk. 7 breed of dog from the land of Dublin, havinga wiry coat Irish Terrier 20071209 In this week's on-air puzzle, every answer is a familiar two-word phrase or name, in which every word contains the consecutive letters R-I. For example: What a person who might fail an FBI background check might pose. The answer would be: secuRIty RIsk. 8 breakfast cereal that goes snap, crackle, pop Rice Krispies 20071209 In this week's on-air puzzle, every answer is a familiar two-word phrase or name, in which every word contains the consecutive letters R-I. For example: What a person who might fail an FBI background check might pose. The answer would be: secuRIty RIsk. 9 capitalism (contrasting with the publicownership of communism) private enterprise 20071209 In this week's on-air puzzle, every answer is a familiar two-word phrase or name, in which every word contains the consecutive letters R-I. For example: What a person who might fail an FBI background check might pose. The answer would be: secuRIty RIsk. 10 illegal reproduction of protected textinfringement copyright 20071209 In this week's on-air puzzle, every answer is a familiar two-word phrase or name, in which every word contains the consecutive letters R-I. For example: What a person who might fail an FBI background check might pose. The answer would be: secuRIty RIsk. 11 a hod brick carrier 20071209 In this week's on-air puzzle, every answer is a familiar two-word phrase or name, in which every word contains the consecutive letters R-I. For example: What a person who might fail an FBI background check might pose. The answer would be: secuRIty RIsk. 12 something on a spool on a Smith Corona typewriter ribbon 20071209 In this week's on-air puzzle, every answer is a familiar two-word phrase or name, in which every word contains the consecutive letters R-I. For example: What a person who might fail an FBI background check might pose. The answer would be: secuRIty RIsk. 13 line that passes through Greenwich, England Prime Meridian 20071209 In this week's on-air puzzle, every answer is a familiar two-word phrase or name, in which every word contains the consecutive letters R-I. For example: What a person who might fail an FBI background check might pose. The answer would be: secuRIty RIsk. 14 another name for Santa Claus (starts with K) Kris Kringle 20090322 "Each clue is three words. For each set, think of a fourth word that can follow each clue to complete a compound word or familiar two-word phrase. For example, given """"cat,"""" """"cattle"""" and """"telephone,"""" the answer would be """"call,"""" as in """"catcall,"""" """"cattle call"""" and """"telephone call."""" Hint: Each answer ends in two L's, like """"call.""""" 0 sleeping, pain, chill pill 20090322 "Each clue is three words. For each set, think of a fourth word that can follow each clue to complete a compound word or familiar two-word phrase. For example, given """"cat,"""" """"cattle"""" and """"telephone,"""" the answer would be """"call,"""" as in """"catcall,"""" """"cattle call"""" and """"telephone call."""" Hint: Each answer ends in two L's, like """"call.""""" 1 cow, bar, dumb bell 20090322 "Each clue is three words. For each set, think of a fourth word that can follow each clue to complete a compound word or familiar two-word phrase. For example, given """"cat,"""" """"cattle"""" and """"telephone,"""" the answer would be """"call,"""" as in """"catcall,"""" """"cattle call"""" and """"telephone call."""" Hint: Each answer ends in two L's, like """"call.""""" 2 tennis, cannon, screw ball 20090322 "Each clue is three words. For each set, think of a fourth word that can follow each clue to complete a compound word or familiar two-word phrase. For example, given """"cat,"""" """"cattle"""" and """"telephone,"""" the answer would be """"call,"""" as in """"catcall,"""" """"cattle call"""" and """"telephone call."""" Hint: Each answer ends in two L's, like """"call.""""" 3 bomb, egg, tortoise shell 20090322 "Each clue is three words. For each set, think of a fourth word that can follow each clue to complete a compound word or familiar two-word phrase. For example, given """"cat,"""" """"cattle"""" and """"telephone,"""" the answer would be """"call,"""" as in """"catcall,"""" """"cattle call"""" and """"telephone call."""" Hint: Each answer ends in two L's, like """"call.""""" 4 duck, play, Buffalo bill 20090322 "Each clue is three words. For each set, think of a fourth word that can follow each clue to complete a compound word or familiar two-word phrase. For example, given """"cat,"""" """"cattle"""" and """"telephone,"""" the answer would be """"call,"""" as in """"catcall,"""" """"cattle call"""" and """"telephone call."""" Hint: Each answer ends in two L's, like """"call.""""" 5 baby, rag, china doll 20090322 "Each clue is three words. For each set, think of a fourth word that can follow each clue to complete a compound word or familiar two-word phrase. For example, given """"cat,"""" """"cattle"""" and """"telephone,"""" the answer would be """"call,"""" as in """"catcall,"""" """"cattle call"""" and """"telephone call."""" Hint: Each answer ends in two L's, like """"call.""""" 6 ant, boot, Capitol hill 20090322 "Each clue is three words. For each set, think of a fourth word that can follow each clue to complete a compound word or familiar two-word phrase. For example, given """"cat,"""" """"cattle"""" and """"telephone,"""" the answer would be """"call,"""" as in """"catcall,"""" """"cattle call"""" and """"telephone call."""" Hint: Each answer ends in two L's, like """"call.""""" 7 ink, stair, oil well 20090322 "Each clue is three words. For each set, think of a fourth word that can follow each clue to complete a compound word or familiar two-word phrase. For example, given """"cat,"""" """"cattle"""" and """"telephone,"""" the answer would be """"call,"""" as in """"catcall,"""" """"cattle call"""" and """"telephone call."""" Hint: Each answer ends in two L's, like """"call.""""" 8 bank, drum, jelly roll 20090322 "Each clue is three words. For each set, think of a fourth word that can follow each clue to complete a compound word or familiar two-word phrase. For example, given """"cat,"""" """"cattle"""" and """"telephone,"""" the answer would be """"call,"""" as in """"catcall,"""" """"cattle call"""" and """"telephone call."""" Hint: Each answer ends in two L's, like """"call.""""" 9 water, wind, free fall 20090322 "Each clue is three words. For each set, think of a fourth word that can follow each clue to complete a compound word or familiar two-word phrase. For example, given """"cat,"""" """"cattle"""" and """"telephone,"""" the answer would be """"call,"""" as in """"catcall,"""" """"cattle call"""" and """"telephone call."""" Hint: Each answer ends in two L's, like """"call.""""" 10 fire, dentists', power drill 20090322 "Each clue is three words. For each set, think of a fourth word that can follow each clue to complete a compound word or familiar two-word phrase. For example, given """"cat,"""" """"cattle"""" and """"telephone,"""" the answer would be """"call,"""" as in """"catcall,"""" """"cattle call"""" and """"telephone call."""" Hint: Each answer ends in two L's, like """"call.""""" 11 fire, stone, white wall 20090322 "Each clue is three words. For each set, think of a fourth word that can follow each clue to complete a compound word or familiar two-word phrase. For example, given """"cat,"""" """"cattle"""" and """"telephone,"""" the answer would be """"call,"""" as in """"catcall,"""" """"cattle call"""" and """"telephone call."""" Hint: Each answer ends in two L's, like """"call.""""" 12 pepper, paper, tread mill 20090322 "Each clue is three words. For each set, think of a fourth word that can follow each clue to complete a compound word or familiar two-word phrase. For example, given """"cat,"""" """"cattle"""" and """"telephone,"""" the answer would be """"call,"""" as in """"catcall,"""" """"cattle call"""" and """"telephone call."""" Hint: Each answer ends in two L's, like """"call.""""" 13 jail, fuel, brain cell 20070930 "In this week's on-air Puzzle, every answer is the name of a famous writer. Given rhymes for the first and last names, you name the writers. For example: given """"Wet Start,"""" you'd say """"Bret Harte.""""" 0 dark stain (19th century) Mark Twain 20070930 "In this week's on-air Puzzle, every answer is the name of a famous writer. Given rhymes for the first and last names, you name the writers. For example: given """"Wet Start,"""" you'd say """"Bret Harte.""""" 1 Mormon sailor Norman Mailer 20070930 "In this week's on-air Puzzle, every answer is the name of a famous writer. Given rhymes for the first and last names, you name the writers. For example: given """"Wet Start,"""" you'd say """"Bret Harte.""""" 2 plain Boston Jane Austen 20070930 "In this week's on-air Puzzle, every answer is the name of a famous writer. Given rhymes for the first and last names, you name the writers. For example: given """"Wet Start,"""" you'd say """"Bret Harte.""""" 3 dame's voice James Joyce 20070930 "In this week's on-air Puzzle, every answer is the name of a famous writer. Given rhymes for the first and last names, you name the writers. For example: given """"Wet Start,"""" you'd say """"Bret Harte.""""" 4 villain promise Dylan Thomas 20070930 "In this week's on-air Puzzle, every answer is the name of a famous writer. Given rhymes for the first and last names, you name the writers. For example: given """"Wet Start,"""" you'd say """"Bret Harte.""""" 5 sharper key Harper Lee 20070930 "In this week's on-air Puzzle, every answer is the name of a famous writer. Given rhymes for the first and last names, you name the writers. For example: given """"Wet Start,"""" you'd say """"Bret Harte.""""" 6 villa blather Willa Cather 20070930 "In this week's on-air Puzzle, every answer is the name of a famous writer. Given rhymes for the first and last names, you name the writers. For example: given """"Wet Start,"""" you'd say """"Bret Harte.""""" 7 tall fellow Saul Bellow 20070930 "In this week's on-air Puzzle, every answer is the name of a famous writer. Given rhymes for the first and last names, you name the writers. For example: given """"Wet Start,"""" you'd say """"Bret Harte.""""" 8 yon beavernot Tom Seaver John Cheever 20070930 "In this week's on-air Puzzle, every answer is the name of a famous writer. Given rhymes for the first and last names, you name the writers. For example: given """"Wet Start,"""" you'd say """"Bret Harte.""""" 9 tan mice Ann Rice 20070930 "In this week's on-air Puzzle, every answer is the name of a famous writer. Given rhymes for the first and last names, you name the writers. For example: given """"Wet Start,"""" you'd say """"Bret Harte.""""" 10 cycle titan Michael Chriton 20070930 "In this week's on-air Puzzle, every answer is the name of a famous writer. Given rhymes for the first and last names, you name the writers. For example: given """"Wet Start,"""" you'd say """"Bret Harte.""""" 11 mules turn Jules Verne 20070422 "The on-air puzzle this week is titled Shifting from One Phrase to Another. It asks for familiar three-word phrases. The middle word of each phrase is ""the."" The clues are words that rhyme with the first and last words. For example, given ""lift the frame,"" the answer would be ""shift the blame."" " 0 hollow the cedar follow the leader 20070422 "The on-air puzzle this week is titled Shifting from One Phrase to Another. It asks for familiar three-word phrases. The middle word of each phrase is ""the."" The clues are words that rhyme with the first and last words. For example, given ""lift the frame,"" the answer would be ""shift the blame."" " 1 cease the deals grease the wheels, not release the seals 20070422 "The on-air puzzle this week is titled Shifting from One Phrase to Another. It asks for familiar three-word phrases. The middle word of each phrase is ""the."" The clues are words that rhyme with the first and last words. For example, given ""lift the frame,"" the answer would be ""shift the blame."" " 2 sketch the tooth stretch the truth 20070422 "The on-air puzzle this week is titled Shifting from One Phrase to Another. It asks for familiar three-word phrases. The middle word of each phrase is ""the."" The clues are words that rhyme with the first and last words. For example, given ""lift the frame,"" the answer would be ""shift the blame."" " 3 split the throttle hit the bottle, not quit the bottle 20070422 "The on-air puzzle this week is titled Shifting from One Phrase to Another. It asks for familiar three-word phrases. The middle word of each phrase is ""the."" The clues are words that rhyme with the first and last words. For example, given ""lift the frame,"" the answer would be ""shift the blame."" " 4 mow the thistle blow the whistle 20070422 "The on-air puzzle this week is titled Shifting from One Phrase to Another. It asks for familiar three-word phrases. The middle word of each phrase is ""the."" The clues are words that rhyme with the first and last words. For example, given ""lift the frame,"" the answer would be ""shift the blame."" " 5 sweep the range keep the change 20070422 "The on-air puzzle this week is titled Shifting from One Phrase to Another. It asks for familiar three-word phrases. The middle word of each phrase is ""the."" The clues are words that rhyme with the first and last words. For example, given ""lift the frame,"" the answer would be ""shift the blame."" " 6 turn the scopes learn the ropes 20070422 "The on-air puzzle this week is titled Shifting from One Phrase to Another. It asks for familiar three-word phrases. The middle word of each phrase is ""the."" The clues are words that rhyme with the first and last words. For example, given ""lift the frame,"" the answer would be ""shift the blame."" " 7 pack the ship crack the whip 20070422 "The on-air puzzle this week is titled Shifting from One Phrase to Another. It asks for familiar three-word phrases. The middle word of each phrase is ""the."" The clues are words that rhyme with the first and last words. For example, given ""lift the frame,"" the answer would be ""shift the blame."" " 8 near the wreck clear the deck 20070422 "The on-air puzzle this week is titled Shifting from One Phrase to Another. It asks for familiar three-word phrases. The middle word of each phrase is ""the."" The clues are words that rhyme with the first and last words. For example, given ""lift the frame,"" the answer would be ""shift the blame."" " 9 feel the glow steal the show 20070422 "The on-air puzzle this week is titled Shifting from One Phrase to Another. It asks for familiar three-word phrases. The middle word of each phrase is ""the."" The clues are words that rhyme with the first and last words. For example, given ""lift the frame,"" the answer would be ""shift the blame."" " 10 cheat the doc beat the clock 20070422 "The on-air puzzle this week is titled Shifting from One Phrase to Another. It asks for familiar three-word phrases. The middle word of each phrase is ""the."" The clues are words that rhyme with the first and last words. For example, given ""lift the frame,"" the answer would be ""shift the blame."" " 11 block the vote rock the boat 20070422 "The on-air puzzle this week is titled Shifting from One Phrase to Another. It asks for familiar three-word phrases. The middle word of each phrase is ""the."" The clues are words that rhyme with the first and last words. For example, given ""lift the frame,"" the answer would be ""shift the blame."" " 12 fight the rust bite the dust 20070422 "The on-air puzzle this week is titled Shifting from One Phrase to Another. It asks for familiar three-word phrases. The middle word of each phrase is ""the."" The clues are words that rhyme with the first and last words. For example, given ""lift the frame,"" the answer would be ""shift the blame."" " 13 miss the guide kiss the bride 20090621 "You are given three four-letter words. Think of three letters that can precede each of them to complete familiar seven-letter words. For example, given """"each,"""" """"rove"""" and """"lode,"""" the answer would be """"imp"""" for """"impeach,"""" """"improve"""" and """"implode.""""" 0 riot, lice, grin cha 20090621 "You are given three four-letter words. Think of three letters that can precede each of them to complete familiar seven-letter words. For example, given """"each,"""" """"rove"""" and """"lode,"""" the answer would be """"imp"""" for """"impeach,"""" """"improve"""" and """"implode.""""" 1 gain, rack, rage bar 20090621 "You are given three four-letter words. Think of three letters that can precede each of them to complete familiar seven-letter words. For example, given """"each,"""" """"rove"""" and """"lode,"""" the answer would be """"imp"""" for """"impeach,"""" """"improve"""" and """"implode.""""" 2 mite, race, mini ter 20090621 "You are given three four-letter words. Think of three letters that can precede each of them to complete familiar seven-letter words. For example, given """"each,"""" """"rove"""" and """"lode,"""" the answer would be """"imp"""" for """"impeach,"""" """"improve"""" and """"implode.""""" 3 roof, lace, tile rep 20090621 "You are given three four-letter words. Think of three letters that can precede each of them to complete familiar seven-letter words. For example, given """"each,"""" """"rove"""" and """"lode,"""" the answer would be """"imp"""" for """"impeach,"""" """"improve"""" and """"implode.""""" 4 verb, pane, fuse pro 20090621 "You are given three four-letter words. Think of three letters that can precede each of them to complete familiar seven-letter words. For example, given """"each,"""" """"rove"""" and """"lode,"""" the answer would be """"imp"""" for """"impeach,"""" """"improve"""" and """"implode.""""" 5 soon, arch, eyed mon 20090621 "You are given three four-letter words. Think of three letters that can precede each of them to complete familiar seven-letter words. For example, given """"each,"""" """"rove"""" and """"lode,"""" the answer would be """"imp"""" for """"impeach,"""" """"improve"""" and """"implode.""""" 6 loon, last, dish bal 20090621 "You are given three four-letter words. Think of three letters that can precede each of them to complete familiar seven-letter words. For example, given """"each,"""" """"rove"""" and """"lode,"""" the answer would be """"imp"""" for """"impeach,"""" """"improve"""" and """"implode.""""" 7 dual, nite, sped gra 20090621 "You are given three four-letter words. Think of three letters that can precede each of them to complete familiar seven-letter words. For example, given """"each,"""" """"rove"""" and """"lode,"""" the answer would be """"imp"""" for """"impeach,"""" """"improve"""" and """"implode.""""" 8 rice, size, able cap 20090621 "You are given three four-letter words. Think of three letters that can precede each of them to complete familiar seven-letter words. For example, given """"each,"""" """"rove"""" and """"lode,"""" the answer would be """"imp"""" for """"impeach,"""" """"improve"""" and """"implode.""""" 9 loin, suit, pose pur 20080713 "You are given three words, starting with the letters A, R and T, respectively. You offer a fourth word, to precede each of the original words, to create a familiar two-word phrase. For example, given """"alarm,"""" """"report"""" and """"teeth,"""" the answer would be """"false,"""" as in """"false alarm,"""" """"false report"""" and """"false teeth.""""" 0 ant, retardant, truck fire 20080713 "You are given three words, starting with the letters A, R and T, respectively. You offer a fourth word, to precede each of the original words, to create a familiar two-word phrase. For example, given """"alarm,"""" """"report"""" and """"teeth,"""" the answer would be """"false,"""" as in """"false alarm,"""" """"false report"""" and """"false teeth.""""" 1 age, retriever, touch golden 20080713 "You are given three words, starting with the letters A, R and T, respectively. You offer a fourth word, to precede each of the original words, to create a familiar two-word phrase. For example, given """"alarm,"""" """"report"""" and """"teeth,"""" the answer would be """"false,"""" as in """"false alarm,"""" """"false report"""" and """"false teeth.""""" 2 attendant, recorder, test flight 20080713 "You are given three words, starting with the letters A, R and T, respectively. You offer a fourth word, to precede each of the original words, to create a familiar two-word phrase. For example, given """"alarm,"""" """"report"""" and """"teeth,"""" the answer would be """"false,"""" as in """"false alarm,"""" """"false report"""" and """"false teeth.""""" 3 angle, reverend, turn right 20080713 "You are given three words, starting with the letters A, R and T, respectively. You offer a fourth word, to precede each of the original words, to create a familiar two-word phrase. For example, given """"alarm,"""" """"report"""" and """"teeth,"""" the answer would be """"false,"""" as in """"false alarm,"""" """"false report"""" and """"false teeth.""""" 4 acres, room, thumb green 20080713 "You are given three words, starting with the letters A, R and T, respectively. You offer a fourth word, to precede each of the original words, to create a familiar two-word phrase. For example, given """"alarm,"""" """"report"""" and """"teeth,"""" the answer would be """"false,"""" as in """"false alarm,"""" """"false report"""" and """"false teeth.""""" 5 alert, rooster, tape red 20080713 "You are given three words, starting with the letters A, R and T, respectively. You offer a fourth word, to precede each of the original words, to create a familiar two-word phrase. For example, given """"alarm,"""" """"report"""" and """"teeth,"""" the answer would be """"false,"""" as in """"false alarm,"""" """"false report"""" and """"false teeth.""""" 6 air, rod, tub hot 20080713 "You are given three words, starting with the letters A, R and T, respectively. You offer a fourth word, to precede each of the original words, to create a familiar two-word phrase. For example, given """"alarm,"""" """"report"""" and """"teeth,"""" the answer would be """"false,"""" as in """"false alarm,"""" """"false report"""" and """"false teeth.""""" 7 atlas, rage, trip road 20080713 "You are given three words, starting with the letters A, R and T, respectively. You offer a fourth word, to precede each of the original words, to create a familiar two-word phrase. For example, given """"alarm,"""" """"report"""" and """"teeth,"""" the answer would be """"false,"""" as in """"false alarm,"""" """"false report"""" and """"false teeth.""""" 8 aspirin, Ruth, talkafternoon, riddance, taste baby good 20080713 "You are given three words, starting with the letters A, R and T, respectively. You offer a fourth word, to precede each of the original words, to create a familiar two-word phrase. For example, given """"alarm,"""" """"report"""" and """"teeth,"""" the answer would be """"false,"""" as in """"false alarm,"""" """"false report"""" and """"false teeth.""""" 9 How come there are no left angles, and left reverends? They 20080713 "You are given three words, starting with the letters A, R and T, respectively. You offer a fourth word, to precede each of the original words, to create a familiar two-word phrase. For example, given """"alarm,"""" """"report"""" and """"teeth,"""" the answer would be """"false,"""" as in """"false alarm,"""" """"false report"""" and """"false teeth.""""" 10 could lead a church I want to visit. 20071028 "In this week's on-air puzzle, we start with a clue in a category, then name something else in the same category that starts with the second and third letters of the original clue. For example, if the clue was Minnesota, the answer would be Indiana — because the second and third letters of Minnesota are """"I-N,"""" which begin Indiana — and both Minnesota and Indiana are states." 0 green red 20071028 "In this week's on-air puzzle, we start with a clue in a category, then name something else in the same category that starts with the second and third letters of the original clue. For example, if the clue was Minnesota, the answer would be Indiana — because the second and third letters of Minnesota are """"I-N,"""" which begin Indiana — and both Minnesota and Indiana are states." 1 Madison Adams, not advertising 20071028 "In this week's on-air puzzle, we start with a clue in a category, then name something else in the same category that starts with the second and third letters of the original clue. For example, if the clue was Minnesota, the answer would be Indiana — because the second and third letters of Minnesota are """"I-N,"""" which begin Indiana — and both Minnesota and Indiana are states." 2 Volvo Oldsmobile 20071028 "In this week's on-air puzzle, we start with a clue in a category, then name something else in the same category that starts with the second and third letters of the original clue. For example, if the clue was Minnesota, the answer would be Indiana — because the second and third letters of Minnesota are """"I-N,"""" which begin Indiana — and both Minnesota and Indiana are states." 3 pound ounce 20071028 "In this week's on-air puzzle, we start with a clue in a category, then name something else in the same category that starts with the second and third letters of the original clue. For example, if the clue was Minnesota, the answer would be Indiana — because the second and third letters of Minnesota are """"I-N,"""" which begin Indiana — and both Minnesota and Indiana are states." 4 Yankees Angels 20071028 "In this week's on-air puzzle, we start with a clue in a category, then name something else in the same category that starts with the second and third letters of the original clue. For example, if the clue was Minnesota, the answer would be Indiana — because the second and third letters of Minnesota are """"I-N,"""" which begin Indiana — and both Minnesota and Indiana are states." 5 crocus rose 20071028 "In this week's on-air puzzle, we start with a clue in a category, then name something else in the same category that starts with the second and third letters of the original clue. For example, if the clue was Minnesota, the answer would be Indiana — because the second and third letters of Minnesota are """"I-N,"""" which begin Indiana — and both Minnesota and Indiana are states." 6 Texaco Exxon 20071028 "In this week's on-air puzzle, we start with a clue in a category, then name something else in the same category that starts with the second and third letters of the original clue. For example, if the clue was Minnesota, the answer would be Indiana — because the second and third letters of Minnesota are """"I-N,"""" which begin Indiana — and both Minnesota and Indiana are states." 7 Moscow Oslo 20071028 "In this week's on-air puzzle, we start with a clue in a category, then name something else in the same category that starts with the second and third letters of the original clue. For example, if the clue was Minnesota, the answer would be Indiana — because the second and third letters of Minnesota are """"I-N,"""" which begin Indiana — and both Minnesota and Indiana are states." 8 Airedale Irish setter 20071028 "In this week's on-air puzzle, we start with a clue in a category, then name something else in the same category that starts with the second and third letters of the original clue. For example, if the clue was Minnesota, the answer would be Indiana — because the second and third letters of Minnesota are """"I-N,"""" which begin Indiana — and both Minnesota and Indiana are states." 9 grape raspberry (no word on raisin) 20071028 "In this week's on-air puzzle, we start with a clue in a category, then name something else in the same category that starts with the second and third letters of the original clue. For example, if the clue was Minnesota, the answer would be Indiana — because the second and third letters of Minnesota are """"I-N,"""" which begin Indiana — and both Minnesota and Indiana are states." 10 Thanksgiving Halloween 20071028 "In this week's on-air puzzle, we start with a clue in a category, then name something else in the same category that starts with the second and third letters of the original clue. For example, if the clue was Minnesota, the answer would be Indiana — because the second and third letters of Minnesota are """"I-N,"""" which begin Indiana — and both Minnesota and Indiana are states." 11 topaz opal 20071028 "In this week's on-air puzzle, we start with a clue in a category, then name something else in the same category that starts with the second and third letters of the original clue. For example, if the clue was Minnesota, the answer would be Indiana — because the second and third letters of Minnesota are """"I-N,"""" which begin Indiana — and both Minnesota and Indiana are states." 12 Trix Rice Crispies 20071028 "In this week's on-air puzzle, we start with a clue in a category, then name something else in the same category that starts with the second and third letters of the original clue. For example, if the clue was Minnesota, the answer would be Indiana — because the second and third letters of Minnesota are """"I-N,"""" which begin Indiana — and both Minnesota and Indiana are states." 13 BeatlesWind and Fire Eagles; Easy Beats; or Earth, 20071028 "In this week's on-air puzzle, we start with a clue in a category, then name something else in the same category that starts with the second and third letters of the original clue. For example, if the clue was Minnesota, the answer would be Indiana — because the second and third letters of Minnesota are """"I-N,"""" which begin Indiana — and both Minnesota and Indiana are states." 14 whiskey sour highball 20071028 "In this week's on-air puzzle, we start with a clue in a category, then name something else in the same category that starts with the second and third letters of the original clue. For example, if the clue was Minnesota, the answer would be Indiana — because the second and third letters of Minnesota are """"I-N,"""" which begin Indiana — and both Minnesota and Indiana are states." 15 Hamlet A Midsummer Night's Dream 20080928 "Every answer is a word, name or familiar phrase starting with K and ending with Y, as in """"Kentucky."""" For example, given the clue """"stereotypical fraternity blowout,"""" the answer would be """"keg party.""""" 0 president after Eisenhower Kennedy 20080928 "Every answer is a word, name or familiar phrase starting with K and ending with Y, as in """"Kentucky."""" For example, given the clue """"stereotypical fraternity blowout,"""" the answer would be """"keg party.""""" 1 home of the Chiefs and Royals Kansas City 20080928 "Every answer is a word, name or familiar phrase starting with K and ending with Y, as in """"Kentucky."""" For example, given the clue """"stereotypical fraternity blowout,"""" the answer would be """"keg party.""""" 2 in World War II graffiti, he was here Kilroy 20080928 "Every answer is a word, name or familiar phrase starting with K and ending with Y, as in """"Kentucky."""" For example, given the clue """"stereotypical fraternity blowout,"""" the answer would be """"keg party.""""" 3 a person who spoils all the fun killjoy 20080928 "Every answer is a word, name or familiar phrase starting with K and ending with Y, as in """"Kentucky."""" For example, given the clue """"stereotypical fraternity blowout,"""" the answer would be """"keg party.""""" 4 money in the middle of a poker table kitty 20080928 "Every answer is a word, name or familiar phrase starting with K and ending with Y, as in """"Kentucky."""" For example, given the clue """"stereotypical fraternity blowout,"""" the answer would be """"keg party.""""" 5 actor Ben, who played Gandhi Kingsley 20080928 "Every answer is a word, name or familiar phrase starting with K and ending with Y, as in """"Kentucky."""" For example, given the clue """"stereotypical fraternity blowout,"""" the answer would be """"keg party.""""" 6 Bush's opponent in 2004 Kerry 20080928 "Every answer is a word, name or familiar phrase starting with K and ending with Y, as in """"Kentucky."""" For example, given the clue """"stereotypical fraternity blowout,"""" the answer would be """"keg party.""""" 7 sound when something is destroyed in an explosion kaplooey 20080302 "In this week's on-air puzzle, clues are given for two words that each ends in an unaccented vowel sound. Change that sound to an """"er,"""" and you'll get a new word that answers the second clue. For example, if the clue were """"sandwich fish"""" and """"piano repairman,"""" the answer would be """"tuna"""" and """"tuner."""" " 1 capital of Peru; a monkey-like animal Lima, lemur 20080302 "In this week's on-air puzzle, clues are given for two words that each ends in an unaccented vowel sound. Change that sound to an """"er,"""" and you'll get a new word that answers the second clue. For example, if the clue were """"sandwich fish"""" and """"piano repairman,"""" the answer would be """"tuna"""" and """"tuner."""" " 2 food from heaven; a fancy home manna, manor 20080302 "In this week's on-air puzzle, clues are given for two words that each ends in an unaccented vowel sound. Change that sound to an """"er,"""" and you'll get a new word that answers the second clue. For example, if the clue were """"sandwich fish"""" and """"piano repairman,"""" the answer would be """"tuna"""" and """"tuner."""" " 3 Latin for Earth; extreme fright terra, terror 20080302 "In this week's on-air puzzle, clues are given for two words that each ends in an unaccented vowel sound. Change that sound to an """"er,"""" and you'll get a new word that answers the second clue. For example, if the clue were """"sandwich fish"""" and """"piano repairman,"""" the answer would be """"tuna"""" and """"tuner."""" " 4 a deep toned instrument; a potato, for example tuba, tuber 20080302 "In this week's on-air puzzle, clues are given for two words that each ends in an unaccented vowel sound. Change that sound to an """"er,"""" and you'll get a new word that answers the second clue. For example, if the clue were """"sandwich fish"""" and """"piano repairman,"""" the answer would be """"tuna"""" and """"tuner."""" " 5 a line dance; a kind of eel conga, conger 20080302 "In this week's on-air puzzle, clues are given for two words that each ends in an unaccented vowel sound. Change that sound to an """"er,"""" and you'll get a new word that answers the second clue. For example, if the clue were """"sandwich fish"""" and """"piano repairman,"""" the answer would be """"tuna"""" and """"tuner."""" " 6 member of an old Andean empire; part of aprinting press Inca, inker 20080302 "In this week's on-air puzzle, clues are given for two words that each ends in an unaccented vowel sound. Change that sound to an """"er,"""" and you'll get a new word that answers the second clue. For example, if the clue were """"sandwich fish"""" and """"piano repairman,"""" the answer would be """"tuna"""" and """"tuner."""" " 7 old Italian currency; one who looks salaciously lira,leerer 20080302 "In this week's on-air puzzle, clues are given for two words that each ends in an unaccented vowel sound. Change that sound to an """"er,"""" and you'll get a new word that answers the second clue. For example, if the clue were """"sandwich fish"""" and """"piano repairman,"""" the answer would be """"tuna"""" and """"tuner."""" " 8 disaster relief organization; thigh bone FEMA, femur 20080210 "In this week's on-air puzzle, every answer is a two-word phrase, in which both words start with the letter """"P"""" and the two words are anagrams of each other. For example, for the clue, """"One hundred percent the land of Lima,"""" the answer would be, """"Pure Peru.""""" 0 couples in the capital of France Paris pairs 20080210 "In this week's on-air puzzle, every answer is a two-word phrase, in which both words start with the letter """"P"""" and the two words are anagrams of each other. For example, for the clue, """"One hundred percent the land of Lima,"""" the answer would be, """"Pure Peru.""""" 1 a little puzzle that is not in verse prose poser 20080210 "In this week's on-air puzzle, every answer is a two-word phrase, in which both words start with the letter """"P"""" and the two words are anagrams of each other. For example, for the clue, """"One hundred percent the land of Lima,"""" the answer would be, """"Pure Peru.""""" 2 less colorful a gem from an oyster paler pearl 20080210 "In this week's on-air puzzle, every answer is a two-word phrase, in which both words start with the letter """"P"""" and the two words are anagrams of each other. For example, for the clue, """"One hundred percent the land of Lima,"""" the answer would be, """"Pure Peru.""""" 3 least colorful parts of a flower palest petals 20080210 "In this week's on-air puzzle, every answer is a two-word phrase, in which both words start with the letter """"P"""" and the two words are anagrams of each other. For example, for the clue, """"One hundred percent the land of Lima,"""" the answer would be, """"Pure Peru.""""" 4 an aviator's gun pilot's pistol 20080210 "In this week's on-air puzzle, every answer is a two-word phrase, in which both words start with the letter """"P"""" and the two words are anagrams of each other. For example, for the clue, """"One hundred percent the land of Lima,"""" the answer would be, """"Pure Peru.""""" 5 one who sets down a package parcel placer 20080210 "In this week's on-air puzzle, every answer is a two-word phrase, in which both words start with the letter """"P"""" and the two words are anagrams of each other. For example, for the clue, """"One hundred percent the land of Lima,"""" the answer would be, """"Pure Peru.""""" 6 social gatherings for buccaneers pirates' parties 20080210 "In this week's on-air puzzle, every answer is a two-word phrase, in which both words start with the letter """"P"""" and the two words are anagrams of each other. For example, for the clue, """"One hundred percent the land of Lima,"""" the answer would be, """"Pure Peru.""""" 7 tweezers for a king's son prince's pincers 20080210 "In this week's on-air puzzle, every answer is a two-word phrase, in which both words start with the letter """"P"""" and the two words are anagrams of each other. For example, for the clue, """"One hundred percent the land of Lima,"""" the answer would be, """"Pure Peru.""""" 8 sit-in staged by makers of clay ware potters' protest 20080210 "In this week's on-air puzzle, every answer is a two-word phrase, in which both words start with the letter """"P"""" and the two words are anagrams of each other. For example, for the clue, """"One hundred percent the land of Lima,"""" the answer would be, """"Pure Peru.""""" 9 not-so-decorated a candy containing nuts plainer pralines 20080210 "In this week's on-air puzzle, every answer is a two-word phrase, in which both words start with the letter """"P"""" and the two words are anagrams of each other. For example, for the clue, """"One hundred percent the land of Lima,"""" the answer would be, """"Pure Peru.""""" 10 fines in a Biblical land (9 letters) Palestine penalties 20070603 "The on-air puzzle this week offers a game of ""accidental acrostics."" Every answer is a multiword title, in which the initial letters spell a word. " 0 CAP, Dostoevsky novel Crime and Punishment 20070603 "The on-air puzzle this week offers a game of ""accidental acrostics."" Every answer is a multiword title, in which the initial letters spell a word. " 1 SLY, Beatles hit She Loves You 20070603 "The on-air puzzle this week offers a game of ""accidental acrostics."" Every answer is a multiword title, in which the initial letters spell a word. " 2 IMP, #1 Leslie Gore hit It's My Party 20070603 "The on-air puzzle this week offers a game of ""accidental acrostics."" Every answer is a multiword title, in which the initial letters spell a word. " 3 "DOH, 1980's TV show, with ""The""" Dukes of Hazard 20070603 "The on-air puzzle this week offers a game of ""accidental acrostics."" Every answer is a multiword title, in which the initial letters spell a word. " 4 NOS, and NFL team New Orleans Saints 20070603 "The on-air puzzle this week offers a game of ""accidental acrostics."" Every answer is a multiword title, in which the initial letters spell a word. " 5 SAW, event of 1898 Spanish American War 20070603 "The on-air puzzle this week offers a game of ""accidental acrostics."" Every answer is a multiword title, in which the initial letters spell a word. " 6 GOO, classic weekly radio and TV countrymusical program Grand Ole Opry 20070603 "The on-air puzzle this week offers a game of ""accidental acrostics."" Every answer is a multiword title, in which the initial letters spell a word. " 7 PITH, Sally Field movie Places in the Heart 20070603 "The on-air puzzle this week offers a game of ""accidental acrostics."" Every answer is a multiword title, in which the initial letters spell a word. " 8 OTIS, 1930's musical comedy whose titleis a line from the song America Of Thee I Sing 20070603 "The on-air puzzle this week offers a game of ""accidental acrostics."" Every answer is a multiword title, in which the initial letters spell a word. " 9 COTS, a brand of tuna Chicken of the Sea 20070603 "The on-air puzzle this week offers a game of ""accidental acrostics."" Every answer is a multiword title, in which the initial letters spell a word. " 10 TSAR, Ernest Hemingway novel The Sun Also Rises 20081207 Every answer is a familiar two-word phrase or title in which one word starts with P-I, as in Pittsburgh, and the other starts with P-A. The P-A and P-I words can be in either order. 1 "1963 Peter Sellers comedy, with ""The""" Pink Panther 20081207 Every answer is a familiar two-word phrase or title in which one word starts with P-I, as in Pittsburgh, and the other starts with P-A. The P-A and P-I words can be in either order. 2 noted Spanish cubist Pablo Picasso 20081207 Every answer is a familiar two-word phrase or title in which one word starts with P-I, as in Pittsburgh, and the other starts with P-A. The P-A and P-I words can be in either order. 3 container that a cherry cobbler might bebaked in pie pan 20081207 Every answer is a familiar two-word phrase or title in which one word starts with P-I, as in Pittsburgh, and the other starts with P-A. The P-A and P-I words can be in either order. 4 pioneering 1990's PDA device PalmPilot 20080817 "In the on-air puzzle, you must complete a familiar three-word phrase that takes the form """"blank of blank."""" You are given the first word and must guess the last. The answer begins with the letter A. For example, given """"law,"""" the answer is """"averages,"""" as in """"law of averages.""""" 0 coat arms 20080817 "In the on-air puzzle, you must complete a familiar three-word phrase that takes the form """"blank of blank."""" You are given the first word and must guess the last. The answer begins with the letter A. For example, given """"law,"""" the answer is """"averages,"""" as in """"law of averages.""""" 2 Department Agriculture 20080817 "In the on-air puzzle, you must complete a familiar three-word phrase that takes the form """"blank of blank."""" You are given the first word and must guess the last. The answer begins with the letter A. For example, given """"law,"""" the answer is """"averages,"""" as in """"law of averages.""""" 3 Horn Africa 20080817 "In the on-air puzzle, you must complete a familiar three-word phrase that takes the form """"blank of blank."""" You are given the first word and must guess the last. The answer begins with the letter A. For example, given """"law,"""" the answer is """"averages,"""" as in """"law of averages.""""" 4 Joan Arc 20080817 "In the on-air puzzle, you must complete a familiar three-word phrase that takes the form """"blank of blank."""" You are given the first word and must guess the last. The answer begins with the letter A. For example, given """"law,"""" the answer is """"averages,"""" as in """"law of averages.""""" 5 plan attack or action 20080817 "In the on-air puzzle, you must complete a familiar three-word phrase that takes the form """"blank of blank."""" You are given the first word and must guess the last. The answer begins with the letter A. For example, given """"law,"""" the answer is """"averages,"""" as in """"law of averages.""""" 6 Voice America 20080817 "In the on-air puzzle, you must complete a familiar three-word phrase that takes the form """"blank of blank."""" You are given the first word and must guess the last. The answer begins with the letter A. For example, given """"law,"""" the answer is """"averages,"""" as in """"law of averages.""""" 7 rock ages 20080817 "In the on-air puzzle, you must complete a familiar three-word phrase that takes the form """"blank of blank."""" You are given the first word and must guess the last. The answer begins with the letter A. For example, given """"law,"""" the answer is """"averages,"""" as in """"law of averages.""""" 8 seal approval 20080817 "In the on-air puzzle, you must complete a familiar three-word phrase that takes the form """"blank of blank."""" You are given the first word and must guess the last. The answer begins with the letter A. For example, given """"law,"""" the answer is """"averages,"""" as in """"law of averages.""""" 9 court appeals 20080817 "In the on-air puzzle, you must complete a familiar three-word phrase that takes the form """"blank of blank."""" You are given the first word and must guess the last. The answer begins with the letter A. For example, given """"law,"""" the answer is """"averages,"""" as in """"law of averages.""""" 10 power attorney 20080817 "In the on-air puzzle, you must complete a familiar three-word phrase that takes the form """"blank of blank."""" You are given the first word and must guess the last. The answer begins with the letter A. For example, given """"law,"""" the answer is """"averages,"""" as in """"law of averages.""""" 11 Lawrence Arabia 20080817 "In the on-air puzzle, you must complete a familiar three-word phrase that takes the form """"blank of blank."""" You are given the first word and must guess the last. The answer begins with the letter A. For example, given """"law,"""" the answer is """"averages,"""" as in """"law of averages.""""" 12 article apparel 20080817 "In the on-air puzzle, you must complete a familiar three-word phrase that takes the form """"blank of blank."""" You are given the first word and must guess the last. The answer begins with the letter A. For example, given """"law,"""" the answer is """"averages,"""" as in """"law of averages.""""" 13 Age Aquarius, anxiety 20080817 "In the on-air puzzle, you must complete a familiar three-word phrase that takes the form """"blank of blank."""" You are given the first word and must guess the last. The answer begins with the letter A. For example, given """"law,"""" the answer is """"averages,"""" as in """"law of averages.""""" 14 Timon Athens 20080817 "In the on-air puzzle, you must complete a familiar three-word phrase that takes the form """"blank of blank."""" You are given the first word and must guess the last. The answer begins with the letter A. For example, given """"law,"""" the answer is """"averages,"""" as in """"law of averages.""""" 15 center attention 20080817 "In the on-air puzzle, you must complete a familiar three-word phrase that takes the form """"blank of blank."""" You are given the first word and must guess the last. The answer begins with the letter A. For example, given """"law,"""" the answer is """"averages,"""" as in """"law of averages.""""" 16 round applause, not ammunition 20070415 This puzzle sounds a little more complicated than it is. I'm going to give you some words. For each one, take the last two letters, reverse them, then think of a word starting with these two letters that can follow mine to complete a familiar two-word phrase. 0 Faberg‚ (second wordsare 3 letters long) Faberg‚ egg 20070415 This puzzle sounds a little more complicated than it is. I'm going to give you some words. For each one, take the last two letters, reverse them, then think of a word starting with these two letters that can follow mine to complete a familiar two-word phrase. 1 martial martial law 20070415 This puzzle sounds a little more complicated than it is. I'm going to give you some words. For each one, take the last two letters, reverse them, then think of a word starting with these two letters that can follow mine to complete a familiar two-word phrase. 2 carrot Carrot Top 20070415 This puzzle sounds a little more complicated than it is. I'm going to give you some words. For each one, take the last two letters, reverse them, then think of a word starting with these two letters that can follow mine to complete a familiar two-word phrase. 3 mad (second words now4 letters long) mad dash 20070415 This puzzle sounds a little more complicated than it is. I'm going to give you some words. For each one, take the last two letters, reverse them, then think of a word starting with these two letters that can follow mine to complete a familiar two-word phrase. 4 daytime daytime Emmy 20070415 This puzzle sounds a little more complicated than it is. I'm going to give you some words. For each one, take the last two letters, reverse them, then think of a word starting with these two letters that can follow mine to complete a familiar two-word phrase. 5 liquid liquid diet 20070415 This puzzle sounds a little more complicated than it is. I'm going to give you some words. For each one, take the last two letters, reverse them, then think of a word starting with these two letters that can follow mine to complete a familiar two-word phrase. 6 honor honor roll 20070415 This puzzle sounds a little more complicated than it is. I'm going to give you some words. For each one, take the last two letters, reverse them, then think of a word starting with these two letters that can follow mine to complete a familiar two-word phrase. 7 glow glow worm 20070415 This puzzle sounds a little more complicated than it is. I'm going to give you some words. For each one, take the last two letters, reverse them, then think of a word starting with these two letters that can follow mine to complete a familiar two-word phrase. 8 scrap (5 letters) scrap paper 20070415 This puzzle sounds a little more complicated than it is. I'm going to give you some words. For each one, take the last two letters, reverse them, then think of a word starting with these two letters that can follow mine to complete a familiar two-word phrase. 9 hot hot toddy, or topic 20070415 This puzzle sounds a little more complicated than it is. I'm going to give you some words. For each one, take the last two letters, reverse them, then think of a word starting with these two letters that can follow mine to complete a familiar two-word phrase. 10 blood blood donor 20070415 This puzzle sounds a little more complicated than it is. I'm going to give you some words. For each one, take the last two letters, reverse them, then think of a word starting with these two letters that can follow mine to complete a familiar two-word phrase. 11 Detroit Detroit Tiger 20070415 This puzzle sounds a little more complicated than it is. I'm going to give you some words. For each one, take the last two letters, reverse them, then think of a word starting with these two letters that can follow mine to complete a familiar two-word phrase. 12 air air rifle 20070415 This puzzle sounds a little more complicated than it is. I'm going to give you some words. For each one, take the last two letters, reverse them, then think of a word starting with these two letters that can follow mine to complete a familiar two-word phrase. 13 human (6 letters) human nature 20070415 This puzzle sounds a little more complicated than it is. I'm going to give you some words. For each one, take the last two letters, reverse them, then think of a word starting with these two letters that can follow mine to complete a familiar two-word phrase. 14 cedar Cedar Rapids 20070415 This puzzle sounds a little more complicated than it is. I'm going to give you some words. For each one, take the last two letters, reverse them, then think of a word starting with these two letters that can follow mine to complete a familiar two-word phrase. 15 wonton wonton noodle 20070415 This puzzle sounds a little more complicated than it is. I'm going to give you some words. For each one, take the last two letters, reverse them, then think of a word starting with these two letters that can follow mine to complete a familiar two-word phrase. 18 civil (7+ letters) civil liberty 20070415 This puzzle sounds a little more complicated than it is. I'm going to give you some words. For each one, take the last two letters, reverse them, then think of a word starting with these two letters that can follow mine to complete a familiar two-word phrase. 19 heart heart trouble, or transplant 20070415 This puzzle sounds a little more complicated than it is. I'm going to give you some words. For each one, take the last two letters, reverse them, then think of a word starting with these two letters that can follow mine to complete a familiar two-word phrase. 20 Eleanor Eleanor Roosevelt 20091011 "This week's puzzle involves the letters P-S-U as in Portland State University, where Will spoke on Saturday. Each answer is a familiar two-word phrase in which the first word starts with """"P"""" and the second word starts with """"SU."""" For example, if the clue is """"What a lawyer conducts to see if an invention has been made before,"""" the answer would be """"patent survey.""""" 1 luxury accommodation at the top of a building penthouse suite 20091011 "This week's puzzle involves the letters P-S-U as in Portland State University, where Will spoke on Saturday. Each answer is a familiar two-word phrase in which the first word starts with """"P"""" and the second word starts with """"SU."""" For example, if the clue is """"What a lawyer conducts to see if an invention has been made before,"""" the answer would be """"patent survey.""""" 2 NBA team in Arizona Phoenix Suns 20091011 "This week's puzzle involves the letters P-S-U as in Portland State University, where Will spoke on Saturday. Each answer is a familiar two-word phrase in which the first word starts with """"P"""" and the second word starts with """"SU."""" For example, if the clue is """"What a lawyer conducts to see if an invention has been made before,"""" the answer would be """"patent survey.""""" 3 medical procedure to improve one's appearance plastic surgery 20091011 "This week's puzzle involves the letters P-S-U as in Portland State University, where Will spoke on Saturday. Each answer is a familiar two-word phrase in which the first word starts with """"P"""" and the second word starts with """"SU."""" For example, if the clue is """"What a lawyer conducts to see if an invention has been made before,"""" the answer would be """"patent survey.""""" 4 "Buddy Holly hit whose title follows thelyrics, ""I love you, gal, yes I love you""" Peggy Sue 20091011 "This week's puzzle involves the letters P-S-U as in Portland State University, where Will spoke on Saturday. Each answer is a familiar two-word phrase in which the first word starts with """"P"""" and the second word starts with """"SU."""" For example, if the clue is """"What a lawyer conducts to see if an invention has been made before,"""" the answer would be """"patent survey.""""" 5 women's business attire that is not a dress pant suit 20091011 "This week's puzzle involves the letters P-S-U as in Portland State University, where Will spoke on Saturday. Each answer is a familiar two-word phrase in which the first word starts with """"P"""" and the second word starts with """"SU."""" For example, if the clue is """"What a lawyer conducts to see if an invention has been made before,"""" the answer would be """"patent survey.""""" 6 a wood court for basketball, or a grass fieldfor football playing surface 20091011 "This week's puzzle involves the letters P-S-U as in Portland State University, where Will spoke on Saturday. Each answer is a familiar two-word phrase in which the first word starts with """"P"""" and the second word starts with """"SU."""" For example, if the clue is """"What a lawyer conducts to see if an invention has been made before,"""" the answer would be """"patent survey.""""" 7 shrub that can give you a rash poison sumac 20091011 "This week's puzzle involves the letters P-S-U as in Portland State University, where Will spoke on Saturday. Each answer is a familiar two-word phrase in which the first word starts with """"P"""" and the second word starts with """"SU."""" For example, if the clue is """"What a lawyer conducts to see if an invention has been made before,"""" the answer would be """"patent survey.""""" 8 former football star and TV sportscaster whowas once the spokesperson for True Value Pat Summerall 20091011 "This week's puzzle involves the letters P-S-U as in Portland State University, where Will spoke on Saturday. Each answer is a familiar two-word phrase in which the first word starts with """"P"""" and the second word starts with """"SU."""" For example, if the clue is """"What a lawyer conducts to see if an invention has been made before,"""" the answer would be """"patent survey.""""" 9 chemical compound represented as K2SO4 potassium sulfate 20091011 "This week's puzzle involves the letters P-S-U as in Portland State University, where Will spoke on Saturday. Each answer is a familiar two-word phrase in which the first word starts with """"P"""" and the second word starts with """"SU."""" For example, if the clue is """"What a lawyer conducts to see if an invention has been made before,"""" the answer would be """"patent survey.""""" 10 community event where everyone brings a dishfor everyone to share potluck supper 20091011 "This week's puzzle involves the letters P-S-U as in Portland State University, where Will spoke on Saturday. Each answer is a familiar two-word phrase in which the first word starts with """"P"""" and the second word starts with """"SU."""" For example, if the clue is """"What a lawyer conducts to see if an invention has been made before,"""" the answer would be """"patent survey.""""" 11 one week before Easter Palm Sunday 20091011 "This week's puzzle involves the letters P-S-U as in Portland State University, where Will spoke on Saturday. Each answer is a familiar two-word phrase in which the first word starts with """"P"""" and the second word starts with """"SU."""" For example, if the clue is """"What a lawyer conducts to see if an invention has been made before,"""" the answer would be """"patent survey.""""" 12 like a weather forecast with some clouds partly sunny 20081123 "Each clue is one word. The answer is a word that can follow the clue to complete a familiar two-word phrase. The first two letters of the answer must be the first and last letter of the clue. For example, given """"pool,"""" the answer would be """"player.""""" 0 pepperoni pizza 20081123 "Each clue is one word. The answer is a word that can follow the clue to complete a familiar two-word phrase. The first two letters of the answer must be the first and last letter of the clue. For example, given """"pool,"""" the answer would be """"player.""""" 1 advertising agent, or agency 20081123 "Each clue is one word. The answer is a word that can follow the clue to complete a familiar two-word phrase. The first two letters of the answer must be the first and last letter of the clue. For example, given """"pool,"""" the answer would be """"player.""""" 2 varicose veins 20081123 "Each clue is one word. The answer is a word that can follow the clue to complete a familiar two-word phrase. The first two letters of the answer must be the first and last letter of the clue. For example, given """"pool,"""" the answer would be """"player.""""" 3 little league 20081123 "Each clue is one word. The answer is a word that can follow the clue to complete a familiar two-word phrase. The first two letters of the answer must be the first and last letter of the clue. For example, given """"pool,"""" the answer would be """"player.""""" 4 balance beam 20081123 "Each clue is one word. The answer is a word that can follow the clue to complete a familiar two-word phrase. The first two letters of the answer must be the first and last letter of the clue. For example, given """"pool,"""" the answer would be """"player.""""" 5 political platform 20081123 "Each clue is one word. The answer is a word that can follow the clue to complete a familiar two-word phrase. The first two letters of the answer must be the first and last letter of the clue. For example, given """"pool,"""" the answer would be """"player.""""" 6 picture perfect 20081123 "Each clue is one word. The answer is a word that can follow the clue to complete a familiar two-word phrase. The first two letters of the answer must be the first and last letter of the clue. For example, given """"pool,"""" the answer would be """"player.""""" 7 China cabinet 20081123 "Each clue is one word. The answer is a word that can follow the clue to complete a familiar two-word phrase. The first two letters of the answer must be the first and last letter of the clue. For example, given """"pool,"""" the answer would be """"player.""""" 8 short staff, stick, story, stack, stop 20081123 "Each clue is one word. The answer is a word that can follow the clue to complete a familiar two-word phrase. The first two letters of the answer must be the first and last letter of the clue. For example, given """"pool,"""" the answer would be """"player.""""" 9 crystal clear 20081123 "Each clue is one word. The answer is a word that can follow the clue to complete a familiar two-word phrase. The first two letters of the answer must be the first and last letter of the clue. For example, given """"pool,"""" the answer would be """"player.""""" 10 love letter 20081123 "Each clue is one word. The answer is a word that can follow the clue to complete a familiar two-word phrase. The first two letters of the answer must be the first and last letter of the clue. For example, given """"pool,"""" the answer would be """"player.""""" 11 ascorbic acid 20081123 "Each clue is one word. The answer is a word that can follow the clue to complete a familiar two-word phrase. The first two letters of the answer must be the first and last letter of the clue. For example, given """"pool,"""" the answer would be """"player.""""" 12 peer pressure 20081123 "Each clue is one word. The answer is a word that can follow the clue to complete a familiar two-word phrase. The first two letters of the answer must be the first and last letter of the clue. For example, given """"pool,"""" the answer would be """"player.""""" 13 table tennis 20090118 "Every answer is a word starting with the letter O. The clues are two words — one that can precede the answer, and one that can follow it to complete familiar two-word phrases. For example, given """"elected"""" and """"statement,"""" the answer would be """"official,"""" as in """"elected official"""" and """"official statement.""""" 0 agent, bowl orange 20090118 "Every answer is a word starting with the letter O. The clues are two words — one that can precede the answer, and one that can follow it to complete familiar two-word phrases. For example, given """"elected"""" and """"statement,"""" the answer would be """"official,"""" as in """"elected official"""" and """"official statement.""""" 1 toaster, mitt oven 20090118 "Every answer is a word starting with the letter O. The clues are two words — one that can precede the answer, and one that can follow it to complete familiar two-word phrases. For example, given """"elected"""" and """"statement,"""" the answer would be """"official,"""" as in """"elected official"""" and """"official statement.""""" 2 vegetable, filter oil 20090118 "Every answer is a word starting with the letter O. The clues are two words — one that can precede the answer, and one that can follow it to complete familiar two-word phrases. For example, given """"elected"""" and """"statement,"""" the answer would be """"official,"""" as in """"elected official"""" and """"official statement.""""" 3 Indian, liner ocean 20090118 "Every answer is a word starting with the letter O. The clues are two words — one that can precede the answer, and one that can follow it to complete familiar two-word phrases. For example, given """"elected"""" and """"statement,"""" the answer would be """"official,"""" as in """"elected official"""" and """"official statement.""""" 4 black, branch olive 20090118 "Every answer is a word starting with the letter O. The clues are two words — one that can precede the answer, and one that can follow it to complete familiar two-word phrases. For example, given """"elected"""" and """"statement,"""" the answer would be """"official,"""" as in """"elected official"""" and """"official statement.""""" 5 stock, play option 20090118 "Every answer is a word starting with the letter O. The clues are two words — one that can precede the answer, and one that can follow it to complete familiar two-word phrases. For example, given """"elected"""" and """"statement,"""" the answer would be """"official,"""" as in """"elected official"""" and """"official statement.""""" 6 electrical, mall outlet 20090118 "Every answer is a word starting with the letter O. The clues are two words — one that can precede the answer, and one that can follow it to complete familiar two-word phrases. For example, given """"elected"""" and """"statement,"""" the answer would be """"official,"""" as in """"elected official"""" and """"official statement.""""" 7 grand, bell opening 20090118 "Every answer is a word starting with the letter O. The clues are two words — one that can precede the answer, and one that can follow it to complete familiar two-word phrases. For example, given """"elected"""" and """"statement,"""" the answer would be """"official,"""" as in """"elected official"""" and """"official statement.""""" 8 pipe, transplant organ 20090118 "Every answer is a word starting with the letter O. The clues are two words — one that can precede the answer, and one that can follow it to complete familiar two-word phrases. For example, given """"elected"""" and """"statement,"""" the answer would be """"official,"""" as in """"elected official"""" and """"official statement.""""" 9 equal, knocks opportunity 20090118 "Every answer is a word starting with the letter O. The clues are two words — one that can precede the answer, and one that can follow it to complete familiar two-word phrases. For example, given """"elected"""" and """"statement,"""" the answer would be """"official,"""" as in """"elected official"""" and """"official statement.""""" 10 chamber, pit orchestra 20080323 "In this week's puzzle, every answer is a familiar two-word phrase in which the first word starts with CA and the second word starts with W. For example, given """"a glass opening in a wall that is attached by hinges,"""" the answer is """"casement window."""" " 1 it melts and drips from a flame candle wax 20080323 "In this week's puzzle, every answer is a familiar two-word phrase in which the first word starts with CA and the second word starts with W. For example, given """"a glass opening in a wall that is attached by hinges,"""" the answer is """"casement window."""" " 2 covering for a Snickers bar or M&M's candy wrapper 20080323 "In this week's puzzle, every answer is a familiar two-word phrase in which the first word starts with CA and the second word starts with W. For example, given """"a glass opening in a wall that is attached by hinges,"""" the answer is """"casement window."""" " 3 what a moat surrounds castle wall 20080323 "In this week's puzzle, every answer is a familiar two-word phrase in which the first word starts with CA and the second word starts with W. For example, given """"a glass opening in a wall that is attached by hinges,"""" the answer is """"casement window."""" " 4 cabernet or other product from Napa Valley California wine 20080323 "In this week's puzzle, every answer is a familiar two-word phrase in which the first word starts with CA and the second word starts with W. For example, given """"a glass opening in a wall that is attached by hinges,"""" the answer is """"casement window."""" " 5 person making a large floor coveringnot worker carpet weaver, 20080323 "In this week's puzzle, every answer is a familiar two-word phrase in which the first word starts with CA and the second word starts with W. For example, given """"a glass opening in a wall that is attached by hinges,"""" the answer is """"casement window."""" " 6 removal of money from a bank cash withdrawal 20080323 "In this week's puzzle, every answer is a familiar two-word phrase in which the first word starts with CA and the second word starts with W. For example, given """"a glass opening in a wall that is attached by hinges,"""" the answer is """"casement window."""" " 7 part of a Surrey or Brougham that rollsnot car wheel carriage wheel, 20080323 "In this week's puzzle, every answer is a familiar two-word phrase in which the first word starts with CA and the second word starts with W. For example, given """"a glass opening in a wall that is attached by hinges,"""" the answer is """"casement window."""" " 8 seltzer carbonated water 20080323 "In this week's puzzle, every answer is a familiar two-word phrase in which the first word starts with CA and the second word starts with W. For example, given """"a glass opening in a wall that is attached by hinges,"""" the answer is """"casement window."""" " 9 luxury time piece originally made by a 19thCentury French jeweler Cartier watch 20080323 "In this week's puzzle, every answer is a familiar two-word phrase in which the first word starts with CA and the second word starts with W. For example, given """"a glass opening in a wall that is attached by hinges,"""" the answer is """"casement window."""" " 10 T-shirt, shorts and sandals, for example casual wear 20080323 "In this week's puzzle, every answer is a familiar two-word phrase in which the first word starts with CA and the second word starts with W. For example, given """"a glass opening in a wall that is attached by hinges,"""" the answer is """"casement window."""" " 11 one who might try to get you to take threeballs for a dollar carnival worker 20080323 "In this week's puzzle, every answer is a familiar two-word phrase in which the first word starts with CA and the second word starts with W. For example, given """"a glass opening in a wall that is attached by hinges,"""" the answer is """"casement window."""" " 12 in a classic quote, one who must be abovesuspicion Caesar's wife 20080323 "In this week's puzzle, every answer is a familiar two-word phrase in which the first word starts with CA and the second word starts with W. For example, given """"a glass opening in a wall that is attached by hinges,"""" the answer is """"casement window."""" " 13 telephone feature allowing you to talk toone person while another person is on hold call waiting 20080323 "In this week's puzzle, every answer is a familiar two-word phrase in which the first word starts with CA and the second word starts with W. For example, given """"a glass opening in a wall that is attached by hinges,"""" the answer is """"casement window."""" " 14 business with soap and big brushes car wash 20080323 "In this week's puzzle, every answer is a familiar two-word phrase in which the first word starts with CA and the second word starts with W. For example, given """"a glass opening in a wall that is attached by hinges,"""" the answer is """"casement window."""" " 15 cry of eagerness regarding a future event can't wait 20100711 "Every answer is a familiar, two-word phrase with the initials I-D. For example: """"A fake gem for a ring."""" The answer is """"Imitation Diamond.""""" 1 July 4th Independence Day 20100711 "Every answer is a familiar, two-word phrase with the initials I-D. For example: """"A fake gem for a ring."""" The answer is """"Imitation Diamond.""""" 2 theory proposed as an alternative toevolution intelligent design 20100711 "Every answer is a familiar, two-word phrase with the initials I-D. For example: """"A fake gem for a ring."""" The answer is """"Imitation Diamond.""""" 3 cause of anemia iron deficiency 20100711 "Every answer is a familiar, two-word phrase with the initials I-D. For example: """"A fake gem for a ring."""" The answer is """"Imitation Diamond.""""" 4 salad topping with water, oil, vinegarand black pepper Italian dressing 20100711 "Every answer is a familiar, two-word phrase with the initials I-D. For example: """"A fake gem for a ring."""" The answer is """"Imitation Diamond.""""" 5 activity that might be done around acampfire in a headdress Indian dance 20100711 "Every answer is a familiar, two-word phrase with the initials I-D. For example: """"A fake gem for a ring."""" The answer is """"Imitation Diamond.""""" 6 cocaine and heroin illegal drugs 20100711 "Every answer is a familiar, two-word phrase with the initials I-D. For example: """"A fake gem for a ring."""" The answer is """"Imitation Diamond.""""" 7 "classic song with the lyric, ""To fightthe unbeatable foe, to bear theunbearable sorry, to run where the bravedare not go."" Liane adds that it is froMan of La Mancha" Impossible Dreamm 20100711 "Every answer is a familiar, two-word phrase with the initials I-D. For example: """"A fake gem for a ring."""" The answer is """"Imitation Diamond.""""" 8 plastic girl you have to blow up inflatable doll 20100711 "Every answer is a familiar, two-word phrase with the initials I-D. For example: """"A fake gem for a ring."""" The answer is """"Imitation Diamond.""""" 9 what a lawyer might plead to have aclient avoid the death penalty,especially if the client is crazy insanity defense 20100711 "Every answer is a familiar, two-word phrase with the initials I-D. For example: """"A fake gem for a ring."""" The answer is """"Imitation Diamond.""""" 10 a person who chooses wall paper andfurniture interior decorator 20100711 "Every answer is a familiar, two-word phrase with the initials I-D. For example: """"A fake gem for a ring."""" The answer is """"Imitation Diamond.""""" 11 it runs approximately along the 180Datelinelongitude International 20100711 "Every answer is a familiar, two-word phrase with the initials I-D. For example: """"A fake gem for a ring."""" The answer is """"Imitation Diamond.""""" 12 something dug to carry water to crops irrigation ditch 20100711 "Every answer is a familiar, two-word phrase with the initials I-D. For example: """"A fake gem for a ring."""" The answer is """"Imitation Diamond.""""" 13 things you list on your IRS form tolower your taxes itemized deductions 20070819 In this week's on-air puzzle every answer is the name of a musical instrument. See if you can get it through its anagram. For example, if the clue was MUD plus R, you would rearrange those letters to get DRUM. 0 rap + h harp 20070819 In this week's on-air puzzle every answer is the name of a musical instrument. See if you can get it through its anagram. For example, if the clue was MUD plus R, you would rearrange those letters to get DRUM. 1 boo + e oboe 20070819 In this week's on-air puzzle every answer is the name of a musical instrument. See if you can get it through its anagram. For example, if the clue was MUD plus R, you would rearrange those letters to get DRUM. 2 but + a tuba 20070819 In this week's on-air puzzle every answer is the name of a musical instrument. See if you can get it through its anagram. For example, if the clue was MUD plus R, you would rearrange those letters to get DRUM. 3 Joan + b banjo 20070819 In this week's on-air puzzle every answer is the name of a musical instrument. See if you can get it through its anagram. For example, if the clue was MUD plus R, you would rearrange those letters to get DRUM. 4 fuel + t flute 20070819 In this week's on-air puzzle every answer is the name of a musical instrument. See if you can get it through its anagram. For example, if the clue was MUD plus R, you would rearrange those letters to get DRUM. 5 pain + o piano 20070819 In this week's on-air puzzle every answer is the name of a musical instrument. See if you can get it through its anagram. For example, if the clue was MUD plus R, you would rearrange those letters to get DRUM. 6 oval + i viola 20070819 In this week's on-air puzzle every answer is the name of a musical instrument. See if you can get it through its anagram. For example, if the clue was MUD plus R, you would rearrange those letters to get DRUM. 7 rang + o organ 20070819 In this week's on-air puzzle every answer is the name of a musical instrument. See if you can get it through its anagram. For example, if the clue was MUD plus R, you would rearrange those letters to get DRUM. 8 Cleo + L cello 20070819 In this week's on-air puzzle every answer is the name of a musical instrument. See if you can get it through its anagram. For example, if the clue was MUD plus R, you would rearrange those letters to get DRUM. 9 blue + g bugle 20070819 In this week's on-air puzzle every answer is the name of a musical instrument. See if you can get it through its anagram. For example, if the clue was MUD plus R, you would rearrange those letters to get DRUM. 10 field + d fiddle 20070819 In this week's on-air puzzle every answer is the name of a musical instrument. See if you can get it through its anagram. For example, if the clue was MUD plus R, you would rearrange those letters to get DRUM. 11 their + z zither 20070819 In this week's on-air puzzle every answer is the name of a musical instrument. See if you can get it through its anagram. For example, if the clue was MUD plus R, you would rearrange those letters to get DRUM. 12 tenor + c cornet 20070819 In this week's on-air puzzle every answer is the name of a musical instrument. See if you can get it through its anagram. For example, if the clue was MUD plus R, you would rearrange those letters to get DRUM. 13 putter + m trumpet 20070819 In this week's on-air puzzle every answer is the name of a musical instrument. See if you can get it through its anagram. For example, if the clue was MUD plus R, you would rearrange those letters to get DRUM. 14 article + n clarinet 20070819 In this week's on-air puzzle every answer is the name of a musical instrument. See if you can get it through its anagram. For example, if the clue was MUD plus R, you would rearrange those letters to get DRUM. 15 nominal + d mandolin 20070819 In this week's on-air puzzle every answer is the name of a musical instrument. See if you can get it through its anagram. For example, if the clue was MUD plus R, you would rearrange those letters to get DRUM. 16 chairman + o harmonica 20070812 "The on-air puzzle is called, ""Where's It At?"" There are clues for two words. The first word contains the consecutive letters ""A-T"" somewhere inside. Remove the ""A-T"" and you'll get a new word that answers the second clue." 0 a saint honored on March 17; a puncturewith a pin or needle Patrick, prick 20070812 "The on-air puzzle is called, ""Where's It At?"" There are clues for two words. The first word contains the consecutive letters ""A-T"" somewhere inside. Remove the ""A-T"" and you'll get a new word that answers the second clue." 2 a grand speech; The Hunter constellation oration, Orion 20070812 "The on-air puzzle is called, ""Where's It At?"" There are clues for two words. The first word contains the consecutive letters ""A-T"" somewhere inside. Remove the ""A-T"" and you'll get a new word that answers the second clue." 3 a layer, as in rock; to play a guitar stratum, strum 20070812 "The on-air puzzle is called, ""Where's It At?"" There are clues for two words. The first word contains the consecutive letters ""A-T"" somewhere inside. Remove the ""A-T"" and you'll get a new word that answers the second clue." 4 a gorilla or human; a good grade of beef primate, prime 20070812 "The on-air puzzle is called, ""Where's It At?"" There are clues for two words. The first word contains the consecutive letters ""A-T"" somewhere inside. Remove the ""A-T"" and you'll get a new word that answers the second clue." 5 grandmother or uncle, for example; toexperience again relative, relive 20070812 "The on-air puzzle is called, ""Where's It At?"" There are clues for two words. The first word contains the consecutive letters ""A-T"" somewhere inside. Remove the ""A-T"" and you'll get a new word that answers the second clue." 6 chewing out; sea off the coast of Alaska berating, Bering 20070812 "The on-air puzzle is called, ""Where's It At?"" There are clues for two words. The first word contains the consecutive letters ""A-T"" somewhere inside. Remove the ""A-T"" and you'll get a new word that answers the second clue." 7 to fill to excess; French composer Erik satiate, Satie 20070812 "The on-air puzzle is called, ""Where's It At?"" There are clues for two words. The first word contains the consecutive letters ""A-T"" somewhere inside. Remove the ""A-T"" and you'll get a new word that answers the second clue." 8 shorthand or musical symbols, for example;idea notation, notion 20070812 "The on-air puzzle is called, ""Where's It At?"" There are clues for two words. The first word contains the consecutive letters ""A-T"" somewhere inside. Remove the ""A-T"" and you'll get a new word that answers the second clue." 9 New Jersey city north of Newark; any manor woman Paterson, person 20070812 "The on-air puzzle is called, ""Where's It At?"" There are clues for two words. The first word contains the consecutive letters ""A-T"" somewhere inside. Remove the ""A-T"" and you'll get a new word that answers the second clue." 10 what a secretary may take when a bosssends a letter; a manner of speaking dictation, diction 20070812 "The on-air puzzle is called, ""Where's It At?"" There are clues for two words. The first word contains the consecutive letters ""A-T"" somewhere inside. Remove the ""A-T"" and you'll get a new word that answers the second clue." 11 what a person may get when hit in themouth (two word phrase); to turn over fat lip, flip 20090607 "Each clue is a list of three things. The answer is what they all have in common. For example, if the clues are: """"a college graduate, a thermometer and longitude,"""" the answer would be """"degrees."""" Hint: Every answer, like """"degrees,"""" is a seven-letter plural." 0 kite, marionette, orchestra strings 20090607 "Each clue is a list of three things. The answer is what they all have in common. For example, if the clues are: """"a college graduate, a thermometer and longitude,"""" the answer would be """"degrees."""" Hint: Every answer, like """"degrees,"""" is a seven-letter plural." 1 pool table, kangaroo, pants pockets 20090607 "Each clue is a list of three things. The answer is what they all have in common. For example, if the clues are: """"a college graduate, a thermometer and longitude,"""" the answer would be """"degrees."""" Hint: Every answer, like """"degrees,"""" is a seven-letter plural." 2 word, post office, school's star athlete letters 20090607 "Each clue is a list of three things. The answer is what they all have in common. For example, if the clues are: """"a college graduate, a thermometer and longitude,"""" the answer would be """"degrees."""" Hint: Every answer, like """"degrees,"""" is a seven-letter plural." 3 disc jockey, hospital, ex-con records 20090607 "Each clue is a list of three things. The answer is what they all have in common. For example, if the clues are: """"a college graduate, a thermometer and longitude,"""" the answer would be """"degrees."""" Hint: Every answer, like """"degrees,"""" is a seven-letter plural." 4 credit card, ion, cavalry charges 20090607 "Each clue is a list of three things. The answer is what they all have in common. For example, if the clues are: """"a college graduate, a thermometer and longitude,"""" the answer would be """"degrees."""" Hint: Every answer, like """"degrees,"""" is a seven-letter plural." 5 jack-in-the-box, mattress, stream springs 20090607 "Each clue is a list of three things. The answer is what they all have in common. For example, if the clues are: """"a college graduate, a thermometer and longitude,"""" the answer would be """"degrees."""" Hint: Every answer, like """"degrees,"""" is a seven-letter plural." 6 April, bride to be, locker room showers 20090607 "Each clue is a list of three things. The answer is what they all have in common. For example, if the clues are: """"a college graduate, a thermometer and longitude,"""" the answer would be """"degrees."""" Hint: Every answer, like """"degrees,"""" is a seven-letter plural." 7 candy store, octopus, line at a side show suckers 20090607 "Each clue is a list of three things. The answer is what they all have in common. For example, if the clues are: """"a college graduate, a thermometer and longitude,"""" the answer would be """"degrees."""" Hint: Every answer, like """"degrees,"""" is a seven-letter plural." 8 porch, computer, suntan lotion screens 20090607 "Each clue is a list of three things. The answer is what they all have in common. For example, if the clues are: """"a college graduate, a thermometer and longitude,"""" the answer would be """"degrees."""" Hint: Every answer, like """"degrees,"""" is a seven-letter plural." 9 Parthenon, newspaper, spreadsheet columns 20090607 "Each clue is a list of three things. The answer is what they all have in common. For example, if the clues are: """"a college graduate, a thermometer and longitude,"""" the answer would be """"degrees."""" Hint: Every answer, like """"degrees,"""" is a seven-letter plural." 10 cigarette, swimming pool, coffee maker filters 20090607 "Each clue is a list of three things. The answer is what they all have in common. For example, if the clues are: """"a college graduate, a thermometer and longitude,"""" the answer would be """"degrees."""" Hint: Every answer, like """"degrees,"""" is a seven-letter plural." 11 ping-pong, canoe, fraternity member paddles 20070916 In this week's on-air puzzle, Will offers three words that start with the letters M, V and P, as in Most Valuable Player. Think of a word that can follow each of his, to complete a compound word or a familiar two-word phrase. For example: Mothers, Veterans and Pay — the answer would be DAY, as in Mother's Day, Veterans Day and Pay Day. 0 mail, voice and Pandora's (3-letter answers) box 20070916 In this week's on-air puzzle, Will offers three words that start with the letters M, V and P, as in Most Valuable Player. Think of a word that can follow each of his, to complete a compound word or a familiar two-word phrase. For example: Mothers, Veterans and Pay — the answer would be DAY, as in Mother's Day, Veterans Day and Pay Day. 1 mountain, volcanic and potMaster, Valentine's and playing (4-letter answers) ash card 20070916 In this week's on-air puzzle, Will offers three words that start with the letters M, V and P, as in Most Valuable Player. Think of a word that can follow each of his, to complete a compound word or a familiar two-word phrase. For example: Mothers, Veterans and Pay — the answer would be DAY, as in Mother's Day, Veterans Day and Pay Day. 2 minus, V and peace sign 20070916 In this week's on-air puzzle, Will offers three words that start with the letters M, V and P, as in Most Valuable Player. Think of a word that can follow each of his, to complete a compound word or a familiar two-word phrase. For example: Mothers, Veterans and Pay — the answer would be DAY, as in Mother's Day, Veterans Day and Pay Day. 3 moth, volley and paddle ball 20070916 In this week's on-air puzzle, Will offers three words that start with the letters M, V and P, as in Most Valuable Player. Think of a word that can follow each of his, to complete a compound word or a familiar two-word phrase. For example: Mothers, Veterans and Pay — the answer would be DAY, as in Mother's Day, Veterans Day and Pay Day. 4 murder, violin and pillow case 20070916 In this week's on-air puzzle, Will offers three words that start with the letters M, V and P, as in Most Valuable Player. Think of a word that can follow each of his, to complete a compound word or a familiar two-word phrase. For example: Mothers, Veterans and Pay — the answer would be DAY, as in Mother's Day, Veterans Day and Pay Day. 5 masking, video and police tape 20070916 In this week's on-air puzzle, Will offers three words that start with the letters M, V and P, as in Most Valuable Player. Think of a word that can follow each of his, to complete a compound word or a familiar two-word phrase. For example: Mothers, Veterans and Pay — the answer would be DAY, as in Mother's Day, Veterans Day and Pay Day. 6 match, vantage and pressure (5-letter answers) point 20070916 In this week's on-air puzzle, Will offers three words that start with the letters M, V and P, as in Most Valuable Player. Think of a word that can follow each of his, to complete a compound word or a familiar two-word phrase. For example: Mothers, Veterans and Pay — the answer would be DAY, as in Mother's Day, Veterans Day and Pay Day. 7 mental, vegetative and Penn state 20070916 In this week's on-air puzzle, Will offers three words that start with the letters M, V and P, as in Most Valuable Player. Think of a word that can follow each of his, to complete a compound word or a familiar two-word phrase. For example: Mothers, Veterans and Pay — the answer would be DAY, as in Mother's Day, Veterans Day and Pay Day. 8 magic, volume and prime (6 letters) number 20070916 In this week's on-air puzzle, Will offers three words that start with the letters M, V and P, as in Most Valuable Player. Think of a word that can follow each of his, to complete a compound word or a familiar two-word phrase. For example: Mothers, Veterans and Pay — the answer would be DAY, as in Mother's Day, Veterans Day and Pay Day. 9 mind, volume and pest (7 letters) control 20071014 "In this week's on-air puzzle, every answer is a seven-letter word or name that contains the consecutive letters, R-I-O, somewhere in it. For example, if the clue is """"inquisitive,"""" the answer is """"curious."""" " 0 lover of one's country patriot 20071014 "In this week's on-air puzzle, every answer is a seven-letter word or name that contains the consecutive letters, R-I-O, somewhere in it. For example, if the clue is """"inquisitive,"""" the answer is """"curious."""" " 1 Toronto's province Ontario 20071014 "In this week's on-air puzzle, every answer is a seven-letter word or name that contains the consecutive letters, R-I-O, somewhere in it. For example, if the clue is """"inquisitive,"""" the answer is """"curious."""" " 2 not joking around serious 20071014 "In this week's on-air puzzle, every answer is a seven-letter word or name that contains the consecutive letters, R-I-O, somewhere in it. For example, if the clue is """"inquisitive,"""" the answer is """"curious."""" " 3 baseball team that plays in Camden Yards Orioles 20071014 "In this week's on-air puzzle, every answer is a seven-letter word or name that contains the consecutive letters, R-I-O, somewhere in it. For example, if the clue is """"inquisitive,"""" the answer is """"curious."""" " 4 screamingly funny riotous 20071014 "In this week's on-air puzzle, every answer is a seven-letter word or name that contains the consecutive letters, R-I-O, somewhere in it. For example, if the clue is """"inquisitive,"""" the answer is """"curious."""" " 5 a man experienced in battle warrior 20071014 "In this week's on-air puzzle, every answer is a seven-letter word or name that contains the consecutive letters, R-I-O, somewhere in it. For example, if the clue is """"inquisitive,"""" the answer is """"curious."""" " 6 very angry furious 20071014 "In this week's on-air puzzle, every answer is a seven-letter word or name that contains the consecutive letters, R-I-O, somewhere in it. For example, if the clue is """"inquisitive,"""" the answer is """"curious."""" " 7 vehicle in Ben Hur chariot 20071014 "In this week's on-air puzzle, every answer is a seven-letter word or name that contains the consecutive letters, R-I-O, somewhere in it. For example, if the clue is """"inquisitive,"""" the answer is """"curious."""" " 8 light French roll or bun brioche 20071014 "In this week's on-air puzzle, every answer is a seven-letter word or name that contains the consecutive letters, R-I-O, somewhere in it. For example, if the clue is """"inquisitive,"""" the answer is """"curious."""" " 9 highly caustic criticism vitriol 20071014 "In this week's on-air puzzle, every answer is a seven-letter word or name that contains the consecutive letters, R-I-O, somewhere in it. For example, if the clue is """"inquisitive,"""" the answer is """"curious."""" " 10 diverse or assorted various 20071014 "In this week's on-air puzzle, every answer is a seven-letter word or name that contains the consecutive letters, R-I-O, somewhere in it. For example, if the clue is """"inquisitive,"""" the answer is """"curious."""" " 11 ancient trumpet or its shrill sound clarion 20071014 "In this week's on-air puzzle, every answer is a seven-letter word or name that contains the consecutive letters, R-I-O, somewhere in it. For example, if the clue is """"inquisitive,"""" the answer is """"curious."""" " 12 resident of Nicosia Cypriot 20071014 "In this week's on-air puzzle, every answer is a seven-letter word or name that contains the consecutive letters, R-I-O, somewhere in it. For example, if the clue is """"inquisitive,"""" the answer is """"curious."""" " 13 Richard who was mayor of Los Angeles from1993 to 2001 Riordan 20071014 "In this week's on-air puzzle, every answer is a seven-letter word or name that contains the consecutive letters, R-I-O, somewhere in it. For example, if the clue is """"inquisitive,"""" the answer is """"curious."""" " 14 deductive, as reasoning (2 word phrase) a priori 20071014 "In this week's on-air puzzle, every answer is a seven-letter word or name that contains the consecutive letters, R-I-O, somewhere in it. For example, if the clue is """"inquisitive,"""" the answer is """"curious."""" " 15 informal British good-bye cheerio 20080810 In the on-air puzzle, you are given several 10-letter words. For each word, rearrange the first five letters to spell a common word and the last five letters to spell another common word. Each pair of answers is unique. 0 unsteadily tunes, daily 20080810 In the on-air puzzle, you are given several 10-letter words. For each word, rearrange the first five letters to spell a common word and the last five letters to spell another common word. Each pair of answers is unique. 1 continuity tonic, unity 20080810 In the on-air puzzle, you are given several 10-letter words. For each word, rearrange the first five letters to spell a common word and the last five letters to spell another common word. Each pair of answers is unique. 2 allegorist legal, riots 20080810 In the on-air puzzle, you are given several 10-letter words. For each word, rearrange the first five letters to spell a common word and the last five letters to spell another common word. Each pair of answers is unique. 3 patriotism tapir, moist 20080810 In the on-air puzzle, you are given several 10-letter words. For each word, rearrange the first five letters to spell a common word and the last five letters to spell another common word. Each pair of answers is unique. 4 mercantile cream, inlet 20080810 In the on-air puzzle, you are given several 10-letter words. For each word, rearrange the first five letters to spell a common word and the last five letters to spell another common word. Each pair of answers is unique. 5 boisterous obits, rouse 20080810 In the on-air puzzle, you are given several 10-letter words. For each word, rearrange the first five letters to spell a common word and the last five letters to spell another common word. Each pair of answers is unique. 6 decahedron ached, drone 20080113 Every answer is a familiar two-word phrase, in which the first word starts with C-A and the second word starts with R. 1 what a store clerk uses to ring up purchases cash register 20080113 Every answer is a familiar two-word phrase, in which the first word starts with C-A and the second word starts with R. 2 a trip by taxi cab ride 20080113 Every answer is a familiar two-word phrase, in which the first word starts with C-A and the second word starts with R. 3 a walk on part in a movie cameo role 20080113 Every answer is a familiar two-word phrase, in which the first word starts with C-A and the second word starts with R. 4 place where they use branding irons cattle ranch 20080113 Every answer is a familiar two-word phrase, in which the first word starts with C-A and the second word starts with R. 5 the first James Bond book, and recent movieremake Casino Royale 20080113 Every answer is a familiar two-word phrase, in which the first word starts with C-A and the second word starts with R. 6 Oriole who set the record for the mostconsecutive games played Cal Ripkin 20080113 Every answer is a familiar two-word phrase, in which the first word starts with C-A and the second word starts with R. 7 dancing fruit in an old TV commercialRaisin California 20080113 Every answer is a familiar two-word phrase, in which the first word starts with C-A and the second word starts with R. 8 part of a typewriter that allows you to gofrom the end of one line to the start of thenext carriage return 20080113 Every answer is a familiar two-word phrase, in which the first word starts with C-A and the second word starts with R. 9 a kind of tube, as in a TV or radio cathode ray 20080113 Every answer is a familiar two-word phrase, in which the first word starts with C-A and the second word starts with R. 10 mountains in Alberta and British ColumbiaRockies Canadian 20080113 Every answer is a familiar two-word phrase, in which the first word starts with C-A and the second word starts with R. 11 John Steinbeck novel set in Monterey (Calif) Cannery Row 20080113 Every answer is a familiar two-word phrase, in which the first word starts with C-A and the second word starts with R. 12 Beatles label before Apple Capitol Records 20080113 Every answer is a familiar two-word phrase, in which the first word starts with C-A and the second word starts with R. 13 a division of an army on horsebackregiment cavalry 20080113 Every answer is a familiar two-word phrase, in which the first word starts with C-A and the second word starts with R. 14 baptism and first communion Catholic rites 20080113 Every answer is a familiar two-word phrase, in which the first word starts with C-A and the second word starts with R. 15 non-profit group devoted to finding fosterhomes for dogs canine rescue 20080113 Every answer is a familiar two-word phrase, in which the first word starts with C-A and the second word starts with R. 16 lakefront or riverfront business that offerspaddles canoe rental 20080406 "In this week's puzzle, every answer is a familiar two-word phrase, where the first word starts with V and the second word starts with A. For example, given """"subject of a tax in Britain"""" the answer would be """"value added.""""" 1 second in command in the navy vice admiral 20080406 "In this week's puzzle, every answer is a familiar two-word phrase, where the first word starts with V and the second word starts with A. For example, given """"subject of a tax in Britain"""" the answer would be """"value added.""""" 2 residue around Mauna Loa, for example volcanic ash 20080406 "In this week's puzzle, every answer is a familiar two-word phrase, where the first word starts with V and the second word starts with A. For example, given """"subject of a tax in Britain"""" the answer would be """"value added.""""" 3 law that ushered in Prohibition Volstead Act 20080406 "In this week's puzzle, every answer is a familiar two-word phrase, where the first word starts with V and the second word starts with A. For example, given """"subject of a tax in Britain"""" the answer would be """"value added.""""" 4 painting, sculpture, photography, etc. visual arts 20070701 The answer is a familiar two-word phrase or name in which the first words starts with L and the second word starts with R. 1 capital of Arkansas Little Rock 20070701 The answer is a familiar two-word phrase or name in which the first words starts with L and the second word starts with R. 2 inflatable item on a ship life raft 20070701 The answer is a familiar two-word phrase or name in which the first words starts with L and the second word starts with R. 3 item on top of a house that is useful in a storm lightning rod 20070701 The answer is a familiar two-word phrase or name in which the first words starts with L and the second word starts with R. 4 a person who knows what you are saying not byhearing lip reader 20070701 The answer is a familiar two-word phrase or name in which the first words starts with L and the second word starts with R. 5 "Good Golly Miss Molly"" singer" Little Richard 20070701 The answer is a familiar two-word phrase or name in which the first words starts with L and the second word starts with R. 6 "singer with the 1977 hit ""Blue Bayou""" Linda Ronstadt 20070701 The answer is a familiar two-word phrase or name in which the first words starts with L and the second word starts with R. 7 "singer with #1 hits ""Truly"" and ""Say You, SayMe""" Lionel Richie 20070701 The answer is a familiar two-word phrase or name in which the first words starts with L and the second word starts with R. 8 "country singer with the #1 hit, ""Something'sGotta Give""" LeAnn Rimes 20070701 The answer is a familiar two-word phrase or name in which the first words starts with L and the second word starts with R. 9 pink light red 20070701 The answer is a familiar two-word phrase or name in which the first words starts with L and the second word starts with R. 10 starring part in a movie or play leading role 20070701 The answer is a familiar two-word phrase or name in which the first words starts with L and the second word starts with R. 11 publication for students doing legal studies law review 20070701 The answer is a familiar two-word phrase or name in which the first words starts with L and the second word starts with R. 12 animal that runs a maze lab rat 20070701 The answer is a familiar two-word phrase or name in which the first words starts with L and the second word starts with R. 13 apparatus on top of a car luggage rack 20070701 The answer is a familiar two-word phrase or name in which the first words starts with L and the second word starts with R. 14 what you turn to when all other efforts failnot last rites last resort, 20080914 "Each answer is a familiar two-word phrase in which the first word ends in R-E and the second word starts with C. For example, if the clue is """"text that comes from the Associated Press or Reuters,"""" the answer would be """"wire copy.""""" 0 what BC means in a date Before Christ 20080914 "Each answer is a familiar two-word phrase in which the first word ends in R-E and the second word starts with C. For example, if the clue is """"text that comes from the Associated Press or Reuters,"""" the answer would be """"wire copy.""""" 1 container for a pirate's jewels and gold treasure chest 20080914 "Each answer is a familiar two-word phrase in which the first word ends in R-E and the second word starts with C. For example, if the clue is """"text that comes from the Associated Press or Reuters,"""" the answer would be """"wire copy.""""" 2 numismatist find rare coin 20080914 "Each answer is a familiar two-word phrase in which the first word ends in R-E and the second word starts with C. For example, if the clue is """"text that comes from the Associated Press or Reuters,"""" the answer would be """"wire copy.""""" 3 group in the second year of high school sophomore class 20080914 "Each answer is a familiar two-word phrase in which the first word ends in R-E and the second word starts with C. For example, if the clue is """"text that comes from the Associated Press or Reuters,"""" the answer would be """"wire copy.""""" 4 French scientist who co-discovered radium Pierre Curie 20080914 "Each answer is a familiar two-word phrase in which the first word ends in R-E and the second word starts with C. For example, if the clue is """"text that comes from the Associated Press or Reuters,"""" the answer would be """"wire copy.""""" 5 rules for building safety in case ofconflagration fire code 20080914 "Each answer is a familiar two-word phrase in which the first word ends in R-E and the second word starts with C. For example, if the clue is """"text that comes from the Associated Press or Reuters,"""" the answer would be """"wire copy.""""" 6 aid for improving traction on snowyhighways tire chains 20080914 "Each answer is a familiar two-word phrase in which the first word ends in R-E and the second word starts with C. For example, if the clue is """"text that comes from the Associated Press or Reuters,"""" the answer would be """"wire copy.""""" 7 pot for preparing stews and meat quickly pressure cooker 20080914 "Each answer is a familiar two-word phrase in which the first word ends in R-E and the second word starts with C. For example, if the clue is """"text that comes from the Associated Press or Reuters,"""" the answer would be """"wire copy.""""" 8 1980's pop group led by Boy George Culture Club 20080914 "Each answer is a familiar two-word phrase in which the first word ends in R-E and the second word starts with C. For example, if the clue is """"text that comes from the Associated Press or Reuters,"""" the answer would be """"wire copy.""""" 9 item a golfer carries with a pencil score card 20080914 "Each answer is a familiar two-word phrase in which the first word ends in R-E and the second word starts with C. For example, if the clue is """"text that comes from the Associated Press or Reuters,"""" the answer would be """"wire copy.""""" 10 pennies, nickels, dimes, etc., that you arefree to give or lend sparechange 20080914 "Each answer is a familiar two-word phrase in which the first word ends in R-E and the second word starts with C. For example, if the clue is """"text that comes from the Associated Press or Reuters,"""" the answer would be """"wire copy.""""" 11 what a thermostat affords temperature control 20080914 "Each answer is a familiar two-word phrase in which the first word ends in R-E and the second word starts with C. For example, if the clue is """"text that comes from the Associated Press or Reuters,"""" the answer would be """"wire copy.""""" 12 grinning animal in Alice in Wonderland Cheshire Cat 20090125 "The clues are sentences. Each sentence conceals the name of a make of an automobile somewhere in consecutive letters inside it. You name the automobile. For example, if the clue is """"give them a 'z,' the answer would be Mazda because it's hidden in consecutive letters." 0 Alex uses hair dye. Lexus 20090125 "The clues are sentences. Each sentence conceals the name of a make of an automobile somewhere in consecutive letters inside it. You name the automobile. For example, if the clue is """"give them a 'z,' the answer would be Mazda because it's hidden in consecutive letters." 1 We were attacked by Ninja guards. Jaguar 20090125 "The clues are sentences. Each sentence conceals the name of a make of an automobile somewhere in consecutive letters inside it. You name the automobile. For example, if the clue is """"give them a 'z,' the answer would be Mazda because it's hidden in consecutive letters." 2 The vapor's chemicals overwhelmed me. Porsche 20090125 "The clues are sentences. Each sentence conceals the name of a make of an automobile somewhere in consecutive letters inside it. You name the automobile. For example, if the clue is """"give them a 'z,' the answer would be Mazda because it's hidden in consecutive letters." 3 At DuPont, I accomplished a lot. Pontiac 20090125 "The clues are sentences. Each sentence conceals the name of a make of an automobile somewhere in consecutive letters inside it. You name the automobile. For example, if the clue is """"give them a 'z,' the answer would be Mazda because it's hidden in consecutive letters." 4 Ellen simply mouthed the words. Plymouth 20090125 "The clues are sentences. Each sentence conceals the name of a make of an automobile somewhere in consecutive letters inside it. You name the automobile. For example, if the clue is """"give them a 'z,' the answer would be Mazda because it's hidden in consecutive letters." 5 It was Harold's mob I let loose. Oldsmobile 20090125 "The clues are sentences. Each sentence conceals the name of a make of an automobile somewhere in consecutive letters inside it. You name the automobile. For example, if the clue is """"give them a 'z,' the answer would be Mazda because it's hidden in consecutive letters." 6 Is the golfer rarin' to go? Ferrari 20090125 "The clues are sentences. Each sentence conceals the name of a make of an automobile somewhere in consecutive letters inside it. You name the automobile. For example, if the clue is """"give them a 'z,' the answer would be Mazda because it's hidden in consecutive letters." 7 The subway has a turnstile. Saturn 20090125 "The clues are sentences. Each sentence conceals the name of a make of an automobile somewhere in consecutive letters inside it. You name the automobile. For example, if the clue is """"give them a 'z,' the answer would be Mazda because it's hidden in consecutive letters." 8 Alan drove really fast. Land Rover 20100829 "You are given sentences with the last two words missing. Add the letter F before the missing word that goes first to get a new word that follows it. For example, given """"Warm weather continued from the entire summer. Now it is expected to last _____ _____,"""" the answer would be """"all fall.""""" 1 If Sinatra were on American Idol, howwould the judges _____ _____? rank Frank 20100829 "You are given sentences with the last two words missing. Add the letter F before the missing word that goes first to get a new word that follows it. For example, given """"Warm weather continued from the entire summer. Now it is expected to last _____ _____,"""" the answer would be """"all fall.""""" 2 The atmospheric conditions were neitherreally good nor really bad, so thepollution monitor rated the _____ _____. air fair 20100829 "You are given sentences with the last two words missing. Add the letter F before the missing word that goes first to get a new word that follows it. For example, given """"Warm weather continued from the entire summer. Now it is expected to last _____ _____,"""" the answer would be """"all fall.""""" 3 With only twenty percent of the plane'sseats filled, the crew called it a __________. light flight 20100829 "You are given sentences with the last two words missing. Add the letter F before the missing word that goes first to get a new word that follows it. For example, given """"Warm weather continued from the entire summer. Now it is expected to last _____ _____,"""" the answer would be """"all fall.""""" 4 The cruise shipped docked at Helsinki'sharbor, after which we got off andtraveled away from the coast to explore_____ _____. inland Finland 20100829 "You are given sentences with the last two words missing. Add the letter F before the missing word that goes first to get a new word that follows it. For example, given """"Warm weather continued from the entire summer. Now it is expected to last _____ _____,"""" the answer would be """"all fall.""""" 5 Of the two kinds of terrain, we wouldcall the former hillier and the __________. latter flatter 20100829 "You are given sentences with the last two words missing. Add the letter F before the missing word that goes first to get a new word that follows it. For example, given """"Warm weather continued from the entire summer. Now it is expected to last _____ _____,"""" the answer would be """"all fall.""""" 6 Some electrical systems have circuitbreakers, but ours _____ _____. uses fuses 20100829 "You are given sentences with the last two words missing. Add the letter F before the missing word that goes first to get a new word that follows it. For example, given """"Warm weather continued from the entire summer. Now it is expected to last _____ _____,"""" the answer would be """"all fall.""""" 7 The newer file didn't have all thepapers I wanted, so I reached for an_____ _____. older folder 20100829 "You are given sentences with the last two words missing. Add the letter F before the missing word that goes first to get a new word that follows it. For example, given """"Warm weather continued from the entire summer. Now it is expected to last _____ _____,"""" the answer would be """"all fall.""""" 8 A person eating a lot of food a weekafter Palm Sunday might be called an_____ _____. Easter feaster 20100829 "You are given sentences with the last two words missing. Add the letter F before the missing word that goes first to get a new word that follows it. For example, given """"Warm weather continued from the entire summer. Now it is expected to last _____ _____,"""" the answer would be """"all fall.""""" 9 In making a horror film, you have tocombine many scary elements to producejust the _____ _____. right fright 20100829 "You are given sentences with the last two words missing. Add the letter F before the missing word that goes first to get a new word that follows it. For example, given """"Warm weather continued from the entire summer. Now it is expected to last _____ _____,"""" the answer would be """"all fall.""""" 10 If you're carrying a concealed weaponaround a police officer, and he'ssuspicious, you are _____ _____. risking frisking 20100829 "You are given sentences with the last two words missing. Add the letter F before the missing word that goes first to get a new word that follows it. For example, given """"Warm weather continued from the entire summer. Now it is expected to last _____ _____,"""" the answer would be """"all fall.""""" 11 The other team had us down for 79points, but there should have been onemore to make _____ _____. (Each of thetwo answers has two words, add the Fonly to the first word.) our score four score 20070520 Each sentence read conceals the name of a world capital in consecutive letters. 0 Captain Nemo scowled. Moscow 20070520 Each sentence read conceals the name of a world capital in consecutive letters. 2 The soil is bone dry. Lisbon 20070520 Each sentence read conceals the name of a world capital in consecutive letters. 3 The jeweler is picking stones. Kingston 20070520 Each sentence read conceals the name of a world capital in consecutive letters. 5 We had pizza, then saw a movie. Athens 20070520 Each sentence read conceals the name of a world capital in consecutive letters. 6 Did you see the Hindu blink? Dublin 20070520 Each sentence read conceals the name of a world capital in consecutive letters. 7 That radio station runs on-air obituaries. Nairobi 20070520 Each sentence read conceals the name of a world capital in consecutive letters. 8 Here is the pheasant I agonized about shooting. Santiago 20070520 Each sentence read conceals the name of a world capital in consecutive letters. 10 Kids don't need satchels in kindergarten. Helsinki 20100613 "Every answer is a familiar two-word phrase in which the first word ends in the letter M and the second word starts with C. For example, given """"Lysol or Comet product,"""" the answer would be """"bathroom cleaner.""""" 1 it might wake you up in the morning alarm clock 20100613 "Every answer is a familiar two-word phrase in which the first word ends in the letter M and the second word starts with C. For example, given """"Lysol or Comet product,"""" the answer would be """"bathroom cleaner.""""" 2 Manhattan or New England food item clam chowder 20100613 "Every answer is a familiar two-word phrase in which the first word ends in the letter M and the second word starts with C. For example, given """"Lysol or Comet product,"""" the answer would be """"bathroom cleaner.""""" 3 part of a s'more graham cracker 20100613 "Every answer is a familiar two-word phrase in which the first word ends in the letter M and the second word starts with C. For example, given """"Lysol or Comet product,"""" the answer would be """"bathroom cleaner.""""" 4 a constant brat; also the name of a movie problem child 20100613 "Every answer is a familiar two-word phrase in which the first word ends in the letter M and the second word starts with C. For example, given """"Lysol or Comet product,"""" the answer would be """"bathroom cleaner.""""" 5 below ground shelter during a tornado storm cellar 20100613 "Every answer is a familiar two-word phrase in which the first word ends in the letter M and the second word starts with C. For example, given """"Lysol or Comet product,"""" the answer would be """"bathroom cleaner.""""" 6 Hoover product vacuum cleaner 20100613 "Every answer is a familiar two-word phrase in which the first word ends in the letter M and the second word starts with C. For example, given """"Lysol or Comet product,"""" the answer would be """"bathroom cleaner.""""" 7 PE gym class 20100613 "Every answer is a familiar two-word phrase in which the first word ends in the letter M and the second word starts with C. For example, given """"Lysol or Comet product,"""" the answer would be """"bathroom cleaner.""""" 8 title for a woman who heads a meetingmadam chairman madam chair or 20100613 "Every answer is a familiar two-word phrase in which the first word ends in the letter M and the second word starts with C. For example, given """"Lysol or Comet product,"""" the answer would be """"bathroom cleaner.""""" 9 cocktail made with gin and lemon juice;it's the name of a person Tom Collins 20100613 "Every answer is a familiar two-word phrase in which the first word ends in the letter M and the second word starts with C. For example, given """"Lysol or Comet product,"""" the answer would be """"bathroom cleaner.""""" 10 what's seen after a nuclear test mushroom cloud 20100613 "Every answer is a familiar two-word phrase in which the first word ends in the letter M and the second word starts with C. For example, given """"Lysol or Comet product,"""" the answer would be """"bathroom cleaner.""""" 11 spread for a bagel cream cheese 20100613 "Every answer is a familiar two-word phrase in which the first word ends in the letter M and the second word starts with C. For example, given """"Lysol or Comet product,"""" the answer would be """"bathroom cleaner.""""" 12 place in the house to keep a mop broom closet 20100613 "Every answer is a familiar two-word phrase in which the first word ends in the letter M and the second word starts with C. For example, given """"Lysol or Comet product,"""" the answer would be """"bathroom cleaner.""""" 13 table salt, chemically sodium chloride 20080831 "Each word has 2 syllables. The first vowel sound in the first word is a long """"E."""" Change this to a short """"E,"""" and phonetically you'll get a new word that answers the second clue. For example: if the clue is """"slang term for an eye, and partner for salt,"""" the answer would be: """"peeper and pepper.""""" 1 top of a room; opposite of buying ceiling, selling 20080831 "Each word has 2 syllables. The first vowel sound in the first word is a long """"E."""" Change this to a short """"E,"""" and phonetically you'll get a new word that answers the second clue. For example: if the clue is """"slang term for an eye, and partner for salt,"""" the answer would be: """"peeper and pepper.""""" 2 prominent polling organization; wrestlinghold Nielson, nelson 20080831 "Each word has 2 syllables. The first vowel sound in the first word is a long """"E."""" Change this to a short """"E,"""" and phonetically you'll get a new word that answers the second clue. For example: if the clue is """"slang term for an eye, and partner for salt,"""" the answer would be: """"peeper and pepper.""""" 3 term of endearment; like gym clothes afterexercising sweetie, sweaty 20080831 "Each word has 2 syllables. The first vowel sound in the first word is a long """"E."""" Change this to a short """"E,"""" and phonetically you'll get a new word that answers the second clue. For example: if the clue is """"slang term for an eye, and partner for salt,"""" the answer would be: """"peeper and pepper.""""" 4 unit of petrol; a postal delivery liter, letter 20080831 "Each word has 2 syllables. The first vowel sound in the first word is a long """"E."""" Change this to a short """"E,"""" and phonetically you'll get a new word that answers the second clue. For example: if the clue is """"slang term for an eye, and partner for salt,"""" the answer would be: """"peeper and pepper.""""" 5 a lab container; former tennis champ Boris beaker, Becker 20080831 "Each word has 2 syllables. The first vowel sound in the first word is a long """"E."""" Change this to a short """"E,"""" and phonetically you'll get a new word that answers the second clue. For example: if the clue is """"slang term for an eye, and partner for salt,"""" the answer would be: """"peeper and pepper.""""" 6 ignoring the consequences; like thehorsemen in The Legend of Sleepy Hollow heedless, headless 20080831 "Each word has 2 syllables. The first vowel sound in the first word is a long """"E."""" Change this to a short """"E,"""" and phonetically you'll get a new word that answers the second clue. For example: if the clue is """"slang term for an eye, and partner for salt,"""" the answer would be: """"peeper and pepper.""""" 7 lighthouse light; call to come closer beacon, beckon 20080831 "Each word has 2 syllables. The first vowel sound in the first word is a long """"E."""" Change this to a short """"E,"""" and phonetically you'll get a new word that answers the second clue. For example: if the clue is """"slang term for an eye, and partner for salt,"""" the answer would be: """"peeper and pepper.""""" 8 butcher's knife; ingenious cleaver, clever 20100214 "Every answer is a word whose second syllable is """"pen."""" For example, given the clue, """"a child, say, on one's tax return,"""" the answer is, """"de-pen-dent.""""" 0 costly expensive 20100214 "Every answer is a word whose second syllable is """"pen."""" For example, given the clue, """"a child, say, on one's tax return,"""" the answer is, """"de-pen-dent.""""" 1 capital of Denmark Copenhagen 20100214 "Every answer is a word whose second syllable is """"pen."""" For example, given the clue, """"a child, say, on one's tax return,"""" the answer is, """"de-pen-dent.""""" 2 something at the end of a book appendix 20100214 "Every answer is a word whose second syllable is """"pen."""" For example, given the clue, """"a child, say, on one's tax return,"""" the answer is, """"de-pen-dent.""""" 3 book that is a compilation compendium 20100214 "Every answer is a word whose second syllable is """"pen."""" For example, given the clue, """"a child, say, on one's tax return,"""" the answer is, """"de-pen-dent.""""" 4 worker with a hammer and nails carpenter 20100214 "Every answer is a word whose second syllable is """"pen."""" For example, given the clue, """"a child, say, on one's tax return,"""" the answer is, """"de-pen-dent.""""" 5 things that hold the pants up suspenders 20100214 "Every answer is a word whose second syllable is """"pen."""" For example, given the clue, """"a child, say, on one's tax return,"""" the answer is, """"de-pen-dent.""""" 6 fluid used in thinning paints turpentine 20100214 "Every answer is a word whose second syllable is """"pen."""" For example, given the clue, """"a child, say, on one's tax return,"""" the answer is, """"de-pen-dent.""""" 7 Pez candy has a fancy one dispenser 20100214 "Every answer is a word whose second syllable is """"pen."""" For example, given the clue, """"a child, say, on one's tax return,"""" the answer is, """"de-pen-dent.""""" 8 snakelike or winding serpentine 20100214 "Every answer is a word whose second syllable is """"pen."""" For example, given the clue, """"a child, say, on one's tax return,"""" the answer is, """"de-pen-dent.""""" 9 event that happens by accident happenstance 20100214 "Every answer is a word whose second syllable is """"pen."""" For example, given the clue, """"a child, say, on one's tax return,"""" the answer is, """"de-pen-dent.""""" 10 device that puts a point on a pencil sharpener 20100214 "Every answer is a word whose second syllable is """"pen."""" For example, given the clue, """"a child, say, on one's tax return,"""" the answer is, """"de-pen-dent.""""" 11 experiencing regret for one's actions repentant 20100214 "Every answer is a word whose second syllable is """"pen."""" For example, given the clue, """"a child, say, on one's tax return,"""" the answer is, """"de-pen-dent.""""" 12 fantastic stupendous 20070624 "In the on-air puzzle this week, clues are given to two words. If you insert the letters ""I-T"" somewhere in the first word, you get the second word." 1 person from Warsaw; courteous Pole, polite 20070624 "In the on-air puzzle this week, clues are given to two words. If you insert the letters ""I-T"" somewhere in the first word, you get the second word." 2 at what time; to blanch when, whiten 20070624 "In the on-air puzzle this week, clues are given to two words. If you insert the letters ""I-T"" somewhere in the first word, you get the second word." 3 penalty for speeding; limited fine, finite 20070624 "In the on-air puzzle this week, clues are given to two words. If you insert the letters ""I-T"" somewhere in the first word, you get the second word." 4 center of the intellect; where London is brain, Britain 20070624 "In the on-air puzzle this week, clues are given to two words. If you insert the letters ""I-T"" somewhere in the first word, you get the second word." 5 topping for mashed potatoes; subject studiedby Sir Isaac Newton gravy, gravity 20070624 "In the on-air puzzle this week, clues are given to two words. If you insert the letters ""I-T"" somewhere in the first word, you get the second word." 6 playing marble; to stir up agate, agitate 20070624 "In the on-air puzzle this week, clues are given to two words. If you insert the letters ""I-T"" somewhere in the first word, you get the second word." 7 a small donkey used as a pack animal; ahand-held Mexican dish burro, burrito 20070624 "In the on-air puzzle this week, clues are given to two words. If you insert the letters ""I-T"" somewhere in the first word, you get the second word." 8 container for oranges and lemons; a salt crate, citrate 20081109 "Every answer is an anagram of a word starting with the letters, A-M. For example, if the clue is """"A-M + sue,"""" then the answer would be, """"amuse.""""" 0 am + hubs ambush 20081109 "Every answer is an anagram of a word starting with the letters, A-M. For example, if the clue is """"A-M + sue,"""" then the answer would be, """"amuse.""""" 1 am + unto amount 20081109 "Every answer is an anagram of a word starting with the letters, A-M. For example, if the clue is """"A-M + sue,"""" then the answer would be, """"amuse.""""" 2 am + Styne amnesty 20081109 "Every answer is an anagram of a word starting with the letters, A-M. For example, if the clue is """"A-M + sue,"""" then the answer would be, """"amuse.""""" 3 am + tee up amputee 20081109 "Every answer is an anagram of a word starting with the letters, A-M. For example, if the clue is """"A-M + sue,"""" then the answer would be, """"amuse.""""" 4 am + be nice ambience 20081109 "Every answer is an anagram of a word starting with the letters, A-M. For example, if the clue is """"A-M + sue,"""" then the answer would be, """"amuse.""""" 5 am + isobar ambrosia 20081109 "Every answer is an anagram of a word starting with the letters, A-M. For example, if the clue is """"A-M + sue,"""" then the answer would be, """"amuse.""""" 6 am + panders ampersand 20081109 "Every answer is an anagram of a word starting with the letters, A-M. For example, if the clue is """"A-M + sue,"""" then the answer would be, """"amuse.""""" 7 am + thesauri amateurish 20100808 "Every answer is a familiar two-word phrase or name with the initials V.C. For example, given """"serving of calf's meat,"""" the answer would be """"veal chop.""""" 1 piece of equipment for a housekeeper vacuum cleaner 20100808 "Every answer is a familiar two-word phrase or name with the initials V.C. For example, given """"serving of calf's meat,"""" the answer would be """"veal chop.""""" 2 what to give someone on February 14Valentine's candy Valentine's card or 20100808 "Every answer is a familiar two-word phrase or name with the initials V.C. For example, given """"serving of calf's meat,"""" the answer would be """"veal chop.""""" 3 a small handbag for women's cosmeticsand toiletries, sometimes with a smallmirror vanity case 20100808 "Every answer is a familiar two-word phrase or name with the initials V.C. For example, given """"serving of calf's meat,"""" the answer would be """"veal chop.""""" 4 women use it to clean the skin andremove age spots. vanishing cream 20100808 "Every answer is a familiar two-word phrase or name with the initials V.C. For example, given """"serving of calf's meat,"""" the answer would be """"veal chop.""""" 5 world's smallest country in size Vatican City 20100808 "Every answer is a familiar two-word phrase or name with the initials V.C. For example, given """"serving of calf's meat,"""" the answer would be """"veal chop.""""" 6 processed dairy product made by Kraft Velveeta Cheese 20070429 Every answer today is a compound word or a familiar two-word phrase in which each half has exactly four letters. And the middle two letters of the first word are the same as the middle two letters of the last. 1 picnic competition that involves hopping sack race 20070429 Every answer today is a compound word or a familiar two-word phrase in which each half has exactly four letters. And the middle two letters of the first word are the same as the middle two letters of the last. 2 It has 366 days, and it comes aroundquadrennially leap year 20070429 Every answer today is a compound word or a familiar two-word phrase in which each half has exactly four letters. And the middle two letters of the first word are the same as the middle two letters of the last. 3 nickname for Hickock in the Old West Wild Bill 20070429 Every answer today is a compound word or a familiar two-word phrase in which each half has exactly four letters. And the middle two letters of the first word are the same as the middle two letters of the last. 4 fenced in area for cows, horses orchickens barnyard or farm yard 20070429 Every answer today is a compound word or a familiar two-word phrase in which each half has exactly four letters. And the middle two letters of the first word are the same as the middle two letters of the last. 5 an absolute tie dead heat 20070429 Every answer today is a compound word or a familiar two-word phrase in which each half has exactly four letters. And the middle two letters of the first word are the same as the middle two letters of the last. 6 each one has a fairway and a green golf hole 20070429 Every answer today is a compound word or a familiar two-word phrase in which each half has exactly four letters. And the middle two letters of the first word are the same as the middle two letters of the last. 7 co-author of the Communist Manifesto Karl Marx 20070429 Every answer today is a compound word or a familiar two-word phrase in which each half has exactly four letters. And the middle two letters of the first word are the same as the middle two letters of the last. 8 a written series of things one wouldlove to get or do wish list 20070429 Every answer today is a compound word or a familiar two-word phrase in which each half has exactly four letters. And the middle two letters of the first word are the same as the middle two letters of the last. 9 an event to unload unwanted Summermerchandise Fall sale 20070429 Every answer today is a compound word or a familiar two-word phrase in which each half has exactly four letters. And the middle two letters of the first word are the same as the middle two letters of the last. 10 middle ground between black and whiteextremes (involves a British spelling) grey area 20070429 Every answer today is a compound word or a familiar two-word phrase in which each half has exactly four letters. And the middle two letters of the first word are the same as the middle two letters of the last. 11 where to play eight ball poolroom 20071111 Every answer in this week's on-air puzzle is the name of an opera. The clues are in the form of anagrams. 0 coat + s Tosca 20071111 Every answer in this week's on-air puzzle is the name of an opera. The clues are in the form of anagrams. 2 cream + n Carmen 20071111 Every answer in this week's on-air puzzle is the name of an opera. The clues are in the form of anagrams. 3 foiled + i Fidelio 20071111 Every answer in this week's on-air puzzle is the name of an opera. The clues are in the form of anagrams. 4 relate + k (classic Greek) Elektra 20071111 Every answer in this week's on-air puzzle is the name of an opera. The clues are in the form of anagrams. 5 rotunda + t (by Puccini; a solid word; a name) Turandot 20071111 Every answer in this week's on-air puzzle is the name of an opera. The clues are in the form of anagrams. 6 colored + q (French; three words) Le Coq d'Or 20071111 Every answer in this week's on-air puzzle is the name of an opera. The clues are in the form of anagrams. 7 fireside + g (Wagner; a solid name; part of TheRing Cycle; first letter is S) Siegfried 20071111 Every answer in this week's on-air puzzle is the name of an opera. The clues are in the form of anagrams. 8 doghandlers + i (Wagner; two words; first word isa German article; second word is a brand of beer) Das Rheingold 20090104 "Every answer is a familiar two-word phrase or name, in which the first word starts with """"PA"""" and the second word ends in """"N."""" For example, if the clue is, """"namesake of a popular pizzeria chain,"""" the answer would be, """"Papa John,"""" because it starts with """"PA"""" and ends in an """"N.""""" 1 body of water along the West Coast Pacific Ocean 20090104 "Every answer is a familiar two-word phrase or name, in which the first word starts with """"PA"""" and the second word ends in """"N."""" For example, if the clue is, """"namesake of a popular pizzeria chain,"""" the answer would be, """"Papa John,"""" because it starts with """"PA"""" and ends in an """"N.""""" 2 socialite and hotel heiress once seen onThe Simple Life Paris Hilton 20090104 "Every answer is a familiar two-word phrase or name, in which the first word starts with """"PA"""" and the second word ends in """"N."""" For example, if the clue is, """"namesake of a popular pizzeria chain,"""" the answer would be, """"Papa John,"""" because it starts with """"PA"""" and ends in an """"N.""""" 3 once the most common bird in America, itbecame extinct in 1914 passenger pigeon 20090104 "Every answer is a familiar two-word phrase or name, in which the first word starts with """"PA"""" and the second word ends in """"N."""" For example, if the clue is, """"namesake of a popular pizzeria chain,"""" the answer would be, """"Papa John,"""" because it starts with """"PA"""" and ends in an """"N.""""" 4 fictional lumberjack famous in Tall Tales Paul Bunyon 20090104 "Every answer is a familiar two-word phrase or name, in which the first word starts with """"PA"""" and the second word ends in """"N."""" For example, if the clue is, """"namesake of a popular pizzeria chain,"""" the answer would be, """"Papa John,"""" because it starts with """"PA"""" and ends in an """"N.""""" 5 First Lady after Lady Bird Johnson Patricia Nixon 20090104 "Every answer is a familiar two-word phrase or name, in which the first word starts with """"PA"""" and the second word ends in """"N."""" For example, if the clue is, """"namesake of a popular pizzeria chain,"""" the answer would be, """"Papa John,"""" because it starts with """"PA"""" and ends in an """"N.""""" 6 When police arrest a lot of people, what paddy wagon, or 20090104 "Every answer is a familiar two-word phrase or name, in which the first word starts with """"PA"""" and the second word ends in """"N."""" For example, if the clue is, """"namesake of a popular pizzeria chain,"""" the answer would be, """"Papa John,"""" because it starts with """"PA"""" and ends in an """"N.""""" 7 they take them away in patrol wagon 20090104 "Every answer is a familiar two-word phrase or name, in which the first word starts with """"PA"""" and the second word ends in """"N."""" For example, if the clue is, """"namesake of a popular pizzeria chain,"""" the answer would be, """"Papa John,"""" because it starts with """"PA"""" and ends in an """"N.""""" 8 container at a Benjamin Moore orSherwin-Williams store paint can 20090104 "Every answer is a familiar two-word phrase or name, in which the first word starts with """"PA"""" and the second word ends in """"N."""" For example, if the clue is, """"namesake of a popular pizzeria chain,"""" the answer would be, """"Papa John,"""" because it starts with """"PA"""" and ends in an """"N.""""" 9 actress and sex symbol who co-starred inTV's Baywatch Pamela Anderson 20090104 "Every answer is a familiar two-word phrase or name, in which the first word starts with """"PA"""" and the second word ends in """"N."""" For example, if the clue is, """"namesake of a popular pizzeria chain,"""" the answer would be, """"Papa John,"""" because it starts with """"PA"""" and ends in an """"N.""""" 10 1973 film starring Ryan and Tatum O'Neal Paper Moon 20090104 "Every answer is a familiar two-word phrase or name, in which the first word starts with """"PA"""" and the second word ends in """"N."""" For example, if the clue is, """"namesake of a popular pizzeria chain,"""" the answer would be, """"Papa John,"""" because it starts with """"PA"""" and ends in an """"N.""""" 11 writing implement that competes with Cross Papermate pen, 20090104 "Every answer is a familiar two-word phrase or name, in which the first word starts with """"PA"""" and the second word ends in """"N."""" For example, if the clue is, """"namesake of a popular pizzeria chain,"""" the answer would be, """"Papa John,"""" because it starts with """"PA"""" and ends in an """"N.""""" 12 and Sheaffer Parker pen 20090104 "Every answer is a familiar two-word phrase or name, in which the first word starts with """"PA"""" and the second word ends in """"N."""" For example, if the clue is, """"namesake of a popular pizzeria chain,"""" the answer would be, """"Papa John,"""" because it starts with """"PA"""" and ends in an """"N.""""" 13 classic arcade game that involves chompingdots in a maze Pac-Man 20090104 "Every answer is a familiar two-word phrase or name, in which the first word starts with """"PA"""" and the second word ends in """"N."""" For example, if the clue is, """"namesake of a popular pizzeria chain,"""" the answer would be, """"Papa John,"""" because it starts with """"PA"""" and ends in an """"N.""""" 14 what a meter maid gives a ticket for parking violation 20090104 "Every answer is a familiar two-word phrase or name, in which the first word starts with """"PA"""" and the second word ends in """"N."""" For example, if the clue is, """"namesake of a popular pizzeria chain,"""" the answer would be, """"Papa John,"""" because it starts with """"PA"""" and ends in an """"N.""""" 15 listen up pay attention 20090329 "Each sentence has two blanks. The word that goes into the first blank starts with an """"H."""" Drop the """"H,"""" and you'll get a new word that goes in the second blank to complete the sentence. For example: With the current price of home fuel, the cost of ______ my house is ______ me alive. The answer: """"heating"""" and """"eating.""""" 1 This old battlefield is _____ ground, socamping on it is not _____. hallowed, allowed 20090329 "Each sentence has two blanks. The word that goes into the first blank starts with an """"H."""" Drop the """"H,"""" and you'll get a new word that goes in the second blank to complete the sentence. For example: With the current price of home fuel, the cost of ______ my house is ______ me alive. The answer: """"heating"""" and """"eating.""""" 2 The pilot with the receding _____ is thebest pilot in the whole _____. hairline, airline 20090329 "Each sentence has two blanks. The word that goes into the first blank starts with an """"H."""" Drop the """"H,"""" and you'll get a new word that goes in the second blank to complete the sentence. For example: With the current price of home fuel, the cost of ______ my house is ______ me alive. The answer: """"heating"""" and """"eating.""""" 3 Any so-called expert who identifies ared-striped fish as a _____ is clearly_____. herring, erring 20090329 "Each sentence has two blanks. The word that goes into the first blank starts with an """"H."""" Drop the """"H,"""" and you'll get a new word that goes in the second blank to complete the sentence. For example: With the current price of home fuel, the cost of ______ my house is ______ me alive. The answer: """"heating"""" and """"eating.""""" 4 During the summer, when temperatures are_____, you'll see an _____ play on the bankof the stream. hotter, otter 20090329 "Each sentence has two blanks. The word that goes into the first blank starts with an """"H."""" Drop the """"H,"""" and you'll get a new word that goes in the second blank to complete the sentence. For example: With the current price of home fuel, the cost of ______ my house is ______ me alive. The answer: """"heating"""" and """"eating.""""" 5 Seemingly every horse was in _____ in theold James _____ western I saw last night. harness, Arness 20090329 "Each sentence has two blanks. The word that goes into the first blank starts with an """"H."""" Drop the """"H,"""" and you'll get a new word that goes in the second blank to complete the sentence. For example: With the current price of home fuel, the cost of ______ my house is ______ me alive. The answer: """"heating"""" and """"eating.""""" 6 Appropriately, a ship carrying thousands of harbor, Arbor 20090329 "Each sentence has two blanks. The word that goes into the first blank starts with an """"H."""" Drop the """"H,"""" and you'll get a new word that goes in the second blank to complete the sentence. For example: With the current price of home fuel, the cost of ______ my house is ______ me alive. The answer: """"heating"""" and """"eating.""""" 7 seedlings arrived in the _____ onDay. _____ 20090329 "Each sentence has two blanks. The word that goes into the first blank starts with an """"H."""" Drop the """"H,"""" and you'll get a new word that goes in the second blank to complete the sentence. For example: With the current price of home fuel, the cost of ______ my house is ______ me alive. The answer: """"heating"""" and """"eating.""""" 8 Before putting new tiles on the roof, theroofer removes and _____ the old ones overthe _____. heaves, eaves 20090329 "Each sentence has two blanks. The word that goes into the first blank starts with an """"H."""" Drop the """"H,"""" and you'll get a new word that goes in the second blank to complete the sentence. For example: With the current price of home fuel, the cost of ______ my house is ______ me alive. The answer: """"heating"""" and """"eating.""""" 9 In exploring the colonial site,archeologists discovered a stone _____buried in the _____. hearth, earth 20090329 "Each sentence has two blanks. The word that goes into the first blank starts with an """"H."""" Drop the """"H,"""" and you'll get a new word that goes in the second blank to complete the sentence. For example: With the current price of home fuel, the cost of ______ my house is ______ me alive. The answer: """"heating"""" and """"eating.""""" 10 In the photograph of the woman at herdressing table, if her _____ is distracting,_____ it out. hairbrush, airbrush 20090329 "Each sentence has two blanks. The word that goes into the first blank starts with an """"H."""" Drop the """"H,"""" and you'll get a new word that goes in the second blank to complete the sentence. For example: With the current price of home fuel, the cost of ______ my house is ______ me alive. The answer: """"heating"""" and """"eating.""""" 11 The girl in the _____ top must _____ herattire before class. halter, alter 20070311 Every answer is a familiar two-word phrase with the consecutive letters A-R-M, and specifically the first word in the phrase will end in A-R, and the second word will start with M.  1 rite of passage for a Jewish boy Bar Mitzvah 20070311 Every answer is a familiar two-word phrase with the consecutive letters A-R-M, and specifically the first word in the phrase will end in A-R, and the second word will start with M.  2 sell off on Wall Street bear market 20070311 Every answer is a familiar two-word phrase with the consecutive letters A-R-M, and specifically the first word in the phrase will end in A-R, and the second word will start with M.  3 man's facial hair that curls up at the ends handlebar mustache 20070311 Every answer is a familiar two-word phrase with the consecutive letters A-R-M, and specifically the first word in the phrase will end in A-R, and the second word will start with M.  4 vehicle that lands on the moon lunar module 20070311 Every answer is a familiar two-word phrase with the consecutive letters A-R-M, and specifically the first word in the phrase will end in A-R, and the second word will start with M.  5 close call near miss 20070311 Every answer is a familiar two-word phrase with the consecutive letters A-R-M, and specifically the first word in the phrase will end in A-R, and the second word will start with M.  6 Rhythm and Blues, Rock and Roll, or Rap popular music 20070311 Every answer is a familiar two-word phrase with the consecutive letters A-R-M, and specifically the first word in the phrase will end in A-R, and the second word will start with M.  8 company that produces Havanasmanufacturer cigar maker or 20071202 "In this week's on-air puzzle, each definition or clue contains a missing word. That word is a homophone of the word that answers the clue. For example: If the clue is """"have (blank) doubt"""" the missing word would be """"no,"""" completing the phrase """"have no doubt."""" """"No"""" and """"know"""" are the homophones." 0 warning _____ a golfer for, fore 20071202 "In this week's on-air puzzle, each definition or clue contains a missing word. That word is a homophone of the word that answers the clue. For example: If the clue is """"have (blank) doubt"""" the missing word would be """"no,"""" completing the phrase """"have no doubt."""" """"No"""" and """"know"""" are the homophones." 1 what a Hawaiian may _____ around your neck lay, lei 20071202 "In this week's on-air puzzle, each definition or clue contains a missing word. That word is a homophone of the word that answers the clue. For example: If the clue is """"have (blank) doubt"""" the missing word would be """"no,"""" completing the phrase """"have no doubt."""" """"No"""" and """"know"""" are the homophones." 2 raw material that may contain gold _____ silver or, ore 20071202 "In this week's on-air puzzle, each definition or clue contains a missing word. That word is a homophone of the word that answers the clue. For example: If the clue is """"have (blank) doubt"""" the missing word would be """"no,"""" completing the phrase """"have no doubt."""" """"No"""" and """"know"""" are the homophones." 3 something _____ in a play seen, scene 20071202 "In this week's on-air puzzle, each definition or clue contains a missing word. That word is a homophone of the word that answers the clue. For example: If the clue is """"have (blank) doubt"""" the missing word would be """"no,"""" completing the phrase """"have no doubt."""" """"No"""" and """"know"""" are the homophones." 4 a place to turn _____ for the night whiletraveling in, inn 20071202 "In this week's on-air puzzle, each definition or clue contains a missing word. That word is a homophone of the word that answers the clue. For example: If the clue is """"have (blank) doubt"""" the missing word would be """"no,"""" completing the phrase """"have no doubt."""" """"No"""" and """"know"""" are the homophones." 5 Put a _____ number of workers on the payroll higher, hire 20071202 "In this week's on-air puzzle, each definition or clue contains a missing word. That word is a homophone of the word that answers the clue. For example: If the clue is """"have (blank) doubt"""" the missing word would be """"no,"""" completing the phrase """"have no doubt."""" """"No"""" and """"know"""" are the homophones." 6 animal with long ears and soft _____ hair, hare 20071202 "In this week's on-air puzzle, each definition or clue contains a missing word. That word is a homophone of the word that answers the clue. For example: If the clue is """"have (blank) doubt"""" the missing word would be """"no,"""" completing the phrase """"have no doubt."""" """"No"""" and """"know"""" are the homophones." 7 groups working on _____ ships cruise, crews 20071202 "In this week's on-air puzzle, each definition or clue contains a missing word. That word is a homophone of the word that answers the clue. For example: If the clue is """"have (blank) doubt"""" the missing word would be """"no,"""" completing the phrase """"have no doubt."""" """"No"""" and """"know"""" are the homophones." 8 showed fear as a _____ coward, cowered 20071202 "In this week's on-air puzzle, each definition or clue contains a missing word. That word is a homophone of the word that answers the clue. For example: If the clue is """"have (blank) doubt"""" the missing word would be """"no,"""" completing the phrase """"have no doubt."""" """"No"""" and """"know"""" are the homophones." 9 what a phone company may try to _____ you sell, cell 20071202 "In this week's on-air puzzle, each definition or clue contains a missing word. That word is a homophone of the word that answers the clue. For example: If the clue is """"have (blank) doubt"""" the missing word would be """"no,"""" completing the phrase """"have no doubt."""" """"No"""" and """"know"""" are the homophones." 10 a large animal that may _____ its teeth bare, bear 20071202 "In this week's on-air puzzle, each definition or clue contains a missing word. That word is a homophone of the word that answers the clue. For example: If the clue is """"have (blank) doubt"""" the missing word would be """"no,"""" completing the phrase """"have no doubt."""" """"No"""" and """"know"""" are the homophones." 11 _____ simple addition some, sum 20080217 Each sentence conceals the name of a U.S. president in consecutive letters. 0 The Lone Ranger was brainwashing Tonto. Washington 20080217 Each sentence conceals the name of a U.S. president in consecutive letters. 1 The two ships share a gangway. Reagan 20080217 Each sentence conceals the name of a U.S. president in consecutive letters. 2 Dad planted an orchard in Georgia. Harding 20080217 Each sentence conceals the name of a U.S. president in consecutive letters. 3 Can Idaho overtake Nebraska? Hoover 20080217 Each sentence conceals the name of a U.S. president in consecutive letters. 4 Get rum and cokes for everyone. Truman 20080217 Each sentence conceals the name of a U.S. president in consecutive letters. 5 Its taken Ned years to improve. Kennedy 20080217 Each sentence conceals the name of a U.S. president in consecutive letters. 7 Grandpa wants a crispier cereal. Pierce 20080217 Each sentence conceals the name of a U.S. president in consecutive letters. 8 The dog ran the gauntlet.Whenever I lean over, I start hurting again. Grant Arthur 20080217 Each sentence conceals the name of a U.S. president in consecutive letters. 9 Florida is big NASCAR territory. Carter 20080217 Each sentence conceals the name of a U.S. president in consecutive letters. 10 I ordered basic level andirons. Cleveland 20070805 "This week's on-air puzzle has a familiar three-word phrase in which the first word is a verb, the second word is ""the"" and the third word is a noun. I'll give you the phrase, but with an anagram of either the first or last word, you give me the phrases. " 0 miss the pinto miss the point 20070805 "This week's on-air puzzle has a familiar three-word phrase in which the first word is a verb, the second word is ""the"" and the third word is a noun. I'll give you the phrase, but with an anagram of either the first or last word, you give me the phrases. " 1 taste the obvious state the obvious 20070805 "This week's on-air puzzle has a familiar three-word phrase in which the first word is a verb, the second word is ""the"" and the third word is a noun. I'll give you the phrase, but with an anagram of either the first or last word, you give me the phrases. " 2 find the item find the time 20070805 "This week's on-air puzzle has a familiar three-word phrase in which the first word is a verb, the second word is ""the"" and the third word is a noun. I'll give you the phrase, but with an anagram of either the first or last word, you give me the phrases. " 3 show the odor show the door 20070805 "This week's on-air puzzle has a familiar three-word phrase in which the first word is a verb, the second word is ""the"" and the third word is a noun. I'll give you the phrase, but with an anagram of either the first or last word, you give me the phrases. " 4 cork the boat rock the boat 20070805 "This week's on-air puzzle has a familiar three-word phrase in which the first word is a verb, the second word is ""the"" and the third word is a noun. I'll give you the phrase, but with an anagram of either the first or last word, you give me the phrases. " 5 stem the diet stem the tide 20070805 "This week's on-air puzzle has a familiar three-word phrase in which the first word is a verb, the second word is ""the"" and the third word is a noun. I'll give you the phrase, but with an anagram of either the first or last word, you give me the phrases. " 6 parse the rod spare the rod 20070805 "This week's on-air puzzle has a familiar three-word phrase in which the first word is a verb, the second word is ""the"" and the third word is a noun. I'll give you the phrase, but with an anagram of either the first or last word, you give me the phrases. " 7 learn the prose learn the ropes 20070805 "This week's on-air puzzle has a familiar three-word phrase in which the first word is a verb, the second word is ""the"" and the third word is a noun. I'll give you the phrase, but with an anagram of either the first or last word, you give me the phrases. " 8 touch the earth touch the heart 20070805 "This week's on-air puzzle has a familiar three-word phrase in which the first word is a verb, the second word is ""the"" and the third word is a noun. I'll give you the phrase, but with an anagram of either the first or last word, you give me the phrases. " 9 follow the dealer follow the leader 20070805 "This week's on-air puzzle has a familiar three-word phrase in which the first word is a verb, the second word is ""the"" and the third word is a noun. I'll give you the phrase, but with an anagram of either the first or last word, you give me the phrases. " 10 grease the lamp grease the palm 20070805 "This week's on-air puzzle has a familiar three-word phrase in which the first word is a verb, the second word is ""the"" and the third word is a noun. I'll give you the phrase, but with an anagram of either the first or last word, you give me the phrases. " 11 prod the subject drop the subject 20100131 "Every answer is the name of a college or university. Given an anagram, the contestant names the school. For example, if the clue is """"lay"""" plus """"e,"""" the answer is """"Yale.""""" 0 worn + b Brown 20100131 "Every answer is the name of a college or university. Given an anagram, the contestant names the school. For example, if the clue is """"lay"""" plus """"e,"""" the answer is """"Yale.""""" 1 royal + b Baylor 20100131 "Every answer is the name of a college or university. Given an anagram, the contestant names the school. For example, if the clue is """"lay"""" plus """"e,"""" the answer is """"Yale.""""" 2 alloy + o Loyola 20100131 "Every answer is the name of a college or university. Given an anagram, the contestant names the school. For example, if the clue is """"lay"""" plus """"e,"""" the answer is """"Yale.""""" 3 prude + u Purdue 20100131 "Every answer is the name of a college or university. Given an anagram, the contestant names the school. For example, if the clue is """"lay"""" plus """"e,"""" the answer is """"Yale.""""" 4 Rhoda + w` Howard 20100131 "Every answer is the name of a college or university. Given an anagram, the contestant names the school. For example, if the clue is """"lay"""" plus """"e,"""" the answer is """"Yale.""""" 5 enroll + c Cornell 20100131 "Every answer is the name of a college or university. Given an anagram, the contestant names the school. For example, if the clue is """"lay"""" plus """"e,"""" the answer is """"Yale.""""" 6 boiler + o Oberlin 20100131 "Every answer is the name of a college or university. Given an anagram, the contestant names the school. For example, if the clue is """"lay"""" plus """"e,"""" the answer is """"Yale.""""" 7 biventral + d Vanderbilt 20100131 "Every answer is the name of a college or university. Given an anagram, the contestant names the school. For example, if the clue is """"lay"""" plus """"e,"""" the answer is """"Yale.""""" 8 enamored + t Notre Dame 20090201 "Every answer is a familiar two-word phrase in which one word starts with S-T, as in Steelers, and the other starts with C-A, as in Cardinals. The words can be in either order. For example, given """"people who work for a political candidate,"""" the answer would be """"campaign staff.""""" 1 racing vehicle stock car 20090201 "Every answer is a familiar two-word phrase in which one word starts with S-T, as in Steelers, and the other starts with C-A, as in Cardinals. The words can be in either order. For example, given """"people who work for a political candidate,"""" the answer would be """"campaign staff.""""" 2 book listing prices for philatelists stamp catalog 20090201 "Every answer is a familiar two-word phrase in which one word starts with S-T, as in Steelers, and the other starts with C-A, as in Cardinals. The words can be in either order. For example, given """"people who work for a political candidate,"""" the answer would be """"campaign staff.""""" 3 What Juneau is to Alaska state capital 20090201 "Every answer is a familiar two-word phrase in which one word starts with S-T, as in Steelers, and the other starts with C-A, as in Cardinals. The words can be in either order. For example, given """"people who work for a political candidate,"""" the answer would be """"campaign staff.""""" 4 place to buy chocolates and peppermint sticks candy store 20090201 "Every answer is a familiar two-word phrase in which one word starts with S-T, as in Steelers, and the other starts with C-A, as in Cardinals. The words can be in either order. For example, given """"people who work for a political candidate,"""" the answer would be """"campaign staff.""""" 5 antique elevator compartment; also acontraption that a shark hunter might dive in steel cage 20090201 "Every answer is a familiar two-word phrase in which one word starts with S-T, as in Steelers, and the other starts with C-A, as in Cardinals. The words can be in either order. For example, given """"people who work for a political candidate,"""" the answer would be """"campaign staff.""""" 6 what Bugs Bunny munches on carrot sticks 20090201 "Every answer is a familiar two-word phrase in which one word starts with S-T, as in Steelers, and the other starts with C-A, as in Cardinals. The words can be in either order. For example, given """"people who work for a political candidate,"""" the answer would be """"campaign staff.""""" 7 long running adventure comic strip by MiltonCaniff, from the 1930's to the 1980's Steve Canyon 20090201 "Every answer is a familiar two-word phrase in which one word starts with S-T, as in Steelers, and the other starts with C-A, as in Cardinals. The words can be in either order. For example, given """"people who work for a political candidate,"""" the answer would be """"campaign staff.""""" 8 1960's 1970's singer now known as Yusuf Islam Cat Stevens 20090201 "Every answer is a familiar two-word phrase in which one word starts with S-T, as in Steelers, and the other starts with C-A, as in Cardinals. The words can be in either order. For example, given """"people who work for a political candidate,"""" the answer would be """"campaign staff.""""" 9 Complete the title: The ___ of Dr. Jekyll andMr. Hyde Strange Case 20090201 "Every answer is a familiar two-word phrase in which one word starts with S-T, as in Steelers, and the other starts with C-A, as in Cardinals. The words can be in either order. For example, given """"people who work for a political candidate,"""" the answer would be """"campaign staff.""""" 10 worker who makes beds and cleans rooms on acruise ship cabin steward 20090201 "Every answer is a familiar two-word phrase in which one word starts with S-T, as in Steelers, and the other starts with C-A, as in Cardinals. The words can be in either order. For example, given """"people who work for a political candidate,"""" the answer would be """"campaign staff.""""" 11 knitted piece of headwear you pull over yourhead; a bank robber might wear one to hide hisidentity stocking cap 20090201 "Every answer is a familiar two-word phrase in which one word starts with S-T, as in Steelers, and the other starts with C-A, as in Cardinals. The words can be in either order. For example, given """"people who work for a political candidate,"""" the answer would be """"campaign staff.""""" 12 an ownerless feline stray cat 20090201 "Every answer is a familiar two-word phrase in which one word starts with S-T, as in Steelers, and the other starts with C-A, as in Cardinals. The words can be in either order. For example, given """"people who work for a political candidate,"""" the answer would be """"campaign staff.""""" 13 what someone who can eat large quantities ofspicy food without getting heartburn is saidto have cast-iron stomach 20080413 "In this week's puzzle, clues are given for two words. The first word has two or more G's in it. Remove all the G's and the remaining letters, in order, will spell the answer to the second clue. For example, given """"joke writers"""" and """"prayer ending,"""" the answer would be """"gag men"""" and """"amen.""""" 0 uses a mouth wash; French city where van Gogh painted Starry Night gargle, Arles 20080413 "In this week's puzzle, clues are given for two words. The first word has two or more G's in it. Remove all the G's and the remaining letters, in order, will spell the answer to the second clue. For example, given """"joke writers"""" and """"prayer ending,"""" the answer would be """"gag men"""" and """"amen.""""" 1 keeps several balls in the air; writer Verne juggles, Jules 20080413 "In this week's puzzle, clues are given for two words. The first word has two or more G's in it. Remove all the G's and the remaining letters, in order, will spell the answer to the second clue. For example, given """"joke writers"""" and """"prayer ending,"""" the answer would be """"gag men"""" and """"amen.""""" 2 long and matted as a dog's hair; rural carriage shaggy; shay 20080413 "In this week's puzzle, clues are given for two words. The first word has two or more G's in it. Remove all the G's and the remaining letters, in order, will spell the answer to the second clue. For example, given """"joke writers"""" and """"prayer ending,"""" the answer would be """"gag men"""" and """"amen.""""" 3 a non-wizzard in the Harry Potter books; an offspring of a horse and donkey muggle, mule 20080413 "In this week's puzzle, clues are given for two words. The first word has two or more G's in it. Remove all the G's and the remaining letters, in order, will spell the answer to the second clue. For example, given """"joke writers"""" and """"prayer ending,"""" the answer would be """"gag men"""" and """"amen.""""" 4 exercise activity on a track or street; become a member of jogging, join 20080413 "In this week's puzzle, clues are given for two words. The first word has two or more G's in it. Remove all the G's and the remaining letters, in order, will spell the answer to the second clue. For example, given """"joke writers"""" and """"prayer ending,"""" the answer would be """"gag men"""" and """"amen.""""" 5 protective eyewear; cries at a bullfight goggles, oles 20080413 "In this week's puzzle, clues are given for two words. The first word has two or more G's in it. Remove all the G's and the remaining letters, in order, will spell the answer to the second clue. For example, given """"joke writers"""" and """"prayer ending,"""" the answer would be """"gag men"""" and """"amen.""""" 6 boasting; intellect bragging, brain 20080413 "In this week's puzzle, clues are given for two words. The first word has two or more G's in it. Remove all the G's and the remaining letters, in order, will spell the answer to the second clue. For example, given """"joke writers"""" and """"prayer ending,"""" the answer would be """"gag men"""" and """"amen.""""" 7 sound of a drowning person (2 words); a lollapalooza glug glug, lulu 20081005 "You are given a sentence that's missing two words. The word that goes in the first blank has a T in it. Change this to a D, and phonetically, you'll get the word that goes in the second blank. For example, given """"The church's wooden _______ was made from an old _______ tree,"""" the answer would be """"altar"""" and """"alder."""" Hints: The answers are always two syllables long, and the T is always inside the word, not at the start or the end." 0 The Broadway musical _____ of the Opera has always had a large _____. Phantom, fandom 20081005 "You are given a sentence that's missing two words. The word that goes in the first blank has a T in it. Change this to a D, and phonetically, you'll get the word that goes in the second blank. For example, given """"The church's wooden _______ was made from an old _______ tree,"""" the answer would be """"altar"""" and """"alder."""" Hints: The answers are always two syllables long, and the T is always inside the word, not at the start or the end." 1 Flowers with the most beautiful _____ are what the florist _____ for a living. petals, pedals 20081005 "You are given a sentence that's missing two words. The word that goes in the first blank has a T in it. Change this to a D, and phonetically, you'll get the word that goes in the second blank. For example, given """"The church's wooden _______ was made from an old _______ tree,"""" the answer would be """"altar"""" and """"alder."""" Hints: The answers are always two syllables long, and the T is always inside the word, not at the start or the end." 2 In England, the rising cost of a _____ of petrol got the _____ voted out of office. litre, leader 20081005 "You are given a sentence that's missing two words. The word that goes in the first blank has a T in it. Change this to a D, and phonetically, you'll get the word that goes in the second blank. For example, given """"The church's wooden _______ was made from an old _______ tree,"""" the answer would be """"altar"""" and """"alder."""" Hints: The answers are always two syllables long, and the T is always inside the word, not at the start or the end." 3 For a serf a thousand years ago, it was _____ to rebel against the _____ system. futile, feudal 20081005 "You are given a sentence that's missing two words. The word that goes in the first blank has a T in it. Change this to a D, and phonetically, you'll get the word that goes in the second blank. For example, given """"The church's wooden _______ was made from an old _______ tree,"""" the answer would be """"altar"""" and """"alder."""" Hints: The answers are always two syllables long, and the T is always inside the word, not at the start or the end." 4 The amusement park prohibits pets, so one cannot take a _____ even on a _____ ride. kitty, kiddy 20081005 "You are given a sentence that's missing two words. The word that goes in the first blank has a T in it. Change this to a D, and phonetically, you'll get the word that goes in the second blank. For example, given """"The church's wooden _______ was made from an old _______ tree,"""" the answer would be """"altar"""" and """"alder."""" Hints: The answers are always two syllables long, and the T is always inside the word, not at the start or the end." 5 In negotiating with the Scandinavian businessman, to _____ the deal, we offered him a free trip home to _____. sweeten, Sweden 20081005 "You are given a sentence that's missing two words. The word that goes in the first blank has a T in it. Change this to a D, and phonetically, you'll get the word that goes in the second blank. For example, given """"The church's wooden _______ was made from an old _______ tree,"""" the answer would be """"altar"""" and """"alder."""" Hints: The answers are always two syllables long, and the T is always inside the word, not at the start or the end." 6 With my own hands, I fashioned a bust of the Greek philosopher _____ out of _____. Plato,Play-Doh 20080511 "In this week's on-air puzzle, you are given clues for six-letter words, each containing the consecutive letters M-A. Remove the M-A, and the remaining four letters, in order, will spell a word answering the second clue. For example, given """"to stay"""" and """"part of a bridle,"""" the answer would be """"remain"""" and """"rein.""""" 0 French city with an annual 24-hour auto race; part of a telescope Le Mans, lens 20080511 "In this week's on-air puzzle, you are given clues for six-letter words, each containing the consecutive letters M-A. Remove the M-A, and the remaining four letters, in order, will spell a word answering the second clue. For example, given """"to stay"""" and """"part of a bridle,"""" the answer would be """"remain"""" and """"rein.""""" 1 sheriff or marshall; something to be mowed lawman, lawn 20080511 "In this week's on-air puzzle, you are given clues for six-letter words, each containing the consecutive letters M-A. Remove the M-A, and the remaining four letters, in order, will spell a word answering the second clue. For example, given """"to stay"""" and """"part of a bridle,"""" the answer would be """"remain"""" and """"rein.""""" 2 frequent hamburger topping; a dog in The Wizard of Oz tomato, Toto 20080511 "In this week's on-air puzzle, you are given clues for six-letter words, each containing the consecutive letters M-A. Remove the M-A, and the remaining four letters, in order, will spell a word answering the second clue. For example, given """"to stay"""" and """"part of a bridle,"""" the answer would be """"remain"""" and """"rein.""""" 3 City in Washington; fried Mexican dish Tacoma, taco 20080511 "In this week's on-air puzzle, you are given clues for six-letter words, each containing the consecutive letters M-A. Remove the M-A, and the remaining four letters, in order, will spell a word answering the second clue. For example, given """"to stay"""" and """"part of a bridle,"""" the answer would be """"remain"""" and """"rein.""""" 4 steamed Mexican dish; a story tamale, tale 20080511 "In this week's on-air puzzle, you are given clues for six-letter words, each containing the consecutive letters M-A. Remove the M-A, and the remaining four letters, in order, will spell a word answering the second clue. For example, given """"to stay"""" and """"part of a bridle,"""" the answer would be """"remain"""" and """"rein.""""" 5 illness; proper woman malady, lady 20080511 "In this week's on-air puzzle, you are given clues for six-letter words, each containing the consecutive letters M-A. Remove the M-A, and the remaining four letters, in order, will spell a word answering the second clue. For example, given """"to stay"""" and """"part of a bridle,"""" the answer would be """"remain"""" and """"rein.""""" 6 class for expectant mothers; take things easy Lamaze, laze 20080518 "In this week's on-air puzzle, you are given two sentences. Each sentence has two blanks. A word starting with """"UP"""" goes in the first blank. Move the """"UP"""" to the end, and you'll get a familiar two-word phrase that goes in the second blank to complete the sentence. " 0 The daily _____ of a castle is expensive, and its hard even for a nobleman to _____ with the bills. upkeep, keep up 20080518 "In this week's on-air puzzle, you are given two sentences. Each sentence has two blanks. A word starting with """"UP"""" goes in the first blank. Move the """"UP"""" to the end, and you'll get a familiar two-word phrase that goes in the second blank to complete the sentence. " 1 The embattled executive managed to remain _____ despite being pretty _____ by the press. upbeat, beat up 20080518 "In this week's on-air puzzle, you are given two sentences. Each sentence has two blanks. A word starting with """"UP"""" goes in the first blank. Move the """"UP"""" to the end, and you'll get a familiar two-word phrase that goes in the second blank to complete the sentence. " 2 If the dull-witted employee is slow on the _____, this is an issue you should _____ with his boss. uptake, take up 20080518 "In this week's on-air puzzle, you are given two sentences. Each sentence has two blanks. A word starting with """"UP"""" goes in the first blank. Move the """"UP"""" to the end, and you'll get a familiar two-word phrase that goes in the second blank to complete the sentence. " 3 The rubber band slipped off the end of the balloon with the _____ being, the balloon _____ into the air. upshot, shot up 20080518 "In this week's on-air puzzle, you are given two sentences. Each sentence has two blanks. A word starting with """"UP"""" goes in the first blank. Move the """"UP"""" to the end, and you'll get a familiar two-word phrase that goes in the second blank to complete the sentence. " 4 The short-tempered hostess was _____ when she saw that the caterer couldn't _____ properly for the banquet. upset,set up 20080518 "In this week's on-air puzzle, you are given two sentences. Each sentence has two blanks. A word starting with """"UP"""" goes in the first blank. Move the """"UP"""" to the end, and you'll get a familiar two-word phrase that goes in the second blank to complete the sentence. " 5 The prospective groom admired his girlfriend's _____ nose, until pre-rhinoplasty pictures of her _____ in a family album. upturned, turned up 20090920 "For each clue, the answer is a four-letter word. The word is an anagram of one of the words in the clue. For example, if the clue is """"main line through Egypt,"""" the answer would be """"Nile,"""" because Nile is a rearrangement of the letters in """"line.""""" 0 group that acts in a play cast 20090920 "For each clue, the answer is a four-letter word. The word is an anagram of one of the words in the clue. For example, if the clue is """"main line through Egypt,"""" the answer would be """"Nile,"""" because Nile is a rearrangement of the letters in """"line.""""" 1 cheese made in Holland edam 20090920 "For each clue, the answer is a four-letter word. The word is an anagram of one of the words in the clue. For example, if the clue is """"main line through Egypt,"""" the answer would be """"Nile,"""" because Nile is a rearrangement of the letters in """"line.""""" 2 person one likes very much chum 20090920 "For each clue, the answer is a four-letter word. The word is an anagram of one of the words in the clue. For example, if the clue is """"main line through Egypt,"""" the answer would be """"Nile,"""" because Nile is a rearrangement of the letters in """"line.""""" 3 musical tone note 20090920 "For each clue, the answer is a four-letter word. The word is an anagram of one of the words in the clue. For example, if the clue is """"main line through Egypt,"""" the answer would be """"Nile,"""" because Nile is a rearrangement of the letters in """"line.""""" 4 clothing item worn inside a shoe hose 20090920 "For each clue, the answer is a four-letter word. The word is an anagram of one of the words in the clue. For example, if the clue is """"main line through Egypt,"""" the answer would be """"Nile,"""" because Nile is a rearrangement of the letters in """"line.""""" 5 overrule, as a congressional vote veto 20090920 "For each clue, the answer is a four-letter word. The word is an anagram of one of the words in the clue. For example, if the clue is """"main line through Egypt,"""" the answer would be """"Nile,"""" because Nile is a rearrangement of the letters in """"line.""""" 6 bit of land that lies in the ocean ilse 20090920 "For each clue, the answer is a four-letter word. The word is an anagram of one of the words in the clue. For example, if the clue is """"main line through Egypt,"""" the answer would be """"Nile,"""" because Nile is a rearrangement of the letters in """"line.""""" 7 vile evil 20090920 "For each clue, the answer is a four-letter word. The word is an anagram of one of the words in the clue. For example, if the clue is """"main line through Egypt,"""" the answer would be """"Nile,"""" because Nile is a rearrangement of the letters in """"line.""""" 8 ayes yeas 20090920 "For each clue, the answer is a four-letter word. The word is an anagram of one of the words in the clue. For example, if the clue is """"main line through Egypt,"""" the answer would be """"Nile,"""" because Nile is a rearrangement of the letters in """"line.""""" 9 wander over rove 20090920 "For each clue, the answer is a four-letter word. The word is an anagram of one of the words in the clue. For example, if the clue is """"main line through Egypt,"""" the answer would be """"Nile,"""" because Nile is a rearrangement of the letters in """"line.""""" 10 what freezers do when the electricity is turned off thaw 20090920 "For each clue, the answer is a four-letter word. The word is an anagram of one of the words in the clue. For example, if the clue is """"main line through Egypt,"""" the answer would be """"Nile,"""" because Nile is a rearrangement of the letters in """"line.""""" 11 drinks for sale at a pub ales 20090920 "For each clue, the answer is a four-letter word. The word is an anagram of one of the words in the clue. For example, if the clue is """"main line through Egypt,"""" the answer would be """"Nile,"""" because Nile is a rearrangement of the letters in """"line.""""" 12 source of money used by clubs dues 20090920 "For each clue, the answer is a four-letter word. The word is an anagram of one of the words in the clue. For example, if the clue is """"main line through Egypt,"""" the answer would be """"Nile,"""" because Nile is a rearrangement of the letters in """"line.""""" 13 last thing added to buttered popcorn salt 20090719 "Three of the most common consonants of the English language are R, S and T. Every answer today is a word, name or phrase that contains each of the letters R, S and T exactly once, along with any number of vowels. For example, if the clue is """"short-winded,"""" the answer would be, """"terse."""" Note: The R, S and T can appear in any order." 0 a fixed look (5 letter answers from now on) stare 20090719 "Three of the most common consonants of the English language are R, S and T. Every answer today is a word, name or phrase that contains each of the letters R, S and T exactly once, along with any number of vowels. For example, if the clue is """"short-winded,"""" the answer would be, """"terse."""" Note: The R, S and T can appear in any order." 1 unit of a mall store 20090719 "Three of the most common consonants of the English language are R, S and T. Every answer today is a word, name or phrase that contains each of the letters R, S and T exactly once, along with any number of vowels. For example, if the clue is """"short-winded,"""" the answer would be, """"terse."""" Note: The R, S and T can appear in any order." 2 the trunk of the body torso 20090719 "Three of the most common consonants of the English language are R, S and T. Every answer today is a word, name or phrase that contains each of the letters R, S and T exactly once, along with any number of vowels. For example, if the clue is """"short-winded,"""" the answer would be, """"terse."""" Note: The R, S and T can appear in any order." 3 where a chicken sits overnight roost 20090719 "Three of the most common consonants of the English language are R, S and T. Every answer today is a word, name or phrase that contains each of the letters R, S and T exactly once, along with any number of vowels. For example, if the clue is """"short-winded,"""" the answer would be, """"terse."""" Note: The R, S and T can appear in any order." 4 stringed musical instrument of India sitar 20090719 "Three of the most common consonants of the English language are R, S and T. Every answer today is a word, name or phrase that contains each of the letters R, S and T exactly once, along with any number of vowels. For example, if the clue is """"short-winded,"""" the answer would be, """"terse."""" Note: The R, S and T can appear in any order." 5 a stitch made after surgery (6 letter answers from now on) suture 20090719 "Three of the most common consonants of the English language are R, S and T. Every answer today is a word, name or phrase that contains each of the letters R, S and T exactly once, along with any number of vowels. For example, if the clue is """"short-winded,"""" the answer would be, """"terse."""" Note: The R, S and T can appear in any order." 6 Zodiac sign after Aries Taurus 20090719 "Three of the most common consonants of the English language are R, S and T. Every answer today is a word, name or phrase that contains each of the letters R, S and T exactly once, along with any number of vowels. For example, if the clue is """"short-winded,"""" the answer would be, """"terse."""" Note: The R, S and T can appear in any order." 7 sound system stereo 20090719 "Three of the most common consonants of the English language are R, S and T. Every answer today is a word, name or phrase that contains each of the letters R, S and T exactly once, along with any number of vowels. For example, if the clue is """"short-winded,"""" the answer would be, """"terse."""" Note: The R, S and T can appear in any order." 9 expulsion ouster 20090719 "Three of the most common consonants of the English language are R, S and T. Every answer today is a word, name or phrase that contains each of the letters R, S and T exactly once, along with any number of vowels. For example, if the clue is """"short-winded,"""" the answer would be, """"terse."""" Note: The R, S and T can appear in any order." 11 very, very funny riotous 20090719 "Three of the most common consonants of the English language are R, S and T. Every answer today is a word, name or phrase that contains each of the letters R, S and T exactly once, along with any number of vowels. For example, if the clue is """"short-winded,"""" the answer would be, """"terse."""" Note: The R, S and T can appear in any order." 12 ventilates (2 word answers from now on) airs out 20090719 "Three of the most common consonants of the English language are R, S and T. Every answer today is a word, name or phrase that contains each of the letters R, S and T exactly once, along with any number of vowels. For example, if the clue is """"short-winded,"""" the answer would be, """"terse."""" Note: The R, S and T can appear in any order." 13 a path across the ocean (8 letters, still two words) sea route 20100110 "Each sentence has a blank. Fill the blank with the name of a vegetable that can complete the sentence in a """"punny"""" way. For example: Apologizing profusely, the boy said, """"You don't know how sorry I _____."""" The answer would be """"yam"""" (""""I am"""")." 0 Marian performed her work so well, the boss increased her _____. celery (salary) 20100110 "Each sentence has a blank. Fill the blank with the name of a vegetable that can complete the sentence in a """"punny"""" way. For example: Apologizing profusely, the boy said, """"You don't know how sorry I _____."""" The answer would be """"yam"""" (""""I am"""")." 1 "During the church service, the minister said, ""please bow your heads and _____ pray.""" lettuce (let us) 20100110 "Each sentence has a blank. Fill the blank with the name of a vegetable that can complete the sentence in a """"punny"""" way. For example: Apologizing profusely, the boy said, """"You don't know how sorry I _____."""" The answer would be """"yam"""" (""""I am"""")." 2 Bob has been looking for a job for over a year, but he is confident something will eventually _____. turnip (turn up) 20100110 "Each sentence has a blank. Fill the blank with the name of a vegetable that can complete the sentence in a """"punny"""" way. For example: Apologizing profusely, the boy said, """"You don't know how sorry I _____."""" The answer would be """"yam"""" (""""I am"""")." 4 "When someone finally found the missing wire for Pete's guitar, Pete asked, ""where has that _____?"" (two word answer)" string bean (string been) 20101010 "You are given two categories and must name two things in both of them. For example, given """"male singers who have No. 1 hits"""" and """"state capitals,"""" the answers would be John Denver (Denver is the capital of Colorado) and Michael Jackson (Jackson is the capital of Mississippi)." 0 kings of Great Britain; first names of Beetles members John, George 20101010 "You are given two categories and must name two things in both of them. For example, given """"male singers who have No. 1 hits"""" and """"state capitals,"""" the answers would be John Denver (Denver is the capital of Colorado) and Michael Jackson (Jackson is the capital of Mississippi)." 1 animals in the zodiac; NFL team animals (teams in the Midwest; Rebecca notes one used to be in California) Lions, Rams 20101010 "You are given two categories and must name two things in both of them. For example, given """"male singers who have No. 1 hits"""" and """"state capitals,"""" the answers would be John Denver (Denver is the capital of Colorado) and Michael Jackson (Jackson is the capital of Mississippi)." 2 holidays; island (one has large stone statues in the South Pacific) Christmas, Easter 20101010 "You are given two categories and must name two things in both of them. For example, given """"male singers who have No. 1 hits"""" and """"state capitals,"""" the answers would be John Denver (Denver is the capital of Colorado) and Michael Jackson (Jackson is the capital of Mississippi)." 3 countries; things seen on a Thanksgiving dinner table Turkey, China 20101010 "You are given two categories and must name two things in both of them. For example, given """"male singers who have No. 1 hits"""" and """"state capitals,"""" the answers would be John Denver (Denver is the capital of Colorado) and Michael Jackson (Jackson is the capital of Mississippi)." 4 Holidays and islands Christmas and Easter 20070318  I'm going to give you two words. Rearrange the letters of the first word to get a new word, and that new word and the second word are both parts of a familiar proverb or saying. 0 shore, water You can lead a horse to water, but you can't make him drink 20070318  I'm going to give you two words. Rearrange the letters of the first word to get a new word, and that new word and the second word are both parts of a familiar proverb or saying. 1 epics, life Variety is the spice of life. 20070318  I'm going to give you two words. Rearrange the letters of the first word to get a new word, and that new word and the second word are both parts of a familiar proverb or saying. 2 super, ear You can't make a silk purse out of a sow's ear. 20070318  I'm going to give you two words. Rearrange the letters of the first word to get a new word, and that new word and the second word are both parts of a familiar proverb or saying. 3 more, roads All roads lead to Rome. 20070318  I'm going to give you two words. Rearrange the letters of the first word to get a new word, and that new word and the second word are both parts of a familiar proverb or saying. 4 least, dead Dead men tell no tales. 20070318  I'm going to give you two words. Rearrange the letters of the first word to get a new word, and that new word and the second word are both parts of a familiar proverb or saying. 5 earth, home Home is where the heart is. 20070318  I'm going to give you two words. Rearrange the letters of the first word to get a new word, and that new word and the second word are both parts of a familiar proverb or saying. 6 hated, taxed The only things that are guaranteed in life are death and taxes. 20070318  I'm going to give you two words. Rearrange the letters of the first word to get a new word, and that new word and the second word are both parts of a familiar proverb or saying. 7 deaf, soldiers Old soldiers never die, they just fade away. 20070318  I'm going to give you two words. Rearrange the letters of the first word to get a new word, and that new word and the second word are both parts of a familiar proverb or saying. 8 runt, another One good turn deserves another. 20070318  I'm going to give you two words. Rearrange the letters of the first word to get a new word, and that new word and the second word are both parts of a familiar proverb or saying. 9 vile, money Money is the root of all evil. 20070318  I'm going to give you two words. Rearrange the letters of the first word to get a new word, and that new word and the second word are both parts of a familiar proverb or saying. 10 evil, stones People who live in glass houses shouldn't throw stones. 20070318  I'm going to give you two words. Rearrange the letters of the first word to get a new word, and that new word and the second word are both parts of a familiar proverb or saying. 11 pears, spoil Spare the rod and spoil the child. 20070318  I'm going to give you two words. Rearrange the letters of the first word to get a new word, and that new word and the second word are both parts of a familiar proverb or saying. 12 relay, healthy Early to bed and early to rise makes a man healthy, wealthy and wise. 20070401 "This is a good puzzle for two people. Because it's April Fools' Day, I brought some classic April Fools' puzzles from Martin Gardner from his 1969 book, ""Perplexing Puzzles and Tantalizing Teasers.""" 0 Farmer Higgs owns three pink pigs, four brown pigs, and one black pig. How many of Higgs' pigs can say that it is the same color as another pig on Higgs' farm? None of the pigs can say anything 20070401 "This is a good puzzle for two people. Because it's April Fools' Day, I brought some classic April Fools' puzzles from Martin Gardner from his 1969 book, ""Perplexing Puzzles and Tantalizing Teasers.""" 1 Tom says his grandfather is only six years older than his father. Is that possible? Yes. The grandfather could be his mother's father. 20070401 "This is a good puzzle for two people. Because it's April Fools' Day, I brought some classic April Fools' puzzles from Martin Gardner from his 1969 book, ""Perplexing Puzzles and Tantalizing Teasers.""" 2 If it takes 20 minutes to hard boil one goose egg, how long does it take to hard boil four goose eggs. 20 minutes, if they are in the same pot. 20070401 "This is a good puzzle for two people. Because it's April Fools' Day, I brought some classic April Fools' puzzles from Martin Gardner from his 1969 book, ""Perplexing Puzzles and Tantalizing Teasers.""" 3 A hobo was walking down a railroad track when he saw a fast express train speeding toward him. Before he jumped off the track, he ran 10 feet toward the train. Why? He was on a bridge or in a tunnel, and the nearest exit was toward the train. 20070401 "This is a good puzzle for two people. Because it's April Fools' Day, I brought some classic April Fools' puzzles from Martin Gardner from his 1969 book, ""Perplexing Puzzles and Tantalizing Teasers.""" 4 Archibald Flapdoodle walked outside through a pouring rain for twenty minutes without getting a single hair on his head wet. He didn't wear a hat or carry an umbrella. He didn't hold anything over his head. His clothes, though, got soaked. How could this be possible? He is bald, so he had no hairs on his head. 20070401 "This is a good puzzle for two people. Because it's April Fools' Day, I brought some classic April Fools' puzzles from Martin Gardner from his 1969 book, ""Perplexing Puzzles and Tantalizing Teasers.""" 5 Mrs. Fumblefinger was working in the kitchen when a loose ring with a big diamond on it slipped off her finger and fell smack into some coffee. The diamond did not get wet. How? It fell into ground coffee that was not wet. 20070401 "This is a good puzzle for two people. Because it's April Fools' Day, I brought some classic April Fools' puzzles from Martin Gardner from his 1969 book, ""Perplexing Puzzles and Tantalizing Teasers.""" 6 What do you sit on, sleep on, and brush your teeth with. A chair, a bed and a toothbrush. 20070401 "This is a good puzzle for two people. Because it's April Fools' Day, I brought some classic April Fools' puzzles from Martin Gardner from his 1969 book, ""Perplexing Puzzles and Tantalizing Teasers.""" 7 What has 18 legs, is covered with red spots and catches flies? A baseball team with the measles. 20070401 "This is a good puzzle for two people. Because it's April Fools' Day, I brought some classic April Fools' puzzles from Martin Gardner from his 1969 book, ""Perplexing Puzzles and Tantalizing Teasers.""" 8 "One day, centuries ago, an alchemist brought a small bottle to the King. ""This bottle,"" said the alchemist, ""holds a liquid so powerful that it will instantly dissolve anything it touches."" How did the King know theman was lying?" Because the liquid did not dissolve the bottle. 20070401 "This is a good puzzle for two people. Because it's April Fools' Day, I brought some classic April Fools' puzzles from Martin Gardner from his 1969 book, ""Perplexing Puzzles and Tantalizing Teasers.""" 9 Baskim turned off the light in his bedroom and was able to get to bed before the room was dark.His bed is 15 feet from the wall switch. How did Baskim do it? He did it during daylight. 20070617 "In the on-air puzzle this week, every answer is the name of a famous person whose first name starts with ""J."" Will Shortz provides the first name and an anagram of the last name. You identify the person." 0 Jay Lone Leno 20070617 "In the on-air puzzle this week, every answer is the name of a famous person whose first name starts with ""J."" Will Shortz provides the first name and an anagram of the last name. You identify the person." 1 Jules Never Verne 20070617 "In the on-air puzzle this week, every answer is the name of a famous person whose first name starts with ""J."" Will Shortz provides the first name and an anagram of the last name. You identify the person." 2 Joe Spice Pesci 20070617 "In the on-air puzzle this week, every answer is the name of a famous person whose first name starts with ""J."" Will Shortz provides the first name and an anagram of the last name. You identify the person." 3 Jefferson Divas Davis 20070617 "In the on-air puzzle this week, every answer is the name of a famous person whose first name starts with ""J."" Will Shortz provides the first name and an anagram of the last name. You identify the person." 4 Jessica Angel Lange 20070617 "In the on-air puzzle this week, every answer is the name of a famous person whose first name starts with ""J."" Will Shortz provides the first name and an anagram of the last name. You identify the person." 5 Jeremy Rosin Irons 20070617 "In the on-air puzzle this week, every answer is the name of a famous person whose first name starts with ""J."" Will Shortz provides the first name and an anagram of the last name. You identify the person." 6 Johnny Acorns Carson 20070617 "In the on-air puzzle this week, every answer is the name of a famous person whose first name starts with ""J."" Will Shortz provides the first name and an anagram of the last name. You identify the person." 7 James Agency Cagney 20070617 "In the on-air puzzle this week, every answer is the name of a famous person whose first name starts with ""J."" Will Shortz provides the first name and an anagram of the last name. You identify the person." 8 Jane Unseat Austen 20070617 "In the on-air puzzle this week, every answer is the name of a famous person whose first name starts with ""J."" Will Shortz provides the first name and an anagram of the last name. You identify the person." 9 Jean Paul Arrest Sartre 20070617 "In the on-air puzzle this week, every answer is the name of a famous person whose first name starts with ""J."" Will Shortz provides the first name and an anagram of the last name. You identify the person." 10 Joseph Dacron Conrad 20070617 "In the on-air puzzle this week, every answer is the name of a famous person whose first name starts with ""J."" Will Shortz provides the first name and an anagram of the last name. You identify the person." 11 Jim Barons Nabors 20070617 "In the on-air puzzle this week, every answer is the name of a famous person whose first name starts with ""J."" Will Shortz provides the first name and an anagram of the last name. You identify the person." 12 Jimmy Unrated Durante 20070617 "In the on-air puzzle this week, every answer is the name of a famous person whose first name starts with ""J."" Will Shortz provides the first name and an anagram of the last name. You identify the person." 13 Jon Swatter Stewart 20070617 "In the on-air puzzle this week, every answer is the name of a famous person whose first name starts with ""J."" Will Shortz provides the first name and an anagram of the last name. You identify the person." 14 Julie Wardens Andrews 20070617 "In the on-air puzzle this week, every answer is the name of a famous person whose first name starts with ""J."" Will Shortz provides the first name and an anagram of the last name. You identify the person." 15 Jennifer Nations   Aniston 20070617 "In the on-air puzzle this week, every answer is the name of a famous person whose first name starts with ""J."" Will Shortz provides the first name and an anagram of the last name. You identify the person." 16 Jason Peristyle Preistley 20071007 In this week's on-air puzzle, every answer is a familiar two-word phrase or name, with the initials C.O. For example: Right to buy a stock later at a specified price. The answer: Call Option. 0 kitchen gadget for removing lids can opener 20071007 In this week's on-air puzzle, every answer is a familiar two-word phrase or name, with the initials C.O. For example: Right to buy a stock later at a specified price. The answer: Call Option. 1 liquid product of a palm tree coconut oil 20071007 In this week's on-air puzzle, every answer is a familiar two-word phrase or name, with the initials C.O. For example: Right to buy a stock later at a specified price. The answer: Call Option. 2 smallish group playing classical music chamber orchestra 20071007 In this week's on-air puzzle, every answer is a familiar two-word phrase or name, with the initials C.O. For example: Right to buy a stock later at a specified price. The answer: Call Option. 3 Rossini's The Barber of Seville, or Strauss' Die Fledermaus comic opera 20071007 In this week's on-air puzzle, every answer is a familiar two-word phrase or name, with the initials C.O. For example: Right to buy a stock later at a specified price. The answer: Call Option. 4 "Chekov play, with ""The""" Cherry Orchard 20071007 In this week's on-air puzzle, every answer is a familiar two-word phrase or name, with the initials C.O. For example: Right to buy a stock later at a specified price. The answer: Call Option. 5 crime that might result in the death penalty capital offense 20071007 In this week's on-air puzzle, every answer is a familiar two-word phrase or name, with the initials C.O. For example: Right to buy a stock later at a specified price. The answer: Call Option. 6 any army person ranking above sergeant commissioned officer 20071007 In this week's on-air puzzle, every answer is a familiar two-word phrase or name, with the initials C.O. For example: Right to buy a stock later at a specified price. The answer: Call Option. 7 musical accompaniment for a hymn church organ 20071007 In this week's on-air puzzle, every answer is a familiar two-word phrase or name, with the initials C.O. For example: Right to buy a stock later at a specified price. The answer: Call Option. 8 state capital between Indianapolis, Indiana, and Harrisburg, Pennsylvania Columbus, Ohio 20071007 In this week's on-air puzzle, every answer is a familiar two-word phrase or name, with the initials C.O. For example: Right to buy a stock later at a specified price. The answer: Call Option. 9 feature of property in a kibbutz community ownership or common ownership 20071007 In this week's on-air puzzle, every answer is a familiar two-word phrase or name, with the initials C.O. For example: Right to buy a stock later at a specified price. The answer: Call Option. 10 a person with religious or moral reasons for not going to war conscientious objector 20071007 In this week's on-air puzzle, every answer is a familiar two-word phrase or name, with the initials C.O. For example: Right to buy a stock later at a specified price. The answer: Call Option. 11 player of Archie Bunker in All in the Family Carroll O'Connor 20071007 In this week's on-air puzzle, every answer is a familiar two-word phrase or name, with the initials C.O. For example: Right to buy a stock later at a specified price. The answer: Call Option. 12 basic brunch dish cheese omelet 20071007 In this week's on-air puzzle, every answer is a familiar two-word phrase or name, with the initials C.O. For example: Right to buy a stock later at a specified price. The answer: Call Option. 13 citrus fruit from the West Coast California orange 20071007 In this week's on-air puzzle, every answer is a familiar two-word phrase or name, with the initials C.O. For example: Right to buy a stock later at a specified price. The answer: Call Option. 14 typical CIA mission covert operation 20071007 In this week's on-air puzzle, every answer is a familiar two-word phrase or name, with the initials C.O. For example: Right to buy a stock later at a specified price. The answer: Call Option. 15 a sign at a nudist camp clothes optional 20071223 "In this week's on-air puzzle, every answer is part of a category. After a category is given, there should be a response with something else in that category that starts with the last two letters of the given category. For example, if the clue is Rigoletto, then the response might be Tosca. They are both Italian operas; one ends with letters """"t"""" and """"o"""" while the other starts with those letters." 0 Wisconsin Indiana 20071223 "In this week's on-air puzzle, every answer is part of a category. After a category is given, there should be a response with something else in that category that starts with the last two letters of the given category. For example, if the clue is Rigoletto, then the response might be Tosca. They are both Italian operas; one ends with letters """"t"""" and """"o"""" while the other starts with those letters." 1 spruce    cedar 20071223 "In this week's on-air puzzle, every answer is part of a category. After a category is given, there should be a response with something else in that category that starts with the last two letters of the given category. For example, if the clue is Rigoletto, then the response might be Tosca. They are both Italian operas; one ends with letters """"t"""" and """"o"""" while the other starts with those letters." 2 Job       Obadiah 20071223 "In this week's on-air puzzle, every answer is part of a category. After a category is given, there should be a response with something else in that category that starts with the last two letters of the given category. For example, if the clue is Rigoletto, then the response might be Tosca. They are both Italian operas; one ends with letters """"t"""" and """"o"""" while the other starts with those letters." 3 square    rectangle 20071223 "In this week's on-air puzzle, every answer is part of a category. After a category is given, there should be a response with something else in that category that starts with the last two letters of the given category. For example, if the clue is Rigoletto, then the response might be Tosca. They are both Italian operas; one ends with letters """"t"""" and """"o"""" while the other starts with those letters." 4 tuba      bassoon, baritone sax 20071223 "In this week's on-air puzzle, every answer is part of a category. After a category is given, there should be a response with something else in that category that starts with the last two letters of the given category. For example, if the clue is Rigoletto, then the response might be Tosca. They are both Italian operas; one ends with letters """"t"""" and """"o"""" while the other starts with those letters." 5 Verona    Naples 20071223 "In this week's on-air puzzle, every answer is part of a category. After a category is given, there should be a response with something else in that category that starts with the last two letters of the given category. For example, if the clue is Rigoletto, then the response might be Tosca. They are both Italian operas; one ends with letters """"t"""" and """"o"""" while the other starts with those letters." 6 sulfur    uranium 20071223 "In this week's on-air puzzle, every answer is part of a category. After a category is given, there should be a response with something else in that category that starts with the last two letters of the given category. For example, if the clue is Rigoletto, then the response might be Tosca. They are both Italian operas; one ends with letters """"t"""" and """"o"""" while the other starts with those letters." 7 Fillmore  Reagan 20071223 "In this week's on-air puzzle, every answer is part of a category. After a category is given, there should be a response with something else in that category that starts with the last two letters of the given category. For example, if the clue is Rigoletto, then the response might be Tosca. They are both Italian operas; one ends with letters """"t"""" and """"o"""" while the other starts with those letters." 8 apple     lemon 20071223 "In this week's on-air puzzle, every answer is part of a category. After a category is given, there should be a response with something else in that category that starts with the last two letters of the given category. For example, if the clue is Rigoletto, then the response might be Tosca. They are both Italian operas; one ends with letters """"t"""" and """"o"""" while the other starts with those letters." 9 Churchill Lloyd George 20071223 "In this week's on-air puzzle, every answer is part of a category. After a category is given, there should be a response with something else in that category that starts with the last two letters of the given category. For example, if the clue is Rigoletto, then the response might be Tosca. They are both Italian operas; one ends with letters """"t"""" and """"o"""" while the other starts with those letters." 10 Pontiac   Acura 20071223 "In this week's on-air puzzle, every answer is part of a category. After a category is given, there should be a response with something else in that category that starts with the last two letters of the given category. For example, if the clue is Rigoletto, then the response might be Tosca. They are both Italian operas; one ends with letters """"t"""" and """"o"""" while the other starts with those letters." 11 Gator     Orange 20071223 "In this week's on-air puzzle, every answer is part of a category. After a category is given, there should be a response with something else in that category that starts with the last two letters of the given category. For example, if the clue is Rigoletto, then the response might be Tosca. They are both Italian operas; one ends with letters """"t"""" and """"o"""" while the other starts with those letters." 12 Indian    Angel (American League baseball player) or Antarctic (oceans) 20071223 "In this week's on-air puzzle, every answer is part of a category. After a category is given, there should be a response with something else in that category that starts with the last two letters of the given category. For example, if the clue is Rigoletto, then the response might be Tosca. They are both Italian operas; one ends with letters """"t"""" and """"o"""" while the other starts with those letters." 13 Huron     Ontario 20080420 For each category Will lists, you must name something that belongs in it that begins with each of the following letters: C, H, A, M and P. 0 US presidents Coolidge, Hoover, Adams, Madison, Polk  20080420 For each category Will lists, you must name something that belongs in it that begins with each of the following letters: C, H, A, M and P. 1 things seen in a church cross, holy water, ambo,  music stand, priest 20080420 For each category Will lists, you must name something that belongs in it that begins with each of the following letters: C, H, A, M and P. 2 Shakespearean plays Comedy of Errors, Hamlet, Antony and Cleopatra, MacBeth, Pericles, Prince of Tyre 20080420 For each category Will lists, you must name something that belongs in it that begins with each of the following letters: C, H, A, M and P. 3 six letter words that start and end with the same letter      cognac, hoorah or hurrah, asthma, museum, pick-up        20080504 "In this week's on-air puzzle, you are given three words that rhyme with three things that start with the same letter and are in the same category. For example, given """"soxer,"""" """"facet"""" and """"regal,"""" the answer would be """"boxer, """"basset"""" and """"beagle."""" " 0 pays, mover, guarding Hayes, Hoover, Harding (US presidents) 20080504 "In this week's on-air puzzle, you are given three words that rhyme with three things that start with the same letter and are in the same category. For example, given """"soxer,"""" """"facet"""" and """"regal,"""" the answer would be """"boxer, """"basset"""" and """"beagle."""" " 1 flunky, deuce, house monkey, moose, mouse (animals) 20080504 "In this week's on-air puzzle, you are given three words that rhyme with three things that start with the same letter and are in the same category. For example, given """"soxer,"""" """"facet"""" and """"regal,"""" the answer would be """"boxer, """"basset"""" and """"beagle."""" " 2 smirch, speech, salsa birch, beech,  balsa (trees) 20080504 "In this week's on-air puzzle, you are given three words that rhyme with three things that start with the same letter and are in the same category. For example, given """"soxer,"""" """"facet"""" and """"regal,"""" the answer would be """"boxer, """"basset"""" and """"beagle."""" " 3 havens, shams, seders Ravens, Rams, Raiders (NFL teams) 20080504 "In this week's on-air puzzle, you are given three words that rhyme with three things that start with the same letter and are in the same category. For example, given """"soxer,"""" """"facet"""" and """"regal,"""" the answer would be """"boxer, """"basset"""" and """"beagle."""" " 4 follow, lift, dawn swallow, swift, swan (birds) 20080504 "In this week's on-air puzzle, you are given three words that rhyme with three things that start with the same letter and are in the same category. For example, given """"soxer,"""" """"facet"""" and """"regal,"""" the answer would be """"boxer, """"basset"""" and """"beagle."""" " 5 mynah,filly, glad China, Chile, Chad (countries) 20080504 "In this week's on-air puzzle, you are given three words that rhyme with three things that start with the same letter and are in the same category. For example, given """"soxer,"""" """"facet"""" and """"regal,"""" the answer would be """"boxer, """"basset"""" and """"beagle."""" " 6 steel, Tennyson veal, venison (meats) 20080504 "In this week's on-air puzzle, you are given three words that rhyme with three things that start with the same letter and are in the same category. For example, given """"soxer,"""" """"facet"""" and """"regal,"""" the answer would be """"boxer, """"basset"""" and """"beagle."""" " 7 squeegees, words Bee Gees, Byrds (musical groups) 20080504 "In this week's on-air puzzle, you are given three words that rhyme with three things that start with the same letter and are in the same category. For example, given """"soxer,"""" """"facet"""" and """"regal,"""" the answer would be """"boxer, """"basset"""" and """"beagle."""" " 8 riot, Milton Hyatt, Hilton (hotel chains) 20080504 "In this week's on-air puzzle, you are given three words that rhyme with three things that start with the same letter and are in the same category. For example, given """"soxer,"""" """"facet"""" and """"regal,"""" the answer would be """"boxer, """"basset"""" and """"beagle."""" " 9 fillings, cute Billings, Butte (cities in Montana) 20080615 This Friday, June 20, is the day of the year that has the most daylight. So, I have a game of categories using the word light. For each category that I give you, name something in that category starting with each of the letters L – I - G – H – T. For example, if the category were chemical elements, you might say: Lead, Iron, Gold, Helium and Tin. 0 US states Louisiana, Indiana, Georgia, Hawai'i, Texas (or Tennessee) 20080615 This Friday, June 20, is the day of the year that has the most daylight. So, I have a game of categories using the word light. For each category that I give you, name something in that category starting with each of the letters L – I - G – H – T. For example, if the category were chemical elements, you might say: Lead, Iron, Gold, Helium and Tin. 1 holidays Labor Day, Independence Day, Grandparents Day (or Groundhog Day or Good Friday), Hanukkah,  Thanksgiving 20080615 This Friday, June 20, is the day of the year that has the most daylight. So, I have a game of categories using the word light. For each category that I give you, name something in that category starting with each of the letters L – I - G – H – T. For example, if the category were chemical elements, you might say: Lead, Iron, Gold, Helium and Tin. 2 words related to weather lightning (or layer), inversion (or ice or isobar), ground fog (or gulf stream or gale), hailstones, thunder  20080615 This Friday, June 20, is the day of the year that has the most daylight. So, I have a game of categories using the word light. For each category that I give you, name something in that category starting with each of the letters L – I - G – H – T. For example, if the category were chemical elements, you might say: Lead, Iron, Gold, Helium and Tin. 3 colleges and universities that do not have states in their names Loyola, Ignatius (or Ithaca or Iona), Georgetown, Harvard, Tulane 20081102 "For each category given, the answers are things within the category that start with the letters, M, I, N, S and K. For example, given """"two-syllable girls' names,"""" the answer could be """"Mary, Ingrid, Norma, Sarah and Kathy.""""" 0 makes of automobiles Mazda, Isuzu (or Infiniti), Nissan, Saturn, Kia 20081102 "For each category given, the answers are things within the category that start with the letters, M, I, N, S and K. For example, given """"two-syllable girls' names,"""" the answer could be """"Mary, Ingrid, Norma, Sarah and Kathy.""""" 1 parts of the human body mouth, instep (or intestines), nose, sinus (or shoulder or spine or stomach), knee 20081102 "For each category given, the answers are things within the category that start with the letters, M, I, N, S and K. For example, given """"two-syllable girls' names,"""" the answer could be """"Mary, Ingrid, Norma, Sarah and Kathy.""""" 2 Indian tribes Mohawk, Inca (or Iroquois or Illinois), Navajo, Sioux, Kiowa 20081102 "For each category given, the answers are things within the category that start with the letters, M, I, N, S and K. For example, given """"two-syllable girls' names,"""" the answer could be """"Mary, Ingrid, Norma, Sarah and Kathy.""""" 3 words of five or more letters matchstick, icepick, noodnik, stock, kiosk 20081221 "This is a game of categories using the word """"Claus."""" For each category, name something in it starting with each of the letters C-L-A-U-S. For example, if the category were girls' names: Carol, Laura, Alice, Ursula and Sarah." 0 African countries Congo, Liberia, Angola, Uganda, Somalia (or South Africa or Swaziland) 20081221 "This is a game of categories using the word """"Claus."""" For each category, name something in it starting with each of the letters C-L-A-U-S. For example, if the category were girls' names: Carol, Laura, Alice, Ursula and Sarah." 1 people at a baseball game customers (or catcher), left fielder, announcer, umpire, short stop (or second baseman) 20081221 "This is a game of categories using the word """"Claus."""" For each category, name something in it starting with each of the letters C-L-A-U-S. For example, if the category were girls' names: Carol, Laura, Alice, Ursula and Sarah." 2 gods or goddesses Cassiopeia (or Concordia or Chaos), Loki (or Leto), Adonis, Uranus, Saturn 20081221 "This is a game of categories using the word """"Claus."""" For each category, name something in it starting with each of the letters C-L-A-U-S. For example, if the category were girls' names: Carol, Laura, Alice, Ursula and Sarah." 3 areas or rooms in a house closet, linen closet, attic, upstairs (or utility room), study (or solarium) 20081221 "This is a game of categories using the word """"Claus."""" For each category, name something in it starting with each of the letters C-L-A-U-S. For example, if the category were girls' names: Carol, Laura, Alice, Ursula and Sarah." 4 things seen in a theater chandelier, libretto (or lounge or lavatory), audience (or aisle or actor), usher, seat (or stage or screen) 20090315 "For each category, name five items, each one beginning with a different letter in the word """"Aries,"""" the first Zodiac sign in spring. For example, given the category """"elements on the periodic table,"""" the answers could be """"argon, radium, iron, einsteinium and silicon.""""" 0 European countries Austria (or Andorra or Albania), Russia, Ireland (or Iceland or Italy), England, Spain 20090315 "For each category, name five items, each one beginning with a different letter in the word """"Aries,"""" the first Zodiac sign in spring. For example, given the category """"elements on the periodic table,"""" the answers could be """"argon, radium, iron, einsteinium and silicon.""""" 1 Old Testament names Abraham, Ruth, Isaac (or Isaiah), Esau, Sarah 20090315 "For each category, name five items, each one beginning with a different letter in the word """"Aries,"""" the first Zodiac sign in spring. For example, given the category """"elements on the periodic table,"""" the answers could be """"argon, radium, iron, einsteinium and silicon.""""" 2 TV sitcoms All in the Family, Rhoda (or Roseanne or Reba), I Remember Mama (or I Love Lucy or I Dream of Jeanie), Ellen (or Eight is Enough or Empty Nest or Evening Shade or Everyone Loves Raymond), Seinfeld 20090315 "For each category, name five items, each one beginning with a different letter in the word """"Aries,"""" the first Zodiac sign in spring. For example, given the category """"elements on the periodic table,"""" the answers could be """"argon, radium, iron, einsteinium and silicon.""""" 3 Terms related to poetry anapest, rhyme, iambic, elegy, sonnet 20090830 "Each answer starts with a clue for a six-letter word. If you drop the first letter and read the remaining letters backward, you'll get a five-letter word that answers a second clue. For example, if the clues are """"flower parts"""" and """"roofing material,"""" the answers would be """"petals"""" and """"slate."""" (Remove the """"p"""" of """"petals"""" and read the rest backward to get """"slate."""")" 0 to state positively; a lock of hair assert, tress 20090830 "Each answer starts with a clue for a six-letter word. If you drop the first letter and read the remaining letters backward, you'll get a five-letter word that answers a second clue. For example, if the clues are """"flower parts"""" and """"roofing material,"""" the answers would be """"petals"""" and """"slate."""" (Remove the """"p"""" of """"petals"""" and read the rest backward to get """"slate."""")" 1 a hotel employee who carries bags; and like old-fashioneds that are in again porter, retro 20090830 "Each answer starts with a clue for a six-letter word. If you drop the first letter and read the remaining letters backward, you'll get a five-letter word that answers a second clue. For example, if the clues are """"flower parts"""" and """"roofing material,"""" the answers would be """"petals"""" and """"slate."""" (Remove the """"p"""" of """"petals"""" and read the rest backward to get """"slate."""")" 2 calamitous; a prop for Grouch Marx tragic, cigar 20090830 "Each answer starts with a clue for a six-letter word. If you drop the first letter and read the remaining letters backward, you'll get a five-letter word that answers a second clue. For example, if the clues are """"flower parts"""" and """"roofing material,"""" the answers would be """"petals"""" and """"slate."""" (Remove the """"p"""" of """"petals"""" and read the rest backward to get """"slate."""")" 3 a big number; a section in a supermarket with eggs and butter myriad, dairy 20090830 "Each answer starts with a clue for a six-letter word. If you drop the first letter and read the remaining letters backward, you'll get a five-letter word that answers a second clue. For example, if the clues are """"flower parts"""" and """"roofing material,"""" the answers would be """"petals"""" and """"slate."""" (Remove the """"p"""" of """"petals"""" and read the rest backward to get """"slate."""")" 4 attic; Latin for earth garret, terra 20090830 "Each answer starts with a clue for a six-letter word. If you drop the first letter and read the remaining letters backward, you'll get a five-letter word that answers a second clue. For example, if the clues are """"flower parts"""" and """"roofing material,"""" the answers would be """"petals"""" and """"slate."""" (Remove the """"p"""" of """"petals"""" and read the rest backward to get """"slate."""")" 5 "the lowest part; ""all for one, and one for all,"" for example" bottom, motto 20090830 "Each answer starts with a clue for a six-letter word. If you drop the first letter and read the remaining letters backward, you'll get a five-letter word that answers a second clue. For example, if the clues are """"flower parts"""" and """"roofing material,"""" the answers would be """"petals"""" and """"slate."""" (Remove the """"p"""" of """"petals"""" and read the rest backward to get """"slate."""")" 6 a leftist European political party; the look on a villain's face Greens, sneer 20090830 "Each answer starts with a clue for a six-letter word. If you drop the first letter and read the remaining letters backward, you'll get a five-letter word that answers a second clue. For example, if the clues are """"flower parts"""" and """"roofing material,"""" the answers would be """"petals"""" and """"slate."""" (Remove the """"p"""" of """"petals"""" and read the rest backward to get """"slate."""")" 7 means or signifies (two-word phrase); one of the five senses gets at, taste 20090928 "Each answer starts with a clue for a six-letter word. If you drop the first letter and read the remaining letters backward, you'll get a five-letter word that answers a second clue. For example, if the clues are """"flower parts"""" and """"roofing material,"""" the answers would be """"petals"""" and """"slate."""" (Remove the """"p"""" of """"petals"""" and read the rest backward to get """"slate."""")" 0 "a familiar two-word phrase that starts with ""u"" and ends with ""p"" and names something that provides black light" ultraviolet lamp 20090928 "Each answer starts with a clue for a six-letter word. If you drop the first letter and read the remaining letters backward, you'll get a five-letter word that answers a second clue. For example, if the clues are """"flower parts"""" and """"roofing material,"""" the answers would be """"petals"""" and """"slate."""" (Remove the """"p"""" of """"petals"""" and read the rest backward to get """"slate."""")" 1 "What four-letter word, ending in ""up,"" has a silent ""p""?" coup 20090928 "Each answer starts with a clue for a six-letter word. If you drop the first letter and read the remaining letters backward, you'll get a five-letter word that answers a second clue. For example, if the clues are """"flower parts"""" and """"roofing material,"""" the answers would be """"petals"""" and """"slate."""" (Remove the """"p"""" of """"petals"""" and read the rest backward to get """"slate."""")" 2 "The letters ""u-p"" rhyme with ""loopy"" and ""soupy"" What kind of doll does it rhyme with?" Kewpie 20090928 "Each answer starts with a clue for a six-letter word. If you drop the first letter and read the remaining letters backward, you'll get a five-letter word that answers a second clue. For example, if the clues are """"flower parts"""" and """"roofing material,"""" the answers would be """"petals"""" and """"slate."""" (Remove the """"p"""" of """"petals"""" and read the rest backward to get """"slate."""")" 3 "The name of what foreign currency rhymes with ""u-p""?" rupee 20090928 "Each answer starts with a clue for a six-letter word. If you drop the first letter and read the remaining letters backward, you'll get a five-letter word that answers a second clue. For example, if the clues are """"flower parts"""" and """"roofing material,"""" the answers would be """"petals"""" and """"slate."""" (Remove the """"p"""" of """"petals"""" and read the rest backward to get """"slate."""")" 4 "Name a section of a newspaper, insert the letters ""up"" after the first letter, and you will get new word that means ""holds up.""" sports/supports 20090928 "Each answer starts with a clue for a six-letter word. If you drop the first letter and read the remaining letters backward, you'll get a five-letter word that answers a second clue. For example, if the clues are """"flower parts"""" and """"roofing material,"""" the answers would be """"petals"""" and """"slate."""" (Remove the """"p"""" of """"petals"""" and read the rest backward to get """"slate."""")" 5 "Name a common exercise in six letters that contains two ""u's"" and two ""p's.""" pull-up 20090928 "Each answer starts with a clue for a six-letter word. If you drop the first letter and read the remaining letters backward, you'll get a five-letter word that answers a second clue. For example, if the clues are """"flower parts"""" and """"roofing material,"""" the answers would be """"petals"""" and """"slate."""" (Remove the """"p"""" of """"petals"""" and read the rest backward to get """"slate."""")" 6 "What classical name from Greek mythology starts phonetically with ""u"" and ends phonetically with ""p""?" Euterpe, muse of music 20090928 "Each answer starts with a clue for a six-letter word. If you drop the first letter and read the remaining letters backward, you'll get a five-letter word that answers a second clue. For example, if the clues are """"flower parts"""" and """"roofing material,"""" the answers would be """"petals"""" and """"slate."""" (Remove the """"p"""" of """"petals"""" and read the rest backward to get """"slate."""")" 7 "a famous general in the Civil War has an ""h"" in his name. Change the ""h"" to ""up,"" and you will have the name of a well-known hero (who was involved with a march to the Atlantic." Sherman/Superman 20100509 This week's on-air puzzle is an extension of last week's challenge in which a group of letters and symbols represent a familiar word or phrase. 0 write down the letters C O, and underneath, the letter S things that shoppers clip from newspapers 20100509 This week's on-air puzzle is an extension of last week's challenge in which a group of letters and symbols represent a familiar word or phrase. 1 write down the letters A S where the first modern Olympics were held 20100509 This week's on-air puzzle is an extension of last week's challenge in which a group of letters and symbols represent a familiar word or phrase. 2 S and then write R beams that support a roof 20100509 This week's on-air puzzle is an extension of last week's challenge in which a group of letters and symbols represent a familiar word or phrase. 3 write down M, an equals sign, a G, and underneath that write N to rule in a bad way 20100509 This week's on-air puzzle is an extension of last week's challenge in which a group of letters and symbols represent a familiar word or phrase. 4 write the letter D. Inside it, do a K over a W brilliant young person 20100509 This week's on-air puzzle is an extension of last week's challenge in which a group of letters and symbols represent a familiar word or phrase. 5 write down the letters W Y what a driver at an intersection might yield. It is a three word phrase, 5, 2, 3 20100509 This week's on-air puzzle is an extension of last week's challenge in which a group of letters and symbols represent a familiar word or phrase. 6 write down the word DESK left in a hurry, a 10-letter word 20100509 This week's on-air puzzle is an extension of last week's challenge in which a group of letters and symbols represent a familiar word or phrase. 7 write down these three letters, HYY a person who is good at fixing things. Its a two word phrase, 5, 4. 20100509 This week's on-air puzzle is an extension of last week's challenge in which a group of letters and symbols represent a familiar word or phrase. 8 Write down the letter C. Underneath it, write ENTS. Underneath that, write G. spies. It's a two word phrase, 10, 6. 20100718 "You are given two words and must provide a third word that can go in between them. The answer will follow the first word and precede the second to complete two familiar two-word phrases. In addition, the answer must fall in between the two words alphabetically. For example, given """"last"""" and """"lines,"""" the answer would be """"laugh"""" as in """"last laugh"""" and """"laugh lines.""""" 0 beer, button belly 20100718 "You are given two words and must provide a third word that can go in between them. The answer will follow the first word and precede the second to complete two familiar two-word phrases. In addition, the answer must fall in between the two words alphabetically. For example, given """"last"""" and """"lines,"""" the answer would be """"laugh"""" as in """"last laugh"""" and """"laugh lines.""""" 1 chemistry, clown class 20100718 "You are given two words and must provide a third word that can go in between them. The answer will follow the first word and precede the second to complete two familiar two-word phrases. In addition, the answer must fall in between the two words alphabetically. For example, given """"last"""" and """"lines,"""" the answer would be """"laugh"""" as in """"last laugh"""" and """"laugh lines.""""" 2 calling, catalog card 20100718 "You are given two words and must provide a third word that can go in between them. The answer will follow the first word and precede the second to complete two familiar two-word phrases. In addition, the answer must fall in between the two words alphabetically. For example, given """"last"""" and """"lines,"""" the answer would be """"laugh"""" as in """"last laugh"""" and """"laugh lines.""""" 3 bad, brother blood 20100718 "You are given two words and must provide a third word that can go in between them. The answer will follow the first word and precede the second to complete two familiar two-word phrases. In addition, the answer must fall in between the two words alphabetically. For example, given """"last"""" and """"lines,"""" the answer would be """"laugh"""" as in """"last laugh"""" and """"laugh lines.""""" 4 shelf, suit space 20100718 "You are given two words and must provide a third word that can go in between them. The answer will follow the first word and precede the second to complete two familiar two-word phrases. In addition, the answer must fall in between the two words alphabetically. For example, given """"last"""" and """"lines,"""" the answer would be """"laugh"""" as in """"last laugh"""" and """"laugh lines.""""" 5 cabin, cut crew 20100718 "You are given two words and must provide a third word that can go in between them. The answer will follow the first word and precede the second to complete two familiar two-word phrases. In addition, the answer must fall in between the two words alphabetically. For example, given """"last"""" and """"lines,"""" the answer would be """"laugh"""" as in """"last laugh"""" and """"laugh lines.""""" 6 safe, symbol sex 20100718 "You are given two words and must provide a third word that can go in between them. The answer will follow the first word and precede the second to complete two familiar two-word phrases. In addition, the answer must fall in between the two words alphabetically. For example, given """"last"""" and """"lines,"""" the answer would be """"laugh"""" as in """"last laugh"""" and """"laugh lines.""""" 7 banana, box bread 20100718 "You are given two words and must provide a third word that can go in between them. The answer will follow the first word and precede the second to complete two familiar two-word phrases. In addition, the answer must fall in between the two words alphabetically. For example, given """"last"""" and """"lines,"""" the answer would be """"laugh"""" as in """"last laugh"""" and """"laugh lines.""""" 8 bed, Bunny bugs 20090524 "Each clue is a pair of four-letter words. Each can be found inside the first and last names, respectively, of a famous person. For example, given """"rend"""" and """"rase,"""" the answer would be """"Brendan Fraser,"""" the actor." 0 arlo, rand Marlon Brando 20090524 "Each clue is a pair of four-letter words. Each can be found inside the first and last names, respectively, of a famous person. For example, given """"rend"""" and """"rase,"""" the answer would be """"Brendan Fraser,"""" the actor." 1 lady, nigh Gladys Knight 20090524 "Each clue is a pair of four-letter words. Each can be found inside the first and last names, respectively, of a famous person. For example, given """"rend"""" and """"rase,"""" the answer would be """"Brendan Fraser,"""" the actor." 2 meld, arco Imelda Marcos 20090524 "Each clue is a pair of four-letter words. Each can be found inside the first and last names, respectively, of a famous person. For example, given """"rend"""" and """"rase,"""" the answer would be """"Brendan Fraser,"""" the actor." 3 acid, ming Plácido Domingo 20090524 "Each clue is a pair of four-letter words. Each can be found inside the first and last names, respectively, of a famous person. For example, given """"rend"""" and """"rase,"""" the answer would be """"Brendan Fraser,"""" the actor." 4 else, lint Chelsea Clinton 20090524 "Each clue is a pair of four-letter words. Each can be found inside the first and last names, respectively, of a famous person. For example, given """"rend"""" and """"rase,"""" the answer would be """"Brendan Fraser,"""" the actor." 5 list, lock Calista Flockhart 20090524 "Each clue is a pair of four-letter words. Each can be found inside the first and last names, respectively, of a famous person. For example, given """"rend"""" and """"rase,"""" the answer would be """"Brendan Fraser,"""" the actor." 6 rove, elan Grover Cleveland 20090524 "Each clue is a pair of four-letter words. Each can be found inside the first and last names, respectively, of a famous person. For example, given """"rend"""" and """"rase,"""" the answer would be """"Brendan Fraser,"""" the actor." 7 rook, bins Brooks Robinson 20090524 "Each clue is a pair of four-letter words. Each can be found inside the first and last names, respectively, of a famous person. For example, given """"rend"""" and """"rase,"""" the answer would be """"Brendan Fraser,"""" the actor." 8 pole, part Napoleon Bonaparte 20090524 "Each clue is a pair of four-letter words. Each can be found inside the first and last names, respectively, of a famous person. For example, given """"rend"""" and """"rase,"""" the answer would be """"Brendan Fraser,"""" the actor." 9 bell, sell Isabella Rossellini 20160603 For each word given, name a famous person, past or present, whose name contains it. In each case, the word will bridge the first and last names. The dividing point is for you to discover. 1 egret Wayne Gretzky 20160603 For each word given, name a famous person, past or present, whose name contains it. In each case, the word will bridge the first and last names. The dividing point is for you to discover. 2 exhale Alex Haley 20160603 For each word given, name a famous person, past or present, whose name contains it. In each case, the word will bridge the first and last names. The dividing point is for you to discover. 3 lined Céline Dion 20160603 For each word given, name a famous person, past or present, whose name contains it. In each case, the word will bridge the first and last names. The dividing point is for you to discover. 4 ragers Ira Gershwin 20160603 For each word given, name a famous person, past or present, whose name contains it. In each case, the word will bridge the first and last names. The dividing point is for you to discover. 5 drums Donald Rumsfeld 20160603 For each word given, name a famous person, past or present, whose name contains it. In each case, the word will bridge the first and last names. The dividing point is for you to discover. 6 livers Oliver Stone 20160603 For each word given, name a famous person, past or present, whose name contains it. In each case, the word will bridge the first and last names. The dividing point is for you to discover. 7 indies Vin Diesel 20160207 Every answer given in this week's puzzle is a compound word or a familiar two-word phrase in which each part contains the consecutive letters A-N. 1 pencil-and-paper word game that involves guessing letters hANgmAN 20160207 Every answer given in this week's puzzle is a compound word or a familiar two-word phrase in which each part contains the consecutive letters A-N. 2 American _____ (old TV show hosted by Dick Clark) BANdstANd 20160207 Every answer given in this week's puzzle is a compound word or a familiar two-word phrase in which each part contains the consecutive letters A-N. 4 a 19th-century government in the West lANd grANt 20160207 Every answer given in this week's puzzle is a compound word or a familiar two-word phrase in which each part contains the consecutive letters A-N. 5 western part of Oklahoma pANhANdle 20160207 Every answer given in this week's puzzle is a compound word or a familiar two-word phrase in which each part contains the consecutive letters A-N. 6 musical group that a crowd might shimmy to dANce bANd 20160207 Every answer given in this week's puzzle is a compound word or a familiar two-word phrase in which each part contains the consecutive letters A-N. 7 a bit of gymnastics in which your palms are on the floor and your legs point upward hANdstANd 20160207 Every answer given in this week's puzzle is a compound word or a familiar two-word phrase in which each part contains the consecutive letters A-N. 8 gambling card game also known as parliament or sevens fAN tAN 20160207 Every answer given in this week's puzzle is a compound word or a familiar two-word phrase in which each part contains the consecutive letters A-N. 9 an employee who mends fences and brands cattle rANch hANd 20160207 Every answer given in this week's puzzle is a compound word or a familiar two-word phrase in which each part contains the consecutive letters A-N. 10 set of covered bleachers at a race track grANdstANd 20160207 Every answer given in this week's puzzle is a compound word or a familiar two-word phrase in which each part contains the consecutive letters A-N. 11 a phrase that means a thousand expressions of gratitude mANy thANks 20180425 Every answer today is an 8-letter word that consists of a 3-letter word inserted inside a 5-letter word. I'll give you a clue to the 3-letter word as well as the 5-letter word itself. You tell me the 8-letter word. 1 George Gallup, for one / Large sign that might be pinned on a wall pollster; poster 20180425 Every answer today is an 8-letter word that consists of a 3-letter word inserted inside a 5-letter word. I'll give you a clue to the 3-letter word as well as the 5-letter word itself. You tell me the 8-letter word. 2 Mussing, as feathers / Court decision ruffle, rule 20180425 Every answer today is an 8-letter word that consists of a 3-letter word inserted inside a 5-letter word. I'll give you a clue to the 3-letter word as well as the 5-letter word itself. You tell me the 8-letter word. 3 ___ column (part of a car) / Thin cord steering; string 20180425 Every answer today is an 8-letter word that consists of a 3-letter word inserted inside a 5-letter word. I'll give you a clue to the 3-letter word as well as the 5-letter word itself. You tell me the 8-letter word. 4 Buying and selling of goods / Force to do something commerce; coerce 20180425 Every answer today is an 8-letter word that consists of a 3-letter word inserted inside a 5-letter word. I'll give you a clue to the 3-letter word as well as the 5-letter word itself. You tell me the 8-letter word. 5 In a gloomy fashion / Sudden disappointment ?? 20180425 Every answer today is an 8-letter word that consists of a 3-letter word inserted inside a 5-letter word. I'll give you a clue to the 3-letter word as well as the 5-letter word itself. You tell me the 8-letter word. 6 Eliding or passing over / Activity at Vail, Colorado skipping; skiing 20180425 Every answer today is an 8-letter word that consists of a 3-letter word inserted inside a 5-letter word. I'll give you a clue to the 3-letter word as well as the 5-letter word itself. You tell me the 8-letter word. 7 Having no errors / Certain salt in chemistry [accurate; aurate] accurate; aurate 20180425 Every answer today is an 8-letter word that consists of a 3-letter word inserted inside a 5-letter word. I'll give you a clue to the 3-letter word as well as the 5-letter word itself. You tell me the 8-letter word. 8 Risququality / City in Wisconsin [Hint: Both answers consist of an 8-letter word with two consecutive pairs of doubled letters. Drop each pair to leave a 4-letter word that answers the second clue. raciness; Racine 20180425 Every answer today is an 8-letter word that consists of a 3-letter word inserted inside a 5-letter word. I'll give you a clue to the 3-letter word as well as the 5-letter word itself. You tell me the 8-letter word. 9 One sharing living quarters, as in a college dorm / Give four stars, for example roommate; rate 20180425 Every answer today is an 8-letter word that consists of a 3-letter word inserted inside a 5-letter word. I'll give you a clue to the 3-letter word as well as the 5-letter word itself. You tell me the 8-letter word. 10 Is victorious / Soapy water succeeds; suds 20180225 "Each sentence has two blanks. Reverse the first two letters of the word that goes in the first blank to get a new word that goes in the second blank to complete the sentence. Example: Until 1917 in Russia, there was no bigger ______ than the ______. --> STAR, TSAR 1. The wide branches of the ______ tree left ______ room for everyone to sit in the shade. 2. The company president guaranteed all her employees a ______ no matter what financial difficulties might ______. 3. Before going out on stage, ______ Presley changed out of his ______ into dress pants. 4. If the tailor runs out of time, he'll have to ______ the pants leg ______. 5. Of all the uncommon cases the school infirmary got, the ______ was cardiac ______. 6. For his Irish fans, the fact that the singer came from the province of ______ added ______ to his reputation. 7. Heinz is planning a new-fangled bottle for their ______, but every time I try it, it ______ (2 wds.). 8. Because of its conflicting statements about the use of ______ weapons, the government's policy on them was ______. 9. The flying instructor gave the student pilot considerable ______ in deciding what ______ to fly at. 10. Just for your ______, the word """"______"""" can be defined as """"the act of treating someone like a god."""" Last week's challenge: This challenge comes from listener Joseph Young, who conducts the weekly blog Puzzleria. Take the start of the name of a country and the end of that country's capital. Put the parts together, one after the other, and you'll get the last name of a character in a very popular movie. It's a character everyone knows. Who is it? Answer: Kenya + Nairobi --> (Obi-Wan) Kenobi Next week's challenge: This week's challenge comes from listener Chris Stuart, who is from the answer to this puzzle. Name a place in the United States that contains a W. Drop the W, and you can rearrange the remaining letters to name two types of mammals, each in the plural form. What place is it, and what are the mammals? Submit Your Answer If you know the answer to next week's challenge, submit it here. Listeners who submit correct answers win a chance to play the on-air puzzle. Important: Include a phone number where we can reach you Thursday, March 1 at 3 p.m. ET." 1 The wide branches of the ______ tree left ______ room for everyone to sit in the shade. maple, ample 20180225 "Each sentence has two blanks. Reverse the first two letters of the word that goes in the first blank to get a new word that goes in the second blank to complete the sentence. Example: Until 1917 in Russia, there was no bigger ______ than the ______. --> STAR, TSAR 1. The wide branches of the ______ tree left ______ room for everyone to sit in the shade. 2. The company president guaranteed all her employees a ______ no matter what financial difficulties might ______. 3. Before going out on stage, ______ Presley changed out of his ______ into dress pants. 4. If the tailor runs out of time, he'll have to ______ the pants leg ______. 5. Of all the uncommon cases the school infirmary got, the ______ was cardiac ______. 6. For his Irish fans, the fact that the singer came from the province of ______ added ______ to his reputation. 7. Heinz is planning a new-fangled bottle for their ______, but every time I try it, it ______ (2 wds.). 8. Because of its conflicting statements about the use of ______ weapons, the government's policy on them was ______. 9. The flying instructor gave the student pilot considerable ______ in deciding what ______ to fly at. 10. Just for your ______, the word """"______"""" can be defined as """"the act of treating someone like a god."""" Last week's challenge: This challenge comes from listener Joseph Young, who conducts the weekly blog Puzzleria. Take the start of the name of a country and the end of that country's capital. Put the parts together, one after the other, and you'll get the last name of a character in a very popular movie. It's a character everyone knows. Who is it? Answer: Kenya + Nairobi --> (Obi-Wan) Kenobi Next week's challenge: This week's challenge comes from listener Chris Stuart, who is from the answer to this puzzle. Name a place in the United States that contains a W. Drop the W, and you can rearrange the remaining letters to name two types of mammals, each in the plural form. What place is it, and what are the mammals? Submit Your Answer If you know the answer to next week's challenge, submit it here. Listeners who submit correct answers win a chance to play the on-air puzzle. Important: Include a phone number where we can reach you Thursday, March 1 at 3 p.m. ET." 2 The company president guaranteed all her employees a ______ no matter what financial difficulties might ______. raise, arise 20180225 "Each sentence has two blanks. Reverse the first two letters of the word that goes in the first blank to get a new word that goes in the second blank to complete the sentence. Example: Until 1917 in Russia, there was no bigger ______ than the ______. --> STAR, TSAR 1. The wide branches of the ______ tree left ______ room for everyone to sit in the shade. 2. The company president guaranteed all her employees a ______ no matter what financial difficulties might ______. 3. Before going out on stage, ______ Presley changed out of his ______ into dress pants. 4. If the tailor runs out of time, he'll have to ______ the pants leg ______. 5. Of all the uncommon cases the school infirmary got, the ______ was cardiac ______. 6. For his Irish fans, the fact that the singer came from the province of ______ added ______ to his reputation. 7. Heinz is planning a new-fangled bottle for their ______, but every time I try it, it ______ (2 wds.). 8. Because of its conflicting statements about the use of ______ weapons, the government's policy on them was ______. 9. The flying instructor gave the student pilot considerable ______ in deciding what ______ to fly at. 10. Just for your ______, the word """"______"""" can be defined as """"the act of treating someone like a god."""" Last week's challenge: This challenge comes from listener Joseph Young, who conducts the weekly blog Puzzleria. Take the start of the name of a country and the end of that country's capital. Put the parts together, one after the other, and you'll get the last name of a character in a very popular movie. It's a character everyone knows. Who is it? Answer: Kenya + Nairobi --> (Obi-Wan) Kenobi Next week's challenge: This week's challenge comes from listener Chris Stuart, who is from the answer to this puzzle. Name a place in the United States that contains a W. Drop the W, and you can rearrange the remaining letters to name two types of mammals, each in the plural form. What place is it, and what are the mammals? Submit Your Answer If you know the answer to next week's challenge, submit it here. Listeners who submit correct answers win a chance to play the on-air puzzle. Important: Include a phone number where we can reach you Thursday, March 1 at 3 p.m. ET." 3 Before going out on stage, ______ Presley changed out of his ______ into dress pants. Elvis, Levis 20180225 "Each sentence has two blanks. Reverse the first two letters of the word that goes in the first blank to get a new word that goes in the second blank to complete the sentence. Example: Until 1917 in Russia, there was no bigger ______ than the ______. --> STAR, TSAR 1. The wide branches of the ______ tree left ______ room for everyone to sit in the shade. 2. The company president guaranteed all her employees a ______ no matter what financial difficulties might ______. 3. Before going out on stage, ______ Presley changed out of his ______ into dress pants. 4. If the tailor runs out of time, he'll have to ______ the pants leg ______. 5. Of all the uncommon cases the school infirmary got, the ______ was cardiac ______. 6. For his Irish fans, the fact that the singer came from the province of ______ added ______ to his reputation. 7. Heinz is planning a new-fangled bottle for their ______, but every time I try it, it ______ (2 wds.). 8. Because of its conflicting statements about the use of ______ weapons, the government's policy on them was ______. 9. The flying instructor gave the student pilot considerable ______ in deciding what ______ to fly at. 10. Just for your ______, the word """"______"""" can be defined as """"the act of treating someone like a god."""" Last week's challenge: This challenge comes from listener Joseph Young, who conducts the weekly blog Puzzleria. Take the start of the name of a country and the end of that country's capital. Put the parts together, one after the other, and you'll get the last name of a character in a very popular movie. It's a character everyone knows. Who is it? Answer: Kenya + Nairobi --> (Obi-Wan) Kenobi Next week's challenge: This week's challenge comes from listener Chris Stuart, who is from the answer to this puzzle. Name a place in the United States that contains a W. Drop the W, and you can rearrange the remaining letters to name two types of mammals, each in the plural form. What place is it, and what are the mammals? Submit Your Answer If you know the answer to next week's challenge, submit it here. Listeners who submit correct answers win a chance to play the on-air puzzle. Important: Include a phone number where we can reach you Thursday, March 1 at 3 p.m. ET." 5 Of all the uncommon cases the school infirmary got, the ______ was cardiac ______. rarest, arrest 20180225 "Each sentence has two blanks. Reverse the first two letters of the word that goes in the first blank to get a new word that goes in the second blank to complete the sentence. Example: Until 1917 in Russia, there was no bigger ______ than the ______. --> STAR, TSAR 1. The wide branches of the ______ tree left ______ room for everyone to sit in the shade. 2. The company president guaranteed all her employees a ______ no matter what financial difficulties might ______. 3. Before going out on stage, ______ Presley changed out of his ______ into dress pants. 4. If the tailor runs out of time, he'll have to ______ the pants leg ______. 5. Of all the uncommon cases the school infirmary got, the ______ was cardiac ______. 6. For his Irish fans, the fact that the singer came from the province of ______ added ______ to his reputation. 7. Heinz is planning a new-fangled bottle for their ______, but every time I try it, it ______ (2 wds.). 8. Because of its conflicting statements about the use of ______ weapons, the government's policy on them was ______. 9. The flying instructor gave the student pilot considerable ______ in deciding what ______ to fly at. 10. Just for your ______, the word """"______"""" can be defined as """"the act of treating someone like a god."""" Last week's challenge: This challenge comes from listener Joseph Young, who conducts the weekly blog Puzzleria. Take the start of the name of a country and the end of that country's capital. Put the parts together, one after the other, and you'll get the last name of a character in a very popular movie. It's a character everyone knows. Who is it? Answer: Kenya + Nairobi --> (Obi-Wan) Kenobi Next week's challenge: This week's challenge comes from listener Chris Stuart, who is from the answer to this puzzle. Name a place in the United States that contains a W. Drop the W, and you can rearrange the remaining letters to name two types of mammals, each in the plural form. What place is it, and what are the mammals? Submit Your Answer If you know the answer to next week's challenge, submit it here. Listeners who submit correct answers win a chance to play the on-air puzzle. Important: Include a phone number where we can reach you Thursday, March 1 at 3 p.m. ET." 8 After its conflicting statements about the use of ______ weapons, the government's policy on them was ______. nuclear, unclear 20180225 "Each sentence has two blanks. Reverse the first two letters of the word that goes in the first blank to get a new word that goes in the second blank to complete the sentence. Example: Until 1917 in Russia, there was no bigger ______ than the ______. --> STAR, TSAR 1. The wide branches of the ______ tree left ______ room for everyone to sit in the shade. 2. The company president guaranteed all her employees a ______ no matter what financial difficulties might ______. 3. Before going out on stage, ______ Presley changed out of his ______ into dress pants. 4. If the tailor runs out of time, he'll have to ______ the pants leg ______. 5. Of all the uncommon cases the school infirmary got, the ______ was cardiac ______. 6. For his Irish fans, the fact that the singer came from the province of ______ added ______ to his reputation. 7. Heinz is planning a new-fangled bottle for their ______, but every time I try it, it ______ (2 wds.). 8. Because of its conflicting statements about the use of ______ weapons, the government's policy on them was ______. 9. The flying instructor gave the student pilot considerable ______ in deciding what ______ to fly at. 10. Just for your ______, the word """"______"""" can be defined as """"the act of treating someone like a god."""" Last week's challenge: This challenge comes from listener Joseph Young, who conducts the weekly blog Puzzleria. Take the start of the name of a country and the end of that country's capital. Put the parts together, one after the other, and you'll get the last name of a character in a very popular movie. It's a character everyone knows. Who is it? Answer: Kenya + Nairobi --> (Obi-Wan) Kenobi Next week's challenge: This week's challenge comes from listener Chris Stuart, who is from the answer to this puzzle. Name a place in the United States that contains a W. Drop the W, and you can rearrange the remaining letters to name two types of mammals, each in the plural form. What place is it, and what are the mammals? Submit Your Answer If you know the answer to next week's challenge, submit it here. Listeners who submit correct answers win a chance to play the on-air puzzle. Important: Include a phone number where we can reach you Thursday, March 1 at 3 p.m. ET." 9 The flying instructor gave the student pilot considerable ______ in deciding what ______ to fly at. latitude, altitude 20180225 "Each sentence has two blanks. Reverse the first two letters of the word that goes in the first blank to get a new word that goes in the second blank to complete the sentence. Example: Until 1917 in Russia, there was no bigger ______ than the ______. --> STAR, TSAR 1. The wide branches of the ______ tree left ______ room for everyone to sit in the shade. 2. The company president guaranteed all her employees a ______ no matter what financial difficulties might ______. 3. Before going out on stage, ______ Presley changed out of his ______ into dress pants. 4. If the tailor runs out of time, he'll have to ______ the pants leg ______. 5. Of all the uncommon cases the school infirmary got, the ______ was cardiac ______. 6. For his Irish fans, the fact that the singer came from the province of ______ added ______ to his reputation. 7. Heinz is planning a new-fangled bottle for their ______, but every time I try it, it ______ (2 wds.). 8. Because of its conflicting statements about the use of ______ weapons, the government's policy on them was ______. 9. The flying instructor gave the student pilot considerable ______ in deciding what ______ to fly at. 10. Just for your ______, the word """"______"""" can be defined as """"the act of treating someone like a god."""" Last week's challenge: This challenge comes from listener Joseph Young, who conducts the weekly blog Puzzleria. Take the start of the name of a country and the end of that country's capital. Put the parts together, one after the other, and you'll get the last name of a character in a very popular movie. It's a character everyone knows. Who is it? Answer: Kenya + Nairobi --> (Obi-Wan) Kenobi Next week's challenge: This week's challenge comes from listener Chris Stuart, who is from the answer to this puzzle. Name a place in the United States that contains a W. Drop the W, and you can rearrange the remaining letters to name two types of mammals, each in the plural form. What place is it, and what are the mammals? Submit Your Answer If you know the answer to next week's challenge, submit it here. Listeners who submit correct answers win a chance to play the on-air puzzle. Important: Include a phone number where we can reach you Thursday, March 1 at 3 p.m. ET." 10 "Just for your ______, the word """"______"""" can be defined as """"the act of treating someone like a god.”" edification, deification 20180204 "Every answer is a three-syllable word or name in which the middle syllable is accented and pronounced """"sigh.""""" 1 Matthew, John or Judas Iscariot  disciple 20180204 "Every answer is a three-syllable word or name in which the middle syllable is accented and pronounced """"sigh.""""" 2 God of the sea, in Greek myth Poseidon 20180204 "Every answer is a three-syllable word or name in which the middle syllable is accented and pronounced """"sigh.""""" 3 Figure out, as a secret message decipher 20180204 "Every answer is a three-syllable word or name in which the middle syllable is accented and pronounced """"sigh.""""" 4 Music or dance performance that a parent might sit through reading 20180204 "Every answer is a three-syllable word or name in which the middle syllable is accented and pronounced """"sigh.""""" 6 Kind of trading that's illegal insider 20180204 "Every answer is a three-syllable word or name in which the middle syllable is accented and pronounced """"sigh.""""" 7 Painful inflammation around joints bursitis 20180204 "Every answer is a three-syllable word or name in which the middle syllable is accented and pronounced """"sigh.""""" 8 Front tooth incisor 20180204 "Every answer is a three-syllable word or name in which the middle syllable is accented and pronounced """"sigh.""""" 9 * Place of safety asylum 20180204 "Every answer is a three-syllable word or name in which the middle syllable is accented and pronounced """"sigh.""""" 10 Clear and compelling, as a victory decisive 20180204 "Every answer is a three-syllable word or name in which the middle syllable is accented and pronounced """"sigh.""""" 11 Very enthusiastic excited 20180204 "Every answer is a three-syllable word or name in which the middle syllable is accented and pronounced """"sigh.""""" 13 # Religious savior messiah 20180128 "Each sentence has two blanks. Put a word starting with """"up"""" in the first blank. Move the """"up"""" to the end, and you'll get a familiar phrase that goes in the second blank to complete the sentence. Example: Jane was visibly _____ to have been _____ on such a terrible blind date. --> UPSET, SET UP 1. The coach was surprisingly _____ after his team had been _____ so badly. 2. Get ready: The musician's new tune played on the _____ piano is coming _____. 3. An outlaw gang rode into the Wild West town, with the _____ being they completely _____ the place. 4. The child's unusual _____ by the parents is a matter the child's therapist keeps _____. 5. The cost of my house's _____ was so high, I was always frantically working to _____ with bills. 6. The teller tried to _____ her principle of treating everyone kindly during the bank _____. 7. The unexpected snowstorm _____ the family's vacation plans, and they _____ staying at home. 8. Even a slight _____ in an ailing economy is nothing to _____ your nose at. 9. A fine, _____ young person is always _____ for his or her ideals. Last week's challenge: This challenge comes from listener Tom Arnold of Eugene, Ore. Take the name of a conveyance in 7 letters. Drop the middle letter, and the remaining letters can be rearranged to name the place where such a conveyance is often used. What is it? Answer: GONDOLA --> LAGOON Winner: Michael LeBlanc of Ontario, Canada. Next week's challenge: This challenge comes from listener Stuart Portnoy of Arlington, Va. Name a famous actor — first and last names. The last name is a well-known brand. Drop the last letter of the first name and you'll get the kind of product it's a brand of. Who is it? Submit Your Answer If you know the answer to next week's challenge, submit it here. Listeners who submit correct answers win a chance to play the on-air puzzle. Important: Include a phone number where we can reach you Thursday, Feb. 1 at 3 p.m. ET." 1 The coach was surprisingly _____ after his team had been _____ so badly. upbeat, beat up 20180128 "Each sentence has two blanks. Put a word starting with """"up"""" in the first blank. Move the """"up"""" to the end, and you'll get a familiar phrase that goes in the second blank to complete the sentence. Example: Jane was visibly _____ to have been _____ on such a terrible blind date. --> UPSET, SET UP 1. The coach was surprisingly _____ after his team had been _____ so badly. 2. Get ready: The musician's new tune played on the _____ piano is coming _____. 3. An outlaw gang rode into the Wild West town, with the _____ being they completely _____ the place. 4. The child's unusual _____ by the parents is a matter the child's therapist keeps _____. 5. The cost of my house's _____ was so high, I was always frantically working to _____ with bills. 6. The teller tried to _____ her principle of treating everyone kindly during the bank _____. 7. The unexpected snowstorm _____ the family's vacation plans, and they _____ staying at home. 8. Even a slight _____ in an ailing economy is nothing to _____ your nose at. 9. A fine, _____ young person is always _____ for his or her ideals. Last week's challenge: This challenge comes from listener Tom Arnold of Eugene, Ore. Take the name of a conveyance in 7 letters. Drop the middle letter, and the remaining letters can be rearranged to name the place where such a conveyance is often used. What is it? Answer: GONDOLA --> LAGOON Winner: Michael LeBlanc of Ontario, Canada. Next week's challenge: This challenge comes from listener Stuart Portnoy of Arlington, Va. Name a famous actor — first and last names. The last name is a well-known brand. Drop the last letter of the first name and you'll get the kind of product it's a brand of. Who is it? Submit Your Answer If you know the answer to next week's challenge, submit it here. Listeners who submit correct answers win a chance to play the on-air puzzle. Important: Include a phone number where we can reach you Thursday, Feb. 1 at 3 p.m. ET." 2 Get ready: The musician's new tune played on the _____ piano is coming _____. [upright, right up]3. An outlaw gang rode into the Wild West town, with the _____ being they completely _____ the place. [upshot, shot up]4. The child's unusual _____ by the parents is a matter the child's therapist keeps _____. [upbringing, bringing up]5. The cost of my house's _____ was so high, I was always frantically working to _____ with bills. [upkeep, keep up]6. The teller tried to _____ her principle of treating everyone kindly during the bank _____. [uphold, hold up]7. The unexpected snowstorm _____ the family's vacation plans, and they _____ staying at home. [upended, ended up]8. Even a slight _____ in an ailing economy is nothing to _____ your nose at. [upturn, turn up]9. A fine, _____ young person is always _____ for his or her ideals. upstanding, standing up 20180128 "Each sentence has two blanks. Put a word starting with """"up"""" in the first blank. Move the """"up"""" to the end, and you'll get a familiar phrase that goes in the second blank to complete the sentence. Example: Jane was visibly _____ to have been _____ on such a terrible blind date. --> UPSET, SET UP 1. The coach was surprisingly _____ after his team had been _____ so badly. 2. Get ready: The musician's new tune played on the _____ piano is coming _____. 3. An outlaw gang rode into the Wild West town, with the _____ being they completely _____ the place. 4. The child's unusual _____ by the parents is a matter the child's therapist keeps _____. 5. The cost of my house's _____ was so high, I was always frantically working to _____ with bills. 6. The teller tried to _____ her principle of treating everyone kindly during the bank _____. 7. The unexpected snowstorm _____ the family's vacation plans, and they _____ staying at home. 8. Even a slight _____ in an ailing economy is nothing to _____ your nose at. 9. A fine, _____ young person is always _____ for his or her ideals. Last week's challenge: This challenge comes from listener Tom Arnold of Eugene, Ore. Take the name of a conveyance in 7 letters. Drop the middle letter, and the remaining letters can be rearranged to name the place where such a conveyance is often used. What is it? Answer: GONDOLA --> LAGOON Winner: Michael LeBlanc of Ontario, Canada. Next week's challenge: This challenge comes from listener Stuart Portnoy of Arlington, Va. Name a famous actor — first and last names. The last name is a well-known brand. Drop the last letter of the first name and you'll get the kind of product it's a brand of. Who is it? Submit Your Answer If you know the answer to next week's challenge, submit it here. Listeners who submit correct answers win a chance to play the on-air puzzle. Important: Include a phone number where we can reach you Thursday, Feb. 1 at 3 p.m. ET." 2 Get ready: The musician's new tune played on the _____ piano is coming _____. upright, right up 20180128 "Each sentence has two blanks. Put a word starting with """"up"""" in the first blank. Move the """"up"""" to the end, and you'll get a familiar phrase that goes in the second blank to complete the sentence. Example: Jane was visibly _____ to have been _____ on such a terrible blind date. --> UPSET, SET UP 1. The coach was surprisingly _____ after his team had been _____ so badly. 2. Get ready: The musician's new tune played on the _____ piano is coming _____. 3. An outlaw gang rode into the Wild West town, with the _____ being they completely _____ the place. 4. The child's unusual _____ by the parents is a matter the child's therapist keeps _____. 5. The cost of my house's _____ was so high, I was always frantically working to _____ with bills. 6. The teller tried to _____ her principle of treating everyone kindly during the bank _____. 7. The unexpected snowstorm _____ the family's vacation plans, and they _____ staying at home. 8. Even a slight _____ in an ailing economy is nothing to _____ your nose at. 9. A fine, _____ young person is always _____ for his or her ideals. Last week's challenge: This challenge comes from listener Tom Arnold of Eugene, Ore. Take the name of a conveyance in 7 letters. Drop the middle letter, and the remaining letters can be rearranged to name the place where such a conveyance is often used. What is it? Answer: GONDOLA --> LAGOON Winner: Michael LeBlanc of Ontario, Canada. Next week's challenge: This challenge comes from listener Stuart Portnoy of Arlington, Va. Name a famous actor — first and last names. The last name is a well-known brand. Drop the last letter of the first name and you'll get the kind of product it's a brand of. Who is it? Submit Your Answer If you know the answer to next week's challenge, submit it here. Listeners who submit correct answers win a chance to play the on-air puzzle. Important: Include a phone number where we can reach you Thursday, Feb. 1 at 3 p.m. ET." 3 An outlaw gang rode into the Wild West town, with the _____ being they completely _____ the place. upshot, shot up 20180128 "Each sentence has two blanks. Put a word starting with """"up"""" in the first blank. Move the """"up"""" to the end, and you'll get a familiar phrase that goes in the second blank to complete the sentence. Example: Jane was visibly _____ to have been _____ on such a terrible blind date. --> UPSET, SET UP 1. The coach was surprisingly _____ after his team had been _____ so badly. 2. Get ready: The musician's new tune played on the _____ piano is coming _____. 3. An outlaw gang rode into the Wild West town, with the _____ being they completely _____ the place. 4. The child's unusual _____ by the parents is a matter the child's therapist keeps _____. 5. The cost of my house's _____ was so high, I was always frantically working to _____ with bills. 6. The teller tried to _____ her principle of treating everyone kindly during the bank _____. 7. The unexpected snowstorm _____ the family's vacation plans, and they _____ staying at home. 8. Even a slight _____ in an ailing economy is nothing to _____ your nose at. 9. A fine, _____ young person is always _____ for his or her ideals. Last week's challenge: This challenge comes from listener Tom Arnold of Eugene, Ore. Take the name of a conveyance in 7 letters. Drop the middle letter, and the remaining letters can be rearranged to name the place where such a conveyance is often used. What is it? Answer: GONDOLA --> LAGOON Winner: Michael LeBlanc of Ontario, Canada. Next week's challenge: This challenge comes from listener Stuart Portnoy of Arlington, Va. Name a famous actor — first and last names. The last name is a well-known brand. Drop the last letter of the first name and you'll get the kind of product it's a brand of. Who is it? Submit Your Answer If you know the answer to next week's challenge, submit it here. Listeners who submit correct answers win a chance to play the on-air puzzle. Important: Include a phone number where we can reach you Thursday, Feb. 1 at 3 p.m. ET." 5 The cost of my house's _____ was so high, I was always frantically working to _____ with bills. upkeep, keep up 20180128 "Each sentence has two blanks. Put a word starting with """"up"""" in the first blank. Move the """"up"""" to the end, and you'll get a familiar phrase that goes in the second blank to complete the sentence. Example: Jane was visibly _____ to have been _____ on such a terrible blind date. --> UPSET, SET UP 1. The coach was surprisingly _____ after his team had been _____ so badly. 2. Get ready: The musician's new tune played on the _____ piano is coming _____. 3. An outlaw gang rode into the Wild West town, with the _____ being they completely _____ the place. 4. The child's unusual _____ by the parents is a matter the child's therapist keeps _____. 5. The cost of my house's _____ was so high, I was always frantically working to _____ with bills. 6. The teller tried to _____ her principle of treating everyone kindly during the bank _____. 7. The unexpected snowstorm _____ the family's vacation plans, and they _____ staying at home. 8. Even a slight _____ in an ailing economy is nothing to _____ your nose at. 9. A fine, _____ young person is always _____ for his or her ideals. Last week's challenge: This challenge comes from listener Tom Arnold of Eugene, Ore. Take the name of a conveyance in 7 letters. Drop the middle letter, and the remaining letters can be rearranged to name the place where such a conveyance is often used. What is it? Answer: GONDOLA --> LAGOON Winner: Michael LeBlanc of Ontario, Canada. Next week's challenge: This challenge comes from listener Stuart Portnoy of Arlington, Va. Name a famous actor — first and last names. The last name is a well-known brand. Drop the last letter of the first name and you'll get the kind of product it's a brand of. Who is it? Submit Your Answer If you know the answer to next week's challenge, submit it here. Listeners who submit correct answers win a chance to play the on-air puzzle. Important: Include a phone number where we can reach you Thursday, Feb. 1 at 3 p.m. ET." 6 The teller tried to _____ her principle of treating everyone kindly during the bank _____. uphold, hold up 20180128 "Each sentence has two blanks. Put a word starting with """"up"""" in the first blank. Move the """"up"""" to the end, and you'll get a familiar phrase that goes in the second blank to complete the sentence. Example: Jane was visibly _____ to have been _____ on such a terrible blind date. --> UPSET, SET UP 1. The coach was surprisingly _____ after his team had been _____ so badly. 2. Get ready: The musician's new tune played on the _____ piano is coming _____. 3. An outlaw gang rode into the Wild West town, with the _____ being they completely _____ the place. 4. The child's unusual _____ by the parents is a matter the child's therapist keeps _____. 5. The cost of my house's _____ was so high, I was always frantically working to _____ with bills. 6. The teller tried to _____ her principle of treating everyone kindly during the bank _____. 7. The unexpected snowstorm _____ the family's vacation plans, and they _____ staying at home. 8. Even a slight _____ in an ailing economy is nothing to _____ your nose at. 9. A fine, _____ young person is always _____ for his or her ideals. Last week's challenge: This challenge comes from listener Tom Arnold of Eugene, Ore. Take the name of a conveyance in 7 letters. Drop the middle letter, and the remaining letters can be rearranged to name the place where such a conveyance is often used. What is it? Answer: GONDOLA --> LAGOON Winner: Michael LeBlanc of Ontario, Canada. Next week's challenge: This challenge comes from listener Stuart Portnoy of Arlington, Va. Name a famous actor — first and last names. The last name is a well-known brand. Drop the last letter of the first name and you'll get the kind of product it's a brand of. Who is it? Submit Your Answer If you know the answer to next week's challenge, submit it here. Listeners who submit correct answers win a chance to play the on-air puzzle. Important: Include a phone number where we can reach you Thursday, Feb. 1 at 3 p.m. ET." 7 The unexpected snowstorm _____ the family's vacation plans, and they _____ staying at home. upended, ended up 20180128 "Each sentence has two blanks. Put a word starting with """"up"""" in the first blank. Move the """"up"""" to the end, and you'll get a familiar phrase that goes in the second blank to complete the sentence. Example: Jane was visibly _____ to have been _____ on such a terrible blind date. --> UPSET, SET UP 1. The coach was surprisingly _____ after his team had been _____ so badly. 2. Get ready: The musician's new tune played on the _____ piano is coming _____. 3. An outlaw gang rode into the Wild West town, with the _____ being they completely _____ the place. 4. The child's unusual _____ by the parents is a matter the child's therapist keeps _____. 5. The cost of my house's _____ was so high, I was always frantically working to _____ with bills. 6. The teller tried to _____ her principle of treating everyone kindly during the bank _____. 7. The unexpected snowstorm _____ the family's vacation plans, and they _____ staying at home. 8. Even a slight _____ in an ailing economy is nothing to _____ your nose at. 9. A fine, _____ young person is always _____ for his or her ideals. Last week's challenge: This challenge comes from listener Tom Arnold of Eugene, Ore. Take the name of a conveyance in 7 letters. Drop the middle letter, and the remaining letters can be rearranged to name the place where such a conveyance is often used. What is it? Answer: GONDOLA --> LAGOON Winner: Michael LeBlanc of Ontario, Canada. Next week's challenge: This challenge comes from listener Stuart Portnoy of Arlington, Va. Name a famous actor — first and last names. The last name is a well-known brand. Drop the last letter of the first name and you'll get the kind of product it's a brand of. Who is it? Submit Your Answer If you know the answer to next week's challenge, submit it here. Listeners who submit correct answers win a chance to play the on-air puzzle. Important: Include a phone number where we can reach you Thursday, Feb. 1 at 3 p.m. ET." 8 Even a slight _____ in an ailing economy is nothing to _____ your nose at. upturn, turn up 20180128 "Each sentence has two blanks. Put a word starting with """"up"""" in the first blank. Move the """"up"""" to the end, and you'll get a familiar phrase that goes in the second blank to complete the sentence. Example: Jane was visibly _____ to have been _____ on such a terrible blind date. --> UPSET, SET UP 1. The coach was surprisingly _____ after his team had been _____ so badly. 2. Get ready: The musician's new tune played on the _____ piano is coming _____. 3. An outlaw gang rode into the Wild West town, with the _____ being they completely _____ the place. 4. The child's unusual _____ by the parents is a matter the child's therapist keeps _____. 5. The cost of my house's _____ was so high, I was always frantically working to _____ with bills. 6. The teller tried to _____ her principle of treating everyone kindly during the bank _____. 7. The unexpected snowstorm _____ the family's vacation plans, and they _____ staying at home. 8. Even a slight _____ in an ailing economy is nothing to _____ your nose at. 9. A fine, _____ young person is always _____ for his or her ideals. Last week's challenge: This challenge comes from listener Tom Arnold of Eugene, Ore. Take the name of a conveyance in 7 letters. Drop the middle letter, and the remaining letters can be rearranged to name the place where such a conveyance is often used. What is it? Answer: GONDOLA --> LAGOON Winner: Michael LeBlanc of Ontario, Canada. Next week's challenge: This challenge comes from listener Stuart Portnoy of Arlington, Va. Name a famous actor — first and last names. The last name is a well-known brand. Drop the last letter of the first name and you'll get the kind of product it's a brand of. Who is it? Submit Your Answer If you know the answer to next week's challenge, submit it here. Listeners who submit correct answers win a chance to play the on-air puzzle. Important: Include a phone number where we can reach you Thursday, Feb. 1 at 3 p.m. ET." 9 A fine, _____ young person is always _____ for his or her ideals. upstanding, standing up 20180107 I'm going to give you three 5-letter words. You give me a fourth 5-letter word that can precede each of mine to complete a compound word or a familiar 2-word phrase. 1 alley, trust, faith blind 20180107 I'm going to give you three 5-letter words. You give me a fourth 5-letter word that can precede each of mine to complete a compound word or a familiar 2-word phrase. 2 whale, noise, pages white 20180107 I'm going to give you three 5-letter words. You give me a fourth 5-letter word that can precede each of mine to complete a compound word or a familiar 2-word phrase. 3 alarm, teeth, front FALSE 20180107 I'm going to give you three 5-letter words. You give me a fourth 5-letter word that can precede each of mine to complete a compound word or a familiar 2-word phrase. 4 speed, opera, touch light 20180107 I'm going to give you three 5-letter words. You give me a fourth 5-letter word that can precede each of mine to complete a compound word or a familiar 2-word phrase. 5 child, piano, total grand 20180107 I'm going to give you three 5-letter words. You give me a fourth 5-letter word that can precede each of mine to complete a compound word or a familiar 2-word phrase. 6 charm, guess, stiff lucky 20180107 I'm going to give you three 5-letter words. You give me a fourth 5-letter word that can precede each of mine to complete a compound word or a familiar 2-word phrase. 7 story, order, bread short 20180107 I'm going to give you three 5-letter words. You give me a fourth 5-letter word that can precede each of mine to complete a compound word or a familiar 2-word phrase. 8 robin, dance, table round 20171224 "NPR stands for National Public Radio, of course. But the letters N-P-R are also the initials of """"nail polish remover."""" I'm going to give you clues to some other three-word names and phrases whose initials stand for better-known things. 1. I.C.C. (Interstate Commerce Commission) — Basic purchase at Dairy Queen2. S.L.R. (Single-Lens Reflex) — Waterway past Montreal and Quebec3. N.F.L. (National Football League) — Principle that an object at rest stays at rest unless acted upon4. B.B.C. (British Broadcasting Corporaton) — Noted Wild West showman5. J.F.K. (John Fitzgerald Kennedy) — Not with any serious purpose in mind6. D.D.S. (Doctor of dental surgery) — Means of classification of library books7. F.T.C. (Federal Trade Commission) — Something you're said to search with if you do a very careful search8. B.F.F. (Best friends forever) — Event for cinephiles in Germany9. L.C.D. (Light-emitting diode or lowest common denominator) — Rimsky-Korsakov opera whose French title translates as """"The Golden Cockerel""""10. N.B.C. (National Broadcasting Company) — Evening of December 24 Last week's challenge: This challenge came from listener Neville Fogarty of Newport News, Va. Think of a convenience introduced in the 19th century that is still around today. Its name has two words. Take the first three letters of the first word and the first letter of its second word, in order, to get a convenience introduced in the 21st century that serves a similar purpose. Their names are otherwise unrelated. What two conveniences are these?Answer: Yellow Pages --> Yelp Winner: Kevin Devine of Marlborough, Mass. Next week's challenge: The name of what well-known U.S. city, in 10 letters, contains only three different letters of the alphabet? Submit Your Answer If you know the answer to next week's challenge, submit it here. Listeners who submit correct answers win a chance to play the on-air puzzle. Important: Include a phone number where we can reach you Thursday, Dec. 28 at 3 p.m. ET." 1 I.C.C. (Interstate Commerce Commission) — Basic purchase at Dairy Queen ice cream cone 20171224 "NPR stands for National Public Radio, of course. But the letters N-P-R are also the initials of """"nail polish remover."""" I'm going to give you clues to some other three-word names and phrases whose initials stand for better-known things. 1. I.C.C. (Interstate Commerce Commission) — Basic purchase at Dairy Queen2. S.L.R. (Single-Lens Reflex) — Waterway past Montreal and Quebec3. N.F.L. (National Football League) — Principle that an object at rest stays at rest unless acted upon4. B.B.C. (British Broadcasting Corporaton) — Noted Wild West showman5. J.F.K. (John Fitzgerald Kennedy) — Not with any serious purpose in mind6. D.D.S. (Doctor of dental surgery) — Means of classification of library books7. F.T.C. (Federal Trade Commission) — Something you're said to search with if you do a very careful search8. B.F.F. (Best friends forever) — Event for cinephiles in Germany9. L.C.D. (Light-emitting diode or lowest common denominator) — Rimsky-Korsakov opera whose French title translates as """"The Golden Cockerel""""10. N.B.C. (National Broadcasting Company) — Evening of December 24 Last week's challenge: This challenge came from listener Neville Fogarty of Newport News, Va. Think of a convenience introduced in the 19th century that is still around today. Its name has two words. Take the first three letters of the first word and the first letter of its second word, in order, to get a convenience introduced in the 21st century that serves a similar purpose. Their names are otherwise unrelated. What two conveniences are these?Answer: Yellow Pages --> Yelp Winner: Kevin Devine of Marlborough, Mass. Next week's challenge: The name of what well-known U.S. city, in 10 letters, contains only three different letters of the alphabet? Submit Your Answer If you know the answer to next week's challenge, submit it here. Listeners who submit correct answers win a chance to play the on-air puzzle. Important: Include a phone number where we can reach you Thursday, Dec. 28 at 3 p.m. ET." 2 S.L.R. (Single-Lens Reflex) — Waterway past Montreal and Quebec St. Lawrence River 20171224 "NPR stands for National Public Radio, of course. But the letters N-P-R are also the initials of """"nail polish remover."""" I'm going to give you clues to some other three-word names and phrases whose initials stand for better-known things. 1. I.C.C. (Interstate Commerce Commission) — Basic purchase at Dairy Queen2. S.L.R. (Single-Lens Reflex) — Waterway past Montreal and Quebec3. N.F.L. (National Football League) — Principle that an object at rest stays at rest unless acted upon4. B.B.C. (British Broadcasting Corporaton) — Noted Wild West showman5. J.F.K. (John Fitzgerald Kennedy) — Not with any serious purpose in mind6. D.D.S. (Doctor of dental surgery) — Means of classification of library books7. F.T.C. (Federal Trade Commission) — Something you're said to search with if you do a very careful search8. B.F.F. (Best friends forever) — Event for cinephiles in Germany9. L.C.D. (Light-emitting diode or lowest common denominator) — Rimsky-Korsakov opera whose French title translates as """"The Golden Cockerel""""10. N.B.C. (National Broadcasting Company) — Evening of December 24 Last week's challenge: This challenge came from listener Neville Fogarty of Newport News, Va. Think of a convenience introduced in the 19th century that is still around today. Its name has two words. Take the first three letters of the first word and the first letter of its second word, in order, to get a convenience introduced in the 21st century that serves a similar purpose. Their names are otherwise unrelated. What two conveniences are these?Answer: Yellow Pages --> Yelp Winner: Kevin Devine of Marlborough, Mass. Next week's challenge: The name of what well-known U.S. city, in 10 letters, contains only three different letters of the alphabet? Submit Your Answer If you know the answer to next week's challenge, submit it here. Listeners who submit correct answers win a chance to play the on-air puzzle. Important: Include a phone number where we can reach you Thursday, Dec. 28 at 3 p.m. ET." 3 N.F.L. (National Football League) — Principle that an object at rest stays at rest unless acted upon Newton’s First Law 20171224 "NPR stands for National Public Radio, of course. But the letters N-P-R are also the initials of """"nail polish remover."""" I'm going to give you clues to some other three-word names and phrases whose initials stand for better-known things. 1. I.C.C. (Interstate Commerce Commission) — Basic purchase at Dairy Queen2. S.L.R. (Single-Lens Reflex) — Waterway past Montreal and Quebec3. N.F.L. (National Football League) — Principle that an object at rest stays at rest unless acted upon4. B.B.C. (British Broadcasting Corporaton) — Noted Wild West showman5. J.F.K. (John Fitzgerald Kennedy) — Not with any serious purpose in mind6. D.D.S. (Doctor of dental surgery) — Means of classification of library books7. F.T.C. (Federal Trade Commission) — Something you're said to search with if you do a very careful search8. B.F.F. (Best friends forever) — Event for cinephiles in Germany9. L.C.D. (Light-emitting diode or lowest common denominator) — Rimsky-Korsakov opera whose French title translates as """"The Golden Cockerel""""10. N.B.C. (National Broadcasting Company) — Evening of December 24 Last week's challenge: This challenge came from listener Neville Fogarty of Newport News, Va. Think of a convenience introduced in the 19th century that is still around today. Its name has two words. Take the first three letters of the first word and the first letter of its second word, in order, to get a convenience introduced in the 21st century that serves a similar purpose. Their names are otherwise unrelated. What two conveniences are these?Answer: Yellow Pages --> Yelp Winner: Kevin Devine of Marlborough, Mass. Next week's challenge: The name of what well-known U.S. city, in 10 letters, contains only three different letters of the alphabet? Submit Your Answer If you know the answer to next week's challenge, submit it here. Listeners who submit correct answers win a chance to play the on-air puzzle. Important: Include a phone number where we can reach you Thursday, Dec. 28 at 3 p.m. ET." 5 J.F.K. (John Fitzgerald Kennedy) — Not with any serious purpose in mind just for kicks 20171224 "NPR stands for National Public Radio, of course. But the letters N-P-R are also the initials of """"nail polish remover."""" I'm going to give you clues to some other three-word names and phrases whose initials stand for better-known things. 1. I.C.C. (Interstate Commerce Commission) — Basic purchase at Dairy Queen2. S.L.R. (Single-Lens Reflex) — Waterway past Montreal and Quebec3. N.F.L. (National Football League) — Principle that an object at rest stays at rest unless acted upon4. B.B.C. (British Broadcasting Corporaton) — Noted Wild West showman5. J.F.K. (John Fitzgerald Kennedy) — Not with any serious purpose in mind6. D.D.S. (Doctor of dental surgery) — Means of classification of library books7. F.T.C. (Federal Trade Commission) — Something you're said to search with if you do a very careful search8. B.F.F. (Best friends forever) — Event for cinephiles in Germany9. L.C.D. (Light-emitting diode or lowest common denominator) — Rimsky-Korsakov opera whose French title translates as """"The Golden Cockerel""""10. N.B.C. (National Broadcasting Company) — Evening of December 24 Last week's challenge: This challenge came from listener Neville Fogarty of Newport News, Va. Think of a convenience introduced in the 19th century that is still around today. Its name has two words. Take the first three letters of the first word and the first letter of its second word, in order, to get a convenience introduced in the 21st century that serves a similar purpose. Their names are otherwise unrelated. What two conveniences are these?Answer: Yellow Pages --> Yelp Winner: Kevin Devine of Marlborough, Mass. Next week's challenge: The name of what well-known U.S. city, in 10 letters, contains only three different letters of the alphabet? Submit Your Answer If you know the answer to next week's challenge, submit it here. Listeners who submit correct answers win a chance to play the on-air puzzle. Important: Include a phone number where we can reach you Thursday, Dec. 28 at 3 p.m. ET." 6 D.D.S. (Doctor of dental surgery) — Means of classification of library books Dewey Decimal System 20171224 "NPR stands for National Public Radio, of course. But the letters N-P-R are also the initials of """"nail polish remover."""" I'm going to give you clues to some other three-word names and phrases whose initials stand for better-known things. 1. I.C.C. (Interstate Commerce Commission) — Basic purchase at Dairy Queen2. S.L.R. (Single-Lens Reflex) — Waterway past Montreal and Quebec3. N.F.L. (National Football League) — Principle that an object at rest stays at rest unless acted upon4. B.B.C. (British Broadcasting Corporaton) — Noted Wild West showman5. J.F.K. (John Fitzgerald Kennedy) — Not with any serious purpose in mind6. D.D.S. (Doctor of dental surgery) — Means of classification of library books7. F.T.C. (Federal Trade Commission) — Something you're said to search with if you do a very careful search8. B.F.F. (Best friends forever) — Event for cinephiles in Germany9. L.C.D. (Light-emitting diode or lowest common denominator) — Rimsky-Korsakov opera whose French title translates as """"The Golden Cockerel""""10. N.B.C. (National Broadcasting Company) — Evening of December 24 Last week's challenge: This challenge came from listener Neville Fogarty of Newport News, Va. Think of a convenience introduced in the 19th century that is still around today. Its name has two words. Take the first three letters of the first word and the first letter of its second word, in order, to get a convenience introduced in the 21st century that serves a similar purpose. Their names are otherwise unrelated. What two conveniences are these?Answer: Yellow Pages --> Yelp Winner: Kevin Devine of Marlborough, Mass. Next week's challenge: The name of what well-known U.S. city, in 10 letters, contains only three different letters of the alphabet? Submit Your Answer If you know the answer to next week's challenge, submit it here. Listeners who submit correct answers win a chance to play the on-air puzzle. Important: Include a phone number where we can reach you Thursday, Dec. 28 at 3 p.m. ET." 7 F.T.C. (Federal Trade Commission) — Something you're said to search with if you do a very careful search fine tooth comb 20171224 "NPR stands for National Public Radio, of course. But the letters N-P-R are also the initials of """"nail polish remover."""" I'm going to give you clues to some other three-word names and phrases whose initials stand for better-known things. 1. I.C.C. (Interstate Commerce Commission) — Basic purchase at Dairy Queen2. S.L.R. (Single-Lens Reflex) — Waterway past Montreal and Quebec3. N.F.L. (National Football League) — Principle that an object at rest stays at rest unless acted upon4. B.B.C. (British Broadcasting Corporaton) — Noted Wild West showman5. J.F.K. (John Fitzgerald Kennedy) — Not with any serious purpose in mind6. D.D.S. (Doctor of dental surgery) — Means of classification of library books7. F.T.C. (Federal Trade Commission) — Something you're said to search with if you do a very careful search8. B.F.F. (Best friends forever) — Event for cinephiles in Germany9. L.C.D. (Light-emitting diode or lowest common denominator) — Rimsky-Korsakov opera whose French title translates as """"The Golden Cockerel""""10. N.B.C. (National Broadcasting Company) — Evening of December 24 Last week's challenge: This challenge came from listener Neville Fogarty of Newport News, Va. Think of a convenience introduced in the 19th century that is still around today. Its name has two words. Take the first three letters of the first word and the first letter of its second word, in order, to get a convenience introduced in the 21st century that serves a similar purpose. Their names are otherwise unrelated. What two conveniences are these?Answer: Yellow Pages --> Yelp Winner: Kevin Devine of Marlborough, Mass. Next week's challenge: The name of what well-known U.S. city, in 10 letters, contains only three different letters of the alphabet? Submit Your Answer If you know the answer to next week's challenge, submit it here. Listeners who submit correct answers win a chance to play the on-air puzzle. Important: Include a phone number where we can reach you Thursday, Dec. 28 at 3 p.m. ET." 8 B.F.F. (Best friends forever) — Event for cinephiles in Germany Berlin (International) Film Festival 20171224 "NPR stands for National Public Radio, of course. But the letters N-P-R are also the initials of """"nail polish remover."""" I'm going to give you clues to some other three-word names and phrases whose initials stand for better-known things. 1. I.C.C. (Interstate Commerce Commission) — Basic purchase at Dairy Queen2. S.L.R. (Single-Lens Reflex) — Waterway past Montreal and Quebec3. N.F.L. (National Football League) — Principle that an object at rest stays at rest unless acted upon4. B.B.C. (British Broadcasting Corporaton) — Noted Wild West showman5. J.F.K. (John Fitzgerald Kennedy) — Not with any serious purpose in mind6. D.D.S. (Doctor of dental surgery) — Means of classification of library books7. F.T.C. (Federal Trade Commission) — Something you're said to search with if you do a very careful search8. B.F.F. (Best friends forever) — Event for cinephiles in Germany9. L.C.D. (Light-emitting diode or lowest common denominator) — Rimsky-Korsakov opera whose French title translates as """"The Golden Cockerel""""10. N.B.C. (National Broadcasting Company) — Evening of December 24 Last week's challenge: This challenge came from listener Neville Fogarty of Newport News, Va. Think of a convenience introduced in the 19th century that is still around today. Its name has two words. Take the first three letters of the first word and the first letter of its second word, in order, to get a convenience introduced in the 21st century that serves a similar purpose. Their names are otherwise unrelated. What two conveniences are these?Answer: Yellow Pages --> Yelp Winner: Kevin Devine of Marlborough, Mass. Next week's challenge: The name of what well-known U.S. city, in 10 letters, contains only three different letters of the alphabet? Submit Your Answer If you know the answer to next week's challenge, submit it here. Listeners who submit correct answers win a chance to play the on-air puzzle. Important: Include a phone number where we can reach you Thursday, Dec. 28 at 3 p.m. ET." 9 "L.C.D. (Light-emitting diode or lowest common denominator) — Rimsky-Korsakov opera whose French title translates as """"The Golden Cockerel”" Le Coq d’Or 20171224 "NPR stands for National Public Radio, of course. But the letters N-P-R are also the initials of """"nail polish remover."""" I'm going to give you clues to some other three-word names and phrases whose initials stand for better-known things. 1. I.C.C. (Interstate Commerce Commission) — Basic purchase at Dairy Queen2. S.L.R. (Single-Lens Reflex) — Waterway past Montreal and Quebec3. N.F.L. (National Football League) — Principle that an object at rest stays at rest unless acted upon4. B.B.C. (British Broadcasting Corporaton) — Noted Wild West showman5. J.F.K. (John Fitzgerald Kennedy) — Not with any serious purpose in mind6. D.D.S. (Doctor of dental surgery) — Means of classification of library books7. F.T.C. (Federal Trade Commission) — Something you're said to search with if you do a very careful search8. B.F.F. (Best friends forever) — Event for cinephiles in Germany9. L.C.D. (Light-emitting diode or lowest common denominator) — Rimsky-Korsakov opera whose French title translates as """"The Golden Cockerel""""10. N.B.C. (National Broadcasting Company) — Evening of December 24 Last week's challenge: This challenge came from listener Neville Fogarty of Newport News, Va. Think of a convenience introduced in the 19th century that is still around today. Its name has two words. Take the first three letters of the first word and the first letter of its second word, in order, to get a convenience introduced in the 21st century that serves a similar purpose. Their names are otherwise unrelated. What two conveniences are these?Answer: Yellow Pages --> Yelp Winner: Kevin Devine of Marlborough, Mass. Next week's challenge: The name of what well-known U.S. city, in 10 letters, contains only three different letters of the alphabet? Submit Your Answer If you know the answer to next week's challenge, submit it here. Listeners who submit correct answers win a chance to play the on-air puzzle. Important: Include a phone number where we can reach you Thursday, Dec. 28 at 3 p.m. ET." 10 N.B.C. (National Broadcasting Company) — Evening of December 24 night before Christmas 20171126 Every answer today is the name of a famous person whose last name is the name of a well-known U.S. city. 1 Only U.S. president to serve two nonconsecutive terms Oh. 20171126 Every answer today is the name of a famous person whose last name is the name of a well-known U.S. city. 2 Author of the Nancy Drew books [N.H.] Keene 20171126 Every answer today is the name of a famous person whose last name is the name of a well-known U.S. city. 3 "Singer with the No.1 hits """"I Will Always Love You"""" and """"Didn't We Almost Have It All""""" Tex. 20171126 Every answer today is the name of a famous person whose last name is the name of a well-known U.S. city. 4 "1960s soul singer with the No.1 hit """"(Sittin' On) The Dock of the Bay"""" [Pa.]" Redding 20171126 Every answer today is the name of a famous person whose last name is the name of a well-known U.S. city. 5 "Who had a hit with """"U Got the Look"""" with Prince?" Pa. 20171126 Every answer today is the name of a famous person whose last name is the name of a well-known U.S. city. 6 "Who sang """"Rocky Mountain High"""" and """"Thank God I'm a Country Boy”?" Colo. 20171126 Every answer today is the name of a famous person whose last name is the name of a well-known U.S. city. 7 Attorney general under Bill Clinton for all eight years Nev. 20171126 Every answer today is the name of a famous person whose last name is the name of a well-known U.S. city. 8 "Star of """"Malcolm X,"""" """"Fences"""" and """"The Equalizer""""" D.C. 20171126 Every answer today is the name of a famous person whose last name is the name of a well-known U.S. city. 9 American botanist who created the Shasta daisy and hundreds of other varieties of plants Cal. 20171126 Every answer today is the name of a famous person whose last name is the name of a well-known U.S. city. 10 1960s-70s singer with Dawn Fla. 20171126 Every answer today is the name of a famous person whose last name is the name of a well-known U.S. city. 11 "Singer with the 33-time platinum album """"Thriller""""" Miss. 20171126 Every answer today is the name of a famous person whose last name is the name of a well-known U.S. city. 12 "Singer with the 1981 hit """"Jessie's Girl""""" Ill., Mass. or Mo. 20171126 Every answer today is the name of a famous person whose last name is the name of a well-known U.S. city. 13 """""Guys and Dolls"""" role played on film by Frank Sinatra" Mich. 20171126 Every answer today is the name of a famous person whose last name is the name of a well-known U.S. city. 14 Silent film star who was married to Mary Pickford Alas. 20171126 Every answer today is the name of a famous person whose last name is the name of a well-known U.S. city. 1 Only U.S. president to serve two nonconsecutive terms [Oh.] Cleveland 20171126 Every answer today is the name of a famous person whose last name is the name of a well-known U.S. city. 2 Author of the Nancy Drew books [N.H.] Keene 20171126 Every answer today is the name of a famous person whose last name is the name of a well-known U.S. city. 3 "Singer with the No.1 hits """"I Will Always Love You"""" and """"Didn't We Almost Have It All"""" [Tex.]" Houston 20171126 Every answer today is the name of a famous person whose last name is the name of a well-known U.S. city. 4 "1960s soul singer with the No.1 hit """"(Sittin' On) The Dock of the Bay"""" [Pa.]" Redding 20171126 Every answer today is the name of a famous person whose last name is the name of a well-known U.S. city. 5 "Who had a hit with """"U Got the Look"""" with Prince [Pa.]" Easton 20171126 Every answer today is the name of a famous person whose last name is the name of a well-known U.S. city. 6 "Who sang """"Rocky Mountain High"""" and """"Thank God I'm a Country Boy"""" [Colo.]" Denver 20171126 Every answer today is the name of a famous person whose last name is the name of a well-known U.S. city. 7 Attorney general under Bill Clinton for all eight years [Nev.] Reno 20171126 Every answer today is the name of a famous person whose last name is the name of a well-known U.S. city. 8 "Star of """"Malcolm X,"""" """"Fences"""" and """"The Equalizer"""" [D.C.]" Washington 20171126 Every answer today is the name of a famous person whose last name is the name of a well-known U.S. city. 9 American botanist who created the Shasta daisy and hundreds of other varieties of plants [Cal.] Burbank 20171126 Every answer today is the name of a famous person whose last name is the name of a well-known U.S. city. 10 1960s-70s singer with Dawn [Fla.] Orlando 20171126 Every answer today is the name of a famous person whose last name is the name of a well-known U.S. city. 11 "Singer with the 33-times platinum album """"Thriller"""" [Miss.]" Jackson 20171126 Every answer today is the name of a famous person whose last name is the name of a well-known U.S. city. 12 "Singer with the 1981 hit """"Jessie's Girl"""" [Ill., Mass. or Mo.]" Springfield 20171126 Every answer today is the name of a famous person whose last name is the name of a well-known U.S. city. 13 """""Guys and Dolls"""" role played on film by Frank Sinatra [Mich.]" Detroit 20171126 Every answer today is the name of a famous person whose last name is the name of a well-known U.S. city. 14 Silent film star who was married to Mary Pickford [Alas.] Fairbanks 20171022 "The word senses start with the letter “s”, and there are two senses, sight and smell, that also start with an s. Will will name a category, and, for each, give two things in it that start with the first letter of the category. " 1 months May, March 20171022 The word senses start with the letter “s”, and there are two senses, sight and smell, that also start with an s. Will will name a category, and, for each, give two things in it that start with the first letter of the category. 2 presidents Polk, Pierce 20171022 The word senses start with the letter “s”, and there are two senses, sight and smell, that also start with an s. Will will name a category, and, for each, give two things in it that start with the first letter of the category. 4 boroughs in New York City Brooklyn, Bronx 20171022 The word senses start with the letter “s”, and there are two senses, sight and smell, that also start with an s. Will will name a category, and, for each, give two things in it that start with the first letter of the category. 5 parts of speech preposition, pronoun; not participle 20171022 The word senses start with the letter “s”, and there are two senses, sight and smell, that also start with an s. Will will name a category, and, for each, give two things in it that start with the first letter of the category. 6 Snow White's dwarves Sleepy, Sneezy 20171022 The word senses start with the letter “s”, and there are two senses, sight and smell, that also start with an s. Will will name a category, and, for each, give two things in it that start with the first letter of the category. 7 astronauts on the first moon landing Aldrin, Armstrong 20171022 The word senses start with the letter “s”, and there are two senses, sight and smell, that also start with an s. Will will name a category, and, for each, give two things in it that start with the first letter of the category. 8 Monopoly properties Mediterranean Avenue, Marvin Gardens 20171022 The word senses start with the letter “s”, and there are two senses, sight and smell, that also start with an s. Will will name a category, and, for each, give two things in it that start with the first letter of the category. 9 NATO members Norway, Netherlands 20171022 The word senses start with the letter “s”, and there are two senses, sight and smell, that also start with an s. Will will name a category, and, for each, give two things in it that start with the first letter of the category. 10 colleges in the Ivy League Columbia, Cornell 20171015 I'm going to give you clues to some 8-letter words. Each word contains a doubled letter. Drop that doubled letter, and the remaining letters in order will spell a 6-letter word that answers the second clue. 1 George Gallup, for one / something pinned on a wall pollster / poster 20171015 I'm going to give you clues to some 8-letter words. Each word contains a doubled letter. Drop that doubled letter, and the remaining letters in order will spell a 6-letter word that answers the second clue. 2 mussing, as feathers / court decision ruffling / ruling 20171015 I'm going to give you clues to some 8-letter words. Each word contains a doubled letter. Drop that doubled letter, and the remaining letters in order will spell a 6-letter word that answers the second clue. 3 [Fill in the blank:] ___ column (part of a car) / lace steering / string 20171015 I'm going to give you clues to some 8-letter words. Each word contains a doubled letter. Drop that doubled letter, and the remaining letters in order will spell a 6-letter word that answers the second clue. 4 buying and selling of goods / force to do something commerce / coerce 20171015 I'm going to give you clues to some 8-letter words. Each word contains a doubled letter. Drop that doubled letter, and the remaining letters in order will spell a 6-letter word that answers the second clue. 5 in a gloomy fashion / sudden disappointment dismally / dismay 20171015 I'm going to give you clues to some 8-letter words. Each word contains a doubled letter. Drop that doubled letter, and the remaining letters in order will spell a 6-letter word that answers the second clue. 6 eliding / activity at Vail skipping / skiing 20171015 I'm going to give you clues to some 8-letter words. Each word contains a doubled letter. Drop that doubled letter, and the remaining letters in order will spell a 6-letter word that answers the second clue. 9 one sharing living quarters / $79 a night, for example roommate / rate 20171015 I'm going to give you clues to some 8-letter words. Each word contains a doubled letter. Drop that doubled letter, and the remaining letters in order will spell a 6-letter word that answers the second clue. 10 doesn’t fail / soapy water succeeds / suds 20171001 Every answer today is a famous person whose first name is also the name of a well-known U.S. city. 1 Author of Jane Eyre North Carolina 20171001 Every answer today is a famous person whose first name is also the name of a well-known U.S. city. 2 *^ Academy Award-winning actress for Moonstruck Washington 20171001 Every answer today is a famous person whose first name is also the name of a well-known U.S. city. 3 Abstract expressionist famous for drip painting Mississippi 20171001 Every answer today is a famous person whose first name is also the name of a well-known U.S. city. 4 Fictional spy played by Mike Myers Texas 20171001 Every answer today is a famous person whose first name is also the name of a well-known U.S. city. 5 * English actor in The Lord of the Rings and The Hobbit trilogy Florida 20171001 Every answer today is a famous person whose first name is also the name of a well-known U.S. city. 6 * Actress who played Queen Elizabeth in The King's Speech Montana 20171001 Every answer today is a famous person whose first name is also the name of a well-known U.S. city. 7 Author who created Rip Van Winkle D.C. 20171001 Every answer today is a famous person whose first name is also the name of a well-known U.S. city. 9 # 2016 Libertarian candidate for president Indiana 20171001 Every answer today is a famous person whose first name is also the name of a well-known U.S. city. 10 # Villain in The Silence of the Lambs Missouri 20171001 Every answer today is a famous person whose first name is also the name of a well-known U.S. city. 11 # Senior senator from Massachusetts New Jersey 20171001 Every answer today is a famous person whose first name is also the name of a well-known U.S. city. 12 # Cartoonist who created Bloom County California 20171001 Every answer today is a famous person whose first name is also the name of a well-known U.S. city. 13 # Classic artist for The Saturday Evening Post Oklahoma 20171001 Every answer today is a famous person whose first name is also the name of a well-known U.S. city. 14 # Liberal pundit who hosts The Last Word on MSNBC Kansas 20171001 Every answer today is a famous person whose first name is also the name of a well-known U.S. city. 1 Author of Jane Eyre North Carolina 20171001 Every answer today is a famous person whose first name is also the name of a well-known U.S. city. 2 *^ Academy Award-winning actress for Moonstruck Washington 20171001 Every answer today is a famous person whose first name is also the name of a well-known U.S. city. 3 Abstract expressionist famous for drip painting Mississippi 20171001 Every answer today is a famous person whose first name is also the name of a well-known U.S. city. 4 Fictional spy played by Mike Myers Texas 20171001 Every answer today is a famous person whose first name is also the name of a well-known U.S. city. 5 * English actor in The Lord of the Rings and The Hobbit trilogy Florida 20171001 Every answer today is a famous person whose first name is also the name of a well-known U.S. city. 6 * Actress who played Queen Elizabeth in The King's Speech Montana 20171001 Every answer today is a famous person whose first name is also the name of a well-known U.S. city. 7 Author who created Rip Van Winkle D.C. 20171001 Every answer today is a famous person whose first name is also the name of a well-known U.S. city. 9 # 2016 Libertarian candidate for president Indiana 20171001 Every answer today is a famous person whose first name is also the name of a well-known U.S. city. 10 # Villain in The Silence of the Lambs Missouri 20171001 Every answer today is a famous person whose first name is also the name of a well-known U.S. city. 11 # Senior senator from Massachusetts New Jersey 20171001 Every answer today is a famous person whose first name is also the name of a well-known U.S. city. 12 # Cartoonist who created Bloom County California 20171001 Every answer today is a famous person whose first name is also the name of a well-known U.S. city. 13 # Classic artist for The Saturday Evening Post Oklahoma 20171001 Every answer today is a famous person whose first name is also the name of a well-known U.S. city. 14 # Liberal pundit who hosts The Last Word on MSNBC Kansas 20170917 "Every answer is a familiar three-word phrase that has """"and"""" in the middle. Each sentence contains two words that have homophones that will complete the phrase." 1 I found a beautiful reed on the river's right bank. read & write 20170917 "Every answer is a familiar three-word phrase that has """"and"""" in the middle. Each sentence contains two words that have homophones that will complete the phrase." 2 "Some friends of mine say """"hi's"""" in Lowe's housewares department." highs & lows 20170917 "Every answer is a familiar three-word phrase that has """"and"""" in the middle. Each sentence contains two words that have homophones that will complete the phrase." 3 "The musician composed a new hymn for the """"Ben Hur"""" soundtrack." him & her 20170917 "Every answer is a familiar three-word phrase that has """"and"""" in the middle. Each sentence contains two words that have homophones that will complete the phrase." 4 The woodworker won a chisel and awl in a lottery. one & all 20170917 "Every answer is a familiar three-word phrase that has """"and"""" in the middle. Each sentence contains two words that have homophones that will complete the phrase." 5 "A dozen tix to """"The Bourne Identity"""" cost a lot of bread." born & bred 20170917 "Every answer is a familiar three-word phrase that has """"and"""" in the middle. Each sentence contains two words that have homophones that will complete the phrase." 6 A vegetable shortage of peas and corn caused long queues at the supermarket. p's & q's 20170917 "Every answer is a familiar three-word phrase that has """"and"""" in the middle. Each sentence contains two words that have homophones that will complete the phrase." 7 The chef had parsley, sage, and thyme tied up in bundles. time & tide 20170917 "Every answer is a familiar three-word phrase that has """"and"""" in the middle. Each sentence contains two words that have homophones that will complete the phrase." 8 The field was mown after the grass had grown too high. moan & groan 20170917 "Every answer is a familiar three-word phrase that has """"and"""" in the middle. Each sentence contains two words that have homophones that will complete the phrase." 9 "Bret Harte was the sole author of """"The Luck of Roaring Camp.""""" Heart & Soul 20170917 "Every answer is a familiar three-word phrase that has """"and"""" in the middle. Each sentence contains two words that have homophones that will complete the phrase." 10 You can always reach me by cell phone. buy & sell 20170827 Each clue is a thing that belongs to two categories. Name something that's in both categories. 1 country + large bird Turkey 20170827 Each clue is a thing that belongs to two categories. Name something that's in both categories. 2 country + popular wedding gift China 20170827 Each clue is a thing that belongs to two categories. Name something that's in both categories. 3 university + color Brown, Auburn 20170827 Each clue is a thing that belongs to two categories. Name something that's in both categories. 4 insect + sport cricket 20170827 Each clue is a thing that belongs to two categories. Name something that's in both categories. 5 card game + construction project bridge 20170827 Each clue is a thing that belongs to two categories. Name something that's in both categories. 6 vice president + foreign money Pence 20170827 Each clue is a thing that belongs to two categories. Name something that's in both categories. 7 fruit + company on the Dow Index Apple 20170827 Each clue is a thing that belongs to two categories. Name something that's in both categories. 8 comic strip + snack food Peanuts 20170827 Each clue is a thing that belongs to two categories. Name something that's in both categories. 9 breed of dog + athlete in a particular Olympic sport Boxer 20170827 Each clue is a thing that belongs to two categories. Name something that's in both categories. 10 alcoholic drink + tool screwdriver 20170827 Each clue is a thing that belongs to two categories. Name something that's in both categories. 11 [2 answers] planet + bygone make of automobile Saturn, Mercury 20170723 Every answer today is a two-word phrase like READY RED, in which the first word is an adjective ending in -y, and the second word sounds like the first one without that -y. Ex. Mideast ruler who trembles --> SHAKY SHEIK1. Hirsute animal that hops2. Flying mammal that's nuts3. Male offspring who has an optimistic outlook4. Numerous fellows5. Nobleman who arrives prematurely6. Wacky singer Malik formerly of One Direction7. Vowels that are simple to draw8. Powerful little arachnid9. Very small prong on a fork 1 hirsute animal that hops hairy hare 20170723 Every answer today is a two-word phrase like READY RED, in which the first word is an adjective ending in -y, and the second word sounds like the first one without that -y. Ex. Mideast ruler who trembles --> SHAKY SHEIK1. Hirsute animal that hops2. Flying mammal that's nuts3. Male offspring who has an optimistic outlook4. Numerous fellows5. Nobleman who arrives prematurely6. Wacky singer Malik formerly of One Direction7. Vowels that are simple to draw8. Powerful little arachnid9. Very small prong on a fork 2 flying mammal that's nuts batty bat 20170723 Every answer today is a two-word phrase like READY RED, in which the first word is an adjective ending in -y, and the second word sounds like the first one without that -y. Ex. Mideast ruler who trembles --> SHAKY SHEIK1. Hirsute animal that hops2. Flying mammal that's nuts3. Male offspring who has an optimistic outlook4. Numerous fellows5. Nobleman who arrives prematurely6. Wacky singer Malik formerly of One Direction7. Vowels that are simple to draw8. Powerful little arachnid9. Very small prong on a fork 3 male offspring who has an optimistic outlook sunny son 20170723 Every answer today is a two-word phrase like READY RED, in which the first word is an adjective ending in -y, and the second word sounds like the first one without that -y. Ex. Mideast ruler who trembles --> SHAKY SHEIK1. Hirsute animal that hops2. Flying mammal that's nuts3. Male offspring who has an optimistic outlook4. Numerous fellows5. Nobleman who arrives prematurely6. Wacky singer Malik formerly of One Direction7. Vowels that are simple to draw8. Powerful little arachnid9. Very small prong on a fork 4 numerous fellows many men 20170723 Every answer today is a two-word phrase like READY RED, in which the first word is an adjective ending in -y, and the second word sounds like the first one without that -y. Ex. Mideast ruler who trembles --> SHAKY SHEIK1. Hirsute animal that hops2. Flying mammal that's nuts3. Male offspring who has an optimistic outlook4. Numerous fellows5. Nobleman who arrives prematurely6. Wacky singer Malik formerly of One Direction7. Vowels that are simple to draw8. Powerful little arachnid9. Very small prong on a fork 5 a nobleman who arrives prematurely early earl 20170723 Every answer today is a two-word phrase like READY RED, in which the first word is an adjective ending in -y, and the second word sounds like the first one without that -y. Ex. Mideast ruler who trembles --> SHAKY SHEIK1. Hirsute animal that hops2. Flying mammal that's nuts3. Male offspring who has an optimistic outlook4. Numerous fellows5. Nobleman who arrives prematurely6. Wacky singer Malik formerly of One Direction7. Vowels that are simple to draw8. Powerful little arachnid9. Very small prong on a fork 6 wacky singer Malik formerly of One Direction zany Zayn 20170723 Every answer today is a two-word phrase like READY RED, in which the first word is an adjective ending in -y, and the second word sounds like the first one without that -y. Ex. Mideast ruler who trembles --> SHAKY SHEIK1. Hirsute animal that hops2. Flying mammal that's nuts3. Male offspring who has an optimistic outlook4. Numerous fellows5. Nobleman who arrives prematurely6. Wacky singer Malik formerly of One Direction7. Vowels that are simple to draw8. Powerful little arachnid9. Very small prong on a fork 7 vowels that are simple to draw easy e’s 20170723 Every answer today is a two-word phrase like READY RED, in which the first word is an adjective ending in -y, and the second word sounds like the first one without that -y. Ex. Mideast ruler who trembles --> SHAKY SHEIK1. Hirsute animal that hops2. Flying mammal that's nuts3. Male offspring who has an optimistic outlook4. Numerous fellows5. Nobleman who arrives prematurely6. Wacky singer Malik formerly of One Direction7. Vowels that are simple to draw8. Powerful little arachnid9. Very small prong on a fork 8 a powerful little arachnid mighty mite 20170723 Every answer today is a two-word phrase like READY RED, in which the first word is an adjective ending in -y, and the second word sounds like the first one without that -y. Ex. Mideast ruler who trembles --> SHAKY SHEIK1. Hirsute animal that hops2. Flying mammal that's nuts3. Male offspring who has an optimistic outlook4. Numerous fellows5. Nobleman who arrives prematurely6. Wacky singer Malik formerly of One Direction7. Vowels that are simple to draw8. Powerful little arachnid9. Very small prong on a fork 9 a very small prong on a fork tiny tine 20170625 "I'm going to give you clues for two words. The first word starts with the letter """"A"""" and is accented on the second syllable. Reverse the order of the syllables and phonetically you'll get the second word. Ex. Similar / Classic German camera --> ALIKE, LEICA1. Standoffish / Exfoliating shower sponge2. Horrify / Comedian Poundstone 3. Ease, as fears / Princess in """"Star Wars"""" 4. State as fact / Brand of mattress5. Make receptive or aware / Fish in a Bumblebee can6. Each / Leaning Tower city7. Not inclined to do something / Fill in the blank: Vice ___8. Public fight / Goddess of love, in Norse myth9. Toward shelter, at sea / Wife of Jacob, in the Bible Last week's challenge: Think of a familiar two-word phrase starting with T and ending with S, in which the interior letters name part of the human body. Remove the first and last letters of that word, and what remains will name another part of the human body. What's the phrase, and what are the body parts?" 1 standoffish / exfoliating shower sponge aloof, loofa 20170625 "I'm going to give you clues for two words. The first word starts with the letter """"A"""" and is accented on the second syllable. Reverse the order of the syllables and phonetically you'll get the second word. Ex. Similar / Classic German camera --> ALIKE, LEICA1. Standoffish / Exfoliating shower sponge2. Horrify / Comedian Poundstone 3. Ease, as fears / Princess in """"Star Wars"""" 4. State as fact / Brand of mattress5. Make receptive or aware / Fish in a Bumblebee can6. Each / Leaning Tower city7. Not inclined to do something / Fill in the blank: Vice ___8. Public fight / Goddess of love, in Norse myth9. Toward shelter, at sea / Wife of Jacob, in the Bible Last week's challenge: Think of a familiar two-word phrase starting with T and ending with S, in which the interior letters name part of the human body. Remove the first and last letters of that word, and what remains will name another part of the human body. What's the phrase, and what are the body parts?" 2 horrify / comedian Poundstone appall, Paula 20170625 "I'm going to give you clues for two words. The first word starts with the letter """"A"""" and is accented on the second syllable. Reverse the order of the syllables and phonetically you'll get the second word. Ex. Similar / Classic German camera --> ALIKE, LEICA1. Standoffish / Exfoliating shower sponge2. Horrify / Comedian Poundstone 3. Ease, as fears / Princess in """"Star Wars"""" 4. State as fact / Brand of mattress5. Make receptive or aware / Fish in a Bumblebee can6. Each / Leaning Tower city7. Not inclined to do something / Fill in the blank: Vice ___8. Public fight / Goddess of love, in Norse myth9. Toward shelter, at sea / Wife of Jacob, in the Bible Last week's challenge: Think of a familiar two-word phrase starting with T and ending with S, in which the interior letters name part of the human body. Remove the first and last letters of that word, and what remains will name another part of the human body. What's the phrase, and what are the body parts?" 3 "ease, as fears / princess in """"Star Wars”" allay, Leia 20170625 "I'm going to give you clues for two words. The first word starts with the letter """"A"""" and is accented on the second syllable. Reverse the order of the syllables and phonetically you'll get the second word. Ex. Similar / Classic German camera --> ALIKE, LEICA1. Standoffish / Exfoliating shower sponge2. Horrify / Comedian Poundstone 3. Ease, as fears / Princess in """"Star Wars"""" 4. State as fact / Brand of mattress5. Make receptive or aware / Fish in a Bumblebee can6. Each / Leaning Tower city7. Not inclined to do something / Fill in the blank: Vice ___8. Public fight / Goddess of love, in Norse myth9. Toward shelter, at sea / Wife of Jacob, in the Bible Last week's challenge: Think of a familiar two-word phrase starting with T and ending with S, in which the interior letters name part of the human body. Remove the first and last letters of that word, and what remains will name another part of the human body. What's the phrase, and what are the body parts?" 5 make receptive or aware / fish in a Bumblebee can attune, tuna 20170625 "I'm going to give you clues for two words. The first word starts with the letter """"A"""" and is accented on the second syllable. Reverse the order of the syllables and phonetically you'll get the second word. Ex. Similar / Classic German camera --> ALIKE, LEICA1. Standoffish / Exfoliating shower sponge2. Horrify / Comedian Poundstone 3. Ease, as fears / Princess in """"Star Wars"""" 4. State as fact / Brand of mattress5. Make receptive or aware / Fish in a Bumblebee can6. Each / Leaning Tower city7. Not inclined to do something / Fill in the blank: Vice ___8. Public fight / Goddess of love, in Norse myth9. Toward shelter, at sea / Wife of Jacob, in the Bible Last week's challenge: Think of a familiar two-word phrase starting with T and ending with S, in which the interior letters name part of the human body. Remove the first and last letters of that word, and what remains will name another part of the human body. What's the phrase, and what are the body parts?" 6 each / leaning tower city apiece, Pisa 20170625 "I'm going to give you clues for two words. The first word starts with the letter """"A"""" and is accented on the second syllable. Reverse the order of the syllables and phonetically you'll get the second word. Ex. Similar / Classic German camera --> ALIKE, LEICA1. Standoffish / Exfoliating shower sponge2. Horrify / Comedian Poundstone 3. Ease, as fears / Princess in """"Star Wars"""" 4. State as fact / Brand of mattress5. Make receptive or aware / Fish in a Bumblebee can6. Each / Leaning Tower city7. Not inclined to do something / Fill in the blank: Vice ___8. Public fight / Goddess of love, in Norse myth9. Toward shelter, at sea / Wife of Jacob, in the Bible Last week's challenge: Think of a familiar two-word phrase starting with T and ending with S, in which the interior letters name part of the human body. Remove the first and last letters of that word, and what remains will name another part of the human body. What's the phrase, and what are the body parts?" 7 not inclined to do something / fill in the blank: vice ___ averse, versa 20170625 "I'm going to give you clues for two words. The first word starts with the letter """"A"""" and is accented on the second syllable. Reverse the order of the syllables and phonetically you'll get the second word. Ex. Similar / Classic German camera --> ALIKE, LEICA1. Standoffish / Exfoliating shower sponge2. Horrify / Comedian Poundstone 3. Ease, as fears / Princess in """"Star Wars"""" 4. State as fact / Brand of mattress5. Make receptive or aware / Fish in a Bumblebee can6. Each / Leaning Tower city7. Not inclined to do something / Fill in the blank: Vice ___8. Public fight / Goddess of love, in Norse myth9. Toward shelter, at sea / Wife of Jacob, in the Bible Last week's challenge: Think of a familiar two-word phrase starting with T and ending with S, in which the interior letters name part of the human body. Remove the first and last letters of that word, and what remains will name another part of the human body. What's the phrase, and what are the body parts?" 8 public fight / goddess of love, in Norse myth affray, Freya 20170625 "I'm going to give you clues for two words. The first word starts with the letter """"A"""" and is accented on the second syllable. Reverse the order of the syllables and phonetically you'll get the second word. Ex. Similar / Classic German camera --> ALIKE, LEICA1. Standoffish / Exfoliating shower sponge2. Horrify / Comedian Poundstone 3. Ease, as fears / Princess in """"Star Wars"""" 4. State as fact / Brand of mattress5. Make receptive or aware / Fish in a Bumblebee can6. Each / Leaning Tower city7. Not inclined to do something / Fill in the blank: Vice ___8. Public fight / Goddess of love, in Norse myth9. Toward shelter, at sea / Wife of Jacob, in the Bible Last week's challenge: Think of a familiar two-word phrase starting with T and ending with S, in which the interior letters name part of the human body. Remove the first and last letters of that word, and what remains will name another part of the human body. What's the phrase, and what are the body parts?" 9 evaluated / basket in jai alai assessed, cesta 20170625 "I'm going to give you clues for two words. The first word starts with the letter """"A"""" and is accented on the second syllable. Reverse the order of the syllables and phonetically you'll get the second word. Ex. Similar / Classic German camera --> ALIKE, LEICA1. Standoffish / Exfoliating shower sponge2. Horrify / Comedian Poundstone 3. Ease, as fears / Princess in """"Star Wars"""" 4. State as fact / Brand of mattress5. Make receptive or aware / Fish in a Bumblebee can6. Each / Leaning Tower city7. Not inclined to do something / Fill in the blank: Vice ___8. Public fight / Goddess of love, in Norse myth9. Toward shelter, at sea / Wife of Jacob, in the Bible Last week's challenge: Think of a familiar two-word phrase starting with T and ending with S, in which the interior letters name part of the human body. Remove the first and last letters of that word, and what remains will name another part of the human body. What's the phrase, and what are the body parts?" 10 toward shelter, at sea / wife of Jacob, in the Bible alee, Leah 20170604 Every answer is a familiar two-word phrase or name in which the first word starts PI- and the second word starts P-. Ex. Sty ---> PIG PEN 1 What Peter Piper picked a peck of PIckled Peppers 20170604 Every answer is a familiar two-word phrase or name in which the first word starts PI- and the second word starts P-. Ex. Sty ---> PIG PEN 2 What you press with your foot on a baby grand PIano Pedal 20170604 Every answer is a familiar two-word phrase or name in which the first word starts PI- and the second word starts P-. Ex. Sty ---> PIG PEN 3 Baseball team that plays at PNC Park PIttsburgh Pirates 20170604 Every answer is a familiar two-word phrase or name in which the first word starts PI- and the second word starts P-. Ex. Sty ---> PIG PEN 4 "Place in Colorado that completes the phrase ""___ or Bust""" PIke's Peak 20170604 Every answer is a familiar two-word phrase or name in which the first word starts PI- and the second word starts P-. Ex. Sty ---> PIG PEN 5 " Early Dickens novel, with ""The""" PIckwick Papers 20170604 Every answer is a familiar two-word phrase or name in which the first word starts PI- and the second word starts P-. Ex. Sty ---> PIG PEN 6 "Religious allegory by John Bunyan, with ""The""" PIlgrim's Progress 20170604 Every answer is a familiar two-word phrase or name in which the first word starts PI- and the second word starts P-. Ex. Sty ---> PIG PEN 7 "1963 comedy featuring Inspector Clouseau, with ""The""" PInk Panthe 20170604 Every answer is a familiar two-word phrase or name in which the first word starts PI- and the second word starts P-. Ex. Sty ---> PIG PEN 8 Practicing frugality PInching Pennies 20170604 Every answer is a familiar two-word phrase or name in which the first word starts PI- and the second word starts P-. Ex. Sty ---> PIG PEN 9 Person who takes lots of uppers and downers PIll Popper 20170604 Every answer is a familiar two-word phrase or name in which the first word starts PI- and the second word starts P-. Ex. Sty ---> PIG PEN 10 10.^ Main newspaper in St. Paul, Minnesota PIoneer Press 20170604 Every answer is a familiar two-word phrase or name in which the first word starts PI- and the second word starts P-. Ex. Sty ---> PIG PEN 11 Place to order a sausage or pepperoni pie PIzza Parlor 20170604 Every answer is a familiar two-word phrase or name in which the first word starts PI- and the second word starts P-. Ex. Sty ---> PIG PEN 12 "Something that might say """"Greetings from ..."""" " PIcture Postcard 20170604 Every answer is a familiar two-word phrase or name in which the first word starts PI- and the second word starts P-. Ex. Sty ---> PIG PEN 13 Another name for table tennis PIng Pong 20170604 Every answer is a familiar two-word phrase or name in which the first word starts PI- and the second word starts P-. Ex. Sty ---> PIG PEN 14 He led the rats out of Hamelin PIed Piper 20170528 I'm going to give you some 6-letter words. For each one insert two letters in the exact center to complete a familiar 8-letter word. Ex. INNATE --> INNOVATE 1. ARGENT2. VANISH3. CORRAL4. ROSARY5. PANAMA6. THOUGH7. CARNAL8. CRUDER 9. MANURE10. CHAISE11. ESTATE12. OFFING13. SUBTLE 1 argent argument 20170528 I'm going to give you some 6-letter words. For each one insert two letters in the exact center to complete a familiar 8-letter word. Ex. INNATE --> INNOVATE 1. ARGENT2. VANISH3. CORRAL4. ROSARY5. PANAMA6. THOUGH7. CARNAL8. CRUDER 9. MANURE10. CHAISE11. ESTATE12. OFFING13. SUBTLE 2 vanish vanquish 20170528 I'm going to give you some 6-letter words. For each one insert two letters in the exact center to complete a familiar 8-letter word. Ex. INNATE --> INNOVATE 1. ARGENT2. VANISH3. CORRAL4. ROSARY5. PANAMA6. THOUGH7. CARNAL8. CRUDER 9. MANURE10. CHAISE11. ESTATE12. OFFING13. SUBTLE 3 corral corporal 20170528 I'm going to give you some 6-letter words. For each one insert two letters in the exact center to complete a familiar 8-letter word. Ex. INNATE --> INNOVATE 1. ARGENT2. VANISH3. CORRAL4. ROSARY5. PANAMA6. THOUGH7. CARNAL8. CRUDER 9. MANURE10. CHAISE11. ESTATE12. OFFING13. SUBTLE 4 rosary rosemary 20170528 I'm going to give you some 6-letter words. For each one insert two letters in the exact center to complete a familiar 8-letter word. Ex. INNATE --> INNOVATE 1. ARGENT2. VANISH3. CORRAL4. ROSARY5. PANAMA6. THOUGH7. CARNAL8. CRUDER 9. MANURE10. CHAISE11. ESTATE12. OFFING13. SUBTLE 5 Panama panorama 20170528 I'm going to give you some 6-letter words. For each one insert two letters in the exact center to complete a familiar 8-letter word. Ex. INNATE --> INNOVATE 1. ARGENT2. VANISH3. CORRAL4. ROSARY5. PANAMA6. THOUGH7. CARNAL8. CRUDER 9. MANURE10. CHAISE11. ESTATE12. OFFING13. SUBTLE 6 though thorough 20170528 I'm going to give you some 6-letter words. For each one insert two letters in the exact center to complete a familiar 8-letter word. Ex. INNATE --> INNOVATE 1. ARGENT2. VANISH3. CORRAL4. ROSARY5. PANAMA6. THOUGH7. CARNAL8. CRUDER 9. MANURE10. CHAISE11. ESTATE12. OFFING13. SUBTLE 7 carnal cardinal 20170528 I'm going to give you some 6-letter words. For each one insert two letters in the exact center to complete a familiar 8-letter word. Ex. INNATE --> INNOVATE 1. ARGENT2. VANISH3. CORRAL4. ROSARY5. PANAMA6. THOUGH7. CARNAL8. CRUDER 9. MANURE10. CHAISE11. ESTATE12. OFFING13. SUBTLE 8 cruder crusader 20170528 I'm going to give you some 6-letter words. For each one insert two letters in the exact center to complete a familiar 8-letter word. Ex. INNATE --> INNOVATE 1. ARGENT2. VANISH3. CORRAL4. ROSARY5. PANAMA6. THOUGH7. CARNAL8. CRUDER 9. MANURE10. CHAISE11. ESTATE12. OFFING13. SUBTLE 9 manure manicure 20170528 I'm going to give you some 6-letter words. For each one insert two letters in the exact center to complete a familiar 8-letter word. Ex. INNATE --> INNOVATE 1. ARGENT2. VANISH3. CORRAL4. ROSARY5. PANAMA6. THOUGH7. CARNAL8. CRUDER 9. MANURE10. CHAISE11. ESTATE12. OFFING13. SUBTLE 10 chaise chastise 20170528 I'm going to give you some 6-letter words. For each one insert two letters in the exact center to complete a familiar 8-letter word. Ex. INNATE --> INNOVATE 1. ARGENT2. VANISH3. CORRAL4. ROSARY5. PANAMA6. THOUGH7. CARNAL8. CRUDER 9. MANURE10. CHAISE11. ESTATE12. OFFING13. SUBTLE 11 estate estimate 20170528 I'm going to give you some 6-letter words. For each one insert two letters in the exact center to complete a familiar 8-letter word. Ex. INNATE --> INNOVATE 1. ARGENT2. VANISH3. CORRAL4. ROSARY5. PANAMA6. THOUGH7. CARNAL8. CRUDER 9. MANURE10. CHAISE11. ESTATE12. OFFING13. SUBTLE 12 offing offering 20170528 I'm going to give you some 6-letter words. For each one insert two letters in the exact center to complete a familiar 8-letter word. Ex. INNATE --> INNOVATE 1. ARGENT2. VANISH3. CORRAL4. ROSARY5. PANAMA6. THOUGH7. CARNAL8. CRUDER 9. MANURE10. CHAISE11. ESTATE12. OFFING13. SUBTLE 13 subtle subtitle 20170507 Every answer today is a familiar two-word phrase or name with the initials T-A. Answer their clues. 1 Israel's second-largest city, after Jerusalem Tel Aviv 20170507 Every answer today is a familiar two-word phrase or name with the initials T-A. Answer their clues. 2 Part of a phonograph that holds the needle tone arm 20170507 Every answer today is a familiar two-word phrase or name with the initials T-A. Answer their clues. 3 Feat in which an ice skater turns 3 1/2 times while jumping triple axel 20170507 Every answer today is a familiar two-word phrase or name with the initials T-A. Answer their clues. 4 Carrier to Istanbul Turkish Airlines 20170507 Every answer today is a familiar two-word phrase or name with the initials T-A. Answer their clues. 5 Fighting group that Hector helped lead in Greek myth Trojan army 20170507 Every answer today is a familiar two-word phrase or name with the initials T-A. Answer their clues. 7 Area in the plains states known for twisters tornado alley 20170507 Every answer today is a familiar two-word phrase or name with the initials T-A. Answer their clues. 8 Actor formerly married to Roseanne Tom Arnold 20170507 Every answer today is a familiar two-word phrase or name with the initials T-A. Answer their clues. 9 A little over 24 hours from Monday morning Tuesday afternoon 20170416 "For each category, I'll name something in the category that closely follows the name of the category alphabetically. For example, """"Shakespeare Plays"""" and """"Tempest."""" You name the only other thing in the category that fits between these two things alphabetically. In the case of the example, you would say """"Taming of the Shrew."""" 1. Days of the Week, Monday2. Coins, dollar3. Beatles, John4. Nobel Prize categories, physics5. Poker hands, straight6. Provinces of Canada, Saskatchewan7. European countries, France8. General Mills cereals, Lucky Charms9. Planets, Uranus10. Signs of the zodiac, virgo11. Hosts of The Tonight Show, O'Brien12. Baseball positions, center field" 1 days of the week, Monday Friday 20170416 "For each category, I'll name something in the category that closely follows the name of the category alphabetically. For example, """"Shakespeare Plays"""" and """"Tempest."""" You name the only other thing in the category that fits between these two things alphabetically. In the case of the example, you would say """"Taming of the Shrew."""" 1. Days of the Week, Monday2. Coins, dollar3. Beatles, John4. Nobel Prize categories, physics5. Poker hands, straight6. Provinces of Canada, Saskatchewan7. European countries, France8. General Mills cereals, Lucky Charms9. Planets, Uranus10. Signs of the zodiac, virgo11. Hosts of The Tonight Show, O'Brien12. Baseball positions, center field" 2 Beatles, John George 20170416 "For each category, I'll name something in the category that closely follows the name of the category alphabetically. For example, """"Shakespeare Plays"""" and """"Tempest."""" You name the only other thing in the category that fits between these two things alphabetically. In the case of the example, you would say """"Taming of the Shrew."""" 1. Days of the Week, Monday2. Coins, dollar3. Beatles, John4. Nobel Prize categories, physics5. Poker hands, straight6. Provinces of Canada, Saskatchewan7. European countries, France8. General Mills cereals, Lucky Charms9. Planets, Uranus10. Signs of the zodiac, virgo11. Hosts of The Tonight Show, O'Brien12. Baseball positions, center field" 3 Nobel Prize categories, physics Peace 20170416 "For each category, I'll name something in the category that closely follows the name of the category alphabetically. For example, """"Shakespeare Plays"""" and """"Tempest."""" You name the only other thing in the category that fits between these two things alphabetically. In the case of the example, you would say """"Taming of the Shrew."""" 1. Days of the Week, Monday2. Coins, dollar3. Beatles, John4. Nobel Prize categories, physics5. Poker hands, straight6. Provinces of Canada, Saskatchewan7. European countries, France8. General Mills cereals, Lucky Charms9. Planets, Uranus10. Signs of the zodiac, virgo11. Hosts of The Tonight Show, O'Brien12. Baseball positions, center field" 4 provinces of Canada, Saskatchewan Quebec 20170416 "For each category, I'll name something in the category that closely follows the name of the category alphabetically. For example, """"Shakespeare Plays"""" and """"Tempest."""" You name the only other thing in the category that fits between these two things alphabetically. In the case of the example, you would say """"Taming of the Shrew."""" 1. Days of the Week, Monday2. Coins, dollar3. Beatles, John4. Nobel Prize categories, physics5. Poker hands, straight6. Provinces of Canada, Saskatchewan7. European countries, France8. General Mills cereals, Lucky Charms9. Planets, Uranus10. Signs of the zodiac, virgo11. Hosts of The Tonight Show, O'Brien12. Baseball positions, center field" 5 signs of the zodiac, Virgo Taurus 20170416 "For each category, I'll name something in the category that closely follows the name of the category alphabetically. For example, """"Shakespeare Plays"""" and """"Tempest."""" You name the only other thing in the category that fits between these two things alphabetically. In the case of the example, you would say """"Taming of the Shrew."""" 1. Days of the Week, Monday2. Coins, dollar3. Beatles, John4. Nobel Prize categories, physics5. Poker hands, straight6. Provinces of Canada, Saskatchewan7. European countries, France8. General Mills cereals, Lucky Charms9. Planets, Uranus10. Signs of the zodiac, virgo11. Hosts of The Tonight Show, O'Brien12. Baseball positions, center field" 6 baseball positions, center field catcher 20170326 Take two-word clues that might appear in a crossword. The first and last letters of each answer are the same as the first and last letters of the clue. 1 snow tool shovel 20170326 Take two-word clues that might appear in a crossword. The first and last letters of each answer are the same as the first and last letters of the clue. 2 autumn flower aster 20170326 Take two-word clues that might appear in a crossword. The first and last letters of each answer are the same as the first and last letters of the clue. 3 warm material wool 20170326 Take two-word clues that might appear in a crossword. The first and last letters of each answer are the same as the first and last letters of the clue. 5 Spanish mister señor 20170326 Take two-word clues that might appear in a crossword. The first and last letters of each answer are the same as the first and last letters of the clue. 6 near relative niece 20170326 Take two-word clues that might appear in a crossword. The first and last letters of each answer are the same as the first and last letters of the clue. 7 cable giant Comcast 20170326 Take two-word clues that might appear in a crossword. The first and last letters of each answer are the same as the first and last letters of the clue. 8 baby's shoe bootie 20170326 Take two-word clues that might appear in a crossword. The first and last letters of each answer are the same as the first and last letters of the clue. 9 chipping tool chisel 20170326 Take two-word clues that might appear in a crossword. The first and last letters of each answer are the same as the first and last letters of the clue. 10 Northeast state (two-word answer) New Hampshire 20170305 "I'm going to read you some sentences. Each sentence ends in two blanks. Put two 4-letter homophones in them to complete the sentence. Homophones, of course, are words that sound alike but are spelled differently. For example: The nautical supplies company that was going out of business had a big ____ ____. --> SAIL SALE 1. Business was strong for most of this month, and then for the last seven days we had a ____ ____. 2. Taxi drivers don't want to gouge passengers with their rates; all they ask for is a ____ ____. 3. Santa Claus started his thank you letter to Dasher, Dancer, Prancer, Vixen, etc.: """"____ ____."""" 4. The hotel manager in charge of the housekeeping staff examined all 47 beds the ____ ____. 5. To measure the opinion of the residents of Antarctica, you'd have to conduct a South ____ ____. 6. A vegetarians' convention is a place where people who shun ____ ____. 7. A survey of the top-selling laundry detergents found that Brand X outperformed Gain, did less well than Wisk, and ____ ____. 8. A circus grizzly not wearing his costume could be described as a ____ ____. 9. The farm that is raising chickens, geese and peacocks stinks from all the ____ ____. 10. The young singing sensation who lost his recording contract is now an ____ ____." 1 Business was strong for most of this month, but then for the last seven days we had a ____ ____. weak week 20170305 "I'm going to read you some sentences. Each sentence ends in two blanks. Put two 4-letter homophones in them to complete the sentence. Homophones, of course, are words that sound alike but are spelled differently. For example: The nautical supplies company that was going out of business had a big ____ ____. --> SAIL SALE 1. Business was strong for most of this month, and then for the last seven days we had a ____ ____. 2. Taxi drivers don't want to gouge passengers with their rates; all they ask for is a ____ ____. 3. Santa Claus started his thank you letter to Dasher, Dancer, Prancer, Vixen, etc.: """"____ ____."""" 4. The hotel manager in charge of the housekeeping staff examined all 47 beds the ____ ____. 5. To measure the opinion of the residents of Antarctica, you'd have to conduct a South ____ ____. 6. A vegetarians' convention is a place where people who shun ____ ____. 7. A survey of the top-selling laundry detergents found that Brand X outperformed Gain, did less well than Wisk, and ____ ____. 8. A circus grizzly not wearing his costume could be described as a ____ ____. 9. The farm that is raising chickens, geese and peacocks stinks from all the ____ ____. 10. The young singing sensation who lost his recording contract is now an ____ ____." 2 Taxi drivers don't want to gouge passengers with their rates; all they ask for is a ____ ____. fair fare 20170305 "I'm going to read you some sentences. Each sentence ends in two blanks. Put two 4-letter homophones in them to complete the sentence. Homophones, of course, are words that sound alike but are spelled differently. For example: The nautical supplies company that was going out of business had a big ____ ____. --> SAIL SALE 1. Business was strong for most of this month, and then for the last seven days we had a ____ ____. 2. Taxi drivers don't want to gouge passengers with their rates; all they ask for is a ____ ____. 3. Santa Claus started his thank you letter to Dasher, Dancer, Prancer, Vixen, etc.: """"____ ____."""" 4. The hotel manager in charge of the housekeeping staff examined all 47 beds the ____ ____. 5. To measure the opinion of the residents of Antarctica, you'd have to conduct a South ____ ____. 6. A vegetarians' convention is a place where people who shun ____ ____. 7. A survey of the top-selling laundry detergents found that Brand X outperformed Gain, did less well than Wisk, and ____ ____. 8. A circus grizzly not wearing his costume could be described as a ____ ____. 9. The farm that is raising chickens, geese and peacocks stinks from all the ____ ____. 10. The young singing sensation who lost his recording contract is now an ____ ____." 3 To measure the opinion of the residents of Antarctica, you'd have to conduct a South ____ ____. Pole poll 20170305 "I'm going to read you some sentences. Each sentence ends in two blanks. Put two 4-letter homophones in them to complete the sentence. Homophones, of course, are words that sound alike but are spelled differently. For example: The nautical supplies company that was going out of business had a big ____ ____. --> SAIL SALE 1. Business was strong for most of this month, and then for the last seven days we had a ____ ____. 2. Taxi drivers don't want to gouge passengers with their rates; all they ask for is a ____ ____. 3. Santa Claus started his thank you letter to Dasher, Dancer, Prancer, Vixen, etc.: """"____ ____."""" 4. The hotel manager in charge of the housekeeping staff examined all 47 beds the ____ ____. 5. To measure the opinion of the residents of Antarctica, you'd have to conduct a South ____ ____. 6. A vegetarians' convention is a place where people who shun ____ ____. 7. A survey of the top-selling laundry detergents found that Brand X outperformed Gain, did less well than Wisk, and ____ ____. 8. A circus grizzly not wearing his costume could be described as a ____ ____. 9. The farm that is raising chickens, geese and peacocks stinks from all the ____ ____. 10. The young singing sensation who lost his recording contract is now an ____ ____." 4 A vegetarians' convention is a place where people who shun ____ ____. meat meet 20170305 "I'm going to read you some sentences. Each sentence ends in two blanks. Put two 4-letter homophones in them to complete the sentence. Homophones, of course, are words that sound alike but are spelled differently. For example: The nautical supplies company that was going out of business had a big ____ ____. --> SAIL SALE 1. Business was strong for most of this month, and then for the last seven days we had a ____ ____. 2. Taxi drivers don't want to gouge passengers with their rates; all they ask for is a ____ ____. 3. Santa Claus started his thank you letter to Dasher, Dancer, Prancer, Vixen, etc.: """"____ ____."""" 4. The hotel manager in charge of the housekeeping staff examined all 47 beds the ____ ____. 5. To measure the opinion of the residents of Antarctica, you'd have to conduct a South ____ ____. 6. A vegetarians' convention is a place where people who shun ____ ____. 7. A survey of the top-selling laundry detergents found that Brand X outperformed Gain, did less well than Wisk, and ____ ____. 8. A circus grizzly not wearing his costume could be described as a ____ ____. 9. The farm that is raising chickens, geese and peacocks stinks from all the ____ ____. 10. The young singing sensation who lost his recording contract is now an ____ ____." 5 A survey of the top-selling laundry detergents found that Brand X outperformed Gain, did less well than Wisk, and ____ ____. tied Tide 20170305 "I'm going to read you some sentences. Each sentence ends in two blanks. Put two 4-letter homophones in them to complete the sentence. Homophones, of course, are words that sound alike but are spelled differently. For example: The nautical supplies company that was going out of business had a big ____ ____. --> SAIL SALE 1. Business was strong for most of this month, and then for the last seven days we had a ____ ____. 2. Taxi drivers don't want to gouge passengers with their rates; all they ask for is a ____ ____. 3. Santa Claus started his thank you letter to Dasher, Dancer, Prancer, Vixen, etc.: """"____ ____."""" 4. The hotel manager in charge of the housekeeping staff examined all 47 beds the ____ ____. 5. To measure the opinion of the residents of Antarctica, you'd have to conduct a South ____ ____. 6. A vegetarians' convention is a place where people who shun ____ ____. 7. A survey of the top-selling laundry detergents found that Brand X outperformed Gain, did less well than Wisk, and ____ ____. 8. A circus grizzly not wearing his costume could be described as a ____ ____. 9. The farm that is raising chickens, geese and peacocks stinks from all the ____ ____. 10. The young singing sensation who lost his recording contract is now an ____ ____." 7 The young singing sensation who lost his recording contract is now an ____ ____. idle idol 20170101 Every year around this time, I do a year-end news quiz — usually on names that sprang into the news during the previous 12 months. Since 2016 broke the mold in so many ways, I decided to break the mold with my year-end quiz. Here are some notable quotes from the previous 12 months. Who said them? 1 """""I could stand in the middle of Fifth Avenue and shoot somebody, and I wouldn't lose any voters.”" Donald Trump 20170101 Every year around this time, I do a year-end news quiz — usually on names that sprang into the news during the previous 12 months. Since 2016 broke the mold in so many ways, I decided to break the mold with my year-end quiz. Here are some notable quotes from the previous 12 months. Who said them? 3 """""I'm not the next Usain Bolt or Michael Phelps. I'm the first ____________."""" (The name that goes in the blank is the person who said this.)" Simone Biles 20170101 Every year around this time, I do a year-end news quiz — usually on names that sprang into the news during the previous 12 months. Since 2016 broke the mold in so many ways, I decided to break the mold with my year-end quiz. Here are some notable quotes from the previous 12 months. Who said them? 4 """""People are always, like 'Why did you get a monkey?' If you could get a monkey, well, you would get a *BLEEPING* monkey, too! Monkeys are awesome!"""" (Hint: popular singer)" Justin Bieber 20170101 Every year around this time, I do a year-end news quiz — usually on names that sprang into the news during the previous 12 months. Since 2016 broke the mold in so many ways, I decided to break the mold with my year-end quiz. Here are some notable quotes from the previous 12 months. Who said them? 5 "(Name the movie this is from:) """"We have hope. Rebellions are built on hope!”" Rogue One 20170101 Every year around this time, I do a year-end news quiz — usually on names that sprang into the news during the previous 12 months. Since 2016 broke the mold in so many ways, I decided to break the mold with my year-end quiz. Here are some notable quotes from the previous 12 months. Who said them? 6 "(For the last quote, I'll tell you who said it. You tell me what he was talking about:) """"We thought the game would be popular, but it obviously struck a nerve."""" — John Hanke, CEO of the company Niantic. [Pokémon Go]“Bonus” quotations, not given on the air:a. """"When they go low, we go high.” [First Lady Michelle Obama]b. """"And love is love is love is love is love is love is love is love, cannot be killed or swept aside.” [Hamilton’s Lin-Manuel Miranda, as he accepted this year’s Tony award for Best Score]c. """"Brexit means Brexit.” [Theresa May, Prime Minister of the United Kingdom]d. """"Ms. Eisenstein, one question.”" Supreme Court Associate Justice Clarence Thomas 20161225 The theme of today's puzzle is giving. I'm going to give you two words. You give each of them a letter — the same letter for each word — in order to complete a familiar two-word phrase. 1 singe fie single file 20161225 The theme of today's puzzle is giving. I'm going to give you two words. You give each of them a letter — the same letter for each word — in order to complete a familiar two-word phrase. 2 raft boar draft board 20161225 The theme of today's puzzle is giving. I'm going to give you two words. You give each of them a letter — the same letter for each word — in order to complete a familiar two-word phrase. 3 dive though drive through 20161225 The theme of today's puzzle is giving. I'm going to give you two words. You give each of them a letter — the same letter for each word — in order to complete a familiar two-word phrase. 4 pone boot phone booth 20161225 The theme of today's puzzle is giving. I'm going to give you two words. You give each of them a letter — the same letter for each word — in order to complete a familiar two-word phrase. 5 back bet black belt 20161225 The theme of today's puzzle is giving. I'm going to give you two words. You give each of them a letter — the same letter for each word — in order to complete a familiar two-word phrase. 8 spry pint spray paint 20160605 For each five-letter word, insert two letters between the first and second letters to make a familiar seven-letter word. 1 solar* sCHolar 20160605 For each five-letter word, insert two letters between the first and second letters to make a familiar seven-letter word. 2 march mONarch 20160605 For each five-letter word, insert two letters between the first and second letters to make a familiar seven-letter word. 3 gland gARland 20160605 For each five-letter word, insert two letters between the first and second letters to make a familiar seven-letter word. 4 spent sERpent 20160605 For each five-letter word, insert two letters between the first and second letters to make a familiar seven-letter word. 5 tight tONight 20160605 For each five-letter word, insert two letters between the first and second letters to make a familiar seven-letter word. 7 fight* fREight 20160605 For each five-letter word, insert two letters between the first and second letters to make a familiar seven-letter word. 8 olive* oUTlive 20160605 For each five-letter word, insert two letters between the first and second letters to make a familiar seven-letter word. 9 elude eXClude 20160605 For each five-letter word, insert two letters between the first and second letters to make a familiar seven-letter word. 10 feign fOReign 20160522 Every answer this week is the name of a newspaper comic strip or cartoon, past or present. Identify the funnies from their anagrams. 1 CHAT + Y Cathy 20160522 Every answer this week is the name of a newspaper comic strip or cartoon, past or present. Identify the funnies from their anagrams. 2 CYAN + N Nancy 20160522 Every answer this week is the name of a newspaper comic strip or cartoon, past or present. Identify the funnies from their anagrams. 3 CHAIR + E Archie 20160522 Every answer this week is the name of a newspaper comic strip or cartoon, past or present. Identify the funnies from their anagrams. 4 UNSEAT + P Peanuts 20160522 Every answer this week is the name of a newspaper comic strip or cartoon, past or present. Identify the funnies from their anagrams. 5 FRAGILE + D Garfield 20160522 Every answer this week is the name of a newspaper comic strip or cartoon, past or present. Identify the funnies from their anagrams. 6 LIBERAL + N Lil’ Abner 20160522 Every answer this week is the name of a newspaper comic strip or cartoon, past or present. Identify the funnies from their anagrams. 7 HEADFIRST + E The Far Side 20160515 Given two kinds of people or things, name something they both do — in three letters. 1 marathoner, woman's stocking run 20160515 Given two kinds of people or things, name something they both do — in three letters. 2 pugilist, worker in a shipping warehouse* box 20160515 Given two kinds of people or things, name something they both do — in three letters. 3 hip hop artist, spirit at a seance rap 20160515 Given two kinds of people or things, name something they both do — in three letters. 4 restaurant patron, boat with too many people in it* tip 20160515 Given two kinds of people or things, name something they both do — in three letters. 5 overfilled balloon, weasel pop 20160515 Given two kinds of people or things, name something they both do — in three letters. 7 seamstress, farmer in the spring sew/sow 20160515 Given two kinds of people or things, name something they both do — in three letters. 8 a plant that doesn't get enough sun, salon worker die/dye 20160424 "Every answer today is a familiar two-word phrase or name in which the first word ends in """"X"""" and the second starts with """"C"""" (as in XC, the Roman numeral for 90). A friend of mine recently turned 90, and I made this as a birthday gift." 1 Iowa’s fourth largest metropolis, named after a native American tribe Sioux City 20160424 "Every answer today is a familiar two-word phrase or name in which the first word ends in """"X"""" and the second starts with """"C"""" (as in XC, the Roman numeral for 90). A friend of mine recently turned 90, and I made this as a birthday gift." 2 a rude sound made by sticking the tongue out and blowing Bronx cheer 20160424 "Every answer today is a familiar two-word phrase or name in which the first word ends in """"X"""" and the second starts with """"C"""" (as in XC, the Roman numeral for 90). A friend of mine recently turned 90, and I made this as a birthday gift." 3 a General Mills product that has a silly wabbit spokes-animal Trix cereal 20160424 "Every answer today is a familiar two-word phrase or name in which the first word ends in """"X"""" and the second starts with """"C"""" (as in XC, the Roman numeral for 90). A friend of mine recently turned 90, and I made this as a birthday gift." 4 something with a wick wax candle 20160424 "Every answer today is a familiar two-word phrase or name in which the first word ends in """"X"""" and the second starts with """"C"""" (as in XC, the Roman numeral for 90). A friend of mine recently turned 90, and I made this as a birthday gift." 5 Greek or Russian religious institution Orthodox church 20160424 "Every answer today is a familiar two-word phrase or name in which the first word ends in """"X"""" and the second starts with """"C"""" (as in XC, the Roman numeral for 90). A friend of mine recently turned 90, and I made this as a birthday gift." 6 part of Virginia that abuts Washington, D.C. Fairfax County 20160424 "Every answer today is a familiar two-word phrase or name in which the first word ends in """"X"""" and the second starts with """"C"""" (as in XC, the Roman numeral for 90). A friend of mine recently turned 90, and I made this as a birthday gift." 7 facsimile of a letter or other paper document Xerox copy 20160424 "Every answer today is a familiar two-word phrase or name in which the first word ends in """"X"""" and the second starts with """"C"""" (as in XC, the Roman numeral for 90). A friend of mine recently turned 90, and I made this as a birthday gift." 8 where to catch the Simpsons or American Idol FOX channel 20160424 "Every answer today is a familiar two-word phrase or name in which the first word ends in """"X"""" and the second starts with """"C"""" (as in XC, the Roman numeral for 90). A friend of mine recently turned 90, and I made this as a birthday gift." 9 one to whom you send money for the government tax collector 20160417 Ignore the vowels in each word given. The consonants in each word are the same consonants in the same order as in the name of a well-known American city. Name the cities. 1 ToLD, 6 ToLeDo 20160417 Ignore the vowels in each word given. The consonants in each word are the same consonants in the same order as in the name of a well-known American city. Name the cities. 2 LiPS, 6, 2 words El Paso 20160417 Ignore the vowels in each word given. The consonants in each word are the same consonants in the same order as in the name of a well-known American city. Name the cities. 3 BLooDieR, 7 BouLDeR 20160417 Ignore the vowels in each word given. The consonants in each word are the same consonants in the same order as in the name of a well-known American city. Name the cities. 4 BaFFLe, 7 BuFFaLo 20160417 Ignore the vowels in each word given. The consonants in each word are the same consonants in the same order as in the name of a well-known American city. Name the cities. 7 DieT RiTe,7 DeTRoiT 20160417 Ignore the vowels in each word given. The consonants in each word are the same consonants in the same order as in the name of a well-known American city. Name the cities. 8 No CHaRGe, 9* ANCHoRaGe 20160403 Every answer consists of two words in the same category that rhyme. For example, two animals whose rhyming names start with B and H are BEAR and HARE. Each clue will give you the initial letters, the words' lengths and the category. Name the things. 1 H,L, kinds of meat (3,4) ham, lamb 20160403 Every answer consists of two words in the same category that rhyme. For example, two animals whose rhyming names start with B and H are BEAR and HARE. Each clue will give you the initial letters, the words' lengths and the category. Name the things. 2 T,P, means of transportation (5,5)* train, plane 20160403 Every answer consists of two words in the same category that rhyme. For example, two animals whose rhyming names start with B and H are BEAR and HARE. Each clue will give you the initial letters, the words' lengths and the category. Name the things. 3 A,B, state capitals (6,6)* Austin, Boston) 20160403 Every answer consists of two words in the same category that rhyme. For example, two animals whose rhyming names start with B and H are BEAR and HARE. Each clue will give you the initial letters, the words' lengths and the category. Name the things. 4 R,D, seas (3,4) Red, Dead 20160403 Every answer consists of two words in the same category that rhyme. For example, two animals whose rhyming names start with B and H are BEAR and HARE. Each clue will give you the initial letters, the words' lengths and the category. Name the things. 5 H,L, parts of the body you have two of (4,4 as plurals)* hips, lips 20160403 Every answer consists of two words in the same category that rhyme. For example, two animals whose rhyming names start with B and H are BEAR and HARE. Each clue will give you the initial letters, the words' lengths and the category. Name the things. 6 H,D, games (6,5) Hearts, darts 20160403 Every answer consists of two words in the same category that rhyme. For example, two animals whose rhyming names start with B and H are BEAR and HARE. Each clue will give you the initial letters, the words' lengths and the category. Name the things. 7 M,N,J, New York professional sports teams (4,4,4) Mets, Nets, Jets 20160327 Every answer is a five-letter word said twice, in two different meanings. Answer the clues to get the phrases. 1 a lamp that doesn’t weigh much light light 20160327 Every answer is a five-letter word said twice, in two different meanings. Answer the clues to get the phrases. 2 the “correct” one of the first ten amendments to the Constitution right right 20160327 Every answer is a five-letter word said twice, in two different meanings. Answer the clues to get the phrases. 3 one who studies the center of the eye pupil pupil 20160327 Every answer is a five-letter word said twice, in two different meanings. Answer the clues to get the phrases. 4 an icky group of 144 things gross gross 20160327 Every answer is a five-letter word said twice, in two different meanings. Answer the clues to get the phrases. 5 a person imposing better financial penalties finer finer 20160327 Every answer is a five-letter word said twice, in two different meanings. Answer the clues to get the phrases. 6 a horse carriage with lots of things about it that don’t really work buggy buggy 20160327 Every answer is a five-letter word said twice, in two different meanings. Answer the clues to get the phrases. 7 a group of directors at a saw mill board board 20160327 Every answer is a five-letter word said twice, in two different meanings. Answer the clues to get the phrases. 8 sound made when prisoner toast each other with their glasses clink clink 20160320 Take the category, then name something in it whose first two letters are the last two letters of the category's name. 1 BeatLE (spelled out) LEnnon 20160320 Take the category, then name something in it whose first two letters are the last two letters of the category's name. 2 Disney musicAL ALaddin 20160320 Take the category, then name something in it whose first two letters are the last two letters of the category's name. 3 country in AfriCA CAmeroon) 20160320 Take the category, then name something in it whose first two letters are the last two letters of the category's name. 4 make of auTO TOyota 20160320 Take the category, then name something in it whose first two letters are the last two letters of the category's name. 5 race horSE SEabiscuit, SEattle Slew or SEcretariat 20160320 Take the category, then name something in it whose first two letters are the last two letters of the category's name. 6 municipal officiAL ALderman 20160320 Take the category, then name something in it whose first two letters are the last two letters of the category's name. 7 Norse explorER ERic the Red 20160320 Take the category, then name something in it whose first two letters are the last two letters of the category's name. 8 bridGE GEorge Washington 20160320 Take the category, then name something in it whose first two letters are the last two letters of the category's name. 9 oceAN ANtarctic 20160320 Take the category, then name something in it whose first two letters are the last two letters of the category's name. 10 best pictuRE REbecca 20160320 Take the category, then name something in it whose first two letters are the last two letters of the category's name. 12 letter of the Greek alphabET [ETa]13. make of automobiLE [LExus]14. an ObaMA [MAlia]15. salad greEN ENdive 20160228 Every answer is the name of a movie that has won in one of the top four categories at the Oscars: best picture, best director, best actor or best actress. Every answer is a solid word or name. Identify the movies from their anagrams. 1 USA-made Amadeus 20160228 Every answer is the name of a movie that has won in one of the top four categories at the Oscars: best picture, best director, best actor or best actress. Every answer is a solid word or name. Identify the movies from their anagrams. 2 on a plot Platoon 20160228 Every answer is the name of a movie that has won in one of the top four categories at the Oscars: best picture, best director, best actor or best actress. Every answer is a solid word or name. Identify the movies from their anagrams. 3 in attic Titanic 20160228 Every answer is the name of a movie that has won in one of the top four categories at the Oscars: best picture, best director, best actor or best actress. Every answer is a solid word or name. Identify the movies from their anagrams. 4 not apt Patton 20160228 Every answer is the name of a movie that has won in one of the top four categories at the Oscars: best picture, best director, best actor or best actress. Every answer is a solid word or name. Identify the movies from their anagrams. 5 toe cap Capote 20160228 Every answer is the name of a movie that has won in one of the top four categories at the Oscars: best picture, best director, best actor or best actress. Every answer is a solid word or name. Identify the movies from their anagrams. 6 gold tiera Gladiator 20160221 Every answer is a familiar two-word phrase or name in which the first word ends in the letter -E, and the second word starts GO-. 1 "An outdoor game, also called ""Putt-Putt""" miniaturE GOlf 20160221 Every answer is a familiar two-word phrase or name in which the first word ends in the letter -E, and the second word starts GO-. 2 Odin or Thor NorsE GOd 20160221 Every answer is a familiar two-word phrase or name in which the first word ends in the letter -E, and the second word starts GO-. 3 a term in economics for  like cars or refrigerators, expected to last a long time* durablE GOods 20160221 Every answer is a familiar two-word phrase or name in which the first word ends in the letter -E, and the second word starts GO-. 4 jacking up the cost of things during an emergency* pricE GOuging 20160221 Every answer is a familiar two-word phrase or name in which the first word ends in the letter -E, and the second word starts GO-. 5 chitchat about rumors and such idlE GOssip 20160221 Every answer is a familiar two-word phrase or name in which the first word ends in the letter -E, and the second word starts GO-. 6 Howard Hughes' airplane SprucE GOose 20160221 Every answer is a familiar two-word phrase or name in which the first word ends in the letter -E, and the second word starts GO-. 7 to silently signal one's party from a distance wavE GOodbye 20160221 Every answer is a familiar two-word phrase or name in which the first word ends in the letter -E, and the second word starts GO-. 8 political entity between federal and local statE GOvernment 20160221 Every answer is a familiar two-word phrase or name in which the first word ends in the letter -E, and the second word starts GO-. 9 another name for the ibex, referencing the mountain range it inhabits AlpinE GOat 20160221 Every answer is a familiar two-word phrase or name in which the first word ends in the letter -E, and the second word starts GO-. 10 singer of the '60s, '70's and '80s, married to Steve Lawrence EydiE GOrme 20160221 Every answer is a familiar two-word phrase or name in which the first word ends in the letter -E, and the second word starts GO-. 11 Words of praise after someone does a fine job* [nicE GOing] (No idea why donE GOod 20160124 I'm going to give you a clue for a word that has two Os. Change both Os to Es to get the answer to the second clue. 1 a portal; a doe or buck door; deer 20160124 I'm going to give you a clue for a word that has two Os. Change both Os to Es to get the answer to the second clue. 2 part of the body between the neck and the waist; succinct torso; terse 20160124 I'm going to give you a clue for a word that has two Os. Change both Os to Es to get the answer to the second clue. 3 select; a dairy product choose; cheese 20160124 I'm going to give you a clue for a word that has two Os. Change both Os to Es to get the answer to the second clue. 4 chicken ______ soup; a sewing item noodle; needle 20160124 I'm going to give you a clue for a word that has two Os. Change both Os to Es to get the answer to the second clue. 5 member of the Church of Latter Day Saints; mythical male creatures that look like fish from the waist down Mormon; mermen 20160124 I'm going to give you a clue for a word that has two Os. Change both Os to Es to get the answer to the second clue. 6 one of the smallest countries in Europe; to threaten Monaco; menace 20160124 I'm going to give you a clue for a word that has two Os. Change both Os to Es to get the answer to the second clue. 7 sciences; some sad poems ologies; elegies 20160124 I'm going to give you a clue for a word that has two Os. Change both Os to Es to get the answer to the second clue. 8 advertiser; English poet “Edmund” who wrote The Faerie Queen sponsor; Spenser 20160117 I'm going to name a category. You name something in the category whose first two letters are the last two letters of the category's name. 1 colOR ORange 20160117 I'm going to name a category. You name something in the category whose first two letters are the last two letters of the category's name. 2 metAL ALuminum 20160117 I'm going to name a category. You name something in the category whose first two letters are the last two letters of the category's name. 3 staTE TExas (or TEnnessee) 20160117 I'm going to name a category. You name something in the category whose first two letters are the last two letters of the category's name. 4 state capitAL ALbany 20160117 I'm going to name a category. You name something in the category whose first two letters are the last two letters of the category's name. 5 shellfiSH SHrimp 20160117 I'm going to name a category. You name something in the category whose first two letters are the last two letters of the category's name. 6 Norse gOD ODin 20160117 I'm going to name a category. You name something in the category whose first two letters are the last two letters of the category's name. 7 wooly animal ALpaca 20160117 I'm going to name a category. You name something in the category whose first two letters are the last two letters of the category's name. 8 languaGE GErman 20160117 I'm going to name a category. You name something in the category whose first two letters are the last two letters of the category's name. 9 TV talent progrAM AMerican Idol 20160117 I'm going to name a category. You name something in the category whose first two letters are the last two letters of the category's name. 10 book of the BibLE LEviticus 20160117 I'm going to name a category. You name something in the category whose first two letters are the last two letters of the category's name. 11 breakfast cereAL ALl Bran (or ALphabits) 20160103 Each of the following sentences has two blanks. The first word has the letter O somewhere in it. Double the O to get the second word, which completes the sentence. 1 To get to an online article about the 31st U.S. president, you should ____________ your cursor over the name Herbert ____________. hover, Hoover 20160103 Each of the following sentences has two blanks. The first word has the letter O somewhere in it. Double the O to get the second word, which completes the sentence. 2 On the team's baseball ____________ was a player who perfectly imitated the crowing of a ____________. roster, rooster 20160103 Each of the following sentences has two blanks. The first word has the letter O somewhere in it. Double the O to get the second word, which completes the sentence. 3 To order craft stockings made in Indiana, I needed a ____________ who was a ____________. hosier, Hoosier 20160103 Each of the following sentences has two blanks. The first word has the letter O somewhere in it. Double the O to get the second word, which completes the sentence. 4 Our instructor on whaling would always ____________ the need for the ____________ to be very, very sharp. harp on, harpoon 20160103 Each of the following sentences has two blanks. The first word has the letter O somewhere in it. Double the O to get the second word, which completes the sentence. 5 Our instructor on whaling would always ____________ the need for the ____________ to be very, very sharp. [note: what goes in the first blank is two words] smother, smoother 20160103 Each of the following sentences has two blanks. The first word has the letter O somewhere in it. Double the O to get the second word, which completes the sentence. 6 John's wife stormed out of the disco after the ____________ dancer made ____________ eyes at her husband. go-go, goo goo 20151220 Every answer today is the name of a Christmas carol or song — mostly carols. Answer the questions. 1 Sounds like a member of King Arthur's Round Table who won't speak* Silent Night 20151220 Every answer today is the name of a Christmas carol or song — mostly carols. Answer the questions. 2 "Rhymes with """"Whittle Summer Toy""""" Little Drummer Boy 20151220 Every answer today is the name of a Christmas carol or song — mostly carols. Answer the questions. 3 "An anagram of """"Beverly Sills,"""" without the Y*" Silver Bells 20151220 Every answer today is the name of a Christmas carol or song — mostly carols. Answer the questions. 4 Sounds like former TV host Arsenio and his family Deck the Halls 20151220 Every answer today is the name of a Christmas carol or song — mostly carols. Answer the questions. 5 """""Joy to the World"""" contains the name of a newspaper; name another carol whose title names a newspaper*" Hark, the Herald Angels Sing 20151206 I'm going to give you some 5-letter words. For each one, change the middle letter to two new letters to get a familiar 6-letter word. 1 eaRly eaSIly 20151206 I'm going to give you some 5-letter words. For each one, change the middle letter to two new letters to get a familiar 6-letter word. 2 tuLip tuRNip 20151206 I'm going to give you some 5-letter words. For each one, change the middle letter to two new letters to get a familiar 6-letter word. 3 moUrn moDErn 20151206 I'm going to give you some 5-letter words. For each one, change the middle letter to two new letters to get a familiar 6-letter word. 4 brOth brEAth 20151206 I'm going to give you some 5-letter words. For each one, change the middle letter to two new letters to get a familiar 6-letter word. 5 fuTon fuSIon 20151206 I'm going to give you some 5-letter words. For each one, change the middle letter to two new letters to get a familiar 6-letter word. 5 fuTon fuSIon 20151206 I'm going to give you some 5-letter words. For each one, change the middle letter to two new letters to get a familiar 6-letter word. 6 deity* dePUty 20151206 I'm going to give you some 5-letter words. For each one, change the middle letter to two new letters to get a familiar 6-letter word. 6 deity* dePUty 20151206 I'm going to give you some 5-letter words. For each one, change the middle letter to two new letters to get a familiar 6-letter word. 7 paNda paGOda 20151206 I'm going to give you some 5-letter words. For each one, change the middle letter to two new letters to get a familiar 6-letter word. 7 paNda paGOda 20151206 I'm going to give you some 5-letter words. For each one, change the middle letter to two new letters to get a familiar 6-letter word. 8 slOth sleuth 20151206 I'm going to give you some 5-letter words. For each one, change the middle letter to two new letters to get a familiar 6-letter word. 8 slOth sleuth 20151206 I'm going to give you some 5-letter words. For each one, change the middle letter to two new letters to get a familiar 6-letter word. 9 visor* viCTor 20151206 I'm going to give you some 5-letter words. For each one, change the middle letter to two new letters to get a familiar 6-letter word. 9 visor* viCTor 20151122 Each sentence contains two words that have homophones that are opposites. For each sentence given, find the homophone opposites. 1 Singer Frankie VALLI couldn't wait to PEEK at the song lyrics valley, peak 20151122 Each sentence contains two words that have homophones that are opposites. For each sentence given, find the homophone opposites. 2 The worker pulled some branches of YEWS up to his WAIST use, waste 20151122 Each sentence contains two words that have homophones that are opposites. For each sentence given, find the homophone opposites. 3 The band's music was so loud I couldn't HEAR THEIR lyrics here, there 20151122 Each sentence contains two words that have homophones that are opposites. For each sentence given, find the homophone opposites. 4 At many stores now you can pay BY CELL phone buy, sell 20151018 For each of the following words and phrases, the consonants in it are the same consonants, in the same order, as those of a world capital. You name the capital. 1 uPRaiSe PaRiS 20151018 For each of the following words and phrases, the consonants in it are the same consonants, in the same order, as those of a world capital. You name the capital. 2 CRoCuS CaRaCaS 20151018 For each of the following words and phrases, the consonants in it are the same consonants, in the same order, as those of a world capital. You name the capital. 3 aBoRT BeiRuiT 20151018 For each of the following words and phrases, the consonants in it are the same consonants, in the same order, as those of a world capital. You name the capital. 4 THRoNe TeHRaN 20151018 For each of the following words and phrases, the consonants in it are the same consonants, in the same order, as those of a world capital. You name the capital. 5 uNRoBe NaiRoBi---Rachel got this one] 20151018 For each of the following words and phrases, the consonants in it are the same consonants, in the same order, as those of a world capital. You name the capital. 6 BiGoT BoGoTa 20151018 For each of the following words and phrases, the consonants in it are the same consonants, in the same order, as those of a world capital. You name the capital. 7 LaGeRS aLGieRS 20151018 For each of the following words and phrases, the consonants in it are the same consonants, in the same order, as those of a world capital. You name the capital. 8 BeLeaGueReD BeLGRaDe 20151018 For each of the following words and phrases, the consonants in it are the same consonants, in the same order, as those of a world capital. You name the capital. 9 SeaL Norway 20151018 For each of the following words and phrases, the consonants in it are the same consonants, in the same order, as those of a world capital. You name the capital. 10 SeaL SeouL 20151004 For each six-letter word given, add two letters at the start to complete a common eight-letter word. The answer never involves adding a regular prefix like RE- or UN-. 1 OGRESS PROGRESS; Will let CONGRESS slip by 20151004 For each six-letter word given, add two letters at the start to complete a common eight-letter word. The answer never involves adding a regular prefix like RE- or UN-. 2 VERITY SEVERITY 20151004 For each six-letter word given, add two letters at the start to complete a common eight-letter word. The answer never involves adding a regular prefix like RE- or UN-. 3 SILICA BASILICA 20151004 For each six-letter word given, add two letters at the start to complete a common eight-letter word. The answer never involves adding a regular prefix like RE- or UN-. 4 RUGGED "SHRUGGED; Will called """"time"""" on this one" 20151004 For each six-letter word given, add two letters at the start to complete a common eight-letter word. The answer never involves adding a regular prefix like RE- or UN-. 5 ANGLER WRANGLER 20151004 For each six-letter word given, add two letters at the start to complete a common eight-letter word. The answer never involves adding a regular prefix like RE- or UN-. 6 RESIDE FIRESIDE 20151004 For each six-letter word given, add two letters at the start to complete a common eight-letter word. The answer never involves adding a regular prefix like RE- or UN-. 7 PETITE APPETITE 20151004 For each six-letter word given, add two letters at the start to complete a common eight-letter word. The answer never involves adding a regular prefix like RE- or UN-. 8 RENTAL PARENTAL 20150920 Every answer this week is the last name of a late-night TV host, past or present. Identify the hosts from their anagrams. 1 ONE + L Jay LENO 20150920 Every answer this week is the last name of a late-night TV host, past or present. Identify the hosts from their anagrams. 2 RAP + A Jack PAAR 20150920 Every answer this week is the last name of a late-night TV host, past or present. Identify the hosts from their anagrams. 3 LEAN + L Steve ALLEN 20150920 Every answer this week is the last name of a late-night TV host, past or present. Identify the hosts from their anagrams. 4 ACORN + S Johnny CARSON 20150920 Every answer this week is the last name of a late-night TV host, past or present. Identify the hosts from their anagrams. 5 LLANO + F Jimmy FALLON 20150920 Every answer this week is the last name of a late-night TV host, past or present. Identify the hosts from their anagrams. 6 NERDY + S Tom SNYDER 20150920 Every answer this week is the last name of a late-night TV host, past or present. Identify the hosts from their anagrams. 7 RISER + V Joan RIVERS 20150920 Every answer this week is the last name of a late-night TV host, past or present. Identify the hosts from their anagrams. 8 ROBIN + E Conan O'BRIEN 20150920 Every answer this week is the last name of a late-night TV host, past or present. Identify the hosts from their anagrams. 9 LECTOR + B Stephen COLBERT 20150920 Every answer this week is the last name of a late-night TV host, past or present. Identify the hosts from their anagrams. 10 SURGEON + F Craig FERGUSON 20150920 Every answer this week is the last name of a late-night TV host, past or present. Identify the hosts from their anagrams. 11 LAMENTER + T David LETTERMAN 20150823 "You'll be given a five-letter word and a six-letter word. Rearrange the letters of the five-letter word to get a synonym of the six-letter one. For example, given """"carve"""" and """"desire,"""" you would say """"crave.""""" 1 south; scream shout 20150823 "You'll be given a five-letter word and a six-letter word. Rearrange the letters of the five-letter word to get a synonym of the six-letter one. For example, given """"carve"""" and """"desire,"""" you would say """"crave.""""" 2 march; amulet charm 20150823 "You'll be given a five-letter word and a six-letter word. Rearrange the letters of the five-letter word to get a synonym of the six-letter one. For example, given """"carve"""" and """"desire,"""" you would say """"crave.""""" 3 peach; frugal cheap 20150823 "You'll be given a five-letter word and a six-letter word. Rearrange the letters of the five-letter word to get a synonym of the six-letter one. For example, given """"carve"""" and """"desire,"""" you would say """"crave.""""" 4 phase; figure shape 20150823 "You'll be given a five-letter word and a six-letter word. Rearrange the letters of the five-letter word to get a synonym of the six-letter one. For example, given """"carve"""" and """"desire,"""" you would say """"crave.""""" 5 serve; poetry verse 20150823 "You'll be given a five-letter word and a six-letter word. Rearrange the letters of the five-letter word to get a synonym of the six-letter one. For example, given """"carve"""" and """"desire,"""" you would say """"crave.""""" 6 earns; entrap snare 20150816 Every answer this week is the name of a state. For all the words given, ignore the vowels in them. The consonants in them are the same consonants, in the same order, as in the states. 1 FLaReD FLoRiDa 20150816 Every answer this week is the name of a state. For all the words given, ignore the vowels in them. The consonants in them are the same consonants, in the same order, as in the states. 2 KiNeSiS KaNSaS 20150816 Every answer this week is the name of a state. For all the words given, ignore the vowels in them. The consonants in them are the same consonants, in the same order, as in the states. 3 GaRaGe GeoRGia 20150816 Every answer this week is the name of a state. For all the words given, ignore the vowels in them. The consonants in them are the same consonants, in the same order, as in the states. 4 iNVaDe NeVaDa 20150816 Every answer this week is the name of a state. For all the words given, ignore the vowels in them. The consonants in them are the same consonants, in the same order, as in the states. 5 CLeaReD CoLoRaDo 20150816 Every answer this week is the name of a state. For all the words given, ignore the vowels in them. The consonants in them are the same consonants, in the same order, as in the states. 6 aMaSSeR MiSSouRi 20150816 Every answer this week is the name of a state. For all the words given, ignore the vowels in them. The consonants in them are the same consonants, in the same order, as in the states. 7 LLaNoS iLLiNoiS 20150816 Every answer this week is the name of a state. For all the words given, ignore the vowels in them. The consonants in them are the same consonants, in the same order, as in the states. 8 uNBRiSK NeBRaSKa 20150816 Every answer this week is the name of a state. For all the words given, ignore the vowels in them. The consonants in them are the same consonants, in the same order, as in the states. 9 eLiSioN LouiSiaNa 20150816 Every answer this week is the name of a state. For all the words given, ignore the vowels in them. The consonants in them are the same consonants, in the same order, as in the states. 10 oiL BooM aLaBaMa 20150816 Every answer this week is the name of a state. For all the words given, ignore the vowels in them. The consonants in them are the same consonants, in the same order, as in the states. 11 oVeR aGaiN ViRGiNia 20150802 Every answer this week is a six-letter word that contains two consecutive F's. Use each anagram of the other four letters to find the full six-letter word. 1 earl raffle 20150802 Every answer this week is a six-letter word that contains two consecutive F's. Use each anagram of the other four letters to find the full six-letter word. 2 tube buffet 20150802 Every answer this week is a six-letter word that contains two consecutive F's. Use each anagram of the other four letters to find the full six-letter word. 3 tore effort 20150802 Every answer this week is a six-letter word that contains two consecutive F's. Use each anagram of the other four letters to find the full six-letter word. 4 weal waffle 20150802 Every answer this week is a six-letter word that contains two consecutive F's. Use each anagram of the other four letters to find the full six-letter word. 5 mira affirm 20150802 Every answer this week is a six-letter word that contains two consecutive F's. Use each anagram of the other four letters to find the full six-letter word. 6 yips spiffy 20150802 Every answer this week is a six-letter word that contains two consecutive F's. Use each anagram of the other four letters to find the full six-letter word. 7 lure ruffle 20150802 Every answer this week is a six-letter word that contains two consecutive F's. Use each anagram of the other four letters to find the full six-letter word. 8 road afford 20150726 Every answer today is a made-up three-word phrase in which all three words rhyme ... and every word has two syllables. 1 T S H: a little rear end that glistens tiny shiny hiney 20150726 Every answer today is a made-up three-word phrase in which all three words rhyme ... and every word has two syllables. 2 J R D: a laughing play thing for a girl in North Carolina’s capital jolly Raleigh dolly 20150726 Every answer today is a made-up three-word phrase in which all three words rhyme ... and every word has two syllables. 3 B D L: superior missive to a person who owes money better debtor letter 20150726 Every answer today is a made-up three-word phrase in which all three words rhyme ... and every word has two syllables. 4 F N D: mindless drawing of a bit of pasta during the Middle Ages feudal noodle doodle 20150726 Every answer today is a made-up three-word phrase in which all three words rhyme ... and every word has two syllables. 5 N S V: more recent observer of a pipe-carrying waste newer sewer viewer 20150726 Every answer today is a made-up three-word phrase in which all three words rhyme ... and every word has two syllables. 6 P W T: appropriate thing to put on a Burger King hamburger proper Whopper topper 20150719 Today's puzzle has a bit of wordplay. Change one letter in each word provided to make two new words. The letter you change must be in the same position in each word of the pair. And the letter you change each of them to will be the same letter of the alphabet. 1 accent, strike accePt, striPe 20150719 Today's puzzle has a bit of wordplay. Change one letter in each word provided to make two new words. The letter you change must be in the same position in each word of the pair. And the letter you change each of them to will be the same letter of the alphabet. 2 trough, waiter tHough, wHiter 20150719 Today's puzzle has a bit of wordplay. Change one letter in each word provided to make two new words. The letter you change must be in the same position in each word of the pair. And the letter you change each of them to will be the same letter of the alphabet. 3 coffer, raving coffeE, ravinE 20150719 Today's puzzle has a bit of wordplay. Change one letter in each word provided to make two new words. The letter you change must be in the same position in each word of the pair. And the letter you change each of them to will be the same letter of the alphabet. 4 gander, stance gEnder, sEance 20150719 Today's puzzle has a bit of wordplay. Change one letter in each word provided to make two new words. The letter you change must be in the same position in each word of the pair. And the letter you change each of them to will be the same letter of the alphabet. 5 simple, decode simIle, decIde 20150719 Today's puzzle has a bit of wordplay. Change one letter in each word provided to make two new words. The letter you change must be in the same position in each word of the pair. And the letter you change each of them to will be the same letter of the alphabet. 7 avenge, radios avenUe, radiUs 20150705 In each pair of clues, the answer to the first clue is a word that contains the consecutive letters A-R. Drop the A-R, and the remaining letters in order will form a word that answers the second clue. 1 practicing in the boxing ring; what follows winter sparing; spring 20150705 In each pair of clues, the answer to the first clue is a word that contains the consecutive letters A-R. Drop the A-R, and the remaining letters in order will form a word that answers the second clue. 2 not easy to see; a husband of an aunt unclear; uncle 20150705 In each pair of clues, the answer to the first clue is a word that contains the consecutive letters A-R. Drop the A-R, and the remaining letters in order will form a word that answers the second clue. 4 a list of specialized words and their definitions; having a smooth, shiny surface such as a photo glossary; glossy 20150705 In each pair of clues, the answer to the first clue is a word that contains the consecutive letters A-R. Drop the A-R, and the remaining letters in order will form a word that answers the second clue. 5 somewhat like a rectangle with four equal sides; to step on like a bug squarish; squish 20150628 "For every word provided (all starting with the letter """"W""""), give a proverb or saying that contains that word." 1 worm The early bird gets the worm. 20150628 "For every word provided (all starting with the letter """"W""""), give a proverb or saying that contains that word." 2 wheel The squeaky wheel gets the grease. 20150628 "For every word provided (all starting with the letter """"W""""), give a proverb or saying that contains that word." 3 waste Waste not want not. 20150628 "For every word provided (all starting with the letter """"W""""), give a proverb or saying that contains that word." 4 wait All things come to those who wait. Time waits for no man. 20150628 "For every word provided (all starting with the letter """"W""""), give a proverb or saying that contains that word." 5 wealthy Early to bed and early to rise makes a man healthy, wealthy and wise. 20150628 "For every word provided (all starting with the letter """"W""""), give a proverb or saying that contains that word." 6 weakest A chain is only as strong as its weakest link. 20150628 "For every word provided (all starting with the letter """"W""""), give a proverb or saying that contains that word." 7 work All work and no play makes Jack a dull boy. 20150628 "For every word provided (all starting with the letter """"W""""), give a proverb or saying that contains that word." 8 water Water, water, everywhere, nor any drop to drink. (David said “…but not a drop to drink.) You can lead a horse to water, but you can’t make it drink. 20150628 "For every word provided (all starting with the letter """"W""""), give a proverb or saying that contains that word." 9 will Where there’s a will, there’s a way. 20150628 "For every word provided (all starting with the letter """"W""""), give a proverb or saying that contains that word." 10 well All’s well that ends well. 20150621 "Today's puzzle is called """"Monkey Business."""" Every answer is a familiar two-word phrase starting with """"M"""" and """"B"""" — as in """"monkey business.""""" 1 spring break locale in Florida Miami Beach 20150621 "Today's puzzle is called """"Monkey Business."""" Every answer is a familiar two-word phrase starting with """"M"""" and """"B"""" — as in """"monkey business.""""" 2 dumpling in Jewish soup Matzoh Ball 20150621 "Today's puzzle is called """"Monkey Business."""" Every answer is a familiar two-word phrase starting with """"M"""" and """"B"""" — as in """"monkey business.""""" 3 Puccini opera set in Japan Madam Butterfly 20150621 "Today's puzzle is called """"Monkey Business."""" Every answer is a familiar two-word phrase starting with """"M"""" and """"B"""" — as in """"monkey business.""""" 4 something to sleep on that's stored in a wall Murphy Bed 20150621 "Today's puzzle is called """"Monkey Business."""" Every answer is a familiar two-word phrase starting with """"M"""" and """"B"""" — as in """"monkey business.""""" 5 something a Boy Scout earns Merit Badge 20150621 "Today's puzzle is called """"Monkey Business."""" Every answer is a familiar two-word phrase starting with """"M"""" and """"B"""" — as in """"monkey business.""""" 6 who's got a lovely daughter in a 1960s Herman's Hermits song Mrs Brown 20150621 "Today's puzzle is called """"Monkey Business."""" Every answer is a familiar two-word phrase starting with """"M"""" and """"B"""" — as in """"monkey business.""""" 7 Sea of Tranquility, for Apollo 11 Moon Base 20150621 "Today's puzzle is called """"Monkey Business."""" Every answer is a familiar two-word phrase starting with """"M"""" and """"B"""" — as in """"monkey business.""""" 8 tallest of the Alps Mont Blanc 20150607 Every answer today is a made-up two-word phrase, in which the two words rhyme. The initials of the two words will be provided, along with a one-word clue. Example: C S, Tennis ---> Court Sport 1 NL, moon Night Light 20150607 Every answer today is a made-up two-word phrase, in which the two words rhyme. The initials of the two words will be provided, along with a one-word clue. Example: C S, Tennis ---> Court Sport 2 BR, semi-trailer Big Rig 20150607 Every answer today is a made-up two-word phrase, in which the two words rhyme. The initials of the two words will be provided, along with a one-word clue. Example: C S, Tennis ---> Court Sport 3 NH, cold Not Hot 20150607 Every answer today is a made-up two-word phrase, in which the two words rhyme. The initials of the two words will be provided, along with a one-word clue. Example: C S, Tennis ---> Court Sport 4 TV, haiku Terse Verse 20150607 Every answer today is a made-up two-word phrase, in which the two words rhyme. The initials of the two words will be provided, along with a one-word clue. Example: C S, Tennis ---> Court Sport 5 HS, bow-wow Hound Sound 20150607 Every answer today is a made-up two-word phrase, in which the two words rhyme. The initials of the two words will be provided, along with a one-word clue. Example: C S, Tennis ---> Court Sport 6 RP, speedway Race Place 20150607 Every answer today is a made-up two-word phrase, in which the two words rhyme. The initials of the two words will be provided, along with a one-word clue. Example: C S, Tennis ---> Court Sport 7 SO, exclaim Shout Out 20150607 Every answer today is a made-up two-word phrase, in which the two words rhyme. The initials of the two words will be provided, along with a one-word clue. Example: C S, Tennis ---> Court Sport 8 PD, Pepto-Bismol Pink Drink 20150524 "Every answer today is the name of a famous, one-named singer like Madonna or Beyoncé. Identify each name from its anagram, to which one extra letter is added. The singers are a mix of past and present. For example, given """"ADS"""" plus """"E,"""" the answer is """"Sade.""""" 1 yen + a Enya 20150524 "Every answer today is the name of a famous, one-named singer like Madonna or Beyoncé. Identify each name from its anagram, to which one extra letter is added. The singers are a mix of past and present. For example, given """"ADS"""" plus """"E,"""" the answer is """"Sade.""""" 2 lead + e Adele 20150524 "Every answer today is the name of a famous, one-named singer like Madonna or Beyoncé. Identify each name from its anagram, to which one extra letter is added. The singers are a mix of past and present. For example, given """"ADS"""" plus """"E,"""" the answer is """"Sade.""""" 3 dare + k Drake 20150524 "Every answer today is the name of a famous, one-named singer like Madonna or Beyoncé. Identify each name from its anagram, to which one extra letter is added. The singers are a mix of past and present. For example, given """"ADS"""" plus """"E,"""" the answer is """"Sade.""""" 4 grief + e Fergie 20150524 "Every answer today is the name of a famous, one-named singer like Madonna or Beyoncé. Identify each name from its anagram, to which one extra letter is added. The singers are a mix of past and present. For example, given """"ADS"""" plus """"E,"""" the answer is """"Sade.""""" 5 caliber + e Liberace 20150503 "Each word provided is an anagram of something you might see in a kitchen. For example, """"skin"""" is an anagram of """"sink.""""" 1 votes stove 20150503 "Each word provided is an anagram of something you might see in a kitchen. For example, """"skin"""" is an anagram of """"sink.""""" 2 anger range 20150503 "Each word provided is an anagram of something you might see in a kitchen. For example, """"skin"""" is an anagram of """"sink.""""" 3 merit timer 20150503 "Each word provided is an anagram of something you might see in a kitchen. For example, """"skin"""" is an anagram of """"sink.""""" 4 trounce counter 20150503 "Each word provided is an anagram of something you might see in a kitchen. For example, """"skin"""" is an anagram of """"sink.""""" 5 snoop spoon 20150503 "Each word provided is an anagram of something you might see in a kitchen. For example, """"skin"""" is an anagram of """"sink.""""" 6 garret grater 20150503 "Each word provided is an anagram of something you might see in a kitchen. For example, """"skin"""" is an anagram of """"sink.""""" 7 terrains strainer 20150503 "Each word provided is an anagram of something you might see in a kitchen. For example, """"skin"""" is an anagram of """"sink.""""" 8 macroview microwave 20150426 Every answer is a familiar two-word phrase or name in which the first word starts with L-O and the second word starts with G. 1 a mirror looking glass 20150426 Every answer is a familiar two-word phrase or name in which the first word starts with L-O and the second word starts with G. 2 what you put a car into when you go up a steep hill low gear 20150426 Every answer is a familiar two-word phrase or name in which the first word starts with L-O and the second word starts with G. 3 a pistol with bullets in it loaded gun 20150426 Every answer is a familiar two-word phrase or name in which the first word starts with L-O and the second word starts with G. 4 what a mayor is head of local government 20150426 Every answer is a familiar two-word phrase or name in which the first word starts with L-O and the second word starts with G. 5 Mastermind or Clue logic game 20150426 Every answer is a familiar two-word phrase or name in which the first word starts with L-O and the second word starts with G. 6 California city whose name is Spanish for “the cats” Los Gatos 20150426 Every answer is a familiar two-word phrase or name in which the first word starts with L-O and the second word starts with G. 7 1920s-1930s Yankee who was known as the Iron Horse Lou Gehrig 20150426 Every answer is a familiar two-word phrase or name in which the first word starts with L-O and the second word starts with G. 8 term that Ernest Hemingway used for those who came of age during WWI Lost Generation 20150426 Every answer is a familiar two-word phrase or name in which the first word starts with L-O and the second word starts with G. 9 lost for more than a while long gone 20150426 Every answer is a familiar two-word phrase or name in which the first word starts with L-O and the second word starts with G. 10 surety when someone borrows money loan guarantee 20150426 Every answer is a familiar two-word phrase or name in which the first word starts with L-O and the second word starts with G. 11 Eros or Cupid love god 20100228 "This game involves two words. The challenge is to add an EE sound at the end of the first word to get the second word. Example: For """"A person with a bank account"""" and """"tasty"""" — the answer would be """"saver"""" and """"savory.""""" 1 "desire for food; where Budapest is [hunger; Hungary]2. the hot season' a recap [summer; summary]3. more """"icky""""'; a supermarket [grosser; grocery]4. stock market partner of Lynch; in a carefree way [Merrill; merrily]5. fellow; genre for Agatha Christie [mister; mystery]6. a talking bird; equivalence in value [parrot; parity]7. a city in Montana; a Miss America, for example [Butte; beauty]8. a place to sit; a topper on an ice cream sundae [chair; cherry]9. loose, as shoelaces; messy [untied; untidy]10. used a swizzle stick; rugged [stirred; sturdy]11. a stand with three legs; with hardly any effort" easel; easily 20100228 "This game involves two words. The challenge is to add an EE sound at the end of the first word to get the second word. Example: For """"A person with a bank account"""" and """"tasty"""" — the answer would be """"saver"""" and """"savory.""""" 3 "more """"icky""""'; a supermarket" grosser; grocery 20100228 "This game involves two words. The challenge is to add an EE sound at the end of the first word to get the second word. Example: For """"A person with a bank account"""" and """"tasty"""" — the answer would be """"saver"""" and """"savory.""""" 4 stock market partner of Lynch; in a carefree way Merrill; merrily 20100228 "This game involves two words. The challenge is to add an EE sound at the end of the first word to get the second word. Example: For """"A person with a bank account"""" and """"tasty"""" — the answer would be """"saver"""" and """"savory.""""" 5 fellow; genre for Agatha Christie mister; mystery 20100228 "This game involves two words. The challenge is to add an EE sound at the end of the first word to get the second word. Example: For """"A person with a bank account"""" and """"tasty"""" — the answer would be """"saver"""" and """"savory.""""" 6 a talking bird; equivalence in value parrot; parity 20100228 "This game involves two words. The challenge is to add an EE sound at the end of the first word to get the second word. Example: For """"A person with a bank account"""" and """"tasty"""" — the answer would be """"saver"""" and """"savory.""""" 7 a city in Montana; a Miss America, for example Butte; beauty 20100228 "This game involves two words. The challenge is to add an EE sound at the end of the first word to get the second word. Example: For """"A person with a bank account"""" and """"tasty"""" — the answer would be """"saver"""" and """"savory.""""" 8 a place to sit; a topper on an ice cream sundae chair; cherry 20100228 "This game involves two words. The challenge is to add an EE sound at the end of the first word to get the second word. Example: For """"A person with a bank account"""" and """"tasty"""" — the answer would be """"saver"""" and """"savory.""""" 9 loose, as shoelaces; messy untied; untidy 20100228 "This game involves two words. The challenge is to add an EE sound at the end of the first word to get the second word. Example: For """"A person with a bank account"""" and """"tasty"""" — the answer would be """"saver"""" and """"savory.""""" 10 used a swizzle stick; rugged stirred; sturdy 20100228 "This game involves two words. The challenge is to add an EE sound at the end of the first word to get the second word. Example: For """"A person with a bank account"""" and """"tasty"""" — the answer would be """"saver"""" and """"savory.""""" 11 a stand with three legs; with hardly any effort easel; easily 20100124 "Every answer is a familiar two-word phrase with the initials J.R. For example, if the clue is """"going for a spin in a stolen car,"""" the answer would be """"joy ride.""""" 1 pastry with preserves inside jelly roll 20100124 "Every answer is a familiar two-word phrase with the initials J.R. For example, if the clue is """"going for a spin in a stolen car,"""" the answer would be """"joy ride.""""" 2 what married taxpayers file with the IRS joint return 20100124 "Every answer is a familiar two-word phrase with the initials J.R. For example, if the clue is """"going for a spin in a stolen car,"""" the answer would be """"joy ride.""""" 3 a pirate flag Jolly Roger 20100124 "Every answer is a familiar two-word phrase with the initials J.R. For example, if the clue is """"going for a spin in a stolen car,"""" the answer would be """"joy ride.""""" 4 what an appellate court exercises over a lower court judicial review 20100124 "Every answer is a familiar two-word phrase with the initials J.R. For example, if the clue is """"going for a spin in a stolen car,"""" the answer would be """"joy ride.""""" 5 item used in double Dutch jump rope 20100124 "Every answer is a familiar two-word phrase with the initials J.R. For example, if the clue is """"going for a spin in a stolen car,"""" the answer would be """"joy ride.""""" 6 What is practiced by synagogue goers Jewish rituals or Jewish religion 20100124 "Every answer is a familiar two-word phrase with the initials J.R. For example, if the clue is """"going for a spin in a stolen car,"""" the answer would be """"joy ride.""""" 7 waterway forming part of Israel's border that goes to the Dead Sea Jordan River 20100124 "Every answer is a familiar two-word phrase with the initials J.R. For example, if the clue is """"going for a spin in a stolen car,"""" the answer would be """"joy ride.""""" 8 Elvis Presley film or the number one song from it Jailhouse Rock, 1957 20100124 "Every answer is a familiar two-word phrase with the initials J.R. For example, if the clue is """"going for a spin in a stolen car,"""" the answer would be """"joy ride.""""" 9 nickname for a SOuthern soldier during the Civil War Johnny Reb 20100124 "Every answer is a familiar two-word phrase with the initials J.R. For example, if the clue is """"going for a spin in a stolen car,"""" the answer would be """"joy ride.""""" 10 Place for deliberations in a trial jury room 20100124 "Every answer is a familiar two-word phrase with the initials J.R. For example, if the clue is """"going for a spin in a stolen car,"""" the answer would be """"joy ride.""""" 11 place where sushi is served Japanese restaurant 20100124 "Every answer is a familiar two-word phrase with the initials J.R. For example, if the clue is """"going for a spin in a stolen car,"""" the answer would be """"joy ride.""""" 12 Dave Brubek or Count Bassie disk jazz record 20100124 "Every answer is a familiar two-word phrase with the initials J.R. For example, if the clue is """"going for a spin in a stolen car,"""" the answer would be """"joy ride.""""" 13 what a crime of passion is often committed in jealous rage 20100124 "Every answer is a familiar two-word phrase with the initials J.R. For example, if the clue is """"going for a spin in a stolen car,"""" the answer would be """"joy ride.""""" 14 state license and two years' experience, for example job requirements 20100124 "Every answer is a familiar two-word phrase with the initials J.R. For example, if the clue is """"going for a spin in a stolen car,"""" the answer would be """"joy ride.""""" 15 follower of the third US president and a believer in agrarian democracy Jeffersonian Republican 20100124 "Every answer is a familiar two-word phrase with the initials J.R. For example, if the clue is """"going for a spin in a stolen car,"""" the answer would be """"joy ride.""""" 16 distinctive alcohol from the Caribbean used to make a Hurricane Jamaican Rum 20100124 "Every answer is a familiar two-word phrase with the initials J.R. For example, if the clue is """"going for a spin in a stolen car,"""" the answer would be """"joy ride.""""" 17 a very fast hare jackrabbit 20100124 "Every answer is a familiar two-word phrase with the initials J.R. For example, if the clue is """"going for a spin in a stolen car,"""" the answer would be """"joy ride.""""" 18 When Goldilocks tried the beds of the three bears, one bed was too hard, one was too soft, and third one was … just right 20091227 "Every answer is a familiar phrase in the form ____ to ____, as in """"bored to tears"""" or """"year to date."""" Each clue is a compound word or common two-word phrase in which the first part rhymes with the first word in the answer and the second part rhymes with the last word in the answer. For example, if the clue is """"rear gate,"""" the answer would be """"year to date.""""" 1 line drive nine to five 20091227 "Every answer is a familiar phrase in the form ____ to ____, as in """"bored to tears"""" or """"year to date."""" Each clue is a compound word or common two-word phrase in which the first part rhymes with the first word in the answer and the second part rhymes with the last word in the answer. For example, if the clue is """"rear gate,"""" the answer would be """"year to date.""""" 1 line drive nine to five 20091227 "Every answer is a familiar phrase in the form ____ to ____, as in """"bored to tears"""" or """"year to date."""" Each clue is a compound word or common two-word phrase in which the first part rhymes with the first word in the answer and the second part rhymes with the last word in the answer. For example, if the clue is """"rear gate,"""" the answer would be """"year to date.""""" 2 small farms call to arms 20091227 "Every answer is a familiar phrase in the form ____ to ____, as in """"bored to tears"""" or """"year to date."""" Each clue is a compound word or common two-word phrase in which the first part rhymes with the first word in the answer and the second part rhymes with the last word in the answer. For example, if the clue is """"rear gate,"""" the answer would be """"year to date.""""" 3 white sheet bite to eat 20091227 "Every answer is a familiar phrase in the form ____ to ____, as in """"bored to tears"""" or """"year to date."""" Each clue is a compound word or common two-word phrase in which the first part rhymes with the first word in the answer and the second part rhymes with the last word in the answer. For example, if the clue is """"rear gate,"""" the answer would be """"year to date.""""" 4 black pool back to school 20091227 "Every answer is a familiar phrase in the form ____ to ____, as in """"bored to tears"""" or """"year to date."""" Each clue is a compound word or common two-word phrase in which the first part rhymes with the first word in the answer and the second part rhymes with the last word in the answer. For example, if the clue is """"rear gate,"""" the answer would be """"year to date.""""" 5 headlights dead to rights 20091227 "Every answer is a familiar phrase in the form ____ to ____, as in """"bored to tears"""" or """"year to date."""" Each clue is a compound word or common two-word phrase in which the first part rhymes with the first word in the answer and the second part rhymes with the last word in the answer. For example, if the clue is """"rear gate,"""" the answer would be """"year to date.""""" 6 flip for ship-to-shore 20091227 "Every answer is a familiar phrase in the form ____ to ____, as in """"bored to tears"""" or """"year to date."""" Each clue is a compound word or common two-word phrase in which the first part rhymes with the first word in the answer and the second part rhymes with the last word in the answer. For example, if the clue is """"rear gate,"""" the answer would be """"year to date.""""" 8 hot curry not to worry 20091227 "Every answer is a familiar phrase in the form ____ to ____, as in """"bored to tears"""" or """"year to date."""" Each clue is a compound word or common two-word phrase in which the first part rhymes with the first word in the answer and the second part rhymes with the last word in the answer. For example, if the clue is """"rear gate,"""" the answer would be """"year to date.""""" 9 west hill dressed to kill 20091227 "Every answer is a familiar phrase in the form ____ to ____, as in """"bored to tears"""" or """"year to date."""" Each clue is a compound word or common two-word phrase in which the first part rhymes with the first word in the answer and the second part rhymes with the last word in the answer. For example, if the clue is """"rear gate,"""" the answer would be """"year to date.""""" 10 cricket side ticket to ride 20091024 "This puzzle is called """"Mix and Match."""" Every answer is a familiar two-word phrase in which the first word starts with M-I and the second word starts with M-A, as in """"Mix and Match."""" For example, for the clue """"a day before St. Patrick's Day,"""" you would say """"mid-March.""""" 1 "on coins, a ""D"" for Denver, for example" Minute Maid 20091024 "This puzzle is called """"Mix and Match."""" Every answer is a familiar two-word phrase in which the first word starts with M-I and the second word starts with M-A, as in """"Mix and Match."""" For example, for the clue """"a day before St. Patrick's Day,"""" you would say """"mid-March.""""" 2 a brand of orange juice mischief maker 20091024 "This puzzle is called """"Mix and Match."""" Every answer is a familiar two-word phrase in which the first word starts with M-I and the second word starts with M-A, as in """"Mix and Match."""" For example, for the clue """"a day before St. Patrick's Day,"""" you would say """"mid-March.""""" 3 Agatha Christie sleuth Miss Marple 20091024 "This puzzle is called """"Mix and Match."""" Every answer is a familiar two-word phrase in which the first word starts with M-I and the second word starts with M-A, as in """"Mix and Match."""" For example, for the clue """"a day before St. Patrick's Day,"""" you would say """"mid-March.""""" 4 a near-sighted cartoon character Mister Magoo 20091024 "This puzzle is called """"Mix and Match."""" Every answer is a familiar two-word phrase in which the first word starts with M-I and the second word starts with M-A, as in """"Mix and Match."""" For example, for the clue """"a day before St. Patrick's Day,"""" you would say """"mid-March.""""" 5 in the corporate hierarchy, those between the chief executives and lowest executives middle management 20091024 "This puzzle is called """"Mix and Match."""" Every answer is a familiar two-word phrase in which the first word starts with M-I and the second word starts with M-A, as in """"Mix and Match."""" For example, for the clue """"a day before St. Patrick's Day,"""" you would say """"mid-March.""""" 6 an apparatus at a dairy farm milking machine 20091024 "This puzzle is called """"Mix and Match."""" Every answer is a familiar two-word phrase in which the first word starts with M-I and the second word starts with M-A, as in """"Mix and Match."""" For example, for the clue """"a day before St. Patrick's Day,"""" you would say """"mid-March.""""" 7 union of two people from different races or religions mixed marriage 20091024 "This puzzle is called """"Mix and Match."""" Every answer is a familiar two-word phrase in which the first word starts with M-I and the second word starts with M-A, as in """"Mix and Match."""" For example, for the clue """"a day before St. Patrick's Day,"""" you would say """"mid-March.""""" 8 Montana Senator who was Senate majority leader from 1961-1977 Mike Mansfield 20091024 "This puzzle is called """"Mix and Match."""" Every answer is a familiar two-word phrase in which the first word starts with M-I and the second word starts with M-A, as in """"Mix and Match."""" For example, for the clue """"a day before St. Patrick's Day,"""" you would say """"mid-March.""""" 9 1950s and 1960s New York Yankee who hit 536 home runs Mickey Mantle 20091024 "This puzzle is called """"Mix and Match."""" Every answer is a familiar two-word phrase in which the first word starts with M-I and the second word starts with M-A, as in """"Mix and Match."""" For example, for the clue """"a day before St. Patrick's Day,"""" you would say """"mid-March.""""" 10 late service at a Catholic Church midnight Mass 20091024 "This puzzle is called """"Mix and Match."""" Every answer is a familiar two-word phrase in which the first word starts with M-I and the second word starts with M-A, as in """"Mix and Match."""" For example, for the clue """"a day before St. Patrick's Day,"""" you would say """"mid-March.""""" 11 on a road, there might be one every 5,280 feet mile marker 20091024 "This puzzle is called """"Mix and Match."""" Every answer is a familiar two-word phrase in which the first word starts with M-I and the second word starts with M-A, as in """"Mix and Match."""" For example, for the clue """"a day before St. Patrick's Day,"""" you would say """"mid-March.""""" 12 1992 Denzel Washington film set in the deep South Mississippi Masala 20091024 "This puzzle is called """"Mix and Match."""" Every answer is a familiar two-word phrase in which the first word starts with M-I and the second word starts with M-A, as in """"Mix and Match."""" For example, for the clue """"a day before St. Patrick's Day,"""" you would say """"mid-March.""""" 13 where the UN and Grand Central Station are located in New York City Midtown Manhattan 20091024 "This puzzle is called """"Mix and Match."""" Every answer is a familiar two-word phrase in which the first word starts with M-I and the second word starts with M-A, as in """"Mix and Match."""" For example, for the clue """"a day before St. Patrick's Day,"""" you would say """"mid-March.""""" 14 an imp or little devil mischief maker 20201129 "I'm going to give you three words starting with """"F."""" You give me another word that could follow each of my three words, and in each case it has to form a compound word OR a familiar two-word phrase." 1 fly, fifth, ferris (5 letters, starts with w) wheel 20201129 "I'm going to give you three words starting with """"F."""" You give me another word that could follow each of my three words, and in each case it has to form a compound word OR a familiar two-word phrase." 2 fresh, flood, fire (5 letters, starts with w) water (fresh water, floodwater, firewater) 20201129 "I'm going to give you three words starting with """"F."""" You give me another word that could follow each of my three words, and in each case it has to form a compound word OR a familiar two-word phrase." 3 fun, full, fraternity (5 letters, starts with h) house (funhouse, full house, fraternity house) 20201129 "I'm going to give you three words starting with """"F."""" You give me another word that could follow each of my three words, and in each case it has to form a compound word OR a familiar two-word phrase." 4 flash, focal, freezing (5 letters, starts with p) point (flashpoint, focal point, freezing point) 20201129 "I'm going to give you three words starting with """"F."""" You give me another word that could follow each of my three words, and in each case it has to form a compound word OR a familiar two-word phrase." 5 fuel, firing, finish (4 letters, starts with l) line (fuel line, firing line, finish line) 20201129 "I'm going to give you three words starting with """"F."""" You give me another word that could follow each of my three words, and in each case it has to form a compound word OR a familiar two-word phrase." 6 feed, flight, flea (3 letters, starts with b) bag (feedbag, flight bag, fleabag) 20201129 "I'm going to give you three words starting with """"F."""" You give me another word that could follow each of my three words, and in each case it has to form a compound word OR a familiar two-word phrase." 7 foul, foot, fur (4 letters, starts with b) ball (foul ball, football, furball) 20201129 "I'm going to give you three words starting with """"F."""" You give me another word that could follow each of my three words, and in each case it has to form a compound word OR a familiar two-word phrase." 8 fig, fir, family (4 letters, starts with t) tree (fig tree, fir tree, family tree) 20201129 "I'm going to give you three words starting with """"F."""" You give me another word that could follow each of my three words, and in each case it has to form a compound word OR a familiar two-word phrase." 9 fat, fair, fighting (6 letters, starts with c) chance (fat chance, fair chance, fighting chance) 20201129 "I'm going to give you three words starting with """"F."""" You give me another word that could follow each of my three words, and in each case it has to form a compound word OR a familiar two-word phrase." 10 fish, funnel, fruit (4 letters, starts with c) cake (fish cake, funnel cake, fruitcake) 20201129 "I'm going to give you three words starting with """"F."""" You give me another word that could follow each of my three words, and in each case it has to form a compound word OR a familiar two-word phrase." 11 first, free, farm (4 letters, starts with h) hand (firsthand, freehand, farmhand) 20091219 "Today's puzzle is called """"Shhhh."""" Every answer is a familiar two-word phrase in which the first word starts with S-H and the second word starts with H. For example, if the clue is """"restraining device with a diagonal strap across the chest,"""" the answer would be """"shoulder harness.""""" 1 something you usually do when you meet someone SHake Hands 20091219 "Today's puzzle is called """"Shhhh."""" Every answer is a familiar two-word phrase in which the first word starts with S-H and the second word starts with H. For example, if the clue is """"restraining device with a diagonal strap across the chest,"""" the answer would be """"shoulder harness.""""" 2 detective who debuted in A Study in Scarlet SHerlock Holmes 20091219 "Today's puzzle is called """"Shhhh."""" Every answer is a familiar two-word phrase in which the first word starts with S-H and the second word starts with H. For example, if the clue is """"restraining device with a diagonal strap across the chest,"""" the answer would be """"shoulder harness.""""" 3 what a crew cut results in SHort Hair 20091219 "Today's puzzle is called """"Shhhh."""" Every answer is a familiar two-word phrase in which the first word starts with S-H and the second word starts with H. For example, if the clue is """"restraining device with a diagonal strap across the chest,"""" the answer would be """"shoulder harness.""""" 4 where the water comes out above you when you're washing yourself SHower Head 20091219 "Today's puzzle is called """"Shhhh."""" Every answer is a familiar two-word phrase in which the first word starts with S-H and the second word starts with H. For example, if the clue is """"restraining device with a diagonal strap across the chest,"""" the answer would be """"shoulder harness.""""" 5 a person in charge of a flock Sheep Herder 20091219 "Today's puzzle is called """"Shhhh."""" Every answer is a familiar two-word phrase in which the first word starts with S-H and the second word starts with H. For example, if the clue is """"restraining device with a diagonal strap across the chest,"""" the answer would be """"shoulder harness.""""" 6 they might be 9 a.m. to 9 p.m. at a mall Shop[ping] Hours 20091219 "Today's puzzle is called """"Shhhh."""" Every answer is a familiar two-word phrase in which the first word starts with S-H and the second word starts with H. For example, if the clue is """"restraining device with a diagonal strap across the chest,"""" the answer would be """"shoulder harness.""""" 7 one who might go looking for great whites Shark Hunter 20091219 "Today's puzzle is called """"Shhhh."""" Every answer is a familiar two-word phrase in which the first word starts with S-H and the second word starts with H. For example, if the clue is """"restraining device with a diagonal strap across the chest,"""" the answer would be """"shoulder harness.""""" 8 medical facility overseen by an organization whose members wear a red fez Shriners Hospital 20091219 "Today's puzzle is called """"Shhhh."""" Every answer is a familiar two-word phrase in which the first word starts with S-H and the second word starts with H. For example, if the clue is """"restraining device with a diagonal strap across the chest,"""" the answer would be """"shoulder harness.""""" 9 he played George Jefferson on The Jeffersons SHerman Hemsley 20091219 "Today's puzzle is called """"Shhhh."""" Every answer is a familiar two-word phrase in which the first word starts with S-H and the second word starts with H. For example, if the clue is """"restraining device with a diagonal strap across the chest,"""" the answer would be """"shoulder harness.""""" 10 author of the 1976 best-selling Report on Female Sexuality Here Hite 20091219 "Today's puzzle is called """"Shhhh."""" Every answer is a familiar two-word phrase in which the first word starts with S-H and the second word starts with H. For example, if the clue is """"restraining device with a diagonal strap across the chest,"""" the answer would be """"shoulder harness.""""" 11 [to] play basketball, informally Shoot Hoops 20091219 "Today's puzzle is called """"Shhhh."""" Every answer is a familiar two-word phrase in which the first word starts with S-H and the second word starts with H. For example, if the clue is """"restraining device with a diagonal strap across the chest,"""" the answer would be """"shoulder harness.""""" 12 suburb of Cleveland, Ohio Shaker Heights 20091219 "Today's puzzle is called """"Shhhh."""" Every answer is a familiar two-word phrase in which the first word starts with S-H and the second word starts with H. For example, if the clue is """"restraining device with a diagonal strap across the chest,"""" the answer would be """"shoulder harness.""""" 13 tonsorial style of Kojak Shaved Head 20200216 "I'm going to give you some words starting with the letter """"L."""" For each one, give me another word starting with """"L"""" that will complete a common two-word phrase." 1 Three-letter answers: Lemon law 20200216 "I'm going to give you some words starting with the letter """"L."""" For each one, give me another word starting with """"L"""" that will complete a common two-word phrase." 2 Four-letter answers: Laundry list 20200216 "I'm going to give you some words starting with the letter """"L."""" For each one, give me another word starting with """"L"""" that will complete a common two-word phrase." 3 Five-letter answers: Lady love 20200216 "I'm going to give you some words starting with the letter """"L."""" For each one, give me another word starting with """"L"""" that will complete a common two-word phrase." 4 Six-letter answers: Lava lamp 20200216 "I'm going to give you some words starting with the letter """"L."""" For each one, give me another word starting with """"L"""" that will complete a common two-word phrase." 5 Seven-letter answers: Living legend 20200216 "I'm going to give you some words starting with the letter """"L."""" For each one, give me another word starting with """"L"""" that will complete a common two-word phrase." 6 Eight-letter answers: Liquor license 20200216 "I'm going to give you some words starting with the letter """"L."""" For each one, give me another word starting with """"L"""" that will complete a common two-word phrase." 7 Nine-letter answers: Lending library 20200216 "I'm going to give you some words starting with the letter """"L."""" For each one, give me another word starting with """"L"""" that will complete a common two-word phrase." 8 Ten-letter answers: Limited liability 20141207 "Think of the old saying: """"That means no, N-O!"""" Every answer today is a familiar two-word phrase or name with the initial letters N and O. Example: Any place that reports on current events: NEWS OUTLET." 1 Mardi Gras city New Orleans 20141207 "Think of the old saying: """"That means no, N-O!"""" Every answer today is a familiar two-word phrase or name with the initial letters N and O. Example: Any place that reports on current events: NEWS OUTLET." 2 a person who stays up late Night Owl 20141207 "Think of the old saying: """"That means no, N-O!"""" Every answer today is a familiar two-word phrase or name with the initial letters N and O. Example: Any place that reports on current events: NEWS OUTLET." 3 technical term for laughing gas Nitrous Oxide 20141207 "Think of the old saying: """"That means no, N-O!"""" Every answer today is a familiar two-word phrase or name with the initial letters N and O. Example: Any place that reports on current events: NEWS OUTLET." 4 the day after Halloween November One 20141207 "Think of the old saying: """"That means no, N-O!"""" Every answer today is a familiar two-word phrase or name with the initial letters N and O. Example: Any place that reports on current events: NEWS OUTLET." 5 standard starting time for offices and stores Nine O'clock 20141207 "Think of the old saying: """"That means no, N-O!"""" Every answer today is a familiar two-word phrase or name with the initial letters N and O. Example: Any place that reports on current events: NEWS OUTLET." 6 captain or admiral Naval Officer 20141207 "Think of the old saying: """"That means no, N-O!"""" Every answer today is a familiar two-word phrase or name with the initial letters N and O. Example: Any place that reports on current events: NEWS OUTLET." 7 citrus fruit, much grown in Florida and California Navel Orange 20141207 "Think of the old saying: """"That means no, N-O!"""" Every answer today is a familiar two-word phrase or name with the initial letters N and O. Example: Any place that reports on current events: NEWS OUTLET." 8 French term for the responsibility of the privileged to those less fortunate Noblesse Oblige 20141207 "Think of the old saying: """"That means no, N-O!"""" Every answer today is a familiar two-word phrase or name with the initial letters N and O. Example: Any place that reports on current events: NEWS OUTLET." 9 area bordering Hudson Bay Northern Ontario 20141207 "Think of the old saying: """"That means no, N-O!"""" Every answer today is a familiar two-word phrase or name with the initial letters N and O. Example: Any place that reports on current events: NEWS OUTLET." 10 female anchor of CBS This Morning Norah O'Donnell 20141207 "Think of the old saying: """"That means no, N-O!"""" Every answer today is a familiar two-word phrase or name with the initial letters N and O. Example: Any place that reports on current events: NEWS OUTLET." 11 third-largest city of the Sooner State Norman, Oklahoma 20141207 "Think of the old saying: """"That means no, N-O!"""" Every answer today is a familiar two-word phrase or name with the initial letters N and O. Example: Any place that reports on current events: NEWS OUTLET." 12 mined rock from which part of five-cent coins are made Nickel Ore 20141207 "Think of the old saying: """"That means no, N-O!"""" Every answer today is a familiar two-word phrase or name with the initial letters N and O. Example: Any place that reports on current events: NEWS OUTLET." 13 a local law that restricts loud parties Noise Ordinance 20150125 "Every answer today is a word starting with the letters A-R, which you will identify from its anagram. For example, given AR plus ROB, the answer would be """"arbor.""""" 1 ar + nog argon 20150125 "Every answer today is a word starting with the letters A-R, which you will identify from its anagram. For example, given AR plus ROB, the answer would be """"arbor.""""" 2 ar + rod ardor 20150125 "Every answer today is a word starting with the letters A-R, which you will identify from its anagram. For example, given AR plus ROB, the answer would be """"arbor.""""" 3 ar + acne arcane 20150125 "Every answer today is a word starting with the letters A-R, which you will identify from its anagram. For example, given AR plus ROB, the answer would be """"arbor.""""" 4 ar + tend ardent 20150125 "Every answer today is a word starting with the letters A-R, which you will identify from its anagram. For example, given AR plus ROB, the answer would be """"arbor.""""" 5 ar + tyre artery 20150125 "Every answer today is a word starting with the letters A-R, which you will identify from its anagram. For example, given AR plus ROB, the answer would be """"arbor.""""" 6 ar + tribe arbiter 20150125 "Every answer today is a word starting with the letters A-R, which you will identify from its anagram. For example, given AR plus ROB, the answer would be """"arbor.""""" 7 ar + lanes arsenal 20150125 "Every answer today is a word starting with the letters A-R, which you will identify from its anagram. For example, given AR plus ROB, the answer would be """"arbor.""""" 8 ar + since arsenic 20150125 "Every answer today is a word starting with the letters A-R, which you will identify from its anagram. For example, given AR plus ROB, the answer would be """"arbor.""""" 9 ar + toss in arsonist 20150125 "Every answer today is a word starting with the letters A-R, which you will identify from its anagram. For example, given AR plus ROB, the answer would be """"arbor.""""" 10 ar + old mail armadillo 20120318 "Name something in a given category where the last two letters of the category's name are the first two letters of your answer. For example, given """"U.S. state,"""" the answer would be either """"Texas"""" or """"Tennessee.""""" 1 state capitAL ALbany 20120318 "Name something in a given category where the last two letters of the category's name are the first two letters of your answer. For example, given """"U.S. state,"""" the answer would be either """"Texas"""" or """"Tennessee.""""" 2 colOR ORange 20120318 "Name something in a given category where the last two letters of the category's name are the first two letters of your answer. For example, given """"U.S. state,"""" the answer would be either """"Texas"""" or """"Tennessee.""""" 3 make of automobiLE LExus 20120318 "Name something in a given category where the last two letters of the category's name are the first two letters of your answer. For example, given """"U.S. state,"""" the answer would be either """"Texas"""" or """"Tennessee.""""" 4 metAL ALuminum 20120318 "Name something in a given category where the last two letters of the category's name are the first two letters of your answer. For example, given """"U.S. state,"""" the answer would be either """"Texas"""" or """"Tennessee.""""" 5 car rentAL ALamo 20120318 "Name something in a given category where the last two letters of the category's name are the first two letters of your answer. For example, given """"U.S. state,"""" the answer would be either """"Texas"""" or """"Tennessee.""""" 6 book of the BibLE LEviticus 20120318 "Name something in a given category where the last two letters of the category's name are the first two letters of your answer. For example, given """"U.S. state,"""" the answer would be either """"Texas"""" or """"Tennessee.""""" 7 English monarCH CHarles (I or II) 20120318 "Name something in a given category where the last two letters of the category's name are the first two letters of your answer. For example, given """"U.S. state,"""" the answer would be either """"Texas"""" or """"Tennessee.""""" 8 portrait artiST STewart 20120318 "Name something in a given category where the last two letters of the category's name are the first two letters of your answer. For example, given """"U.S. state,"""" the answer would be either """"Texas"""" or """"Tennessee.""""" 9 salad greEN ENdive 20120318 "Name something in a given category where the last two letters of the category's name are the first two letters of your answer. For example, given """"U.S. state,"""" the answer would be either """"Texas"""" or """"Tennessee.""""" 10 NCAA tournament-winning basketball colleGE Georgetown 20120318 "Name something in a given category where the last two letters of the category's name are the first two letters of your answer. For example, given """"U.S. state,"""" the answer would be either """"Texas"""" or """"Tennessee.""""" 11 woman in ShakespeaRE REagan 20120318 "Name something in a given category where the last two letters of the category's name are the first two letters of your answer. For example, given """"U.S. state,"""" the answer would be either """"Texas"""" or """"Tennessee.""""" 12 breakfast cereAL ALl-Bra 20100403 "Every answer today is a familiar two-word phrase or name with the initials """"E and """"S.""""" 1 boy who has earned at least 21 merit badges EAGLE SCOUT 20100403 "Every answer today is a familiar two-word phrase or name with the initials """"E and """"S.""""" 2 nickname for New York EMPIRE STATE 20100403 "Every answer today is a familiar two-word phrase or name with the initials """"E and """"S.""""" 3 what follows kindergarten ELEMENTARY SCHOOL 20100403 "Every answer today is a familiar two-word phrase or name with the initials """"E and """"S.""""" 4 the middle part of an algebraic statement EQUALS SIGN 20100403 "Every answer today is a familiar two-word phrase or name with the initials """"E and """"S.""""" 5 Venus EVENING STAR 20100403 "Every answer today is a familiar two-word phrase or name with the initials """"E and """"S.""""" 6 someone coming from abroad for a year of education EXCHANGE STUDENT 20100403 "Every answer today is a familiar two-word phrase or name with the initials """"E and """"S.""""" 7 some makeup EYE SHADOW 20100403 "Every answer today is a familiar two-word phrase or name with the initials """"E and """"S.""""" 8 familiar name for magnesium sulfate EPSOM SALT 20100403 "Every answer today is a familiar two-word phrase or name with the initials """"E and """"S.""""" 9 cabinet member who oversees gas, oil, and utilities ENERGY SECRETARY 20100403 "Every answer today is a familiar two-word phrase or name with the initials """"E and """"S.""""" 10 it can give you a leg up ELEVATOR SHOES 20100403 "Every answer today is a familiar two-word phrase or name with the initials """"E and """"S.""""" 12 host of a weekly TV variety show from the 1940s to 1970s ED SULLIVAN 20170820 The answer to each clue is a 6-letter word that rhymes with the last word: 1 time of year when birds start to sing spring 20170820 The answer to each clue is a 6-letter word that rhymes with the last word: 2 having glaring light bright 20170820 The answer to each clue is a 6-letter word that rhymes with the last word: 3 what follows sunday monday 20170820 The answer to each clue is a 6-letter word that rhymes with the last word: 4 what's raised in a mound ground 20170820 The answer to each clue is a 6-letter word that rhymes with the last word: 6 place where you might find a vassal castle 20170820 The answer to each clue is a 6-letter word that rhymes with the last word: 7 tool on a mechanic's bench wrench 20170820 The answer to each clue is a 6-letter word that rhymes with the last word: 8 what you can use to fill in a stencil pencil 20170820 The answer to each clue is a 6-letter word that rhymes with the last word: 9 bank feller teller 20170820 The answer to each clue is a 6-letter word that rhymes with the last word: 10 traveling theater group troupe 20170820 The answer to each clue is a 6-letter word that rhymes with the last word: 11 like the hair of a horse coarse 20170820 The answer to each clue is a 6-letter word that rhymes with the last word: 12 oops whoops 20170820 The answer to each clue is a 6-letter word that rhymes with the last word: 13 cover as in a cloud shroud 20170820 The answer to each clue is a 6-letter word that rhymes with the last word: 14 where many french films first ran cannes* 20100515 "Each answer is a familiar two-word phrase containing the consecutive letters G-E-L. For example, given the clue """"where a university student may study,"""" the answer would be a """"college library.""""" 1 Theater direction to an actor staGE Left 20100515 "Each answer is a familiar two-word phrase containing the consecutive letters G-E-L. For example, given the clue """"where a university student may study,"""" the answer would be a """"college library.""""" 2 Where a student might listen to tapes in French or Spanish languaGE Lab 20100515 "Each answer is a familiar two-word phrase containing the consecutive letters G-E-L. For example, given the clue """"where a university student may study,"""" the answer would be a """"college library.""""" 3 Company that makes money available to homebuyers at interest mortgaGE Lender 20100515 "Each answer is a familiar two-word phrase containing the consecutive letters G-E-L. For example, given the clue """"where a university student may study,"""" the answer would be a """"college library.""""" 4 What and editor and art director prepare for a magazine paGE Layout 20100515 "Each answer is a familiar two-word phrase containing the consecutive letters G-E-L. For example, given the clue """"where a university student may study,"""" the answer would be a """"college library.""""" 5 One who hangs out at a piano bar, say lounGE Lizard 20100515 "Each answer is a familiar two-word phrase containing the consecutive letters G-E-L. For example, given the clue """"where a university student may study,"""" the answer would be a """"college library.""""" 6 What people rent for household items they don't have room for storaGE Locker 20100515 "Each answer is a familiar two-word phrase containing the consecutive letters G-E-L. For example, given the clue """"where a university student may study,"""" the answer would be a """"college library.""""" 7 Bit of greenery used in coleslaw cabbaGE Leaf 20100515 "Each answer is a familiar two-word phrase containing the consecutive letters G-E-L. For example, given the clue """"where a university student may study,"""" the answer would be a """"college library.""""" 8 Illumination of the front of a horse-drawn vehicle carriaGE Lamp (or Lantern) 20100515 "Each answer is a familiar two-word phrase containing the consecutive letters G-E-L. For example, given the clue """"where a university student may study,"""" the answer would be a """"college library.""""" 9 Anti-theft device for a traveler luggaGE Lock 20070527 Every answer today is a familiar, two-word phrase with the letters R-O in the second and third positions of each of the words. That is, blank R-O something and then blank R-O something. 1 Big seller of fine men's suits Brooks Brothers 20070527 Every answer today is a familiar, two-word phrase with the letters R-O in the second and third positions of each of the words. That is, blank R-O something and then blank R-O something. 2 Metal for a balcony railing wrought iron 20070527 Every answer today is a familiar, two-word phrase with the letters R-O in the second and third positions of each of the words. That is, blank R-O something and then blank R-O something. 3 An oath that is not followed broken promise 20070527 Every answer today is a familiar, two-word phrase with the letters R-O in the second and third positions of each of the words. That is, blank R-O something and then blank R-O something. 4 A place where extremely high-speed vehicles are tested proving ground 20070527 Every answer today is a familiar, two-word phrase with the letters R-O in the second and third positions of each of the words. That is, blank R-O something and then blank R-O something. 5 Traveled here and there by car drove around 20070527 Every answer today is a familiar, two-word phrase with the letters R-O in the second and third positions of each of the words. That is, blank R-O something and then blank R-O something. 6 Where two companies advertise each other's products cross promotion 20070527 Every answer today is a familiar, two-word phrase with the letters R-O in the second and third positions of each of the words. That is, blank R-O something and then blank R-O something. 7 A fish from a babbling stream brown trout 20070527 Every answer today is a familiar, two-word phrase with the letters R-O in the second and third positions of each of the words. That is, blank R-O something and then blank R-O something. 8 A local unit of young Girl Scouts Brownie troop 20070527 Every answer today is a familiar, two-word phrase with the letters R-O in the second and third positions of each of the words. That is, blank R-O something and then blank R-O something. 9 Wearing away of soil, as from a stream's bank ground erosion 20070527 Every answer today is a familiar, two-word phrase with the letters R-O in the second and third positions of each of the words. That is, blank R-O something and then blank R-O something. 10 Refute (prove wrong) prove wrong 20070527 Every answer today is a familiar, two-word phrase with the letters R-O in the second and third positions of each of the words. That is, blank R-O something and then blank R-O something. 11 An old-fashioned cable car from a New York suburb Brooklyn trolley 20070527 Every answer today is a familiar, two-word phrase with the letters R-O in the second and third positions of each of the words. That is, blank R-O something and then blank R-O something. 12 A slangy order to remove one's pants drop trou 20120115 "Think of a word that can follow a given word to complete a familiar two-word phrase or name. The first two letters of your word must be the second and last letters, respectively, of the given word. For example, if given """"fallen,"""" the answer would be """"angel.""""" 1 European Union 20120115 "Think of a word that can follow a given word to complete a familiar two-word phrase or name. The first two letters of your word must be the second and last letters, respectively, of the given word. For example, if given """"fallen,"""" the answer would be """"angel.""""" 2 soap opera 20120115 "Think of a word that can follow a given word to complete a familiar two-word phrase or name. The first two letters of your word must be the second and last letters, respectively, of the given word. For example, if given """"fallen,"""" the answer would be """"angel.""""" 3 Second Edition 20120115 "Think of a word that can follow a given word to complete a familiar two-word phrase or name. The first two letters of your word must be the second and last letters, respectively, of the given word. For example, if given """"fallen,"""" the answer would be """"angel.""""" 4 casting agent 20120115 "Think of a word that can follow a given word to complete a familiar two-word phrase or name. The first two letters of your word must be the second and last letters, respectively, of the given word. For example, if given """"fallen,"""" the answer would be """"angel.""""" 5 free rein 20120115 "Think of a word that can follow a given word to complete a familiar two-word phrase or name. The first two letters of your word must be the second and last letters, respectively, of the given word. For example, if given """"fallen,"""" the answer would be """"angel.""""" 6 beady eyes 20120115 "Think of a word that can follow a given word to complete a familiar two-word phrase or name. The first two letters of your word must be the second and last letters, respectively, of the given word. For example, if given """"fallen,"""" the answer would be """"angel.""""" 7 Auburn University 20120115 "Think of a word that can follow a given word to complete a familiar two-word phrase or name. The first two letters of your word must be the second and last letters, respectively, of the given word. For example, if given """"fallen,"""" the answer would be """"angel.""""" 8 Chicago Hope 20120115 "Think of a word that can follow a given word to complete a familiar two-word phrase or name. The first two letters of your word must be the second and last letters, respectively, of the given word. For example, if given """"fallen,"""" the answer would be """"angel.""""" 9 Star Trek 20200105 Every answer is the brand name of a food or beverage that you might buy at the grocery store. 1 preserves consists of a word meaning “lollipops” around the letter m smucker’s 20200105 Every answer is the brand name of a food or beverage that you might buy at the grocery store. 2 soup conceals the word ogres in consecutive letters progresso 20200105 Every answer is the brand name of a food or beverage that you might buy at the grocery store. 3 breakfast cereal is an anagram of white sea wheaties 20200105 Every answer is the brand name of a food or beverage that you might buy at the grocery store. 4 beer completes the phrase “older ___” in a punny way budweiser 20200105 Every answer is the brand name of a food or beverage that you might buy at the grocery store. 5 juice also names a kind of car engine v-8 20200105 Every answer is the brand name of a food or beverage that you might buy at the grocery store. 6 beverage product hidden inside the phrase t-bone steak nestea 20121125 "Every answer consists of a made-up two-word phrase in which the first word starts with CH, and the second word is pronounced the same as the first except with an SH sound. (The spelling may or may not change.) For example, given the clue """"some Central African fish,"""" the answer would be """"Chad shad.""""" 1 inexpensive wooly animals cheap sheep 20121125 "Every answer consists of a made-up two-word phrase in which the first word starts with CH, and the second word is pronounced the same as the first except with an SH sound. (The spelling may or may not change.) For example, given the clue """"some Central African fish,"""" the answer would be """"Chad shad.""""" 2 the most important bundle of grain stalks chief sheaf 20121125 "Every answer consists of a made-up two-word phrase in which the first word starts with CH, and the second word is pronounced the same as the first except with an SH sound. (The spelling may or may not change.) For example, given the clue """"some Central African fish,"""" the answer would be """"Chad shad.""""" 3 picked plays and musicals to see chose shows 20121125 "Every answer consists of a made-up two-word phrase in which the first word starts with CH, and the second word is pronounced the same as the first except with an SH sound. (The spelling may or may not change.) For example, given the clue """"some Central African fish,"""" the answer would be """"Chad shad.""""" 4 actor norris' shy, self-effacing comments chuck's shucks 20121125 "Every answer consists of a made-up two-word phrase in which the first word starts with CH, and the second word is pronounced the same as the first except with an SH sound. (The spelling may or may not change.) For example, given the clue """"some Central African fish,"""" the answer would be """"Chad shad.""""" 5 young pig at a prestigious connecticut prep school choate shoat 20121125 "Every answer consists of a made-up two-word phrase in which the first word starts with CH, and the second word is pronounced the same as the first except with an SH sound. (The spelling may or may not change.) For example, given the clue """"some Central African fish,"""" the answer would be """"Chad shad.""""" 6 a ups employee who is in a good mood chipper shipper 20121125 "Every answer consists of a made-up two-word phrase in which the first word starts with CH, and the second word is pronounced the same as the first except with an SH sound. (The spelling may or may not change.) For example, given the clue """"some Central African fish,"""" the answer would be """"Chad shad.""""" 7 one looking to buy a helicopter chopper shopper 20121125 "Every answer consists of a made-up two-word phrase in which the first word starts with CH, and the second word is pronounced the same as the first except with an SH sound. (The spelling may or may not change.) For example, given the clue """"some Central African fish,"""" the answer would be """"Chad shad.""""" 8 a place where stolen vehicles are dismantled chop shop 20121125 "Every answer consists of a made-up two-word phrase in which the first word starts with CH, and the second word is pronounced the same as the first except with an SH sound. (The spelling may or may not change.) For example, given the clue """"some Central African fish,"""" the answer would be """"Chad shad.""""" 9 a piece of paper with notes used surreptitiously when taking a test cheat sheet 20201220 I'm going to read you some sentences about Christmas. For each one, change one letter in one of the words to reveal a state capital. 1 The kids made a lot of noise unwrapping their presents. Boise 20201220 I'm going to read you some sentences about Christmas. For each one, change one letter in one of the words to reveal a state capital. 2 Christmas is aseason of traditionalsacraments. Sacramento 20201220 I'm going to read you some sentences about Christmas. For each one, change one letter in one of the words to reveal a state capital. 3 You'll have to pierce the packagingto get out the Christmas ornament. Pierre 20201220 I'm going to read you some sentences about Christmas. For each one, change one letter in one of the words to reveal a state capital. 4 Julys and Augusts are when stores place their Christmas orders. Augusta 20201220 I'm going to read you some sentences about Christmas. For each one, change one letter in one of the words to reveal a state capital. 5 Is this the same Santa we ran into at the other mall? Santa Fe 20110501 "Every answer is a familiar two-word phrase with AT in the middle. The letter A ends the first word of the phrase, and the letter T begins the second word. For the clue, """"trying out of something while changes are still being made,"""" the answer would be """"beta test.""""" 1 area in theWestern Atlantic where ships mysteriously disappear BERMUDA TRIANGLE 20110501 "Every answer is a familiar two-word phrase with AT in the middle. The letter A ends the first word of the phrase, and the letter T begins the second word. For the clue, """"trying out of something while changes are still being made,"""" the answer would be """"beta test.""""" 2 area explored by lewis and clark louisiana territory 20110501 "Every answer is a familiar two-word phrase with AT in the middle. The letter A ends the first word of the phrase, and the letter T begins the second word. For the clue, """"trying out of something while changes are still being made,"""" the answer would be """"beta test.""""" 3 sausage, pepperoni, and mushrooms, for example pizza toppings 20110501 "Every answer is a familiar two-word phrase with AT in the middle. The letter A ends the first word of the phrase, and the letter T begins the second word. For the clue, """"trying out of something while changes are still being made,"""" the answer would be """"beta test.""""" 4 in school, it might carry food for a student's lunch cafeteria tray 20110501 "Every answer is a familiar two-word phrase with AT in the middle. The letter A ends the first word of the phrase, and the letter T begins the second word. For the clue, """"trying out of something while changes are still being made,"""" the answer would be """"beta test.""""" 5 very thinly-slicedcrisp piece of bread, named after an australian opera singer melba toast 20110501 "Every answer is a familiar two-word phrase with AT in the middle. The letter A ends the first word of the phrase, and the letter T begins the second word. For the clue, """"trying out of something while changes are still being made,"""" the answer would be """"beta test.""""" 6 atlanta school for engineers georgia tech 20110501 "Every answer is a familiar two-word phrase with AT in the middle. The letter A ends the first word of the phrase, and the letter T begins the second word. For the clue, """"trying out of something while changes are still being made,"""" the answer would be """"beta test.""""" 7 american league team, that until recently played at thehubert h. humphreymetrodome minnesota twins 20110501 "Every answer is a familiar two-word phrase with AT in the middle. The letter A ends the first word of the phrase, and the letter T begins the second word. For the clue, """"trying out of something while changes are still being made,"""" the answer would be """"beta test.""""" 8 teenage mutants from comic books, television, and movies ninja turtles 20110501 "Every answer is a familiar two-word phrase with AT in the middle. The letter A ends the first word of the phrase, and the letter T begins the second word. For the clue, """"trying out of something while changes are still being made,"""" the answer would be """"beta test.""""" 9 on clocks, it's4 hours behind new york and washington alaska time 20110501 "Every answer is a familiar two-word phrase with AT in the middle. The letter A ends the first word of the phrase, and the letter T begins the second word. For the clue, """"trying out of something while changes are still being made,"""" the answer would be """"beta test.""""" 10 female singer, with the number one country album _comeon over_ shania twain 20110501 "Every answer is a familiar two-word phrase with AT in the middle. The letter A ends the first word of the phrase, and the letter T begins the second word. For the clue, """"trying out of something while changes are still being made,"""" the answer would be """"beta test.""""" 11 best [supporting] actress oscar-winner for _my cousin vinnie_ marisa tomei 20110501 "Every answer is a familiar two-word phrase with AT in the middle. The letter A ends the first word of the phrase, and the letter T begins the second word. For the clue, """"trying out of something while changes are still being made,"""" the answer would be """"beta test.""""" 12 best actress oscar-winner for _howard's end_ emma thompson 20110501 "Every answer is a familiar two-word phrase with AT in the middle. The letter A ends the first word of the phrase, and the letter T begins the second word. For the clue, """"trying out of something while changes are still being made,"""" the answer would be """"beta test.""""" 13 inventor rival of thomas edison, who specialized in electromagnetism nikola tesla 20110501 "Every answer is a familiar two-word phrase with AT in the middle. The letter A ends the first word of the phrase, and the letter T begins the second word. For the clue, """"trying out of something while changes are still being made,"""" the answer would be """"beta test.""""" 14 legal affairs correspondent for npr nina totenberg 20080427 "In this week's on-air puzzle, you are given anagrams of the names of constellations. You name the constellations. For example, given """"ray"""" plus L, the answer would be """"Lyra.""""" 1 sire + a Aries 20080427 "In this week's on-air puzzle, you are given anagrams of the names of constellations. You name the constellations. For example, given """"ray"""" plus L, the answer would be """"Lyra.""""" 2 bail + r Libra 20080427 "In this week's on-air puzzle, you are given anagrams of the names of constellations. You name the constellations. For example, given """"ray"""" plus L, the answer would be """"Lyra.""""" 3 iron + o Orion 20080427 "In this week's on-air puzzle, you are given anagrams of the names of constellations. You name the constellations. For example, given """"ray"""" plus L, the answer would be """"Lyra.""""" 4 card + o Draco 20080427 "In this week's on-air puzzle, you are given anagrams of the names of constellations. You name the constellations. For example, given """"ray"""" plus L, the answer would be """"Lyra.""""" 5 voir + g Virgo 20080427 "In this week's on-air puzzle, you are given anagrams of the names of constellations. You name the constellations. For example, given """"ray"""" plus L, the answer would be """"Lyra.""""" 6 sutra + a Taurus 20080427 "In this week's on-air puzzle, you are given anagrams of the names of constellations. You name the constellations. For example, given """"ray"""" plus L, the answer would be """"Lyra.""""" 7 pauses + g Pegasus 20080427 "In this week's on-air puzzle, you are given anagrams of the names of constellations. You name the constellations. For example, given """"ray"""" plus L, the answer would be """"Lyra.""""" 8 rosarium + n Ursa Minor 20080427 "In this week's on-air puzzle, you are given anagrams of the names of constellations. You name the constellations. For example, given """"ray"""" plus L, the answer would be """"Lyra.""""" 9 macaronis + j Canis Major 20101002 "You are given two sentences, each with two blanks. The word that goes in the first blank ends with the letter D. Drop the D, and phonetically, you'll get a new word that goes in the second blank to complete the sentence. For example, answers might be """"field"""" to """"feel"""" or """"beard"""" to """"beer.""""" 1 in napa valley, this road will ___ fifteen miles through blank country WIND to WINE 20101002 "You are given two sentences, each with two blanks. The word that goes in the first blank ends with the letter D. Drop the D, and phonetically, you'll get a new word that goes in the second blank to complete the sentence. For example, answers might be """"field"""" to """"feel"""" or """"beard"""" to """"beer.""""" 2 Even though I have a phobia about yellow foods, I added ___ to my hot dog with all the strength I could ___. WIND to WINE 20101002 "You are given two sentences, each with two blanks. The word that goes in the first blank ends with the letter D. Drop the D, and phonetically, you'll get a new word that goes in the second blank to complete the sentence. For example, answers might be """"field"""" to """"feel"""" or """"beard"""" to """"beer.""""" 4 In Colonial New Hampshire, before the European immigrants could settle in ___, they first had to ___ the Indians. CONQUER to CONCORD 20101002 "You are given two sentences, each with two blanks. The word that goes in the first blank ends with the letter D. Drop the D, and phonetically, you'll get a new word that goes in the second blank to complete the sentence. For example, answers might be """"field"""" to """"feel"""" or """"beard"""" to """"beer.""""" 5 During Seniors Week on the TV game show, one old ___ just couldn't push his ___ fast enough to get any answers. BUZZARD to BUZZER 20101002 "You are given two sentences, each with two blanks. The word that goes in the first blank ends with the letter D. Drop the D, and phonetically, you'll get a new word that goes in the second blank to complete the sentence. For example, answers might be """"field"""" to """"feel"""" or """"beard"""" to """"beer.""""" 6 To improve city hygiene, ___, Alaska has installed a new ___ system. SEWARD to SEWER 20101002 "You are given two sentences, each with two blanks. The word that goes in the first blank ends with the letter D. Drop the D, and phonetically, you'll get a new word that goes in the second blank to complete the sentence. For example, answers might be """"field"""" to """"feel"""" or """"beard"""" to """"beer.""""" 7 In return for her long-time service to the Food Network chef, Sarah personally received an ___ broach from ___ himself. EMERALD to EMERIL 20101002 "You are given two sentences, each with two blanks. The word that goes in the first blank ends with the letter D. Drop the D, and phonetically, you'll get a new word that goes in the second blank to complete the sentence. For example, answers might be """"field"""" to """"feel"""" or """"beard"""" to """"beer.""""" 8 With the increase of money into the English royal treasury, King ___ was considerably ___. RICHARD to RICHER 20141026 A series of sentences will be given; each one contains a blank. Put the name of an occupation in the blank to complete the sentence in a punny way. As a help, the occupation itself is hinted at, directly or indirectly, near the start of the sentence. 1 the buff law officer was an expert in the _____ arts. martial; marshal 20141026 A series of sentences will be given; each one contains a blank. Put the name of an occupation in the blank to complete the sentence in a punny way. As a help, the occupation itself is hinted at, directly or indirectly, near the start of the sentence. 2 the clergy woman is so sneaky, i wouldn't put anything _____. pastor; pastor 20141026 A series of sentences will be given; each one contains a blank. Put the name of an occupation in the blank to complete the sentence in a punny way. As a help, the occupation itself is hinted at, directly or indirectly, near the start of the sentence. 3 "one military man asked another who was headed overseas, """"have you _____ house yet?""""" soldier; soldier 20141026 A series of sentences will be given; each one contains a blank. Put the name of an occupation in the blank to complete the sentence in a punny way. As a help, the occupation itself is hinted at, directly or indirectly, near the start of the sentence. 4 "after the writer had finished her latest novel, her agent asked, """"_____ any other books you want to do?""""" author; author 20141026 A series of sentences will be given; each one contains a blank. Put the name of an occupation in the blank to complete the sentence in a punny way. As a help, the occupation itself is hinted at, directly or indirectly, near the start of the sentence. 5 after my assistant optometrist missed three straight days of work without explanation, _____ pay. i docked her; eye doctor 20070225 I'm going to read you some clues. The answer to each clue rhymes with the last word in the clue.  1 wheels for a tike bike or trike 20070225 I'm going to read you some clues. The answer to each clue rhymes with the last word in the clue.  2 what a bee does buzz 20070225 I'm going to read you some clues. The answer to each clue rhymes with the last word in the clue.  3 where bees thrive hive 20070225 I'm going to read you some clues. The answer to each clue rhymes with the last word in the clue.  5 it's balled at the end of a wrist fist 20070225 I'm going to read you some clues. The answer to each clue rhymes with the last word in the clue.  6 what March 21 will bring Spring 20070225 I'm going to read you some clues. The answer to each clue rhymes with the last word in the clue.  7 in hockey, something that is struck puck 20070225 I'm going to read you some clues. The answer to each clue rhymes with the last word in the clue.  8 animal in a pound hound 20070225 I'm going to read you some clues. The answer to each clue rhymes with the last word in the clue.  9 animal at a zoo kangaroo, or gnu 20070225 I'm going to read you some clues. The answer to each clue rhymes with the last word in the clue.  10 animal you might take to a vet pet 20070225 I'm going to read you some clues. The answer to each clue rhymes with the last word in the clue.  11 avoid, as work shirk 20070225 I'm going to read you some clues. The answer to each clue rhymes with the last word in the clue.  12 event in which athletes compete meet 20070225 I'm going to read you some clues. The answer to each clue rhymes with the last word in the clue.  14 evaluate rate 20070225 I'm going to read you some clues. The answer to each clue rhymes with the last word in the clue.  15 feeling suspense tense 20111023 "You'll be given three words. Name the fourth word that, when added to each of these words, creates a familiar two-word phrase. The answer will rhyme with one of the three words. For example, if you're given """"boob,"""" """"inner"""" and """"test,"""" the fourth word would be """"tube.""""" 1 bald, legal, spread eagle 20111023 "You'll be given three words. Name the fourth word that, when added to each of these words, creates a familiar two-word phrase. The answer will rhyme with one of the three words. For example, if you're given """"boob,"""" """"inner"""" and """"test,"""" the fourth word would be """"tube.""""" 2 funny, hush, prize money 20111023 "You'll be given three words. Name the fourth word that, when added to each of these words, creates a familiar two-word phrase. The answer will rhyme with one of the three words. For example, if you're given """"boob,"""" """"inner"""" and """"test,"""" the fourth word would be """"tube.""""" 3 air, junk, snail mail 20111023 "You'll be given three words. Name the fourth word that, when added to each of these words, creates a familiar two-word phrase. The answer will rhyme with one of the three words. For example, if you're given """"boob,"""" """"inner"""" and """"test,"""" the fourth word would be """"tube.""""" 4 father, prime, spare time 20111023 "You'll be given three words. Name the fourth word that, when added to each of these words, creates a familiar two-word phrase. The answer will rhyme with one of the three words. For example, if you're given """"boob,"""" """"inner"""" and """"test,"""" the fourth word would be """"tube.""""" 5 creature, double, facial feature 20111023 "You'll be given three words. Name the fourth word that, when added to each of these words, creates a familiar two-word phrase. The answer will rhyme with one of the three words. For example, if you're given """"boob,"""" """"inner"""" and """"test,"""" the fourth word would be """"tube.""""" 6 cheat, cookie, rap sheet 20111023 "You'll be given three words. Name the fourth word that, when added to each of these words, creates a familiar two-word phrase. The answer will rhyme with one of the three words. For example, if you're given """"boob,"""" """"inner"""" and """"test,"""" the fourth word would be """"tube.""""" 7 drag, sack, space race 20111023 "You'll be given three words. Name the fourth word that, when added to each of these words, creates a familiar two-word phrase. The answer will rhyme with one of the three words. For example, if you're given """"boob,"""" """"inner"""" and """"test,"""" the fourth word would be """"tube.""""" 8 "rhymes with """"cheat""""" cheat 20140427 For each word provided, give a word that can follow it to complete a familiar two-word phrase. The first two letters of the provided word should be the last two letters of the answer. Example: Red Square 1 ARmored (3 letters) cAR 20140427 For each word provided, give a word that can follow it to complete a familiar two-word phrase. The first two letters of the provided word should be the last two letters of the answer. Example: Red Square 2 ELectric eEL eEL 20140427 For each word provided, give a word that can follow it to complete a familiar two-word phrase. The first two letters of the provided word should be the last two letters of the answer. Example: Red Square 3 UNearned rUN rUN 20140427 For each word provided, give a word that can follow it to complete a familiar two-word phrase. The first two letters of the provided word should be the last two letters of the answer. Example: Red Square 4 SKi maSK maSK 20140427 For each word provided, give a word that can follow it to complete a familiar two-word phrase. The first two letters of the provided word should be the last two letters of the answer. Example: Red Square 5 SEcond baSE baSE 20140427 For each word provided, give a word that can follow it to complete a familiar two-word phrase. The first two letters of the provided word should be the last two letters of the answer. Example: Red Square 6 NEutral zoNE zoNE 20140427 For each word provided, give a word that can follow it to complete a familiar two-word phrase. The first two letters of the provided word should be the last two letters of the answer. Example: Red Square 7 STress teST teST 20140427 For each word provided, give a word that can follow it to complete a familiar two-word phrase. The first two letters of the provided word should be the last two letters of the answer. Example: Red Square 8 INsert coIN coIN 20140427 For each word provided, give a word that can follow it to complete a familiar two-word phrase. The first two letters of the provided word should be the last two letters of the answer. Example: Red Square 9 CEntrifugal forCE forCE 20140427 For each word provided, give a word that can follow it to complete a familiar two-word phrase. The first two letters of the provided word should be the last two letters of the answer. Example: Red Square 10 ORgan donOR donOR 20140427 For each word provided, give a word that can follow it to complete a familiar two-word phrase. The first two letters of the provided word should be the last two letters of the answer. Example: Red Square 11 TEctonic plaTE plaTE 20140427 For each word provided, give a word that can follow it to complete a familiar two-word phrase. The first two letters of the provided word should be the last two letters of the answer. Example: Red Square 12 LEgal eagLE (titLE) eagLE 20140427 For each word provided, give a word that can follow it to complete a familiar two-word phrase. The first two letters of the provided word should be the last two letters of the answer. Example: Red Square 13 CHess matCH matCH 20140427 For each word provided, give a word that can follow it to complete a familiar two-word phrase. The first two letters of the provided word should be the last two letters of the answer. Example: Red Square 14 THree-toed sloTH sloTH 20140427 For each word provided, give a word that can follow it to complete a familiar two-word phrase. The first two letters of the provided word should be the last two letters of the answer. Example: Red Square 15 OLd schoOL schoOL 20140427 For each word provided, give a word that can follow it to complete a familiar two-word phrase. The first two letters of the provided word should be the last two letters of the answer. Example: Red Square 16 SHoe poliSH (fetiSH) poliSH 20140427 For each word provided, give a word that can follow it to complete a familiar two-word phrase. The first two letters of the provided word should be the last two letters of the answer. Example: Red Square 17 IRan-contra affaIR affaIR 20140427 For each word provided, give a word that can follow it to complete a familiar two-word phrase. The first two letters of the provided word should be the last two letters of the answer. Example: Red Square 18 LEafy vegetabLE vegetabLE 20140427 For each word provided, give a word that can follow it to complete a familiar two-word phrase. The first two letters of the provided word should be the last two letters of the answer. Example: Red Square 19 GEneral outraGE (knowledGE) outraGE 20120923 "Every answer is the name of a TV show, past or present. Each can be found in consecutive letters in the sentences read. Name the TV shows. For example, in the sentence, """"We watched the acrobat many times,"""" the hidden TV show is BATMAN. Hint: Each answer has at least six letters." 1 the straw hid everything rawhide 20120923 "Every answer is the name of a TV show, past or present. Each can be found in consecutive letters in the sentences read. Name the TV shows. For example, in the sentence, """"We watched the acrobat many times,"""" the hidden TV show is BATMAN. Hint: Each answer has at least six letters." 2 the desert is not the environs i desire ironside 20120923 "Every answer is the name of a TV show, past or present. Each can be found in consecutive letters in the sentences read. Name the TV shows. For example, in the sentence, """"We watched the acrobat many times,"""" the hidden TV show is BATMAN. Hint: Each answer has at least six letters." 3 the waitress always forgets martinis get smart 20120923 "Every answer is the name of a TV show, past or present. Each can be found in consecutive letters in the sentences read. Name the TV shows. For example, in the sentence, """"We watched the acrobat many times,"""" the hidden TV show is BATMAN. Hint: Each answer has at least six letters." 4 did you wear that sandal last week dallas 20120923 "Every answer is the name of a TV show, past or present. Each can be found in consecutive letters in the sentences read. Name the TV shows. For example, in the sentence, """"We watched the acrobat many times,"""" the hidden TV show is BATMAN. Hint: Each answer has at least six letters." 5 the kids gave daddy nasty looks dynasty 20120923 "Every answer is the name of a TV show, past or present. Each can be found in consecutive letters in the sentences read. Name the TV shows. For example, in the sentence, """"We watched the acrobat many times,"""" the hidden TV show is BATMAN. Hint: Each answer has at least six letters." 6 to save your marriage, you'll have to restart, rekindle your romance star trek 20120923 "Every answer is the name of a TV show, past or present. Each can be found in consecutive letters in the sentences read. Name the TV shows. For example, in the sentence, """"We watched the acrobat many times,"""" the hidden TV show is BATMAN. Hint: Each answer has at least six letters." 7 josÃ, after seeing what the mexican did, came racing over candid camera 20161120 Every answer is the name of an NPR host — that is, one of the colleagues of this week's special guest, NPR's Lulu Garcia-Navarro, who will be taking over as host of Weekend Edition Sunday in January. Given some wordplay on their names, you name the hosts. 1 His last name contains all the letters of his first name + POSH Ari Shapiro 20161120 Every answer is the name of an NPR host — that is, one of the colleagues of this week's special guest, NPR's Lulu Garcia-Navarro, who will be taking over as host of Weekend Edition Sunday in January. Given some wordplay on their names, you name the hosts. 2 His name contains the letters VEINS consecutively Steve Innskeep 20161120 Every answer is the name of an NPR host — that is, one of the colleagues of this week's special guest, NPR's Lulu Garcia-Navarro, who will be taking over as host of Weekend Edition Sunday in January. Given some wordplay on their names, you name the hosts. 3 "This host's full [nick]name can be anagrammed to spell """"Oracular vulgarian.""""" Lulu Garcia-Navarro 20161120 Every answer is the name of an NPR host — that is, one of the colleagues of this week's special guest, NPR's Lulu Garcia-Navarro, who will be taking over as host of Weekend Edition Sunday in January. Given some wordplay on their names, you name the hosts. 4 This host's first name has five letters, of which four are vowels Audie Cornish 20161120 Every answer is the name of an NPR host — that is, one of the colleagues of this week's special guest, NPR's Lulu Garcia-Navarro, who will be taking over as host of Weekend Edition Sunday in January. Given some wordplay on their names, you name the hosts. 5 A former host, now a special correspondent, her name has the word BASSI backward inside it Melissa Block 20161120 Every answer is the name of an NPR host — that is, one of the colleagues of this week's special guest, NPR's Lulu Garcia-Navarro, who will be taking over as host of Weekend Edition Sunday in January. Given some wordplay on their names, you name the hosts. 6 If you add an I to the end of this host's last name you'll have an alcoholic drink Rachel Martin 20090705 "The theme today is """"cow."""" Every answer is a familiar two-word phrase or name in which the first word starts with """"C-O"""" and the second word starts with """"W."""" For example, if the clue is """"person who can fix a hard drive in 10 minutes,"""" the answer would be, """"computer whiz.""""" 1 former conflict between Communist countries and the West Cold War 20090705 "The theme today is """"cow."""" Every answer is a familiar two-word phrase or name in which the first word starts with """"C-O"""" and the second word starts with """"W."""" For example, if the clue is """"person who can fix a hard drive in 10 minutes,"""" the answer would be, """"computer whiz.""""" 2 volume that has all of the plays and poems of shakespeare, for example complete/collected works 20090705 "The theme today is """"cow."""" Every answer is a familiar two-word phrase or name in which the first word starts with """"C-O"""" and the second word starts with """"W."""" For example, if the clue is """"person who can fix a hard drive in 10 minutes,"""" the answer would be, """"computer whiz.""""" 3 person who works for (David) Letterman or Jon Stewart comedy writer 20090705 "The theme today is """"cow."""" Every answer is a familiar two-word phrase or name in which the first word starts with """"C-O"""" and the second word starts with """"W."""" For example, if the clue is """"person who can fix a hard drive in 10 minutes,"""" the answer would be, """"computer whiz.""""" 4 music genre for Dolly Parton and Garth Brooks Country-Western 20090705 "The theme today is """"cow."""" Every answer is a familiar two-word phrase or name in which the first word starts with """"C-O"""" and the second word starts with """"W."""" For example, if the clue is """"person who can fix a hard drive in 10 minutes,"""" the answer would be, """"computer whiz.""""" 5 woman who serves drinks before dinner (typically in a restaurant) cocktail waitress 20090705 "The theme today is """"cow."""" Every answer is a familiar two-word phrase or name in which the first word starts with """"C-O"""" and the second word starts with """"W."""" For example, if the clue is """"person who can fix a hard drive in 10 minutes,"""" the answer would be, """"computer whiz.""""" 6 a term made up of two or more shorter parts, such as highway or football compound word 20090705 "The theme today is """"cow."""" Every answer is a familiar two-word phrase or name in which the first word starts with """"C-O"""" and the second word starts with """"W."""" For example, if the clue is """"person who can fix a hard drive in 10 minutes,"""" the answer would be, """"computer whiz.""""" 7 a gun carried in one's pocket, for example concealed weapon 20090705 "The theme today is """"cow."""" Every answer is a familiar two-word phrase or name in which the first word starts with """"C-O"""" and the second word starts with """"W."""" For example, if the clue is """"person who can fix a hard drive in 10 minutes,"""" the answer would be, """"computer whiz.""""" 8 "Edgar Allan Poe poem with ""The""" The Conqueror Worm 20090705 "The theme today is """"cow."""" Every answer is a familiar two-word phrase or name in which the first word starts with """"C-O"""" and the second word starts with """"W."""" For example, if the clue is """"person who can fix a hard drive in 10 minutes,"""" the answer would be, """"computer whiz.""""" 9 a vehicle that carried pioneers across the West Conestoga/covered wagon 20090705 "The theme today is """"cow."""" Every answer is a familiar two-word phrase or name in which the first word starts with """"C-O"""" and the second word starts with """"W."""" For example, if the clue is """"person who can fix a hard drive in 10 minutes,"""" the answer would be, """"computer whiz.""""" 10 a hard hat construction worker 20090705 "The theme today is """"cow."""" Every answer is a familiar two-word phrase or name in which the first word starts with """"C-O"""" and the second word starts with """"W."""" For example, if the clue is """"person who can fix a hard drive in 10 minutes,"""" the answer would be, """"computer whiz.""""" 11 The FTC (Federal Trade Commission), for example consumer watchdog 20090705 "The theme today is """"cow."""" Every answer is a familiar two-word phrase or name in which the first word starts with """"C-O"""" and the second word starts with """"W."""" For example, if the clue is """"person who can fix a hard drive in 10 minutes,"""" the answer would be, """"computer whiz.""""" 12 brand of non-dairy dessert topping Cool Whip 20090705 "The theme today is """"cow."""" Every answer is a familiar two-word phrase or name in which the first word starts with """"C-O"""" and the second word starts with """"W."""" For example, if the clue is """"person who can fix a hard drive in 10 minutes,"""" the answer would be, """"computer whiz.""""" 13 good material for an electrical circuit copper wire 20090705 "The theme today is """"cow."""" Every answer is a familiar two-word phrase or name in which the first word starts with """"C-O"""" and the second word starts with """"W."""" For example, if the clue is """"person who can fix a hard drive in 10 minutes,"""" the answer would be, """"computer whiz.""""" 14 a person who takes a prize for entering a competition contest winner 20131006 "For each given category, name things in the category starting with the letters R, H, Y, M, E. For example, if the category were """"chemical elements with names ending in -ium,"""" you might say: radium, helium, yttrium, magnesium and einsteinium. You can give the answers in any order, and any answer that works is fine." 1 rivers Rhine, Hudson, Meuse, Yser, Euphrates 20131006 "For each given category, name things in the category starting with the letters R, H, Y, M, E. For example, if the category were """"chemical elements with names ending in -ium,"""" you might say: radium, helium, yttrium, magnesium and einsteinium. You can give the answers in any order, and any answer that works is fine." 2 animals seen in the zoo rhino, hyena, yak (Rachel), moose, 20131006 "For each given category, name things in the category starting with the letters R, H, Y, M, E. For example, if the category were """"chemical elements with names ending in -ium,"""" you might say: radium, helium, yttrium, magnesium and einsteinium. You can give the answers in any order, and any answer that works is fine." 3 traffic signs route, merge, yield, merge, exit 20131006 "For each given category, name things in the category starting with the letters R, H, Y, M, E. For example, if the category were """"chemical elements with names ending in -ium,"""" you might say: radium, helium, yttrium, magnesium and einsteinium. You can give the answers in any order, and any answer that works is fine." 4 songs by the Beatles Revolution, Help, Yellow Submarine, 20131006 "For each given category, name things in the category starting with the letters R, H, Y, M, E. For example, if the category were """"chemical elements with names ending in -ium,"""" you might say: radium, helium, yttrium, magnesium and einsteinium. You can give the answers in any order, and any answer that works is fine." 5 things to eat for breakfast Rice Krispies, honey, yogurt, 20100116 "Every answer is a word, name or familiar phrase with alternating A's. For example, if the clue is """"Woody Allen movie,"""" the answer is """"Bananas.""""" 1 Group of wagons traveling through the desert caravan 20100116 "Every answer is a word, name or familiar phrase with alternating A's. For example, if the clue is """"Woody Allen movie,"""" the answer is """"Bananas.""""" 2 A knight at King Arthur's Round Table (Sir) Galahad 20100116 "Every answer is a word, name or familiar phrase with alternating A's. For example, if the clue is """"Woody Allen movie,"""" the answer is """"Bananas.""""" 3 Islamic month of fasting Ramadan 20100116 "Every answer is a word, name or familiar phrase with alternating A's. For example, if the clue is """"Woody Allen movie,"""" the answer is """"Bananas.""""" 4 Capital of Venezuela Caracas 20100116 "Every answer is a word, name or familiar phrase with alternating A's. For example, if the clue is """"Woody Allen movie,"""" the answer is """"Bananas.""""" 5 Language of Barcelona Catalan 20100116 "Every answer is a word, name or familiar phrase with alternating A's. For example, if the clue is """"Woody Allen movie,"""" the answer is """"Bananas.""""" 6 Sleepwear Pajamas 20100116 "Every answer is a word, name or familiar phrase with alternating A's. For example, if the clue is """"Woody Allen movie,"""" the answer is """"Bananas.""""" 7 A hard road surface Macadam 20100116 "Every answer is a word, name or familiar phrase with alternating A's. For example, if the clue is """"Woody Allen movie,"""" the answer is """"Bananas.""""" 8 Distant (two word answer) Far away 20100116 "Every answer is a word, name or familiar phrase with alternating A's. For example, if the clue is """"Woody Allen movie,"""" the answer is """"Bananas.""""" 9 Marathon runner Alberto Salazar 20100116 "Every answer is a word, name or familiar phrase with alternating A's. For example, if the clue is """"Woody Allen movie,"""" the answer is """"Bananas.""""" 10 Telly who was on the telly Savalas 20100116 "Every answer is a word, name or familiar phrase with alternating A's. For example, if the clue is """"Woody Allen movie,"""" the answer is """"Bananas.""""" 11 July first, as celebrated north of the border Canada Day 20100116 "Every answer is a word, name or familiar phrase with alternating A's. For example, if the clue is """"Woody Allen movie,"""" the answer is """"Bananas.""""" 12 Light summer headwear (two words) Panama Hat 20100116 "Every answer is a word, name or familiar phrase with alternating A's. For example, if the clue is """"Woody Allen movie,"""" the answer is """"Bananas.""""" 13 Body of water adjoining Cuba's capital Havana Bay 20100116 "Every answer is a word, name or familiar phrase with alternating A's. For example, if the clue is """"Woody Allen movie,"""" the answer is """"Bananas.""""" 14 Largest city in eastern Afghanistan Jalalabad 20100116 "Every answer is a word, name or familiar phrase with alternating A's. For example, if the clue is """"Woody Allen movie,"""" the answer is """"Bananas.""""" 15 A light, multi-hulled boat used for fishing or sport Catamaran 20100116 "Every answer is a word, name or familiar phrase with alternating A's. For example, if the clue is """"Woody Allen movie,"""" the answer is """"Bananas.""""" 16 Connection between the Atlantic and Pacific oceans Panama Canal 20170212 """""S-V"""" are the initials of St. Valentine. They're also the initials of some other familiar two-word phrases and names. See if you can get them from the clues." 1 Center of the tech industry in California Silicon Valley 20170212 """""S-V"""" are the initials of St. Valentine. They're also the initials of some other familiar two-word phrases and names. See if you can get them from the clues." 2 Unofficial ballot conducted as a test of opinion Straw Vote 20170212 """""S-V"""" are the initials of St. Valentine. They're also the initials of some other familiar two-word phrases and names. See if you can get them from the clues." 3 Poison from a viper Serpent/Snake Venom 20170212 """""S-V"""" are the initials of St. Valentine. They're also the initials of some other familiar two-word phrases and names. See if you can get them from the clues." 4 Saigon was its capital South Vietnam 20170212 """""S-V"""" are the initials of St. Valentine. They're also the initials of some other familiar two-word phrases and names. See if you can get them from the clues." 5 "Italian phrase meaning """"said quietly so as not to be overheard""""" Sotto Voce 20170212 """""S-V"""" are the initials of St. Valentine. They're also the initials of some other familiar two-word phrases and names. See if you can get them from the clues." 6 Lettuce, carrots, radishes, etc. Salad Vegetables 20170212 """""S-V"""" are the initials of St. Valentine. They're also the initials of some other familiar two-word phrases and names. See if you can get them from the clues." 7 "20th-century jazz singer who was nicknamed """"The Divine One""""" Sarah Vaughn 20170212 """""S-V"""" are the initials of St. Valentine. They're also the initials of some other familiar two-word phrases and names. See if you can get them from the clues." 8 Member of the Sex Pistols Sid Vicious 20170212 """""S-V"""" are the initials of St. Valentine. They're also the initials of some other familiar two-word phrases and names. See if you can get them from the clues." 9 Arena, for example Sports Venue 20170212 """""S-V"""" are the initials of St. Valentine. They're also the initials of some other familiar two-word phrases and names. See if you can get them from the clues." 10 Extremely valuable stringed instrument from Italy Stradivarius Violin 20170212 """""S-V"""" are the initials of St. Valentine. They're also the initials of some other familiar two-word phrases and names. See if you can get them from the clues." 11 B, in a set of encyclopedias Second Volume 20170212 """""S-V"""" are the initials of St. Valentine. They're also the initials of some other familiar two-word phrases and names. See if you can get them from the clues." 12 Kind of mirror on a car Side View 20170212 """""S-V"""" are the initials of St. Valentine. They're also the initials of some other familiar two-word phrases and names. See if you can get them from the clues." 13 Pull-down item that reduces glare for a driver Sun Visor 20170212 """""S-V"""" are the initials of St. Valentine. They're also the initials of some other familiar two-word phrases and names. See if you can get them from the clues." 14 Necessary paperwork for a foreigner attending an American university Student Visa 20150830 This week's puzzle was inspired by crossword constructor Merl Reagle, who died earlier this month. 1 I’ve got you under my sink I’ve got you under my skin 20150830 This week's puzzle was inspired by crossword constructor Merl Reagle, who died earlier this month. 2 The shadow of your slime The shadow of your smile. 20150830 This week's puzzle was inspired by crossword constructor Merl Reagle, who died earlier this month. 3 You oughta be in piecrust You oughta be in pictures. 20150830 This week's puzzle was inspired by crossword constructor Merl Reagle, who died earlier this month. 4 The rates of a clown The tears of a clown. 20150830 This week's puzzle was inspired by crossword constructor Merl Reagle, who died earlier this month. 5 How can you mend a broken Earth? How can you mend a broken heart. 20150830 This week's puzzle was inspired by crossword constructor Merl Reagle, who died earlier this month. 6 Hello Lloyd Hello Dolly. 20150830 This week's puzzle was inspired by crossword constructor Merl Reagle, who died earlier this month. 7 A shore with no name A horse with no name. 20150830 This week's puzzle was inspired by crossword constructor Merl Reagle, who died earlier this month. 8 The first time I ever saw your cafe The first time I ever saw your 20150830 This week's puzzle was inspired by crossword constructor Merl Reagle, who died earlier this month. 9 Skis on my list Kiss on my list. 20150830 This week's puzzle was inspired by crossword constructor Merl Reagle, who died earlier this month. 10 Before the next predator falls Before the next teardrop falls. 20150830 This week's puzzle was inspired by crossword constructor Merl Reagle, who died earlier this month. 11 Every bather you take Every breathe you take. 20190217 In celebration of Presidents Day, every answer is the name of a U.S. president. I'm going to give you some words and familiar phrases. For each one, name a president in which the consonants are the same — and in the same order — as the consonants in the word or phrase. 1 afraid ford 20190217 In celebration of Presidents Day, every answer is the name of a U.S. president. I'm going to give you some words and familiar phrases. For each one, name a president in which the consonants are the same — and in the same order — as the consonants in the word or phrase. 2 oregon reagan 20190217 In celebration of Presidents Day, every answer is the name of a U.S. president. I'm going to give you some words and familiar phrases. For each one, name a president in which the consonants are the same — and in the same order — as the consonants in the word or phrase. 3 beam obama 20190217 In celebration of Presidents Day, every answer is the name of a U.S. president. I'm going to give you some words and familiar phrases. For each one, name a president in which the consonants are the same — and in the same order — as the consonants in the word or phrase. 4 meaner monroe 20190217 In celebration of Presidents Day, every answer is the name of a U.S. president. I'm going to give you some words and familiar phrases. For each one, name a president in which the consonants are the same — and in the same order — as the consonants in the word or phrase. 5 heavier hoover 20190217 In celebration of Presidents Day, every answer is the name of a U.S. president. I'm going to give you some words and familiar phrases. For each one, name a president in which the consonants are the same — and in the same order — as the consonants in the word or phrase. 6 hurries in harrison 20190217 In celebration of Presidents Day, every answer is the name of a U.S. president. I'm going to give you some words and familiar phrases. For each one, name a president in which the consonants are the same — and in the same order — as the consonants in the word or phrase. 7 price pierce 20190217 In celebration of Presidents Day, every answer is the name of a U.S. president. I'm going to give you some words and familiar phrases. For each one, name a president in which the consonants are the same — and in the same order — as the consonants in the word or phrase. 8 guarantee grant 20190217 In celebration of Presidents Day, every answer is the name of a U.S. president. I'm going to give you some words and familiar phrases. For each one, name a president in which the consonants are the same — and in the same order — as the consonants in the word or phrase. 9 rather arthur 20190217 In celebration of Presidents Day, every answer is the name of a U.S. president. I'm going to give you some words and familiar phrases. For each one, name a president in which the consonants are the same — and in the same order — as the consonants in the word or phrase. 10 ice ledge coolidge 20190217 In celebration of Presidents Day, every answer is the name of a U.S. president. I'm going to give you some words and familiar phrases. For each one, name a president in which the consonants are the same — and in the same order — as the consonants in the word or phrase. 11 outer moon truman 20120226 Every answer is the name of a film that won an Academy Award for Best Picture. Identify the films from their anagrams. 1 cork+y rocky, 1976 20120226 Every answer is the name of a film that won an Academy Award for Best Picture. Identify the films from their anagrams. 2 ruben+h ben hur, 1959 20120226 Every answer is the name of a film that won an Academy Award for Best Picture. Identify the films from their anagrams. 3 medusa+a amadeus, 1984 20120226 Every answer is the name of a film that won an Academy Award for Best Picture. Identify the films from their anagrams. 4 marina +n rain man, 1988 20120226 Every answer is the name of a film that won an Academy Award for Best Picture. Identify the films from their anagrams. 5 dog trail+a gladiator, 1988 20120226 Every answer is the name of a film that won an Academy Award for Best Picture. Identify the films from their anagrams. 6 a vertebra+h braveheart, 1992 20120226 Every answer is the name of a film that won an Academy Award for Best Picture. Identify the films from their anagrams. 7 no gunfire +v unforgiven, 1992 20120226 Every answer is the name of a film that won an Academy Award for Best Picture. Identify the films from their anagrams. 8 had together+f the godfather, 1972 20120506 "You'll be given a series of categories. For each one, name something in the category beginning with each of the letters of the word """"robin."""" For example, given the category """"two-syllable boys' names,"""" the answers could be """"Roger,"""" """"Omar,"""" """"Barry,"""" """"Isaac"""" and """"Neville.""""" 1 islands Rarotonga {Cook Islands} (Rhodes, Reunion), Oahu, Bahamas, Iceland (Ireland, Ios), Niihau (New Guinea, Newfoundland) 20130630 "For the Sunday before the Fourth of July weekend, every answer is the last name of a U.S. president, which comes from their anagrams. For example, """"shrub"""" without R is """"Bush.""""" 1 "frond minus ""n""" ford 20130630 "For the Sunday before the Fourth of July weekend, every answer is the last name of a U.S. president, which comes from their anagrams. For example, """"shrub"""" without R is """"Bush.""""" 2 "fatty minus ""y""" taft 20130630 "For the Sunday before the Fourth of July weekend, every answer is the last name of a U.S. president, which comes from their anagrams. For example, """"shrub"""" without R is """"Bush.""""" 3 "hyenas minus ""n""" hayes 20130630 "For the Sunday before the Fourth of July weekend, every answer is the last name of a U.S. president, which comes from their anagrams. For example, """"shrub"""" without R is """"Bush.""""" 4 "garnet minus ""e""" grant 20130630 "For the Sunday before the Fourth of July weekend, every answer is the last name of a U.S. president, which comes from their anagrams. For example, """"shrub"""" without R is """"Bush.""""" 5 "realty minus ""a""" tyler 20130630 "For the Sunday before the Fourth of July weekend, every answer is the last name of a U.S. president, which comes from their anagrams. For example, """"shrub"""" without R is """"Bush.""""" 6 "amoeba minus ""e""" obama 20130630 "For the Sunday before the Fourth of July weekend, every answer is the last name of a U.S. president, which comes from their anagrams. For example, """"shrub"""" without R is """"Bush.""""" 7 "terrace minus ""e""" carter 20130630 "For the Sunday before the Fourth of July weekend, every answer is the last name of a U.S. president, which comes from their anagrams. For example, """"shrub"""" without R is """"Bush.""""" 8 "tantrum minus ""t""" truman 20130630 "For the Sunday before the Fourth of July weekend, every answer is the last name of a U.S. president, which comes from their anagrams. For example, """"shrub"""" without R is """"Bush.""""" 9 "slowing minus ""g""" wilson 20130630 "For the Sunday before the Fourth of July weekend, every answer is the last name of a U.S. president, which comes from their anagrams. For example, """"shrub"""" without R is """"Bush.""""" 10 "arrange minus ""r""" reagan 20130630 "For the Sunday before the Fourth of July weekend, every answer is the last name of a U.S. president, which comes from their anagrams. For example, """"shrub"""" without R is """"Bush.""""" 11 "royalty minus ""y""" taylor 20130630 "For the Sunday before the Fourth of July weekend, every answer is the last name of a U.S. president, which comes from their anagrams. For example, """"shrub"""" without R is """"Bush.""""" 12 "diamonds minus ""d""" madison 20130630 "For the Sunday before the Fourth of July weekend, every answer is the last name of a U.S. president, which comes from their anagrams. For example, """"shrub"""" without R is """"Bush.""""" 13 "handgrip minus ""p""" harding 20130630 "For the Sunday before the Fourth of July weekend, every answer is the last name of a U.S. president, which comes from their anagrams. For example, """"shrub"""" without R is """"Bush.""""" 14 "narrowish minus ""w""" harrison 20120617 "Every answer is a word, phrase or name starting with the letter """"B,"""" ending in """"Y"""" and having """"A"""" and """"B"""" inside, in that order, although not necessarily consecutively. For example, if I said """"assistant to a baseball team,"""" the answer would be either """"batboy"""" or """"ballboy.""""" 1 Late science fiction author Ray Bradbury 20120617 "Every answer is a word, phrase or name starting with the letter """"B,"""" ending in """"Y"""" and having """"A"""" and """"B"""" inside, in that order, although not necessarily consecutively. For example, if I said """"assistant to a baseball team,"""" the answer would be either """"batboy"""" or """"ballboy.""""" 2 North Africa's ____ Coast Barbary [or Berber] 20120617 "Every answer is a word, phrase or name starting with the letter """"B,"""" ending in """"Y"""" and having """"A"""" and """"B"""" inside, in that order, although not necessarily consecutively. For example, if I said """"assistant to a baseball team,"""" the answer would be either """"batboy"""" or """"ballboy.""""" 3 iPhone competitor Blackberry 20120617 "Every answer is a word, phrase or name starting with the letter """"B,"""" ending in """"Y"""" and having """"A"""" and """"B"""" inside, in that order, although not necessarily consecutively. For example, if I said """"assistant to a baseball team,"""" the answer would be either """"batboy"""" or """"ballboy.""""" 4 Herman Melville's story, ____ the Scrivener Bartleby 20120617 "Every answer is a word, phrase or name starting with the letter """"B,"""" ending in """"Y"""" and having """"A"""" and """"B"""" inside, in that order, although not necessarily consecutively. For example, if I said """"assistant to a baseball team,"""" the answer would be either """"batboy"""" or """"ballboy.""""" 5 Where ships arrive in Sydney, Australia Botany Bay 20120617 "Every answer is a word, phrase or name starting with the letter """"B,"""" ending in """"Y"""" and having """"A"""" and """"B"""" inside, in that order, although not necessarily consecutively. For example, if I said """"assistant to a baseball team,"""" the answer would be either """"batboy"""" or """"ballboy.""""" 6 An attractive woman in a swimsuit at the shore bathing beauty 20120617 "Every answer is a word, phrase or name starting with the letter """"B,"""" ending in """"Y"""" and having """"A"""" and """"B"""" inside, in that order, although not necessarily consecutively. For example, if I said """"assistant to a baseball team,"""" the answer would be either """"batboy"""" or """"ballboy.""""" 7 "Any of the singers of """"California Girls""""" Beach Boy 20120617 "Every answer is a word, phrase or name starting with the letter """"B,"""" ending in """"Y"""" and having """"A"""" and """"B"""" inside, in that order, although not necessarily consecutively. For example, if I said """"assistant to a baseball team,"""" the answer would be either """"batboy"""" or """"ballboy.""""" 8 "Classic song by the Who, lyric, """"it's only teenage wasteland""""" Baba O'Riley 20120617 "Every answer is a word, phrase or name starting with the letter """"B,"""" ending in """"Y"""" and having """"A"""" and """"B"""" inside, in that order, although not necessarily consecutively. For example, if I said """"assistant to a baseball team,"""" the answer would be either """"batboy"""" or """"ballboy.""""" 9 Interstate 695 in central Maryland Baltimore Beltway 20120617 "Every answer is a word, phrase or name starting with the letter """"B,"""" ending in """"Y"""" and having """"A"""" and """"B"""" inside, in that order, although not necessarily consecutively. For example, if I said """"assistant to a baseball team,"""" the answer would be either """"batboy"""" or """"ballboy.""""" 10 A crime that might involve giving a note to a teller bank robbery 20120617 "Every answer is a word, phrase or name starting with the letter """"B,"""" ending in """"Y"""" and having """"A"""" and """"B"""" inside, in that order, although not necessarily consecutively. For example, if I said """"assistant to a baseball team,"""" the answer would be either """"batboy"""" or """"ballboy.""""" 11 Anna Sewell novel about a horse Black Beauty 20120617 "Every answer is a word, phrase or name starting with the letter """"B,"""" ending in """"Y"""" and having """"A"""" and """"B"""" inside, in that order, although not necessarily consecutively. For example, if I said """"assistant to a baseball team,"""" the answer would be either """"batboy"""" or """"ballboy.""""" 12 "Lead actress in TV's """"Leave it to Beaver""""" Barbara Billingsley 20120617 "Every answer is a word, phrase or name starting with the letter """"B,"""" ending in """"Y"""" and having """"A"""" and """"B"""" inside, in that order, although not necessarily consecutively. For example, if I said """"assistant to a baseball team,"""" the answer would be either """"batboy"""" or """"ballboy.""""" 13 At some eventual time by and by 20080525 "In this week's on-air puzzle, every answer is a compound word or a familiar two-word phrase in which the first part has a long I vowel sound and the second part has a long O sound. Both parts have just one syllable. For example, given """"a small pink flower growing in a field,"""" the answer would be """"wild rose.""""" 1 the fruit of an evergreen tree PINE CONE 20080525 "In this week's on-air puzzle, every answer is a compound word or a familiar two-word phrase in which the first part has a long I vowel sound and the second part has a long O sound. Both parts have just one syllable. For example, given """"a small pink flower growing in a field,"""" the answer would be """"wild rose.""""" 2 sign on a semi trailer, carrying a house down a highway WIDE LOAD 20080525 "In this week's on-air puzzle, every answer is a compound word or a familiar two-word phrase in which the first part has a long I vowel sound and the second part has a long O sound. Both parts have just one syllable. For example, given """"a small pink flower growing in a field,"""" the answer would be """"wild rose.""""" 3 sound you hear when you pick up a telephone receiver DIAL TONE 20080525 "In this week's on-air puzzle, every answer is a compound word or a familiar two-word phrase in which the first part has a long I vowel sound and the second part has a long O sound. Both parts have just one syllable. For example, given """"a small pink flower growing in a field,"""" the answer would be """"wild rose.""""" 4 set of safety specifications for a building FIRE CODE 20080525 "In this week's on-air puzzle, every answer is a compound word or a familiar two-word phrase in which the first part has a long I vowel sound and the second part has a long O sound. Both parts have just one syllable. For example, given """"a small pink flower growing in a field,"""" the answer would be """"wild rose.""""" 5 half a round of golf NINE HOLES 20080525 "In this week's on-air puzzle, every answer is a compound word or a familiar two-word phrase in which the first part has a long I vowel sound and the second part has a long O sound. Both parts have just one syllable. For example, given """"a small pink flower growing in a field,"""" the answer would be """"wild rose.""""" 6 something a circus performer walks on TIGHT ROPE 20080525 "In this week's on-air puzzle, every answer is a compound word or a familiar two-word phrase in which the first part has a long I vowel sound and the second part has a long O sound. Both parts have just one syllable. For example, given """"a small pink flower growing in a field,"""" the answer would be """"wild rose.""""" 7 browned bread thatTMs not whole wheat, rye, or pumpernickel WHITE 20080525 "In this week's on-air puzzle, every answer is a compound word or a familiar two-word phrase in which the first part has a long I vowel sound and the second part has a long O sound. Both parts have just one syllable. For example, given """"a small pink flower growing in a field,"""" the answer would be """"wild rose.""""" 8 a small emergency vessel at sea LIFE BOAT 20080525 "In this week's on-air puzzle, every answer is a compound word or a familiar two-word phrase in which the first part has a long I vowel sound and the second part has a long O sound. Both parts have just one syllable. For example, given """"a small pink flower growing in a field,"""" the answer would be """"wild rose.""""" 9 a swimming style in which the legs do a scissors kick SIDE STROKE 20080525 "In this week's on-air puzzle, every answer is a compound word or a familiar two-word phrase in which the first part has a long I vowel sound and the second part has a long O sound. Both parts have just one syllable. For example, given """"a small pink flower growing in a field,"""" the answer would be """"wild rose.""""" 10 like some of Nathaniel HawthorneTMs tales TWICE TOLD 20080525 "In this week's on-air puzzle, every answer is a compound word or a familiar two-word phrase in which the first part has a long I vowel sound and the second part has a long O sound. Both parts have just one syllable. For example, given """"a small pink flower growing in a field,"""" the answer would be """"wild rose.""""" 11 what the pyramids of Egypt are made of LIMESTONE 20080525 "In this week's on-air puzzle, every answer is a compound word or a familiar two-word phrase in which the first part has a long I vowel sound and the second part has a long O sound. Both parts have just one syllable. For example, given """"a small pink flower growing in a field,"""" the answer would be """"wild rose.""""" 12 area a pitcher aims for STRIKE ZONE 20080525 "In this week's on-air puzzle, every answer is a compound word or a familiar two-word phrase in which the first part has a long I vowel sound and the second part has a long O sound. Both parts have just one syllable. For example, given """"a small pink flower growing in a field,"""" the answer would be """"wild rose.""""" 13 complete the line œBe it ever so humble, thereTMs no place¦ LIKE 20140504 "Every answer is a five-letter word. You will be given a clue for the word. Besides describing the answer, the clue will also contain the answer in consecutive letters. For example, given """"It's near the planet Mars,"""" you would say, """"Earth.""""" 1 what chicks do hatch 20140504 "Every answer is a five-letter word. You will be given a clue for the word. Besides describing the answer, the clue will also contain the answer in consecutive letters. For example, given """"It's near the planet Mars,"""" you would say, """"Earth.""""" 2 from antoninus' home, say roman 20140504 "Every answer is a five-letter word. You will be given a clue for the word. Besides describing the answer, the clue will also contain the answer in consecutive letters. For example, given """"It's near the planet Mars,"""" you would say, """"Earth.""""" 3 one was awarded to scarlett o'hara's portrayer. oscar 20140504 "Every answer is a five-letter word. You will be given a clue for the word. Besides describing the answer, the clue will also contain the answer in consecutive letters. For example, given """"It's near the planet Mars,"""" you would say, """"Earth.""""" 4 what to call a german brew lager 20140504 "Every answer is a five-letter word. You will be given a clue for the word. Besides describing the answer, the clue will also contain the answer in consecutive letters. For example, given """"It's near the planet Mars,"""" you would say, """"Earth.""""" 5 number of times fdr was rewarded with re-election. three 20140504 "Every answer is a five-letter word. You will be given a clue for the word. Besides describing the answer, the clue will also contain the answer in consecutive letters. For example, given """"It's near the planet Mars,"""" you would say, """"Earth.""""" 6 score a gleeful golfer might have eagle 20140504 "Every answer is a five-letter word. You will be given a clue for the word. Besides describing the answer, the clue will also contain the answer in consecutive letters. For example, given """"It's near the planet Mars,"""" you would say, """"Earth.""""" 7 where a captain lets a boat moor inlet 20140504 "Every answer is a five-letter word. You will be given a clue for the word. Besides describing the answer, the clue will also contain the answer in consecutive letters. For example, given """"It's near the planet Mars,"""" you would say, """"Earth.""""" 8 bout of imbibing excessively binge 20140504 "Every answer is a five-letter word. You will be given a clue for the word. Besides describing the answer, the clue will also contain the answer in consecutive letters. For example, given """"It's near the planet Mars,"""" you would say, """"Earth.""""" 9 they get their shares of an estate heirs 20140504 "Every answer is a five-letter word. You will be given a clue for the word. Besides describing the answer, the clue will also contain the answer in consecutive letters. For example, given """"It's near the planet Mars,"""" you would say, """"Earth.""""" 10 spicy dish which i like chili 20170205 Every answer today is a word containing the letters of SUPER + one or two other letters. 1 + U = Follow PURSUE 20170205 Every answer today is a word containing the letters of SUPER + one or two other letters. 2 + H = A drug dealer PUSHER 20170205 Every answer today is a word containing the letters of SUPER + one or two other letters. 3 + T = Explodes ERUPTS 20170205 Every answer today is a word containing the letters of SUPER + one or two other letters. 5 + T = Get some sleep before a big event REST UP 20170205 Every answer today is a word containing the letters of SUPER + one or two other letters. 6 + C,W = Make a really big mistake SCREW UP 20170205 Every answer today is a word containing the letters of SUPER + one or two other letters. 7 + M,T = Really hard puzzle STUMPER 20170205 Every answer today is a word containing the letters of SUPER + one or two other letters. 8 + I = Revolt RISE UP 20170205 Every answer today is a word containing the letters of SUPER + one or two other letters. 9 + B = Excellent SUPERB 20170205 Every answer today is a word containing the letters of SUPER + one or two other letters. 10 + O = Person who pretends to be someone he or she is not POSEUR 20170205 Every answer today is a word containing the letters of SUPER + one or two other letters. 11 + B,M = Parts of pinball machines BUMPERS 20170205 Every answer today is a word containing the letters of SUPER + one or two other letters. 12 + A,T = Field for grazing PASTURE 20170205 Every answer today is a word containing the letters of SUPER + one or two other letters. 13 + O,P = Reason for doing something PURPOSE 20170205 Every answer today is a word containing the letters of SUPER + one or two other letters. 14 + G,L = Spend extravagantly SPLURGE 20170205 Every answer today is a word containing the letters of SUPER + one or two other letters. 15 + E,L = Drive back by force REPULSE 20170205 Every answer today is a word containing the letters of SUPER + one or two other letters. 16 + N,T = Person making plays on words PUNSTER 20170205 Every answer today is a word containing the letters of SUPER + one or two other letters. 17 + N = Fruit touted as a digestive aid PRUNES 20170205 Every answer today is a word containing the letters of SUPER + one or two other letters. 18 + E = Reduces to a paste PUREES 20100619 "Every answer is a word or name that contains the syllable """"port."""" For example, given the clue """"consequential,"""" the answer would be """"important.""""" 1 a newspaper employee reporter 20100619 "Every answer is a word or name that contains the syllable """"port."""" For example, given the clue """"consequential,"""" the answer would be """"important.""""" 2 a sofa davenport 20100619 "Every answer is a word or name that contains the syllable """"port."""" For example, given the clue """"consequential,"""" the answer would be """"important.""""" 3 a travel device on Star Trek transporter 20100619 "Every answer is a word or name that contains the syllable """"port."""" For example, given the clue """"consequential,"""" the answer would be """"important.""""" 4 a language of Brazil Portugese 20100619 "Every answer is a word or name that contains the syllable """"port."""" For example, given the clue """"consequential,"""" the answer would be """"important.""""" 5 a home of the Trailblazers Portland 20100619 "Every answer is a word or name that contains the syllable """"port."""" For example, given the clue """"consequential,"""" the answer would be """"important.""""" 6 a political candidate needs support[ers] 20100619 "Every answer is a word or name that contains the syllable """"port."""" For example, given the clue """"consequential,"""" the answer would be """"important.""""" 7 a window on a ship porthole 20100619 "Every answer is a word or name that contains the syllable """"port."""" For example, given the clue """"consequential,"""" the answer would be """"important.""""" 8 a Leonardo da Vinci Mona Lisa, for oneportrait 20100619 "Every answer is a word or name that contains the syllable """"port."""" For example, given the clue """"consequential,"""" the answer would be """"important.""""" 9 a travel document passport 20100619 "Every answer is a word or name that contains the syllable """"port."""" For example, given the clue """"consequential,"""" the answer would be """"important.""""" 10 a company that makes goods and sells them overseas exporter 20100619 "Every answer is a word or name that contains the syllable """"port."""" For example, given the clue """"consequential,"""" the answer would be """"important.""""" 11 a variety of mushroom portobello 20100619 "Every answer is a word or name that contains the syllable """"port."""" For example, given the clue """"consequential,"""" the answer would be """"important.""""" 12 a steak porterhouse 20100619 "Every answer is a word or name that contains the syllable """"port."""" For example, given the clue """"consequential,"""" the answer would be """"important.""""" 13 an artist's showcase portfolio 20100619 "Every answer is a word or name that contains the syllable """"port."""" For example, given the clue """"consequential,"""" the answer would be """"important.""""" 14 a word "like """"brunch,"""" which is a blending of breakfast and lunchportmanteau" 20110605 "Every answer is a compound word or familiar two-word phrase, in which the first word has a long O for its vowel sound and the second word has a long U. For example, given the clue """"a traditional Christmas entrée,"""" the answer would be """"goose.""""" 1 a mushroom TOADSTOOL 20110605 "Every answer is a compound word or familiar two-word phrase, in which the first word has a long O for its vowel sound and the second word has a long U. For example, given the clue """"a traditional Christmas entrée,"""" the answer would be """"goose.""""" 2 area in a building CLOAKROOM 20110605 "Every answer is a compound word or familiar two-word phrase, in which the first word has a long O for its vowel sound and the second word has a long U. For example, given the clue """"a traditional Christmas entrée,"""" the answer would be """"goose.""""" 3 teach children HOME SCHOOL 20110605 "Every answer is a compound word or familiar two-word phrase, in which the first word has a long O for its vowel sound and the second word has a long U. For example, given the clue """"a traditional Christmas entrée,"""" the answer would be """"goose.""""" 4 a skating jump TOE LOOP 20110605 "Every answer is a compound word or familiar two-word phrase, in which the first word has a long O for its vowel sound and the second word has a long U. For example, given the clue """"a traditional Christmas entrée,"""" the answer would be """"goose.""""" 5 pithy sign NO NUKES 20110605 "Every answer is a compound word or familiar two-word phrase, in which the first word has a long O for its vowel sound and the second word has a long U. For example, given the clue """"a traditional Christmas entrée,"""" the answer would be """"goose.""""" 6 songs from Broadway SHOW TUNES 20110605 "Every answer is a compound word or familiar two-word phrase, in which the first word has a long O for its vowel sound and the second word has a long U. For example, given the clue """"a traditional Christmas entrée,"""" the answer would be """"goose.""""" 7 footwear for walking in the Arctic SNOWBOOTS 20110605 "Every answer is a compound word or familiar two-word phrase, in which the first word has a long O for its vowel sound and the second word has a long U. For example, given the clue """"a traditional Christmas entrée,"""" the answer would be """"goose.""""" 8 nickname for Snoopy JOE COOL 20110605 "Every answer is a compound word or familiar two-word phrase, in which the first word has a long O for its vowel sound and the second word has a long U. For example, given the clue """"a traditional Christmas entrée,"""" the answer would be """"goose.""""" 9 place to pay to cross a bridge TOLL BOOTH 20110605 "Every answer is a compound word or familiar two-word phrase, in which the first word has a long O for its vowel sound and the second word has a long U. For example, given the clue """"a traditional Christmas entrée,"""" the answer would be """"goose.""""" 10 If it takes a long time for you to explode in anger SLOW FUSE 20111218 "Rearrange a series of anagrams to identify some well-known magazines. For example, if given """"never point,"""" rearrange the letters to spell """"Prevention,"""" the name of a popular health magazine." 1 clear icy film [6,6] Family Circle 20111218 "Rearrange a series of anagrams to identify some well-known magazines. For example, if given """"never point,"""" rearrange the letters to spell """"Prevention,"""" the name of a popular health magazine." 2 registered ads [7,6] Reader's Digest 20111218 "Rearrange a series of anagrams to identify some well-known magazines. For example, if given """"never point,"""" rearrange the letters to spell """"Prevention,"""" the name of a popular health magazine." 3 not in heat [3,6] The Nation 20111218 "Rearrange a series of anagrams to identify some well-known magazines. For example, if given """"never point,"""" rearrange the letters to spell """"Prevention,"""" the name of a popular health magazine." 4 this mansion [11] Smithsonian 20111218 "Rearrange a series of anagrams to identify some well-known magazines. For example, if given """"never point,"""" rearrange the letters to spell """"Prevention,"""" the name of a popular health magazine." 5 grade A opposites [4,5,6] Soap Opera Digest 20111218 "Rearrange a series of anagrams to identify some well-known magazines. For example, if given """"never point,"""" rearrange the letters to spell """"Prevention,"""" the name of a popular health magazine." 6 more corruptness [8,7] Consumer Reports 20140810 You'll be given some four-letter words. For each one, insert two letters exactly in the middle to complete a common, uncaplitalized six-letter word. 1 club, er cherub 20140810 You'll be given some four-letter words. For each one, insert two letters exactly in the middle to complete a common, uncaplitalized six-letter word. 2 poet, ck pocket 20140810 You'll be given some four-letter words. For each one, insert two letters exactly in the middle to complete a common, uncaplitalized six-letter word. 3 rain, is raisin 20140810 You'll be given some four-letter words. For each one, insert two letters exactly in the middle to complete a common, uncaplitalized six-letter word. 4 when, it whiten 20140810 You'll be given some four-letter words. For each one, insert two letters exactly in the middle to complete a common, uncaplitalized six-letter word. 5 hose, rs horse 20140810 You'll be given some four-letter words. For each one, insert two letters exactly in the middle to complete a common, uncaplitalized six-letter word. 6 show, ad shadow 20140810 You'll be given some four-letter words. For each one, insert two letters exactly in the middle to complete a common, uncaplitalized six-letter word. 7 door, ct doctor 20140810 You'll be given some four-letter words. For each one, insert two letters exactly in the middle to complete a common, uncaplitalized six-letter word. 8 toil, ns tonsil 20140810 You'll be given some four-letter words. For each one, insert two letters exactly in the middle to complete a common, uncaplitalized six-letter word. 9 joey, ck jockey 20140810 You'll be given some four-letter words. For each one, insert two letters exactly in the middle to complete a common, uncaplitalized six-letter word. 10 fish, ni finish 20101205 "Every answer is a six-letter word or phrase that contains the consecutive letters S-B. For example, given the clue """"comic Bill and others,"""" the answer would be """"Cosbys.""""" 1 the capital of portugal lisbon 20101205 "Every answer is a six-letter word or phrase that contains the consecutive letters S-B. For example, given the clue """"comic Bill and others,"""" the answer would be """"Cosbys.""""" 2 """""white christmas"""" singer" crosby 20101205 "Every answer is a six-letter word or phrase that contains the consecutive letters S-B. For example, given the clue """"comic Bill and others,"""" the answer would be """"Cosbys.""""" 3 old, walled section of algiers casbah (or kasbah) 20101205 "Every answer is a six-letter word or phrase that contains the consecutive letters S-B. For example, given the clue """"comic Bill and others,"""" the answer would be """"Cosbys.""""" 4 great character of literature gatsby 20101205 "Every answer is a six-letter word or phrase that contains the consecutive letters S-B. For example, given the clue """"comic Bill and others,"""" the answer would be """"Cosbys.""""" 5 __________ park, new jersey asbury 20101205 "Every answer is a six-letter word or phrase that contains the consecutive letters S-B. For example, given the clue """"comic Bill and others,"""" the answer would be """"Cosbys.""""" 6 one who talks and talks and talks gasbag 20101205 "Every answer is a six-letter word or phrase that contains the consecutive letters S-B. For example, given the clue """"comic Bill and others,"""" the answer would be """"Cosbys.""""" 7 a clearer of dishes at a restaurant busboy 20101205 "Every answer is a six-letter word or phrase that contains the consecutive letters S-B. For example, given the clue """"comic Bill and others,"""" the answer would be """"Cosbys.""""" 8 lover of pyramus, in myth thisbe 20101205 "Every answer is a six-letter word or phrase that contains the consecutive letters S-B. For example, given the clue """"comic Bill and others,"""" the answer would be """"Cosbys.""""" 9 island where the poet sappho was born lesbos 20101205 "Every answer is a six-letter word or phrase that contains the consecutive letters S-B. For example, given the clue """"comic Bill and others,"""" the answer would be """"Cosbys.""""" 10 in _spiderman_, peter parker's closest friend, harry __________ osborn 20101205 "Every answer is a six-letter word or phrase that contains the consecutive letters S-B. For example, given the clue """"comic Bill and others,"""" the answer would be """"Cosbys.""""" 11 (2-word phrase) to elapse pass by 20101205 "Every answer is a six-letter word or phrase that contains the consecutive letters S-B. For example, given the clue """"comic Bill and others,"""" the answer would be """"Cosbys.""""" 12 composer of the brandenburg concertos j.s. bach 20101205 "Every answer is a six-letter word or phrase that contains the consecutive letters S-B. For example, given the clue """"comic Bill and others,"""" the answer would be """"Cosbys.""""" 13 toy company that makes g.i. joe and lincoln logs hasbro 20150201 Every answer is a made-up two-word phrase, where the second and third letters of the first word are switched to get the second word. Example: Serene bivalve would be calm clam 1 lab container in a Colorado ski town Vail vial 20150201 Every answer is a made-up two-word phrase, where the second and third letters of the first word are switched to get the second word. Example: Serene bivalve would be calm clam 2 the only fruit used to make a gin fizz sole sloe 20150201 Every answer is a made-up two-word phrase, where the second and third letters of the first word are switched to get the second word. Example: Serene bivalve would be calm clam 3 a bonus benefit at a nursery school pre-K perk 20150201 Every answer is a made-up two-word phrase, where the second and third letters of the first word are switched to get the second word. Example: Serene bivalve would be calm clam 4 personal journal from a milkmaid dairy diary 20150201 Every answer is a made-up two-word phrase, where the second and third letters of the first word are switched to get the second word. Example: Serene bivalve would be calm clam 5 cunningly inane slily silly 20150201 Every answer is a made-up two-word phrase, where the second and third letters of the first word are switched to get the second word. Example: Serene bivalve would be calm clam 6 a skater on wheels who has less hair on his head balder blader 20150201 Every answer is a made-up two-word phrase, where the second and third letters of the first word are switched to get the second word. Example: Serene bivalve would be calm clam 7 a thin piece of a precious metal silver sliver 20150201 Every answer is a made-up two-word phrase, where the second and third letters of the first word are switched to get the second word. Example: Serene bivalve would be calm clam 8 a person who sings love songs while he performs autopsies coroner--crooner 20150201 Every answer is a made-up two-word phrase, where the second and third letters of the first word are switched to get the second word. Example: Serene bivalve would be calm clam 9 an ideal government official in Japan or France perfect prefect 20120610 "Given a sentence, change one letter in one word to make a new word which completely reverses the meaning of the sentence. For example, given """"The singer is not coming on stage."""" Changing the """"T"""" in not to a """"W"""" in the word """"not"""" makes the sentence, """"The singer is now coming on stage.""""" 1 the poker player took one look at his cards and decided to hold fold 20120610 "Given a sentence, change one letter in one word to make a new word which completely reverses the meaning of the sentence. For example, given """"The singer is not coming on stage."""" Changing the """"T"""" in not to a """"W"""" in the word """"not"""" makes the sentence, """"The singer is now coming on stage.""""" 2 a new company manager arrived and immediately hired ten more workers. fired 20120610 "Given a sentence, change one letter in one word to make a new word which completely reverses the meaning of the sentence. For example, given """"The singer is not coming on stage."""" Changing the """"T"""" in not to a """"W"""" in the word """"not"""" makes the sentence, """"The singer is now coming on stage.""""" 3 the moon will be waning for the next seven days. waxing 20120610 "Given a sentence, change one letter in one word to make a new word which completely reverses the meaning of the sentence. For example, given """"The singer is not coming on stage."""" Changing the """"T"""" in not to a """"W"""" in the word """"not"""" makes the sentence, """"The singer is now coming on stage.""""" 4 when the gymnast came to do her most difficult jump, she failed it. nailed 20120610 "Given a sentence, change one letter in one word to make a new word which completely reverses the meaning of the sentence. For example, given """"The singer is not coming on stage."""" Changing the """"T"""" in not to a """"W"""" in the word """"not"""" makes the sentence, """"The singer is now coming on stage.""""" 5 when it comes to dealing with other people, george is always the sort to go along. alone 20120610 "Given a sentence, change one letter in one word to make a new word which completely reverses the meaning of the sentence. For example, given """"The singer is not coming on stage."""" Changing the """"T"""" in not to a """"W"""" in the word """"not"""" makes the sentence, """"The singer is now coming on stage.""""" 6 when margaret got back from vacation, she was still raving about her hotel accommodations. raging 20120610 "Given a sentence, change one letter in one word to make a new word which completely reverses the meaning of the sentence. For example, given """"The singer is not coming on stage."""" Changing the """"T"""" in not to a """"W"""" in the word """"not"""" makes the sentence, """"The singer is now coming on stage.""""" 7 generals ordered the troops to march at the start of night. light 20120610 "Given a sentence, change one letter in one word to make a new word which completely reverses the meaning of the sentence. For example, given """"The singer is not coming on stage."""" Changing the """"T"""" in not to a """"W"""" in the word """"not"""" makes the sentence, """"The singer is now coming on stage.""""" 8 the civic group toasted the mayor for her controversial position. roasted 20120610 "Given a sentence, change one letter in one word to make a new word which completely reverses the meaning of the sentence. For example, given """"The singer is not coming on stage."""" Changing the """"T"""" in not to a """"W"""" in the word """"not"""" makes the sentence, """"The singer is now coming on stage.""""" 9 when grandmother was a little girl, she was rather homely. comely 20120610 "Given a sentence, change one letter in one word to make a new word which completely reverses the meaning of the sentence. For example, given """"The singer is not coming on stage."""" Changing the """"T"""" in not to a """"W"""" in the word """"not"""" makes the sentence, """"The singer is now coming on stage.""""" 10 to open the jammed lock, you will have to use your brain. brawn 20150329 "The challenge is a game of Categories based on the word """"watch."""" For each category provided, name something in the category starting with each of the letters W-A-T-C-H. For example, parts of the human body would be """"waist,"""" """"arm,"""" """"thigh,"""" """"chest"""" and """"head.""""" 1 presidential first names William, Abraham, Theodore. Calvin, Harry 20150329 "The challenge is a game of Categories based on the word """"watch."""" For each category provided, name something in the category starting with each of the letters W-A-T-C-H. For example, parts of the human body would be """"waist,"""" """"arm,"""" """"thigh,"""" """"chest"""" and """"head.""""" 2 insects Wasp, Ant, Tick, Caterpillar, Horne 20150329 "The challenge is a game of Categories based on the word """"watch."""" For each category provided, name something in the category starting with each of the letters W-A-T-C-H. For example, parts of the human body would be """"waist,"""" """"arm,"""" """"thigh,"""" """"chest"""" and """"head.""""" 3 TV channels WOR [Will intended Weather], AMC, TNN, CBS, HBO 20150329 "The challenge is a game of Categories based on the word """"watch."""" For each category provided, name something in the category starting with each of the letters W-A-T-C-H. For example, parts of the human body would be """"waist,"""" """"arm,"""" """"thigh,"""" """"chest"""" and """"head.""""" 4 words that can precede BALL Whiffle, air, tennis, croquet, har 20180401 Editor's note on April 2, 2018: If you've listened to the audio, you may have figured it out quickly: Part of this broadcast was an April Fools' joke. We hope you enjoyed it. 1 It could ______ only bad things when hundreds of enemy ships sailed into ______. It could PORTEND only bad things when hundreds of enemy ships sailed into PORT. 20180401 Editor's note on April 2, 2018: If you've listened to the audio, you may have figured it out quickly: Part of this broadcast was an April Fools' joke. We hope you enjoyed it. 2 In the children's rhyme, I intended to buy ______ for a salad when I was going to St. ______. In the children's rhyme, I intended to buy ENDIVES for a salad when I was going to St. IVES. 20180401 Editor's note on April 2, 2018: If you've listened to the audio, you may have figured it out quickly: Part of this broadcast was an April Fools' joke. We hope you enjoyed it. 3 The rap singer wore several ______ around his neck, and his ______ were held up by a large, shiny belt. The rap singer wore several PENDANTS around his neck, and his PANTS were held up by a large, shiny belt. 20180401 Editor's note on April 2, 2018: If you've listened to the audio, you may have figured it out quickly: Part of this broadcast was an April Fools' joke. We hope you enjoyed it. 4 The stylist had a hard time BLENDING the client's minimalist asthetic with his need to wear some BLING. The stylist had a hard time BLENDING the client's minimalist asthetic with his need to wear some BLING. 20151227 It's the end of the year, so it's time for a challenge featuring the names of people who made news in 2015. For each description given, name the person (or people) described. 1 kim davis rowan county, kentucky, county clerk who refused to issue marriage certificates to gay couples, based on her religious principles 20151227 It's the end of the year, so it's time for a challenge featuring the names of people who made news in 2015. For each description given, name the person (or people) described. 2 justin trudeau new prime minister of canada 20151227 It's the end of the year, so it's time for a challenge featuring the names of people who made news in 2015. For each description given, name the person (or people) described. 3 anthony sadler, spencer stone; alek skaralatos aboard a high-speed train en route to paris from amsterdam, they overpowered a moroccan terrorist who had opened fire 20151227 It's the end of the year, so it's time for a challenge featuring the names of people who made news in 2015. For each description given, name the person (or people) described. 4 kylo ren; rey; finn characters in star wars: the force awakens 20151227 It's the end of the year, so it's time for a challenge featuring the names of people who made news in 2015. For each description given, name the person (or people) described. 5 ariana gutierrez miss columbia who was mistakenly crowned as miss universe by steve harvey 20070408 I'm going to give you some words. For each one, name a world capital that rhymes 1 mussels Brussels 20070408 I'm going to give you some words. For each one, name a world capital that rhymes 2 sienna Vienna 20070408 I'm going to give you some words. For each one, name a world capital that rhymes 3 Savannah Havana 20070408 I'm going to give you some words. For each one, name a world capital that rhymes 4 gnome Rome 20070408 I'm going to give you some words. For each one, name a world capital that rhymes 5 belly Delhi 20070408 I'm going to give you some words. For each one, name a world capital that rhymes 6 neato Quito 20070408 I'm going to give you some words. For each one, name a world capital that rhymes 7 tyro Cairo 20070408 I'm going to give you some words. For each one, name a world capital that rhymes 8 schema Lima 20070408 I'm going to give you some words. For each one, name a world capital that rhymes 9 Roscoe Moscow 20070408 I'm going to give you some words. For each one, name a world capital that rhymes 10 troll Seoul 20070408 I'm going to give you some words. For each one, name a world capital that rhymes 11 vanilla Manilla 20070408 I'm going to give you some words. For each one, name a world capital that rhymes 12 runic Munich 20070408 I'm going to give you some words. For each one, name a world capital that rhymes 13 staples Naples 20070408 I'm going to give you some words. For each one, name a world capital that rhymes 14 kidney Sydney 20070408 I'm going to give you some words. For each one, name a world capital that rhymes 15 Dennis Venice 20070408 I'm going to give you some words. For each one, name a world capital that rhymes 16 dorky Gorky 20070408 I'm going to give you some words. For each one, name a world capital that rhymes 17 alone Cologne 20070408 I'm going to give you some words. For each one, name a world capital that rhymes 18 Yemen Breme 20160619 Every answer today is a word that starts with Para-. First, I'll define it in a regular way and then in a punny way. You tell me the words. 1 gear for skydivers / two photography sessions parachutes/pair o' shoots 20160619 Every answer today is a word that starts with Para-. First, I'll define it in a regular way and then in a punny way. You tell me the words. 2 a perfect place / equipment in the game monopoly paradise/pair o' dice 20160619 Every answer today is a word that starts with Para-. First, I'll define it in a regular way and then in a punny way. You tell me the words. 3 to render incapable of moving / two untruths paralyze/pair o' lies 20160619 Every answer today is a word that starts with Para-. First, I'll define it in a regular way and then in a punny way. You tell me the words. 4 low protective walls / a dog and a cast parapets/pair o' pets 20160619 Every answer today is a word that starts with Para-. First, I'll define it in a regular way and then in a punny way. You tell me the words. 5 units in an article / two explanatory diagrams paragraphs/pair o' graphs 20160619 Every answer today is a word that starts with Para-. First, I'll define it in a regular way and then in a punny way. You tell me the words. 6 gets pulled in a harness through the air / back-to-back events to clear inventory parasails/pair o' sales 20160619 Every answer today is a word that starts with Para-. First, I'll define it in a regular way and then in a punny way. You tell me the words. 7 lovers / othello and a fellow countryman* paramours/pair o' moors 20160619 Every answer today is a word that starts with Para-. First, I'll define it in a regular way and then in a punny way. You tell me the words. 8 self-contradictory statement / small medical practice* paradox/pair o' docs 20160731 I'm going to give you two four-letter words. Rearrange the letters in each of them to make two synonyms. 1 lore / trap role /part 20160731 I'm going to give you two four-letter words. Rearrange the letters in each of them to make two synonyms. 2 foal / zeal loaf / laze 20160731 I'm going to give you two four-letter words. Rearrange the letters in each of them to make two synonyms. 3 dune / bear nude / bare 20160731 I'm going to give you two four-letter words. Rearrange the letters in each of them to make two synonyms. 4 care / shad race / dash 20160731 I'm going to give you two four-letter words. Rearrange the letters in each of them to make two synonyms. 5 bake / ones nose / beak 20160731 I'm going to give you two four-letter words. Rearrange the letters in each of them to make two synonyms. 6 drag /maul grad / alum 20160731 I'm going to give you two four-letter words. Rearrange the letters in each of them to make two synonyms. 7 rate / nerd tear* / rend 20160731 I'm going to give you two four-letter words. Rearrange the letters in each of them to make two synonyms. 8 leap / grin ring / peal 20160731 I'm going to give you two four-letter words. Rearrange the letters in each of them to make two synonyms. 9 leap / shay pale / ashy 20160731 I'm going to give you two four-letter words. Rearrange the letters in each of them to make two synonyms. 11 cool / stun loco ? nuts 20160731 I'm going to give you two four-letter words. Rearrange the letters in each of them to make two synonyms. 12 rare / shut rear / tush* 20160731 I'm going to give you two four-letter words. Rearrange the letters in each of them to make two synonyms. 13 lath / post halt / stop 20190908 Every answer today is a word, name, or familiar phrase in which the only consonants are G and L — each of which may be repeated any number of times. Besides G's and L's, all the other letters are vowels — A, E, I, O, or U. 1 Ice home in the Arctic igloo or iglu 20190908 Every answer today is a word, name, or familiar phrase in which the only consonants are G and L — each of which may be repeated any number of times. Besides G's and L's, all the other letters are vowels — A, E, I, O, or U. 2 Scum on top of a pond algae 20190908 Every answer today is a word, name, or familiar phrase in which the only consonants are G and L — each of which may be repeated any number of times. Besides G's and L's, all the other letters are vowels — A, E, I, O, or U. 3 Group of geese gaggle 20190908 Every answer today is a word, name, or familiar phrase in which the only consonants are G and L — each of which may be repeated any number of times. Besides G's and L's, all the other letters are vowels — A, E, I, O, or U. 4 Popular search engine Google 20190908 Every answer today is a word, name, or familiar phrase in which the only consonants are G and L — each of which may be repeated any number of times. Besides G's and L's, all the other letters are vowels — A, E, I, O, or U. 5 Russian dramatist Nikolai Gogol Galileo 20190908 Every answer today is a word, name, or familiar phrase in which the only consonants are G and L — each of which may be repeated any number of times. Besides G's and L's, all the other letters are vowels — A, E, I, O, or U. 6 Bags packed for the airport luggage 20190908 Every answer today is a word, name, or familiar phrase in which the only consonants are G and L — each of which may be repeated any number of times. Besides G's and L's, all the other letters are vowels — A, E, I, O, or U. 7 Winemaker Ernest or Julio Gallo Ernest 20190908 Every answer today is a word, name, or familiar phrase in which the only consonants are G and L — each of which may be repeated any number of times. Besides G's and L's, all the other letters are vowels — A, E, I, O, or U. 8 Pioneering astronomer from Pisa Galileo 20190908 Every answer today is a word, name, or familiar phrase in which the only consonants are G and L — each of which may be repeated any number of times. Besides G's and L's, all the other letters are vowels — A, E, I, O, or U. 9 Sea where Jesus preached Galilee 20190908 Every answer today is a word, name, or familiar phrase in which the only consonants are G and L — each of which may be repeated any number of times. Besides G's and L's, all the other letters are vowels — A, E, I, O, or U. 10 Assert wrongdoing without proof allege 20190908 Every answer today is a word, name, or familiar phrase in which the only consonants are G and L — each of which may be repeated any number of times. Besides G's and L's, all the other letters are vowels — A, E, I, O, or U. 11 Dashboard item showing a fluid level (two words) oil gauge 20190908 Every answer today is a word, name, or familiar phrase in which the only consonants are G and L — each of which may be repeated any number of times. Besides G's and L's, all the other letters are vowels — A, E, I, O, or U. 12 Sound of someone drinking from a bottle (three words) glug glug glug 20111127 "Each answer is a compound word or a familiar two-word phrase containing the consecutive letters H-H-O, as in H2O. For example, given the clue """"Dutch province containing Rotterdam and The Hague,"""" the answer would be """"South Holland.""""" 1 a period from noon to 1:00 for many workers lunch hour 20161204 Every answer this week is the last name of a world leader of the past 100 years. For each country and an anagram of the name, find the name of the leader. 1 Russia, input Putin 20161204 Every answer this week is the last name of a world leader of the past 100 years. For each country and an anagram of the name, find the name of the leader. 2 Cuba, actors Castro 20161204 Every answer this week is the last name of a world leader of the past 100 years. For each country and an anagram of the name, find the name of the leader. 3 Philippines, caroms Marcos 20161204 Every answer this week is the last name of a world leader of the past 100 years. For each country and an anagram of the name, find the name of the leader. 4 Soviet Union, Latins Stalin 20161204 Every answer this week is the last name of a world leader of the past 100 years. For each country and an anagram of the name, find the name of the leader. 5 Pakistan, hot tub Bhutto 20161204 Every answer this week is the last name of a world leader of the past 100 years. For each country and an anagram of the name, find the name of the leader. 6 South Africa, lead man Mandela 20161204 Every answer this week is the last name of a world leader of the past 100 years. For each country and an anagram of the name, find the name of the leader. 7 Ethiopia, sea isles Selassie 20200329 I'm going to give you a word and an extra letter. Anagram everything into a new word ... in which the added letter is silent. Example: MOW + B --> WOMB1. GRIN + W2. TOGS + H3. FINE + K4. TENS + C5. HANS + G6. BRIDE + S7. NOMAD + L8. DOTER + B9. PEDLAR + O10. SIMONE + T11. RETINT + W12. AIMING + E 13. NO SHAME + D14. CHEAP MAN + G 1 grin + w wring 20150104 Given a clue, each response is a two-word answer with the first word starting with B-R and the second word starting with R 1 lead character in a story told by Uncle Remus BR'er Rabbit 20150104 Given a clue, each response is a two-word answer with the first word starting with B-R and the second word starting with R 2 a restaurant freebie that usually comes in a basket BRead Roll 20150104 Given a clue, each response is a two-word answer with the first word starting with B-R and the second word starting with R 3 what a woman about to be married signs up for in a store BRidal Registry 20150104 Given a clue, each response is a two-word answer with the first word starting with B-R and the second word starting with R 4 23-year third baseman for the Baltimore Orioles BRooks Robinson 20150104 Given a clue, each response is a two-word answer with the first word starting with B-R and the second word starting with R 5 a leafy vegetable whose buds have small heads BRoccoli Raab 20150104 Given a clue, each response is a two-word answer with the first word starting with B-R and the second word starting with R 6 area of the house in which to have the morning meal BBreakfast Room 20150104 Given a clue, each response is a two-word answer with the first word starting with B-R and the second word starting with R 7 A nominal thing you win when there's no actual prize BRagging Rights 20110626 "Each answer is the name of a well-known film. You are given three words. Say the words out loud quickly to identify the film. For example, given the words """"booty,"""" """"Andy"""" and """"pieced,"""" the answer would be Beauty and the Beast." 1 sadder, denied, viva saturday night fever 20110626 "Each answer is the name of a well-known film. You are given three words. Say the words out loud quickly to identify the film. For example, given the words """"booty,"""" """"Andy"""" and """"pieced,"""" the answer would be Beauty and the Beast." 2 you're, hassock, bark jurassic park 20110626 "Each answer is the name of a well-known film. You are given three words. Say the words out loud quickly to identify the film. For example, given the words """"booty,"""" """"Andy"""" and """"pieced,"""" the answer would be Beauty and the Beast." 3 thick, erotic, id the karate kid 20110626 "Each answer is the name of a well-known film. You are given three words. Say the words out loud quickly to identify the film. For example, given the words """"booty,"""" """"Andy"""" and """"pieced,"""" the answer would be Beauty and the Beast." 5 fiat, cup, pull the odd couple 20110626 "Each answer is the name of a well-known film. You are given three words. Say the words out loud quickly to identify the film. For example, given the words """"booty,"""" """"Andy"""" and """"pieced,"""" the answer would be Beauty and the Beast." 6 toast, hoary, do to kill a mockingbird 20110626 "Each answer is the name of a well-known film. You are given three words. Say the words out loud quickly to identify the film. For example, given the words """"booty,"""" """"Andy"""" and """"pieced,"""" the answer would be Beauty and the Beast." 7 tequila, mucking, purred to kill a mockingbird 20190512 Every answer today is a familiar two-word phrase in 6 letters. The middle 4 letters spell a word. I'll give you the middle 4 letters and a clue to the 6-letter phrase. You tell me the phrase. 1 Metal container, INCA tin can 20190512 Every answer today is a familiar two-word phrase in 6 letters. The middle 4 letters spell a word. I'll give you the middle 4 letters and a clue to the 6-letter phrase. You tell me the phrase. 2 Idle, TRES at rest 20190512 Every answer today is a familiar two-word phrase in 6 letters. The middle 4 letters spell a word. I'll give you the middle 4 letters and a clue to the 6-letter phrase. You tell me the phrase. 3 Recently, FLAT of late 20190512 Every answer today is a familiar two-word phrase in 6 letters. The middle 4 letters spell a word. I'll give you the middle 4 letters and a clue to the 6-letter phrase. You tell me the phrase. 4 "Latin phrase meaning """"for the time being,"""" ROTE" pro tem 20190512 Every answer today is a familiar two-word phrase in 6 letters. The middle 4 letters spell a word. I'll give you the middle 4 letters and a clue to the 6-letter phrase. You tell me the phrase. 5 Kind of tie, LIPO clip on 20190512 Every answer today is a familiar two-word phrase in 6 letters. The middle 4 letters spell a word. I'll give you the middle 4 letters and a clue to the 6-letter phrase. You tell me the phrase. 6 Last two words of the Pledge of Allegiance, ORAL for all 20190512 Every answer today is a familiar two-word phrase in 6 letters. The middle 4 letters spell a word. I'll give you the middle 4 letters and a clue to the 6-letter phrase. You tell me the phrase. 7 Resurrect, as an old issue, RAGU drag up 20190512 Every answer today is a familiar two-word phrase in 6 letters. The middle 4 letters spell a word. I'll give you the middle 4 letters and a clue to the 6-letter phrase. You tell me the phrase. 8 Zodiac symbol, HERA the ram 20190512 Every answer today is a familiar two-word phrase in 6 letters. The middle 4 letters spell a word. I'll give you the middle 4 letters and a clue to the 6-letter phrase. You tell me the phrase. 9 Old telephone company, ABEL Ma Bell 20190512 Every answer today is a familiar two-word phrase in 6 letters. The middle 4 letters spell a word. I'll give you the middle 4 letters and a clue to the 6-letter phrase. You tell me the phrase. 10 Mars, for example, ARGO war god 20190512 Every answer today is a familiar two-word phrase in 6 letters. The middle 4 letters spell a word. I'll give you the middle 4 letters and a clue to the 6-letter phrase. You tell me the phrase. 11 Crawl, OSLO go slow 20190512 Every answer today is a familiar two-word phrase in 6 letters. The middle 4 letters spell a word. I'll give you the middle 4 letters and a clue to the 6-letter phrase. You tell me the phrase. 12 Angrily rebuke, NAPA snap at 20190512 Every answer today is a familiar two-word phrase in 6 letters. The middle 4 letters spell a word. I'll give you the middle 4 letters and a clue to the 6-letter phrase. You tell me the phrase. 13 Some china, EASE tea set 20190512 Every answer today is a familiar two-word phrase in 6 letters. The middle 4 letters spell a word. I'll give you the middle 4 letters and a clue to the 6-letter phrase. You tell me the phrase. 14 Variety of poker, OLDE hold ̃em 20140119 " ""Name a word that" 1 berg, bucket, breaker ice 20140119 " ""Name a word that" 2 brow, ball, bank eye 20140119 " ""Name a word that" 3 bed, breeze, biscuit sea 20140119 " ""Name a word that" 4 board, bird, blower snow 20140119 " ""Name a word that" 5 bubble, box, bar soap 20140119 " ""Name a word that" 6 bottle, balloon, buffalo water 20140119 " ""Name a word that" 7 bean, beret, briar green 20140119 " ""Name a word that" 8 bulb, beer, brigade light 20140119 " ""Name a word that" 9 beet, baby, bowl sugar 20140119 " ""Name a word that" 10 bagger, beetle, bomb carpet 20140119 " ""Name a word that" 11 blade, bag, bone shoulder 20100221 "This is a game of categories. For each one, name something in it starting with each of the letters """"L-I-G-H-T"""" in any order. For example, if the category is two-syllable girls' names, the answer might be """"Lila,"""" """"Irene,"""" """"Georgette,"""" """"Holly"""" and """"Tina.""""" 1 Sports Luge, Ice skating, Giant slalom, Hockey, Tennis 20100221 "This is a game of categories. For each one, name something in it starting with each of the letters """"L-I-G-H-T"""" in any order. For example, if the category is two-syllable girls' names, the answer might be """"Lila,"""" """"Irene,"""" """"Georgette,"""" """"Holly"""" and """"Tina.""""" 2 Parts of a car Lights, Ignition, Gas tank, Headlights, Trunk 20100221 "This is a game of categories. For each one, name something in it starting with each of the letters """"L-I-G-H-T"""" in any order. For example, if the category is two-syllable girls' names, the answer might be """"Lila,"""" """"Irene,"""" """"Georgette,"""" """"Holly"""" and """"Tina.""""" 4 License plate Horn, Tachometer 20100221 "This is a game of categories. For each one, name something in it starting with each of the letters """"L-I-G-H-T"""" in any order. For example, if the category is two-syllable girls' names, the answer might be """"Lila,"""" """"Irene,"""" """"Georgette,"""" """"Holly"""" and """"Tina.""""" 5 Islands Long Island, Ireland, Guam, Haiti, Tahiti, Iceland, Hawaii, Tasmania 20100221 "This is a game of categories. For each one, name something in it starting with each of the letters """"L-I-G-H-T"""" in any order. For example, if the category is two-syllable girls' names, the answer might be """"Lila,"""" """"Irene,"""" """"Georgette,"""" """"Holly"""" and """"Tina.""""" 6 Things in a hotel room Lamp, Ice bucket, Gideon Bible, Heater, Tub, Iro, Glasses, Hanger, Towel, Ironing board 20100221 "This is a game of categories. For each one, name something in it starting with each of the letters """"L-I-G-H-T"""" in any order. For example, if the category is two-syllable girls' names, the answer might be """"Lila,"""" """"Irene,"""" """"Georgette,"""" """"Holly"""" and """"Tina.""""" 7 Synonyms for big Large, Immense, Giant, Huge, Tremendous 20111127 "Each answer is a compound word or a familiar two-word phrase containing the consecutive letters H-H-O, as in H2O. For example, given the clue """"Dutch province containing Rotterdam and The Hague,"""" the answer would be """"South Holland.""""" 1 a period from noon to 1:00 for many workers lunch hour 20111127 "Each answer is a compound word or a familiar two-word phrase containing the consecutive letters H-H-O, as in H2O. For example, given the clue """"Dutch province containing Rotterdam and The Hague,"""" the answer would be """"South Holland.""""" 2 a familiar name for a bar at a golf club nineteenth hole 20111127 "Each answer is a compound word or a familiar two-word phrase containing the consecutive letters H-H-O, as in H2O. For example, given the clue """"Dutch province containing Rotterdam and The Hague,"""" the answer would be """"South Holland.""""" 3 place to put a worm fish hook 20111127 "Each answer is a compound word or a familiar two-word phrase containing the consecutive letters H-H-O, as in H2O. For example, given the clue """"Dutch province containing Rotterdam and The Hague,"""" the answer would be """"South Holland.""""" 4 domicile along a sandy shore beach house 20111127 "Each answer is a compound word or a familiar two-word phrase containing the consecutive letters H-H-O, as in H2O. For example, given the clue """"Dutch province containing Rotterdam and The Hague,"""" the answer would be """"South Holland.""""" 5 low-priced lodging for vacationing kids youth hostel 20111127 "Each answer is a compound word or a familiar two-word phrase containing the consecutive letters H-H-O, as in H2O. For example, given the clue """"Dutch province containing Rotterdam and The Hague,"""" the answer would be """"South Holland.""""" 6 "if you were haughty or contemptuous, someone might tell you to """"get of your ... """"" high horse 20111127 "Each answer is a compound word or a familiar two-word phrase containing the consecutive letters H-H-O, as in H2O. For example, given the clue """"Dutch province containing Rotterdam and The Hague,"""" the answer would be """"South Holland.""""" 7 former name for belize british honduras 20111127 "Each answer is a compound word or a familiar two-word phrase containing the consecutive letters H-H-O, as in H2O. For example, given the clue """"Dutch province containing Rotterdam and The Hague,"""" the answer would be """"South Holland.""""" 8 "words before """"and charity"""" in an old expression" faith, hope 20111127 "Each answer is a compound word or a familiar two-word phrase containing the consecutive letters H-H-O, as in H2O. For example, given the clue """"Dutch province containing Rotterdam and The Hague,"""" the answer would be """"South Holland.""""" 9 something fed to livestock to make them get bigger artificially growth hormone 20111127 "Each answer is a compound word or a familiar two-word phrase containing the consecutive letters H-H-O, as in H2O. For example, given the clue """"Dutch province containing Rotterdam and The Hague,"""" the answer would be """"South Holland.""""" 10 rosh hashanah or passover jewish holiday 20111127 "Each answer is a compound word or a familiar two-word phrase containing the consecutive letters H-H-O, as in H2O. For example, given the clue """"Dutch province containing Rotterdam and The Hague,"""" the answer would be """"South Holland.""""" 11 elasticized stockings stretch hose or hosiery 20111127 "Each answer is a compound word or a familiar two-word phrase containing the consecutive letters H-H-O, as in H2O. For example, given the clue """"Dutch province containing Rotterdam and The Hague,"""" the answer would be """"South Holland.""""" 12 "translation of """"cum laude""""" with honors 20111127 "Each answer is a compound word or a familiar two-word phrase containing the consecutive letters H-H-O, as in H2O. For example, given the clue """"Dutch province containing Rotterdam and The Hague,"""" the answer would be """"South Holland.""""" 13 "exclamation from the seven dwarfs as ""off to work they go""" heigh-ho 20170618 Every answer today is a word that contains part of the human body in the exact middle. Ex. Group of Native Americans, starting with T and ending with E --> TRIBE, which contains RIB between the T and the E1. E ____ Y Mournful poem2. W ____ Y Tired3. A ____ G Very sore4. EL ____ SE Geometrical shape5. LE ____ ES Beans and peas6. RE ____ AL Opposing argument in a debate7. OB ____ TE Out of date8. RA ____ SS Quality of a harsh voice9. FLA ____ ESS Showy display 1 E ____ Y Mournful poem ElegY 20170618 Every answer today is a word that contains part of the human body in the exact middle. Ex. Group of Native Americans, starting with T and ending with E --> TRIBE, which contains RIB between the T and the E1. E ____ Y Mournful poem2. W ____ Y Tired3. A ____ G Very sore4. EL ____ SE Geometrical shape5. LE ____ ES Beans and peas6. RE ____ AL Opposing argument in a debate7. OB ____ TE Out of date8. RA ____ SS Quality of a harsh voice9. FLA ____ ESS Showy display 2 W ____ Y Tired WearY Tired 20170618 Every answer today is a word that contains part of the human body in the exact middle. Ex. Group of Native Americans, starting with T and ending with E --> TRIBE, which contains RIB between the T and the E1. E ____ Y Mournful poem2. W ____ Y Tired3. A ____ G Very sore4. EL ____ SE Geometrical shape5. LE ____ ES Beans and peas6. RE ____ AL Opposing argument in a debate7. OB ____ TE Out of date8. RA ____ SS Quality of a harsh voice9. FLA ____ ESS Showy display 3 A ____ G Very sore AchinG Very sore 20170618 Every answer today is a word that contains part of the human body in the exact middle. Ex. Group of Native Americans, starting with T and ending with E --> TRIBE, which contains RIB between the T and the E1. E ____ Y Mournful poem2. W ____ Y Tired3. A ____ G Very sore4. EL ____ SE Geometrical shape5. LE ____ ES Beans and peas6. RE ____ AL Opposing argument in a debate7. OB ____ TE Out of date8. RA ____ SS Quality of a harsh voice9. FLA ____ ESS Showy display 4 EL ____ SE Geometrical shape ELlipSE Geometrical shape 20170618 Every answer today is a word that contains part of the human body in the exact middle. Ex. Group of Native Americans, starting with T and ending with E --> TRIBE, which contains RIB between the T and the E1. E ____ Y Mournful poem2. W ____ Y Tired3. A ____ G Very sore4. EL ____ SE Geometrical shape5. LE ____ ES Beans and peas6. RE ____ AL Opposing argument in a debate7. OB ____ TE Out of date8. RA ____ SS Quality of a harsh voice9. FLA ____ ESS Showy display 5 LE ____ ES Beans and peas LEgumES Beans and peas 20170618 Every answer today is a word that contains part of the human body in the exact middle. Ex. Group of Native Americans, starting with T and ending with E --> TRIBE, which contains RIB between the T and the E1. E ____ Y Mournful poem2. W ____ Y Tired3. A ____ G Very sore4. EL ____ SE Geometrical shape5. LE ____ ES Beans and peas6. RE ____ AL Opposing argument in a debate7. OB ____ TE Out of date8. RA ____ SS Quality of a harsh voice9. FLA ____ ESS Showy display 6 RE ____ AL Opposing argument in a debate REbuttAL Opposing argument in a debate 20170618 Every answer today is a word that contains part of the human body in the exact middle. Ex. Group of Native Americans, starting with T and ending with E --> TRIBE, which contains RIB between the T and the E1. E ____ Y Mournful poem2. W ____ Y Tired3. A ____ G Very sore4. EL ____ SE Geometrical shape5. LE ____ ES Beans and peas6. RE ____ AL Opposing argument in a debate7. OB ____ TE Out of date8. RA ____ SS Quality of a harsh voice9. FLA ____ ESS Showy display 7 OB ____ TE Out of date OBsoleTE Out of date 20170618 Every answer today is a word that contains part of the human body in the exact middle. Ex. Group of Native Americans, starting with T and ending with E --> TRIBE, which contains RIB between the T and the E1. E ____ Y Mournful poem2. W ____ Y Tired3. A ____ G Very sore4. EL ____ SE Geometrical shape5. LE ____ ES Beans and peas6. RE ____ AL Opposing argument in a debate7. OB ____ TE Out of date8. RA ____ SS Quality of a harsh voice9. FLA ____ ESS Showy display 8 RA ____ SS Quality of a harsh voice RAspineSS Quality of a harsh voice 20200329 I'm going to give you a word and an extra letter. Anagram everything into a new word ... in which the added letter is silent. Example: MOW + B --> WOMB1. GRIN + W2. TOGS + H3. FINE + K4. TENS + C5. HANS + G6. BRIDE + S7. NOMAD + L8. DOTER + B9. PEDLAR + O10. SIMONE + T11. RETINT + W12. AIMING + E 13. NO SHAME + D14. CHEAP MAN + G 1 grin + w wring 20200329 I'm going to give you a word and an extra letter. Anagram everything into a new word ... in which the added letter is silent. Example: MOW + B --> WOMB1. GRIN + W2. TOGS + H3. FINE + K4. TENS + C5. HANS + G6. BRIDE + S7. NOMAD + L8. DOTER + B9. PEDLAR + O10. SIMONE + T11. RETINT + W12. AIMING + E 13. NO SHAME + D14. CHEAP MAN + G 1 grin + w wring 20200329 I'm going to give you a word and an extra letter. Anagram everything into a new word ... in which the added letter is silent. Example: MOW + B --> WOMB1. GRIN + W2. TOGS + H3. FINE + K4. TENS + C5. HANS + G6. BRIDE + S7. NOMAD + L8. DOTER + B9. PEDLAR + O10. SIMONE + T11. RETINT + W12. AIMING + E 13. NO SHAME + D14. CHEAP MAN + G 2 togs + h phost 20200329 I'm going to give you a word and an extra letter. Anagram everything into a new word ... in which the added letter is silent. Example: MOW + B --> WOMB1. GRIN + W2. TOGS + H3. FINE + K4. TENS + C5. HANS + G6. BRIDE + S7. NOMAD + L8. DOTER + B9. PEDLAR + O10. SIMONE + T11. RETINT + W12. AIMING + E 13. NO SHAME + D14. CHEAP MAN + G 3 fine + k knife 20200329 I'm going to give you a word and an extra letter. Anagram everything into a new word ... in which the added letter is silent. Example: MOW + B --> WOMB1. GRIN + W2. TOGS + H3. FINE + K4. TENS + C5. HANS + G6. BRIDE + S7. NOMAD + L8. DOTER + B9. PEDLAR + O10. SIMONE + T11. RETINT + W12. AIMING + E 13. NO SHAME + D14. CHEAP MAN + G 4 tens + c scent 20200329 I'm going to give you a word and an extra letter. Anagram everything into a new word ... in which the added letter is silent. Example: MOW + B --> WOMB1. GRIN + W2. TOGS + H3. FINE + K4. TENS + C5. HANS + G6. BRIDE + S7. NOMAD + L8. DOTER + B9. PEDLAR + O10. SIMONE + T11. RETINT + W12. AIMING + E 13. NO SHAME + D14. CHEAP MAN + G 5 hans + g gnash 20200329 I'm going to give you a word and an extra letter. Anagram everything into a new word ... in which the added letter is silent. Example: MOW + B --> WOMB1. GRIN + W2. TOGS + H3. FINE + K4. TENS + C5. HANS + G6. BRIDE + S7. NOMAD + L8. DOTER + B9. PEDLAR + O10. SIMONE + T11. RETINT + W12. AIMING + E 13. NO SHAME + D14. CHEAP MAN + G 6 bride + s debris 20200329 I'm going to give you a word and an extra letter. Anagram everything into a new word ... in which the added letter is silent. Example: MOW + B --> WOMB1. GRIN + W2. TOGS + H3. FINE + K4. TENS + C5. HANS + G6. BRIDE + S7. NOMAD + L8. DOTER + B9. PEDLAR + O10. SIMONE + T11. RETINT + W12. AIMING + E 13. NO SHAME + D14. CHEAP MAN + G 7 nomad + l nice to eat 20200329 I'm going to give you a word and an extra letter. Anagram everything into a new word ... in which the added letter is silent. Example: MOW + B --> WOMB1. GRIN + W2. TOGS + H3. FINE + K4. TENS + C5. HANS + G6. BRIDE + S7. NOMAD + L8. DOTER + B9. PEDLAR + O10. SIMONE + T11. RETINT + W12. AIMING + E 13. NO SHAME + D14. CHEAP MAN + G 8 retint + w written 20200329 I'm going to give you a word and an extra letter. Anagram everything into a new word ... in which the added letter is silent. Example: MOW + B --> WOMB1. GRIN + W2. TOGS + H3. FINE + K4. TENS + C5. HANS + G6. BRIDE + S7. NOMAD + L8. DOTER + B9. PEDLAR + O10. SIMONE + T11. RETINT + W12. AIMING + E 13. NO SHAME + D14. CHEAP MAN + G 9 no shame + d handsome 20200329 I'm going to give you a word and an extra letter. Anagram everything into a new word ... in which the added letter is silent. Example: MOW + B --> WOMB1. GRIN + W2. TOGS + H3. FINE + K4. TENS + C5. HANS + G6. BRIDE + S7. NOMAD + L8. DOTER + B9. PEDLAR + O10. SIMONE + T11. RETINT + W12. AIMING + E 13. NO SHAME + D14. CHEAP MAN + G 10 doter + b debtor 20200329 I'm going to give you a word and an extra letter. Anagram everything into a new word ... in which the added letter is silent. Example: MOW + B --> WOMB1. GRIN + W2. TOGS + H3. FINE + K4. TENS + C5. HANS + G6. BRIDE + S7. NOMAD + L8. DOTER + B9. PEDLAR + O10. SIMONE + T11. RETINT + W12. AIMING + E 13. NO SHAME + D14. CHEAP MAN + G 11 pedlar + o leopard 20200329 I'm going to give you a word and an extra letter. Anagram everything into a new word ... in which the added letter is silent. Example: MOW + B --> WOMB1. GRIN + W2. TOGS + H3. FINE + K4. TENS + C5. HANS + G6. BRIDE + S7. NOMAD + L8. DOTER + B9. PEDLAR + O10. SIMONE + T11. RETINT + W12. AIMING + E 13. NO SHAME + D14. CHEAP MAN + G 12 simone + t moisten 20200329 I'm going to give you a word and an extra letter. Anagram everything into a new word ... in which the added letter is silent. Example: MOW + B --> WOMB1. GRIN + W2. TOGS + H3. FINE + K4. TENS + C5. HANS + G6. BRIDE + S7. NOMAD + L8. DOTER + B9. PEDLAR + O10. SIMONE + T11. RETINT + W12. AIMING + E 13. NO SHAME + D14. CHEAP MAN + G 13 aiming + e imagine 20200329 I'm going to give you a word and an extra letter. Anagram everything into a new word ... in which the added letter is silent. Example: MOW + B --> WOMB1. GRIN + W2. TOGS + H3. FINE + K4. TENS + C5. HANS + G6. BRIDE + S7. NOMAD + L8. DOTER + B9. PEDLAR + O10. SIMONE + T11. RETINT + W12. AIMING + E 13. NO SHAME + D14. CHEAP MAN + G 14 cheap man + g champagne 20140706 Two clues will be given for two five-letter answers. Move the middle letter of the first answer to the end of the word to get the second answer. Example: A weapon that's thrown; a tire in the trunk. Answer: spear/spare 1 start; any living creature begin; being 20140706 Two clues will be given for two five-letter answers. Move the middle letter of the first answer to the end of the word to get the second answer. Example: A weapon that's thrown; a tire in the trunk. Answer: spear/spare 2 bringer of presents at christmas; the devil santa; satan 20140706 Two clues will be given for two five-letter answers. Move the middle letter of the first answer to the end of the word to get the second answer. Example: A weapon that's thrown; a tire in the trunk. Answer: spear/spare 3 handed out as cards; the area around the mouth of a river dealt; delta 20140706 Two clues will be given for two five-letter answers. Move the middle letter of the first answer to the end of the word to get the second answer. Example: A weapon that's thrown; a tire in the trunk. Answer: spear/spare 4 discovered; melted in french cooking found; fondu 20140706 Two clues will be given for two five-letter answers. Move the middle letter of the first answer to the end of the word to get the second answer. Example: A weapon that's thrown; a tire in the trunk. Answer: spear/spare 5 to change; to warn alter; alert 20140706 Two clues will be given for two five-letter answers. Move the middle letter of the first answer to the end of the word to get the second answer. Example: A weapon that's thrown; a tire in the trunk. Answer: spear/spare 6 a series of links; and where shanghai is chain; china 20140706 Two clues will be given for two five-letter answers. Move the middle letter of the first answer to the end of the word to get the second answer. Example: A weapon that's thrown; a tire in the trunk. Answer: spear/spare 7 wash cycle after wash; up and about rinse; risen 20140706 Two clues will be given for two five-letter answers. Move the middle letter of the first answer to the end of the word to get the second answer. Example: A weapon that's thrown; a tire in the trunk. Answer: spear/spare 8 a monthly bill; a spy in the bible cable; caleb 20140706 Two clues will be given for two five-letter answers. Move the middle letter of the first answer to the end of the word to get the second answer. Example: A weapon that's thrown; a tire in the trunk. Answer: spear/spare 9 astrological sign after leo; strengths virgo; vigor 20140706 Two clues will be given for two five-letter answers. Move the middle letter of the first answer to the end of the word to get the second answer. Example: A weapon that's thrown; a tire in the trunk. Answer: spear/spare 10 "a fur wrap; an early italian pope, the first to be called """"the great""""" stole; st. leo 20090705 "The theme today is """"cow."""" Every answer is a familiar two-word phrase or name in which the first word starts with """"C-O"""" and the second word starts with """"W."""" For example, if the clue is """"person who can fix a hard drive in 10 minutes,"""" the answer would be, """"computer whiz.""""" 1 former conflict between communist countries and the west cold war 20090705 "The theme today is """"cow."""" Every answer is a familiar two-word phrase or name in which the first word starts with """"C-O"""" and the second word starts with """"W."""" For example, if the clue is """"person who can fix a hard drive in 10 minutes,"""" the answer would be, """"computer whiz.""""" 2 volume that has all of the plays and poems of shakespeare, for example complete/collected works 20090705 "The theme today is """"cow."""" Every answer is a familiar two-word phrase or name in which the first word starts with """"C-O"""" and the second word starts with """"W."""" For example, if the clue is """"person who can fix a hard drive in 10 minutes,"""" the answer would be, """"computer whiz.""""" 3 person who works for (david) letterman or jon stewart comedy writer 20090705 "The theme today is """"cow."""" Every answer is a familiar two-word phrase or name in which the first word starts with """"C-O"""" and the second word starts with """"W."""" For example, if the clue is """"person who can fix a hard drive in 10 minutes,"""" the answer would be, """"computer whiz.""""" 4 music genre for dolly parton and garth brooks country-western 20090705 "The theme today is """"cow."""" Every answer is a familiar two-word phrase or name in which the first word starts with """"C-O"""" and the second word starts with """"W."""" For example, if the clue is """"person who can fix a hard drive in 10 minutes,"""" the answer would be, """"computer whiz.""""" 5 woman who serves drinks before dinner (typically in a restaurant) cocktail waitress 20090705 "The theme today is """"cow."""" Every answer is a familiar two-word phrase or name in which the first word starts with """"C-O"""" and the second word starts with """"W."""" For example, if the clue is """"person who can fix a hard drive in 10 minutes,"""" the answer would be, """"computer whiz.""""" 6 a term made up of two or more shorter parts, such as highway or football compound word 20090705 "The theme today is """"cow."""" Every answer is a familiar two-word phrase or name in which the first word starts with """"C-O"""" and the second word starts with """"W."""" For example, if the clue is """"person who can fix a hard drive in 10 minutes,"""" the answer would be, """"computer whiz.""""" 7 a gun carried in one's pocket, for example concealed weapon 20090705 "The theme today is """"cow."""" Every answer is a familiar two-word phrase or name in which the first word starts with """"C-O"""" and the second word starts with """"W."""" For example, if the clue is """"person who can fix a hard drive in 10 minutes,"""" the answer would be, """"computer whiz.""""" 8 a vehicle that carried pioneers across the west conestoga/covered wagon 20090705 "The theme today is """"cow."""" Every answer is a familiar two-word phrase or name in which the first word starts with """"C-O"""" and the second word starts with """"W."""" For example, if the clue is """"person who can fix a hard drive in 10 minutes,"""" the answer would be, """"computer whiz.""""" 9 "edgar allan poe poem with """"the""""" the conqueror worm 20090705 "The theme today is """"cow."""" Every answer is a familiar two-word phrase or name in which the first word starts with """"C-O"""" and the second word starts with """"W."""" For example, if the clue is """"person who can fix a hard drive in 10 minutes,"""" the answer would be, """"computer whiz.""""" 10 a hard hat construction worker 20090705 "The theme today is """"cow."""" Every answer is a familiar two-word phrase or name in which the first word starts with """"C-O"""" and the second word starts with """"W."""" For example, if the clue is """"person who can fix a hard drive in 10 minutes,"""" the answer would be, """"computer whiz.""""" 11 the ftc (federal trade commission), for example consumer watchdog 20090705 "The theme today is """"cow."""" Every answer is a familiar two-word phrase or name in which the first word starts with """"C-O"""" and the second word starts with """"W."""" For example, if the clue is """"person who can fix a hard drive in 10 minutes,"""" the answer would be, """"computer whiz.""""" 12 brand of non-dairy dessert topping cool whip 20090705 "The theme today is """"cow."""" Every answer is a familiar two-word phrase or name in which the first word starts with """"C-O"""" and the second word starts with """"W."""" For example, if the clue is """"person who can fix a hard drive in 10 minutes,"""" the answer would be, """"computer whiz.""""" 13 good material for an electrical circuit copper wire 20090705 "The theme today is """"cow."""" Every answer is a familiar two-word phrase or name in which the first word starts with """"C-O"""" and the second word starts with """"W."""" For example, if the clue is """"person who can fix a hard drive in 10 minutes,"""" the answer would be, """"computer whiz.""""" 14 a person who takes a prize for entering a competition contest winner 20191020 Every answer today is something you see in a hotel room — always, or at least often. Name these things from their anagrams, minus one letter. 1 debt + t bed 20191020 Every answer today is something you see in a hotel room — always, or at least often. Name these things from their anagrams, minus one letter. 2 maple + e lamp 20191020 Every answer today is something you see in a hotel room — always, or at least often. Name these things from their anagrams, minus one letter. 3 opals + l soap 20191020 Every answer today is something you see in a hotel room — always, or at least often. Name these things from their anagrams, minus one letter. 4 feast + t safe 20191020 Every answer today is something you see in a hotel room — always, or at least often. Name these things from their anagrams, minus one letter. 5 robin + b iron 20191020 Every answer today is something you see in a hotel room — always, or at least often. Name these things from their anagrams, minus one letter. 6 achier + e chair 20191020 Every answer today is something you see in a hotel room — always, or at least often. Name these things from their anagrams, minus one letter. 7 lowest + s towel 20191020 Every answer today is something you see in a hotel room — always, or at least often. Name these things from their anagrams, minus one letter. 8 nibble + n bible 20191020 Every answer today is something you see in a hotel room — always, or at least often. Name these things from their anagrams, minus one letter. 9 howlers + l shower 20191020 Every answer today is something you see in a hotel room — always, or at least often. Name these things from their anagrams, minus one letter. 10 midbrain + d minibar 20191020 Every answer today is something you see in a hotel room — always, or at least often. Name these things from their anagrams, minus one letter. 11 teamsters + e mattress 20191020 Every answer today is something you see in a hotel room — always, or at least often. Name these things from their anagrams, minus one letter. 12 longevities + g television 20141116 Every answer today is a rhyming two-word phrase in which each word has two syllables and the first word ends with a doubled consonant plus the letter Y. 1 a hilarious rabbit funny bunny 20141116 Every answer today is a rhyming two-word phrase in which each word has two syllables and the first word ends with a doubled consonant plus the letter Y. 2 a taxi driver who complains a lot crabby cabby 20141116 Every answer today is a rhyming two-word phrase in which each word has two syllables and the first word ends with a doubled consonant plus the letter Y. 3 an odorous sandwich shop smelly deli 20141116 Every answer today is a rhyming two-word phrase in which each word has two syllables and the first word ends with a doubled consonant plus the letter Y. 4 a haughty London policeman snobby Bobby 20141116 Every answer today is a rhyming two-word phrase in which each word has two syllables and the first word ends with a doubled consonant plus the letter Y. 5 a stupid '60s guy with long hair and a tie-dyed shirt dippy hippie 20141116 Every answer today is a rhyming two-word phrase in which each word has two syllables and the first word ends with a doubled consonant plus the letter Y. 6 wifie's overweight spouse tubby hubby 20141116 Every answer today is a rhyming two-word phrase in which each word has two syllables and the first word ends with a doubled consonant plus the letter Y. 7 an enbalmed body that is in a pyramid and in poor condition crummy mummy 20181216 "This Sunday's New York Times has a special 14-page section called """"Puzzle Mania,"""" which I edited, containing puzzles of all sorts. Here's the start of one I made called """"Silence Is Golden."""" I'm going to give you a word and a letter of the alphabet. Rearrange everything to make a new word in which the added letter is silent." 1 MUD + B DUMB 20181216 "This Sunday's New York Times has a special 14-page section called """"Puzzle Mania,"""" which I edited, containing puzzles of all sorts. Here's the start of one I made called """"Silence Is Golden."""" I'm going to give you a word and a letter of the alphabet. Rearrange everything to make a new word in which the added letter is silent." 2 PAR + W WRAP 20181216 "This Sunday's New York Times has a special 14-page section called """"Puzzle Mania,"""" which I edited, containing puzzles of all sorts. Here's the start of one I made called """"Silence Is Golden."""" I'm going to give you a word and a letter of the alphabet. Rearrange everything to make a new word in which the added letter is silent." 3 FINE + K KNIFE 20181216 "This Sunday's New York Times has a special 14-page section called """"Puzzle Mania,"""" which I edited, containing puzzles of all sorts. Here's the start of one I made called """"Silence Is Golden."""" I'm going to give you a word and a letter of the alphabet. Rearrange everything to make a new word in which the added letter is silent." 4 RODS + W SWORD 20181216 "This Sunday's New York Times has a special 14-page section called """"Puzzle Mania,"""" which I edited, containing puzzles of all sorts. Here's the start of one I made called """"Silence Is Golden."""" I'm going to give you a word and a letter of the alphabet. Rearrange everything to make a new word in which the added letter is silent." 5 ALMS + P PSALM 20181216 "This Sunday's New York Times has a special 14-page section called """"Puzzle Mania,"""" which I edited, containing puzzles of all sorts. Here's the start of one I made called """"Silence Is Golden."""" I'm going to give you a word and a letter of the alphabet. Rearrange everything to make a new word in which the added letter is silent." 7 * SNARE + W ANSWER 20200119 I'm going to give you two four-letter words. Anagram each of them to get two new words that rhyme. 1 Lure, Polo rule, pool 20200119 I'm going to give you two four-letter words. Anagram each of them to get two new words that rhyme. 2 Opus, Polo soup, loop 20200119 I'm going to give you two four-letter words. Anagram each of them to get two new words that rhyme. 3 Knee, Neal keen, lean 20200119 I'm going to give you two four-letter words. Anagram each of them to get two new words that rhyme. 4 Rage, Ripe gear, pier 20200119 I'm going to give you two four-letter words. Anagram each of them to get two new words that rhyme. 5 Luge, Hose glue, shoe 20200119 I'm going to give you two four-letter words. Anagram each of them to get two new words that rhyme. 6 Flue, Laud fuel, dual 20200920 I'm going to give you some six-letter words. For each one, change one of the consonants to a vowel to make another familiar six-letter word. 1 AVENgE avenue 20200920 I'm going to give you some six-letter words. For each one, change one of the consonants to a vowel to make another familiar six-letter word. 2 DIpPER diaper 20200920 I'm going to give you some six-letter words. For each one, change one of the consonants to a vowel to make another familiar six-letter word. 3 mIGHTY eighty 20200920 I'm going to give you some six-letter words. For each one, change one of the consonants to a vowel to make another familiar six-letter word. 4 PREPPY prepay 20200920 I'm going to give you some six-letter words. For each one, change one of the consonants to a vowel to make another familiar six-letter word. 5 gROUSE arouse 20200920 I'm going to give you some six-letter words. For each one, change one of the consonants to a vowel to make another familiar six-letter word. 6 MAdDEN maiden 20200920 I'm going to give you some six-letter words. For each one, change one of the consonants to a vowel to make another familiar six-letter word. 7 BIKINg bikini 20200920 I'm going to give you some six-letter words. For each one, change one of the consonants to a vowel to make another familiar six-letter word. 8 CARtER career 20200920 I'm going to give you some six-letter words. For each one, change one of the consonants to a vowel to make another familiar six-letter word. 9 gRANGE orange 20200920 I'm going to give you some six-letter words. For each one, change one of the consonants to a vowel to make another familiar six-letter word. 10 DECEnT deceit 20200920 I'm going to give you some six-letter words. For each one, change one of the consonants to a vowel to make another familiar six-letter word. 11 STANCh stance 20200920 I'm going to give you some six-letter words. For each one, change one of the consonants to a vowel to make another familiar six-letter word. 12 StANCE seance 20160724 When I give you three words starting with the letters H, O and T, you tell me a word that can follow each of mine to complete a compound word or a familiar two-word phrase. 1 haunted, opera, tree house 20160724 When I give you three words starting with the letters H, O and T, you tell me a word that can follow each of mine to complete a compound word or a familiar two-word phrase. 2 head, one night, taxi stand 20160724 When I give you three words starting with the letters H, O and T, you tell me a word that can follow each of mine to complete a compound word or a familiar two-word phrase. 3 hospital, oyster, twin beds 20160724 When I give you three words starting with the letters H, O and T, you tell me a word that can follow each of mine to complete a compound word or a familiar two-word phrase. 4 honor, onion, tootsie roll/roll 20160724 When I give you three words starting with the letters H, O and T, you tell me a word that can follow each of mine to complete a compound word or a familiar two-word phrase. 5 high, old, trade school 20160724 When I give you three words starting with the letters H, O and T, you tell me a word that can follow each of mine to complete a compound word or a familiar two-word phrase. 6 hand, ox, tea cart 20160724 When I give you three words starting with the letters H, O and T, you tell me a word that can follow each of mine to complete a compound word or a familiar two-word phrase. 7 horse, oxford, tennis shoe 20160724 When I give you three words starting with the letters H, O and T, you tell me a word that can follow each of mine to complete a compound word or a familiar two-word phrase. 8 hail, oliver, tomb stone/stone 20160724 When I give you three words starting with the letters H, O and T, you tell me a word that can follow each of mine to complete a compound word or a familiar two-word phrase. 9 hot water, oxygen, think tank 20160724 When I give you three words starting with the letters H, O and T, you tell me a word that can follow each of mine to complete a compound word or a familiar two-word phrase. 10 horse, one act, triple play 20160724 When I give you three words starting with the letters H, O and T, you tell me a word that can follow each of mine to complete a compound word or a familiar two-word phrase. 11 handle, open, topless bar 20160724 When I give you three words starting with the letters H, O and T, you tell me a word that can follow each of mine to complete a compound word or a familiar two-word phrase. 12 head, ocean, trash can liner 20160724 When I give you three words starting with the letters H, O and T, you tell me a word that can follow each of mine to complete a compound word or a familiar two-word phrase. 13 hen, opposition, tupper ware party 20200816 Every answer today is the name of a major foreign city that is not the capital of its country. I'll give you anagrams. You name the cities. 1 ROCK, Ireland Cork 20200816 Every answer today is the name of a major foreign city that is not the capital of its country. I'll give you anagrams. You name the cities. 2 AVENGE, Switzerland Geneva 20200816 Every answer today is the name of a major foreign city that is not the capital of its country. I'll give you anagrams. You name the cities. 3 GOALS, Nigeria Lagos 20200816 Every answer today is the name of a major foreign city that is not the capital of its country. I'll give you anagrams. You name the cities. 4 REDDENS, Germany Dresden 20200816 Every answer today is the name of a major foreign city that is not the capital of its country. I'll give you anagrams. You name the cities. 5 PLANES, Italy Naples 20200816 Every answer today is the name of a major foreign city that is not the capital of its country. I'll give you anagrams. You name the cities. 6 DRAMAS, India Madras 20200816 Every answer today is the name of a major foreign city that is not the capital of its country. I'll give you anagrams. You name the cities. 7 ONLY, France Lyon 20200816 Every answer today is the name of a major foreign city that is not the capital of its country. I'll give you anagrams. You name the cities. 8 NEW PART, Belgium Antwerp 20200816 Every answer today is the name of a major foreign city that is not the capital of its country. I'll give you anagrams. You name the cities. 9 VALIANCE, Spain Valencia 20200816 Every answer today is the name of a major foreign city that is not the capital of its country. I'll give you anagrams. You name the cities. 10 APROPOS, Japan Sapporo 20130915 Every answer is a familiar two-word phrase or name with the initials M.N. For example: Be pleasant in order to appease someone? Answer: Make Nice. 1 Michelle Robinson, for the current First Lady Maiden Name 20130915 Every answer is a familiar two-word phrase or name with the initials M.N. For example: Be pleasant in order to appease someone? Answer: Make Nice. 2 Something you put around a bed to prevent being bitten Mosquito Net 20130915 Every answer is a familiar two-word phrase or name with the initials M.N. For example: Be pleasant in order to appease someone? Answer: Make Nice. 3 Do, re, mi, fa, sol, la, or ti Musical Note 20130915 Every answer is a familiar two-word phrase or name with the initials M.N. For example: Be pleasant in order to appease someone? Answer: Make Nice. 4 It's not nice to fool her, according to ... Mother Nature 20130915 Every answer is a familiar two-word phrase or name with the initials M.N. For example: Be pleasant in order to appease someone? Answer: Make Nice. 5 A hard-shelled snack item from Hawaii Macadamia Nut 20130915 Every answer is a familiar two-word phrase or name with the initials M.N. For example: Be pleasant in order to appease someone? Answer: Make Nice. 6 Where a compass needle points Magnetic North 20130915 Every answer is a familiar two-word phrase or name with the initials M.N. For example: Be pleasant in order to appease someone? Answer: Make Nice. 7 Oread Mountain Nymph 20130915 Every answer is a familiar two-word phrase or name with the initials M.N. For example: Be pleasant in order to appease someone? Answer: Make Nice. 8 Women's Wimbledon champion nine times Martina Navratilova 20130915 Every answer is a familiar two-word phrase or name with the initials M.N. For example: Be pleasant in order to appease someone? Answer: Make Nice. 9 Former Panamanian dictator overthrown in a 1989 U.S. invasion ManuelNoriega 20130915 Every answer is a familiar two-word phrase or name with the initials M.N. For example: Be pleasant in order to appease someone? Answer: Make Nice. 10 One of the Monkees Michael Nesmith 20130915 Every answer is a familiar two-word phrase or name with the initials M.N. For example: Be pleasant in order to appease someone? Answer: Make Nice. 11 Daily newspaper in San Jose Mercury News 20130915 Every answer is a familiar two-word phrase or name with the initials M.N. For example: Be pleasant in order to appease someone? Answer: Make Nice. 12 When ESPN broadcasts football games Monday Night 20190811 Every answer is a familiar phrase in the form of ___ and ___ in which the first word starts with the letter C. I'll give you the last word of the phrase, you tell me the missing first word. 1 MOUSE cat 20190811 Every answer is a familiar phrase in the form of ___ and ___ in which the first word starts with the letter C. I'll give you the last word of the phrase, you tell me the missing first word. 2 ROBBERS cops 20190811 Every answer is a familiar phrase in the form of ___ and ___ in which the first word starts with the letter C. I'll give you the last word of the phrase, you tell me the missing first word. 3 SAUCER cup 20190811 Every answer is a familiar phrase in the form of ___ and ___ in which the first word starts with the letter C. I'll give you the last word of the phrase, you tell me the missing first word. 4 TIE coat 20190811 Every answer is a familiar phrase in the form of ___ and ___ in which the first word starts with the letter C. I'll give you the last word of the phrase, you tell me the missing first word. 5 BULL cock 20190811 Every answer is a familiar phrase in the form of ___ and ___ in which the first word starts with the letter C. I'll give you the last word of the phrase, you tell me the missing first word. 6 CARRY cash (not conceal) 20190811 Every answer is a familiar phrase in the form of ___ and ___ in which the first word starts with the letter C. I'll give you the last word of the phrase, you tell me the missing first word. 7 ALE cakes 20190811 Every answer is a familiar phrase in the form of ___ and ___ in which the first word starts with the letter C. I'll give you the last word of the phrase, you tell me the missing first word. 8 PUNISHMENT crime 20190811 Every answer is a familiar phrase in the form of ___ and ___ in which the first word starts with the letter C. I'll give you the last word of the phrase, you tell me the missing first word. 9 DAGGER cloak 20190811 Every answer is a familiar phrase in the form of ___ and ___ in which the first word starts with the letter C. I'll give you the last word of the phrase, you tell me the missing first word. 10 UNUSUAL cruel 20190811 Every answer is a familiar phrase in the form of ___ and ___ in which the first word starts with the letter C. I'll give you the last word of the phrase, you tell me the missing first word. 11 EFFECT cause 20190811 Every answer is a familiar phrase in the form of ___ and ___ in which the first word starts with the letter C. I'll give you the last word of the phrase, you tell me the missing first word. 12 WHEY curds 20190811 Every answer is a familiar phrase in the form of ___ and ___ in which the first word starts with the letter C. I'll give you the last word of the phrase, you tell me the missing first word. 13 WESTERN country 20190811 Every answer is a familiar phrase in the form of ___ and ___ in which the first word starts with the letter C. I'll give you the last word of the phrase, you tell me the missing first word. 14 BURN crash 20190811 Every answer is a familiar phrase in the form of ___ and ___ in which the first word starts with the letter C. I'll give you the last word of the phrase, you tell me the missing first word. 15 GOWN cap 20190811 Every answer is a familiar phrase in the form of ___ and ___ in which the first word starts with the letter C. I'll give you the last word of the phrase, you tell me the missing first word. 16 SUGAR cream 20190811 Every answer is a familiar phrase in the form of ___ and ___ in which the first word starts with the letter C. I'll give you the last word of the phrase, you tell me the missing first word. 17 RELEASE catch 20190811 Every answer is a familiar phrase in the form of ___ and ___ in which the first word starts with the letter C. I'll give you the last word of the phrase, you tell me the missing first word. 18 POLLUX Castor 20190811 Every answer is a familiar phrase in the form of ___ and ___ in which the first word starts with the letter C. I'll give you the last word of the phrase, you tell me the missing first word. 19 DRIVER car 20190811 Every answer is a familiar phrase in the form of ___ and ___ in which the first word starts with the letter C. I'll give you the last word of the phrase, you tell me the missing first word. 20 GET IT come 20100529 "For each word you are given, change one letter to get a common name for a newspaper. For example, given """"tomes"""" the answer is """"Times.""""" 1 tribute Tribune 20100529 "For each word you are given, change one letter to get a common name for a newspaper. For example, given """"tomes"""" the answer is """"Times.""""" 2 posh Post 20100529 "For each word you are given, change one letter to get a common name for a newspaper. For example, given """"tomes"""" the answer is """"Times.""""" 3 hedger Ledger 20100529 "For each word you are given, change one letter to get a common name for a newspaper. For example, given """"tomes"""" the answer is """"Times.""""" 4 glove Globe 20100529 "For each word you are given, change one letter to get a common name for a newspaper. For example, given """"tomes"""" the answer is """"Times.""""" 5 resister Register 20100529 "For each word you are given, change one letter to get a common name for a newspaper. For example, given """"tomes"""" the answer is """"Times.""""" 6 empress Express 20100529 "For each word you are given, change one letter to get a common name for a newspaper. For example, given """"tomes"""" the answer is """"Times.""""" 7 would World 20100529 "For each word you are given, change one letter to get a common name for a newspaper. For example, given """"tomes"""" the answer is """"Times.""""" 8 reword Record 20100529 "For each word you are given, change one letter to get a common name for a newspaper. For example, given """"tomes"""" the answer is """"Times.""""" 9 deporter Reporter 20100529 "For each word you are given, change one letter to get a common name for a newspaper. For example, given """"tomes"""" the answer is """"Times.""""" 10 lender Leader 20100529 "For each word you are given, change one letter to get a common name for a newspaper. For example, given """"tomes"""" the answer is """"Times.""""" 11 onion "Union (Liane mentioned, """"There's already a newspaper named The Onion!"""")" 20100529 "For each word you are given, change one letter to get a common name for a newspaper. For example, given """"tomes"""" the answer is """"Times.""""" 12 preps Press 20100529 "For each word you are given, change one letter to get a common name for a newspaper. For example, given """"tomes"""" the answer is """"Times.""""" 13 nets News 20100529 "For each word you are given, change one letter to get a common name for a newspaper. For example, given """"tomes"""" the answer is """"Times.""""" 14 sue Sun 20100529 "For each word you are given, change one letter to get a common name for a newspaper. For example, given """"tomes"""" the answer is """"Times.""""" 15 stay Star 20120916 "Every answer is a familiar phrase in the form of """"___ and ___."""" You'll be given the two missing words, each with a letter removed, and you give the phrases. For example, given """"lot and fund,"""" the answer would be """"lost and found.""""" 1 BAT and WITCH BAIT and SWITCH 20120916 "Every answer is a familiar phrase in the form of """"___ and ___."""" You'll be given the two missing words, each with a letter removed, and you give the phrases. For example, given """"lot and fund,"""" the answer would be """"lost and found.""""" 2 FAT and LOSE FAST and LOOSE 20120916 "Every answer is a familiar phrase in the form of """"___ and ___."""" You'll be given the two missing words, each with a letter removed, and you give the phrases. For example, given """"lot and fund,"""" the answer would be """"lost and found.""""" 3 PINT and LICK POINT and CLICK 20120916 "Every answer is a familiar phrase in the form of """"___ and ___."""" You'll be given the two missing words, each with a letter removed, and you give the phrases. For example, given """"lot and fund,"""" the answer would be """"lost and found.""""" 4 CASH and BUN CRASH and BURN 20120916 "Every answer is a familiar phrase in the form of """"___ and ___."""" You'll be given the two missing words, each with a letter removed, and you give the phrases. For example, given """"lot and fund,"""" the answer would be """"lost and found.""""" 5 TOOT and AIL TOOTH and NAIL 20120916 "Every answer is a familiar phrase in the form of """"___ and ___."""" You'll be given the two missing words, each with a letter removed, and you give the phrases. For example, given """"lot and fund,"""" the answer would be """"lost and found.""""" 6 READ and BUTTE BREAD and BUTTER 20120916 "Every answer is a familiar phrase in the form of """"___ and ___."""" You'll be given the two missing words, each with a letter removed, and you give the phrases. For example, given """"lot and fund,"""" the answer would be """"lost and found.""""" 7 HEAR and SOL HEART and SOUL 20120916 "Every answer is a familiar phrase in the form of """"___ and ___."""" You'll be given the two missing words, each with a letter removed, and you give the phrases. For example, given """"lot and fund,"""" the answer would be """"lost and found.""""" 8 PEN and HUT OPEN and SHUT 20120916 "Every answer is a familiar phrase in the form of """"___ and ___."""" You'll be given the two missing words, each with a letter removed, and you give the phrases. For example, given """"lot and fund,"""" the answer would be """"lost and found.""""" 9 SKI and ONES SKIN and BONES 20120916 "Every answer is a familiar phrase in the form of """"___ and ___."""" You'll be given the two missing words, each with a letter removed, and you give the phrases. For example, given """"lot and fund,"""" the answer would be """"lost and found.""""" 10 WAS and WAR WASH and WEAR 20120916 "Every answer is a familiar phrase in the form of """"___ and ___."""" You'll be given the two missing words, each with a letter removed, and you give the phrases. For example, given """"lot and fund,"""" the answer would be """"lost and found.""""" 11 TIE and GAIN TIME and AGAIN 20120916 "Every answer is a familiar phrase in the form of """"___ and ___."""" You'll be given the two missing words, each with a letter removed, and you give the phrases. For example, given """"lot and fund,"""" the answer would be """"lost and found.""""" 12 TARS and STRIPS STARS and STRIPES 20120916 "Every answer is a familiar phrase in the form of """"___ and ___."""" You'll be given the two missing words, each with a letter removed, and you give the phrases. For example, given """"lot and fund,"""" the answer would be """"lost and found.""""" 13 SON and DANE SONG and DANCE 20120916 "Every answer is a familiar phrase in the form of """"___ and ___."""" You'll be given the two missing words, each with a letter removed, and you give the phrases. For example, given """"lot and fund,"""" the answer would be """"lost and found.""""" 14 CASE and DEIST CEASE and DESIST 20131201 "Today's puzzle is a game of categories based on the word """"thank,"""" in honor of Thanksgiving weekend. For each category, name something beginning with each of the letters T, H, A, N and K. For example, if the category were """"U.S. States,"""" you might say Tennessee, Hawaii, Alaska, Nevada and Kentucky." 1 U.S. presidents Truman, Hoover, Adams, Nixon, Kennedy 20131201 "Today's puzzle is a game of categories based on the word """"thank,"""" in honor of Thanksgiving weekend. For each category, name something beginning with each of the letters T, H, A, N and K. For example, if the category were """"U.S. States,"""" you might say Tennessee, Hawaii, Alaska, Nevada and Kentucky." 2 fruits tangerine, honeydew, apple, nectarine, kiwi 20131201 "Today's puzzle is a game of categories based on the word """"thank,"""" in honor of Thanksgiving weekend. For each category, name something beginning with each of the letters T, H, A, N and K. For example, if the category were """"U.S. States,"""" you might say Tennessee, Hawaii, Alaska, Nevada and Kentucky." 3 African capitals Tunis, Harare, Addis Ababa, Nairobi, Kinshasa 20131201 "Today's puzzle is a game of categories based on the word """"thank,"""" in honor of Thanksgiving weekend. For each category, name something beginning with each of the letters T, H, A, N and K. For example, if the category were """"U.S. States,"""" you might say Tennessee, Hawaii, Alaska, Nevada and Kentucky." 4 uncapitalized four-letter words whose first and last letters are the same trot, hash, aura, naan, kook 20100925 "Every answer is a familiar two-word phrase, in which the first word starts with P-I and the second word starts with T. For example, given """"path taken by early settlers in the West,"""" the answer would be """"pioneer trail.""""" 1 a vehicle, say from chevrolet or ford, that might carry a load of dirt pick-up truck 20100925 "Every answer is a familiar two-word phrase, in which the first word starts with P-I and the second word starts with T. For example, given """"path taken by early settlers in the West,"""" the answer would be """"pioneer trail.""""" 2 a place to have lunch outdoors, such as at a highway rest area picnic table 20100925 "Every answer is a familiar two-word phrase, in which the first word starts with P-I and the second word starts with T. For example, given """"path taken by early settlers in the West,"""" the answer would be """"pioneer trail.""""" 3 chit-chat by a couple in bed pillow talk 20100925 "Every answer is a familiar two-word phrase, in which the first word starts with P-I and the second word starts with T. For example, given """"path taken by early settlers in the West,"""" the answer would be """"pioneer trail.""""" 4 producer of needles and cones pine tree 20100925 "Every answer is a familiar two-word phrase, in which the first word starts with P-I and the second word starts with T. For example, given """"path taken by early settlers in the West,"""" the answer would be """"pioneer trail.""""" 5 sausage, pepperoni or anchovies pizza topping 20100925 "Every answer is a familiar two-word phrase, in which the first word starts with P-I and the second word starts with T. For example, given """"path taken by early settlers in the West,"""" the answer would be """"pioneer trail.""""" 6 baking accessory that can be used as a frisbee pie tin 20100925 "Every answer is a familiar two-word phrase, in which the first word starts with P-I and the second word starts with T. For example, given """"path taken by early settlers in the West,"""" the answer would be """"pioneer trail.""""" 7 prince albert product for smokers pipe tobacco 20100925 "Every answer is a familiar two-word phrase, in which the first word starts with P-I and the second word starts with T. For example, given """"path taken by early settlers in the West,"""" the answer would be """"pioneer trail.""""" 8 person repairing a large musical instrument piano tuner 20100925 "Every answer is a familiar two-word phrase, in which the first word starts with P-I and the second word starts with T. For example, given """"path taken by early settlers in the West,"""" the answer would be """"pioneer trail.""""" 9 major part of an old-fashioned tv picture tube 20100925 "Every answer is a familiar two-word phrase, in which the first word starts with P-I and the second word starts with T. For example, given """"path taken by early settlers in the West,"""" the answer would be """"pioneer trail.""""" 10 girls' hair-do with braids and bows at the side of the head pig tails 20100925 "Every answer is a familiar two-word phrase, in which the first word starts with P-I and the second word starts with T. For example, given """"path taken by early settlers in the West,"""" the answer would be """"pioneer trail.""""" 11 condition in which the bottoms of legs point inward pigeon-toes 20100925 "Every answer is a familiar two-word phrase, in which the first word starts with P-I and the second word starts with T. For example, given """"path taken by early settlers in the West,"""" the answer would be """"pioneer trail.""""" 12 baked dessert, using a fruit filling from dole pineapple tart or turnover 20100925 "Every answer is a familiar two-word phrase, in which the first word starts with P-I and the second word starts with T. For example, given """"path taken by early settlers in the West,"""" the answer would be """"pioneer trail.""""" 13 it may be in a chest at the bottom of the sea. pirate treasure 20200531 I'm going to give you some words. Anagram each one to make a new word. And as a help, the first three letters of my word will be the first three letters of your answer. Ex. Sublet --> Subtle1. Platen2. Thereat 3. Declaim4. Parleys 5. Magnate 6. Trained (hyph.)7. Martial 8. Chorale 9. Senescent 10. Pedantries Last week's challenge: This challenge came from listener Patrick Berry, of Jasper, Ala. Think of a well-known European city in seven letters. If you remove the third letter, you'll get a two-word phrase describing what you must do to win a race. If instead you remove the fourth letter, you'll get a two-word phrase describing what you can't do to win a race. What's the city? 1 Platen Planet 20200531 I'm going to give you some words. Anagram each one to make a new word. And as a help, the first three letters of my word will be the first three letters of your answer. Ex. Sublet --> Subtle1. Platen2. Thereat 3. Declaim4. Parleys 5. Magnate 6. Trained (hyph.)7. Martial 8. Chorale 9. Senescent 10. Pedantries Last week's challenge: This challenge came from listener Patrick Berry, of Jasper, Ala. Think of a well-known European city in seven letters. If you remove the third letter, you'll get a two-word phrase describing what you must do to win a race. If instead you remove the fourth letter, you'll get a two-word phrase describing what you can't do to win a race. What's the city? 2 Thereat Theater or theatre 20200531 I'm going to give you some words. Anagram each one to make a new word. And as a help, the first three letters of my word will be the first three letters of your answer. Ex. Sublet --> Subtle1. Platen2. Thereat 3. Declaim4. Parleys 5. Magnate 6. Trained (hyph.)7. Martial 8. Chorale 9. Senescent 10. Pedantries Last week's challenge: This challenge came from listener Patrick Berry, of Jasper, Ala. Think of a well-known European city in seven letters. If you remove the third letter, you'll get a two-word phrase describing what you must do to win a race. If instead you remove the fourth letter, you'll get a two-word phrase describing what you can't do to win a race. What's the city? 3 Declaim Decimal 20200531 I'm going to give you some words. Anagram each one to make a new word. And as a help, the first three letters of my word will be the first three letters of your answer. Ex. Sublet --> Subtle1. Platen2. Thereat 3. Declaim4. Parleys 5. Magnate 6. Trained (hyph.)7. Martial 8. Chorale 9. Senescent 10. Pedantries Last week's challenge: This challenge came from listener Patrick Berry, of Jasper, Ala. Think of a well-known European city in seven letters. If you remove the third letter, you'll get a two-word phrase describing what you must do to win a race. If instead you remove the fourth letter, you'll get a two-word phrase describing what you can't do to win a race. What's the city? 4 Parleys Parsley 20200531 I'm going to give you some words. Anagram each one to make a new word. And as a help, the first three letters of my word will be the first three letters of your answer. Ex. Sublet --> Subtle1. Platen2. Thereat 3. Declaim4. Parleys 5. Magnate 6. Trained (hyph.)7. Martial 8. Chorale 9. Senescent 10. Pedantries Last week's challenge: This challenge came from listener Patrick Berry, of Jasper, Ala. Think of a well-known European city in seven letters. If you remove the third letter, you'll get a two-word phrase describing what you must do to win a race. If instead you remove the fourth letter, you'll get a two-word phrase describing what you can't do to win a race. What's the city? 5 Magnate Magenta 20200531 I'm going to give you some words. Anagram each one to make a new word. And as a help, the first three letters of my word will be the first three letters of your answer. Ex. Sublet --> Subtle1. Platen2. Thereat 3. Declaim4. Parleys 5. Magnate 6. Trained (hyph.)7. Martial 8. Chorale 9. Senescent 10. Pedantries Last week's challenge: This challenge came from listener Patrick Berry, of Jasper, Ala. Think of a well-known European city in seven letters. If you remove the third letter, you'll get a two-word phrase describing what you must do to win a race. If instead you remove the fourth letter, you'll get a two-word phrase describing what you can't do to win a race. What's the city? 6 Trained Trade-in 20200531 I'm going to give you some words. Anagram each one to make a new word. And as a help, the first three letters of my word will be the first three letters of your answer. Ex. Sublet --> Subtle1. Platen2. Thereat 3. Declaim4. Parleys 5. Magnate 6. Trained (hyph.)7. Martial 8. Chorale 9. Senescent 10. Pedantries Last week's challenge: This challenge came from listener Patrick Berry, of Jasper, Ala. Think of a well-known European city in seven letters. If you remove the third letter, you'll get a two-word phrase describing what you must do to win a race. If instead you remove the fourth letter, you'll get a two-word phrase describing what you can't do to win a race. What's the city? 7 Martial Marital 20200531 I'm going to give you some words. Anagram each one to make a new word. And as a help, the first three letters of my word will be the first three letters of your answer. Ex. Sublet --> Subtle1. Platen2. Thereat 3. Declaim4. Parleys 5. Magnate 6. Trained (hyph.)7. Martial 8. Chorale 9. Senescent 10. Pedantries Last week's challenge: This challenge came from listener Patrick Berry, of Jasper, Ala. Think of a well-known European city in seven letters. If you remove the third letter, you'll get a two-word phrase describing what you must do to win a race. If instead you remove the fourth letter, you'll get a two-word phrase describing what you can't do to win a race. What's the city? 8 Chorale Cholera 20200531 I'm going to give you some words. Anagram each one to make a new word. And as a help, the first three letters of my word will be the first three letters of your answer. Ex. Sublet --> Subtle1. Platen2. Thereat 3. Declaim4. Parleys 5. Magnate 6. Trained (hyph.)7. Martial 8. Chorale 9. Senescent 10. Pedantries Last week's challenge: This challenge came from listener Patrick Berry, of Jasper, Ala. Think of a well-known European city in seven letters. If you remove the third letter, you'll get a two-word phrase describing what you must do to win a race. If instead you remove the fourth letter, you'll get a two-word phrase describing what you can't do to win a race. What's the city? 9 Senescent Sentences 20200531 I'm going to give you some words. Anagram each one to make a new word. And as a help, the first three letters of my word will be the first three letters of your answer. Ex. Sublet --> Subtle1. Platen2. Thereat 3. Declaim4. Parleys 5. Magnate 6. Trained (hyph.)7. Martial 8. Chorale 9. Senescent 10. Pedantries Last week's challenge: This challenge came from listener Patrick Berry, of Jasper, Ala. Think of a well-known European city in seven letters. If you remove the third letter, you'll get a two-word phrase describing what you must do to win a race. If instead you remove the fourth letter, you'll get a two-word phrase describing what you can't do to win a race. What's the city? 10 Pedantries Pedestria 20190407 Every answer is the name of a country, for which I'll give you an anagram. For example, PANE + L = NEPAL. The added letter at the end of the anagram will always be the last letter in the name of the country. 1 INCH + A China 20190407 Every answer is the name of a country, for which I'll give you an anagram. For example, PANE + L = NEPAL. The added letter at the end of the anagram will always be the last letter in the name of the country. 2 TAIL + Y Italy 20190407 Every answer is the name of a country, for which I'll give you an anagram. For example, PANE + L = NEPAL. The added letter at the end of the anagram will always be the last letter in the name of the country. 3 VITAL + A Latvia 20190407 Every answer is the name of a country, for which I'll give you an anagram. For example, PANE + L = NEPAL. The added letter at the end of the anagram will always be the last letter in the name of the country. 4 ARISE + L Israel 20190407 Every answer is the name of a country, for which I'll give you an anagram. For example, PANE + L = NEPAL. The added letter at the end of the anagram will always be the last letter in the name of the country. 5 LINEAR + D Ireland 20190407 Every answer is the name of a country, for which I'll give you an anagram. For example, PANE + L = NEPAL. The added letter at the end of the anagram will always be the last letter in the name of the country. 6 LOOMING + A Mongolia 20190407 Every answer is the name of a country, for which I'll give you an anagram. For example, PANE + L = NEPAL. The added letter at the end of the anagram will always be the last letter in the name of the country. 7 LIVES ON + A Slovenia 20190407 Every answer is the name of a country, for which I'll give you an anagram. For example, PANE + L = NEPAL. The added letter at the end of the anagram will always be the last letter in the name of the country. 8 ACROSTIC + A Costa Rica 20140720 Two clues will be provided. The first is for a brand name that ends in the letter S and sounds like it's plural. Change the first letter to spell a new word that is plural and answers the second clue. Example: tennis shoes, places to sleep; the answer would be Keds and beds. 1 brand of diapers; people sleeping outside at night Pampers Campers 20140720 Two clues will be provided. The first is for a brand name that ends in the letter S and sounds like it's plural. Change the first letter to spell a new word that is plural and answers the second clue. Example: tennis shoes, places to sleep; the answer would be Keds and beds. 2 brand of diapers; carriages Huggies Buggies 20140720 Two clues will be provided. The first is for a brand name that ends in the letter S and sounds like it's plural. Change the first letter to spell a new word that is plural and answers the second clue. Example: tennis shoes, places to sleep; the answer would be Keds and beds. 3 brand of candy bar; old-fashioned pants Snickers Knickers 20140720 Two clues will be provided. The first is for a brand name that ends in the letter S and sounds like it's plural. Change the first letter to spell a new word that is plural and answers the second clue. Example: tennis shoes, places to sleep; the answer would be Keds and beds. 4 brand of pants; places to store athlete's clothes Dockers Lockers 20140720 Two clues will be provided. The first is for a brand name that ends in the letter S and sounds like it's plural. Change the first letter to spell a new word that is plural and answers the second clue. Example: tennis shoes, places to sleep; the answer would be Keds and beds. 5 brand of candy; groups of buffalo Nerds Herds 20140720 Two clues will be provided. The first is for a brand name that ends in the letter S and sounds like it's plural. Change the first letter to spell a new word that is plural and answers the second clue. Example: tennis shoes, places to sleep; the answer would be Keds and beds. 6 brand of facial tissue; what police slap on suspects' wrists Puffs Cuffs 20140720 Two clues will be provided. The first is for a brand name that ends in the letter S and sounds like it's plural. Change the first letter to spell a new word that is plural and answers the second clue. Example: tennis shoes, places to sleep; the answer would be Keds and beds. 7 brand of cream-filled cakes; people who pose for pictures Yodels Models 20091205 "Every answer is the name of a famous person. The person's last name is a plural noun that is part of a category. The clues are the person's first name and the category of the last name. The player gives the full name of the person. For example, if the clues are """"Britney"""" and """"weapons,"""" the answer is """"Britney Spears,"""" because spears are weapons." 1 Joyce Relatives Joyce Brothers 20091205 "Every answer is the name of a famous person. The person's last name is a plural noun that is part of a category. The clues are the person's first name and the category of the last name. The player gives the full name of the person. For example, if the clues are """"Britney"""" and """"weapons,"""" the answer is """"Britney Spears,"""" because spears are weapons." 2 Jonathan Seasons Jonathan Winters 20091205 "Every answer is the name of a famous person. The person's last name is a plural noun that is part of a category. The clues are the person's first name and the category of the last name. The player gives the full name of the person. For example, if the clues are """"Britney"""" and """"weapons,"""" the answer is """"Britney Spears,"""" because spears are weapons." 3 Joan Waterways Joan Rivers 20091205 "Every answer is the name of a famous person. The person's last name is a plural noun that is part of a category. The clues are the person's first name and the category of the last name. The player gives the full name of the person. For example, if the clues are """"Britney"""" and """"weapons,"""" the answer is """"Britney Spears,"""" because spears are weapons." 4 Mel Waterways Mel Brooks 20091205 "Every answer is the name of a famous person. The person's last name is a plural noun that is part of a category. The clues are the person's first name and the category of the last name. The player gives the full name of the person. For example, if the clues are """"Britney"""" and """"weapons,"""" the answer is """"Britney Spears,"""" because spears are weapons." 5 Beverly Window Parts Beverly Sills 20091205 "Every answer is the name of a famous person. The person's last name is a plural noun that is part of a category. The clues are the person's first name and the category of the last name. The player gives the full name of the person. For example, if the clues are """"Britney"""" and """"weapons,"""" the answer is """"Britney Spears,"""" because spears are weapons." 6 Jeremy Golf Clubs Jeremy Irons 20091205 "Every answer is the name of a famous person. The person's last name is a plural noun that is part of a category. The clues are the person's first name and the category of the last name. The player gives the full name of the person. For example, if the clues are """"Britney"""" and """"weapons,"""" the answer is """"Britney Spears,"""" because spears are weapons." 7 Tiger Golf Clubs Tiger Woods 20091205 "Every answer is the name of a famous person. The person's last name is a plural noun that is part of a category. The clues are the person's first name and the category of the last name. The player gives the full name of the person. For example, if the clues are """"Britney"""" and """"weapons,"""" the answer is """"Britney Spears,"""" because spears are weapons." 8 Naomi Electric Units Naomi Watts 20091205 "Every answer is the name of a famous person. The person's last name is a plural noun that is part of a category. The clues are the person's first name and the category of the last name. The player gives the full name of the person. For example, if the clues are """"Britney"""" and """"weapons,"""" the answer is """"Britney Spears,"""" because spears are weapons." 9 Alicia Piano Parts Alicia Keys 20091205 "Every answer is the name of a famous person. The person's last name is a plural noun that is part of a category. The clues are the person's first name and the category of the last name. The player gives the full name of the person. For example, if the clues are """"Britney"""" and """"weapons,"""" the answer is """"Britney Spears,"""" because spears are weapons." 10 Bert City Areas (HINT: or Rosa City Areas) Bert Parks (or Rosa Parks) 20091205 "Every answer is the name of a famous person. The person's last name is a plural noun that is part of a category. The clues are the person's first name and the category of the last name. The player gives the full name of the person. For example, if the clues are """"Britney"""" and """"weapons,"""" the answer is """"Britney Spears,"""" because spears are weapons." 11 George Injuries George Burns 20091205 "Every answer is the name of a famous person. The person's last name is a plural noun that is part of a category. The clues are the person's first name and the category of the last name. The player gives the full name of the person. For example, if the clues are """"Britney"""" and """"weapons,"""" the answer is """"Britney Spears,"""" because spears are weapons." 12 Stevie Injuries Stevie Nicks 20091205 "Every answer is the name of a famous person. The person's last name is a plural noun that is part of a category. The clues are the person's first name and the category of the last name. The player gives the full name of the person. For example, if the clues are """"Britney"""" and """"weapons,"""" the answer is """"Britney Spears,"""" because spears are weapons." 13 Jeff Engineering Projects Jeff Bridges 20091205 "Every answer is the name of a famous person. The person's last name is a plural noun that is part of a category. The clues are the person's first name and the category of the last name. The player gives the full name of the person. For example, if the clues are """"Britney"""" and """"weapons,"""" the answer is """"Britney Spears,"""" because spears are weapons." 14 Phil Metals Phil Silvers 20091205 "Every answer is the name of a famous person. The person's last name is a plural noun that is part of a category. The clues are the person's first name and the category of the last name. The player gives the full name of the person. For example, if the clues are """"Britney"""" and """"weapons,"""" the answer is """"Britney Spears,"""" because spears are weapons." 15 Hugh Football Terms Hugh Downs 20120715 "For each category, name something in the category starting with each of the letters in the word """"trail."""" For example, if the category were """"books of the Bible,"""" you might say Timothy, Ruth, Amos, Isaiah and Leviticus." 1 foreign makes of automobiles Toyota, Range Rover, Audi, Isuzu, Land Rover 20120715 "For each category, name something in the category starting with each of the letters in the word """"trail."""" For example, if the category were """"books of the Bible,"""" you might say Timothy, Ruth, Amos, Isaiah and Leviticus." 2 islands Tasmania, Rhodes, Antigua, Iceland, Lanai 20120715 "For each category, name something in the category starting with each of the letters in the word """"trail."""" For example, if the category were """"books of the Bible,"""" you might say Timothy, Ruth, Amos, Isaiah and Leviticus." 3 things you find in a motel room telephone, room, key, air conditioner, ice bucket, lamp 20120715 "For each category, name something in the category starting with each of the letters in the word """"trail."""" For example, if the category were """"books of the Bible,"""" you might say Timothy, Ruth, Amos, Isaiah and Leviticus." 4 seven-letter words that contain a Q tequila, racquet, antique, inquire, liqueur 20181230 "This is my annual """"New Names in the News Quiz."""" I'll name some people and things you probably never heard of until 2018, but who sprang to prominence during the past 12 months. You tell me who and what they are. My list was compiled with the help of Kathie Baker, who played one of my year-end quizzes in the past." 1 Greta Thunberg A young climate activist from Sweden who was just named Times Person of the Year 20181230 "This is my annual """"New Names in the News Quiz."""" I'll name some people and things you probably never heard of until 2018, but who sprang to prominence during the past 12 months. You tell me who and what they are. My list was compiled with the help of Kathie Baker, who played one of my year-end quizzes in the past." 2 Marie Yovanovitch Former ambassador to Ukraine 20181230 "This is my annual """"New Names in the News Quiz."""" I'll name some people and things you probably never heard of until 2018, but who sprang to prominence during the past 12 months. You tell me who and what they are. My list was compiled with the help of Kathie Baker, who played one of my year-end quizzes in the past." 3 Chasten Glezman Husband of Democratic Presidential candidate Pete Buttigieg 20181230 "This is my annual """"New Names in the News Quiz."""" I'll name some people and things you probably never heard of until 2018, but who sprang to prominence during the past 12 months. You tell me who and what they are. My list was compiled with the help of Kathie Baker, who played one of my year-end quizzes in the past." 4 Christina Koch and Jessica Meir The first two women to conduct an all-female spacewalk 20181230 "This is my annual """"New Names in the News Quiz."""" I'll name some people and things you probably never heard of until 2018, but who sprang to prominence during the past 12 months. You tell me who and what they are. My list was compiled with the help of Kathie Baker, who played one of my year-end quizzes in the past." 5 Naruhito The new emperor of Japan 20181230 "This is my annual """"New Names in the News Quiz."""" I'll name some people and things you probably never heard of until 2018, but who sprang to prominence during the past 12 months. You tell me who and what they are. My list was compiled with the help of Kathie Baker, who played one of my year-end quizzes in the past." 6 Lizzo American singer of œJuice and œTruth Hurts, a viral sleeper hit that topped the charts two years after its release 20181230 "This is my annual """"New Names in the News Quiz."""" I'll name some people and things you probably never heard of until 2018, but who sprang to prominence during the past 12 months. You tell me who and what they are. My list was compiled with the help of Kathie Baker, who played one of my year-end quizzes in the past." 7 Whakaari The New Zealand volcano in the news after its deadly Dec. 9th eruption 20200621 I'm going to read you some words. For each one, change the initial consonant or consonants to a new consonant or new consonants to get a new word that looks like it should rhyme with the original but doesn't. Every answer is unique. 1 Howl bowl 20200621 I'm going to read you some words. For each one, change the initial consonant or consonants to a new consonant or new consonants to get a new word that looks like it should rhyme with the original but doesn't. Every answer is unique. 2 Golf wolf 20200621 I'm going to read you some words. For each one, change the initial consonant or consonants to a new consonant or new consonants to get a new word that looks like it should rhyme with the original but doesn't. Every answer is unique. 3 Brave have 20200621 I'm going to read you some words. For each one, change the initial consonant or consonants to a new consonant or new consonants to get a new word that looks like it should rhyme with the original but doesn't. Every answer is unique. 4 Toad broad 20200621 I'm going to read you some words. For each one, change the initial consonant or consonants to a new consonant or new consonants to get a new word that looks like it should rhyme with the original but doesn't. Every answer is unique. 5 Gasp wasp 20141102 A series of names of famous people will be given. For each name, change either the first or last letter of the last name to a new letter, and rearrange the result to name a country. 1 pete seeger greece 20141102 A series of names of famous people will be given. For each name, change either the first or last letter of the last name to a new letter, and rearrange the result to name a country. 2 art carney france 20141102 A series of names of famous people will be given. For each name, change either the first or last letter of the last name to a new letter, and rearrange the result to name a country. 3 robert reich chile 20141102 A series of names of famous people will be given. For each name, change either the first or last letter of the last name to a new letter, and rearrange the result to name a country. 4 jennifer aniston estonia 20141102 A series of names of famous people will be given. For each name, change either the first or last letter of the last name to a new letter, and rearrange the result to name a country. 5 stephanie meyer yemen 20151025 This week's theme is SPAs. Every answer is a familiar two-word phrase or name in which the first word starts with S- and the second word starts with PA-. 1 twin city of Minneapolis St. Paul 20151025 This week's theme is SPAs. Every answer is a familiar two-word phrase or name in which the first word starts with S- and the second word starts with PA-. 2 John McCainas running mate in 2008 Sarah Palin 20151025 This week's theme is SPAs. Every answer is a familiar two-word phrase or name in which the first word starts with S- and the second word starts with PA-. 3 What good things come in, they say. small parcel/package 20151025 This week's theme is SPAs. Every answer is a familiar two-word phrase or name in which the first word starts with S- and the second word starts with PA-. 4 Rogers and Hammerstein musical, with the song Some Enchanted Evening South Pacific 20151025 This week's theme is SPAs. Every answer is a familiar two-word phrase or name in which the first word starts with S- and the second word starts with PA-. 5 money source for a presidential candidate Super PAC 20151025 This week's theme is SPAs. Every answer is a familiar two-word phrase or name in which the first word starts with S- and the second word starts with PA-. 6 somebody to practice boxing with sparring partner 20151025 This week's theme is SPAs. Every answer is a familiar two-word phrase or name in which the first word starts with S- and the second word starts with PA-. 7 lepton or muon subatomic particle 20151025 This week's theme is SPAs. Every answer is a familiar two-word phrase or name in which the first word starts with S- and the second word starts with PA-. 8 bit of facial hair on the lower lip soul patch 20151025 This week's theme is SPAs. Every answer is a familiar two-word phrase or name in which the first word starts with S- and the second word starts with PA-. 9 standard beer purchase at a grocery six pack 20151025 This week's theme is SPAs. Every answer is a familiar two-word phrase or name in which the first word starts with S- and the second word starts with PA-. 10 social occasion for men only stag party 20151025 This week's theme is SPAs. Every answer is a familiar two-word phrase or name in which the first word starts with S- and the second word starts with PA-. 11 contents of a can used by a graffiti artist spray paint 20081025 "Each answer is a familiar word, name or phrase that begins with the letter P and ends in X. For example, if the clue is """"showy flower,"""" the answer would be """"phlox.""""" 1 capital of Arizona Phoenix 20081025 "Each answer is a familiar word, name or phrase that begins with the letter P and ends in X. For example, if the clue is """"showy flower,"""" the answer would be """"phlox.""""" 2 Aunt Jemima pancake mix 20081025 "Each answer is a familiar word, name or phrase that begins with the letter P and ends in X. For example, if the clue is """"showy flower,"""" the answer would be """"phlox.""""" 3 hockey players sit after an penalty box 20081025 "Each answer is a familiar word, name or phrase that begins with the letter P and ends in X. For example, if the clue is """"showy flower,"""" the answer would be """"phlox.""""" 4 a body of troops phalanx 20081025 "Each answer is a familiar word, name or phrase that begins with the letter P and ends in X. For example, if the clue is """"showy flower,"""" the answer would be """"phlox.""""" 5 Castor's brother myth Pollux 20081025 "Each answer is a familiar word, name or phrase that begins with the letter P and ends in X. For example, if the clue is """"showy flower,"""" the answer would be """"phlox.""""" 6 maker of Cross Your Heart Playtex 20081025 "Each answer is a familiar word, name or phrase that begins with the letter P and ends in X. For example, if the clue is """"showy flower,"""" the answer would be """"phlox.""""" 7 what super is in superintelligent prefix 20130519 Every answer is a familiar two-word phrase or name in which the first word starts with H-A and the second word starts with T. 1 informal woman's garment tied behind the neck HAlter Top 20130519 Every answer is a familiar two-word phrase or name in which the first word starts with H-A and the second word starts with T. 2 a small item on a rod in the bathroom HAnd Towel 20130519 Every answer is a familiar two-word phrase or name in which the first word starts with H-A and the second word starts with T. 3 to perform a tricky surfing maneuver HAng Ten 20130519 Every answer is a familiar two-word phrase or name in which the first word starts with H-A and the second word starts with T. 4 international court situated in the Netherlands HAgue Tribunal 20130519 Every answer is a familiar two-word phrase or name in which the first word starts with H-A and the second word starts with T. 5 liable to be set off by the slightest thing, as a temper HAir Trigger 20130519 Every answer is a familiar two-word phrase or name in which the first word starts with H-A and the second word starts with T. 6 statement that's only partly accurate HALF Truth 20130519 Every answer is a familiar two-word phrase or name in which the first word starts with H-A and the second word starts with T. 7 theme song of the Roy Rogers Show HAppy Trails 20130519 Every answer is a familiar two-word phrase or name in which the first word starts with H-A and the second word starts with T. 8 woolen fabric used in traditional sports coats HArris Tweed 20130519 Every answer is a familiar two-word phrase or name in which the first word starts with H-A and the second word starts with T. 9 "words before """"... the Chief"""" in a presidential salute" HAil To 20130519 Every answer is a familiar two-word phrase or name in which the first word starts with H-A and the second word starts with T. 10 if you make things difficult for someone, you're giving him a ... HArry Truman 20130519 Every answer is a familiar two-word phrase or name in which the first word starts with H-A and the second word starts with T. 11 female abolitionist who helped run the underground railroad HArriet Tubman 20130519 Every answer is a familiar two-word phrase or name in which the first word starts with H-A and the second word starts with T. 12 president before Dwight Eisenhower HArry Truman 20130519 Every answer is a familiar two-word phrase or name in which the first word starts with H-A and the second word starts with T. 13 a symbol in many Twitter messages HAsh Tag 20120520 "This week's challenge is a twist on """"Characteristic Initials."""" We will gives clues for some famous people, past and present. The initial letters of the clues are also the initials of the answers. For example """"Wrote Sonnets"""" would be """"William Shakespeare.""""" 1 created snoopy Charles Shulz 20120520 "This week's challenge is a twist on """"Characteristic Initials."""" We will gives clues for some famous people, past and present. The initial letters of the clues are also the initials of the answers. For example """"Wrote Sonnets"""" would be """"William Shakespeare.""""" 2 helped beat slavery Harriet Beecher Stowe 20120520 "This week's challenge is a twist on """"Characteristic Initials."""" We will gives clues for some famous people, past and present. The initial letters of the clues are also the initials of the answers. For example """"Wrote Sonnets"""" would be """"William Shakespeare.""""" 3 turned america electric Thomas Edison 20120520 "This week's challenge is a twist on """"Characteristic Initials."""" We will gives clues for some famous people, past and present. The initial letters of the clues are also the initials of the answers. For example """"Wrote Sonnets"""" would be """"William Shakespeare.""""" 4 broke watergate Bob Woodward 20120520 "This week's challenge is a twist on """"Characteristic Initials."""" We will gives clues for some famous people, past and present. The initial letters of the clues are also the initials of the answers. For example """"Wrote Sonnets"""" would be """"William Shakespeare.""""" 5 readily wrote essays Ralph Waldo Emerson 20120520 "This week's challenge is a twist on """"Characteristic Initials."""" We will gives clues for some famous people, past and present. The initial letters of the clues are also the initials of the answers. For example """"Wrote Sonnets"""" would be """"William Shakespeare.""""" 6 constructed hotels Conrad Hilton 20120520 "This week's challenge is a twist on """"Characteristic Initials."""" We will gives clues for some famous people, past and present. The initial letters of the clues are also the initials of the answers. For example """"Wrote Sonnets"""" would be """"William Shakespeare.""""" 7 knows republicans Karl Rove 20120520 "This week's challenge is a twist on """"Characteristic Initials."""" We will gives clues for some famous people, past and present. The initial letters of the clues are also the initials of the answers. For example """"Wrote Sonnets"""" would be """"William Shakespeare.""""" 8 married schwarzenegger Maria Shriver 20120520 "This week's challenge is a twist on """"Characteristic Initials."""" We will gives clues for some famous people, past and present. The initial letters of the clues are also the initials of the answers. For example """"Wrote Sonnets"""" would be """"William Shakespeare.""""" 9 advised lovelorn Ann Landers 20120520 "This week's challenge is a twist on """"Characteristic Initials."""" We will gives clues for some famous people, past and present. The initial letters of the clues are also the initials of the answers. For example """"Wrote Sonnets"""" would be """"William Shakespeare.""""" 10 painted prolifically Pablo Picasso 20120520 "This week's challenge is a twist on """"Characteristic Initials."""" We will gives clues for some famous people, past and present. The initial letters of the clues are also the initials of the answers. For example """"Wrote Sonnets"""" would be """"William Shakespeare.""""" 11 authored classic detective Arthur Conan Doyle 20120520 "This week's challenge is a twist on """"Characteristic Initials."""" We will gives clues for some famous people, past and present. The initial letters of the clues are also the initials of the answers. For example """"Wrote Sonnets"""" would be """"William Shakespeare.""""" 12 supplies christmas Santa Claus 20120520 "This week's challenge is a twist on """"Characteristic Initials."""" We will gives clues for some famous people, past and present. The initial letters of the clues are also the initials of the answers. For example """"Wrote Sonnets"""" would be """"William Shakespeare.""""" 13 fashioned depression recovery Franklin Delano Roosevelt 20120520 "This week's challenge is a twist on """"Characteristic Initials."""" We will gives clues for some famous people, past and present. The initial letters of the clues are also the initials of the answers. For example """"Wrote Sonnets"""" would be """"William Shakespeare.""""" 14 asks trivia Alex Trebek 20100821 "Remove the vowel in the (unaccented) first syllable of one word to come up with a new word that's a syllable shorter but sounds the same. For example, """"succumb"""" becomes """"scum.""""" 1 a big event before the rose bowl; made in treatise with god parade; prayed 20100821 "Remove the vowel in the (unaccented) first syllable of one word to come up with a new word that's a syllable shorter but sounds the same. For example, """"succumb"""" becomes """"scum.""""" 2 author of walden; to fling or pitch thoreau; throw 20100821 "Remove the vowel in the (unaccented) first syllable of one word to come up with a new word that's a syllable shorter but sounds the same. For example, """"succumb"""" becomes """"scum.""""" 3 like music of bach, handel or vivaldi; flat out of money baroque; broke 20100821 "Remove the vowel in the (unaccented) first syllable of one word to come up with a new word that's a syllable shorter but sounds the same. For example, """"succumb"""" becomes """"scum.""""" 4 special forces cap; sound like a donkey beret; bray 20100821 "Remove the vowel in the (unaccented) first syllable of one word to come up with a new word that's a syllable shorter but sounds the same. For example, """"succumb"""" becomes """"scum.""""" 5 to hit, as two cars in an accident; bonnie's partner in crime collide; clyde 20100821 "Remove the vowel in the (unaccented) first syllable of one word to come up with a new word that's a syllable shorter but sounds the same. For example, """"succumb"""" becomes """"scum.""""" 6 awkward fellows, in slang; muscles of the buttocks, informally galoots; glutes 20100821 "Remove the vowel in the (unaccented) first syllable of one word to come up with a new word that's a syllable shorter but sounds the same. For example, """"succumb"""" becomes """"scum.""""" 7 messes around with other women; french-speaking part of belgium philanders; flanders 20181125 "This week's puzzle is called """"Big Mo'."""" Every answer is a word or name in which the second syllable is an accented """"mo.""""" 1 love or hate emotion 20181125 "This week's puzzle is called """"Big Mo'."""" Every answer is a word or name in which the second syllable is an accented """"mo.""""" 2 japanese robe kimono 20181125 "This week's puzzle is called """"Big Mo'."""" Every answer is a word or name in which the second syllable is an accented """"mo.""""" 3 home of pago-pago samoa 20181125 "This week's puzzle is called """"Big Mo'."""" Every answer is a word or name in which the second syllable is an accented """"mo.""""" 4 stuffed indian pastry samosa 20181125 "This week's puzzle is called """"Big Mo'."""" Every answer is a word or name in which the second syllable is an accented """"mo.""""" 5 cocktail often served at brunch mimosa 20181125 "This week's puzzle is called """"Big Mo'."""" Every answer is a word or name in which the second syllable is an accented """"mo.""""" 6 unable to move immobile 20181125 "This week's puzzle is called """"Big Mo'."""" Every answer is a word or name in which the second syllable is an accented """"mo.""""" 7 hullaballoo commotion 20181125 "This week's puzzle is called """"Big Mo'."""" Every answer is a word or name in which the second syllable is an accented """"mo.""""" 8 prestigious college in claremont, calif. pomona 20181125 "This week's puzzle is called """"Big Mo'."""" Every answer is a word or name in which the second syllable is an accented """"mo.""""" 9 reduction in rank demotion 20181125 "This week's puzzle is called """"Big Mo'."""" Every answer is a word or name in which the second syllable is an accented """"mo.""""" 10 former name for taiwan formosa 20181125 "This week's puzzle is called """"Big Mo'."""" Every answer is a word or name in which the second syllable is an accented """"mo.""""" 11 popular anti-diarrhea product imodium 20181125 "This week's puzzle is called """"Big Mo'."""" Every answer is a word or name in which the second syllable is an accented """"mo.""""" 12 train engine locomotive 20181125 "This week's puzzle is called """"Big Mo'."""" Every answer is a word or name in which the second syllable is an accented """"mo.""""" 13 formal statement in support of someone's qualifications testimonial 20181125 "This week's puzzle is called """"Big Mo'."""" Every answer is a word or name in which the second syllable is an accented """"mo.""""" 14 chip dip made with avocados guacamole 20160313 Every answer is a familiar two-word phrase or name in which the first word ends in -D and the second word starts with ST-. 1 our country United States 20160313 Every answer is a familiar two-word phrase or name in which the first word ends in -D and the second word starts with ST-. 2 shock jock, king of all media howard stern 20160313 Every answer is a familiar two-word phrase or name in which the first word ends in -D and the second word starts with ST-. 3 shirley temple, for example, when she was seven child star 20160313 Every answer is a familiar two-word phrase or name in which the first word ends in -D and the second word starts with ST-. 4 place to buy lps record store 20160313 Every answer is a familiar two-word phrase or name in which the first word ends in -D and the second word starts with ST-. 5 what lumunol can reveal at a crime scene blood stain 20160313 Every answer is a familiar two-word phrase or name in which the first word ends in -D and the second word starts with ST-. 6 informal name for the supplemental nutrition assistance program food stamps 20160313 Every answer is a familiar two-word phrase or name in which the first word ends in -D and the second word starts with ST-. 7 monetary system that the us abandoned in 1934 gold standard 20160313 Every answer is a familiar two-word phrase or name in which the first word ends in -D and the second word starts with ST-. 8 singer of the number 1 hits œmaggie may and œdo you think i'm sexy? rod stewart 20160313 Every answer is a familiar two-word phrase or name in which the first word ends in -D and the second word starts with ST-. 9 slang for whiskey or rum, but not beer or wine hard stuff 20160313 Every answer is a familiar two-word phrase or name in which the first word ends in -D and the second word starts with ST-. 10 someone working on a masterTMs degree grad student 20090816 "This puzzle involves meditation. Every answer is a familiar two-word phrase that starts with the initials """"O-M."""" For example: protection for the hand while cooking? Answer: oven mitt." 1 a kid’s card game OLD MAID 20090816 "This puzzle involves meditation. Every answer is a familiar two-word phrase that starts with the initials """"O-M."""" For example: protection for the hand while cooking? Answer: oven mitt." 2 what you should maintain when considering a new idea OPEN MIND 20090816 "This puzzle involves meditation. Every answer is a familiar two-word phrase that starts with the initials """"O-M."""" For example: protection for the hand while cooking? Answer: oven mitt." 3 big name in hot dogs and lunch meats OSCAR MEYER 20090816 "This puzzle involves meditation. Every answer is a familiar two-word phrase that starts with the initials """"O-M."""" For example: protection for the hand while cooking? Answer: oven mitt." 4 a fruity preserve you might put on toast ORANGE MARMALADE 20090816 "This puzzle involves meditation. Every answer is a familiar two-word phrase that starts with the initials """"O-M."""" For example: protection for the hand while cooking? Answer: oven mitt." 5 it might propel a boat carrying a water-skier OUTBOARD MOTOR 20090816 "This puzzle involves meditation. Every answer is a familiar two-word phrase that starts with the initials """"O-M."""" For example: protection for the hand while cooking? Answer: oven mitt." 6 chain of peaks in Arkansas and Missouri OZARK MOUNTAINS 20090816 "This puzzle involves meditation. Every answer is a familiar two-word phrase that starts with the initials """"O-M."""" For example: protection for the hand while cooking? Answer: oven mitt." 7 supervisor for secretaries and administrative assistants OFFICE MANAGER 20090816 "This puzzle involves meditation. Every answer is a familiar two-word phrase that starts with the initials """"O-M."""" For example: protection for the hand while cooking? Answer: oven mitt." 8 nickname for a Southern university OLE MISS 20090816 "This puzzle involves meditation. Every answer is a familiar two-word phrase that starts with the initials """"O-M."""" For example: protection for the hand while cooking? Answer: oven mitt." 9 what a conservative jurist looks to in the Constitution when deciding cases ORIGINAL MEANING 20090816 "This puzzle involves meditation. Every answer is a familiar two-word phrase that starts with the initials """"O-M."""" For example: protection for the hand while cooking? Answer: oven mitt." 10 NBA team for which Shaquille OTMNeal first played (hint: in Florida) ORLANDO MAGIC 20090816 "This puzzle involves meditation. Every answer is a familiar two-word phrase that starts with the initials """"O-M."""" For example: protection for the hand while cooking? Answer: oven mitt." 11 cry equivalent to “Good Heavens!” OH, MY! 20170226 I'm going to give you three 5-letter words. You tell me a 5-letter word that can precede each of mine to complete a compound word or a familiar two-word phrase. 1 glass, slide, melon water 20170226 I'm going to give you three 5-letter words. You tell me a 5-letter word that can precede each of mine to complete a compound word or a familiar two-word phrase. 2 count, donor, sport blood 20170226 I'm going to give you three 5-letter words. You tell me a 5-letter word that can precede each of mine to complete a compound word or a familiar two-word phrase. 3 sense, power, laugh horse 20170226 I'm going to give you three 5-letter words. You tell me a 5-letter word that can precede each of mine to complete a compound word or a familiar two-word phrase. 4 shift, shirt, stand night 20170226 I'm going to give you three 5-letter words. You tell me a 5-letter word that can precede each of mine to complete a compound word or a familiar two-word phrase. 5 sauce, cider, store apple 20170226 I'm going to give you three 5-letter words. You tell me a 5-letter word that can precede each of mine to complete a compound word or a familiar two-word phrase. 6 blank, guard, taken point 20121021 "You will be given two words. Change one letter in each of them to make two new words that name things that are in the same category. (Hint: In each pair, the letter that you change to — that is, the new letter — is the same in each pair.) For example, given the words """"poked"""" and """"tummy,"""" the answer would be """"poker"""" and """"rummy.""""" 1 VENUE and MARK VenuS and MarS 20121021 "You will be given two words. Change one letter in each of them to make two new words that name things that are in the same category. (Hint: In each pair, the letter that you change to — that is, the new letter — is the same in each pair.) For example, given the words """"poked"""" and """"tummy,"""" the answer would be """"poker"""" and """"rummy.""""" 2 SWELTER and PINTS sweAter and pAnts 20121021 "You will be given two words. Change one letter in each of them to make two new words that name things that are in the same category. (Hint: In each pair, the letter that you change to — that is, the new letter — is the same in each pair.) For example, given the words """"poked"""" and """"tummy,"""" the answer would be """"poker"""" and """"rummy.""""" 3 GRANGE and MANGY Orange and mangO 20121021 "You will be given two words. Change one letter in each of them to make two new words that name things that are in the same category. (Hint: In each pair, the letter that you change to — that is, the new letter — is the same in each pair.) For example, given the words """"poked"""" and """"tummy,"""" the answer would be """"poker"""" and """"rummy.""""" 4 FIANCE and NIGEL France and NigeR 20121021 "You will be given two words. Change one letter in each of them to make two new words that name things that are in the same category. (Hint: In each pair, the letter that you change to — that is, the new letter — is the same in each pair.) For example, given the words """"poked"""" and """"tummy,"""" the answer would be """"poker"""" and """"rummy.""""" 5 HORNET and JELLO Cornet and Cello 20121021 "You will be given two words. Change one letter in each of them to make two new words that name things that are in the same category. (Hint: In each pair, the letter that you change to — that is, the new letter — is the same in each pair.) For example, given the words """"poked"""" and """"tummy,"""" the answer would be """"poker"""" and """"rummy.""""" 6 STARLET and PURE sCarlet and puCe 20140921 Given a five-letter word, insert two new letters between the second and third letters of the given word to complete a common seven-letter word. For example: Amble - Am(ia)ble. 1 ANIME anyTIme 20140921 Given a five-letter word, insert two new letters between the second and third letters of the given word to complete a common seven-letter word. For example: Amble - Am(ia)ble. 2 BRIDE brOMide 20140921 Given a five-letter word, insert two new letters between the second and third letters of the given word to complete a common seven-letter word. For example: Amble - Am(ia)ble. 3 CAMEL caRAmel 20140921 Given a five-letter word, insert two new letters between the second and third letters of the given word to complete a common seven-letter word. For example: Amble - Am(ia)ble. 4 EXUDE exTRude (or exCLude) 20140921 Given a five-letter word, insert two new letters between the second and third letters of the given word to complete a common seven-letter word. For example: Amble - Am(ia)ble. 5 MANTA maGEnta 20140921 Given a five-letter word, insert two new letters between the second and third letters of the given word to complete a common seven-letter word. For example: Amble - Am(ia)ble. 6 REIGN reALign 20140921 Given a five-letter word, insert two new letters between the second and third letters of the given word to complete a common seven-letter word. For example: Amble - Am(ia)ble. 7 TRIAL trIVial 20140817 "Every answer is a made up of a two-word phrase, in which the second word has three syllables, and the first word sounds like the last two of these syllables. For example, given the clue, """"What the Italians smell in their capital city,"""" you would say, """"Roma aroma.""""" 1 someone who says actor O'Toole's name over and over Peter repeater 20140817 "Every answer is a made up of a two-word phrase, in which the second word has three syllables, and the first word sounds like the last two of these syllables. For example, given the clue, """"What the Italians smell in their capital city,"""" you would say, """"Roma aroma.""""" 2 biting insect in Ecuador's capital Quito mosquito 20140817 "Every answer is a made up of a two-word phrase, in which the second word has three syllables, and the first word sounds like the last two of these syllables. For example, given the clue, """"What the Italians smell in their capital city,"""" you would say, """"Roma aroma.""""" 3 Mexican dish wrapped in a corn husk made in an African country Mali tamale 20140817 "Every answer is a made up of a two-word phrase, in which the second word has three syllables, and the first word sounds like the last two of these syllables. For example, given the clue, """"What the Italians smell in their capital city,"""" you would say, """"Roma aroma.""""" 4 breed of sheep on a large Nevada city Reno Merino 20140817 "Every answer is a made up of a two-word phrase, in which the second word has three syllables, and the first word sounds like the last two of these syllables. For example, given the clue, """"What the Italians smell in their capital city,"""" you would say, """"Roma aroma.""""" 5 remover of pencil marks that is in the shape of an Indy racing car racer eraser 20140817 "Every answer is a made up of a two-word phrase, in which the second word has three syllables, and the first word sounds like the last two of these syllables. For example, given the clue, """"What the Italians smell in their capital city,"""" you would say, """"Roma aroma.""""" 6 very small cocktail with an olive teeny martini 20130825 You're given some sentences. Each sentence conceals the name of a language in consecutive letters. Name the language. Each answer has five or more letters. 1 Sudan is hot this time of year. danish 20130825 You're given some sentences. Each sentence conceals the name of a language in consecutive letters. Name the language. Each answer has five or more letters. 2 In whispers, I answered the phone. persian 20130825 You're given some sentences. Each sentence conceals the name of a language in consecutive letters. Name the language. Each answer has five or more letters. 3 I have to disagree, knowing you. greek 20130825 You're given some sentences. Each sentence conceals the name of a language in consecutive letters. Name the language. Each answer has five or more letters. 4 The noodles we dished out were overcooked. swedish 20130825 You're given some sentences. Each sentence conceals the name of a language in consecutive letters. Name the language. Each answer has five or more letters. 5 I demand a rinse for my hair. mandarin 20130825 You're given some sentences. Each sentence conceals the name of a language in consecutive letters. Name the language. Each answer has five or more letters. 7 """""get down, or we giants will come up.""""" norwegian 20130825 You're given some sentences. Each sentence conceals the name of a language in consecutive letters. Name the language. Each answer has five or more letters. 8 """""that's all lies,"""" per antony and cleopatra." esparanto 20130825 You're given some sentences. Each sentence conceals the name of a language in consecutive letters. Name the language. Each answer has five or more letters. 9 Are soviet names ever used anymore? vietnamese 20120902 Answer riddles from The Amusing Puzzle Book, published circa the 1840s: 1 i know a word of letters three, add two, and fewer there will be. few 20120902 Answer riddles from The Amusing Puzzle Book, published circa the 1840s: 2 without a bridle or a saddle, across a thing i ride astraddle. and those i ride, by help of me, though almost blind, are made to see. a pair of spectacles 20120902 Answer riddles from The Amusing Puzzle Book, published circa the 1840s: 3 what is that which has been tomorrow and will be yesterday? today 20120902 Answer riddles from The Amusing Puzzle Book, published circa the 1840s: 4 clothed in yellow, red and green, i prate before the king and queen. of neither house nor land possessed, by lords and ladies, i'm caressed. a parrot 20110403 "Every answer is a familiar proverb or saying that contains a word that starts with the letter T. You are given the T word and must guess the saying. For some words, there are multiple answers. For example, given the word """"try,"""" the answer could be, """"If at first you don't succeed, try, try again.""""" 1 THOUSAND A picture is worth a thousand words. 20110403 "Every answer is a familiar proverb or saying that contains a word that starts with the letter T. You are given the T word and must guess the saying. For some words, there are multiple answers. For example, given the word """"try,"""" the answer could be, """"If at first you don't succeed, try, try again.""""" 2 THROW People who live in glass houses shouldn't throw stones. 20110403 "Every answer is a familiar proverb or saying that contains a word that starts with the letter T. You are given the T word and must guess the saying. For some words, there are multiple answers. For example, given the word """"try,"""" the answer could be, """"If at first you don't succeed, try, try again.""""" 3 TURN One good turn deserves another. 20110403 "Every answer is a familiar proverb or saying that contains a word that starts with the letter T. You are given the T word and must guess the saying. For some words, there are multiple answers. For example, given the word """"try,"""" the answer could be, """"If at first you don't succeed, try, try again.""""" 4 TREAD Fools rush in where angels fear to tread. 20110403 "Every answer is a familiar proverb or saying that contains a word that starts with the letter T. You are given the T word and must guess the saying. For some words, there are multiple answers. For example, given the word """"try,"""" the answer could be, """"If at first you don't succeed, try, try again.""""" 6 TWICE Measure twice; cut once. 20110403 "Every answer is a familiar proverb or saying that contains a word that starts with the letter T. You are given the T word and must guess the saying. For some words, there are multiple answers. For example, given the word """"try,"""" the answer could be, """"If at first you don't succeed, try, try again.""""" 7 TIME A stitch in time saves nine. 20110403 "Every answer is a familiar proverb or saying that contains a word that starts with the letter T. You are given the T word and must guess the saying. For some words, there are multiple answers. For example, given the word """"try,"""" the answer could be, """"If at first you don't succeed, try, try again.""""" 8 THINK Great minds think alike. 20121216 "Every answer is a familiar two-word phrase or name in which the first word starts with """"F"""" and the second word starts with """"LA.""""" 1 Michelle Obama, for one First LAdy 20121216 "Every answer is a familiar two-word phrase or name in which the first word starts with """"F"""" and the second word starts with """"LA.""""" 2 French, Spanish or Chinese Foreign LAnguage 20121216 "Every answer is a familiar two-word phrase or name in which the first word starts with """"F"""" and the second word starts with """"LA.""""" 3 arriving at 9 for an 8 o'clock party Fashionably LAte 20121216 "Every answer is a familiar two-word phrase or name in which the first word starts with """"F"""" and the second word starts with """"LA.""""" 4 former NYC mayor for whom an airport is named Fiorello LAGuardia Fiorello LAGuardia 20121216 "Every answer is a familiar two-word phrase or name in which the first word starts with """"F"""" and the second word starts with """"LA.""""" 5 long, narrow bodies of water in Upstate NY Finger LAkes 20121216 "Every answer is a familiar two-word phrase or name in which the first word starts with """"F"""" and the second word starts with """"LA.""""" 6 reading light that does not rest on a table Floor LAmp 20121216 "Every answer is a familiar two-word phrase or name in which the first word starts with """"F"""" and the second word starts with """"LA.""""" 7 bringing down of a plane in an emergency Forced LAnding 20121216 "Every answer is a familiar two-word phrase or name in which the first word starts with """"F"""" and the second word starts with """"LA.""""" 8 it lists ingredients and calorie information Food LAbel 20121216 "Every answer is a familiar two-word phrase or name in which the first word starts with """"F"""" and the second word starts with """"LA.""""" 9 subj. that includes adoption, divorce &alimony Family LAw 20121216 "Every answer is a familiar two-word phrase or name in which the first word starts with """"F"""" and the second word starts with """"LA.""""" 10 agricultural workers Farm LAborers 20121216 "Every answer is a familiar two-word phrase or name in which the first word starts with """"F"""" and the second word starts with """"LA.""""" 11 old Argentine-born actor known for lover roles Fernando LAmas 20121216 "Every answer is a familiar two-word phrase or name in which the first word starts with """"F"""" and the second word starts with """"LA.""""" 12 maker of Cheetos and Ruffles potato chips Frito-Lay 20121216 "Every answer is a familiar two-word phrase or name in which the first word starts with """"F"""" and the second word starts with """"LA.""""" 13 Calif. cemetery where many celebs are buried Forest LAwn 20150222 Every answer today is the name of an Academy Award winner or nominee for best picture. Using the given anagram, decipher the title of the film. The films will go from oldest to newest. Example: OUTWORN (1940) (2 words). Answer: OUR TOWN 1 thelma (1948) hamlet 20150222 Every answer today is the name of an Academy Award winner or nominee for best picture. Using the given anagram, decipher the title of the film. The films will go from oldest to newest. Example: OUTWORN (1940) (2 words). Answer: OUR TOWN 2 ashen (1953) shane 20150222 Every answer today is the name of an Academy Award winner or nominee for best picture. Using the given anagram, decipher the title of the film. The films will go from oldest to newest. Example: OUTWORN (1940) (2 words). Answer: OUR TOWN 3 a pectoral (1963) cleopatra 20150222 Every answer today is the name of an Academy Award winner or nominee for best picture. Using the given anagram, decipher the title of the film. The films will go from oldest to newest. Example: OUTWORN (1940) (2 words). Answer: OUR TOWN 4 a life (1966) alfie 20150222 Every answer today is the name of an Academy Award winner or nominee for best picture. Using the given anagram, decipher the title of the film. The films will go from oldest to newest. Example: OUTWORN (1940) (2 words). Answer: OUR TOWN 5 sham (1971) mash 20150222 Every answer today is the name of an Academy Award winner or nominee for best picture. Using the given anagram, decipher the title of the film. The films will go from oldest to newest. Example: OUTWORN (1940) (2 words). Answer: OUR TOWN 6 bearcat (1972) cabaret 20150222 Every answer today is the name of an Academy Award winner or nominee for best picture. Using the given anagram, decipher the title of the film. The films will go from oldest to newest. Example: OUTWORN (1940) (2 words). Answer: OUR TOWN 7 corky (1976) rocky 20150222 Every answer today is the name of an Academy Award winner or nominee for best picture. Using the given anagram, decipher the title of the film. The films will go from oldest to newest. Example: OUTWORN (1940) (2 words). Answer: OUR TOWN 8 roman era (1979; two words) norma rae 20150222 Every answer today is the name of an Academy Award winner or nominee for best picture. Using the given anagram, decipher the title of the film. The films will go from oldest to newest. Example: OUTWORN (1940) (2 words). Answer: OUR TOWN 9 abbe (1995) babe 20150222 Every answer today is the name of an Academy Award winner or nominee for best picture. Using the given anagram, decipher the title of the film. The films will go from oldest to newest. Example: OUTWORN (1940) (2 words). Answer: OUR TOWN 10 toe cap (2005) capote 20150222 Every answer today is the name of an Academy Award winner or nominee for best picture. Using the given anagram, decipher the title of the film. The films will go from oldest to newest. Example: OUTWORN (1940) (2 words). Answer: OUR TOWN 11 males (2014) selma 20140525 The theme of today's puzzle is May. Every answer is a familiar two-word phrase or name in which the first word starts with MA and the second word ends with Y. Example: Alcoholic beverage made from fermented mash: Malt Whiskey 1 republicans or democrats, vis-Ã-vis libertarians majority (or major) party 20140525 The theme of today's puzzle is May. Every answer is a familiar two-word phrase or name in which the first word starts with MA and the second word ends with Y. Example: Alcoholic beverage made from fermented mash: Malt Whiskey 2 an opener that works on all locks master key 20140525 The theme of today's puzzle is May. Every answer is a familiar two-word phrase or name in which the first word starts with MA and the second word ends with Y. Example: Alcoholic beverage made from fermented mash: Malt Whiskey 3 title for a female member of the president's cabinet madame secretary 20140525 The theme of today's puzzle is May. Every answer is a familiar two-word phrase or name in which the first word starts with MA and the second word ends with Y. Example: Alcoholic beverage made from fermented mash: Malt Whiskey 4 a red item, often put on top of an ice cream sundae maraschino cherry 20140525 The theme of today's puzzle is May. Every answer is a familiar two-word phrase or name in which the first word starts with MA and the second word ends with Y. Example: Alcoholic beverage made from fermented mash: Malt Whiskey 5 postal service mail delivery 20140525 The theme of today's puzzle is May. Every answer is a familiar two-word phrase or name in which the first word starts with MA and the second word ends with Y. Example: Alcoholic beverage made from fermented mash: Malt Whiskey 6 lover of cleopatra mark anthony 20140525 The theme of today's puzzle is May. Every answer is a familiar two-word phrase or name in which the first word starts with MA and the second word ends with Y. Example: Alcoholic beverage made from fermented mash: Malt Whiskey 7 singer who was once married to jennifer lopez marc anthony 20140525 The theme of today's puzzle is May. Every answer is a familiar two-word phrase or name in which the first word starts with MA and the second word ends with Y. Example: Alcoholic beverage made from fermented mash: Malt Whiskey 8 best actor oscar winner for dallas buyers club matthew mcconaughey 20140525 The theme of today's puzzle is May. Every answer is a familiar two-word phrase or name in which the first word starts with MA and the second word ends with Y. Example: Alcoholic beverage made from fermented mash: Malt Whiskey 9 religious commemoration three days before easter maunday thursday 20140525 The theme of today's puzzle is May. Every answer is a familiar two-word phrase or name in which the first word starts with MA and the second word ends with Y. Example: Alcoholic beverage made from fermented mash: Malt Whiskey 10 title md of 1970's tv marcus welby 20140525 The theme of today's puzzle is May. Every answer is a familiar two-word phrase or name in which the first word starts with MA and the second word ends with Y. Example: Alcoholic beverage made from fermented mash: Malt Whiskey 11 complete the title: bridges of ... madison county 20140525 The theme of today's puzzle is May. Every answer is a familiar two-word phrase or name in which the first word starts with MA and the second word ends with Y. Example: Alcoholic beverage made from fermented mash: Malt Whiskey 12 "number 1 rod stewart hit with the lyrics, """"it's late september...""""" maggie may 20140525 The theme of today's puzzle is May. Every answer is a familiar two-word phrase or name in which the first word starts with MA and the second word ends with Y. Example: Alcoholic beverage made from fermented mash: Malt Whiskey 13 a puccini opera set in japan madame/madama butterfly 20120826 "Every answer is a two-word phrase in which one of the words starts with W and the other word is the same with the W removed. For example, if you were given the clue """"desires scurrying insects,"""" the answer would be """"wants ants.""""" 1 composes religious observances writes rites 20120826 "Every answer is a two-word phrase in which one of the words starts with W and the other word is the same with the W removed. For example, if you were given the clue """"desires scurrying insects,"""" the answer would be """"wants ants.""""" 2 hits cab drivers hard whacks hacks 20120826 "Every answer is a two-word phrase in which one of the words starts with W and the other word is the same with the W removed. For example, if you were given the clue """"desires scurrying insects,"""" the answer would be """"wants ants.""""" 3 something that is inside a heated sleeve warm arm 20120826 "Every answer is a two-word phrase in which one of the words starts with W and the other word is the same with the W removed. For example, if you were given the clue """"desires scurrying insects,"""" the answer would be """"wants ants.""""" 4 a question regarding an item of head wear what hat 20120826 "Every answer is a two-word phrase in which one of the words starts with W and the other word is the same with the W removed. For example, if you were given the clue """"desires scurrying insects,"""" the answer would be """"wants ants.""""" 5 a question regarding a woman's stockings whose hose 20120826 "Every answer is a two-word phrase in which one of the words starts with W and the other word is the same with the W removed. For example, if you were given the clue """"desires scurrying insects,"""" the answer would be """"wants ants.""""" 6 entirely a green christmas decoration wholly holly 20120826 "Every answer is a two-word phrase in which one of the words starts with W and the other word is the same with the W removed. For example, if you were given the clue """"desires scurrying insects,"""" the answer would be """"wants ants.""""" 7 a very breezy 500-mile auto race windy indy 20120826 "Every answer is a two-word phrase in which one of the words starts with W and the other word is the same with the W removed. For example, if you were given the clue """"desires scurrying insects,"""" the answer would be """"wants ants.""""" 8 a healthy, large sea mammal hale whale 20120826 "Every answer is a two-word phrase in which one of the words starts with W and the other word is the same with the W removed. For example, if you were given the clue """"desires scurrying insects,"""" the answer would be """"wants ants.""""" 9 a very cool party leader in a legislature hip whip 20120826 "Every answer is a two-word phrase in which one of the words starts with W and the other word is the same with the W removed. For example, if you were given the clue """"desires scurrying insects,"""" the answer would be """"wants ants.""""" 10 animosity ill will/will 20120826 "Every answer is a two-word phrase in which one of the words starts with W and the other word is the same with the W removed. For example, if you were given the clue """"desires scurrying insects,"""" the answer would be """"wants ants.""""" 11 permitting rolling around in the mud allowing wallowing 20120826 "Every answer is a two-word phrase in which one of the words starts with W and the other word is the same with the W removed. For example, if you were given the clue """"desires scurrying insects,"""" the answer would be """"wants ants.""""" 12 attacking the singing of christmas carols assailing wassailing 20170122 Today I've brought a game of categories based on the word COMBS. You probably know how this works. I'm going to give you a series of categories. For each one, name something in it starting with each of the letters C-O-M-B-S. 1 Musical instruments cello; oboe; maracas, marimba, mandolin; bongo drums; saxophone 20170122 Today I've brought a game of categories based on the word COMBS. You probably know how this works. I'm going to give you a series of categories. For each one, name something in it starting with each of the letters C-O-M-B-S. 2 Cities in Florida Cape Canaveral, Clearwater, Cocoa Beach; Orlando; Miami; Boca Raton; Sebring, St. Petersburg, St. Augustine, Sarasota 20170122 Today I've brought a game of categories based on the word COMBS. You probably know how this works. I'm going to give you a series of categories. For each one, name something in it starting with each of the letters C-O-M-B-S. 3 Wild mammals in America cougar; otter; mountain lion; badger; skunk, squirrel 20170122 Today I've brought a game of categories based on the word COMBS. You probably know how this works. I'm going to give you a series of categories. For each one, name something in it starting with each of the letters C-O-M-B-S. 4 People and things seen in a church choir; organ; *,minister; bible; string guitar, sacraments, stained glass windows 20170122 Today I've brought a game of categories based on the word COMBS. You probably know how this works. I'm going to give you a series of categories. For each one, name something in it starting with each of the letters C-O-M-B-S. 5 Shakespearean characters Caesar, Casca, Cornelius; Ophelia, Othello, Oliver; Macbeth,Mercurio; Balthasar, Brutus, Bianca; Salisbury, Solanio, Shylock 20110522 "You are given an item in an unnamed category and must name something else in the category. The answer must start with the first two letters in the clue, only reversed. For example, given """"orchid,"""" the answer would be """"rose"""" (the category being flowers)." 1 angola namibia 20110522 "You are given an item in an unnamed category and must name something else in the category. The answer must start with the first two letters in the clue, only reversed. For example, given """"orchid,"""" the answer would be """"rose"""" (the category being flowers)." 2 royals orioles 20110522 "You are given an item in an unnamed category and must name something else in the category. The answer must start with the first two letters in the clue, only reversed. For example, given """"orchid,"""" the answer would be """"rose"""" (the category being flowers)." 3 zechariah ezekiel; ezra 20110522 "You are given an item in an unnamed category and must name something else in the category. The answer must start with the first two letters in the clue, only reversed. For example, given """"orchid,"""" the answer would be """"rose"""" (the category being flowers)." 4 macbeth a midsummer night's dream 20141005 "The word cho means """"beautiful"""" in Korean and """"butterfly"""" in Japanese. Every answer is a familiar two-word phrase or name based around """"cho."""" Specifically, the first word of the answer starts with C and the second word starts HO." 1 pirate in Peter Pan Captain HOok 20141005 "The word cho means """"beautiful"""" in Korean and """"butterfly"""" in Japanese. Every answer is a familiar two-word phrase or name based around """"cho."""" Specifically, the first word of the answer starts with C and the second word starts HO." 2 It produces a toot on the street Car HOrn 20141005 "The word cho means """"beautiful"""" in Korean and """"butterfly"""" in Japanese. Every answer is a familiar two-word phrase or name based around """"cho."""" Specifically, the first word of the answer starts with C and the second word starts HO." 3 Caesar's Palace or the Tropicana Casino HOtel 20141005 "The word cho means """"beautiful"""" in Korean and """"butterfly"""" in Japanese. Every answer is a familiar two-word phrase or name based around """"cho."""" Specifically, the first word of the answer starts with C and the second word starts HO." 4 something to ride on a merry-go-round Carousel HOrse 20141005 "The word cho means """"beautiful"""" in Korean and """"butterfly"""" in Japanese. Every answer is a familiar two-word phrase or name based around """"cho."""" Specifically, the first word of the answer starts with C and the second word starts HO." 5 1978 Jon Voight--Jane Fonda film about a Viet Nam vet Coming HOme 20141005 "The word cho means """"beautiful"""" in Korean and """"butterfly"""" in Japanese. Every answer is a familiar two-word phrase or name based around """"cho."""" Specifically, the first word of the answer starts with C and the second word starts HO." 6 an institution where a pediatrician might work Children's HOspital 20141005 "The word cho means """"beautiful"""" in Korean and """"butterfly"""" in Japanese. Every answer is a familiar two-word phrase or name based around """"cho."""" Specifically, the first word of the answer starts with C and the second word starts HO." 7 support for tapered lights made of wax Candle HOlder 20141005 "The word cho means """"beautiful"""" in Korean and """"butterfly"""" in Japanese. Every answer is a familiar two-word phrase or name based around """"cho."""" Specifically, the first word of the answer starts with C and the second word starts HO." 8 title characters of a cartoon strip about a boy and his stuffed tiger Calvin [and] HObbes 20140824 Every answer is a made-up, two-word phrase, in which the first word has 5 letters. Drop its last letter and read the remaining 4 letters backward, and you'll get the second word of the phrase. 1 male sheep who are intelligent smart rams 20140824 Every answer is a made-up, two-word phrase, in which the first word has 5 letters. Drop its last letter and read the remaining 4 letters backward, and you'll get the second word of the phrase. 2 light items of headwear worn by astronauts space caps 20140824 Every answer is a made-up, two-word phrase, in which the first word has 5 letters. Drop its last letter and read the remaining 4 letters backward, and you'll get the second word of the phrase. 3 a feudal worker who gave people sass fresh serf 20140824 Every answer is a made-up, two-word phrase, in which the first word has 5 letters. Drop its last letter and read the remaining 4 letters backward, and you'll get the second word of the phrase. 4 a feudal master with a dry, amusing sense of humor droll lord 20140824 Every answer is a made-up, two-word phrase, in which the first word has 5 letters. Drop its last letter and read the remaining 4 letters backward, and you'll get the second word of the phrase. 5 one recognized as a policy expert known wonk 20140824 Every answer is a made-up, two-word phrase, in which the first word has 5 letters. Drop its last letter and read the remaining 4 letters backward, and you'll get the second word of the phrase. 6 a whole week of nighttimes seven eves 20140824 Every answer is a made-up, two-word phrase, in which the first word has 5 letters. Drop its last letter and read the remaining 4 letters backward, and you'll get the second word of the phrase. 7 where peruvian pack animals shop llama mall 20161127 One bears a question, while the other questioned bears. Now, that headline may seem like a non sequitur, but it's also a little clue to the Sunday Puzzle — which concludes a two-week conundrum. 1 Phonograph, Track, Arres record 20161127 One bears a question, while the other questioned bears. Now, that headline may seem like a non sequitur, but it's also a little clue to the Sunday Puzzle — which concludes a two-week conundrum. 2 Peer, Air, Tire pressure 20161127 One bears a question, while the other questioned bears. Now, that headline may seem like a non sequitur, but it's also a little clue to the Sunday Puzzle — which concludes a two-week conundrum. 3 Phillips, Talking, Arrow head 20161127 One bears a question, while the other questioned bears. Now, that headline may seem like a non sequitur, but it's also a little clue to the Sunday Puzzle — which concludes a two-week conundrum. 4 Power, Time, Assembly line 20161127 One bears a question, while the other questioned bears. Now, that headline may seem like a non sequitur, but it's also a little clue to the Sunday Puzzle — which concludes a two-week conundrum. 5 Prairie, Top, Attack dog 20161127 One bears a question, while the other questioned bears. Now, that headline may seem like a non sequitur, but it's also a little clue to the Sunday Puzzle — which concludes a two-week conundrum. 6 Prime, Telephone, Atomic number 20161127 One bears a question, while the other questioned bears. Now, that headline may seem like a non sequitur, but it's also a little clue to the Sunday Puzzle — which concludes a two-week conundrum. 7 Possessive, Test, Attache case 20161127 One bears a question, while the other questioned bears. Now, that headline may seem like a non sequitur, but it's also a little clue to the Sunday Puzzle — which concludes a two-week conundrum. 8 Pants, Three-piece, Antitrust suit 20161127 One bears a question, while the other questioned bears. Now, that headline may seem like a non sequitur, but it's also a little clue to the Sunday Puzzle — which concludes a two-week conundrum. 9 Pound, Traffic, Astrological sign 20200315 Every answer today is a word, name or phrase in which the only consonants are C and T — repeated as often as necessary. All the other letters are vowels. 1 room at the top of a house attic 20200315 Every answer today is a word, name or phrase in which the only consonants are C and T — repeated as often as necessary. All the other letters are vowels. 2 like an angle that's less than 90 degrees acute 20200315 Every answer today is a word, name or phrase in which the only consonants are C and T — repeated as often as necessary. All the other letters are vowels. 3 group of eight musicians octet 20200315 Every answer today is a word, name or phrase in which the only consonants are C and T — repeated as often as necessary. All the other letters are vowels. 4 desert plants with needles cacti 20200315 Every answer today is a word, name or phrase in which the only consonants are C and T — repeated as often as necessary. All the other letters are vowels. 5 adorable person cutie 20200315 Every answer today is a word, name or phrase in which the only consonants are C and T — repeated as often as necessary. All the other letters are vowels. 6 something invisible a grade schooler doesn't want to get cootie 20200315 Every answer today is a word, name or phrase in which the only consonants are C and T — repeated as often as necessary. All the other letters are vowels. 7 brand of breath mint (two words) tic tac 20200315 Every answer today is a word, name or phrase in which the only consonants are C and T — repeated as often as necessary. All the other letters are vowels. 8 kind of acid acetic 20200315 Every answer today is a word, name or phrase in which the only consonants are C and T — repeated as often as necessary. All the other letters are vowels. 9 ancient greek state with athens attica 20200315 Every answer today is a word, name or phrase in which the only consonants are C and T — repeated as often as necessary. All the other letters are vowels. 10 *virtuoso musical piece played on a piano toccata 20200315 Every answer today is a word, name or phrase in which the only consonants are C and T — repeated as often as necessary. All the other letters are vowels. 11 large lake between bolivia and peru titicaca 20200315 Every answer today is a word, name or phrase in which the only consonants are C and T — repeated as often as necessary. All the other letters are vowels. 12 "cry meaning """"stop! that's enough!"""" (three words)" cut it out 20200315 Every answer today is a word, name or phrase in which the only consonants are C and T — repeated as often as necessary. All the other letters are vowels. 13 game with x's and o's (hyphen) tic-tac-toe 20090913 "This puzzle is called """"Dividing the Pie."""" Every answer is a familiar two-word phrase in which the first word starts """"pi"""" and the second word ends in an """"e."""" So, the """"pi-e"""" is divided. For example, if the clue is """"power source for most automobiles,"""" the answer would be """"piston engine.""""" 2 It surrounds the yard of a traditional suburban home PIcket fencE 20090913 "This puzzle is called """"Dividing the Pie."""" Every answer is a familiar two-word phrase in which the first word starts """"pi"""" and the second word ends in an """"e."""" So, the """"pi-e"""" is divided. For example, if the clue is """"power source for most automobiles,"""" the answer would be """"piston engine.""""" 3 Dole ( Foods ) drink that comes in a can PIneapple juicE 20090913 "This puzzle is called """"Dividing the Pie."""" Every answer is a familiar two-word phrase in which the first word starts """"pi"""" and the second word ends in an """"e."""" So, the """"pi-e"""" is divided. For example, if the clue is """"power source for most automobiles,"""" the answer would be """"piston engine.""""" 4 Something that tilts (Something you find in an arcade) PInball machinE 20090913 "This puzzle is called """"Dividing the Pie."""" Every answer is a familiar two-word phrase in which the first word starts """"pi"""" and the second word ends in an """"e."""" So, the """"pi-e"""" is divided. For example, if the clue is """"power source for most automobiles,"""" the answer would be """"piston engine.""""" 5 "A comment made before the question, """"Am I dreaming?""""" PInch mE 20090913 "This puzzle is called """"Dividing the Pie."""" Every answer is a familiar two-word phrase in which the first word starts """"pi"""" and the second word ends in an """"e."""" So, the """"pi-e"""" is divided. For example, if the clue is """"power source for most automobiles,"""" the answer would be """"piston engine.""""" 6 It has needles and cones PIne treE 20090913 "This puzzle is called """"Dividing the Pie."""" Every answer is a familiar two-word phrase in which the first word starts """"pi"""" and the second word ends in an """"e."""" So, the """"pi-e"""" is divided. For example, if the clue is """"power source for most automobiles,"""" the answer would be """"piston engine.""""" 7 Place to sit and eat an outdoor meal PIcnic tablE 20090913 "This puzzle is called """"Dividing the Pie."""" Every answer is a familiar two-word phrase in which the first word starts """"pi"""" and the second word ends in an """"e."""" So, the """"pi-e"""" is divided. For example, if the clue is """"power source for most automobiles,"""" the answer would be """"piston engine.""""" 8 Cause of sciatica PInched nervE 20090913 "This puzzle is called """"Dividing the Pie."""" Every answer is a familiar two-word phrase in which the first word starts """"pi"""" and the second word ends in an """"e."""" So, the """"pi-e"""" is divided. For example, if the clue is """"power source for most automobiles,"""" the answer would be """"piston engine.""""" 9 Player at PNC Park PIttsburgh PiratE 20090913 "This puzzle is called """"Dividing the Pie."""" Every answer is a familiar two-word phrase in which the first word starts """"pi"""" and the second word ends in an """"e."""" So, the """"pi-e"""" is divided. For example, if the clue is """"power source for most automobiles,"""" the answer would be """"piston engine.""""" 10 Colorful, bubbly beverage on New Years Eve PInk champagnE 20090913 "This puzzle is called """"Dividing the Pie."""" Every answer is a familiar two-word phrase in which the first word starts """"pi"""" and the second word ends in an """"e."""" So, the """"pi-e"""" is divided. For example, if the clue is """"power source for most automobiles,"""" the answer would be """"piston engine.""""" 11 "String material for a baby grand (2nd word begins with """"W"""")" PIano wirE 20090913 "This puzzle is called """"Dividing the Pie."""" Every answer is a familiar two-word phrase in which the first word starts """"pi"""" and the second word ends in an """"e."""" So, the """"pi-e"""" is divided. For example, if the clue is """"power source for most automobiles,"""" the answer would be """"piston engine.""""" 12 It grew when its owner told a lie PInocchio's nosE 20110619 "The four rarest letters in the alphabet are J, Q, X and Z. You are given a familiar word and must change one letter in it to a J, Q, X or Z to get another familiar word. For example, given the clue """"enact,"""" the answer would be """"exact.""""" 1 gauge gauZe 20110619 "The four rarest letters in the alphabet are J, Q, X and Z. You are given a familiar word and must change one letter in it to a J, Q, X or Z to get another familiar word. For example, given the clue """"enact,"""" the answer would be """"exact.""""" 2 insure inJure 20110619 "The four rarest letters in the alphabet are J, Q, X and Z. You are given a familiar word and must change one letter in it to a J, Q, X or Z to get another familiar word. For example, given the clue """"enact,"""" the answer would be """"exact.""""" 3 guest Quest 20110619 "The four rarest letters in the alphabet are J, Q, X and Z. You are given a familiar word and must change one letter in it to a J, Q, X or Z to get another familiar word. For example, given the clue """"enact,"""" the answer would be """"exact.""""" 4 contest conteXt 20110619 "The four rarest letters in the alphabet are J, Q, X and Z. You are given a familiar word and must change one letter in it to a J, Q, X or Z to get another familiar word. For example, given the clue """"enact,"""" the answer would be """"exact.""""" 5 Panamas paJamas 20110619 "The four rarest letters in the alphabet are J, Q, X and Z. You are given a familiar word and must change one letter in it to a J, Q, X or Z to get another familiar word. For example, given the clue """"enact,"""" the answer would be """"exact.""""" 6 coating coaXing 20110619 "The four rarest letters in the alphabet are J, Q, X and Z. You are given a familiar word and must change one letter in it to a J, Q, X or Z to get another familiar word. For example, given the clue """"enact,"""" the answer would be """"exact.""""" 7 tweeter tweeZer 20110619 "The four rarest letters in the alphabet are J, Q, X and Z. You are given a familiar word and must change one letter in it to a J, Q, X or Z to get another familiar word. For example, given the clue """"enact,"""" the answer would be """"exact.""""" 8 suaver Quaver 20110619 "The four rarest letters in the alphabet are J, Q, X and Z. You are given a familiar word and must change one letter in it to a J, Q, X or Z to get another familiar word. For example, given the clue """"enact,"""" the answer would be """"exact.""""" 9 election eJection 20110619 "The four rarest letters in the alphabet are J, Q, X and Z. You are given a familiar word and must change one letter in it to a J, Q, X or Z to get another familiar word. For example, given the clue """"enact,"""" the answer would be """"exact.""""" 10 convey conveX 20110619 "The four rarest letters in the alphabet are J, Q, X and Z. You are given a familiar word and must change one letter in it to a J, Q, X or Z to get another familiar word. For example, given the clue """"enact,"""" the answer would be """"exact.""""" 11 seige seiZe 20110619 "The four rarest letters in the alphabet are J, Q, X and Z. You are given a familiar word and must change one letter in it to a J, Q, X or Z to get another familiar word. For example, given the clue """"enact,"""" the answer would be """"exact.""""" 12 empress eXpress 20110619 "The four rarest letters in the alphabet are J, Q, X and Z. You are given a familiar word and must change one letter in it to a J, Q, X or Z to get another familiar word. For example, given the clue """"enact,"""" the answer would be """"exact.""""" 14 suite Quite 20110619 "The four rarest letters in the alphabet are J, Q, X and Z. You are given a familiar word and must change one letter in it to a J, Q, X or Z to get another familiar word. For example, given the clue """"enact,"""" the answer would be """"exact.""""" 15 quarto quartZ 20110619 "The four rarest letters in the alphabet are J, Q, X and Z. You are given a familiar word and must change one letter in it to a J, Q, X or Z to get another familiar word. For example, given the clue """"enact,"""" the answer would be """"exact.""""" 16 function Junction 20110619 "The four rarest letters in the alphabet are J, Q, X and Z. You are given a familiar word and must change one letter in it to a J, Q, X or Z to get another familiar word. For example, given the clue """"enact,"""" the answer would be """"exact.""""" 17 tenon Xenon 20110619 "The four rarest letters in the alphabet are J, Q, X and Z. You are given a familiar word and must change one letter in it to a J, Q, X or Z to get another familiar word. For example, given the clue """"enact,"""" the answer would be """"exact.""""" 18 stargate stargaZe 20121028 "Every answer is a three-word phrase, in which each word has four letters. All three words end in the same three letters, and they rhyme. For example, given the clue, """"Series of offerings of excellent chardonnays and Rieslings,"""" the answer would be """"fine wine line.""""" 1 letters received at a prison in a colorado ski town vail jail mail 20121028 "Every answer is a three-word phrase, in which each word has four letters. All three words end in the same three letters, and they rhyme. For example, given the clue, """"Series of offerings of excellent chardonnays and Rieslings,"""" the answer would be """"fine wine line.""""" 2 additional family stories about former vice-president al more gore lore 20121028 "Every answer is a three-word phrase, in which each word has four letters. All three words end in the same three letters, and they rhyme. For example, given the clue, """"Series of offerings of excellent chardonnays and Rieslings,"""" the answer would be """"fine wine line.""""" 3 the principle profits made by governor romney's investment company main bain gain 20121028 "Every answer is a three-word phrase, in which each word has four letters. All three words end in the same three letters, and they rhyme. For example, given the clue, """"Series of offerings of excellent chardonnays and Rieslings,"""" the answer would be """"fine wine line.""""" 4 grooming of a hairless female horse bare mare care 20121028 "Every answer is a three-word phrase, in which each word has four letters. All three words end in the same three letters, and they rhyme. For example, given the clue, """"Series of offerings of excellent chardonnays and Rieslings,"""" the answer would be """"fine wine line.""""" 5 an optimal check-up of part of a three-piece suit best vest test 20121028 "Every answer is a three-word phrase, in which each word has four letters. All three words end in the same three letters, and they rhyme. For example, given the clue, """"Series of offerings of excellent chardonnays and Rieslings,"""" the answer would be """"fine wine line.""""" 6 a hip implement used on a billiard table cool pool tool 20121028 "Every answer is a three-word phrase, in which each word has four letters. All three words end in the same three letters, and they rhyme. For example, given the clue, """"Series of offerings of excellent chardonnays and Rieslings,"""" the answer would be """"fine wine line.""""" 7 medicinal tablet used in a quentin tarantino film kill bill pill 20121028 "Every answer is a three-word phrase, in which each word has four letters. All three words end in the same three letters, and they rhyme. For example, given the clue, """"Series of offerings of excellent chardonnays and Rieslings,"""" the answer would be """"fine wine line.""""" 8 a thin and nasty college administrator lean mean dean 20121028 "Every answer is a three-word phrase, in which each word has four letters. All three words end in the same three letters, and they rhyme. For example, given the clue, """"Series of offerings of excellent chardonnays and Rieslings,"""" the answer would be """"fine wine line.""""" 9 a crazy tool used for fixing a flat tire on a big truck wack mack jack 20101113 "You are given a sentence with two blanks. The word that goes in the first blank contains an """"oy"""" sound. Change this to an """"oo"""" sound, and phonetically, you'll get a new word that goes in the second blank to complete the sentence. For example, given, """"Anyone who thinks ground chuck tastes better than ____ is clearly a ____,"""" the answer would be """"loin"""" and """"loon.""""" 1 Mother is reading James _____ this morning, over her breakfast bagel and _____. JOYCE and JUICE 20101113 "You are given a sentence with two blanks. The word that goes in the first blank contains an """"oy"""" sound. Change this to an """"oo"""" sound, and phonetically, you'll get a new word that goes in the second blank to complete the sentence. For example, given, """"Anyone who thinks ground chuck tastes better than ____ is clearly a ____,"""" the answer would be """"loin"""" and """"loon.""""" 2 Martin plans to _____ the Navy in mid-may or _____. JOIN and JUNE 20101113 "You are given a sentence with two blanks. The word that goes in the first blank contains an """"oy"""" sound. Change this to an """"oo"""" sound, and phonetically, you'll get a new word that goes in the second blank to complete the sentence. For example, given, """"Anyone who thinks ground chuck tastes better than ____ is clearly a ____,"""" the answer would be """"loin"""" and """"loon.""""" 3 If you answer that Brad Pitt starred as Achilles in the film _____, that would be _____. TROY and TRUE 20101113 "You are given a sentence with two blanks. The word that goes in the first blank contains an """"oy"""" sound. Change this to an """"oo"""" sound, and phonetically, you'll get a new word that goes in the second blank to complete the sentence. For example, given, """"Anyone who thinks ground chuck tastes better than ____ is clearly a ____,"""" the answer would be """"loin"""" and """"loon.""""" 4 Regarding the mystery novel...it would _____ the plot if I told you the murder hinged on a _____ of thread. SPOIL and SPOOL 20101113 "You are given a sentence with two blanks. The word that goes in the first blank contains an """"oy"""" sound. Change this to an """"oo"""" sound, and phonetically, you'll get a new word that goes in the second blank to complete the sentence. For example, given, """"Anyone who thinks ground chuck tastes better than ____ is clearly a ____,"""" the answer would be """"loin"""" and """"loon.""""" 5 For an athletic team on the field, it is not _____ to their morale for the crowd to be _____. BUOYING and BOOING 20101113 "You are given a sentence with two blanks. The word that goes in the first blank contains an """"oy"""" sound. Change this to an """"oo"""" sound, and phonetically, you'll get a new word that goes in the second blank to complete the sentence. For example, given, """"Anyone who thinks ground chuck tastes better than ____ is clearly a ____,"""" the answer would be """"loin"""" and """"loon.""""" 6 Your perfect placement of a lace _____ on the dinner table is _____ noted. DOILY and DULY 20210131 "Every answer today is a pair of phrases in the form of """"___ of ___,"""" where the phrases can end in two ways. I'll give you the two ways those phrases can end. You give me the starting word. Every starting word is five letters long. Ex. Bread / Life --> SLICE (slice of bread, slice of life)1. Way / First refusal2. Mind / The Union3. Purchase / The pudding4. Wheat / The crop5. Cards / Representatives6. Paper / Cake7. View / No return8. May / All9. Golf / Applause10. Sheba / Hearts11. Mind / Music" 1 way / first refusal right of 20210131 "Every answer today is a pair of phrases in the form of """"___ of ___,"""" where the phrases can end in two ways. I'll give you the two ways those phrases can end. You give me the starting word. Every starting word is five letters long. Ex. Bread / Life --> SLICE (slice of bread, slice of life)1. Way / First refusal2. Mind / The Union3. Purchase / The pudding4. Wheat / The crop5. Cards / Representatives6. Paper / Cake7. View / No return8. May / All9. Golf / Applause10. Sheba / Hearts11. Mind / Music" 2 mind / the union state of 20210131 "Every answer today is a pair of phrases in the form of """"___ of ___,"""" where the phrases can end in two ways. I'll give you the two ways those phrases can end. You give me the starting word. Every starting word is five letters long. Ex. Bread / Life --> SLICE (slice of bread, slice of life)1. Way / First refusal2. Mind / The Union3. Purchase / The pudding4. Wheat / The crop5. Cards / Representatives6. Paper / Cake7. View / No return8. May / All9. Golf / Applause10. Sheba / Hearts11. Mind / Music" 3 purchase / the pudding proof of 20210131 "Every answer today is a pair of phrases in the form of """"___ of ___,"""" where the phrases can end in two ways. I'll give you the two ways those phrases can end. You give me the starting word. Every starting word is five letters long. Ex. Bread / Life --> SLICE (slice of bread, slice of life)1. Way / First refusal2. Mind / The Union3. Purchase / The pudding4. Wheat / The crop5. Cards / Representatives6. Paper / Cake7. View / No return8. May / All9. Golf / Applause10. Sheba / Hearts11. Mind / Music" 4 wheat / the crop cream of 20210131 "Every answer today is a pair of phrases in the form of """"___ of ___,"""" where the phrases can end in two ways. I'll give you the two ways those phrases can end. You give me the starting word. Every starting word is five letters long. Ex. Bread / Life --> SLICE (slice of bread, slice of life)1. Way / First refusal2. Mind / The Union3. Purchase / The pudding4. Wheat / The crop5. Cards / Representatives6. Paper / Cake7. View / No return8. May / All9. Golf / Applause10. Sheba / Hearts11. Mind / Music" 5 cards / representatives house of 20210131 "Every answer today is a pair of phrases in the form of """"___ of ___,"""" where the phrases can end in two ways. I'll give you the two ways those phrases can end. You give me the starting word. Every starting word is five letters long. Ex. Bread / Life --> SLICE (slice of bread, slice of life)1. Way / First refusal2. Mind / The Union3. Purchase / The pudding4. Wheat / The crop5. Cards / Representatives6. Paper / Cake7. View / No return8. May / All9. Golf / Applause10. Sheba / Hearts11. Mind / Music" 6 paper / cake piece of 20210131 "Every answer today is a pair of phrases in the form of """"___ of ___,"""" where the phrases can end in two ways. I'll give you the two ways those phrases can end. You give me the starting word. Every starting word is five letters long. Ex. Bread / Life --> SLICE (slice of bread, slice of life)1. Way / First refusal2. Mind / The Union3. Purchase / The pudding4. Wheat / The crop5. Cards / Representatives6. Paper / Cake7. View / No return8. May / All9. Golf / Applause10. Sheba / Hearts11. Mind / Music" 7 view / no return point of 20210131 "Every answer today is a pair of phrases in the form of """"___ of ___,"""" where the phrases can end in two ways. I'll give you the two ways those phrases can end. You give me the starting word. Every starting word is five letters long. Ex. Bread / Life --> SLICE (slice of bread, slice of life)1. Way / First refusal2. Mind / The Union3. Purchase / The pudding4. Wheat / The crop5. Cards / Representatives6. Paper / Cake7. View / No return8. May / All9. Golf / Applause10. Sheba / Hearts11. Mind / Music" 8 may / all first of 20210131 "Every answer today is a pair of phrases in the form of """"___ of ___,"""" where the phrases can end in two ways. I'll give you the two ways those phrases can end. You give me the starting word. Every starting word is five letters long. Ex. Bread / Life --> SLICE (slice of bread, slice of life)1. Way / First refusal2. Mind / The Union3. Purchase / The pudding4. Wheat / The crop5. Cards / Representatives6. Paper / Cake7. View / No return8. May / All9. Golf / Applause10. Sheba / Hearts11. Mind / Music" 9 golf / applause round of 20210131 "Every answer today is a pair of phrases in the form of """"___ of ___,"""" where the phrases can end in two ways. I'll give you the two ways those phrases can end. You give me the starting word. Every starting word is five letters long. Ex. Bread / Life --> SLICE (slice of bread, slice of life)1. Way / First refusal2. Mind / The Union3. Purchase / The pudding4. Wheat / The crop5. Cards / Representatives6. Paper / Cake7. View / No return8. May / All9. Golf / Applause10. Sheba / Hearts11. Mind / Music" 10 sheba / hearts queen of 20210131 "Every answer today is a pair of phrases in the form of """"___ of ___,"""" where the phrases can end in two ways. I'll give you the two ways those phrases can end. You give me the starting word. Every starting word is five letters long. Ex. Bread / Life --> SLICE (slice of bread, slice of life)1. Way / First refusal2. Mind / The Union3. Purchase / The pudding4. Wheat / The crop5. Cards / Representatives6. Paper / Cake7. View / No return8. May / All9. Golf / Applause10. Sheba / Hearts11. Mind / Music" 11 mind / music sound of 20141019 "This Tuesday, Oct. 21, would have been the 100th birthday of Martin Gardner, a longtime """"Mathematical Games"""" columnist for Scientific American. He was also a well-known writer on recreational mathematics, puzzles, stage magic and debunking. Today's challenge consists of classic brainteasers from Martin Gardner books." 1 What familiar word starts with IS, ends with AND and has LA in the middle? island 20141019 "This Tuesday, Oct. 21, would have been the 100th birthday of Martin Gardner, a longtime """"Mathematical Games"""" columnist for Scientific American. He was also a well-known writer on recreational mathematics, puzzles, stage magic and debunking. Today's challenge consists of classic brainteasers from Martin Gardner books." 2 How can you throw a ball so it goes a short distance, comes to a dead stop and then reverses its motion? straight up 20141019 "This Tuesday, Oct. 21, would have been the 100th birthday of Martin Gardner, a longtime """"Mathematical Games"""" columnist for Scientific American. He was also a well-known writer on recreational mathematics, puzzles, stage magic and debunking. Today's challenge consists of classic brainteasers from Martin Gardner books." 3 """""I guarantee,"""" said the salesman in the pet shop, """"That this purple parrot will repeat every word that it hears."""" A customer bought the bird and it didn't repeat a single word ... but what the salesman said was true. How is this possible?" The bird is deaf. 20141019 "This Tuesday, Oct. 21, would have been the 100th birthday of Martin Gardner, a longtime """"Mathematical Games"""" columnist for Scientific American. He was also a well-known writer on recreational mathematics, puzzles, stage magic and debunking. Today's challenge consists of classic brainteasers from Martin Gardner books." 4 A truck is stuck beneath an underpass that is an inch too low for the truck to pass through. A gas station is in sight down the road and, as the driver starts toward it he gets an idea. A few minutes later he's on his way again. How'd he do it? Let some air out of the tires. 20150118 "Every answer is a familiar two-word phrase or name with the initials S.V. For example, given """"noted Idaho ski resort,"""" you would say """"Sun Valley.""""" 1 Who is honored on February 14? St Valentine 20150118 "Every answer is a familiar two-word phrase or name with the initials S.V. For example, given """"noted Idaho ski resort,"""" you would say """"Sun Valley.""""" 2 Its capital was Saigon South Vietnam 20150118 "Every answer is a familiar two-word phrase or name with the initials S.V. For example, given """"noted Idaho ski resort,"""" you would say """"Sun Valley.""""" 3 a ship or boat Sailing Vessel 20150118 "Every answer is a familiar two-word phrase or name with the initials S.V. For example, given """"noted Idaho ski resort,"""" you would say """"Sun Valley.""""" 4 bass guitarist and vocalist for the Sex Pistols Sid Vicious 20150118 "Every answer is a familiar two-word phrase or name with the initials S.V. For example, given """"noted Idaho ski resort,"""" you would say """"Sun Valley.""""" 5 lettuce, carrot or cucumber, as part of a dinner starter Salad Vegetable 20150118 "Every answer is a familiar two-word phrase or name with the initials S.V. For example, given """"noted Idaho ski resort,"""" you would say """"Sun Valley.""""" 6 as a group they are listed as patience, chastity, ... Seven Virtues 20140601 Every answer to the given clue is a made-up, two-word phrase in which you add IN to the front of the first word to get the second word. For example: shoe insert; sole insole 1 physical harm to a court panel JURY INJURY 20140601 Every answer to the given clue is a made-up, two-word phrase in which you add IN to the front of the first word to get the second word. For example: shoe insert; sole insole 2 more-competent company that protects against financial loss SURER INSURER 20140601 Every answer to the given clue is a made-up, two-word phrase in which you add IN to the front of the first word to get the second word. For example: shoe insert; sole insole 3 previous person who rats FORMER INFORMER 20140601 Every answer to the given clue is a made-up, two-word phrase in which you add IN to the front of the first word to get the second word. For example: shoe insert; sole insole 4 creates openings for air INVENTS VENTS 20140601 Every answer to the given clue is a made-up, two-word phrase in which you add IN to the front of the first word to get the second word. For example: shoe insert; sole insole 5 adds to the number of lines made by folding INCREASES CREASES 20140601 Every answer to the given clue is a made-up, two-word phrase in which you add IN to the front of the first word to get the second word. For example: shoe insert; sole insole 6 establishes compartments for horses in a bar INSTALLS STALLS 20140601 Every answer to the given clue is a made-up, two-word phrase in which you add IN to the front of the first word to get the second word. For example: shoe insert; sole insole 7 religious offshoot of insects and bees INBSECT SECT 20140601 Every answer to the given clue is a made-up, two-word phrase in which you add IN to the front of the first word to get the second word. For example: shoe insert; sole insole 8 large geographical area where people keep going to the bathroom INCONTINENT CONTINENT 20110703 "You are given a series of sentences, each of which is missing three words. The word in the first blank is five letters long. Drop the last letter to get a four-letter word for the second blank. Drop the last letter to get a three-letter answer for the third blank. For example, given the sentence, """"While I was filming at the Egyptian pyramids, a ____ with a rider on it ____ into view of my ____,"""" the words would be """"camel,"""" """"came"""" and """"cam.""""" 1 At my birthday_______, playing miniature golf, my favorite _______ of the course was the _______ five hole. PARTY---PART---PAR 20110703 "You are given a series of sentences, each of which is missing three words. The word in the first blank is five letters long. Drop the last letter to get a four-letter word for the second blank. Drop the last letter to get a three-letter answer for the third blank. For example, given the sentence, """"While I was filming at the Egyptian pyramids, a ____ with a rider on it ____ into view of my ____,"""" the words would be """"camel,"""" """"came"""" and """"cam.""""" 2 """""Go _______,"""" the general cried as his troops left the _______ _______ battle." FORTH---FORT---FOR 20110703 "You are given a series of sentences, each of which is missing three words. The word in the first blank is five letters long. Drop the last letter to get a four-letter word for the second blank. Drop the last letter to get a three-letter answer for the third blank. For example, given the sentence, """"While I was filming at the Egyptian pyramids, a ____ with a rider on it ____ into view of my ____,"""" the words would be """"camel,"""" """"came"""" and """"cam.""""" 3 George was driving his old Ford _______ to the store for a _______ of milk when a _______ in the steering column broke. PINTO---PINT---PIN 20110703 "You are given a series of sentences, each of which is missing three words. The word in the first blank is five letters long. Drop the last letter to get a four-letter word for the second blank. Drop the last letter to get a three-letter answer for the third blank. For example, given the sentence, """"While I was filming at the Egyptian pyramids, a ____ with a rider on it ____ into view of my ____,"""" the words would be """"camel,"""" """"came"""" and """"cam.""""" 4 During the recent recession, our family decided to _______ on unnecessary things like _______ milk andour planned_______ vacation. SKIMP---SKIM---SKI 20110703 "You are given a series of sentences, each of which is missing three words. The word in the first blank is five letters long. Drop the last letter to get a four-letter word for the second blank. Drop the last letter to get a three-letter answer for the third blank. For example, given the sentence, """"While I was filming at the Egyptian pyramids, a ____ with a rider on it ____ into view of my ____,"""" the words would be """"camel,"""" """"came"""" and """"cam.""""" 5 During the _______17th century, the _______ of Sandwich lost his right _______ in a swordfight. EARLY---EARL---EAR 20110703 "You are given a series of sentences, each of which is missing three words. The word in the first blank is five letters long. Drop the last letter to get a four-letter word for the second blank. Drop the last letter to get a three-letter answer for the third blank. For example, given the sentence, """"While I was filming at the Egyptian pyramids, a ____ with a rider on it ____ into view of my ____,"""" the words would be """"camel,"""" """"came"""" and """"cam.""""" 6 In the announcers' _______, the commentators couldn't believe the team manager had been given the _______, causing the fans to _______. BOOTH---BOOT---BOO 20110703 "You are given a series of sentences, each of which is missing three words. The word in the first blank is five letters long. Drop the last letter to get a four-letter word for the second blank. Drop the last letter to get a three-letter answer for the third blank. For example, given the sentence, """"While I was filming at the Egyptian pyramids, a ____ with a rider on it ____ into view of my ____,"""" the words would be """"camel,"""" """"came"""" and """"cam.""""" 7 Hennessy _______ is an internationally-known _______, but not as well-known as Kellogg's Raisin _______ or the Maidenform _______. BRANDY---BRAND---BRAN---BRA 20141228 Since this is the last puzzle of the year, it's time for the annual new-names-in-the-news quiz. Here's how it works: You're given names that you probably never heard of before 2014, but were in the news during the past 12 months. Answer with who the people are. 1 Michael Sam first openly gay athlete to be drafted by the national football league 20141228 Since this is the last puzzle of the year, it's time for the annual new-names-in-the-news quiz. Here's how it works: You're given names that you probably never heard of before 2014, but were in the news during the past 12 months. Answer with who the people are. 2 Cliven Bundy the nevada cattlerancher who engaged in an armed standoff with the bureau of land management over grazing fees 20141228 Since this is the last puzzle of the year, it's time for the annual new-names-in-the-news quiz. Here's how it works: You're given names that you probably never heard of before 2014, but were in the news during the past 12 months. Answer with who the people are. 3 Narendra Modi new prime minister of india 20141228 Since this is the last puzzle of the year, it's time for the annual new-names-in-the-news quiz. Here's how it works: You're given names that you probably never heard of before 2014, but were in the news during the past 12 months. Answer with who the people are. 4 MoTune Davis first girl to earn a win and to pitch a shutout in little leagueworld series history 20141228 Since this is the last puzzle of the year, it's time for the annual new-names-in-the-news quiz. Here's how it works: You're given names that you probably never heard of before 2014, but were in the news during the past 12 months. Answer with who the people are. 5 Amal Al-Mamudin the womanwho married george clooney 20141228 Since this is the last puzzle of the year, it's time for the annual new-names-in-the-news quiz. Here's how it works: You're given names that you probably never heard of before 2014, but were in the news during the past 12 months. Answer with who the people are. 6 67p (churyumov-gerasimenko) the comet on which the rosetta spacecraft landed 20140629 For each set of three words, find a word that can precede each one to complete a familiar two-word phrase or name. The first word in each set will name an animal. Example: turtle, spring, office. The answer would be box — box turtle, box spring, box office. 1 (three-letter answer): dog, chocolate, potato hot 20140629 For each set of three words, find a word that can precede each one to complete a familiar two-word phrase or name. The first word in each set will name an animal. Example: turtle, spring, office. The answer would be box — box turtle, box spring, box office. 2 (four-letter answers): horse, school, voltage high 20140629 For each set of three words, find a word that can precede each one to complete a familiar two-word phrase or name. The first word in each set will name an animal. Example: turtle, spring, office. The answer would be box — box turtle, box spring, box office. 3 panther, slip, floyd pink 20140629 For each set of three words, find a word that can precede each one to complete a familiar two-word phrase or name. The first word in each set will name an animal. Example: turtle, spring, office. The answer would be box — box turtle, box spring, box office. 4 sheep, market, friday black 20140629 For each set of three words, find a word that can precede each one to complete a familiar two-word phrase or name. The first word in each set will name an animal. Example: turtle, spring, office. The answer would be box — box turtle, box spring, box office. 5 elephant, bread, white collar 20140629 For each set of three words, find a word that can precede each one to complete a familiar two-word phrase or name. The first word in each set will name an animal. Example: turtle, spring, office. The answer would be box — box turtle, box spring, box office. 6 bear, cap, opposites polar 20140629 For each set of three words, find a word that can precede each one to complete a familiar two-word phrase or name. The first word in each set will name an animal. Example: turtle, spring, office. The answer would be box — box turtle, box spring, box office. 7 goat, club, crystal billy 20140629 For each set of three words, find a word that can precede each one to complete a familiar two-word phrase or name. The first word in each set will name an animal. Example: turtle, spring, office. The answer would be box — box turtle, box spring, box office. 8 tiger, route, towel paper 20140629 For each set of three words, find a word that can precede each one to complete a familiar two-word phrase or name. The first word in each set will name an animal. Example: turtle, spring, office. The answer would be box — box turtle, box spring, box office. 9 monkey, pencil, gun grease 20080601 "In this week's on-air challenge, you are given anagrams plus an extra letter that you must unscramble to name a river. For example, given """"roil"""" plus E, the answer would be """"Loire,"""" the river in France." 1 goal + v volga 20080601 "In this week's on-air challenge, you are given anagrams plus an extra letter that you must unscramble to name a river. For example, given """"roil"""" plus E, the answer would be """"Loire,"""" the river in France." 2 heir + n rhine 20080601 "In this week's on-air challenge, you are given anagrams plus an extra letter that you must unscramble to name a river. For example, given """"roil"""" plus E, the answer would be """"Loire,"""" the river in France." 7 grits + i tigris 20121104 The on-air challenge included an incorrect anagram. Forest, which is the first name of actor Forest Whitaker, is not an anagram of Forster, which is the last name of novelist E.M. Forster. 1 ogden __ christian anderson nash/hans 20121104 The on-air challenge included an incorrect anagram. Forest, which is the first name of actor Forest Whitaker, is not an anagram of Forster, which is the last name of novelist E.M. Forster. 2 condoleezza__clapton rice/eric 20121104 The on-air challenge included an incorrect anagram. Forest, which is the first name of actor Forest Whitaker, is not an anagram of Forster, which is the last name of novelist E.M. Forster. 3 william__bader ginsberg hurt/ruth 20121104 The on-air challenge included an incorrect anagram. Forest, which is the first name of actor Forest Whitaker, is not an anagram of Forster, which is the last name of novelist E.M. Forster. 4 sally___castro field/fidel 20121104 The on-air challenge included an incorrect anagram. Forest, which is the first name of actor Forest Whitaker, is not an anagram of Forster, which is the last name of novelist E.M. Forster. 5 ralph___agassi nader/andre 20121104 The on-air challenge included an incorrect anagram. Forest, which is the first name of actor Forest Whitaker, is not an anagram of Forster, which is the last name of novelist E.M. Forster. 6 peter___flynn lloyd/dolly 20121104 The on-air challenge included an incorrect anagram. Forest, which is the first name of actor Forest Whitaker, is not an anagram of Forster, which is the last name of novelist E.M. Forster. 7 christopher___parton lorre/errol 20121104 The on-air challenge included an incorrect anagram. Forest, which is the first name of actor Forest Whitaker, is not an anagram of Forster, which is the last name of novelist E.M. Forster. 8 katie___asch sholem/yasser 20121104 The on-air challenge included an incorrect anagram. Forest, which is the first name of actor Forest Whitaker, is not an anagram of Forster, which is the last name of novelist E.M. Forster. 9 dorothy___arafat sayers/yasser 20121104 The on-air challenge included an incorrect anagram. Forest, which is the first name of actor Forest Whitaker, is not an anagram of Forster, which is the last name of novelist E.M. Forster. 10 e. m. ___whittaker forster/forrest 20121104 The on-air challenge included an incorrect anagram. Forest, which is the first name of actor Forest Whitaker, is not an anagram of Forster, which is the last name of novelist E.M. Forster. 11 ogden __ christian anderson nash/hans 20161218 Every answer is an anagram of a geographical feature. 1 kale lake 20161218 Every answer is an anagram of a geographical feature. 2 same mesa 20161218 Every answer is an anagram of a geographical feature. 3 lies isle 20161218 Every answer is an anagram of a geographical feature. 4 spas pass 20161218 Every answer is an anagram of a geographical feature. 5 allot atoll 20161218 Every answer is an anagram of a geographical feature. 6 canoe ocean 20161218 Every answer is an anagram of a geographical feature. 7 softer forest 20161218 Every answer is an anagram of a geographical feature. 8 sectional coastline 20161218 Every answer is an anagram of a geographical feature. 9 real force coral reef 20161218 Every answer is an anagram of a geographical feature. 10 ROOM moor 20161218 Every answer is an anagram of a geographical feature. 11 DEALT delta 20161218 Every answer is an anagram of a geographical feature. 12 HARMS marsh 20161218 Every answer is an anagram of a geographical feature. 13 DIRGE ridge 20161218 Every answer is an anagram of a geographical feature. 14 LAPIN plain 20161218 Every answer is an anagram of a geographical feature. 15 RESTED desert 20161218 Every answer is an anagram of a geographical feature. 16 MASTER stream 20161218 Every answer is an anagram of a geographical feature. 17 ARTIST strait 20161218 Every answer is an anagram of a geographical feature. 18 NO GOAL lagoon 20210103 "Every year around this time I do a year-end """"New Names in the News Quiz."""" Here's how it works. I'll name some people and things you probably never heard of until 2020, but who sprang to prominence during the past 12 months. You tell me who or what they are. This list was compiled with the help of Kathie Baker, who played a similar quiz in the past." 1 Amy Coney Barrett Replaced Ruth Bader Ginsberg on the Supreme Court 20210103 "Every year around this time I do a year-end """"New Names in the News Quiz."""" Here's how it works. I'll name some people and things you probably never heard of until 2020, but who sprang to prominence during the past 12 months. You tell me who or what they are. This list was compiled with the help of Kathie Baker, who played a similar quiz in the past." 2 Debra Birx coordinator of the White House coronavirus response 20210103 "Every year around this time I do a year-end """"New Names in the News Quiz."""" Here's how it works. I'll name some people and things you probably never heard of until 2020, but who sprang to prominence during the past 12 months. You tell me who or what they are. This list was compiled with the help of Kathie Baker, who played a similar quiz in the past." 3 Li Wenliang Chinese physician who issued a warning about coronavirus, was suppressed and died from the disease 20210103 "Every year around this time I do a year-end """"New Names in the News Quiz."""" Here's how it works. I'll name some people and things you probably never heard of until 2020, but who sprang to prominence during the past 12 months. You tell me who or what they are. This list was compiled with the help of Kathie Baker, who played a similar quiz in the past." 4 Doug Emhoff Husband of Vice President-elect Kamala Harris 20210103 "Every year around this time I do a year-end """"New Names in the News Quiz."""" Here's how it works. I'll name some people and things you probably never heard of until 2020, but who sprang to prominence during the past 12 months. You tell me who or what they are. This list was compiled with the help of Kathie Baker, who played a similar quiz in the past." 5 Sarah Fuller first woman to play and to score in a Power Five football game 20210103 "Every year around this time I do a year-end """"New Names in the News Quiz."""" Here's how it works. I'll name some people and things you probably never heard of until 2020, but who sprang to prominence during the past 12 months. You tell me who or what they are. This list was compiled with the help of Kathie Baker, who played a similar quiz in the past." 6 Isaias August hurricane that caused extensive damage across the Caribbean and the East Coast of the United States 20170716 I'm going to give you a 4-letter word and a 5-letter word. Rearrange the letters in each one to make two opposites. 1 SLOT & FONDU LOST & FOUND 20170716 I'm going to give you a 4-letter word and a 5-letter word. Rearrange the letters in each one to make two opposites. 3 ROVE & NUDER OVER & UNDER 20170716 I'm going to give you a 4-letter word and a 5-letter word. Rearrange the letters in each one to make two opposites. 4 DINK & LUCRE KIND & CRUEL 20170716 I'm going to give you a 4-letter word and a 5-letter word. Rearrange the letters in each one to make two opposites. 6 VASE & PENDS SAVE & SPEND 20170716 I'm going to give you a 4-letter word and a 5-letter word. Rearrange the letters in each one to make two opposites. 7 SKIN & ALOFT SINK & FLOAT 20170716 I'm going to give you a 4-letter word and a 5-letter word. Rearrange the letters in each one to make two opposites. 8 FILE & HATED LIFE & DEATH 20160828 Every answer today is a familiar 8-letter word. We're going to give you two 3-letter words that are somewhere in it. You tell me the full word. 1 vet, ail dovetail 20160828 Every answer today is a familiar 8-letter word. We're going to give you two 3-letter words that are somewhere in it. You tell me the full word. 2 leg, ram telegram 20160828 Every answer today is a familiar 8-letter word. We're going to give you two 3-letter words that are somewhere in it. You tell me the full word. 3 pen, ago pentagon 20160828 Every answer today is a familiar 8-letter word. We're going to give you two 3-letter words that are somewhere in it. You tell me the full word. 4 urn, oat turncoat 20160828 Every answer today is a familiar 8-letter word. We're going to give you two 3-letter words that are somewhere in it. You tell me the full word. 5 pip, any epiphany 20160828 Every answer today is a familiar 8-letter word. We're going to give you two 3-letter words that are somewhere in it. You tell me the full word. 6 not, one monotone 20131208 "Every answer is the name of a famous person whose first and last names start with the same consonant or group of consonants. You're given rhymes for the two names. You name the people. For example, if given """"cycle four,"""" the answer would be """"Michael Moore.""""" 1 messy saxon jesse jackson 20131208 "Every answer is the name of a famous person whose first and last names start with the same consonant or group of consonants. You're given rhymes for the two names. You name the people. For example, if given """"cycle four,"""" the answer would be """"Michael Moore.""""" 2 melon craze helen hayes 20131208 "Every answer is the name of a famous person whose first and last names start with the same consonant or group of consonants. You're given rhymes for the two names. You name the people. For example, if given """"cycle four,"""" the answer would be """"Michael Moore.""""" 3 porous tray doris day 20131208 "Every answer is the name of a famous person whose first and last names start with the same consonant or group of consonants. You're given rhymes for the two names. You name the people. For example, if given """"cycle four,"""" the answer would be """"Michael Moore.""""" 4 since dawn vince vaughan 20131208 "Every answer is the name of a famous person whose first and last names start with the same consonant or group of consonants. You're given rhymes for the two names. You name the people. For example, if given """"cycle four,"""" the answer would be """"Michael Moore.""""" 5 harry fronds barry bonds 20131208 "Every answer is the name of a famous person whose first and last names start with the same consonant or group of consonants. You're given rhymes for the two names. You name the people. For example, if given """"cycle four,"""" the answer would be """"Michael Moore.""""" 6 bed burner ted turner 20131208 "Every answer is the name of a famous person whose first and last names start with the same consonant or group of consonants. You're given rhymes for the two names. You name the people. For example, if given """"cycle four,"""" the answer would be """"Michael Moore.""""" 7 even chills steven stills 20131208 "Every answer is the name of a famous person whose first and last names start with the same consonant or group of consonants. You're given rhymes for the two names. You name the people. For example, if given """"cycle four,"""" the answer would be """"Michael Moore.""""" 8 bike tires mike myers 20131208 "Every answer is the name of a famous person whose first and last names start with the same consonant or group of consonants. You're given rhymes for the two names. You name the people. For example, if given """"cycle four,"""" the answer would be """"Michael Moore.""""" 9 dames voice james joyce 20131208 "Every answer is the name of a famous person whose first and last names start with the same consonant or group of consonants. You're given rhymes for the two names. You name the people. For example, if given """"cycle four,"""" the answer would be """"Michael Moore.""""" 10 ratty cage patti page 20131208 "Every answer is the name of a famous person whose first and last names start with the same consonant or group of consonants. You're given rhymes for the two names. You name the people. For example, if given """"cycle four,"""" the answer would be """"Michael Moore.""""" 11 german dress herman hess 20131208 "Every answer is the name of a famous person whose first and last names start with the same consonant or group of consonants. You're given rhymes for the two names. You name the people. For example, if given """"cycle four,"""" the answer would be """"Michael Moore.""""" 12 heavy base chevy chase 20101023 "Each answer is a two-word phrase in which the first word begins with P-O and the second word begins with P (and is not followed by an O). For example, given the clue """"his or hers,"""" the answer would be """"possessive pronoun.""""" 1 another term for a billiard hall pool parlor 20101023 "Each answer is a two-word phrase in which the first word begins with P-O and the second word begins with P (and is not followed by an O). For example, given the clue """"his or hers,"""" the answer would be """"possessive pronoun.""""" 2 judge who, by tradition, sentenced jesus to crucifixion pontius pilate 20101023 "Each answer is a two-word phrase in which the first word begins with P-O and the second word begins with P (and is not followed by an O). For example, given the clue """"his or hers,"""" the answer would be """"possessive pronoun.""""" 3 what someone who is receiving death threats might request police protection 20101023 "Each answer is a two-word phrase in which the first word begins with P-O and the second word begins with P (and is not followed by an O). For example, given the clue """"his or hers,"""" the answer would be """"possessive pronoun.""""" 4 a person in a game of stud or texas hold 'em poker player 20101023 "Each answer is a two-word phrase in which the first word begins with P-O and the second word begins with P (and is not followed by an O). For example, given the clue """"his or hers,"""" the answer would be """"possessive pronoun.""""" 5 a kind of letter that is nasty poison pen 20101023 "Each answer is a two-word phrase in which the first word begins with P-O and the second word begins with P (and is not followed by an O). For example, given the clue """"his or hers,"""" the answer would be """"possessive pronoun.""""" 6 what a stereotypical nerd has on his shirt pocket protector 20101023 "Each answer is a two-word phrase in which the first word begins with P-O and the second word begins with P (and is not followed by an O). For example, given the clue """"his or hers,"""" the answer would be """"possessive pronoun.""""" 7 what a native of warsaw needs to travel abroad polish passport 20101023 "Each answer is a two-word phrase in which the first word begins with P-O and the second word begins with P (and is not followed by an O). For example, given the clue """"his or hers,"""" the answer would be """"possessive pronoun.""""" 8 a baked entrÃe, made by swanson, for example pot pie 20101023 "Each answer is a two-word phrase in which the first word begins with P-O and the second word begins with P (and is not followed by an O). For example, given the clue """"his or hers,"""" the answer would be """"possessive pronoun.""""" 9 """""why not?"""" in french" porquoi pas 20101023 "Each answer is a two-word phrase in which the first word begins with P-O and the second word begins with P (and is not followed by an O). For example, given the clue """"his or hers,"""" the answer would be """"possessive pronoun.""""" 10 in ice hockey, when one team has a numerical advantage on the ice over the other power play 20101023 "Each answer is a two-word phrase in which the first word begins with P-O and the second word begins with P (and is not followed by an O). For example, given the clue """"his or hers,"""" the answer would be """"possessive pronoun.""""" 11 a fine ball for applying cosmetic powder puff 20101023 "Each answer is a two-word phrase in which the first word begins with P-O and the second word begins with P (and is not followed by an O). For example, given the clue """"his or hers,"""" the answer would be """"possessive pronoun.""""" 12 the outer part of a spud potato peel 20101023 "Each answer is a two-word phrase in which the first word begins with P-O and the second word begins with P (and is not followed by an O). For example, given the clue """"his or hers,"""" the answer would be """"possessive pronoun.""""" 13 republicans, democrats or greens political party 20101023 "Each answer is a two-word phrase in which the first word begins with P-O and the second word begins with P (and is not followed by an O). For example, given the clue """"his or hers,"""" the answer would be """"possessive pronoun.""""" 14 where to go to vote polling place 20190804 I'm going to read you some sentences. Each sentence has two blanks. The word that goes in the first blank starts GR-. Change the GR- to CR- and phonetically you'll get a new word that goes in the second blank to complete the sentence. 1 folding my bus ticket to athens, ___, left a permanent ___ in it. folding my bus ticket to athens, greece, left a permanent crease in it. 20190804 I'm going to read you some sentences. Each sentence has two blanks. The word that goes in the first blank starts GR-. Change the GR- to CR- and phonetically you'll get a new word that goes in the second blank to complete the sentence. 2 at school i always put off studying sentence structure until the last minute so you might call me a ___ ___. at school i always put off studying sentence structure until the last minute so you might call me a grammar crammer. 20190804 I'm going to read you some sentences. Each sentence has two blanks. The word that goes in the first blank starts GR-. Change the GR- to CR- and phonetically you'll get a new word that goes in the second blank to complete the sentence. 3 my dad's dad always got pains in his legs while swimming and we never found a way to prevent ___ ___. the baker would ___ under her breath if her apple ___ turned out bad. 20190804 I'm going to read you some sentences. Each sentence has two blanks. The word that goes in the first blank starts GR-. Change the GR- to CR- and phonetically you'll get a new word that goes in the second blank to complete the sentence. 4 the first asteroid left a big hole in the moon but the second left an even ___ ___. the first asteroid left a big hole in the moon but the second left an even greater crater. 20070506 Today I brought a game of categories based on the word coast, C-O-A-S-T. I'm going to give you some categories. For each one, name something in that category starting with each of the letters in coast.  1 Midwest cities (from Ohio to the Plains States) CHICAGO, OLNEY or OMAHA, OSHKOSH or OTTUMWA, AKRON, ST. LOUIS, TOPEKA or TOLEDO 20070506 Today I brought a game of categories based on the word coast, C-O-A-S-T. I'm going to give you some categories. For each one, name something in that category starting with each of the letters in coast.  2 ___________ bar (fill in the blank) CAPTAIN'S BAR OR CASH BAR OR CANDY BAR OR CROW BAR 20070506 Today I brought a game of categories based on the word coast, C-O-A-S-T. I'm going to give you some categories. For each one, name something in that category starting with each of the letters in coast.  3 Comic strip characters, past or present CAPTAIN AMERICA or CALVIN 20070506 Today I brought a game of categories based on the word coast, C-O-A-S-T. I'm going to give you some categories. For each one, name something in that category starting with each of the letters in coast.  4 things to have for breakfast CEREAL or CANTELOUPE OR COFFEE OR CROISSANT 20150315 """""Yes"""" is supposed to be the most pleasing word in the English language. And if that's true, today's puzzle will be very pleasing indeed. Every answer is an anagram of """"yes"""" plus two or three other letters." 1 YES + LT style 20150315 """""Yes"""" is supposed to be the most pleasing word in the English language. And if that's true, today's puzzle will be very pleasing indeed. Every answer is an anagram of """"yes"""" plus two or three other letters." 2 YES + KP pesky (or Skype) 20150315 """""Yes"""" is supposed to be the most pleasing word in the English language. And if that's true, today's puzzle will be very pleasing indeed. Every answer is an anagram of """"yes"""" plus two or three other letters." 3 YES + AQU queasy 20150315 """""Yes"""" is supposed to be the most pleasing word in the English language. And if that's true, today's puzzle will be very pleasing indeed. Every answer is an anagram of """"yes"""" plus two or three other letters." 4 YES + FIT feisty 20150315 """""Yes"""" is supposed to be the most pleasing word in the English language. And if that's true, today's puzzle will be very pleasing indeed. Every answer is an anagram of """"yes"""" plus two or three other letters." 5 YES + CHE cheesy 20150315 """""Yes"""" is supposed to be the most pleasing word in the English language. And if that's true, today's puzzle will be very pleasing indeed. Every answer is an anagram of """"yes"""" plus two or three other letters." 6 YES + EJR Jersey 20150315 """""Yes"""" is supposed to be the most pleasing word in the English language. And if that's true, today's puzzle will be very pleasing indeed. Every answer is an anagram of """"yes"""" plus two or three other letters." 7 YES + OPR osprey 20150315 """""Yes"""" is supposed to be the most pleasing word in the English language. And if that's true, today's puzzle will be very pleasing indeed. Every answer is an anagram of """"yes"""" plus two or three other letters." 8 YES + ILM smiley 20090712 The challenge is a game of categories. The word is beach. Each clue is a category, and for each category, the goal is to name something in it starting with each of the letters in beach. 1 Countries in Central and South America Bolivia, Ecuador, Argentina, Colombia, Honduras 20090712 The challenge is a game of categories. The word is beach. Each clue is a category, and for each category, the goal is to name something in it starting with each of the letters in beach. 2 Things to eat or drink at breakfast Bacon, eggs, apple juice, cherries, ham 20090712 The challenge is a game of categories. The word is beach. Each clue is a category, and for each category, the goal is to name something in it starting with each of the letters in beach. 3 Symbols on a computer keyboard other than letters and numbers bracket, equal sign, apostrophe, colon, hyphen 20090712 The challenge is a game of categories. The word is beach. Each clue is a category, and for each category, the goal is to name something in it starting with each of the letters in beach. 4 Red___ (compound term or two-word phrase) Brigade, eye, Army, coats, head 20130224 " ""You will be given some words starting with the letter R. You name a proverb or saying that contains each one.""" 1 Rome When in Rome, do as the Romans do. Rome was not built in a day. all roads lead to Rome. 20130224 " ""You will be given some words starting with the letter R. You name a proverb or saying that contains each one.""" 2 Rolling A rolling stone gathers no moss. 20130224 " ""You will be given some words starting with the letter R. You name a proverb or saying that contains each one.""" 3 Reaping As you sow, so shall ye reap. 20130224 " ""You will be given some words starting with the letter R. You name a proverb or saying that contains each one.""" 4 Rise Early to bed and early to rise helps makes a man healthy, wealthy and wise. 20130224 " ""You will be given some words starting with the letter R. You name a proverb or saying that contains each one.""" 5 Rush Fools rush in where angels fear to tread. 20130224 " ""You will be given some words starting with the letter R. You name a proverb or saying that contains each one.""" 6 Revenge Revenge is sweet. Living well is the best revenge. 20130224 " ""You will be given some words starting with the letter R. You name a proverb or saying that contains each one.""" 7 Rod Spare the rod and spoil the child. 20130224 " ""You will be given some words starting with the letter R. You name a proverb or saying that contains each one.""" 8 Reward Virtue is its own reward. 20130224 " ""You will be given some words starting with the letter R. You name a proverb or saying that contains each one.""" 9 Receive It is better to give than to receive. 20130224 " ""You will be given some words starting with the letter R. You name a proverb or saying that contains each one.""" 10 Rest Don't rest on your laurels. No rest for the weary. No rest for the wicked. 20161023 " ""This is a game of categories based on the word """"guard."""" For each category" 1 first names of us president gerald, ulysses, alexander, ronald, dwight 20161023 " ""This is a game of categories based on the word """"guard."""" For each category" 2 rooms and areas of the house garage, upstairs, attic, rec room, driveway/dining room/den 20161023 " ""This is a game of categories based on the word """"guard."""" For each category" 3 three-letter exclamations gah/gee/grr, ugh, ack/aha, rah, duh 20161023 " ""This is a game of categories based on the word """"guard."""" For each category" 4 people and things seen at a wedding groom, usher, aisle, rice, doily/diamond/dress 20100103 It's time for Will Shortz's annual name-in-the-news quiz. 1 Chesley Sullenberger "the """"Miracle on the Hudson"""" pilot who safely landed a plane with 150 passengers, a co-pilot, and 3 flight attendants in the Hudson River minutes after a bird strike crippled both engines" 20100103 It's time for Will Shortz's annual name-in-the-news quiz. 2 Nadya Suleman "the """"Octomom""""" 20100103 It's time for Will Shortz's annual name-in-the-news quiz. 3 Carrie Prejean former Miss California, whose sex tape and opinion on same-sex marriage both ignited controversy 20100103 It's time for Will Shortz's annual name-in-the-news quiz. 4 Falcon Heene "the """"Balloon Boy""""" 20100103 It's time for Will Shortz's annual name-in-the-news quiz. 5 Euna Lee & Laura Ling the journalists detained in North Korea, brought home by Bill Clinton 20100103 It's time for Will Shortz's annual name-in-the-news quiz. 6 María BelÃn Chapur Argentinian mistress of South Carolina Gov. Mark Sanford 20100103 It's time for Will Shortz's annual name-in-the-news quiz. 7 Joe Wilson "who yelled """"You lie!"""" at President Barack Obama during an address to Congress" 20100103 It's time for Will Shortz's annual name-in-the-news quiz. 8 Bo a Portugese water dog who became First Dog 20100103 It's time for Will Shortz's annual name-in-the-news quiz. 9 Chesley Sullenberger "the """"Miracle on the Hudson"""" pilot who safely landed a plane with 150 passengers, a co-pilot, and 3 flight attendants in the Hudson River minutes after a bird strike crippled both engines" 20140126 For each word given, name a synonym in which the first two letters are the same as the second and third letters of the given word. For example, spin and pirouette. 1 though however 20140126 For each word given, name a synonym in which the first two letters are the same as the second and third letters of the given word. For example, spin and pirouette. 2 delete eliminate 20140126 For each word given, name a synonym in which the first two letters are the same as the second and third letters of the given word. For example, spin and pirouette. 3 preacher reverend 20140126 For each word given, name a synonym in which the first two letters are the same as the second and third letters of the given word. For example, spin and pirouette. 4 sort organize or order 20140126 For each word given, name a synonym in which the first two letters are the same as the second and third letters of the given word. For example, spin and pirouette. 5 squelch quiet or quash 20140126 For each word given, name a synonym in which the first two letters are the same as the second and third letters of the given word. For example, spin and pirouette. 6 statuesque tall 20140126 For each word given, name a synonym in which the first two letters are the same as the second and third letters of the given word. For example, spin and pirouette. 7 minor insignificant 20140126 For each word given, name a synonym in which the first two letters are the same as the second and third letters of the given word. For example, spin and pirouette. 8 place lay 20140126 For each word given, name a synonym in which the first two letters are the same as the second and third letters of the given word. For example, spin and pirouette. 9 cancel annul 20140126 For each word given, name a synonym in which the first two letters are the same as the second and third letters of the given word. For example, spin and pirouette. 10 stingy tight 20140126 For each word given, name a synonym in which the first two letters are the same as the second and third letters of the given word. For example, spin and pirouette. 11 afraid frightened 20140126 For each word given, name a synonym in which the first two letters are the same as the second and third letters of the given word. For example, spin and pirouette. 12 habitation tolerate 20140126 For each word given, name a synonym in which the first two letters are the same as the second and third letters of the given word. For example, spin and pirouette. 13 stomach tolerate 20140126 For each word given, name a synonym in which the first two letters are the same as the second and third letters of the given word. For example, spin and pirouette. 14 perfect error-free 20140126 For each word given, name a synonym in which the first two letters are the same as the second and third letters of the given word. For example, spin and pirouette. 15 little itzy-bitzy 20161002 " ""Today's on-air challenge has the regularly verbose title" 1 malted dairy drink/person who hasn't reached the age of 20 oval teen (Ovaltine) 20161002 " ""Today's on-air challenge has the regularly verbose title" 2 "peanut butter/if you read the first 7 letters of the alphabet ""A,B,C,D,F,G,H,"" what you do" skip E (Skippy) 20161002 " ""Today's on-air challenge has the regularly verbose title" 3 frozen meals/a clever person working at a financial institution bank wit (Banquet) 20161002 " ""Today's on-air challenge has the regularly verbose title" 4 "frozen foods/""""alas"""" uttered by former WVa Senator Robert" Byrd sigh (Birdseye) 20161002 " ""Today's on-air challenge has the regularly verbose title" 5 frozen dinners/to store animal coats in a safe place stow furs (Stouffer's) 20161002 " ""Today's on-air challenge has the regularly verbose title" 6 beer/every year I'm older but wiser (Budweiser) 20161002 " ""Today's on-air challenge has the regularly verbose title" 7 breakfats cereal/bank drafts made at a university in Houston Rice checks (Rice Chex) 20161002 " ""Today's on-air challenge has the regularly verbose title" 8 chewing gum/tiny amount of raw metal from a mine ore bit (Orbit) 20161002 " ""Today's on-air challenge has the regularly verbose title" 9 dessert topping/what singer Helen uses to make a horse go faster Reddy whip (Ready Whip) 20161002 " ""Today's on-air challenge has the regularly verbose title" 10 ice cream on a stick/where cooing birds go for a drink dove bar (Dove Bar) 20140622 "Every answer is a compound word or familiar two-word phrase or title in which each word has OU as its second and third letters. Example: Given """"heading to Antarctica,"""" you would say, """"Southbound.""""" 1 Thick bread popularized in America during the California gold rush sOUrdOUgh 20140622 "Every answer is a compound word or familiar two-word phrase or title in which each word has OU as its second and third letters. Example: Given """"heading to Antarctica,"""" you would say, """"Southbound.""""" 2 one who tlks a lot and at high volume lOUdmOUth 20140622 "Every answer is a compound word or familiar two-word phrase or title in which each word has OU as its second and third letters. Example: Given """"heading to Antarctica,"""" you would say, """"Southbound.""""" 3 where the king and his money are is """""Sing a Song of Sixpence"""" cOUnting hOUse" 20140622 "Every answer is a compound word or familiar two-word phrase or title in which each word has OU as its second and third letters. Example: Given """"heading to Antarctica,"""" you would say, """"Southbound.""""" 4 1963 hit for the Kingsmen LOUie LOUie 20140622 "Every answer is a compound word or familiar two-word phrase or title in which each word has OU as its second and third letters. Example: Given """"heading to Antarctica,"""" you would say, """"Southbound.""""" 5 start of a multipart meal, served in a bowl sOUp cOUrse 20140622 "Every answer is a compound word or familiar two-word phrase or title in which each word has OU as its second and third letters. Example: Given """"heading to Antarctica,"""" you would say, """"Southbound.""""" 6 classic French cabaret MOUlin ROUge 20140622 "Every answer is a compound word or familiar two-word phrase or title in which each word has OU as its second and third letters. Example: Given """"heading to Antarctica,"""" you would say, """"Southbound.""""" 7 playing area in tennis in which two players play dOUbles cOUrt 20140622 "Every answer is a compound word or familiar two-word phrase or title in which each word has OU as its second and third letters. Example: Given """"heading to Antarctica,"""" you would say, """"Southbound.""""" 8 activity of showing someone around the home hOUse tOUr 20140622 "Every answer is a compound word or familiar two-word phrase or title in which each word has OU as its second and third letters. Example: Given """"heading to Antarctica,"""" you would say, """"Southbound.""""" 9 in Patti LaBelle's 1975 hit """""Lady Marmalade,"""" the words before """"... avec moi"""" vOUlez vOUs cOUcher" 20140622 "Every answer is a compound word or familiar two-word phrase or title in which each word has OU as its second and third letters. Example: Given """"heading to Antarctica,"""" you would say, """"Southbound.""""" 10 the largest-circulation newspaper in Kentucky LOUisville JOUrnal-COUrier 20160807 Every answer today is a word or name containing the consecutive letters R-I-O. The first set of answers will end in -rio. 1 one less than a quartet trio 20160807 Every answer today is a word or name containing the consecutive letters R-I-O. The first set of answers will end in -rio. 2 where toronto is ontario 20160807 Every answer today is a word or name containing the consecutive letters R-I-O. The first set of answers will end in -rio. 3 a ladyTMs man (specifically a womanizer) lothario 20160807 Every answer today is a word or name containing the consecutive letters R-I-O. The first set of answers will end in -rio. 4 a written outline or a film or play scenario 20160807 Every answer today is a word or name containing the consecutive letters R-I-O. The first set of answers will end in -rio. 5 one who goes to battle warrior 20160807 Every answer today is a word or name containing the consecutive letters R-I-O. The first set of answers will end in -rio. 6 puppet on a string marionette 20160807 Every answer today is a word or name containing the consecutive letters R-I-O. The first set of answers will end in -rio. 7 very, very funny "riotous; will was looking for œhilarious.""""]" 20160807 Every answer today is a word or name containing the consecutive letters R-I-O. The first set of answers will end in -rio. 8 winning over everyone else victorious 20191215 " ""Write down eight different letters of the alphabet. Add an apostrophe. Then write the same eight letters in a different order. With proper spacing" 1 Compound words that become new words when their sides are switched: Bankroll, Bookwork, Downturn, Overhang Bankroll 20191215 " ""Write down eight different letters of the alphabet. Add an apostrophe. Then write the same eight letters in a different order. With proper spacing" 2 Words that name countries if you change a letter: Child, Fiance, Mother, Spawn Chile, France, Spain 20191215 " ""Write down eight different letters of the alphabet. Add an apostrophe. Then write the same eight letters in a different order. With proper spacing" 3 "Words containing """"A-I"""" that become new words if the A and I are reversed: Dairy, Plaint, Strain, Trail" Diary, Plaint, Trial 20191215 " ""Write down eight different letters of the alphabet. Add an apostrophe. Then write the same eight letters in a different order. With proper spacing" 4 Words that rhyme with names of domestic fowl: Bacon, Juice, Perky, Thicken Goose, Turkey, Chicken 20110227 "Each answer is a familiar word that starts with A-D and is an anagram of the given letters. For example, given the """"pat,"""" the answer would be """"adapt.""""" 1 led adle 20110227 "Each answer is a familiar word that starts with A-D and is an anagram of the given letters. For example, given the """"pat,"""" the answer would be """"adapt.""""" 2 tim admit 20110227 "Each answer is a familiar word that starts with A-D and is an anagram of the given letters. For example, given the """"pat,"""" the answer would be """"adapt.""""" 3 nor adorn 20110227 "Each answer is a familiar word that starts with A-D and is an anagram of the given letters. For example, given the """"pat,"""" the answer would be """"adapt.""""" 4 juts adjust 20110227 "Each answer is a familiar word that starts with A-D and is an anagram of the given letters. For example, given the """"pat,"""" the answer would be """"adapt.""""" 5 riot adroit 20110227 "Each answer is a familiar word that starts with A-D and is an anagram of the given letters. For example, given the """"pat,"""" the answer would be """"adapt.""""" 6 frit adrift 20110227 "Each answer is a familiar word that starts with A-D and is an anagram of the given letters. For example, given the """"pat,"""" the answer would be """"adapt.""""" 7 manta adamant 20110227 "Each answer is a familiar word that starts with A-D and is an anagram of the given letters. For example, given the """"pat,"""" the answer would be """"adapt.""""" 8 learn adrenal 20110227 "Each answer is a familiar word that starts with A-D and is an anagram of the given letters. For example, given the """"pat,"""" the answer would be """"adapt.""""" 9 serve adverse 20110227 "Each answer is a familiar word that starts with A-D and is an anagram of the given letters. For example, given the """"pat,"""" the answer would be """"adapt.""""" 10 nether adherent 20110227 "Each answer is a familiar word that starts with A-D and is an anagram of the given letters. For example, given the """"pat,"""" the answer would be """"adapt.""""" 11 Shimon admonish 20110227 "Each answer is a familiar word that starts with A-D and is an anagram of the given letters. For example, given the """"pat,"""" the answer would be """"adapt.""""" 12 octave advocate 20110227 "Each answer is a familiar word that starts with A-D and is an anagram of the given letters. For example, given the """"pat,"""" the answer would be """"adapt.""""" 13 balmier admirable 20110227 "Each answer is a familiar word that starts with A-D and is an anagram of the given letters. For example, given the """"pat,"""" the answer would be """"adapt.""""" 14 dilation additional 20110227 "Each answer is a familiar word that starts with A-D and is an anagram of the given letters. For example, given the """"pat,"""" the answer would be """"adapt.""""" 15 (two-word clue) event timers advertisement 20181118 I'm going to give you two words. Change one letter in the first word to name a category of things. And change one letter in the second word to name something in that category. 1 STAGE MAIZE state, Maine 20181118 I'm going to give you two words. Change one letter in the first word to name a category of things. And change one letter in the second word to name something in that category. 2 CORN QUARTET coin, quarter 20181118 I'm going to give you two words. Change one letter in the first word to name a category of things. And change one letter in the second word to name something in that category. 3 DUMBER FORTE number forty 20181118 I'm going to give you two words. Change one letter in the first word to name a category of things. And change one letter in the second word to name something in that category. 4 RING CHARGES King Charles 20181118 I'm going to give you two words. Change one letter in the first word to name a category of things. And change one letter in the second word to name something in that category. 5 RATIONALITY SWIMS nationality, Swiss 20121202 "Every answer is a familiar three-word phrase in the form """"____ of ____."""" The letters in the first and last words of each phrase are rearranged. You give the phrases. For example, """"Cat of Dog"""" becomes """"Act of God.""""" 1 taco of Mars coat of arms 20121202 "Every answer is a familiar three-word phrase in the form """"____ of ____."""" The letters in the first and last words of each phrase are rearranged. You give the phrases. For example, """"Cat of Dog"""" becomes """"Act of God.""""" 2 cork of sage Rock of Ages 20121202 "Every answer is a familiar three-word phrase in the form """"____ of ____."""" The letters in the first and last words of each phrase are rearranged. You give the phrases. For example, """"Cat of Dog"""" becomes """"Act of God.""""" 3 dies of charm ides of March 20121202 "Every answer is a familiar three-word phrase in the form """"____ of ____."""" The letters in the first and last words of each phrase are rearranged. You give the phrases. For example, """"Cat of Dog"""" becomes """"Act of God.""""" 4 danger of need Garden of Eden 20121202 "Every answer is a familiar three-word phrase in the form """"____ of ____."""" The letters in the first and last words of each phrase are rearranged. You give the phrases. For example, """"Cat of Dog"""" becomes """"Act of God.""""" 5 taste of hocks state of shock 20121202 "Every answer is a familiar three-word phrase in the form """"____ of ____."""" The letters in the first and last words of each phrase are rearranged. You give the phrases. For example, """"Cat of Dog"""" becomes """"Act of God.""""" 6 the A of tablet heat of battle 20121202 "Every answer is a familiar three-word phrase in the form """"____ of ____."""" The letters in the first and last words of each phrase are rearranged. You give the phrases. For example, """"Cat of Dog"""" becomes """"Act of God.""""" 7 mage of hornets Game of Thrones 20121202 "Every answer is a familiar three-word phrase in the form """"____ of ____."""" The letters in the first and last words of each phrase are rearranged. You give the phrases. For example, """"Cat of Dog"""" becomes """"Act of God.""""" 8 stapler of pairs plaster of Paris 20110417 "Every answer is a simile, in the form """"___ as a ___."""" You are given a rhyme for the first and last words. For example, given """"dead as a street,"""" the answer would be """"red as a beet.""""" 1 snappy as a tram HAPPY as a CLAM 20110417 "Every answer is a simile, in the form """"___ as a ___."""" You are given a rhyme for the first and last words. For example, given """"dead as a street,"""" the answer would be """"red as a beet.""""" 2 green as as thistle CLEAN as a WHISTLE 20110417 "Every answer is a simile, in the form """"___ as a ___."""" You are given a rhyme for the first and last words. For example, given """"dead as a street,"""" the answer would be """"red as a beet.""""" 4 White as a tether LIGHT as a FEATHER 20110417 "Every answer is a simile, in the form """"___ as a ___."""" You are given a rhyme for the first and last words. For example, given """"dead as a street,"""" the answer would be """"red as a beet.""""" 5 frail as a host PALE as a GHOST 20110417 "Every answer is a simile, in the form """"___ as a ___."""" You are given a rhyme for the first and last words. For example, given """"dead as a street,"""" the answer would be """"red as a beet.""""" 6 mute as a glutton CUTE as a BUTTON 20110417 "Every answer is a simile, in the form """"___ as a ___."""" You are given a rhyme for the first and last words. For example, given """"dead as a street,"""" the answer would be """"red as a beet.""""" 8 lit as a griddle FIT as a FIDDLE 20110417 "Every answer is a simile, in the form """"___ as a ___."""" You are given a rhyme for the first and last words. For example, given """"dead as a street,"""" the answer would be """"red as a beet.""""" 9 mental as a ram GENTLE as a LAMB 20110417 "Every answer is a simile, in the form """"___ as a ___."""" You are given a rhyme for the first and last words. For example, given """"dead as a street,"""" the answer would be """"red as a beet.""""" 10 bright as a thumb TIGHT as a DRUM 20110417 "Every answer is a simile, in the form """"___ as a ___."""" You are given a rhyme for the first and last words. For example, given """"dead as a street,"""" the answer would be """"red as a beet.""""" 11 lazy as a 'coon CRAZY as a LOON 20110417 "Every answer is a simile, in the form """"___ as a ___."""" You are given a rhyme for the first and last words. For example, given """"dead as a street,"""" the answer would be """"red as a beet.""""" 12 sweet as a fin NEAT as a PIN 20160626 "Every answer this week consists of two familiar phrases in the form """"___ of ___."""" In each case, the first word of the two phrases is the same. I'll give you the ending words. You tell me the full phrases." 1 cards, representatives house 20160626 "Every answer this week consists of two familiar phrases in the form """"___ of ___."""" In each case, the first word of the two phrases is the same. I'll give you the ending words. You tell me the full phrases." 2 thumb, law rule 20160626 "Every answer this week consists of two familiar phrases in the form """"___ of ___."""" In each case, the first word of the two phrases is the same. I'll give you the ending words. You tell me the full phrases." 3 view, order point 20160626 "Every answer this week consists of two familiar phrases in the form """"___ of ___."""" In each case, the first word of the two phrases is the same. I'll give you the ending words. You tell me the full phrases." 4 fare, rights bill 20160626 "Every answer this week consists of two familiar phrases in the form """"___ of ___."""" In each case, the first word of the two phrases is the same. I'll give you the ending words. You tell me the full phrases." 5 fire, wax ball 20160626 "Every answer this week consists of two familiar phrases in the form """"___ of ___."""" In each case, the first word of the two phrases is the same. I'll give you the ending words. You tell me the full phrases." 6 steel, war man 20160626 "Every answer this week consists of two familiar phrases in the form """"___ of ___."""" In each case, the first word of the two phrases is the same. I'll give you the ending words. You tell me the full phrases." 7 arms, mail coat 20160626 "Every answer this week consists of two familiar phrases in the form """"___ of ___."""" In each case, the first word of the two phrases is the same. I'll give you the ending words. You tell me the full phrases." 8 africa, plenty horn 20160626 "Every answer this week consists of two familiar phrases in the form """"___ of ___."""" In each case, the first word of the two phrases is the same. I'll give you the ending words. You tell me the full phrases." 9 golf, drinks round 20160626 "Every answer this week consists of two familiar phrases in the form """"___ of ___."""" In each case, the first word of the two phrases is the same. I'll give you the ending words. You tell me the full phrases." 10 stairs, fancy flight 20160626 "Every answer this week consists of two familiar phrases in the form """"___ of ___."""" In each case, the first word of the two phrases is the same. I'll give you the ending words. You tell me the full phrases." 11 chance, thrones game 20160821 Each answer today consists of the first and last names of a famous person, each with two syllables. Here are some made-up two-word phrases. The first word in the phrase rhymes with the person's first name, and the last word rhymes with the last name. 1 wavy rocket Davy Crockett 20160821 Each answer today consists of the first and last names of a famous person, each with two syllables. Here are some made-up two-word phrases. The first word in the phrase rhymes with the person's first name, and the last word rhymes with the last name. 2 melon seller* Helen Keller 20160821 Each answer today consists of the first and last names of a famous person, each with two syllables. Here are some made-up two-word phrases. The first word in the phrase rhymes with the person's first name, and the last word rhymes with the last name. 3 jolly bunter Holly Hunter 20160821 Each answer today consists of the first and last names of a famous person, each with two syllables. Here are some made-up two-word phrases. The first word in the phrase rhymes with the person's first name, and the last word rhymes with the last name. 4 many dodgers Kenny Rogers 20160821 Each answer today consists of the first and last names of a famous person, each with two syllables. Here are some made-up two-word phrases. The first word in the phrase rhymes with the person's first name, and the last word rhymes with the last name. 5 muddy dolly Buddy Holly 20160821 Each answer today consists of the first and last names of a famous person, each with two syllables. Here are some made-up two-word phrases. The first word in the phrase rhymes with the person's first name, and the last word rhymes with the last name. 6 Mormon tailor Norman Mailer 20160821 Each answer today consists of the first and last names of a famous person, each with two syllables. Here are some made-up two-word phrases. The first word in the phrase rhymes with the person's first name, and the last word rhymes with the last name. 7 journey landers Bernie Sanders 20160821 Each answer today consists of the first and last names of a famous person, each with two syllables. Here are some made-up two-word phrases. The first word in the phrase rhymes with the person's first name, and the last word rhymes with the last name. 8 fancy pagan Nancy Reagan 20160821 Each answer today consists of the first and last names of a famous person, each with two syllables. Here are some made-up two-word phrases. The first word in the phrase rhymes with the person's first name, and the last word rhymes with the last name. 9 messy Saxon Jesse Jackson 20160821 Each answer today consists of the first and last names of a famous person, each with two syllables. Here are some made-up two-word phrases. The first word in the phrase rhymes with the person's first name, and the last word rhymes with the last name. 10 trolley carton Dolly Parton 20160821 Each answer today consists of the first and last names of a famous person, each with two syllables. Here are some made-up two-word phrases. The first word in the phrase rhymes with the person's first name, and the last word rhymes with the last name. 11 Haiti Zurich Katie Couric 20140803 Today's puzzle is ba-a-a-ad. Every answer is a familiar two-word phrase or name in which the first word starts with BA- and the second word starts with D-. 1 an area of play for a pitcher, catcher or shortstop baseball diamond 20140803 Today's puzzle is ba-a-a-ad. Every answer is a familiar two-word phrase or name in which the first word starts with BA- and the second word starts with D-. 2 a rear entrance back door 20140803 Today's puzzle is ba-a-a-ad. Every answer is a familiar two-word phrase or name in which the first word starts with BA- and the second word starts with D-. 3 container for a casserole baking dish 20140803 Today's puzzle is ba-a-a-ad. Every answer is a familiar two-word phrase or name in which the first word starts with BA- and the second word starts with D-. 4 basic thing earned after four years of college bachelor's degree 20140803 Today's puzzle is ba-a-a-ad. Every answer is a familiar two-word phrase or name in which the first word starts with BA- and the second word starts with D-. 5 a waltz or minuet ballroom dance 20140803 Today's puzzle is ba-a-a-ad. Every answer is a familiar two-word phrase or name in which the first word starts with BA- and the second word starts with D-. 6 july 14 Bastille Day 20140803 Today's puzzle is ba-a-a-ad. Every answer is a familiar two-word phrase or name in which the first word starts with BA- and the second word starts with D-. 7 regimen of eating from all the five basic food groups balanced diet 20140803 Today's puzzle is ba-a-a-ad. Every answer is a familiar two-word phrase or name in which the first word starts with BA- and the second word starts with D-. 8 placing of money into a financial institution bank deposit 20140803 Today's puzzle is ba-a-a-ad. Every answer is a familiar two-word phrase or name in which the first word starts with BA- and the second word starts with D-. 9 nickname of the haitian dictator who was overthrown in 1986 baby doc 20140803 Today's puzzle is ba-a-a-ad. Every answer is a familiar two-word phrase or name in which the first word starts with BA- and the second word starts with D-. 10 what to do to the hatches before a storm batten down 20140803 Today's puzzle is ba-a-a-ad. Every answer is a familiar two-word phrase or name in which the first word starts with BA- and the second word starts with D-. 11 an annoying person in the car back seat driver 20140803 Today's puzzle is ba-a-a-ad. Every answer is a familiar two-word phrase or name in which the first word starts with BA- and the second word starts with D-. 12 leader of a musical group who waves a baton band director 20140803 Today's puzzle is ba-a-a-ad. Every answer is a familiar two-word phrase or name in which the first word starts with BA- and the second word starts with D-. 13 a large, deep-sounding percussion instrument bass drum 20140803 Today's puzzle is ba-a-a-ad. Every answer is a familiar two-word phrase or name in which the first word starts with BA- and the second word starts with D-. 14 performer in a tutu ballet dancer 20140803 Today's puzzle is ba-a-a-ad. Every answer is a familiar two-word phrase or name in which the first word starts with BA- and the second word starts with D-. 15 13 baker's dozen 20140803 Today's puzzle is ba-a-a-ad. Every answer is a familiar two-word phrase or name in which the first word starts with BA- and the second word starts with D-. 16 a companion of ken Barbie doll 20110925 "Name the well-known island concealed in consecutive letters of each sentence. For example: """"Helga is a saucy Prussian."""" The answer: """"Cyprus.""""" 1 It was the defendant I guarded Antigua 20110925 "Name the well-known island concealed in consecutive letters of each sentence. For example: """"Helga is a saucy Prussian."""" The answer: """"Cyprus.""""" 2 Listen to rita's maniacal laugh. Tasmania 20110925 "Name the well-known island concealed in consecutive letters of each sentence. For example: """"Helga is a saucy Prussian."""" The answer: """"Cyprus.""""" 3 The plane made a nice landing. Iceland 20110925 "Name the well-known island concealed in consecutive letters of each sentence. For example: """"Helga is a saucy Prussian."""" The answer: """"Cyprus.""""" 4 It's motrin I'd add to the shopping list. Trinidad 20110925 "Name the well-known island concealed in consecutive letters of each sentence. For example: """"Helga is a saucy Prussian."""" The answer: """"Cyprus.""""" 5 These are decors I can live with. Corsica 20110925 "Name the well-known island concealed in consecutive letters of each sentence. For example: """"Helga is a saucy Prussian."""" The answer: """"Cyprus.""""" 6 Did the baseball player get a hit, I wondered? Tahiti 20110925 "Name the well-known island concealed in consecutive letters of each sentence. For example: """"Helga is a saucy Prussian."""" The answer: """"Cyprus.""""" 7 The german hat tanked at the fashion show. Manhattan 20110925 "Name the well-known island concealed in consecutive letters of each sentence. For example: """"Helga is a saucy Prussian."""" The answer: """"Cyprus.""""" 8 Conan, tuck Ethan in bed. Nantucket 20110102 It's our annual year-end news quiz, prepared with the help of Kathy Baker and Tim Goodman. You are given names you've probably never heard of before 2010, but that became famous during the past 12 months. You must identify each person. 2 Tony Hayward former BP Oil executive 20110102 It's our annual year-end news quiz, prepared with the help of Kathy Baker and Tim Goodman. You are given names you've probably never heard of before 2010, but that became famous during the past 12 months. You must identify each person. 3 Shirley Sherrod former Agriculture Department official, fired for comments taken out of context 20110102 It's our annual year-end news quiz, prepared with the help of Kathy Baker and Tim Goodman. You are given names you've probably never heard of before 2010, but that became famous during the past 12 months. You must identify each person. 4 Elena Kagan Supreme Court justice who took office August 7th, 2010 20110102 It's our annual year-end news quiz, prepared with the help of Kathy Baker and Tim Goodman. You are given names you've probably never heard of before 2010, but that became famous during the past 12 months. You must identify each person. 5 Steven Slater JetBlue flight attendant who endeda flight in a rather unorthodoxmanner 20110102 It's our annual year-end news quiz, prepared with the help of Kathy Baker and Tim Goodman. You are given names you've probably never heard of before 2010, but that became famous during the past 12 months. You must identify each person. 6 Julian Assange founder of WikiLeaks 20110102 It's our annual year-end news quiz, prepared with the help of Kathy Baker and Tim Goodman. You are given names you've probably never heard of before 2010, but that became famous during the past 12 months. You must identify each person. 7 Kathryn Bigelow first woman to win Best Director Oscar for _The Hurt Locker_ 20110102 It's our annual year-end news quiz, prepared with the help of Kathy Baker and Tim Goodman. You are given names you've probably never heard of before 2010, but that became famous during the past 12 months. You must identify each person. 8 Liu Xiaobo Nobel Peace Prize winner from China, currently serving an 11-year prison sentenc 20110102 It's our annual year-end news quiz, prepared with the help of Kathy Baker and Tim Goodman. You are given names you've probably never heard of before 2010, but that became famous during the past 12 months. You must identify each person. 9 John Tyner "traveler famous for his quote aboutnot wanting his""""junk"""" touched by a TSA agent" 20110102 It's our annual year-end news quiz, prepared with the help of Kathy Baker and Tim Goodman. You are given names you've probably never heard of before 2010, but that became famous during the past 12 months. You must identify each person. 10 Mark Mezvinsky married Chelsea Clinton in July 20110102 It's our annual year-end news quiz, prepared with the help of Kathy Baker and Tim Goodman. You are given names you've probably never heard of before 2010, but that became famous during the past 12 months. You must identify each person. 11 Paul the Octopus famous for his World Cup predictions 20110102 It's our annual year-end news quiz, prepared with the help of Kathy Baker and Tim Goodman. You are given names you've probably never heard of before 2010, but that became famous during the past 12 months. You must identify each person. 12 Eyjafjallajokull the Icelandic volcanothat snarledinternationalair travelin 2010 20200802 I'm going to give you two words. The letters in the first word appear in left-to-right order, although not consecutively, in a state capital. The letters in the second word appear in left-to-right order in that capital's state. 1 Annals, Aryan Annapolis, Maryland 20200802 I'm going to give you two words. The letters in the first word appear in left-to-right order, although not consecutively, in a state capital. The letters in the second word appear in left-to-right order in that capital's state. 2 Singed, Lois Springfield, Illinois 20200802 I'm going to give you two words. The letters in the first word appear in left-to-right order, although not consecutively, in a state capital. The letters in the second word appear in left-to-right order in that capital's state. 3 Alas, Ford Tallahassee, Florida 20200802 I'm going to give you two words. The letters in the first word appear in left-to-right order, although not consecutively, in a state capital. The letters in the second word appear in left-to-right order in that capital's state. 4 Saul, Mines St Paul, Minnesota 20200802 I'm going to give you two words. The letters in the first word appear in left-to-right order, although not consecutively, in a state capital. The letters in the second word appear in left-to-right order in that capital's state. 5 Boon, Assets Boston, Massachusetts 20200802 I'm going to give you two words. The letters in the first word appear in left-to-right order, although not consecutively, in a state capital. The letters in the second word appear in left-to-right order in that capital's state. 6 Motor, Lama Mobile, Alabama 20200802 I'm going to give you two words. The letters in the first word appear in left-to-right order, although not consecutively, in a state capital. The letters in the second word appear in left-to-right order in that capital's state. 7 Mason, Coin Madison, Wisconsin 20200802 I'm going to give you two words. The letters in the first word appear in left-to-right order, although not consecutively, in a state capital. The letters in the second word appear in left-to-right order in that capital's state. 8 Prince, Resand Providence, Rhode Island 20140323 For each geographical place provided, change one letter to make a new, common word that has a different number of syllables than the geographical name. Note: The answer word can have either fewer or more syllables than the geographical name. 1 Peru perk; perp 20140323 For each geographical place provided, change one letter to make a new, common word that has a different number of syllables than the geographical name. Note: The answer word can have either fewer or more syllables than the geographical name. 2 Iran bran 20140323 For each geographical place provided, change one letter to make a new, common word that has a different number of syllables than the geographical name. Note: The answer word can have either fewer or more syllables than the geographical name. 3 Bonn bony 20140323 For each geographical place provided, change one letter to make a new, common word that has a different number of syllables than the geographical name. Note: The answer word can have either fewer or more syllables than the geographical name. 4 Quito quite; quits 20140323 For each geographical place provided, change one letter to make a new, common word that has a different number of syllables than the geographical name. Note: The answer word can have either fewer or more syllables than the geographical name. 5 Chile child; chill 20140323 For each geographical place provided, change one letter to make a new, common word that has a different number of syllables than the geographical name. Note: The answer word can have either fewer or more syllables than the geographical name. 6 Samoa samba 20140323 For each geographical place provided, change one letter to make a new, common word that has a different number of syllables than the geographical name. Note: The answer word can have either fewer or more syllables than the geographical name. 7 Tangier rangier; mangier 20140323 For each geographical place provided, change one letter to make a new, common word that has a different number of syllables than the geographical name. Note: The answer word can have either fewer or more syllables than the geographical name. 8 Texas texts 20140323 For each geographical place provided, change one letter to make a new, common word that has a different number of syllables than the geographical name. Note: The answer word can have either fewer or more syllables than the geographical name. 9 Boise poise; noise 20140323 For each geographical place provided, change one letter to make a new, common word that has a different number of syllables than the geographical name. Note: The answer word can have either fewer or more syllables than the geographical name. 10 Maui maul 20140323 For each geographical place provided, change one letter to make a new, common word that has a different number of syllables than the geographical name. Note: The answer word can have either fewer or more syllables than the geographical name. 11 Paris parts 20140323 For each geographical place provided, change one letter to make a new, common word that has a different number of syllables than the geographical name. Note: The answer word can have either fewer or more syllables than the geographical name. 12 France fiancà 20201115 "Today's puzzle is a tribute to Alex Trebek, the longtime host of """"Jeopardy!,"""" whom we lost last Sunday. Every answer is a familiar two-word phrase or name with the initials A-T." 1 Part of an orchard apple tree 20201115 "Today's puzzle is a tribute to Alex Trebek, the longtime host of """"Jeopardy!,"""" whom we lost last Sunday. Every answer is a familiar two-word phrase or name with the initials A-T." 2 Something controlled by a tower at O'Hare or LAX air traffic 20201115 "Today's puzzle is a tribute to Alex Trebek, the longtime host of """"Jeopardy!,"""" whom we lost last Sunday. Every answer is a familiar two-word phrase or name with the initials A-T." 3 Clumsy all thumbs 20201115 "Today's puzzle is a tribute to Alex Trebek, the longtime host of """"Jeopardy!,"""" whom we lost last Sunday. Every answer is a familiar two-word phrase or name with the initials A-T." 4 Clock setting in Anchorage or Fairbanks Alaska Time 20201115 "Today's puzzle is a tribute to Alex Trebek, the longtime host of """"Jeopardy!,"""" whom we lost last Sunday. Every answer is a familiar two-word phrase or name with the initials A-T." 5 Something you can hike from Georgia to Maine Appalachian Trail 20201115 "Today's puzzle is a tribute to Alex Trebek, the longtime host of """"Jeopardy!,"""" whom we lost last Sunday. Every answer is a familiar two-word phrase or name with the initials A-T." 6 LSD experience acid trip 20201115 "Today's puzzle is a tribute to Alex Trebek, the longtime host of """"Jeopardy!,"""" whom we lost last Sunday. Every answer is a familiar two-word phrase or name with the initials A-T." 7 Band of tissue that connects the calf muscles to the heel bone Achilles tendon 20201115 "Today's puzzle is a tribute to Alex Trebek, the longtime host of """"Jeopardy!,"""" whom we lost last Sunday. Every answer is a familiar two-word phrase or name with the initials A-T." 8 Sticky material for fastening things adhesive tape 20201115 "Today's puzzle is a tribute to Alex Trebek, the longtime host of """"Jeopardy!,"""" whom we lost last Sunday. Every answer is a familiar two-word phrase or name with the initials A-T." 9 Worker with tigers and elephants at old circuses animal trainer 20201115 "Today's puzzle is a tribute to Alex Trebek, the longtime host of """"Jeopardy!,"""" whom we lost last Sunday. Every answer is a familiar two-word phrase or name with the initials A-T." 10 Something affixed to a wall or ceiling in a sound studio acoustic tile 20110821 "Every word is a compound word, or a familiar two-word phrase or name, with the consecutive letters L — E — S — T. Specifically, the first word ends in L-E and second part starts with S-T. For example, """"activity in a seminary."""" The answer would be """"BibLE STudy.""""" 1 a place to park a schwinn or raleigh bicycle stand 20110821 "Every word is a compound word, or a familiar two-word phrase or name, with the consecutive letters L — E — S — T. Specifically, the first word ends in L-E and second part starts with S-T. For example, """"activity in a seminary."""" The answer would be """"BibLE STudy.""""" 2 nickname for oklahoma, based on its distinctive shape panhandle state 20110821 "Every word is a compound word, or a familiar two-word phrase or name, with the consecutive letters L — E — S — T. Specifically, the first word ends in L-E and second part starts with S-T. For example, """"activity in a seminary."""" The answer would be """"BibLE STudy.""""" 3 it has a wick running through it. candle stick 20110821 "Every word is a compound word, or a familiar two-word phrase or name, with the consecutive letters L — E — S — T. Specifically, the first word ends in L-E and second part starts with S-T. For example, """"activity in a seminary."""" The answer would be """"BibLE STudy.""""" 4 where to buy an ipad or ipod apple store 20110821 "Every word is a compound word, or a familiar two-word phrase or name, with the consecutive letters L — E — S — T. Specifically, the first word ends in L-E and second part starts with S-T. For example, """"activity in a seminary."""" The answer would be """"BibLE STudy.""""" 5 ______ galactica battlestar 20110821 "Every word is a compound word, or a familiar two-word phrase or name, with the consecutive letters L — E — S — T. Specifically, the first word ends in L-E and second part starts with S-T. For example, """"activity in a seminary."""" The answer would be """"BibLE STudy.""""" 6 road marker every 5,280 feet milestone 20110821 "Every word is a compound word, or a familiar two-word phrase or name, with the consecutive letters L — E — S — T. Specifically, the first word ends in L-E and second part starts with S-T. For example, """"activity in a seminary."""" The answer would be """"BibLE STudy.""""" 7 michelangelo's david or the venus de milo marble statue 20110821 "Every word is a compound word, or a familiar two-word phrase or name, with the consecutive letters L — E — S — T. Specifically, the first word ends in L-E and second part starts with S-T. For example, """"activity in a seminary."""" The answer would be """"BibLE STudy.""""" 8 "exclamation meaning """"nonsense!""""" fiddlesticks 20110821 "Every word is a compound word, or a familiar two-word phrase or name, with the consecutive letters L — E — S — T. Specifically, the first word ends in L-E and second part starts with S-T. For example, """"activity in a seminary."""" The answer would be """"BibLE STudy.""""" 9 romantic novelist who has had more than 25 number one hard-cover(ed) fiction best-sellers danielle steel 20110821 "Every word is a compound word, or a familiar two-word phrase or name, with the consecutive letters L — E — S — T. Specifically, the first word ends in L-E and second part starts with S-T. For example, """"activity in a seminary."""" The answer would be """"BibLE STudy.""""" 10 road-safety feature, often stretching across a highway that causes a car to vibrate rumble strip 20110821 "Every word is a compound word, or a familiar two-word phrase or name, with the consecutive letters L — E — S — T. Specifically, the first word ends in L-E and second part starts with S-T. For example, """"activity in a seminary."""" The answer would be """"BibLE STudy.""""" 11 legendary venue for blues, in memphis beale street 20110821 "Every word is a compound word, or a familiar two-word phrase or name, with the consecutive letters L — E — S — T. Specifically, the first word ends in L-E and second part starts with S-T. For example, """"activity in a seminary."""" The answer would be """"BibLE STudy.""""" 12 tour for a political candidate traveling by train whistle stop 20171119 "Today's theme is the World Boxing Association — the W.B.A. Every answer is a compound word or a familiar two words in which the first word starts with """"W"""" and the second word starts """"BA-.""""" 1 small cart with handles that a gardener pushes WheelBArrow 20171119 "Today's theme is the World Boxing Association — the W.B.A. Every answer is a compound word or a familiar two words in which the first word starts with """"W"""" and the second word starts """"BA-.""""" 2 top of a pair of pants that might be elasticized WaistBAnd 20171119 "Today's theme is the World Boxing Association — the W.B.A. Every answer is a compound word or a familiar two words in which the first word starts with """"W"""" and the second word starts """"BA-.""""" 3 round, white toy with holes that you hit with a bat Whiffle BAll 20171119 "Today's theme is the World Boxing Association — the W.B.A. Every answer is a compound word or a familiar two words in which the first word starts with """"W"""" and the second word starts """"BA-.""""" 4 missile in a summer fight that lands with a splat Water BAlloon 20171119 "Today's theme is the World Boxing Association — the W.B.A. Every answer is a compound word or a familiar two words in which the first word starts with """"W"""" and the second word starts """"BA-.""""" 5 area disputed by Israelis and Palestinians West BAnk 20171119 "Today's theme is the World Boxing Association — the W.B.A. Every answer is a compound word or a familiar two words in which the first word starts with """"W"""" and the second word starts """"BA-.""""" 6 container for indoor trash Waste BAsket 20190331 I'm going to give you two words. Change one letter in the first word to name a category of things. And change one letter in the second word to name something in that category. 1 foal & thicken fowl & chicken 20190331 I'm going to give you two words. Change one letter in the first word to name a category of things. And change one letter in the second word to name something in that category. 2 blower & phony flower & peony 20190331 I'm going to give you two words. Change one letter in the first word to name a category of things. And change one letter in the second word to name something in that category. 3 ration & fiance nation & france 20190331 I'm going to give you two words. Change one letter in the first word to name a category of things. And change one letter in the second word to name something in that category. 4 alloy & bronze weight & ounce 20190331 I'm going to give you two words. Change one letter in the first word to name a category of things. And change one letter in the second word to name something in that category. 5 weight & pound material & nylon 20170514 Every answer today is the name of an airline. Name each one from its anagram. Ex. ALL + E (2 words) i--> EL AL1. LEAD + T2. TUNED + I3. SANTA + Q4. CARMINE + A5. FOR RENT + I6. STEAMER + I 7. LEAF ROT + O8. HALF NUTS + A 1 LEAD + T Delta 20170514 Every answer today is the name of an airline. Name each one from its anagram. Ex. ALL + E (2 words) i--> EL AL1. LEAD + T2. TUNED + I3. SANTA + Q4. CARMINE + A5. FOR RENT + I6. STEAMER + I 7. LEAF ROT + O8. HALF NUTS + A 2 TUNED + I United 20170514 Every answer today is the name of an airline. Name each one from its anagram. Ex. ALL + E (2 words) i--> EL AL1. LEAD + T2. TUNED + I3. SANTA + Q4. CARMINE + A5. FOR RENT + I6. STEAMER + I 7. LEAF ROT + O8. HALF NUTS + A 3 SANTA + Q Qantas 20170514 Every answer today is the name of an airline. Name each one from its anagram. Ex. ALL + E (2 words) i--> EL AL1. LEAD + T2. TUNED + I3. SANTA + Q4. CARMINE + A5. FOR RENT + I6. STEAMER + I 7. LEAF ROT + O8. HALF NUTS + A 4 CARMINE + A American 20170514 Every answer today is the name of an airline. Name each one from its anagram. Ex. ALL + E (2 words) i--> EL AL1. LEAD + T2. TUNED + I3. SANTA + Q4. CARMINE + A5. FOR RENT + I6. STEAMER + I 7. LEAF ROT + O8. HALF NUTS + A 5 FOR RENT + I Frontier 20170514 Every answer today is the name of an airline. Name each one from its anagram. Ex. ALL + E (2 words) i--> EL AL1. LEAD + T2. TUNED + I3. SANTA + Q4. CARMINE + A5. FOR RENT + I6. STEAMER + I 7. LEAF ROT + O8. HALF NUTS + A 6 STEAMER + I Emirates 20170514 Every answer today is the name of an airline. Name each one from its anagram. Ex. ALL + E (2 words) i--> EL AL1. LEAD + T2. TUNED + I3. SANTA + Q4. CARMINE + A5. FOR RENT + I6. STEAMER + I 7. LEAF ROT + O8. HALF NUTS + A 7 LEAF ROT + O Aeroflot 20170514 Every answer today is the name of an airline. Name each one from its anagram. Ex. ALL + E (2 words) i--> EL AL1. LEAD + T2. TUNED + I3. SANTA + Q4. CARMINE + A5. FOR RENT + I6. STEAMER + I 7. LEAF ROT + O8. HALF NUTS + A 8 HALF NUTS + A Lufthansa 20101212 "You are given some words and names from a specific category and must name something else in the category that can be spelled from the letters in the clue. For example, given """"Dodge,"""" the answer would be """"Geo."""" Both are cars, and """"Geo"""" can be formed from the letters of """"Dodge.""""" 1 grape (4 letter answer) pear fruits 20101212 "You are given some words and names from a specific category and must name something else in the category that can be spelled from the letters in the clue. For example, given """"Dodge,"""" the answer would be """"Geo."""" Both are cars, and """"Geo"""" can be formed from the letters of """"Dodge.""""" 2 Stockholm (4 letters) Oslo Scandinavian capitals 20101212 "You are given some words and names from a specific category and must name something else in the category that can be spelled from the letters in the clue. For example, given """"Dodge,"""" the answer would be """"Geo."""" Both are cars, and """"Geo"""" can be formed from the letters of """"Dodge.""""" 3 Yom Kippur (5 letters) Purim Jewish holidays 20101212 "You are given some words and names from a specific category and must name something else in the category that can be spelled from the letters in the clue. For example, given """"Dodge,"""" the answer would be """"Geo."""" Both are cars, and """"Geo"""" can be formed from the letters of """"Dodge.""""" 4 grackle (4 letters) lark birds 20101212 "You are given some words and names from a specific category and must name something else in the category that can be spelled from the letters in the clue. For example, given """"Dodge,"""" the answer would be """"Geo."""" Both are cars, and """"Geo"""" can be formed from the letters of """"Dodge.""""" 5 Cherokee (5 letters) Creek Native American tribes 20101212 "You are given some words and names from a specific category and must name something else in the category that can be spelled from the letters in the clue. For example, given """"Dodge,"""" the answer would be """"Geo."""" Both are cars, and """"Geo"""" can be formed from the letters of """"Dodge.""""" 6 boa constrictor (5) cobra snakes 20101212 "You are given some words and names from a specific category and must name something else in the category that can be spelled from the letters in the clue. For example, given """"Dodge,"""" the answer would be """"Geo."""" Both are cars, and """"Geo"""" can be formed from the letters of """"Dodge.""""" 7 Cather (5) Harte American authors 20101212 "You are given some words and names from a specific category and must name something else in the category that can be spelled from the letters in the clue. For example, given """"Dodge,"""" the answer would be """"Geo."""" Both are cars, and """"Geo"""" can be formed from the letters of """"Dodge.""""" 8 maple (3 and 4) elm, palm trees 20100724 "Every answer is a familiar phrase or title in the form of """"____ of ____,"""" where the word before """"of"""" starts with the letter R. You are given the word that follows """"of"""" and must come up with the phrase. For example, given """"Saturn,"""" the answer would be """"rings.""""" 1 hope ray 20100724 "Every answer is a familiar phrase or title in the form of """"____ of ____,"""" where the word before """"of"""" starts with the letter R. You are given the word that follows """"of"""" and must come up with the phrase. For example, given """"Saturn,"""" the answer would be """"rings.""""" 2 passage rite 20100724 "Every answer is a familiar phrase or title in the form of """"____ of ____,"""" where the word before """"of"""" starts with the letter R. You are given the word that follows """"of"""" and must come up with the phrase. For example, given """"Saturn,"""" the answer would be """"rings.""""" 4 applause round 20100724 "Every answer is a familiar phrase or title in the form of """"____ of ____,"""" where the word before """"of"""" starts with the letter R. You are given the word that follows """"of"""" and must come up with the phrase. For example, given """"Saturn,"""" the answer would be """"rings.""""" 5 law rule 20100724 "Every answer is a familiar phrase or title in the form of """"____ of ____,"""" where the word before """"of"""" starts with the letter R. You are given the word that follows """"of"""" and must come up with the phrase. For example, given """"Saturn,"""" the answer would be """"rings.""""" 6 fortune reversal 20100724 "Every answer is a familiar phrase or title in the form of """"____ of ____,"""" where the word before """"of"""" starts with the letter R. You are given the word that follows """"of"""" and must come up with the phrase. For example, given """"Saturn,"""" the answer would be """"rings.""""" 7 terror reign 20100724 "Every answer is a familiar phrase or title in the form of """"____ of ____,"""" where the word before """"of"""" starts with the letter R. You are given the word that follows """"of"""" and must come up with the phrase. For example, given """"Saturn,"""" the answer would be """"rings.""""" 8 fire ring 20100724 "Every answer is a familiar phrase or title in the form of """"____ of ____,"""" where the word before """"of"""" starts with the letter R. You are given the word that follows """"of"""" and must come up with the phrase. For example, given """"Saturn,"""" the answer would be """"rings.""""" 9 no return river 20100724 "Every answer is a familiar phrase or title in the form of """"____ of ____,"""" where the word before """"of"""" starts with the letter R. You are given the word that follows """"of"""" and must come up with the phrase. For example, given """"Saturn,"""" the answer would be """"rings.""""" 10 the mill run 20100724 "Every answer is a familiar phrase or title in the form of """"____ of ____,"""" where the word before """"of"""" starts with the letter R. You are given the word that follows """"of"""" and must come up with the phrase. For example, given """"Saturn,"""" the answer would be """"rings.""""" 11 the road rules 20100724 "Every answer is a familiar phrase or title in the form of """"____ of ____,"""" where the word before """"of"""" starts with the letter R. You are given the word that follows """"of"""" and must come up with the phrase. For example, given """"Saturn,"""" the answer would be """"rings.""""" 12 the jedi return 20100724 "Every answer is a familiar phrase or title in the form of """"____ of ____,"""" where the word before """"of"""" starts with the letter R. You are given the word that follows """"of"""" and must come up with the phrase. For example, given """"Saturn,"""" the answer would be """"rings.""""" 13 the sphinx riddle 20100724 "Every answer is a familiar phrase or title in the form of """"____ of ____,"""" where the word before """"of"""" starts with the letter R. You are given the word that follows """"of"""" and must come up with the phrase. For example, given """"Saturn,"""" the answer would be """"rings.""""" 14 the ancient mariner rhyme 20100724 "Every answer is a familiar phrase or title in the form of """"____ of ____,"""" where the word before """"of"""" starts with the letter R. You are given the word that follows """"of"""" and must come up with the phrase. For example, given """"Saturn,"""" the answer would be """"rings.""""" 15 the lost arc raiders 20100724 "Every answer is a familiar phrase or title in the form of """"____ of ____,"""" where the word before """"of"""" starts with the letter R. You are given the word that follows """"of"""" and must come up with the phrase. For example, given """"Saturn,"""" the answer would be """"rings.""""" 16 order robert's rules 20120408 "Every answer is the name of a popular music group, past or present. You'll be given clues in which two letters in the group's name have been changed. For example, given """"The Bench Boss,"""" the answer would be """"The Beach Boys,"""" after changing the N in """"Bench"""" to an A and the first S of """"Boss"""" to a Y." 1 the bentley the beatles 20120408 "Every answer is the name of a popular music group, past or present. You'll be given clues in which two letters in the group's name have been changed. For example, given """"The Bench Boss,"""" the answer would be """"The Beach Boys,"""" after changing the N in """"Bench"""" to an A and the first S of """"Boss"""" to a Y." 2 the donkeys the monkees 20120408 "Every answer is the name of a popular music group, past or present. You'll be given clues in which two letters in the group's name have been changed. For example, given """"The Bench Boss,"""" the answer would be """"The Beach Boys,"""" after changing the N in """"Bench"""" to an A and the first S of """"Boss"""" to a Y." 3 the novice the police 20120408 "Every answer is the name of a popular music group, past or present. You'll be given clues in which two letters in the group's name have been changed. For example, given """"The Bench Boss,"""" the answer would be """"The Beach Boys,"""" after changing the N in """"Bench"""" to an A and the first S of """"Boss"""" to a Y." 4 the pet shop toys the pet shop boys 20120408 "Every answer is the name of a popular music group, past or present. You'll be given clues in which two letters in the group's name have been changed. For example, given """"The Bench Boss,"""" the answer would be """"The Beach Boys,"""" after changing the N in """"Bench"""" to an A and the first S of """"Boss"""" to a Y." 5 the banjoes the bangles 20120408 "Every answer is the name of a popular music group, past or present. You'll be given clues in which two letters in the group's name have been changed. For example, given """"The Bench Boss,"""" the answer would be """"The Beach Boys,"""" after changing the N in """"Bench"""" to an A and the first S of """"Boss"""" to a Y." 6 the blank eyed pear the black eyed peas 20120408 "Every answer is the name of a popular music group, past or present. You'll be given clues in which two letters in the group's name have been changed. For example, given """"The Bench Boss,"""" the answer would be """"The Beach Boys,"""" after changing the N in """"Bench"""" to an A and the first S of """"Boss"""" to a Y." 7 the printer sitters the pointer sisters 20120408 "Every answer is the name of a popular music group, past or present. You'll be given clues in which two letters in the group's name have been changed. For example, given """"The Bench Boss,"""" the answer would be """"The Beach Boys,"""" after changing the N in """"Bench"""" to an A and the first S of """"Boss"""" to a Y." 8 the glass rooms the grass roots 20120408 "Every answer is the name of a popular music group, past or present. You'll be given clues in which two letters in the group's name have been changed. For example, given """"The Bench Boss,"""" the answer would be """"The Beach Boys,"""" after changing the N in """"Bench"""" to an A and the first S of """"Boss"""" to a Y." 9 the slashing bumpkins the smashing pumpkins 20120408 "Every answer is the name of a popular music group, past or present. You'll be given clues in which two letters in the group's name have been changed. For example, given """"The Bench Boss,"""" the answer would be """"The Beach Boys,"""" after changing the N in """"Bench"""" to an A and the first S of """"Boss"""" to a Y." 10 the thongs the troggs 20120408 "Every answer is the name of a popular music group, past or present. You'll be given clues in which two letters in the group's name have been changed. For example, given """"The Bench Boss,"""" the answer would be """"The Beach Boys,"""" after changing the N in """"Bench"""" to an A and the first S of """"Boss"""" to a Y." 11 the bad city hollers the bay city roller 20120408 "Every answer is the name of a popular music group, past or present. You'll be given clues in which two letters in the group's name have been changed. For example, given """"The Bench Boss,"""" the answer would be """"The Beach Boys,"""" after changing the N in """"Bench"""" to an A and the first S of """"Boss"""" to a Y." 12 the manacles the miracles 20120408 "Every answer is the name of a popular music group, past or present. You'll be given clues in which two letters in the group's name have been changed. For example, given """"The Bench Boss,"""" the answer would be """"The Beach Boys,"""" after changing the N in """"Bench"""" to an A and the first S of """"Boss"""" to a Y." 13 the walking herds the talking heads 20110116 "Every answer is a familiar two-word phrase, name or title containing the consecutive letters I-N-D, as in """"independent."""" For example, given the clue """"19th-century British prime minister,"""" the answer would be """"Benjamin Disraeli.""""" 1 ceremony performed by Native Americans to end a drought raIN Dance 20110116 "Every answer is a familiar two-word phrase, name or title containing the consecutive letters I-N-D, as in """"independent."""" For example, given the clue """"19th-century British prime minister,"""" the answer would be """"Benjamin Disraeli.""""" 2 beauty is only this skIN Deep 20110116 "Every answer is a familiar two-word phrase, name or title containing the consecutive letters I-N-D, as in """"independent."""" For example, given the clue """"19th-century British prime minister,"""" the answer would be """"Benjamin Disraeli.""""" 3 President Roosevelt's given names FranklIN Delano 20110116 "Every answer is a familiar two-word phrase, name or title containing the consecutive letters I-N-D, as in """"independent."""" For example, given the clue """"19th-century British prime minister,"""" the answer would be """"Benjamin Disraeli.""""" 4 title character in a Charles Dickens mystery EdwIN Drood 20110116 "Every answer is a familiar two-word phrase, name or title containing the consecutive letters I-N-D, as in """"independent."""" For example, given the clue """"19th-century British prime minister,"""" the answer would be """"Benjamin Disraeli.""""" 5 fast food franchise with baked goods and coffee DunkIN' Donuts 20110116 "Every answer is a familiar two-word phrase, name or title containing the consecutive letters I-N-D, as in """"independent."""" For example, given the clue """"19th-century British prime minister,"""" the answer would be """"Benjamin Disraeli.""""" 6 "action star in """"The Fast and the Furious""""" VIN Diesel 20110116 "Every answer is a familiar two-word phrase, name or title containing the consecutive letters I-N-D, as in """"independent."""" For example, given the clue """"19th-century British prime minister,"""" the answer would be """"Benjamin Disraeli.""""" 7 a railroad station traIN Depot 20110116 "Every answer is a familiar two-word phrase, name or title containing the consecutive letters I-N-D, as in """"independent."""" For example, given the clue """"19th-century British prime minister,"""" the answer would be """"Benjamin Disraeli.""""" 8 Coke, Pepsi or 7-Up, for example, when bought at a restaurant fountaIN Drink 20110116 "Every answer is a familiar two-word phrase, name or title containing the consecutive letters I-N-D, as in """"independent."""" For example, given the clue """"19th-century British prime minister,"""" the answer would be """"Benjamin Disraeli.""""" 9 soft drink made by Pepsi MountaIN Dew 20110116 "Every answer is a familiar two-word phrase, name or title containing the consecutive letters I-N-D, as in """"independent."""" For example, given the clue """"19th-century British prime minister,"""" the answer would be """"Benjamin Disraeli.""""" 10 Cleveland's leading newspaper PlaIN Dealer 20110116 "Every answer is a familiar two-word phrase, name or title containing the consecutive letters I-N-D, as in """"independent."""" For example, given the clue """"19th-century British prime minister,"""" the answer would be """"Benjamin Disraeli.""""" 11 Duke Ellington jazz standard SatIN Doll 20110116 "Every answer is a familiar two-word phrase, name or title containing the consecutive letters I-N-D, as in """"independent."""" For example, given the clue """"19th-century British prime minister,"""" the answer would be """"Benjamin Disraeli.""""" 12 person who interprets the news to his political advantage spIN Doctor 20110116 "Every answer is a familiar two-word phrase, name or title containing the consecutive letters I-N-D, as in """"independent."""" For example, given the clue """"19th-century British prime minister,"""" the answer would be """"Benjamin Disraeli.""""" 13 departure of scientists and intellectuals from one country braIN Drain 20110116 "Every answer is a familiar two-word phrase, name or title containing the consecutive letters I-N-D, as in """"independent."""" For example, given the clue """"19th-century British prime minister,"""" the answer would be """"Benjamin Disraeli.""""" 14 book in which you might look up Anno Domini or e pluribus unum LatIN Dictionary 20131020 This week we have a celebrity edition of the Puzzle. Comedian Paula Poundstone is taking on our challenge. Poundstone is also a regular panelist on NPR's Wait Wait Don't Tell Me. 1 jupiter earth 20131020 This week we have a celebrity edition of the Puzzle. Comedian Paula Poundstone is taking on our challenge. Poundstone is also a regular panelist on NPR's Wait Wait Don't Tell Me. 2 monday friday 20131020 This week we have a celebrity edition of the Puzzle. Comedian Paula Poundstone is taking on our challenge. Poundstone is also a regular panelist on NPR's Wait Wait Don't Tell Me. 3 aries aquarius 20131020 This week we have a celebrity edition of the Puzzle. Comedian Paula Poundstone is taking on our challenge. Poundstone is also a regular panelist on NPR's Wait Wait Don't Tell Me. 4 center fielder catcher 20131020 This week we have a celebrity edition of the Puzzle. Comedian Paula Poundstone is taking on our challenge. Poundstone is also a regular panelist on NPR's Wait Wait Don't Tell Me. 5 antarctica africa 20131020 This week we have a celebrity edition of the Puzzle. Comedian Paula Poundstone is taking on our challenge. Poundstone is also a regular panelist on NPR's Wait Wait Don't Tell Me. 6 columbia brown 20170312 Here are some categories. For each one, name something in the category whose first and last letters are the same as the first and last letters of the category. For example: College --> COLGATE, among other answers [both start with C and end in E] 1 MontH MarcH 20170312 Here are some categories. For each one, name something in the category whose first and last letters are the same as the first and last letters of the category. For example: College --> COLGATE, among other answers [both start with C and end in E] 3 Hotel chaiN HiltoN 20170312 Here are some categories. For each one, name something in the category whose first and last letters are the same as the first and last letters of the category. For example: College --> COLGATE, among other answers [both start with C and end in E] 4 Country in AsiA ChinA 20170312 Here are some categories. For each one, name something in the category whose first and last letters are the same as the first and last letters of the category. For example: College --> COLGATE, among other answers [both start with C and end in E] 5 Dog breeD DachshunD 20170312 Here are some categories. For each one, name something in the category whose first and last letters are the same as the first and last letters of the category. For example: College --> COLGATE, among other answers [both start with C and end in E] 6 Chemical elemenT CobalT 20170312 Here are some categories. For each one, name something in the category whose first and last letters are the same as the first and last letters of the category. For example: College --> COLGATE, among other answers [both start with C and end in E] 7 CarmakeR ChrysleR 20170312 Here are some categories. For each one, name something in the category whose first and last letters are the same as the first and last letters of the category. For example: College --> COLGATE, among other answers [both start with C and end in E] 8 ChesspiecE CastlE 20170312 Here are some categories. For each one, name something in the category whose first and last letters are the same as the first and last letters of the category. For example: College --> COLGATE, among other answers [both start with C and end in E] 9 Washington localE (2 words) White HousE 20170312 Here are some categories. For each one, name something in the category whose first and last letters are the same as the first and last letters of the category. For example: College --> COLGATE, among other answers [both start with C and end in E] 10 Federal holidaY (3 words) Fourth of JulY 20100626 "Every answer is the first name of one famous person and the last name of another, in which the names are anagrams. For example, """"Dolly Lloyd"""" for """"Dolly Parton and Christopher Lloyd."""" Will Shortz gives the listener the people's other names, the listener gives the names that are anagrams. For example, given """"Christopher Parton,"""" the answer would be """"Dolly Lloyd.""""" 1 Ralph Agassi Andre Nader 20100626 "Every answer is the first name of one famous person and the last name of another, in which the names are anagrams. For example, """"Dolly Lloyd"""" for """"Dolly Parton and Christopher Lloyd."""" Will Shortz gives the listener the people's other names, the listener gives the names that are anagrams. For example, given """"Christopher Parton,"""" the answer would be """"Dolly Lloyd.""""" 2 Yitzhak Williams Brian Rabin 20100626 "Every answer is the first name of one famous person and the last name of another, in which the names are anagrams. For example, """"Dolly Lloyd"""" for """"Dolly Parton and Christopher Lloyd."""" Will Shortz gives the listener the people's other names, the listener gives the names that are anagrams. For example, given """"Christopher Parton,"""" the answer would be """"Dolly Lloyd.""""" 3 Sophia Michaels Lorne Loren 20100626 "Every answer is the first name of one famous person and the last name of another, in which the names are anagrams. For example, """"Dolly Lloyd"""" for """"Dolly Parton and Christopher Lloyd."""" Will Shortz gives the listener the people's other names, the listener gives the names that are anagrams. For example, given """"Christopher Parton,"""" the answer would be """"Dolly Lloyd.""""" 4 Suze Polanski Roman Orman 20100626 "Every answer is the first name of one famous person and the last name of another, in which the names are anagrams. For example, """"Dolly Lloyd"""" for """"Dolly Parton and Christopher Lloyd."""" Will Shortz gives the listener the people's other names, the listener gives the names that are anagrams. For example, given """"Christopher Parton,"""" the answer would be """"Dolly Lloyd.""""" 5 Benedict Reagan Ronald Arnold 20100626 "Every answer is the first name of one famous person and the last name of another, in which the names are anagrams. For example, """"Dolly Lloyd"""" for """"Dolly Parton and Christopher Lloyd."""" Will Shortz gives the listener the people's other names, the listener gives the names that are anagrams. For example, given """"Christopher Parton,"""" the answer would be """"Dolly Lloyd.""""" 6 Johnny Powers Austin Unitas 20100626 "Every answer is the first name of one famous person and the last name of another, in which the names are anagrams. For example, """"Dolly Lloyd"""" for """"Dolly Parton and Christopher Lloyd."""" Will Shortz gives the listener the people's other names, the listener gives the names that are anagrams. For example, given """"Christopher Parton,"""" the answer would be """"Dolly Lloyd.""""" 7 Walt Poitier Sidney Disney 20100626 "Every answer is the first name of one famous person and the last name of another, in which the names are anagrams. For example, """"Dolly Lloyd"""" for """"Dolly Parton and Christopher Lloyd."""" Will Shortz gives the listener the people's other names, the listener gives the names that are anagrams. For example, given """"Christopher Parton,"""" the answer would be """"Dolly Lloyd.""""" 8 Laurence Hemingway Ernest Stern 20100626 "Every answer is the first name of one famous person and the last name of another, in which the names are anagrams. For example, """"Dolly Lloyd"""" for """"Dolly Parton and Christopher Lloyd."""" Will Shortz gives the listener the people's other names, the listener gives the names that are anagrams. For example, given """"Christopher Parton,"""" the answer would be """"Dolly Lloyd.""""" 9 Frankie Hansen Liane Laine 20090905 "For each word given, add the letter """"a"""" and rearrange all the letters to name a well-known island. For example, if the clue is """"trees,"""" add an """"a"""" to get the answer: Easter." 1 mug guam 20090905 "For each word given, add the letter """"a"""" and rearrange all the letters to name a well-known island. For example, if the clue is """"trees,"""" add an """"a"""" to get the answer: Easter." 2 kew wake 20090905 "For each word given, add the letter """"a"""" and rearrange all the letters to name a well-known island. For example, if the clue is """"trees,"""" add an """"a"""" to get the answer: Easter." 3 amos samoa 20090905 "For each word given, add the letter """"a"""" and rearrange all the letters to name a well-known island. For example, if the clue is """"trees,"""" add an """"a"""" to get the answer: Easter." 4 crip capri 20090905 "For each word given, add the letter """"a"""" and rearrange all the letters to name a well-known island. For example, if the clue is """"trees,"""" add an """"a"""" to get the answer: Easter." 5 gobot tobago 20090905 "For each word given, add the letter """"a"""" and rearrange all the letters to name a well-known island. For example, if the clue is """"trees,"""" add an """"a"""" to get the answer: Easter." 7 danger grenada 20090905 "For each word given, add the letter """"a"""" and rearrange all the letters to name a well-known island. For example, if the clue is """"trees,"""" add an """"a"""" to get the answer: Easter." 8 stamina tasmania 20090905 "For each word given, add the letter """"a"""" and rearrange all the letters to name a well-known island. For example, if the clue is """"trees,"""" add an """"a"""" to get the answer: Easter." 9 reposing singapore 20110807 "Every answer is a familiar two-word phrase or name in which the first word starts with T-O and the second word starts with P. For example, given, """"a person who is responsible for organizing a series of live concerts,"""" the answer would be """"tour promoter.""""" 1 classic scene in Animal House TOga Party 20110807 "Every answer is a familiar two-word phrase or name in which the first word starts with T-O and the second word starts with P. For example, given, """"a person who is responsible for organizing a series of live concerts,"""" the answer would be """"tour promoter.""""" 2 Charmin or Cottonelle product TOilet Paper 20110807 "Every answer is a familiar two-word phrase or name in which the first word starts with T-O and the second word starts with P. For example, given, """"a person who is responsible for organizing a series of live concerts,"""" the answer would be """"tour promoter.""""" 3 an Indian carving with many faces TOtem Pole 20110807 "Every answer is a familiar two-word phrase or name in which the first word starts with T-O and the second word starts with P. For example, given, """"a person who is responsible for organizing a series of live concerts,"""" the answer would be """"tour promoter.""""" 4 TOurnament Poker TOwnament Poker 20110807 "Every answer is a familiar two-word phrase or name in which the first word starts with T-O and the second word starts with P. For example, given, """"a person who is responsible for organizing a series of live concerts,"""" the answer would be """"tour promoter.""""" 5 line of destruction after a violent storm TOll Plaza 20110807 "Every answer is a familiar two-word phrase or name in which the first word starts with T-O and the second word starts with P. For example, given, """"a person who is responsible for organizing a series of live concerts,"""" the answer would be """"tour promoter.""""" 6 singer with the Heartbreakers TOdd Palin 20110807 "Every answer is a familiar two-word phrase or name in which the first word starts with T-O and the second word starts with P. For example, given, """"a person who is responsible for organizing a series of live concerts,"""" the answer would be """"tour promoter.""""" 7 TOmato Paste TOmato Paste 20110807 "Every answer is a familiar two-word phrase or name in which the first word starts with T-O and the second word starts with P. For example, given, """"a person who is responsible for organizing a series of live concerts,"""" the answer would be """"tour promoter.""""" 8 walkway along a canal TOe Path 20110807 "Every answer is a familiar two-word phrase or name in which the first word starts with T-O and the second word starts with P. For example, given, """"a person who is responsible for organizing a series of live concerts,"""" the answer would be """"tour promoter.""""" 9 popular hybrid car TOyota Prius 20110807 "Every answer is a familiar two-word phrase or name in which the first word starts with T-O and the second word starts with P. For example, given, """"a person who is responsible for organizing a series of live concerts,"""" the answer would be """"tour promoter.""""" 10 chocolaty candy on a stick TOotsie Pop 20110807 "Every answer is a familiar two-word phrase or name in which the first word starts with T-O and the second word starts with P. For example, given, """"a person who is responsible for organizing a series of live concerts,"""" the answer would be """"tour promoter.""""" 11 classic scene in Animal House TOga Party 20110807 "Every answer is a familiar two-word phrase or name in which the first word starts with T-O and the second word starts with P. For example, given, """"a person who is responsible for organizing a series of live concerts,"""" the answer would be """"tour promoter.""""" 12 Charmin or Cottonelle product TOilet Paper 20180121 "I'm going to give you some four-letter words. For each one, add one letter in front and one letter in back to complete a familiar six-letter word. No proper names are allowed, and you can't add an """"s"""" or """"d"""" at the end of a five-letter word to get the six-letter one." 1 MOOT SmootH 20180121 "I'm going to give you some four-letter words. For each one, add one letter in front and one letter in back to complete a familiar six-letter word. No proper names are allowed, and you can't add an """"s"""" or """"d"""" at the end of a five-letter word to get the six-letter one." 2 HANG ChangE 20180121 "I'm going to give you some four-letter words. For each one, add one letter in front and one letter in back to complete a familiar six-letter word. No proper names are allowed, and you can't add an """"s"""" or """"d"""" at the end of a five-letter word to get the six-letter one." 3 WENT TwentY 20180121 "I'm going to give you some four-letter words. For each one, add one letter in front and one letter in back to complete a familiar six-letter word. No proper names are allowed, and you can't add an """"s"""" or """"d"""" at the end of a five-letter word to get the six-letter one." 4 NIGH KnighT 20180121 "I'm going to give you some four-letter words. For each one, add one letter in front and one letter in back to complete a familiar six-letter word. No proper names are allowed, and you can't add an """"s"""" or """"d"""" at the end of a five-letter word to get the six-letter one." 5 TREE StreeT 20180121 "I'm going to give you some four-letter words. For each one, add one letter in front and one letter in back to complete a familiar six-letter word. No proper names are allowed, and you can't add an """"s"""" or """"d"""" at the end of a five-letter word to get the six-letter one." 6 REFI PrefiX 20180121 "I'm going to give you some four-letter words. For each one, add one letter in front and one letter in back to complete a familiar six-letter word. No proper names are allowed, and you can't add an """"s"""" or """"d"""" at the end of a five-letter word to get the six-letter one." 7 BRAD AbradE 20180121 "I'm going to give you some four-letter words. For each one, add one letter in front and one letter in back to complete a familiar six-letter word. No proper names are allowed, and you can't add an """"s"""" or """"d"""" at the end of a five-letter word to get the six-letter one." 8 CRAW ScrawL 20180121 "I'm going to give you some four-letter words. For each one, add one letter in front and one letter in back to complete a familiar six-letter word. No proper names are allowed, and you can't add an """"s"""" or """"d"""" at the end of a five-letter word to get the six-letter one." 9 ISLE Misled 20180121 "I'm going to give you some four-letter words. For each one, add one letter in front and one letter in back to complete a familiar six-letter word. No proper names are allowed, and you can't add an """"s"""" or """"d"""" at the end of a five-letter word to get the six-letter one." 10 PLAN UplanD 20180121 "I'm going to give you some four-letter words. For each one, add one letter in front and one letter in back to complete a familiar six-letter word. No proper names are allowed, and you can't add an """"s"""" or """"d"""" at the end of a five-letter word to get the six-letter one." 11 AFAR SafarI 20131215 "Every answer is a five-letter word. You'll be given a clue for the word. Besides giving you a direct hint to the answer, the clue will also contain the answer in consecutive letters. For example, given """"push over hard,"""" you would say """"shove.""""" 1 It's taken to kill a vampire STAKE 20131215 "Every answer is a five-letter word. You'll be given a clue for the word. Besides giving you a direct hint to the answer, the clue will also contain the answer in consecutive letters. For example, given """"push over hard,"""" you would say """"shove.""""" 2 Title for Theresa in the Catholic Church SAIN 20131215 "Every answer is a five-letter word. You'll be given a clue for the word. Besides giving you a direct hint to the answer, the clue will also contain the answer in consecutive letters. For example, given """"push over hard,"""" you would say """"shove.""""" 3 Using shorTERERESentences, say SHORT 20131215 "Every answer is a five-letter word. You'll be given a clue for the word. Besides giving you a direct hint to the answer, the clue will also contain the answer in consecutive letters. For example, given """"push over hard,"""" you would say """"shove.""""" 4 Animal with much impish behavior IMPISH 20131215 "Every answer is a five-letter word. You'll be given a clue for the word. Besides giving you a direct hint to the answer, the clue will also contain the answer in consecutive letters. For example, given """"push over hard,"""" you would say """"shove.""""" 5 Beverage put on ice with gin PUT 20131215 "Every answer is a five-letter word. You'll be given a clue for the word. Besides giving you a direct hint to the answer, the clue will also contain the answer in consecutive letters. For example, given """"push over hard,"""" you would say """"shove.""""" 6 It can lead rains away from a house. LEAD 20131215 "Every answer is a five-letter word. You'll be given a clue for the word. Besides giving you a direct hint to the answer, the clue will also contain the answer in consecutive letters. For example, given """"push over hard,"""" you would say """"shove.""""" 7 One of the places to rent in a mall RENT 20131215 "Every answer is a five-letter word. You'll be given a clue for the word. Besides giving you a direct hint to the answer, the clue will also contain the answer in consecutive letters. For example, given """"push over hard,"""" you would say """"shove.""""" 8 It might be built with big, loose blocks of ice. BUILD 20131215 "Every answer is a five-letter word. You'll be given a clue for the word. Besides giving you a direct hint to the answer, the clue will also contain the answer in consecutive letters. For example, given """"push over hard,"""" you would say """"shove.""""" 9 Kellogg's All-Bran, Drano or Twinkies BRAN 20131215 "Every answer is a five-letter word. You'll be given a clue for the word. Besides giving you a direct hint to the answer, the clue will also contain the answer in consecutive letters. For example, given """"push over hard,"""" you would say """"shove.""""" 10 Home of Dali, Velasquez and other famOUSPAINters FAMOUS 20131215 "Every answer is a five-letter word. You'll be given a clue for the word. Besides giving you a direct hint to the answer, the clue will also contain the answer in consecutive letters. For example, given """"push over hard,"""" you would say """"shove.""""" 11 Song that worships almighty God WORSHIP 20131215 "Every answer is a five-letter word. You'll be given a clue for the word. Besides giving you a direct hint to the answer, the clue will also contain the answer in consecutive letters. For example, given """"push over hard,"""" you would say """"shove.""""" 12 Pleasant annual visitor VISIT 20190106 I'm going to give you some 6-letter words. Insert 2 letters in the exact center of each one to complete a common, uncapitalized 8-letter word. 1 Barque BarBEque 20190106 I'm going to give you some 6-letter words. Insert 2 letters in the exact center of each one to complete a common, uncapitalized 8-letter word. 2 Corral CorPOral 20190106 I'm going to give you some 6-letter words. Insert 2 letters in the exact center of each one to complete a common, uncapitalized 8-letter word. 3 Evince EviDEnce 20190106 I'm going to give you some 6-letter words. Insert 2 letters in the exact center of each one to complete a common, uncapitalized 8-letter word. 4 Innate InnOVate 20190106 I'm going to give you some 6-letter words. Insert 2 letters in the exact center of each one to complete a common, uncapitalized 8-letter word. 5 Intact IntERact 20190106 I'm going to give you some 6-letter words. Insert 2 letters in the exact center of each one to complete a common, uncapitalized 8-letter word. 6 Oblate OblIGate 20190106 I'm going to give you some 6-letter words. Insert 2 letters in the exact center of each one to complete a common, uncapitalized 8-letter word. 7 Panama PanORama 20190106 I'm going to give you some 6-letter words. Insert 2 letters in the exact center of each one to complete a common, uncapitalized 8-letter word. 8 Vanish VanQUish 20190922 I'm going to give you clues for two words. The first word has an R somewhere within it. Change the R to two S's, and you'll get a new word that answers the second clue. 1 ruck / Game on a 64-square board Cher, chess 20190922 I'm going to give you clues for two words. The first word has an R somewhere within it. Change the R to two S's, and you'll get a new word that answers the second clue. 2 Tiny opening in the skin / Group helping a sheriff pore, posse 20190922 I'm going to give you clues for two words. The first word has an R somewhere within it. Change the R to two S's, and you'll get a new word that answers the second clue. 3 Cry on a golf course / Choreographer Bob fore, Fosse 20190922 I'm going to give you clues for two words. The first word has an R somewhere within it. Change the R to two S's, and you'll get a new word that answers the second clue. 4 State as fact / Having fallen in social status (from the French) declare, declasse 20190922 I'm going to give you clues for two words. The first word has an R somewhere within it. Change the R to two S's, and you'll get a new word that answers the second clue. 5 "The U.S. flag is called """"Old ___"""" / Photo with a shiny surface" Glory, glossy 20130901 Every answer is a familiar two-word phrase with the consecutive letters of S-H-H. Specifically, the first word in the answer will end in SH, and the second will start with H. 1 period in the morning and evening when traffic is the busiest rush hour 20130901 Every answer is a familiar two-word phrase with the consecutive letters of S-H-H. Specifically, the first word in the answer will end in SH, and the second will start with H. 2 former name of belize british honduras 20130901 Every answer is a familiar two-word phrase with the consecutive letters of S-H-H. Specifically, the first word in the answer will end in SH, and the second will start with H. 3 head wear for a motorcyclist or a race car driver crash helmet 20130901 Every answer is a familiar two-word phrase with the consecutive letters of S-H-H. Specifically, the first word in the answer will end in SH, and the second will start with H. 4 yom kippur or hanukkah jewish holidays 20130901 Every answer is a familiar two-word phrase with the consecutive letters of S-H-H. Specifically, the first word in the answer will end in SH, and the second will start with H. 5 a place to throw garbage trash/ash/rubbish heap 20130901 Every answer is a familiar two-word phrase with the consecutive letters of S-H-H. Specifically, the first word in the answer will end in SH, and the second will start with H. 6 a place to stick a worm fish hook 20130901 Every answer is a familiar two-word phrase with the consecutive letters of S-H-H. Specifically, the first word in the answer will end in SH, and the second will start with H. 7 woodwind instrument resembling an oboe english horn 20130901 Every answer is a familiar two-word phrase with the consecutive letters of S-H-H. Specifically, the first word in the answer will end in SH, and the second will start with H. 8 a cheap restaurant or diner hash house 20130901 Every answer is a familiar two-word phrase with the consecutive letters of S-H-H. Specifically, the first word in the answer will end in SH, and the second will start with H. 9 instructions above a sink to restaurant employees wash hands 20130901 Every answer is a familiar two-word phrase with the consecutive letters of S-H-H. Specifically, the first word in the answer will end in SH, and the second will start with H. 10 best-selling song by a wide margin smash hit 20130901 Every answer is a familiar two-word phrase with the consecutive letters of S-H-H. Specifically, the first word in the answer will end in SH, and the second will start with H. 11 the jewish new year rosh hashanah 20130901 Every answer is a familiar two-word phrase with the consecutive letters of S-H-H. Specifically, the first word in the answer will end in SH, and the second will start with H. 12 building next to a church used for social activities parish house 20130901 Every answer is a familiar two-word phrase with the consecutive letters of S-H-H. Specifically, the first word in the answer will end in SH, and the second will start with H. 13 1960's-70's pitcher for the a's and the yankees who played on five world series winning teams catfish hunter 20130901 Every answer is a familiar two-word phrase with the consecutive letters of S-H-H. Specifically, the first word in the answer will end in SH, and the second will start with H. 14 elevated, far-north part of the british isle scottish highlands 20130901 Every answer is a familiar two-word phrase with the consecutive letters of S-H-H. Specifically, the first word in the answer will end in SH, and the second will start with H. 15 part of manhattan with a large latino population spanish harlem 20130901 Every answer is a familiar two-word phrase with the consecutive letters of S-H-H. Specifically, the first word in the answer will end in SH, and the second will start with H. 16 a place to call if you need tech assistance for an apple computer macintosh help 20201206 I'm going to read you some sentences. Three consecutive words somewhere in each sentence are the first three words of a familiar proverb or saying. Tell me what it is. Example: Put out a saucer of milk when the cat's hungry. --> When the cat's away the mice will play.1. As meteorologists know, every cloud has water droplets.2. Variety is the daily publication of show business.3. The surgeon put a stitch in the gaping wound.4. Through the mountains the road to the next town is very twisty.5. On average the proof of most whiskey is 80.6. In this motel a picture is hanging over every bed.7. The joke starts: a priest, a fool, and a lawyer walk into a bar. 1 As meteorologists know, every cloud has water droplets. Every cloud has a silver lining 20201206 I'm going to read you some sentences. Three consecutive words somewhere in each sentence are the first three words of a familiar proverb or saying. Tell me what it is. Example: Put out a saucer of milk when the cat's hungry. --> When the cat's away the mice will play.1. As meteorologists know, every cloud has water droplets.2. Variety is the daily publication of show business.3. The surgeon put a stitch in the gaping wound.4. Through the mountains the road to the next town is very twisty.5. On average the proof of most whiskey is 80.6. In this motel a picture is hanging over every bed.7. The joke starts: a priest, a fool, and a lawyer walk into a bar. 2 Variety is the daily publication of show business Variety is the spice of life 20201206 I'm going to read you some sentences. Three consecutive words somewhere in each sentence are the first three words of a familiar proverb or saying. Tell me what it is. Example: Put out a saucer of milk when the cat's hungry. --> When the cat's away the mice will play.1. As meteorologists know, every cloud has water droplets.2. Variety is the daily publication of show business.3. The surgeon put a stitch in the gaping wound.4. Through the mountains the road to the next town is very twisty.5. On average the proof of most whiskey is 80.6. In this motel a picture is hanging over every bed.7. The joke starts: a priest, a fool, and a lawyer walk into a bar. 3 The surgeon put a stitch in the gaping wound A stitch in time saves nine 20201206 I'm going to read you some sentences. Three consecutive words somewhere in each sentence are the first three words of a familiar proverb or saying. Tell me what it is. Example: Put out a saucer of milk when the cat's hungry. --> When the cat's away the mice will play.1. As meteorologists know, every cloud has water droplets.2. Variety is the daily publication of show business.3. The surgeon put a stitch in the gaping wound.4. Through the mountains the road to the next town is very twisty.5. On average the proof of most whiskey is 80.6. In this motel a picture is hanging over every bed.7. The joke starts: a priest, a fool, and a lawyer walk into a bar. 4 The proof of most whiskey is 80 The proof of the pudding is in the eating 20201206 I'm going to read you some sentences. Three consecutive words somewhere in each sentence are the first three words of a familiar proverb or saying. Tell me what it is. Example: Put out a saucer of milk when the cat's hungry. --> When the cat's away the mice will play.1. As meteorologists know, every cloud has water droplets.2. Variety is the daily publication of show business.3. The surgeon put a stitch in the gaping wound.4. Through the mountains the road to the next town is very twisty.5. On average the proof of most whiskey is 80.6. In this motel a picture is hanging over every bed.7. The joke starts: a priest, a fool, and a lawyer walk into a bar. 5 In this motel a picture is hanging over every bed A picture is worth a thousand words 20201206 I'm going to read you some sentences. Three consecutive words somewhere in each sentence are the first three words of a familiar proverb or saying. Tell me what it is. Example: Put out a saucer of milk when the cat's hungry. --> When the cat's away the mice will play.1. As meteorologists know, every cloud has water droplets.2. Variety is the daily publication of show business.3. The surgeon put a stitch in the gaping wound.4. Through the mountains the road to the next town is very twisty.5. On average the proof of most whiskey is 80.6. In this motel a picture is hanging over every bed.7. The joke starts: a priest, a fool, and a lawyer walk into a bar. 6 The joke starts: a priest, a fool, and a lawyer walk into a bar A fool and his money are soon parted 20170924 For each word, change one letter in it to get a common boy's name. 1 charges charles 20170924 For each word, change one letter in it to get a common boy's name. 2 tennis dennis 20170924 For each word, change one letter in it to get a common boy's name. 3 cargos carlos 20170924 For each word, change one letter in it to get a common boy's name. 4 everest everett 20170924 For each word, change one letter in it to get a common boy's name. 5 wavier javier or xavier 20170924 For each word, change one letter in it to get a common boy's name. 6 coward howard 20170924 For each word, change one letter in it to get a common boy's name. 7 change theœg. norman 20170924 For each word, change one letter in it to get a common boy's name. 8 english prime minister chamberlin walter 20110424 "You are given the first names of famous people and must figure out their last names. The last two letters of the first name, when reversed, are the first two letters of the last name. For example, given """"Brad,"""" the answer would be """"Davis,"""" as in Brad Davis, the actor." 1 mamie eisenhower 20110424 "You are given the first names of famous people and must figure out their last names. The last two letters of the first name, when reversed, are the first two letters of the last name. For example, given """"Brad,"""" the answer would be """"Davis,"""" as in Brad Davis, the actor." 2 arthur rubinstein 20110424 "You are given the first names of famous people and must figure out their last names. The last two letters of the first name, when reversed, are the first two letters of the last name. For example, given """"Brad,"""" the answer would be """"Davis,"""" as in Brad Davis, the actor." 3 christopher reeve 20110424 "You are given the first names of famous people and must figure out their last names. The last two letters of the first name, when reversed, are the first two letters of the last name. For example, given """"Brad,"""" the answer would be """"Davis,"""" as in Brad Davis, the actor." 4 richard dreyfus 20110424 "You are given the first names of famous people and must figure out their last names. The last two letters of the first name, when reversed, are the first two letters of the last name. For example, given """"Brad,"""" the answer would be """"Davis,"""" as in Brad Davis, the actor." 5 dana andrews 20110424 "You are given the first names of famous people and must figure out their last names. The last two letters of the first name, when reversed, are the first two letters of the last name. For example, given """"Brad,"""" the answer would be """"Davis,"""" as in Brad Davis, the actor." 6 georges seurat 20110424 "You are given the first names of famous people and must figure out their last names. The last two letters of the first name, when reversed, are the first two letters of the last name. For example, given """"Brad,"""" the answer would be """"Davis,"""" as in Brad Davis, the actor." 7 gertrude ederle 20110424 "You are given the first names of famous people and must figure out their last names. The last two letters of the first name, when reversed, are the first two letters of the last name. For example, given """"Brad,"""" the answer would be """"Davis,"""" as in Brad Davis, the actor." 8 jonas salk 20110424 "You are given the first names of famous people and must figure out their last names. The last two letters of the first name, when reversed, are the first two letters of the last name. For example, given """"Brad,"""" the answer would be """"Davis,"""" as in Brad Davis, the actor." 9 laila ali 20110424 "You are given the first names of famous people and must figure out their last names. The last two letters of the first name, when reversed, are the first two letters of the last name. For example, given """"Brad,"""" the answer would be """"Davis,"""" as in Brad Davis, the actor." 10 david dinkins 20110424 "You are given the first names of famous people and must figure out their last names. The last two letters of the first name, when reversed, are the first two letters of the last name. For example, given """"Brad,"""" the answer would be """"Davis,"""" as in Brad Davis, the actor." 11 deborah harry 20110424 "You are given the first names of famous people and must figure out their last names. The last two letters of the first name, when reversed, are the first two letters of the last name. For example, given """"Brad,"""" the answer would be """"Davis,"""" as in Brad Davis, the actor." 12 eleanor roosevelt 20171203 I'm going to give you clues for two words. Reverse the last four letters of the answer to the first clue to get the word that answers the second clue. 1 apportion, coral island allot, atoll 20171203 I'm going to give you clues for two words. Reverse the last four letters of the answer to the first clue to get the word that answers the second clue. 2 distress signal shot into the air, wild, as an animal flare, feral 20171203 I'm going to give you clues for two words. Reverse the last four letters of the answer to the first clue to get the word that answers the second clue. 3 rugged, many-headed serpent of myth hardy, hydra 20171203 I'm going to give you clues for two words. Reverse the last four letters of the answer to the first clue to get the word that answers the second clue. 4 nosy person / implement for eating soup snoop, spoon 20171203 I'm going to give you clues for two words. Reverse the last four letters of the answer to the first clue to get the word that answers the second clue. 5 call the wrong telephone number, lost misdial, mislaid 20171203 I'm going to give you clues for two words. Reverse the last four letters of the answer to the first clue to get the word that answers the second clue. 6 fragility, place to order a sandwich to go delicateness, delicatessen 20130804 "This week's puzzle is called """"What's in a Name?"""" Every answer consists of the names of two famous people. The last name of the first person is an anagram of the first name of the last person. Given the non-anagram parts of the names, you identify the people. For example, given """"Madeleine"""" and """"Aaron,"""" you would say """"Kahn"""" and """"Hank.""""" 1 Jay, Coward Jay LENO, NOEL Coward 20130804 "This week's puzzle is called """"What's in a Name?"""" Every answer consists of the names of two famous people. The last name of the first person is an anagram of the first name of the last person. Given the non-anagram parts of the names, you identify the people. For example, given """"Madeleine"""" and """"Aaron,"""" you would say """"Kahn"""" and """"Hank.""""" 2 Janet, Wolfe Janet RENO, NERO Wolfe 20130804 "This week's puzzle is called """"What's in a Name?"""" Every answer consists of the names of two famous people. The last name of the first person is an anagram of the first name of the last person. Given the non-anagram parts of the names, you identify the people. For example, given """"Madeleine"""" and """"Aaron,"""" you would say """"Kahn"""" and """"Hank.""""" 3 Yogi, McEntire Yogi BEAR, REBA McEntire 20130804 "This week's puzzle is called """"What's in a Name?"""" Every answer consists of the names of two famous people. The last name of the first person is an anagram of the first name of the last person. Given the non-anagram parts of the names, you identify the people. For example, given """"Madeleine"""" and """"Aaron,"""" you would say """"Kahn"""" and """"Hank.""""" 4 Bob, Carter Bob DYLAN, LYNDA Carter 20130804 "This week's puzzle is called """"What's in a Name?"""" Every answer consists of the names of two famous people. The last name of the first person is an anagram of the first name of the last person. Given the non-anagram parts of the names, you identify the people. For example, given """"Madeleine"""" and """"Aaron,"""" you would say """"Kahn"""" and """"Hank.""""" 5 Roger, Montague Roger MOORE, ROMEO Montague 20130804 "This week's puzzle is called """"What's in a Name?"""" Every answer consists of the names of two famous people. The last name of the first person is an anagram of the first name of the last person. Given the non-anagram parts of the names, you identify the people. For example, given """"Madeleine"""" and """"Aaron,"""" you would say """"Kahn"""" and """"Hank.""""" 6 Lily, Berle Lily TOMLIN, MILTON Berle 20130804 "This week's puzzle is called """"What's in a Name?"""" Every answer consists of the names of two famous people. The last name of the first person is an anagram of the first name of the last person. Given the non-anagram parts of the names, you identify the people. For example, given """"Madeleine"""" and """"Aaron,"""" you would say """"Kahn"""" and """"Hank.""""" 7 Benedict, Reagan Benedict ARNOLD, RONALD Reaga 20160918 For the following words starting with the letters S, E and P — as in September — find a word that can precede each to complete a familiar two-word phrase. 1 space, entendre, parking double 20160918 For the following words starting with the letters S, E and P — as in September — find a word that can precede each to complete a familiar two-word phrase. 3 sheep, eye, Panther black 20160918 For the following words starting with the letters S, E and P — as in September — find a word that can precede each to complete a familiar two-word phrase. 4 shoe, elbow, pro* tennis 20160918 For the following words starting with the letters S, E and P — as in September — find a word that can precede each to complete a familiar two-word phrase. 5 saddle, effect, pocket side 20160918 For the following words starting with the letters S, E and P — as in September — find a word that can precede each to complete a familiar two-word phrase. 6 second, ends, personality split 20160918 For the following words starting with the letters S, E and P — as in September — find a word that can precede each to complete a familiar two-word phrase. 7 street, examination, purposes cross 20160918 For the following words starting with the letters S, E and P — as in September — find a word that can precede each to complete a familiar two-word phrase. 8 strike, edition, place first 20160918 For the following words starting with the letters S, E and P — as in September — find a word that can precede each to complete a familiar two-word phrase. 9 stocking, Eve, present Christmas 20160918 For the following words starting with the letters S, E and P — as in September — find a word that can precede each to complete a familiar two-word phrase. 10 Sox, elephant, Pages White 20190127 I'm going to read you some famous advertising slogans, past and present. Each contains the advertiser's name, but anagrammed. You name the advertiser. 1 Please don't squeeze the RICH MAN Charmin 20190127 I'm going to read you some famous advertising slogans, past and present. Each contains the advertiser's name, but anagrammed. You name the advertiser. 2 You're in good hands with A TALL SET Allstate 20190127 I'm going to read you some famous advertising slogans, past and present. Each contains the advertiser's name, but anagrammed. You name the advertiser. 3 The best part of waking up is GOLFERS in your cup Folgers 20190127 I'm going to read you some famous advertising slogans, past and present. Each contains the advertiser's name, but anagrammed. You name the advertiser. 4 I don't want to grow up. I'm a U.S. STORY kid Toys 'R' Us 20190127 I'm going to read you some famous advertising slogans, past and present. Each contains the advertiser's name, but anagrammed. You name the advertiser. 5 This is not your father's IDLE BLOOMS Oldsmobile 20190127 I'm going to read you some famous advertising slogans, past and present. Each contains the advertiser's name, but anagrammed. You name the advertiser. 6 Nobody better lay a finger on my BETTER FIG URN Butterfinger 20190127 I'm going to read you some famous advertising slogans, past and present. Each contains the advertiser's name, but anagrammed. You name the advertiser. 7 Pardon me ” Do you have any YOUNGER POP? Grey Poupon 20190127 I'm going to read you some famous advertising slogans, past and present. Each contains the advertiser's name, but anagrammed. You name the advertiser. 8 Nobody doesn't like A RESALE Sara Lee 20190127 I'm going to read you some famous advertising slogans, past and present. Each contains the advertiser's name, but anagrammed. You name the advertiser. 9 There are some things money can't buy. For everything else, there's DREAM CARTS Mastercard 20111204 Change one letter in each word of a made-up, two-word phrase to get two new words that will start a familiar proverb or saying. Determining which letters to change is up to you. 1 TREAT MINUS Great minds think alike. 20111204 Change one letter in each word of a made-up, two-word phrase to get two new words that will start a familiar proverb or saying. Determining which letters to change is up to you. 2 MOOSELIES Loose lips sink ships. 20111204 Change one letter in each word of a made-up, two-word phrase to get two new words that will start a familiar proverb or saying. Determining which letters to change is up to you. 3 GET SWEEPING Let sleeping dogs lie. 20111204 Change one letter in each word of a made-up, two-word phrase to get two new words that will start a familiar proverb or saying. Determining which letters to change is up to you. 4 EASTER SAND Easier said than done. 20111204 Change one letter in each word of a made-up, two-word phrase to get two new words that will start a familiar proverb or saying. Determining which letters to change is up to you. 5 GOLD THONGS Good things come to those who wait. 20111204 Change one letter in each word of a made-up, two-word phrase to get two new words that will start a familiar proverb or saying. Determining which letters to change is up to you. 6 TAO HEAPS Two heads are better than one. 20120219 "Every answer consists of two adjoining U.S. states. Each clue is a four-letter word formed by one or more letters starting one of the state names plus one or more letters starting the other state name. For example, given """"mist,"""" the answer would be """"Mississippi"""" and """"Tennessee,"""" or """"Missouri"""" and """"Tennessee.""""" 1 COOK COlorado, OKlahoma 20120219 "Every answer consists of two adjoining U.S. states. Each clue is a four-letter word formed by one or more letters starting one of the state names plus one or more letters starting the other state name. For example, given """"mist,"""" the answer would be """"Mississippi"""" and """"Tennessee,"""" or """"Missouri"""" and """"Tennessee.""""" 2 FLOG FLOrida, Georgia 20120219 "Every answer consists of two adjoining U.S. states. Each clue is a four-letter word formed by one or more letters starting one of the state names plus one or more letters starting the other state name. For example, given """"mist,"""" the answer would be """"Mississippi"""" and """"Tennessee,"""" or """"Missouri"""" and """"Tennessee.""""" 3 CANE California, Nevada 20120219 "Every answer consists of two adjoining U.S. states. Each clue is a four-letter word formed by one or more letters starting one of the state names plus one or more letters starting the other state name. For example, given """"mist,"""" the answer would be """"Mississippi"""" and """"Tennessee,"""" or """"Missouri"""" and """"Tennessee.""""" 4 MADE Maryland, Delaware 20120219 "Every answer consists of two adjoining U.S. states. Each clue is a four-letter word formed by one or more letters starting one of the state names plus one or more letters starting the other state name. For example, given """"mist,"""" the answer would be """"Mississippi"""" and """"Tennessee,"""" or """"Missouri"""" and """"Tennessee.""""" 5 NEWT New (M)exico, Texas 20120219 "Every answer consists of two adjoining U.S. states. Each clue is a four-letter word formed by one or more letters starting one of the state names plus one or more letters starting the other state name. For example, given """"mist,"""" the answer would be """"Mississippi"""" and """"Tennessee,"""" or """"Missouri"""" and """"Tennessee.""""" 6 WILL Wisconsin, Illinois 20120219 "Every answer consists of two adjoining U.S. states. Each clue is a four-letter word formed by one or more letters starting one of the state names plus one or more letters starting the other state name. For example, given """"mist,"""" the answer would be """"Mississippi"""" and """"Tennessee,"""" or """"Missouri"""" and """"Tennessee.""""" 7 MACON Massachusetts, Connecticut 20120219 "Every answer consists of two adjoining U.S. states. Each clue is a four-letter word formed by one or more letters starting one of the state names plus one or more letters starting the other state name. For example, given """"mist,"""" the answer would be """"Mississippi"""" and """"Tennessee,"""" or """"Missouri"""" and """"Tennessee.""""" 8 MINOR Minnesota, North (D)akota 20120219 "Every answer consists of two adjoining U.S. states. Each clue is a four-letter word formed by one or more letters starting one of the state names plus one or more letters starting the other state name. For example, given """"mist,"""" the answer would be """"Mississippi"""" and """"Tennessee,"""" or """"Missouri"""" and """"Tennessee.""""" 9 PENNE Pennsylvania, New (Y)ork (or NEw (J)ersey) 20100703 "You are given categories and must name items within those categories that begin with the letters G, A, M, U and T. For example, given the category """"girls' names,"""" possible answers would be Gloria, Amelia, Martha, Ursula and Theresa." 1 Presidential first names George, Andrew, Martin, Ulysses, Thomas 20100703 "You are given categories and must name items within those categories that begin with the letters G, A, M, U and T. For example, given the category """"girls' names,"""" possible answers would be Gloria, Amelia, Martha, Ursula and Theresa." 2 Countries in Africa Gabon, Algeria, Mali, Uganda, Tanzania 20100703 "You are given categories and must name items within those categories that begin with the letters G, A, M, U and T. For example, given the category """"girls' names,"""" possible answers would be Gloria, Amelia, Martha, Ursula and Theresa." 3 Blood relatives grandfather, aunt, mother, uncle, twin 20100703 "You are given categories and must name items within those categories that begin with the letters G, A, M, U and T. For example, given the category """"girls' names,"""" possible answers would be Gloria, Amelia, Martha, Ursula and Theresa." 4 Musical instruments guitar, accordion, mandolin, ukulele, tiple 20100703 "You are given categories and must name items within those categories that begin with the letters G, A, M, U and T. For example, given the category """"girls' names,"""" possible answers would be Gloria, Amelia, Martha, Ursula and Theresa." 5 Things to pack in a suitcase gloves, ascot, mirror, underwear, T-shirt 20120205 Each clue contains at least one seven-letter word. Rearrange the letters in that word to answer the clue. 1 magazine that answers men's fashion-related QUERIES Esquire 20120205 Each clue contains at least one seven-letter word. Rearrange the letters in that word to answer the clue. 2 like some deductions CLAIMED on tax returns medical 20120205 Each clue contains at least one seven-letter word. Rearrange the letters in that word to answer the clue. 3 tree with a FIR-CONE (extreme example of puzzler's license) conifer 20120205 Each clue contains at least one seven-letter word. Rearrange the letters in that word to answer the clue. 4 where bidders should exhibit CAUTION auction 20120205 Each clue contains at least one seven-letter word. Rearrange the letters in that word to answer the clue. 5 It's LARGELY filled with paintings. gallery 20120205 Each clue contains at least one seven-letter word. Rearrange the letters in that word to answer the clue. 6 They give the PRECISE way to prepare dishes. recipes 20170319 Take two four-letter words. Rearrange the letters in each of them to make two new words that are opposites. 1 NAME CINE mean & nice 20170319 Take two four-letter words. Rearrange the letters in each of them to make two new words that are opposites. 2 VOLE HEAT love & hate 20170319 Take two four-letter words. Rearrange the letters in each of them to make two new words that are opposites. 3 FLIT PROD lift & drop 20170319 Take two four-letter words. Rearrange the letters in each of them to make two new words that are opposites. 4 OWLS FATS slow & fast 20170319 Take two four-letter words. Rearrange the letters in each of them to make two new words that are opposites. 5 VEIL DADE live & dead 20170319 Take two four-letter words. Rearrange the letters in each of them to make two new words that are opposites. 6 PEEK SOTS keep & toss 20170319 Take two four-letter words. Rearrange the letters in each of them to make two new words that are opposites. 7 AGIN SOLE gain & lose 20170319 Take two four-letter words. Rearrange the letters in each of them to make two new words that are opposites. 8 REAM DUST mare & stud 20170319 Take two four-letter words. Rearrange the letters in each of them to make two new words that are opposites. 9 SEAN STUN sane & nuts 20170319 Take two four-letter words. Rearrange the letters in each of them to make two new words that are opposites. 10 ^SEAT STEW east & west 20170319 Take two four-letter words. Rearrange the letters in each of them to make two new words that are opposites. 11 ^HEIR RIFE hire & fire 20170319 Take two four-letter words. Rearrange the letters in each of them to make two new words that are opposites. 12 ^WHOS HIED show & hide 20170319 Take two four-letter words. Rearrange the letters in each of them to make two new words that are opposites. 13 ^BUYS LIED busy & idle 20170319 Take two four-letter words. Rearrange the letters in each of them to make two new words that are opposites. 14 ACME NEWT came & went 20150301 "Rearrange the letters in a four-letter word and a five-letter word to get a pair of synonyms. For example, given """"time"""" and """"night,"""" you would say """"item"""" and """"thing.""""" 1 mope; sever poem; verse 20150301 "Rearrange the letters in a four-letter word and a five-letter word to get a pair of synonyms. For example, given """"time"""" and """"night,"""" you would say """"item"""" and """"thing.""""" 2 tuba; couth abut, touch 20150301 "Rearrange the letters in a four-letter word and a five-letter word to get a pair of synonyms. For example, given """"time"""" and """"night,"""" you would say """"item"""" and """"thing.""""" 3 diva; agree avid; eager 20150301 "Rearrange the letters in a four-letter word and a five-letter word to get a pair of synonyms. For example, given """"time"""" and """"night,"""" you would say """"item"""" and """"thing.""""" 5 skat; ochre task; chore 20150301 "Rearrange the letters in a four-letter word and a five-letter word to get a pair of synonyms. For example, given """"time"""" and """"night,"""" you would say """"item"""" and """"thing.""""" 6 fare; adder fear; dread 20150301 "Rearrange the letters in a four-letter word and a five-letter word to get a pair of synonyms. For example, given """"time"""" and """"night,"""" you would say """"item"""" and """"thing.""""" 7 ruby; inert bury; inter 20150301 "Rearrange the letters in a four-letter word and a five-letter word to get a pair of synonyms. For example, given """"time"""" and """"night,"""" you would say """"item"""" and """"thing.""""" 8 rave; taste aver; state 20150301 "Rearrange the letters in a four-letter word and a five-letter word to get a pair of synonyms. For example, given """"time"""" and """"night,"""" you would say """"item"""" and """"thing.""""" 9 gear; range rage; anger 20150301 "Rearrange the letters in a four-letter word and a five-letter word to get a pair of synonyms. For example, given """"time"""" and """"night,"""" you would say """"item"""" and """"thing.""""" 10 cork; onset rock; stone 20150301 "Rearrange the letters in a four-letter word and a five-letter word to get a pair of synonyms. For example, given """"time"""" and """"night,"""" you would say """"item"""" and """"thing.""""" 11 name; tansy mean; nasty 20181223 I'm going to give you two words. Think of a third word that can follow each of mine to complete compound words or familiar two-word phrases. And as help, the initial letters of my two words will be the first two letters of your answer. 1 Easter Goose egg 20181223 I'm going to give you two words. Think of a third word that can follow each of mine to complete compound words or familiar two-word phrases. And as help, the initial letters of my two words will be the first two letters of your answer. 2 Service Television station 20181223 I'm going to give you two words. Think of a third word that can follow each of mine to complete compound words or familiar two-word phrases. And as help, the initial letters of my two words will be the first two letters of your answer. 3 Oval Front office 20181223 I'm going to give you two words. Think of a third word that can follow each of mine to complete compound words or familiar two-word phrases. And as help, the initial letters of my two words will be the first two letters of your answer. 4 Fancy Risk free 20181223 I'm going to give you two words. Think of a third word that can follow each of mine to complete compound words or familiar two-word phrases. And as help, the initial letters of my two words will be the first two letters of your answer. 5 Halfway Open house 20130922 "You will be given two words. Think of a third word that can follow each to complete a familiar two-word phrase. The third word will rhyme with one of the given words. For example, given """"blame"""" and """"board,"""" you would say """"game,"""" as in """"blame game"""" and """"board game.""""" 1 fan, snail mail 20130922 "You will be given two words. Think of a third word that can follow each to complete a familiar two-word phrase. The third word will rhyme with one of the given words. For example, given """"blame"""" and """"board,"""" you would say """"game,"""" as in """"blame game"""" and """"board game.""""" 2 chill, sleeping pill 20130922 "You will be given two words. Think of a third word that can follow each to complete a familiar two-word phrase. The third word will rhyme with one of the given words. For example, given """"blame"""" and """"board,"""" you would say """"game,"""" as in """"blame game"""" and """"board game.""""" 3 poster, wild child 20130922 "You will be given two words. Think of a third word that can follow each to complete a familiar two-word phrase. The third word will rhyme with one of the given words. For example, given """"blame"""" and """"board,"""" you would say """"game,"""" as in """"blame game"""" and """"board game.""""" 4 fair, market share 20130922 "You will be given two words. Think of a third word that can follow each to complete a familiar two-word phrase. The third word will rhyme with one of the given words. For example, given """"blame"""" and """"board,"""" you would say """"game,"""" as in """"blame game"""" and """"board game.""""" 5 grease, stun gun 20130922 "You will be given two words. Think of a third word that can follow each to complete a familiar two-word phrase. The third word will rhyme with one of the given words. For example, given """"blame"""" and """"board,"""" you would say """"game,"""" as in """"blame game"""" and """"board game.""""" 6 near, root beer 20130922 "You will be given two words. Think of a third word that can follow each to complete a familiar two-word phrase. The third word will rhyme with one of the given words. For example, given """"blame"""" and """"board,"""" you would say """"game,"""" as in """"blame game"""" and """"board game.""""" 7 flower, solar power 20130922 "You will be given two words. Think of a third word that can follow each to complete a familiar two-word phrase. The third word will rhyme with one of the given words. For example, given """"blame"""" and """"board,"""" you would say """"game,"""" as in """"blame game"""" and """"board game.""""" 8 alley, fat cat 20130922 "You will be given two words. Think of a third word that can follow each to complete a familiar two-word phrase. The third word will rhyme with one of the given words. For example, given """"blame"""" and """"board,"""" you would say """"game,"""" as in """"blame game"""" and """"board game.""""" 9 brain, storm drain 20120805 "You are given two five-letter words. Put the same pair of letters in front of each of them to complete two familiar seven-letter words. The letters that go in front will never be a standard prefix, like """"re-."""" For example, given """"quire"""" and """"tress,"""" the answer would be """"ac"""" to make """"acquire"""" and """"actress.""""" 1 eight, antic FReight, FRantic 20120805 "You are given two five-letter words. Put the same pair of letters in front of each of them to complete two familiar seven-letter words. The letters that go in front will never be a standard prefix, like """"re-."""" For example, given """"quire"""" and """"tress,"""" the answer would be """"ac"""" to make """"acquire"""" and """"actress.""""" 2 ounce, avail TRounce, TRavail 20120805 "You are given two five-letter words. Put the same pair of letters in front of each of them to complete two familiar seven-letter words. The letters that go in front will never be a standard prefix, like """"re-."""" For example, given """"quire"""" and """"tress,"""" the answer would be """"ac"""" to make """"acquire"""" and """"actress.""""" 3 hoots, price CAhoots, CAprice 20120805 "You are given two five-letter words. Put the same pair of letters in front of each of them to complete two familiar seven-letter words. The letters that go in front will never be a standard prefix, like """"re-."""" For example, given """"quire"""" and """"tress,"""" the answer would be """"ac"""" to make """"acquire"""" and """"actress.""""" 4 Latin, state GElatin, GEstate 20120805 "You are given two five-letter words. Put the same pair of letters in front of each of them to complete two familiar seven-letter words. The letters that go in front will never be a standard prefix, like """"re-."""" For example, given """"quire"""" and """"tress,"""" the answer would be """"ac"""" to make """"acquire"""" and """"actress.""""" 5 heist, tempt ATheist, ATtempt 20120805 "You are given two five-letter words. Put the same pair of letters in front of each of them to complete two familiar seven-letter words. The letters that go in front will never be a standard prefix, like """"re-."""" For example, given """"quire"""" and """"tress,"""" the answer would be """"ac"""" to make """"acquire"""" and """"actress.""""" 6 event, orate PRevent, PRorate 20120805 "You are given two five-letter words. Put the same pair of letters in front of each of them to complete two familiar seven-letter words. The letters that go in front will never be a standard prefix, like """"re-."""" For example, given """"quire"""" and """"tress,"""" the answer would be """"ac"""" to make """"acquire"""" and """"actress.""""" 7 stain, alone ABstain, ABalone 20120805 "You are given two five-letter words. Put the same pair of letters in front of each of them to complete two familiar seven-letter words. The letters that go in front will never be a standard prefix, like """"re-."""" For example, given """"quire"""" and """"tress,"""" the answer would be """"ac"""" to make """"acquire"""" and """"actress.""""" 8 verge, sable DIverge, DIsable 20120805 "You are given two five-letter words. Put the same pair of letters in front of each of them to complete two familiar seven-letter words. The letters that go in front will never be a standard prefix, like """"re-."""" For example, given """"quire"""" and """"tress,"""" the answer would be """"ac"""" to make """"acquire"""" and """"actress.""""" 9 amble, other BRamble, BRother 20190721 I'm going to give you two 4-letter words. Rearrange the letters in each of them to make two new words that rhyme. 1 Cola, Lose Coal, Sole 20190721 I'm going to give you two 4-letter words. Rearrange the letters in each of them to make two new words that rhyme. 2 Near, Rent Earn, Term 20190721 I'm going to give you two 4-letter words. Rearrange the letters in each of them to make two new words that rhyme. 4 Dais, Deal Said, Lead 20190721 I'm going to give you two 4-letter words. Rearrange the letters in each of them to make two new words that rhyme. 5 Sing, Deny Sign, Dyne 20190721 I'm going to give you two 4-letter words. Rearrange the letters in each of them to make two new words that rhyme. 6 Each, Beak Ache, Bake 20190721 I'm going to give you two 4-letter words. Rearrange the letters in each of them to make two new words that rhyme. 7 Opus, Polo Soup, Loop 20190721 I'm going to give you two 4-letter words. Rearrange the letters in each of them to make two new words that rhyme. 8 Pale, Keen Plea, Knee 20190721 I'm going to give you two 4-letter words. Rearrange the letters in each of them to make two new words that rhyme. 9 Rely, Rite Lyre, Tire 20190721 I'm going to give you two 4-letter words. Rearrange the letters in each of them to make two new words that rhyme. 10 Once, News Cone, Sewn 20190721 I'm going to give you two 4-letter words. Rearrange the letters in each of them to make two new words that rhyme. 11 Oars, Ogre Soar, Gore 20190721 I'm going to give you two 4-letter words. Rearrange the letters in each of them to make two new words that rhyme. 12 Went, Roto Newt, Root 20190721 I'm going to give you two 4-letter words. Rearrange the letters in each of them to make two new words that rhyme. 13 Thaw, Thus What, Shut 20190721 I'm going to give you two 4-letter words. Rearrange the letters in each of them to make two new words that rhyme. 14 Isle, Zeal Leis, Laze 20091004 "Each clue consists of two words starting with the letter """"C."""" The answer is a third word starting with """"C"""" that can follow the first word and precede the second one to complete a compound word or familiar two-word phrase. For example, if the clue is """"comedy"""" and """"car,"""" the answer would be """"club"""" as in """"comedy club"""" and """"Club Car.""""" 1 cotton//cane candy 20091004 "Each clue consists of two words starting with the letter """"C."""" The answer is a third word starting with """"C"""" that can follow the first word and precede the second one to complete a compound word or familiar two-word phrase. For example, if the clue is """"comedy"""" and """"car,"""" the answer would be """"club"""" as in """"comedy club"""" and """"Club Car.""""" 2 chemistry//clown class 20091004 "Each clue consists of two words starting with the letter """"C."""" The answer is a third word starting with """"C"""" that can follow the first word and precede the second one to complete a compound word or familiar two-word phrase. For example, if the clue is """"comedy"""" and """"car,"""" the answer would be """"club"""" as in """"comedy club"""" and """"Club Car.""""" 3 cumulus//cover cloud 20091004 "Each clue consists of two words starting with the letter """"C."""" The answer is a third word starting with """"C"""" that can follow the first word and precede the second one to complete a compound word or familiar two-word phrase. For example, if the clue is """"comedy"""" and """"car,"""" the answer would be """"club"""" as in """"comedy club"""" and """"Club Car.""""" 4 code//crumbs cracker 20091004 "Each clue consists of two words starting with the letter """"C."""" The answer is a third word starting with """"C"""" that can follow the first word and precede the second one to complete a compound word or familiar two-word phrase. For example, if the clue is """"comedy"""" and """"car,"""" the answer would be """"club"""" as in """"comedy club"""" and """"Club Car.""""" 5 coaxial//car cable 20091004 "Each clue consists of two words starting with the letter """"C."""" The answer is a third word starting with """"C"""" that can follow the first word and precede the second one to complete a compound word or familiar two-word phrase. For example, if the clue is """"comedy"""" and """"car,"""" the answer would be """"club"""" as in """"comedy club"""" and """"Club Car.""""" 6 cold//cow cash 20091004 "Each clue consists of two words starting with the letter """"C."""" The answer is a third word starting with """"C"""" that can follow the first word and precede the second one to complete a compound word or familiar two-word phrase. For example, if the clue is """"comedy"""" and """"car,"""" the answer would be """"club"""" as in """"comedy club"""" and """"Club Car.""""" 7 cross//club country 20091004 "Each clue consists of two words starting with the letter """"C."""" The answer is a third word starting with """"C"""" that can follow the first word and precede the second one to complete a compound word or familiar two-word phrase. For example, if the clue is """"comedy"""" and """"car,"""" the answer would be """"club"""" as in """"comedy club"""" and """"Club Car.""""" 8 Caribbean//control cruise 20091004 "Each clue consists of two words starting with the letter """"C."""" The answer is a third word starting with """"C"""" that can follow the first word and precede the second one to complete a compound word or familiar two-word phrase. For example, if the clue is """"comedy"""" and """"car,"""" the answer would be """"club"""" as in """"comedy club"""" and """"Club Car.""""" 9 credit//catalog card 20091004 "Each clue consists of two words starting with the letter """"C."""" The answer is a third word starting with """"C"""" that can follow the first word and precede the second one to complete a compound word or familiar two-word phrase. For example, if the clue is """"comedy"""" and """"car,"""" the answer would be """"club"""" as in """"comedy club"""" and """"Club Car.""""" 10 creamed//cob corn 20091004 "Each clue consists of two words starting with the letter """"C."""" The answer is a third word starting with """"C"""" that can follow the first word and precede the second one to complete a compound word or familiar two-word phrase. For example, if the clue is """"comedy"""" and """"car,"""" the answer would be """"club"""" as in """"comedy club"""" and """"Club Car.""""" 11 cabin//cut crew 20091004 "Each clue consists of two words starting with the letter """"C."""" The answer is a third word starting with """"C"""" that can follow the first word and precede the second one to complete a compound word or familiar two-word phrase. For example, if the clue is """"comedy"""" and """"car,"""" the answer would be """"club"""" as in """"comedy club"""" and """"Club Car.""""" 12 clothes//case closet 20091004 "Each clue consists of two words starting with the letter """"C."""" The answer is a third word starting with """"C"""" that can follow the first word and precede the second one to complete a compound word or familiar two-word phrase. For example, if the clue is """"comedy"""" and """"car,"""" the answer would be """"club"""" as in """"comedy club"""" and """"Club Car.""""" 13 coffee//cake cup 20091004 "Each clue consists of two words starting with the letter """"C."""" The answer is a third word starting with """"C"""" that can follow the first word and precede the second one to complete a compound word or familiar two-word phrase. For example, if the clue is """"comedy"""" and """"car,"""" the answer would be """"club"""" as in """"comedy club"""" and """"Club Car.""""" 14 crystal//conscience clear 20091004 "Each clue consists of two words starting with the letter """"C."""" The answer is a third word starting with """"C"""" that can follow the first word and precede the second one to complete a compound word or familiar two-word phrase. For example, if the clue is """"comedy"""" and """"car,"""" the answer would be """"club"""" as in """"comedy club"""" and """"Club Car.""""" 15 crash//correction course 20140202 For each single letter given 1 one + X oxen 20140202 For each single letter given 2 two + N won't [town] 20140202 For each single letter given 3 three + I either 20140202 For each single letter given 4 four + M forum 20140202 For each single letter given 5 five + R + Y verify 20140202 For each single letter given 6 six + A axis 20140202 For each single letter given 7 seven + U venues 20140202 For each single letter given 8 eight + O hogtie 20140202 For each single letter given 9 nine + L linen 20140202 For each single letter given 10 ten + X next 20140202 For each single letter given 11 eleven + O + P envelope 20160703 With the Fourth of July and cookouts on my mind, I brought a game of Categories — based on the word PATIO. I'm going to give you some categories. For each one, name something in the category starting with each of the letters P, A, T, I and O. 3 things seen in a church pew, altar, transept (tabernacle), incense, organ (offering) 20200628 I'm going to read you some sentences. Three consecutive words somewhere in each sentence are the first three words of a familiar proverb or saying. Tell me what it is. 1 The queen attends every royal function, so her absence makes the crowd concerned. Absence makes the heart grow fonder. 20200628 I'm going to read you some sentences. Three consecutive words somewhere in each sentence are the first three words of a familiar proverb or saying. Tell me what it is. 2 The cows with two heads are the big attraction at the carnival. Two heads are better than one. 20200628 I'm going to read you some sentences. Three consecutive words somewhere in each sentence are the first three words of a familiar proverb or saying. Tell me what it is. 3 A nice Scottish lad is what a miss is looking for. A miss is as good as a mile. 20200628 I'm going to read you some sentences. Three consecutive words somewhere in each sentence are the first three words of a familiar proverb or saying. Tell me what it is. 4 I'm reading a Rolling Stone article about a new country artist. A rolling stone gathers no moss. 20200628 I'm going to read you some sentences. Three consecutive words somewhere in each sentence are the first three words of a familiar proverb or saying. Tell me what it is. 5 At the amusement park, children should be at least eight to ride the rollercoaster. Children should be seen and not heard. 20200628 I'm going to read you some sentences. Three consecutive words somewhere in each sentence are the first three words of a familiar proverb or saying. Tell me what it is. 6 At swim camp, Tammy learned the butterfly, while the counselors taught different strokes for the other swimmers. Different strokes for different folks. 20200628 I'm going to read you some sentences. Three consecutive words somewhere in each sentence are the first three words of a familiar proverb or saying. Tell me what it is. 7 At the Westminster show, every dog has a leash. Every dog has its day. 20200628 I'm going to read you some sentences. Three consecutive words somewhere in each sentence are the first three words of a familiar proverb or saying. Tell me what it is. 8 While marijuana prices vary from state to state, studies show the grass is cheapest in Oregon. The grass is always greener on the other side of the fence. 20200628 I'm going to read you some sentences. Three consecutive words somewhere in each sentence are the first three words of a familiar proverb or saying. Tell me what it is. 9 A foster home is where a child gets a new life. Home is where the heart is. 20200628 I'm going to read you some sentences. Three consecutive words somewhere in each sentence are the first three words of a familiar proverb or saying. Tell me what it is. 10 Of all the qualities you want in a business partner, honesty is the most important. Honesty is the best policy. 20201018 The theme of this puzzle is P, B & J. I'm going to give you three words starting with the letters P, B and J. You give me a fourth word that can follow each of mine to complete a compound word or a familiar two-word phrase. 1 Penalty, Boom, Juke box 20201018 The theme of this puzzle is P, B & J. I'm going to give you three words starting with the letters P, B and J. You give me a fourth word that can follow each of mine to complete a compound word or a familiar two-word phrase. 2 Power, Buzz, Jig saw 20201018 The theme of this puzzle is P, B & J. I'm going to give you three words starting with the letters P, B and J. You give me a fourth word that can follow each of mine to complete a compound word or a familiar two-word phrase. 3 Present, Birth, Judgment day 20201018 The theme of this puzzle is P, B & J. I'm going to give you three words starting with the letters P, B and J. You give me a fourth word that can follow each of mine to complete a compound word or a familiar two-word phrase. 4 Pill, Bed, Jitter bug 20201018 The theme of this puzzle is P, B & J. I'm going to give you three words starting with the letters P, B and J. You give me a fourth word that can follow each of mine to complete a compound word or a familiar two-word phrase. 5 Perp, Board, Jay walk 20201018 The theme of this puzzle is P, B & J. I'm going to give you three words starting with the letters P, B and J. You give me a fourth word that can follow each of mine to complete a compound word or a familiar two-word phrase. 6 Paddle, Basket, Jump ball 20201018 The theme of this puzzle is P, B & J. I'm going to give you three words starting with the letters P, B and J. You give me a fourth word that can follow each of mine to complete a compound word or a familiar two-word phrase. 7 Plymouth, Bed, Jailhouse rock 20201018 The theme of this puzzle is P, B & J. I'm going to give you three words starting with the letters P, B and J. You give me a fourth word that can follow each of mine to complete a compound word or a familiar two-word phrase. 8 Pogo, Broom, Joy stick 20201018 The theme of this puzzle is P, B & J. I'm going to give you three words starting with the letters P, B and J. You give me a fourth word that can follow each of mine to complete a compound word or a familiar two-word phrase. 9 Pocket, Butter, Jack knife 20201018 The theme of this puzzle is P, B & J. I'm going to give you three words starting with the letters P, B and J. You give me a fourth word that can follow each of mine to complete a compound word or a familiar two-word phrase. 10 Passion, Bearing, Juicy fruit 20201018 The theme of this puzzle is P, B & J. I'm going to give you three words starting with the letters P, B and J. You give me a fourth word that can follow each of mine to complete a compound word or a familiar two-word phrase. 11 Pickled, Bell, Jalapeno pepper 20100815 "Each sentence conceals the name of a tree in consecutive letters. For example, given, """"Use soap, please,"""" the answer would be """"apple.""""" 1 Is the lollipop large? poplar 20100815 "Each sentence conceals the name of a tree in consecutive letters. For example, given, """"Use soap, please,"""" the answer would be """"apple.""""" 2 Tomorrow, I'll own the company. willow 20100815 "Each sentence conceals the name of a tree in consecutive letters. For example, given, """"Use soap, please,"""" the answer would be """"apple.""""" 3 The agency pressed for more money. cypress 20100815 "Each sentence conceals the name of a tree in consecutive letters. For example, given, """"Use soap, please,"""" the answer would be """"apple.""""" 4 School officials must give them lockers. hemlock 20100815 "Each sentence conceals the name of a tree in consecutive letters. For example, given, """"Use soap, please,"""" the answer would be """"apple.""""" 5 Don't film a hog anymore. mahogany 20100815 "Each sentence conceals the name of a tree in consecutive letters. For example, given, """"Use soap, please,"""" the answer would be """"apple.""""" 6 Some pueblo customs are unusual. locust 20100815 "Each sentence conceals the name of a tree in consecutive letters. For example, given, """"Use soap, please,"""" the answer would be """"apple.""""" 7 The car raced around the track. cedar 20100815 "Each sentence conceals the name of a tree in consecutive letters. For example, given, """"Use soap, please,"""" the answer would be """"apple.""""" 8 Remove any greasy cam or engine part that's loose. sycamore 20100815 "Each sentence conceals the name of a tree in consecutive letters. For example, given, """"Use soap, please,"""" the answer would be """"apple.""""" 9 Can I access the web on your computer? ebony 20100815 "Each sentence conceals the name of a tree in consecutive letters. For example, given, """"Use soap, please,"""" the answer would be """"apple.""""" 10 Albany and Buffalo are in upstate New York. banyan 20100815 "Each sentence conceals the name of a tree in consecutive letters. For example, given, """"Use soap, please,"""" the answer would be """"apple.""""" 11 Who would Mike Huckabee choose as a running mate? beech 20100207 "Every answer today is a word used in football. Given three words, you give a word that can follow each to give a familiar two word phrase. The answer will always be a football term. For example, if the clue is """"year, tag and dead,"""" the answer would be """"end.""""" 1 STUMBLING, BUILDING, & WRITER'S block 20100207 "Every answer today is a word used in football. Given three words, you give a word that can follow each to give a familiar two word phrase. The answer will always be a football term. For example, if the clue is """"year, tag and dead,"""" the answer would be """"end.""""" 2 GOLD, BUM'S, & MAD rush 20100207 "Every answer today is a word used in football. Given three words, you give a word that can follow each to give a familiar two word phrase. The answer will always be a football term. For example, if the clue is """"year, tag and dead,"""" the answer would be """"end.""""" 3 CATTLE, HARD, & FOUR-WHEEL drive 20100207 "Every answer today is a word used in football. Given three words, you give a word that can follow each to give a familiar two word phrase. The answer will always be a football term. For example, if the clue is """"year, tag and dead,"""" the answer would be """"end.""""" 4 CORN, LEFT, & ELECTROMAGNETIC field 20100207 "Every answer today is a word used in football. Given three words, you give a word that can follow each to give a familiar two word phrase. The answer will always be a football term. For example, if the clue is """"year, tag and dead,"""" the answer would be """"end.""""" 5 FOUL, CHILD'S, & WORD play 20100207 "Every answer today is a word used in football. Given three words, you give a word that can follow each to give a familiar two word phrase. The answer will always be a football term. For example, if the clue is """"year, tag and dead,"""" the answer would be """"end.""""" 6 SHOPPING, DEAD, & DAYCARE center 20100207 "Every answer today is a word used in football. Given three words, you give a word that can follow each to give a familiar two word phrase. The answer will always be a football term. For example, if the clue is """"year, tag and dead,"""" the answer would be """"end.""""" 7 PRISON, NATIONAL, & REAR "guard (I admit giggling at Mr. Coyne's """"rear end"""" suggestion)" 20100207 "Every answer today is a word used in football. Given three words, you give a word that can follow each to give a familiar two word phrase. The answer will always be a football term. For example, if the clue is """"year, tag and dead,"""" the answer would be """"end.""""" 8 BOUNCE, STAND, & TALK back 20100207 "Every answer today is a word used in football. Given three words, you give a word that can follow each to give a familiar two word phrase. The answer will always be a football term. For example, if the clue is """"year, tag and dead,"""" the answer would be """"end.""""" 9 MOUNTAIN, BOARDING & BUS pass 20100207 "Every answer today is a word used in football. Given three words, you give a word that can follow each to give a familiar two word phrase. The answer will always be a football term. For example, if the clue is """"year, tag and dead,"""" the answer would be """"end.""""" 10 BROKEN, DUMB, & THUMBS down 20100207 "Every answer today is a word used in football. Given three words, you give a word that can follow each to give a familiar two word phrase. The answer will always be a football term. For example, if the clue is """"year, tag and dead,"""" the answer would be """"end.""""" 11 PUNCH, DUST, & CEREAL bowl 20100207 "Every answer today is a word used in football. Given three words, you give a word that can follow each to give a familiar two word phrase. The answer will always be a football term. For example, if the clue is """"year, tag and dead,"""" the answer would be """"end.""""" 12 NEWSPAPER, COUPON, & TOENAIL clipping 20130721 Every answer is the name of a famous person, past or present, with five letters in the first and last names. One letter in each name is changed to make a new word. You name the people. 1 julep verse Jules Verne 20130721 Every answer is the name of a famous person, past or present, with five letters in the first and last names. One letter in each name is changed to make a new word. You name the people. 2 Marne cutie Marie Curie 20130721 Every answer is the name of a famous person, past or present, with five letters in the first and last names. One letter in each name is changed to make a new word. You name the people. 3 fancy grade Nancy Grace 20130721 Every answer is the name of a famous person, past or present, with five letters in the first and last names. One letter in each name is changed to make a new word. You name the people. 4 delta geese Della Reese 20130721 Every answer is the name of a famous person, past or present, with five letters in the first and last names. One letter in each name is changed to make a new word. You name the people. 5 dinar store Dinah Shore 20130721 Every answer is the name of a famous person, past or present, with five letters in the first and last names. One letter in each name is changed to make a new word. You name the people. 6 milks Dalis Miles Davis 20130721 Every answer is the name of a famous person, past or present, with five letters in the first and last names. One letter in each name is changed to make a new word. You name the people. 7 grape slice Grace Slick 20130721 Every answer is the name of a famous person, past or present, with five letters in the first and last names. One letter in each name is changed to make a new word. You name the people. 8 sharp Levis Shari Lewis 20130721 Every answer is the name of a famous person, past or present, with five letters in the first and last names. One letter in each name is changed to make a new word. You name the people. 9 early limon Carly Simon 20130721 Every answer is the name of a famous person, past or present, with five letters in the first and last names. One letter in each name is changed to make a new word. You name the people. 10 jades blown James Brow 20110220 "Each word you are given conceals the name of a U.S. president in left-to-right order, but the letters are not consecutive. For example, given the clue """"hairdressing, in seven letters,"""" the answer would be """"Harding.""""" 1 bluefish Bush 20110220 "Each word you are given conceals the name of a U.S. president in left-to-right order, but the letters are not consecutive. For example, given the clue """"hairdressing, in seven letters,"""" the answer would be """"Harding.""""" 2 daydreams Adams 20110220 "Each word you are given conceals the name of a U.S. president in left-to-right order, but the letters are not consecutive. For example, given the clue """"hairdressing, in seven letters,"""" the answer would be """"Harding.""""" 3 monochrome Monroe 20110220 "Each word you are given conceals the name of a U.S. president in left-to-right order, but the letters are not consecutive. For example, given the clue """"hairdressing, in seven letters,"""" the answer would be """"Harding.""""" 5 transfixion Nixon 20110220 "Each word you are given conceals the name of a U.S. president in left-to-right order, but the letters are not consecutive. For example, given the clue """"hairdressing, in seven letters,"""" the answer would be """"Harding.""""" 6 registrant Grant 20110220 "Each word you are given conceals the name of a U.S. president in left-to-right order, but the letters are not consecutive. For example, given the clue """"hairdressing, in seven letters,"""" the answer would be """"Harding.""""" 7 maldistribution Madison 20110220 "Each word you are given conceals the name of a U.S. president in left-to-right order, but the letters are not consecutive. For example, given the clue """"hairdressing, in seven letters,"""" the answer would be """"Harding.""""" 8 howsoever Hoover 20110220 "Each word you are given conceals the name of a U.S. president in left-to-right order, but the letters are not consecutive. For example, given the clue """"hairdressing, in seven letters,"""" the answer would be """"Harding.""""" 9 potluck Polk 20110220 "Each word you are given conceals the name of a U.S. president in left-to-right order, but the letters are not consecutive. For example, given the clue """"hairdressing, in seven letters,"""" the answer would be """"Harding.""""" 10 culmination Clinton 20110220 "Each word you are given conceals the name of a U.S. president in left-to-right order, but the letters are not consecutive. For example, given the clue """"hairdressing, in seven letters,"""" the answer would be """"Harding.""""" 11 talkfest Taft 20110220 "Each word you are given conceals the name of a U.S. president in left-to-right order, but the letters are not consecutive. For example, given the clue """"hairdressing, in seven letters,"""" the answer would be """"Harding.""""" 12 papier-mache Pierce 20140316 "A series of paired words will be provided. For each pair, think of a third word that can follow the first one and precede the second to complete a familiar two-word phrase. Every answer starts with """"W."""" Example: Open and Awake; Answer: Wide." 1 humpback, oil whale 20140316 "A series of paired words will be provided. For each pair, think of a third word that can follow the first one and precede the second to complete a familiar two-word phrase. Every answer starts with """"W."""" Example: Open and Awake; Answer: Wide." 2 station, train wagon 20140316 "A series of paired words will be provided. For each pair, think of a third word that can follow the first one and precede the second to complete a familiar two-word phrase. Every answer starts with """"W."""" Example: Open and Awake; Answer: Wide." 3 wrist, tower watch 20140316 "A series of paired words will be provided. For each pair, think of a third word that can follow the first one and precede the second to complete a familiar two-word phrase. Every answer starts with """"W."""" Example: Open and Awake; Answer: Wide." 5 shotgun, bells wedding 20140316 "A series of paired words will be provided. For each pair, think of a third word that can follow the first one and precede the second to complete a familiar two-word phrase. Every answer starts with """"W."""" Example: Open and Awake; Answer: Wide." 6 timber, whistle wolf 20140316 "A series of paired words will be provided. For each pair, think of a third word that can follow the first one and precede the second to complete a familiar two-word phrase. Every answer starts with """"W."""" Example: Open and Awake; Answer: Wide." 7 atomic, lifter weight 20140316 "A series of paired words will be provided. For each pair, think of a third word that can follow the first one and precede the second to complete a familiar two-word phrase. Every answer starts with """"W."""" Example: Open and Awake; Answer: Wide." 8 third, Series world 20140316 "A series of paired words will be provided. For each pair, think of a third word that can follow the first one and precede the second to complete a familiar two-word phrase. Every answer starts with """"W."""" Example: Open and Awake; Answer: Wide." 9 dish, dryer washer 20140316 "A series of paired words will be provided. For each pair, think of a third word that can follow the first one and precede the second to complete a familiar two-word phrase. Every answer starts with """"W."""" Example: Open and Awake; Answer: Wide." 10 barbed, service wire 20140316 "A series of paired words will be provided. For each pair, think of a third word that can follow the first one and precede the second to complete a familiar two-word phrase. Every answer starts with """"W."""" Example: Open and Awake; Answer: Wide." 11 stormy, map weather 20140316 "A series of paired words will be provided. For each pair, think of a third word that can follow the first one and precede the second to complete a familiar two-word phrase. Every answer starts with """"W."""" Example: Open and Awake; Answer: Wide." 12 Old Man, break winter 20140316 "A series of paired words will be provided. For each pair, think of a third word that can follow the first one and precede the second to complete a familiar two-word phrase. Every answer starts with """"W."""" Example: Open and Awake; Answer: Wide." 13 last, processor word 20131222 Every answer is a familiar two-word phrase or name in which, like Santa Claus, the first word starts with the letters S-A, and the second word starts with C. 1 kids' elaborate construction on a beach sand castle 20131222 Every answer is a familiar two-word phrase or name in which, like Santa Claus, the first word starts with the letters S-A, and the second word starts with C. 2 a person who works at a store sales clerk 20131222 Every answer is a familiar two-word phrase or name in which, like Santa Claus, the first word starts with the letters S-A, and the second word starts with C. 3 something considered immune from questioning or criticism sacred cow 20131222 Every answer is a familiar two-word phrase or name in which, like Santa Claus, the first word starts with the letters S-A, and the second word starts with C. 4 what nabisco premium is a brand of saltine cracker 20131222 Every answer is a familiar two-word phrase or name in which, like Santa Claus, the first word starts with the letters S-A, and the second word starts with C. 5 metal container tightly packed with fish sardine can 20131222 Every answer is a familiar two-word phrase or name in which, like Santa Claus, the first word starts with the letters S-A, and the second word starts with C. 6 city, south of los angeles, that was the home of nixon's western white house san clemente 20131222 Every answer is a familiar two-word phrase or name in which, like Santa Claus, the first word starts with the letters S-A, and the second word starts with C. 7 fill in the blank: carson mccullers' book, the ballad of the ________ sad cafà 20131222 Every answer is a familiar two-word phrase or name in which, like Santa Claus, the first word starts with the letters S-A, and the second word starts with C. 8 mark twain's real name samuel clemens 20131222 Every answer is a familiar two-word phrase or name in which, like Santa Claus, the first word starts with the letters S-A, and the second word starts with C. 9 photographer's request, just before snapping a picture say cheese! 20200405 I'm going to give you clues for two words. The first word starts with AB. Drop the AB and you'll get a new word that answers the second clue. 1 missing, as from a class / mailed absent and sent 20200405 I'm going to give you clues for two words. The first word starts with AB. Drop the AB and you'll get a new word that answers the second clue. 2 leader of nuns / first lady between eleanor and mamie abbess and bess 20200405 I'm going to give you clues for two words. The first word starts with AB. Drop the AB and you'll get a new word that answers the second clue. 3 on fire / lie around idly ablaze and laze 20200405 I'm going to give you clues for two words. The first word starts with AB. Drop the AB and you'll get a new word that answers the second clue. 4 like flowers in the spring / weaving apparatus abroad and road 20200405 I'm going to give you clues for two words. The first word starts with AB. Drop the AB and you'll get a new word that answers the second clue. 5 in another country / highway avoid liquor / laundry problem 20200405 I'm going to give you clues for two words. The first word starts with AB. Drop the AB and you'll get a new word that answers the second clue. 6 to kidnap / passage for air shorten / chain of hills 20200405 I'm going to give you clues for two words. The first word starts with AB. Drop the AB and you'll get a new word that answers the second clue. 7 to avoid liquor / laundry problem a source of mother-of-pearl / by oneself 20200405 I'm going to give you clues for two words. The first word starts with AB. Drop the AB and you'll get a new word that answers the second clue. 8 to shorten* / chain of hills abridge and ridge 20200405 I'm going to give you clues for two words. The first word starts with AB. Drop the AB and you'll get a new word that answers the second clue. 9 a source of mother-of-pearl / by oneself abalone and alone 20110918 "Every answer is the name of a college or university. You must identify the schools from their anagrams. For example: """"ICER."""" The answer: """"RICE.""""" 1 moyer Emory 20110918 "Every answer is the name of a college or university. You must identify the schools from their anagrams. For example: """"ICER."""" The answer: """"RICE.""""" 2 hamster Amherst 20110918 "Every answer is the name of a college or university. You must identify the schools from their anagrams. For example: """"ICER."""" The answer: """"RICE.""""" 3 gelcoat Colgate 20110918 "Every answer is the name of a college or university. You must identify the schools from their anagrams. For example: """"ICER."""" The answer: """"RICE.""""" 4 to meander Notre Dame 20110918 "Every answer is the name of a college or university. You must identify the schools from their anagrams. For example: """"ICER."""" The answer: """"RICE.""""" 5 lead up de Paul 20110918 "Every answer is the name of a college or university. You must identify the schools from their anagrams. For example: """"ICER."""" The answer: """"RICE.""""" 6 dialect Citadel 20110918 "Every answer is the name of a college or university. You must identify the schools from their anagrams. For example: """"ICER."""" The answer: """"RICE.""""" 7 water scenes Case Western 20110918 "Every answer is the name of a college or university. You must identify the schools from their anagrams. For example: """"ICER."""" The answer: """"RICE.""""" 8 earthworms Swarthmore 20131117 "Every answer is a made-up, two-word phrase in which the vowel in the first word is a short """"e"""" and the vowel in the second word is a long """"o."""" For example: A place to meditate would be a """"zen zone.""""" 1 crimson-colored highway red road 20131117 "Every answer is a made-up, two-word phrase in which the vowel in the first word is a short """"e"""" and the vowel in the second word is a long """"o."""" For example: A place to meditate would be a """"zen zone.""""" 2 strangulation in Prague Czech choke 20131117 "Every answer is a made-up, two-word phrase in which the vowel in the first word is a short """"e"""" and the vowel in the second word is a long """"o."""" For example: A place to meditate would be a """"zen zone.""""" 3 acquires farm animals with beards gets goats 20131117 "Every answer is a made-up, two-word phrase in which the vowel in the first word is a short """"e"""" and the vowel in the second word is a long """"o."""" For example: A place to meditate would be a """"zen zone.""""" 4 phantom who's just visiting guest ghost 20131117 "Every answer is a made-up, two-word phrase in which the vowel in the first word is a short """"e"""" and the vowel in the second word is a long """"o."""" For example: A place to meditate would be a """"zen zone.""""" 5 superior yachtsman better boater 20131117 "Every answer is a made-up, two-word phrase in which the vowel in the first word is a short """"e"""" and the vowel in the second word is a long """"o."""" For example: A place to meditate would be a """"zen zone.""""" 6 someone who hates clothes made from animal hides leather loather 20131117 "Every answer is a made-up, two-word phrase in which the vowel in the first word is a short """"e"""" and the vowel in the second word is a long """"o."""" For example: A place to meditate would be a """"zen zone.""""" 7 small horse that costs one cent penny pony 20131117 "Every answer is a made-up, two-word phrase in which the vowel in the first word is a short """"e"""" and the vowel in the second word is a long """"o."""" For example: A place to meditate would be a """"zen zone.""""" 8 ingenious plant with a sweet blossom clever clover 20131117 "Every answer is a made-up, two-word phrase in which the vowel in the first word is a short """"e"""" and the vowel in the second word is a long """"o."""" For example: A place to meditate would be a """"zen zone.""""" 9 Former Senator Kennedy's yes men Teddy's toadies 20131117 "Every answer is a made-up, two-word phrase in which the vowel in the first word is a short """"e"""" and the vowel in the second word is a long """"o."""" For example: A place to meditate would be a """"zen zone.""""" 10 entraps team trainers catches coaches 20200301 I'm going to give you clues for two words. The first word ends in GO. Drop the GO, and what's left will answer the second clue. 1 Tropical fruit / Guy or fellow mango, man 20200301 I'm going to give you clues for two words. The first word ends in GO. Drop the GO, and what's left will answer the second clue. 2 Australian sheep menace / Cacophony dingo, din 20200301 I'm going to give you clues for two words. The first word ends in GO. Drop the GO, and what's left will answer the second clue. 3 Ballroom dance / Light brown Tango, tan 20200301 I'm going to give you clues for two words. The first word ends in GO. Drop the GO, and what's left will answer the second clue. 4 Second-longest river in Africa / Opposed to Congo, con 20200301 I'm going to give you clues for two words. The first word ends in GO. Drop the GO, and what's left will answer the second clue. 5 North Dakota city / At a distance Fargo, far 20200301 I'm going to give you clues for two words. The first word ends in GO. Drop the GO, and what's left will answer the second clue. 6 Game with a 25-square card / Large container Bingo, bin 20200301 I'm going to give you clues for two words. The first word ends in GO. Drop the GO, and what's left will answer the second clue. 7 Foreigner in Mexico / Smile gringo, grin 20200301 I'm going to give you clues for two words. The first word ends in GO. Drop the GO, and what's left will answer the second clue. 8 Italian cheese / Where China is asiago, Asia 20200301 I'm going to give you clues for two words. The first word ends in GO. Drop the GO, and what's left will answer the second clue. 9 Third-largest city in America / Latin-American girl Chicago, chica 20200301 I'm going to give you clues for two words. The first word ends in GO. Drop the GO, and what's left will answer the second clue. 10 Name of a bay in Jamaica / Card game associated with cheats Montego, monte 20200301 I'm going to give you clues for two words. The first word ends in GO. Drop the GO, and what's left will answer the second clue. 11 California city / Cookie often made with pecans San Diego, sandie 20140727 Today's puzzle is a game of categories based on the word peony. For each category, name something in the category beginning with each of the letters P-E-O-N-Y. 1 color pink, ebony, orange, nectarine, yellow 20140727 Today's puzzle is a game of categories based on the word peony. For each category, name something in the category beginning with each of the letters P-E-O-N-Y. 2 street and highway signs pass with care, exit, one-way, no parking, yield 20140727 Today's puzzle is a game of categories based on the word peony. For each category, name something in the category beginning with each of the letters P-E-O-N-Y. 3 place names in Canada Prince Edward Island, Edmonton, Ontario, Newfoundland, Yukon Territory 20140727 Today's puzzle is a game of categories based on the word peony. For each category, name something in the category beginning with each of the letters P-E-O-N-Y. 4 "words of five or fewer letters ending in """"k""""" pink, elk, oak, neck, yak/yolk/yuck/yank 20090628 "Every answer is a familiar three-word phrase. The clues are two-word phrases. The first word of each phrase rhymes with the first word of the answer. The last word of the phrase rhymes with the last word of the answer. For example, if the clue is, """"Split this,"""" the answer would be, """"Hit or miss.""""" 1 TORE DRESS MORE OR LESS 20090628 "Every answer is a familiar three-word phrase. The clues are two-word phrases. The first word of each phrase rhymes with the first word of the answer. The last word of the phrase rhymes with the last word of the answer. For example, if the clue is, """"Split this,"""" the answer would be, """"Hit or miss.""""" 2 MAIN LINE RAIN OR SHINE 20090628 "Every answer is a familiar three-word phrase. The clues are two-word phrases. The first word of each phrase rhymes with the first word of the answer. The last word of the phrase rhymes with the last word of the answer. For example, if the clue is, """"Split this,"""" the answer would be, """"Hit or miss.""""" 3 BRICK STREET TRICK OR TREAT 20090628 "Every answer is a familiar three-word phrase. The clues are two-word phrases. The first word of each phrase rhymes with the first word of the answer. The last word of the phrase rhymes with the last word of the answer. For example, if the clue is, """"Split this,"""" the answer would be, """"Hit or miss.""""" 4 PRIME SEASON RHYME OR REASON 20090628 "Every answer is a familiar three-word phrase. The clues are two-word phrases. The first word of each phrase rhymes with the first word of the answer. The last word of the phrase rhymes with the last word of the answer. For example, if the clue is, """"Split this,"""" the answer would be, """"Hit or miss.""""" 5 LEATHER COT WHETHER OR NOT 20090628 "Every answer is a familiar three-word phrase. The clues are two-word phrases. The first word of each phrase rhymes with the first word of the answer. The last word of the phrase rhymes with the last word of the answer. For example, if the clue is, """"Split this,"""" the answer would be, """"Hit or miss.""""" 6 FIGHT SONG RIGHT OR WRONG 20090628 "Every answer is a familiar three-word phrase. The clues are two-word phrases. The first word of each phrase rhymes with the first word of the answer. The last word of the phrase rhymes with the last word of the answer. For example, if the clue is, """"Split this,"""" the answer would be, """"Hit or miss.""""" 7 SMASH BARGE YES OR NO 20090628 "Every answer is a familiar three-word phrase. The clues are two-word phrases. The first word of each phrase rhymes with the first word of the answer. The last word of the phrase rhymes with the last word of the answer. For example, if the clue is, """"Split this,"""" the answer would be, """"Hit or miss.""""" 8 GUESS SO SINK OR SWIM 20090628 "Every answer is a familiar three-word phrase. The clues are two-word phrases. The first word of each phrase rhymes with the first word of the answer. The last word of the phrase rhymes with the last word of the answer. For example, if the clue is, """"Split this,"""" the answer would be, """"Hit or miss.""""" 9 PINK GYM PLUS OR MINUS 20090628 "Every answer is a familiar three-word phrase. The clues are two-word phrases. The first word of each phrase rhymes with the first word of the answer. The last word of the phrase rhymes with the last word of the answer. For example, if the clue is, """"Split this,"""" the answer would be, """"Hit or miss.""""" 10 FREDTMS SCALES HEADS OR TAILS 20090628 "Every answer is a familiar three-word phrase. The clues are two-word phrases. The first word of each phrase rhymes with the first word of the answer. The last word of the phrase rhymes with the last word of the answer. For example, if the clue is, """"Split this,"""" the answer would be, """"Hit or miss.""""" 11 LEAST SALMON TRUTH OR DARE 20090628 "Every answer is a familiar three-word phrase. The clues are two-word phrases. The first word of each phrase rhymes with the first word of the answer. The last word of the phrase rhymes with the last word of the answer. For example, if the clue is, """"Split this,"""" the answer would be, """"Hit or miss.""""" 12 BUS LINUS PLUS OR MINUS 20090628 "Every answer is a familiar three-word phrase. The clues are two-word phrases. The first word of each phrase rhymes with the first word of the answer. The last word of the phrase rhymes with the last word of the answer. For example, if the clue is, """"Split this,"""" the answer would be, """"Hit or miss.""""" 13 TOOTH SCARE CASH OR CHARGE 20090628 "Every answer is a familiar three-word phrase. The clues are two-word phrases. The first word of each phrase rhymes with the first word of the answer. The last word of the phrase rhymes with the last word of the answer. For example, if the clue is, """"Split this,"""" the answer would be, """"Hit or miss.""""" 14 SOXERS CHIEFS BOXERS OR BRIEFS 20090628 "Every answer is a familiar three-word phrase. The clues are two-word phrases. The first word of each phrase rhymes with the first word of the answer. The last word of the phrase rhymes with the last word of the answer. For example, if the clue is, """"Split this,"""" the answer would be, """"Hit or miss.""""" 15 LUNAR CRATER SOONER OR LATER 20100904 "You are given a five-letter word and a seven-letter word. Rearrange the letters of one of the words to get a synonym of the other word. For example, given """"alloy"""" and """"devoted,"""" the answer would be """"loyal."""" (Loyal is an anagram of """"alloy"""" and a synonym of """"devoted.)" 1 THING & EVENING NIGHT 20100904 "You are given a five-letter word and a seven-letter word. Rearrange the letters of one of the words to get a synonym of the other word. For example, given """"alloy"""" and """"devoted,"""" the answer would be """"loyal."""" (Loyal is an anagram of """"alloy"""" and a synonym of """"devoted.)" 4 IRATE & GRENADE ENRAGED or ANGERED 20100904 "You are given a five-letter word and a seven-letter word. Rearrange the letters of one of the words to get a synonym of the other word. For example, given """"alloy"""" and """"devoted,"""" the answer would be """"loyal."""" (Loyal is an anagram of """"alloy"""" and a synonym of """"devoted.)" 5 NADIR & DEPLETE DRAIN 20100904 "You are given a five-letter word and a seven-letter word. Rearrange the letters of one of the words to get a synonym of the other word. For example, given """"alloy"""" and """"devoted,"""" the answer would be """"loyal."""" (Loyal is an anagram of """"alloy"""" and a synonym of """"devoted.)" 6 ONSET & BOULDER STONE 20100904 "You are given a five-letter word and a seven-letter word. Rearrange the letters of one of the words to get a synonym of the other word. For example, given """"alloy"""" and """"devoted,"""" the answer would be """"loyal."""" (Loyal is an anagram of """"alloy"""" and a synonym of """"devoted.)" 7 AWFUL & INHOUSE HEINOUS 20100904 "You are given a five-letter word and a seven-letter word. Rearrange the letters of one of the words to get a synonym of the other word. For example, given """"alloy"""" and """"devoted,"""" the answer would be """"loyal."""" (Loyal is an anagram of """"alloy"""" and a synonym of """"devoted.)" 8 GIRTH & CORRECT RIGHT 20100904 "You are given a five-letter word and a seven-letter word. Rearrange the letters of one of the words to get a synonym of the other word. For example, given """"alloy"""" and """"devoted,"""" the answer would be """"loyal."""" (Loyal is an anagram of """"alloy"""" and a synonym of """"devoted.)" 9 AGREE & ITCHING EAGER 20121014 "Every answer is a two-word phrase in which the letter """"O"""" is added at the end of the first word to make the second word. For example, given the clue """"pack animal owned by Thomas Jefferson's first vice president,"""" the answer would be """"Burr burro.""""" 1 a teaser for an annual school dance prom, promo 20121014 "Every answer is a two-word phrase in which the letter """"O"""" is added at the end of the first word to make the second word. For example, given the clue """"pack animal owned by Thomas Jefferson's first vice president,"""" the answer would be """"Burr burro.""""" 2 leave Ecuador's capital quit Quito 20121014 "Every answer is a two-word phrase in which the letter """"O"""" is added at the end of the first word to make the second word. For example, given the clue """"pack animal owned by Thomas Jefferson's first vice president,"""" the answer would be """"Burr burro.""""" 3 sound output of a German automobile Audi audio 20121014 "Every answer is a two-word phrase in which the letter """"O"""" is added at the end of the first word to make the second word. For example, given the clue """"pack animal owned by Thomas Jefferson's first vice president,"""" the answer would be """"Burr burro.""""" 4 gambling game promoted by a rival of 20121014 "Every answer is a two-word phrase in which the letter """"O"""" is added at the end of the first word to make the second word. For example, given the clue """"pack animal owned by Thomas Jefferson's first vice president,"""" the answer would be """"Burr burro.""""" 5 greeting from Satan hell hello 20121014 "Every answer is a two-word phrase in which the letter """"O"""" is added at the end of the first word to make the second word. For example, given the clue """"pack animal owned by Thomas Jefferson's first vice president,"""" the answer would be """"Burr burro.""""" 6 a stylish Marx brother chic Chico 20121014 "Every answer is a two-word phrase in which the letter """"O"""" is added at the end of the first word to make the second word. For example, given the clue """"pack animal owned by Thomas Jefferson's first vice president,"""" the answer would be """"Burr burro.""""" 7 old classroom handout about a silent 20121014 "Every answer is a two-word phrase in which the letter """"O"""" is added at the end of the first word to make the second word. For example, given the clue """"pack animal owned by Thomas Jefferson's first vice president,"""" the answer would be """"Burr burro.""""" 8 two-for-one deal for several items of 20121014 "Every answer is a two-word phrase in which the letter """"O"""" is added at the end of the first word to make the second word. For example, given the clue """"pack animal owned by Thomas Jefferson's first vice president,"""" the answer would be """"Burr burro.""""" 9 personal grooming comb combo 20121014 "Every answer is a two-word phrase in which the letter """"O"""" is added at the end of the first word to make the second word. For example, given the clue """"pack animal owned by Thomas Jefferson's first vice president,"""" the answer would be """"Burr burro.""""" 10 carried an East Indian island borne Borneo 20121014 "Every answer is a two-word phrase in which the letter """"O"""" is added at the end of the first word to make the second word. For example, given the clue """"pack animal owned by Thomas Jefferson's first vice president,"""" the answer would be """"Burr burro.""""" 11 a wading bird that's on fire flamingo 20130317 " ""This week's puzzle was entitled (on the Web) """"Take Your Pics"""" and Will introduced it: """"This week we're going to take some P-I Cs."""" The answer to each clue given is a familiar two-word* phrase or name in which the first word starts with the letters P-I and the second word starts with C. For example" 1 It comes from an evergreen PIne Cone 20130317 " ""This week's puzzle was entitled (on the Web) """"Take Your Pics"""" and Will introduced it: """"This week we're going to take some P-I Cs."""" The answer to each clue given is a familiar two-word* phrase or name in which the first word starts with the letters P-I and the second word starts with C. For example" 2 a linen item accompanying a sheet PIllow Case 20130317 " ""This week's puzzle was entitled (on the Web) """"Take Your Pics"""" and Will introduced it: """"This week we're going to take some P-I Cs."""" The answer to each clue given is a familiar two-word* phrase or name in which the first word starts with the letters P-I and the second word starts with C. For example" 3 group that services a race car during a race PIt Crew 20130317 " ""This week's puzzle was entitled (on the Web) """"Take Your Pics"""" and Will introduced it: """"This week we're going to take some P-I Cs."""" The answer to each clue given is a familiar two-word* phrase or name in which the first word starts with the letters P-I and the second word starts with C. For example" 4 item in which to stick unused needles PInCushion 20130317 " ""This week's puzzle was entitled (on the Web) """"Take Your Pics"""" and Will introduced it: """"This week we're going to take some P-I Cs."""" The answer to each clue given is a familiar two-word* phrase or name in which the first word starts with the letters P-I and the second word starts with C. For example" 5 place to hear birds coo PIgeon Coop 20130317 " ""This week's puzzle was entitled (on the Web) """"Take Your Pics"""" and Will introduced it: """"This week we're going to take some P-I Cs."""" The answer to each clue given is a familiar two-word* phrase or name in which the first word starts with the letters P-I and the second word starts with C. For example" 6 where Regent St and Shaftsbury Ave meet PIccadilly Circus 20130317 " ""This week's puzzle was entitled (on the Web) """"Take Your Pics"""" and Will introduced it: """"This week we're going to take some P-I Cs."""" The answer to each clue given is a familiar two-word* phrase or name in which the first word starts with the letters P-I and the second word starts with C. For example" 7 title car in a Bruce Springsteen song PInk Cadillac 20130317 " ""This week's puzzle was entitled (on the Web) """"Take Your Pics"""" and Will introduced it: """"This week we're going to take some P-I Cs."""" The answer to each clue given is a familiar two-word* phrase or name in which the first word starts with the letters P-I and the second word starts with C. For example" 8 Soo-oo-ee! PIg Call 20130317 " ""This week's puzzle was entitled (on the Web) """"Take Your Pics"""" and Will introduced it: """"This week we're going to take some P-I Cs."""" The answer to each clue given is a familiar two-word* phrase or name in which the first word starts with the letters P-I and the second word starts with C. For example" 9 classic two-seat airplane Piper Cub 20130317 " ""This week's puzzle was entitled (on the Web) """"Take Your Pics"""" and Will introduced it: """"This week we're going to take some P-I Cs."""" The answer to each clue given is a familiar two-word* phrase or name in which the first word starts with the letters P-I and the second word starts with C. For example" 10 hard outer part of a baked dessert PIe Crust 20130317 " ""This week's puzzle was entitled (on the Web) """"Take Your Pics"""" and Will introduced it: """"This week we're going to take some P-I Cs."""" The answer to each clue given is a familiar two-word* phrase or name in which the first word starts with the letters P-I and the second word starts with C. For example" 11 cocktail: rum, cream of coconut, pineapple juice PIn(~)a Colada 20101016 "Every answer is a word that starts with the letter Y. You are given a clue that is an anagram of the answer before Y is added. For example, given """"wan,"""" the answer would be """"yawn.""""" 1 AGO YOGA 20101016 "Every answer is a word that starts with the letter Y. You are given a clue that is an anagram of the answer before Y is added. For example, given """"wan,"""" the answer would be """"yawn.""""" 2 RAN YARN 20101016 "Every answer is a word that starts with the letter Y. You are given a clue that is an anagram of the answer before Y is added. For example, given """"wan,"""" the answer would be """"yawn.""""" 3 DOLE YODEL 20101016 "Every answer is a word that starts with the letter Y. You are given a clue that is an anagram of the answer before Y is added. For example, given """"wan,"""" the answer would be """"yawn.""""" 4 LIED YIELD 20101016 "Every answer is a word that starts with the letter Y. You are given a clue that is an anagram of the answer before Y is added. For example, given """"wan,"""" the answer would be """"yawn.""""" 5 CHAT YACHT 20101016 "Every answer is a word that starts with the letter Y. You are given a clue that is an anagram of the answer before Y is added. For example, given """"wan,"""" the answer would be """"yawn.""""" 6 THOU YOUTH 20101016 "Every answer is a word that starts with the letter Y. You are given a clue that is an anagram of the answer before Y is added. For example, given """"wan,"""" the answer would be """"yawn.""""" 7 NEAR YEARN 20101016 "Every answer is a word that starts with the letter Y. You are given a clue that is an anagram of the answer before Y is added. For example, given """"wan,"""" the answer would be """"yawn.""""" 8 DRONE YONDER 20101016 "Every answer is a word that starts with the letter Y. You are given a clue that is an anagram of the answer before Y is added. For example, given """"wan,"""" the answer would be """"yawn.""""" 9 GROUT YOGURT 20101016 "Every answer is a word that starts with the letter Y. You are given a clue that is an anagram of the answer before Y is added. For example, given """"wan,"""" the answer would be """"yawn.""""" 10 STURGEON YOUNGSTER 20101016 "Every answer is a word that starts with the letter Y. You are given a clue that is an anagram of the answer before Y is added. For example, given """"wan,"""" the answer would be """"yawn.""""" 11 DIRT SACK YARDSTICK 20201004 I'm going to read you pairs of incomplete phrases. Put the same word in each blank to complete them. Every answer is a four-letter word. Ex. ___ of wolves / ___ of cigarettes --> PACK1. ___ of sale / ___ of rights2. ___ of arms / ___ of paint3. ___ of thought / ___ of scrimmage4. ___ of fire / ___ of wax5. ___ of thumb / ___ of law6. ___ of mouth / ___ of God7. ___ of hands / ___ of force8. ___ of Africa / ___ of plenty9. ___ of paper / ___ of the tongue10. ___ of luck / ___ of both worlds 1 . ___ of sale / ___ of rights bill 20201004 I'm going to read you pairs of incomplete phrases. Put the same word in each blank to complete them. Every answer is a four-letter word. Ex. ___ of wolves / ___ of cigarettes --> PACK1. ___ of sale / ___ of rights2. ___ of arms / ___ of paint3. ___ of thought / ___ of scrimmage4. ___ of fire / ___ of wax5. ___ of thumb / ___ of law6. ___ of mouth / ___ of God7. ___ of hands / ___ of force8. ___ of Africa / ___ of plenty9. ___ of paper / ___ of the tongue10. ___ of luck / ___ of both worlds 2 . ___ of arms / ___ of paint coat 20201004 I'm going to read you pairs of incomplete phrases. Put the same word in each blank to complete them. Every answer is a four-letter word. Ex. ___ of wolves / ___ of cigarettes --> PACK1. ___ of sale / ___ of rights2. ___ of arms / ___ of paint3. ___ of thought / ___ of scrimmage4. ___ of fire / ___ of wax5. ___ of thumb / ___ of law6. ___ of mouth / ___ of God7. ___ of hands / ___ of force8. ___ of Africa / ___ of plenty9. ___ of paper / ___ of the tongue10. ___ of luck / ___ of both worlds 3 . ___ of thought / ___ of scrimmage line 20201004 I'm going to read you pairs of incomplete phrases. Put the same word in each blank to complete them. Every answer is a four-letter word. Ex. ___ of wolves / ___ of cigarettes --> PACK1. ___ of sale / ___ of rights2. ___ of arms / ___ of paint3. ___ of thought / ___ of scrimmage4. ___ of fire / ___ of wax5. ___ of thumb / ___ of law6. ___ of mouth / ___ of God7. ___ of hands / ___ of force8. ___ of Africa / ___ of plenty9. ___ of paper / ___ of the tongue10. ___ of luck / ___ of both worlds 4 . ___ of fire / ___ of wax ball 20201004 I'm going to read you pairs of incomplete phrases. Put the same word in each blank to complete them. Every answer is a four-letter word. Ex. ___ of wolves / ___ of cigarettes --> PACK1. ___ of sale / ___ of rights2. ___ of arms / ___ of paint3. ___ of thought / ___ of scrimmage4. ___ of fire / ___ of wax5. ___ of thumb / ___ of law6. ___ of mouth / ___ of God7. ___ of hands / ___ of force8. ___ of Africa / ___ of plenty9. ___ of paper / ___ of the tongue10. ___ of luck / ___ of both worlds 5 . ___ of thumb / ___ of law rule 20201004 I'm going to read you pairs of incomplete phrases. Put the same word in each blank to complete them. Every answer is a four-letter word. Ex. ___ of wolves / ___ of cigarettes --> PACK1. ___ of sale / ___ of rights2. ___ of arms / ___ of paint3. ___ of thought / ___ of scrimmage4. ___ of fire / ___ of wax5. ___ of thumb / ___ of law6. ___ of mouth / ___ of God7. ___ of hands / ___ of force8. ___ of Africa / ___ of plenty9. ___ of paper / ___ of the tongue10. ___ of luck / ___ of both worlds 6 . ___ of mouth / ___ of God word 20201004 I'm going to read you pairs of incomplete phrases. Put the same word in each blank to complete them. Every answer is a four-letter word. Ex. ___ of wolves / ___ of cigarettes --> PACK1. ___ of sale / ___ of rights2. ___ of arms / ___ of paint3. ___ of thought / ___ of scrimmage4. ___ of fire / ___ of wax5. ___ of thumb / ___ of law6. ___ of mouth / ___ of God7. ___ of hands / ___ of force8. ___ of Africa / ___ of plenty9. ___ of paper / ___ of the tongue10. ___ of luck / ___ of both worlds 7 . ___ of hands / ___ of force show 20201004 I'm going to read you pairs of incomplete phrases. Put the same word in each blank to complete them. Every answer is a four-letter word. Ex. ___ of wolves / ___ of cigarettes --> PACK1. ___ of sale / ___ of rights2. ___ of arms / ___ of paint3. ___ of thought / ___ of scrimmage4. ___ of fire / ___ of wax5. ___ of thumb / ___ of law6. ___ of mouth / ___ of God7. ___ of hands / ___ of force8. ___ of Africa / ___ of plenty9. ___ of paper / ___ of the tongue10. ___ of luck / ___ of both worlds 8 . ___ of Africa / ___ of plenty horn 20201004 I'm going to read you pairs of incomplete phrases. Put the same word in each blank to complete them. Every answer is a four-letter word. Ex. ___ of wolves / ___ of cigarettes --> PACK1. ___ of sale / ___ of rights2. ___ of arms / ___ of paint3. ___ of thought / ___ of scrimmage4. ___ of fire / ___ of wax5. ___ of thumb / ___ of law6. ___ of mouth / ___ of God7. ___ of hands / ___ of force8. ___ of Africa / ___ of plenty9. ___ of paper / ___ of the tongue10. ___ of luck / ___ of both worlds 9 . ___ of paper / ___ of the tongue slip 20201004 I'm going to read you pairs of incomplete phrases. Put the same word in each blank to complete them. Every answer is a four-letter word. Ex. ___ of wolves / ___ of cigarettes --> PACK1. ___ of sale / ___ of rights2. ___ of arms / ___ of paint3. ___ of thought / ___ of scrimmage4. ___ of fire / ___ of wax5. ___ of thumb / ___ of law6. ___ of mouth / ___ of God7. ___ of hands / ___ of force8. ___ of Africa / ___ of plenty9. ___ of paper / ___ of the tongue10. ___ of luck / ___ of both worlds 10 . ___ of luck / ___ of both worlds best 20140223 " ""Every answer is the name of a famous person with four letters in his or her first name and four letters in the last. For each person" 1 what you need to watch hbo and showtime cable tv 20140223 " ""Every answer is the name of a famous person with four letters in his or her first name and four letters in the last. For each person" 2 "the song """"born to run"""" from the album ""born to run,"" for example" title track 20140223 " ""Every answer is the name of a famous person with four letters in his or her first name and four letters in the last. For each person" 3 a rustler cattle thief 20140223 " ""Every answer is the name of a famous person with four letters in his or her first name and four letters in the last. For each person" 4 a cap for a glass container bottle top 20140223 " ""Every answer is the name of a famous person with four letters in his or her first name and four letters in the last. For each person" 5 "what you swear to tell in court, ""...so help you g-d""" the whole truth 20140223 " ""Every answer is the name of a famous person with four letters in his or her first name and four letters in the last. For each person" 6 original name for the popeye comic strip thimble theatre 20140223 " ""Every answer is the name of a famous person with four letters in his or her first name and four letters in the last. For each person" 7 the guttenberg bible was the first major book to be printed using this. movable type 20140223 " ""Every answer is the name of a famous person with four letters in his or her first name and four letters in the last. For each person" 8 nickname for charlie chaplin little tramp 20140223 " ""Every answer is the name of a famous person with four letters in his or her first name and four letters in the last. For each person" 9 purchase for a drawing raffle ticket 20140223 " ""Every answer is the name of a famous person with four letters in his or her first name and four letters in the last. For each person" 10 a delayed reaction to something surprising, immediately after the first reaction double take 20070722 "Every answer is a familiar two-word phase, with the consecutive letters ""A-L-P."" In particular, the first word ends in ""A-L"" and the second word starts with ""P.""" 1 Yellowstone or Yosemite national park 20070722 "Every answer is a familiar two-word phase, with the consecutive letters ""A-L-P."" In particular, the first word ends in ""A-L"" and the second word starts with ""P.""" 2 A board game of answering questions trivial pursuit 20070722 "Every answer is a familiar two-word phase, with the consecutive letters ""A-L-P."" In particular, the first word ends in ""A-L"" and the second word starts with ""P.""" 3 Republicans or Democrats political party 20070722 "Every answer is a familiar two-word phase, with the consecutive letters ""A-L-P."" In particular, the first word ends in ""A-L"" and the second word starts with ""P.""" 4 The death penalty capital punishment 20070722 "Every answer is a familiar two-word phase, with the consecutive letters ""A-L-P."" In particular, the first word ends in ""A-L"" and the second word starts with ""P.""" 5 July 1 to June 30 fiscal period 20070722 "Every answer is a familiar two-word phase, with the consecutive letters ""A-L-P."" In particular, the first word ends in ""A-L"" and the second word starts with ""P.""" 6 Tossing the ball side ways in football lateral pass 20070722 "Every answer is a familiar two-word phase, with the consecutive letters ""A-L-P."" In particular, the first word ends in ""A-L"" and the second word starts with ""P.""" 7 In a number it comes between the ones and the tenths place decimal point 20070722 "Every answer is a familiar two-word phase, with the consecutive letters ""A-L-P."" In particular, the first word ends in ""A-L"" and the second word starts with ""P.""" 8 Group of sheets of 8-1/2 x 14 paper legal pad 20070722 "Every answer is a familiar two-word phase, with the consecutive letters ""A-L-P."" In particular, the first word ends in ""A-L"" and the second word starts with ""P.""" 9 Girls calf-length pants pedal pushers 20070722 "Every answer is a familiar two-word phase, with the consecutive letters ""A-L-P."" In particular, the first word ends in ""A-L"" and the second word starts with ""P.""" 10 Dish of calf's meat covered with tomato sauce and cheese veal parmigiana 20070722 "Every answer is a familiar two-word phase, with the consecutive letters ""A-L-P."" In particular, the first word ends in ""A-L"" and the second word starts with ""P.""" 11 Doctors, collectively medical profession 20070722 "Every answer is a familiar two-word phase, with the consecutive letters ""A-L-P."" In particular, the first word ends in ""A-L"" and the second word starts with ""P.""" 12 What person may be curled up in as a baby fetal position 20070722 "Every answer is a familiar two-word phase, with the consecutive letters ""A-L-P."" In particular, the first word ends in ""A-L"" and the second word starts with ""P.""" 13 Copyrights and patents intellectual property 20070722 "Every answer is a familiar two-word phase, with the consecutive letters ""A-L-P."" In particular, the first word ends in ""A-L"" and the second word starts with ""P.""" 14 What QED may finish mathematical proof 20070722 "Every answer is a familiar two-word phase, with the consecutive letters ""A-L-P."" In particular, the first word ends in ""A-L"" and the second word starts with ""P.""" 15 An image in your head mental picture 20070722 "Every answer is a familiar two-word phase, with the consecutive letters ""A-L-P."" In particular, the first word ends in ""A-L"" and the second word starts with ""P.""" 16 The first one is in New Hampshire every four years presidential primary 20120422 "You'll be given classic advertising slogans and catch phrases in which the letters of the last word are scrambled. First, unscramble the word. Then name the product or company that is the advertiser. For example, given """"Get a piece of the cork,"""" the answer would be """"Get a piece of the rock,"""" which is a slogan of the Prudential Insurance Company." 4 the breakfast of nacho imps champions; wheaties, a competitor to the company at which don works 20120422 "You'll be given classic advertising slogans and catch phrases in which the letters of the last word are scrambled. First, unscramble the word. Then name the product or company that is the advertiser. For example, given """"Get a piece of the cork,"""" the answer would be """"Get a piece of the rock,"""" which is a slogan of the Prudential Insurance Company." 5 please don't squeeze the rich man charmin 20120422 "You'll be given classic advertising slogans and catch phrases in which the letters of the last word are scrambled. First, unscramble the word. Then name the product or company that is the advertiser. For example, given """"Get a piece of the cork,"""" the answer would be """"Get a piece of the rock,"""" which is a slogan of the Prudential Insurance Company." 6 there are some things that money can't buy. for everything else, there is mad craters. mastercard 20190317 "I've brought a game of Categories based on the word THINK. For each category I give, name something in it starting with each of the letters T-H-I-N-K. For example, if the category were """"Boys' Names,"""" you might say Tom, Hank, Ike, Nick and Kent. Any answer that works is OK, and you can give the answers in any order." 1 US State Texas, Hawaii, Idaho, Nebraska, Kansas 20190317 "I've brought a game of Categories based on the word THINK. For each category I give, name something in it starting with each of the letters T-H-I-N-K. For example, if the category were """"Boys' Names,"""" you might say Tom, Hank, Ike, Nick and Kent. Any answer that works is OK, and you can give the answers in any order." 2 Football team huddle, interception, nose guard, kick 20190317 "I've brought a game of Categories based on the word THINK. For each category I give, name something in it starting with each of the letters T-H-I-N-K. For example, if the category were """"Boys' Names,"""" you might say Tom, Hank, Ike, Nick and Kent. Any answer that works is OK, and you can give the answers in any order." 3 Parts of the Human Body toes, heart, iris, nose, knee 20190317 "I've brought a game of Categories based on the word THINK. For each category I give, name something in it starting with each of the letters T-H-I-N-K. For example, if the category were """"Boys' Names,"""" you might say Tom, Hank, Ike, Nick and Kent. Any answer that works is OK, and you can give the answers in any order." 4 Things Seen on a Dinner Table tablecloth, hamburger, ice cream, napkin, knife 20190317 "I've brought a game of Categories based on the word THINK. For each category I give, name something in it starting with each of the letters T-H-I-N-K. For example, if the category were """"Boys' Names,"""" you might say Tom, Hank, Ike, Nick and Kent. Any answer that works is OK, and you can give the answers in any order." 5 "Five-Letter Words Ending in """"O""""" tempo, hello, imago, negro, kazoo 20080921 "You are given two clues. The first answer is a word that starts with the sound T. Change this to a D, and phonetically you'll get a new word that answers the second clue. For example, if the clue is """"to work hard"""" and """"author Conan _______,"""" the answer would be """"toil"""" and """"Doyle.""""" 2 "A heavy weight; poet who wrote ""Death Be Not Proud""" ton; Donne 20080921 "You are given two clues. The first answer is a word that starts with the sound T. Change this to a D, and phonetically you'll get a new word that answers the second clue. For example, if the clue is """"to work hard"""" and """"author Conan _______,"""" the answer would be """"toil"""" and """"Doyle.""""" 3 Traveling group of actors; to sag troupe; droop 20080921 "You are given two clues. The first answer is a word that starts with the sound T. Change this to a D, and phonetically you'll get a new word that answers the second clue. For example, if the clue is """"to work hard"""" and """"author Conan _______,"""" the answer would be """"toil"""" and """"Doyle.""""" 5 A force that's applied to a wheel; a nerd torque; dork 20080921 "You are given two clues. The first answer is a word that starts with the sound T. Change this to a D, and phonetically you'll get a new word that answers the second clue. For example, if the clue is """"to work hard"""" and """"author Conan _______,"""" the answer would be """"toil"""" and """"Doyle.""""" 6 Easily offended; area overseen by a nobleman touchy; duchy 20080921 "You are given two clues. The first answer is a word that starts with the sound T. Change this to a D, and phonetically you'll get a new word that answers the second clue. For example, if the clue is """"to work hard"""" and """"author Conan _______,"""" the answer would be """"toil"""" and """"Doyle.""""" 7 What you might watch the BBC on; supermarket section telly; deli 20080921 "You are given two clues. The first answer is a word that starts with the sound T. Change this to a D, and phonetically you'll get a new word that answers the second clue. For example, if the clue is """"to work hard"""" and """"author Conan _______,"""" the answer would be """"toil"""" and """"Doyle.""""" 8 Popular food fish; woe for farmers trout; drought 20080921 "You are given two clues. The first answer is a word that starts with the sound T. Change this to a D, and phonetically you'll get a new word that answers the second clue. For example, if the clue is """"to work hard"""" and """"author Conan _______,"""" the answer would be """"toil"""" and """"Doyle.""""" 9 Difficult; rear end tough; duff 20080921 "You are given two clues. The first answer is a word that starts with the sound T. Change this to a D, and phonetically you'll get a new word that answers the second clue. For example, if the clue is """"to work hard"""" and """"author Conan _______,"""" the answer would be """"toil"""" and """"Doyle.""""" 10 Pants; people who have nodded off trousers; drowsers 20080921 "You are given two clues. The first answer is a word that starts with the sound T. Change this to a D, and phonetically you'll get a new word that answers the second clue. For example, if the clue is """"to work hard"""" and """"author Conan _______,"""" the answer would be """"toil"""" and """"Doyle.""""" 11 A certain poison; realm ptomaine; domai 20120819 "This week's puzzle is called """"Anagram K-pers."""" Every answer is a familiar word starting with the letter """"K."""" You identify the words from their anagrams. For example, K + vane will make """"knave.""""" 1 K + FINE KNIFE 20120819 "This week's puzzle is called """"Anagram K-pers."""" Every answer is a familiar word starting with the letter """"K."""" You identify the words from their anagrams. For example, K + vane will make """"knave.""""" 2 K + PAPA KAPPA 20120819 "This week's puzzle is called """"Anagram K-pers."""" Every answer is a familiar word starting with the letter """"K."""" You identify the words from their anagrams. For example, K + vane will make """"knave.""""" 3 K + LENT KNELT 20120819 "This week's puzzle is called """"Anagram K-pers."""" Every answer is a familiar word starting with the letter """"K."""" You identify the words from their anagrams. For example, K + vane will make """"knave.""""" 4 K + NERO KRONE 20120819 "This week's puzzle is called """"Anagram K-pers."""" Every answer is a familiar word starting with the letter """"K."""" You identify the words from their anagrams. For example, K + vane will make """"knave.""""" 5 K + RAISE KAISER 20120819 "This week's puzzle is called """"Anagram K-pers."""" Every answer is a familiar word starting with the letter """"K."""" You identify the words from their anagrams. For example, K + vane will make """"knave.""""" 6 K + THING KNIGHT 20120819 "This week's puzzle is called """"Anagram K-pers."""" Every answer is a familiar word starting with the letter """"K."""" You identify the words from their anagrams. For example, K + vane will make """"knave.""""" 7 K + LINED KINDLE 20120819 "This week's puzzle is called """"Anagram K-pers."""" Every answer is a familiar word starting with the letter """"K."""" You identify the words from their anagrams. For example, K + vane will make """"knave.""""" 8 K + TIMES KISMET 20120819 "This week's puzzle is called """"Anagram K-pers."""" Every answer is a familiar word starting with the letter """"K."""" You identify the words from their anagrams. For example, K + vane will make """"knave.""""" 9 K + CHITS KITSCH 20120819 "This week's puzzle is called """"Anagram K-pers."""" Every answer is a familiar word starting with the letter """"K."""" You identify the words from their anagrams. For example, K + vane will make """"knave.""""" 10 K + PINING KINGPIN 20190707 Every answer is a word containing the accented syllable, PEN. 1 A 1-cent coin PENNY 20190707 Every answer is a word containing the accented syllable, PEN. 2 A part added on at the end of a book APPENDIX 20190707 Every answer is a word containing the accented syllable, PEN. 3 Straps to hold one's pants up SUSPENDERS 20190707 Every answer is a word containing the accented syllable, PEN. 4 Pricey EXPENSIVE 20190707 Every answer is a word containing the accented syllable, PEN. 5 Money for a retired person PENSION 20190707 Every answer is a word containing the accented syllable, PEN. 6 Expressing sincere remorse REPENTING 20190707 Every answer is a word containing the accented syllable, PEN. 7 A city on the Florida Panhandle PENSACOLA 20190707 Every answer is a word containing the accented syllable, PEN. 8 The capital of Cambodia PHNOM-PENH 20190707 Every answer is a word containing the accented syllable, PEN. 9 A facility where drugs are given out DISPENSARY 20190707 Every answer is a word containing the accented syllable, PEN. 10 What a red flag means in sports PENALTY 20091018 "I'm going to give you some clues. The answer to each clue is a four-letter word which can be found in consecutive letters inside the clue. For example, if I said """"a sail boat's part"""", you would say """"spar"""", because a sail boat's part is a spar, and it's concealed in consecutive letters inside """"sail boat's"""" parts." 1 create a rip TEAR 20091018 "I'm going to give you some clues. The answer to each clue is a four-letter word which can be found in consecutive letters inside the clue. For example, if I said """"a sail boat's part"""", you would say """"spar"""", because a sail boat's part is a spar, and it's concealed in consecutive letters inside """"sail boat's"""" parts." 2 emotion that enemies may harbor HATE 20091018 "I'm going to give you some clues. The answer to each clue is a four-letter word which can be found in consecutive letters inside the clue. For example, if I said """"a sail boat's part"""", you would say """"spar"""", because a sail boat's part is a spar, and it's concealed in consecutive letters inside """"sail boat's"""" parts." 3 close a letter with wax SEAL 20091018 "I'm going to give you some clues. The answer to each clue is a four-letter word which can be found in consecutive letters inside the clue. For example, if I said """"a sail boat's part"""", you would say """"spar"""", because a sail boat's part is a spar, and it's concealed in consecutive letters inside """"sail boat's"""" parts." 4 top level of the barn LOFT 20091018 "I'm going to give you some clues. The answer to each clue is a four-letter word which can be found in consecutive letters inside the clue. For example, if I said """"a sail boat's part"""", you would say """"spar"""", because a sail boat's part is a spar, and it's concealed in consecutive letters inside """"sail boat's"""" parts." 5 Edgar Allan Poe, to name one POET 20091018 "I'm going to give you some clues. The answer to each clue is a four-letter word which can be found in consecutive letters inside the clue. For example, if I said """"a sail boat's part"""", you would say """"spar"""", because a sail boat's part is a spar, and it's concealed in consecutive letters inside """"sail boat's"""" parts." 6 utensil for kitchen use FORK 20091018 "I'm going to give you some clues. The answer to each clue is a four-letter word which can be found in consecutive letters inside the clue. For example, if I said """"a sail boat's part"""", you would say """"spar"""", because a sail boat's part is a spar, and it's concealed in consecutive letters inside """"sail boat's"""" parts." 9 they espy things EYES 20091018 "I'm going to give you some clues. The answer to each clue is a four-letter word which can be found in consecutive letters inside the clue. For example, if I said """"a sail boat's part"""", you would say """"spar"""", because a sail boat's part is a spar, and it's concealed in consecutive letters inside """"sail boat's"""" parts." 10 where a Siamese came from ASIA 20091018 "I'm going to give you some clues. The answer to each clue is a four-letter word which can be found in consecutive letters inside the clue. For example, if I said """"a sail boat's part"""", you would say """"spar"""", because a sail boat's part is a spar, and it's concealed in consecutive letters inside """"sail boat's"""" parts." 11 insane Roman emperor NERO 20080127 "In this week's on-air puzzle, every answer is a three-word phrase in the form """"______ the ______."""" Will offers rhymes for the first and last words in the phrases. For example, given """"lease the reels,"""" the answer would be """"grease the wheels.""""" 1 swallow the reader follow the leader 20080127 "In this week's on-air puzzle, every answer is a three-word phrase in the form """"______ the ______."""" Will offers rhymes for the first and last words in the phrases. For example, given """"lease the reels,"""" the answer would be """"grease the wheels.""""" 3 grace the station Face the Nation 20080127 "In this week's on-air puzzle, every answer is a three-word phrase in the form """"______ the ______."""" Will offers rhymes for the first and last words in the phrases. For example, given """"lease the reels,"""" the answer would be """"grease the wheels.""""" 4 ruffle the check shuffle the deck 20080127 "In this week's on-air puzzle, every answer is a three-word phrase in the form """"______ the ______."""" Will offers rhymes for the first and last words in the phrases. For example, given """"lease the reels,"""" the answer would be """"grease the wheels.""""" 5 bring the booze sing the Blues 20080127 "In this week's on-air puzzle, every answer is a three-word phrase in the form """"______ the ______."""" Will offers rhymes for the first and last words in the phrases. For example, given """"lease the reels,"""" the answer would be """"grease the wheels.""""" 6 shake the sponge take the plunge 20080127 "In this week's on-air puzzle, every answer is a three-word phrase in the form """"______ the ______."""" Will offers rhymes for the first and last words in the phrases. For example, given """"lease the reels,"""" the answer would be """"grease the wheels.""""" 7 turn the hopes learn the ropes 20080127 "In this week's on-air puzzle, every answer is a three-word phrase in the form """"______ the ______."""" Will offers rhymes for the first and last words in the phrases. For example, given """"lease the reels,"""" the answer would be """"grease the wheels.""""" 8 best the daughters test the waters 20080127 "In this week's on-air puzzle, every answer is a three-word phrase in the form """"______ the ______."""" Will offers rhymes for the first and last words in the phrases. For example, given """"lease the reels,"""" the answer would be """"grease the wheels.""""" 9 thrill the queens spill the beans 20080127 "In this week's on-air puzzle, every answer is a three-word phrase in the form """"______ the ______."""" Will offers rhymes for the first and last words in the phrases. For example, given """"lease the reels,"""" the answer would be """"grease the wheels.""""" 11 take the yank break the bank 20080127 "In this week's on-air puzzle, every answer is a three-word phrase in the form """"______ the ______."""" Will offers rhymes for the first and last words in the phrases. For example, given """"lease the reels,"""" the answer would be """"grease the wheels.""""" 12 praise the spoof raise the roof 20080127 "In this week's on-air puzzle, every answer is a three-word phrase in the form """"______ the ______."""" Will offers rhymes for the first and last words in the phrases. For example, given """"lease the reels,"""" the answer would be """"grease the wheels.""""" 13 mute the sleaze shoot the breeze 20080127 "In this week's on-air puzzle, every answer is a three-word phrase in the form """"______ the ______."""" Will offers rhymes for the first and last words in the phrases. For example, given """"lease the reels,"""" the answer would be """"grease the wheels.""""" 14 cheat the jock beat the clock/Beat the Clock 21040105 "This week's puzzle is entitled """"Two Times Harder"""" on the Web site; Will, as usual, assigned no name. Every answer is a pair of two-syllable words. The first syllable of the word responsive to the first clue has the letters A-R, pronounced """"are."""" Change these phonetically to """"er,"""" and you'll get a new word that answers the second clue. For example, to the prompts """"hair-cutter"""" and """"a North African,"""" the answers would be """"barber"""" and """"Berber.""""" 1 drape and carton curtain 21040105 "This week's puzzle is entitled """"Two Times Harder"""" on the Web site; Will, as usual, assigned no name. Every answer is a pair of two-syllable words. The first syllable of the word responsive to the first clue has the letters A-R, pronounced """"are."""" Change these phonetically to """"er,"""" and you'll get a new word that answers the second clue. For example, to the prompts """"hair-cutter"""" and """"a North African,"""" the answers would be """"barber"""" and """"Berber.""""" 2 large and muscular--barley burly 21040105 "This week's puzzle is entitled """"Two Times Harder"""" on the Web site; Will, as usual, assigned no name. Every answer is a pair of two-syllable words. The first syllable of the word responsive to the first clue has the letters A-R, pronounced """"are."""" Change these phonetically to """"er,"""" and you'll get a new word that answers the second clue. For example, to the prompts """"hair-cutter"""" and """"a North African,"""" the answers would be """"barber"""" and """"Berber.""""" 3 an individual--parson person 21040105 "This week's puzzle is entitled """"Two Times Harder"""" on the Web site; Will, as usual, assigned no name. Every answer is a pair of two-syllable words. The first syllable of the word responsive to the first clue has the letters A-R, pronounced """"are."""" Change these phonetically to """"er,"""" and you'll get a new word that answers the second clue. For example, to the prompts """"hair-cutter"""" and """"a North African,"""" the answers would be """"barber"""" and """"Berber.""""" 4 actress Elizabeth--Harley Hurley 21040105 "This week's puzzle is entitled """"Two Times Harder"""" on the Web site; Will, as usual, assigned no name. Every answer is a pair of two-syllable words. The first syllable of the word responsive to the first clue has the letters A-R, pronounced """"are."""" Change these phonetically to """"er,"""" and you'll get a new word that answers the second clue. For example, to the prompts """"hair-cutter"""" and """"a North African,"""" the answers would be """"barber"""" and """"Berber.""""" 5 Civil War general--Charmin Sherman 21040105 "This week's puzzle is entitled """"Two Times Harder"""" on the Web site; Will, as usual, assigned no name. Every answer is a pair of two-syllable words. The first syllable of the word responsive to the first clue has the letters A-R, pronounced """"are."""" Change these phonetically to """"er,"""" and you'll get a new word that answers the second clue. For example, to the prompts """"hair-cutter"""" and """"a North African,"""" the answers would be """"barber"""" and """"Berber.""""" 6 brand of skin cream--jargons Jergens 21040105 "This week's puzzle is entitled """"Two Times Harder"""" on the Web site; Will, as usual, assigned no name. Every answer is a pair of two-syllable words. The first syllable of the word responsive to the first clue has the letters A-R, pronounced """"are."""" Change these phonetically to """"er,"""" and you'll get a new word that answers the second clue. For example, to the prompts """"hair-cutter"""" and """"a North African,"""" the answers would be """"barber"""" and """"Berber.""""" 7 a bit of corn--carnal kernel 21040105 "This week's puzzle is entitled """"Two Times Harder"""" on the Web site; Will, as usual, assigned no name. Every answer is a pair of two-syllable words. The first syllable of the word responsive to the first clue has the letters A-R, pronounced """"are."""" Change these phonetically to """"er,"""" and you'll get a new word that answers the second clue. For example, to the prompts """"hair-cutter"""" and """"a North African,"""" the answers would be """"barber"""" and """"Berber.""""" 8 like some silver--starling Sterling 21040105 "This week's puzzle is entitled """"Two Times Harder"""" on the Web site; Will, as usual, assigned no name. Every answer is a pair of two-syllable words. The first syllable of the word responsive to the first clue has the letters A-R, pronounced """"are."""" Change these phonetically to """"er,"""" and you'll get a new word that answers the second clue. For example, to the prompts """"hair-cutter"""" and """"a North African,"""" the answers would be """"barber"""" and """"Berber.""""" 9 a certain wine--Marlowe merlot